Está en la página 1de 885

NEUROLOGÍA

1. Dentro de las causas de hemorragia cerebral, señale el verdadero:


a. La hipertensión crónica produce daño en las paredes vasculares de arterias
pequeñas
b. La coagulopatía produce hemorragia en sistema ventricular y epidural solamente
c. La angiopatía amiloide produce hemorragia en caudado o putámen
d. Los infartos lacunares son causa de hemorragia mediante una conversión
hemorrágica
2. En relación a las manifestaciones clínicas de hemorragia cerebral, señale el verdadero:
a. Producen síntomas de hipotensión de líquido cefalorraquídeo como cefalea
b. La focalidad neurológica debuta súbitamente y puede empeorar luego de 30-90
minutos
c. La hemorragia del caudado del hemisferio no dominante produce afasia global
d. Inclusive hemorragias de pequeño volumen pueden inducir estupor y llegar al
coma
3. En relación a los siguientes enunciados, señale el verdadero:
a. Los aneurismas intracraneales no rotos siempre son sintomáticos
b. En el 100% de casos el diagnóstico de hemorragia subaracnoidea se la hace
mediante la detección tomográfica del sangrado
c. La cefalea de inicio progresivo es el principal dato para sospechar hemorragia
subaracnoidea
d. El vasoespasmo es un estrechamiento de las arterias que puede ser
clínicamente sintomático o asintomático
4. En relación a la hemorragia subaracnoidea, señale el verdadero:
a. La re-ruptura o resangrado aneurismática se asocia con un pobre desenlace o
mortalidad del 50%
b. Si la etiología es aneurismática la reparación trombolítica es indicada luego de
5 días de evolución
c. Los aneurismas no rotos tienen una tasa de riesgo de ruptura más alta conforme
su diámetro sea más pequeño
d. El estudio de líquido cefalorraquídeo no está indicado pese a una tomografía
cerebral normal
5. De los siguientes enunciados, señale el verdadero:
a. El término locked-in o enclaustramiento define un estado de coma con respuesta
motora al estímulo doloroso
b. La hernia del uncus del hipocampo produce compresión ipsilateral del III nervio
craneal
c. El estado vegetativo se alcanza luego de un estado de conciencia mínimo
d. Los síndromes de herniación destruyen el tejido adyacente pero no comprimen
arterias
6. En relación al trauma craneal y las fracturas de cráneo, señale el verdadero:
a. La fractura traduce necesariamente gravedad del trauma y lesión intracraneal
b. La mayoría de las fracturas son desplazadas o hundidas, la lineales son raras
c. Si se compromete por la fractura el espacio subaracnoideo existe riesgo de
hipotensión de líquido cefalorraquideo
d. Las fracturas basilares o de la base del cráneo son complicadas y presentan el
signo de Battle junto a hemovitreo
7. En relación a la esclerosis múltiple, señale lo verdadero:
a. Uno de los síntomas es la neuritis óptica la cuál no altera la percepción de
colores
b. Dentro de las múltiples manifestaciones puede haber ataxia, vértigo, diplopía
c. La forma clínica secundaria progresiva es la forma más frecuente
d. La edad de inicio más frecuente es en personas mayores de 50 años
8. En relación a la neuromielitis óptica, señale lo verdadero:
a. El curso clínico en más del 90% de pacientes es monofásico
b. No existe asociación con enfermedades autoinmunes sistémicas
c. Los ataques agudos deben ser tratados con metilprednisolona IV
d. La presentación como mielitis aguda produce extensión de lesión de 2 cuerpos
vertebrales
9. En relación al síndrome de Guillain Barré, señale el verdadero:
a. El subtipo más común es la polineuropatía desmielinizante inflamatoria aguda
b. El síndrome de Miller Fisher se caracteriza por trastorno deglutorio y debilidad
braquial
c. Las variantes axonales son las que mejor pronóstico tienen
d. La determinación de anticuerpos anti-GQ1b enfocan el diagnóstico al subtipo
pandisautonómico
10. En relación al síndrome de Guillain Barré, señale el verdadero:
a. Entre los gérmenes que constan como causantes de antecedentes infecciosos
se mencionan: arbovirus
b. Un hallazgo distintivo en estudios de líquido cefalorraquideo es: células elevadas
con hipoproteinorraquia
c. Se considera un cuadro bifásico que dura normalmente 60 días
d. El tratamiento es a base de inmunoterapia: plasmaféresis o inmunoglobulina IV
11. En relación a los siguientes enunciados, señale el verdadero:
a. El vértigo postural paroxístico benigno dura pocos segundos
b. La migraña vestibular produce sd vertiginoso único
c. Los síntomas acompañantes al vértigo no ayudan a la diferenciación entre
central y periférico
d. Síntomas como diplopía o ataxia señalan causas periféricas de vértigo
12. En relación al síndrome vertiginoso, señale el verdadero:
a. El nistagmus downbeat es característico de vértigos periféricos
b. Vértigo episódico provocado por cambios posturales deben someterse a
maniobra de Dix-Hallpike
c. El vértigo, la náusea entre otros síntomas se originan desde información que
llega desde la coclea
d. El uso de glucocortidoides deben ser usado como segunda línea luego de los
primeros 4 días de evolución
13. De los siguientes enunciados, señale el verdadero:
a. El edema citotóxico se produce por disfunción o alteración de la barrera
hematoencefálica
b. Los tipos de edema existentes son: citotóxico, endotelial y tisular
c. El edema puede llevar a incremento de la presión intracraneal
d. El edema es una alteración celular que no se traduce en neuroimagen
14. De los siguientes enunciados, señale el verdadero:
a. Los adultos mayores son particularmente vulnerables a presentar delirio o
síndrome confusional agudo
b. El aparecimiento de un síndrome confusional agudo no influye en el pronóstico
de enfermedades críticas
c. La sedación en pacientes críticamente enfermos, mediante el uso de
benzodiacepinas pueden reducir el delirio
d. La encefalopatía hipercárbica no evoluciona al estado de coma, pese a una
disminución de CO2
15. En mielopatía, señale el verdadero:
a. La exploración del reflejo bicipital traduce un nivel de funcionamiento de C5-C6
b. El nivel de lesión umbilical o del ombligo representa T4
c. El síndrome de cono medular produce anestesia unilateral
d. El estudio de elección es la tomografía con contraste
16. En relación al mareo o vértigo, al realizar la historia clínica, el primer paso es averiguar:
a. Averiguar si la lesión es uni o bilateral
b. Delinear más precisamente la naturaleza del síntoma
c. Averiguar Síntomas acompañantes
d. Averiguar antecedentes familiares
17. En el examen físico de un paciente con queja de vértigo o mareo, señale el enunciado
verdadero:
a. La desalineación de los ojos en el plano vertical puede indicar lesión del tronco
cerebral
b. En el nistagmus es imprescindible determinar su única fase de sacudida que es
la fase lenta
c. El test de impulso cefálico evalúa el reflejo cerebelo ponto rubral
d. La maniobra de Dix-Hallpike se realiza en vértigo único no provocado por cambio
postural
18. En relación al delirio o síndrome confusional agudo, señale el enunciado verdadero:
a. El diagnóstico se basa en la realización de una neuroimagen en fase aguda
b. La amplia manifestación clínica se basa sobre todo en la detección de debilidad
y parestesias
c. La clave de reconocimiento o clave diagnóstica es el déficit de atención
d. El peligro del diagnóstico de este síndrome es la irreversibilidad que conlleva
19. En relación a los factores de riesgo en delirio, señale el enunciado verdadero:
a. Los 2 factores independientes son: edad (adultos mayores) y deterioro del
estado cognitivo basal
b. La falta de sueño o la deprivación sensorial no intervienen en incrementar el
riesgo
c. Hay muchos tests que sirven para detectar los factores de riesgo y deben ser
aplicados al alta
d. Los factores de riesgo no son factores predisponentes y solo son detectados al
examen físico
20. Un paciente que ingresa con una enfermedad febril aguda con cuadro confusional agudo,
cambios de personalidad y agitación con manifestaciones psicóticas, usted debe
sospechar en:
a. Absceso cerebral
b. Neurocisticercosis
c. Meningitis viral
d. Encefalitis viral
21. En relación a la encefalitis por herpes virus y sus tests diagnósticos:
a. La Resonancia magnética demuestra alteraciones en el lóbulo parietal
b. Si la determinación de PCR del virus herpes simple fue negativo se puede repetir
si la sospecha es alta
c. La realización de la secuencia FLAIR o difusión en la resonancia no incrementa
la posibilidad diagnóstica
d. La detección de anticuerpos incrementa si se la realiza antes de las 48 horas
22. En relación al tratamiento de encefalitis viral:
a. El uso de antipiréticos no es necesario pues la infección es leve
b. El aparecimiento de crisis convulsivas no requiere manejo farmacológico
c. El ganciclovir está indicado en el tratamiento de enterovirus
d. El aciclovir se usa para virus herpes simple, varicela zoster o Epstein Barr
23. En relación a la meningitis bacteriana:
a. Es un proceso infeccioso purulento agudo localizado en el espacio subdural
b. De entre las bacterias de infección adquirida en la comunidad consta por ej el
streptococo pneumoniae
c. La tasa de mortalidad permanece alta en aproximadamente un 60% pese al
tratamiento antibiótico
d. Las vacunas para varios gérmenes como el Haemophilus no han logrado
disminuir los casos
24. En un paciente con un síndrome febril agudo, al que añade cefalea y letargia, su análisis
sería:
a. La sospecha de meningitis bacteriana no debe ser considerada pues le falta
signos meníngeos o rigidez nucal
b. La alteración de la conciencia ocurre en < de 75% de pacientes, aunque puede
llegar al coma
c. Los signos clásicos de irritación meníngea siempre traducen procesos
infecciosos
d. El signo de Kernig no se encuentra en pacientes ancianos o
inmunocomprometidos
25. En el análisis del líquido cefalorraquídeo en una meningitis bacteriana:
a. La pleocitosis no debe superar las 500 células
b. Inicialmente predominan los mononucleares
c. La hipoproteinorraquia es mandatoria
d. La glucosa debe ser < 40 mg/dl
26. En relación a las causas y factores de riesgo para enfermedad cerebrovascular
isquémica:
a. La causa más frecuente alrededor del mundo es la fibrilación auricular reumática
b. un infarto de miocardio reciente puede ser una fuente de émbolos
c. La aterosclerosis carotídea es más frecuente en el segmento intracraneal
d. La aterosclerosis carotídea asintomática puede producir un accidente isquémico
transitorio
27. En relación al diagnóstico de la enfermedad cerebrovascular isquémica:
a. La tomografía puede ser normal en las primeras horas, pero sirve para descartar
hemorragia cerebral
b. La secuencia FLAIR de difusión es más sensible a la detección temprana del
infarto
c. La angio RM cerebral es útil para diagnosticar estenosis carotídea intracraneal
d. La técnica de dúplex carotídeo se basa en la determinación del volumen de la
sangre
28. En relación al trauma craneoencefálico:
a. La pérdida de la memoria que existe puede ser anterógrada o post-traumática
b. Según la epidemiología el más común es el trauma moderado
c. La existencia de fractura del cráneo invariablemente traduce hematoma
parenquimatoso
d. Los hematomas subdurales son producidos por lesión de la arteria meníngea
media
29. En relación a la injuria que se produce por el trauma craneoencefálico:
a. La injuria axonal traumática con menos de 2 lesiones se conoce como lesión
axonal difusa
b. La injuria axonal traumática se produce por un mecanismo de desaceleración de
alta velocidad
c. La lesión axonal difusa produce lesiones micro-isquémicas solo visibles en
resonancia
d. La injuria de nervios craneales es infrecuente y no produce síntomas
30. En relación a la esclerosis múltiple:
a. La edad de inicio es típicamente entre 20-40 años de edad
b. La forma de presentación: secundaria progresiva es la más frecuente
c. Pese a la gran relación con factores medioambientales no se considera que
existan factores de riesgo
d. Los ataques de desmielinización deben ser tratados con fármacos
inmunosupresores

NEFROLOGÍA:

1. Un hombre de 45 años, previamente sano, desarrolla un cuadro de fiebre y gastroenteritis


aguda con importante diarrea líquida. En el análisis de sangre se observa sodio 140 mmol/L,
potasio 3,2 mmol/L, cloro 85 mmol/L y bicarbonato 38 mmol/L. El pH arterial es 7,60 y la
pCO2 arterial 42 mmHg. ¿Qué opción sería la más indicada en el paciente?
a. El paciente tiene una alcalosis metabólica.
b. Los valores de pCO2 reflejan una acidosis respiratoria compensadora.
c. El anión gap es de 17 mmol/L.
d. La hipocloremia e hipokalemia se debe a las pérdidas gastrointestinales.
2. Hombre de 83 años que acude a urgencias por astenia, malestar general y diarrea.
Antecedentes de HTA, diabético con enfermedad renal crónica (EFG 32 mL/min) e
insuficiencia cardiaca (FE 30%). En tratamiento con digoxina, nebivolol, furosemida,
alopurinol, espironolactona, enalapril, atorvastatina y sitagliptina. En la exploración destacan
signos de deshidratación de mucosas. PA 90/44 mm/Hg, FC 67 lpm. Análisis: Hb 10,8 g/dL,
glucosa 156 mg/dL, urea 125 mg/dL, creatinina 3,25 mg/dL, K 7,5 mEq/L, Na 138 mEq/L, pH
7,31, bicarbonato 16 mmol/L. Se realiza el siguiente ECG. ¿Cuál es la primera medida a
instaurar en este paciente?
a. Administrar gluconato cálcico al 10% intravenoso.
b. Corregir la acidosis metabólica con bicarbonato intravenoso.
c. Retirada de fármacos y añadir resinas de intercambio iónico.
d. Hemodiálisis.
3. ¿Cuál afirmación considera la más adecuada respecto a la fisiología renal?
a. Los segmentos de asa de Henle reabsorben ClNa con un exceso de agua, un efecto
esencial para la excreción de orina con osmolaridad diferente a la del plasma.
b. La reabsorción del bicarbonato en la nefrona se realiza principalmente en el túbulo
contorneado distal.
c. La excreción urinaria de Na y agua es igual a la suma de la cantidad filtrada a través
de los glomérulos y la cantidad reabsorbida por los túbulos.
d. En condiciones normales el 80% de la glucosa filtrada se reabsorbe en el túbulo
contorneado proximal y regresa a la circulación sistémica por los capilares
peritubulares.
4. ¿Cuál parámetro de la ecuación MDRD, NO se incluye para el cálculo del filtrado
glomerular?
a. Edad.
b. Talla.
c. Sexo.
d. Raza.
5. ¿Cuál de las substancias vasoactivas mencionadas contrae preferentemente las arteriolas
eferentes glomerulares en la mayoría de estados fisiológicos?
a. Adrenalina.
b. Noradrenalina.
c. Endotelina.
d. Angiotensina II
6. Paciente femenina de 60 años diagnosticada de broncopatía crónica, diabetes mellitus,
hipertensión arterial y tratada desde hace 4 meses con Omeprazol, Metformina, Salbutamol,
Bromuro de Ipratropio y Enalapril 20 mg + Hidroclorotiazida 25 mg. Acude a su médica por
cansancio, disminución de apetito, con ligera disnea y tos ocasional, deposiciones variables,
a veces blandas y sin síntomas urinarios. Unos análisis muestran leucocitos 10.000/mm3,
Hto 35%, VCM 80, Glucosa 150 mg/dl, Urea 80 mg/dl, Creatinina 1,6 mg/dl, Sodio 133 mq/l
y Potasio 2,9 mEq/l. ¿Cuál es la causa más probable de la hipopotasemia?
a. Insuficiencia renal aguda.
b. Hiponatremia hipoosmolar euvolémica.
c. Déficit de aporte de potasio.
d. El uso de sus antihipertensivos.
7. Usted desea calcular el aclaramiento de creatinina de un paciente de 75 años para ajustar la
dosis del antibiótico que ha de prescribirle para su cuadro de infección respiratoria. ¿La
fórmula de Cockcroft Gault permite la estimación del aclaramiento de creatinina usando lo
siguientes parámetros?
a. Edad, peso (en kg), género y creatinina plasmática.
b. Edad, etnia, género y creatinina plasmática.
c. Edad, etnia, peso (en kg) y creatinina plasmática.
d. Edad, peso (en kg), etnia y género.
8. En una nefrona, el 60% del cloruro de sodio es reabsorbido en?
a. Túbulo proximal
b. Rama descendente del asa de Henle.
c. Rama ascendente del asa de Henle.
d. Túbulo contorneado distal.
9. Anciana que llevan sus vecinos a Urgencias porque la ven algo atontada y con manchas de
deposición en la ropa. TA 100/60 mmHg, FC 100 lpm; sentada TA 70/30 mmHg, FC 105 lpm.
PVY normal. Respiración de Kussmaul. No focalidad neurológica. Peso de 50 kg. Lab: pH
7,25, PCO2 14 mmHg, Bicarbonato 5 mg/dl, Na 133 mEq/l, k 2,5 mEq/l, Cl 118 mE/l, Cr 3,4
mg/dl, NUS 60, Prot. 8 g/dl. ¿Cuál de las siguientes respuestas sería la más indicada?
a. El trastorno ácido-base que presenta es una acidosis respiratoria.
b. La compensación para corregir la acidosis no es adecuada.
c. Con esta exploración descartamos que esté deshidratada.
d. Tiene insuficiencia renal aguda de causa prerrenal.
10. En un paciente con hiperpotasemia y disminución de la excreción urinaria de potasio. ¿Cuál
de las siguientes es la causa más probable?
a. Tratamiento con espirolactona.
b. Intoxicación digitálica.
c. Síndrome de lisis tumoral.
d. Ejercicio físico.
11. ¿Cuál NO es una causa de alcalosis respiratoria?
a. Tratamiento con salicilatos.
b. Aldosteronismo primario.
c. Exposición a grandes alturas.
d. Crisis asmática.
12. Hombre de 28 años de edad que acude a urgencias del hospital por presentar hematuria
macroscópica. ¿Cuál de las siguientes alteraciones en el análisis de orina apoyaría el
diagnóstico de glomerulonefritis?
a. Hematíes dismórficos y/o cilindros hemáticos.
b. Proteinuria de 1 g/día, con resultado negativo en tira reactiva y con microalbuminuria
mayor de 300 mg/24 horas.
c. Coexistencia de hematuria con piuria sin bacteriuria.
d. Coágulos en la orina a simple vista
13. Hombre de 25 años sin antecedentes conocidos de interés. Lleva 2 meses con orinas
espumosas y cargadas, sin otra sintomatología. Exploración física sin hallazgos. Orina
elemental: pH 5; glucosa -; hemoglobina ++; proteinas ++; esterasa leucocitaria -; proteinuria
1,5 g/24 horas; Na urinario 60 mEq/L; K urinario 30 mEq/L; Cl urinario 100 mEq/L. Bioquímica
sanguínea: creatinina 1,6 mg/dL; urea 80 mg/dL; Na 140 mEq/L; K 3,8 mEq/L. Estudio
inmunológico: antiDNA negativo; ANCA negativo; antiMBG negativo; factor reumatoide
negativo; C3 20 mg/dL (normal 60-120); C4 10 mg/dL (normal 20-40). Se realiza biopsia
renal. ¿Cuál de estos diagnósticos le parece más probable encontrar en la biopsia?
a. Glomerulonefritis membranosa.
b. Glomerulonefritis de cambios mínimos.
c. Glomerulonefritis mesangial IgA.
d. Glomerulonefritis membranoproliferativa.
14. ¿Cuál de las siguientes cifras de albúmina en orina se define como microalbuminuria?
a. Menos de 30 mg en 24 horas.
b. Menos de 300 mg en 24 horas.
c. Entre 30 y 300 mg/g de creatinina.
d. Entre 300 y 1000 mg/g de creatinina.
15. ¿Cuál NO es un índice urinario en el diagnóstico del fracaso renal agudo prerrenal?
a. La osmolalidad urinaria es superior a 400 mOsm/Kg.
b. El sodio urinario es inferior a 20 mEq/l.
c. La excreción fraccional de Sodio (EFNa) es mayor al 1%.
d. El cociente entre la urea urinaria y la urea plasmática es superior a 10.

NEUMOLOGÍA:

1. Cual es el principal medicamento controlador para el Asma


a. Beta 2 agonista de acción rápida
b. Glucocorticoide Inhalado
c. Glucocorticoide sistémico
d. Anticolinérgico
2. Cual de las siguientes aseveraciones es correcta en el manejo de una crisis de Asma
a. Se debe administrar siempre 5 litros de oxígeno
b. Para revertir la obstrucción se debe administrar Adrenalina
c. El Glucocorticoide sistémico oral es equivalente al venoso
d. Debo hospitalizarlo si no responde a los 30 minutos de iniciado el tratamiento
3. Qué sucede en la fase de inflamación agua del asma?
a. Hiperreactividad bronquial parasimpática
b. Liberación de histamina y otros mediadores
c. Remodelación y reparación de tejido dañado
d. Hiperplasia de elementos celulares del pulmón
4. Cuál es la característica de las bronquiectasias saculares?
a. Están presenten en un 80% de todas las bronquiectasias
b. Alternan zonas de dilatación con zonas de constricción localizadas
c. La dilatación se acentúa distalmente terminando en un fondo
d. Tienen un aspecto de ristra de perlas
5. Escoja la respuesta correcta con relación a la hemoptisis
a. La sangre expulsada es de color rojo oscuro
b. Su clínica se compone de náuseas junto con dolor abdominal
c. El pH de la sangre es alcalino
d. La anemia es un síntoma frecuente
6. Cual de los siguientes exámenes es mas especìfico para detectar Bronquiectasias
a. Rx de Tòrax
b. Broncoscopìa
c. TAC-AR
d. Espirometrìa
7. Señale lo correcto con respecto a la quimioprofilaxis en TB
a. Isoniazida 300mg/día durante 6- 12 meses
b. Rifampicina 600mg/día por 2 meses
c. Pirazinamida 15 mg/kg/día por 5 meses
d. Etambutol 5mg/kg/día por 17 semanas
8. El riesgo de un individuo para enfermarse de tuberculosis depende de:
a. Inmunidad celular, CD4
b. Prevalencia de la tuberculosis
c. El estilo de vida de la persona
d. La genética del paciente
9. Señale la causa por la que se ve una tos seca en el derrame pleural:
a. Por la ocupación de 1/3 de la pleura
b. Por la ineficiente mecánica muscular respiratoria
c. Por la distorsión pulmonar
d. Por el origen embrionario
10. Cómo determino el sitio de punción en la toracocentesis?
a. Identifico límite superior y bajo 3 espacios intercostales
b. Identifico el límite lateral y bajo 2 espacios intercostales
c. Identifico limite inferior y subo 2 espacios intercostales
d. Identifico límite superior y bajo 2 espacios intercostales
11. En la radiografía de tórax como se identifica el límite superior del derrame pleural
a. Signo del Menisco de Damoisseau
b. Opacidad apical bilateral
c. Signo de la S de Golden
d. Signo del Diafragma Continuo
12. Los dos mecanismos fisiopatológicos de la TEP son:
a. Obstrucción funcional pulmonar y obstrucción mecánica
b. Obstrucción funcional y aumento de mediadores químicos.
c. Obstrucción mecánica y aumento de mediadores químicos.
d. Obstrucción mecánica y colapso de la vía aérea
13. Indique una de las características para clasificar a una TEP como submasiva
a. Hipotensión (TAS <90 mmHg)
b. Disfunción de ventrículo derecho
c. Shock
d. Sin descompensación hemodinámica
14. La Espirometría en una enfermedad intersticial como se encontraría
a. FEV1/FVC en 65%
b. FVC en 72%
c. FEV1 en 85%
d. Es normal
15. Dentro de los criterios diagnósticos ALAT para la fibrosis pulmonar idiopática incluye
encontrar un patrón de ¨Panal de abeja¨ que consiste en:
a. Hipertransparencia pulmonar
b. Consolidaciones de quistes de 1cm agrupados en racimos
c. Engrosamiento del tramo broncovascular
d. Espacios aéreos quísticos de diámetros variables (3-20 mm) agrupados en
varias hileras
EXAMENES CUESTIONARIO

1) Existe fármacos que pueden aumentar o disminuir la acción de un anticoagulante ¿Cuál de


estos fármacos inhibe la acción de los anticoagulantes?

A) Barbitúrico

B) Ácido acetíl salicílico

C) Rifampicina

D) Vitamina K

ANSWER: D

2) Un paciente con una Trombosis de sistema nervioso central recibe tratamiento, existe una
contraindicación absoluta de estos medicamentos ¿Cuál de estos está contraindicado?

A) Esteroide

B) Heparina de bajo peso molecular


C) Fibrinolíticos

D) Antihipertensivos

ANSWER: C

3) ¿En cuál de las enfermedades siguientes sería más probable encontrar un tiempo de
protrombina (TP) anormal?

A) Hemofilia B

B) intravascular diseminada CID

C) Hemofilia A

D) Enfermedad de Von Willebrand

ANSWER: B

4) El Hiperesplenismo es una hiperfunción del bazo, cursa con pancitopenia, esplenomegalia y


reticulocitos altos, responda la opción afirmativa.

A) Es una anemia congénita

B) Tiene un IPR < 2

C) Es una anemia arregenerativa

D) El índice de producción de reticulocitos es (IPR) > de 2

ANSWER: D

5) Un joven de 14 años con signos de varicela. ¿Cuál de estos medicamentos está


contraindicado su uso?

A) Aciclovir

B) Ibuprofeno
C) Ácido acetíl salicílico

D) Paracetamol

ANSWER: C

¿Qué evalúan los criterios de Wells para tromboembolia pulmonar?

A) La probabilidad clínica de tener tromboembolia pulmonar

B) Diagnóstico definitivo de tromboembolia pulmonar

C) Pronóstico de supervivencia para pacientes con tromboembolia pulmonar

D) Criterios radiológicos para diagnosticar tromboembolia pulmonar

ANSWER: A

Un valor de > 4 en la escala de Wells indica que:

A) El paciente está estable y es poco probable que presente embolia pulmonar

B) El paciente está inestable pero la probabilidad de que presente embolia pulmonar es baja
C) Clínicamente el paciente tiene probabilidad de Embolismo Pulmonar

D) Clínicamente el paciente no tiene probabilidad de Embolismo Pulmonar

ANSWER: C

Respecto a la magnitud del daño causado por una tromboembolia pulmonar, seleccione la
respuesta correcta:

A) Es determinante el tamaño del émbolo y el diámetro de la arteria afectada (obstruida)

B) Es únicamente importante la arteria que está siendo afectada (obstruida)

C) Lo principal es el tamaño, y son de importancia únicamente émbolos mayores a 2mm

D) Ningún factor es importante ya que el diagnóstico siempre es post mortem

ANSWER: A

¿Con cuál de los siguientes fármacos NO interacciona la rifampicina?:

A) Fenitoína

B) Metadona

C) Cloranfenicol

D) Hidróxido de aluminio

ANSWER: D

Cuál de las siguientes afirmaciones corresponden a Neumonía Adquirida en la Comunidad.


A) Pacientes que viven en hogares de cuidados crónicos.

B) Pacientes no hospitalizados en los 14 días anteriores

C) Pacientes que presentan síntomas durante las 48 primeras horas de la admisión al hospital

D) Cuadro de presentación aguda.

ANSWER: A

Cuál de las siguientes afirmaciones NO corresponde al diagnóstico diferencial de la EPOC.

A) Trombo embolismo pulmonar

B) Paro respiratorio

C) Infarto agudo de miocardio

D) Obstrucción de vía aérea superior

ANSWER: B

Uno de los siguientes NO es síntoma general de la tuberculosis.


A) Pérdida de peso

B) Síndrome febril

C) Hematemesis

D) Sudoración nocturna

ANSWER: C

En cuanto al diagnóstico funcional del asma:

A) Si la relación VEF1/CVF es mayor 0.7 se considera patrón obstructivo, diagnosticamos asma

B) Si la relación VEF1/CVF es menor a 0.7 (patrón restrictivo) y post broncodilatador obtenemos


un aumento igual o mayor al 12% en el VEF1, nos orienta hacia diagnóstico de asma

C) Si la relación VEF1/CVF es menor a 0.7 (patrón obstructivo) y post broncodilatador obtenemos


un aumento igual o mayor al 12% en el VEF1, nos orienta hacia diagnóstico de asma

D) Valores espirométricos no tienen importancia en el diagnóstico del asma.

ANSWER: C

Un paciente hipertenso con descompensación aguda “típica” de insuficiencia cardiaca sin


sobrecarga de volumen, que fármaco usaría:

A) Furosemida

B) Clortalidona
C) Dobutamina

D) Nitroprusiato

ANSWER: D

Cuál síndrome genético es común en la insuficiencia aórtica:

A) Anomalía de Ebstein

B) Síndrome de Eisenmenger

C) Síndrome de Marfán

D) Síndrome de Rastelli

ANSWER: C

Qué enfermedad reumatológica causa insuficiencia aórtica:

A) Espondilitis anquilosante

B) Artritis reumatoidea
C) Lupus eritematoso sistémico

D) Vasculitis leucocitoclástica

ANSWER: A

Cuánto es la dosis sublingual de nitroglicerina en angina de pecho:

A) 1.3 a 1.6 mg

B) 0.8 a 1.0 mg

C) 3 a 6 mg

D) 0.3 a 0.6 mg

ANSWER: D

Cuánto es la dosis máxima de dinitrato de isosorbide de liberación prolongada en angina de


pecho:

A) 160 mg

B) 120 mg

C) 300 mg

D) 40 mg

ANSWER: C
El Síndrome de Alport, que nefropatía produce?

A) Glomerulonefritis membranoproliferativa

B) Granulomatosis de Wegener

C) Glomeruloesclerosis segmentaria focal

D) Nefropatía por IgA

ANSWER: C

¿Cuál no se considera un tratamiento para hiperpotasemia?

A) Furosemida

B) Bicarbonato de calcio

C) Gluconato de calcio

D) Solución polarizante

ANSWER: B

Si el cálculo se ubica en la porción inferior del uréter, el dolor puede irradiarse en mujeres hacia?

A) Miometrio

B) Labio mayor ipsilateral

C) Útero

D) Pubis

ANSWER: B

La obstrucción ureteral bilateral crónica se caracteriza por?

A) Diuresis posterior a la obstrucción


B) Anuria

C) Hipertensión

D) Dolor renal

ANSWER: C

Cuántos mEq de Na contiene 500 ml de Cloruro de sodio al 0.09%?

A) 154 mEq/L

B) 250 mEq/L
C) 130 mEq/L

D) 132 mEq/L

ANSWER: A

En que valores se encuentra una hiponatremia leve?

A) 115 a 125 mEq/l

B) 105 a 115 mEq/l

C) 125 a 135 mEq/l

D) 95 a 105 mEq/l

ANSWER: C

La furosemida ¿qué categoría de riesgo en gestantes tiene?

A) C

B) D
C) A

D) B

ANSWER: A

De las siguientes causas de vértigo, señale cuál corresponde a causa de vértigo agudo:

A) Ataque isquémico transitorio

B) Enfermedad de Méniére´s

C) Migraña vestibular

D) Hipofunción vestibular bilateral

ANSWER: A

Dentro de las causas de delirio, señale la verdadera:

A) Hiperglicemia

B) Antipsicóticos

C) Estatus convulsivo

D) Trastornos del sueño

ANSWER: A

De los siguientes enunciados en relación a crisis y epilepsia, señale el correcto:


A) La epilepsia es una condición que produce riesgo de recurrencia de crisis

B) La crisis se produce por una repolarización asincrónica neuronal

C) Las crisis focales se originan de descargas en ambos hemisferios cerebrales

D) La parálisis de Todd´s es un fenómeno infrecuente en crisis generalizada

ANSWER: A

El síndrome de Wallenberg se produce por infarto en:

A) Arteria cerebelosa posteroinferior

B) Arteria cerebelosa anteroinferior

C) Arteria cerebelosa superior

D) Arterias paramedianas basilares

ANSWER: A

Una contraindicación para la administración de rtpa (alteplasa) es:


A) Cifras de TA >185/110 mm Hg a pesar del tratamiento

B) Tiempo de evolución desde el inicio de los síntomas <4,5h

C) Edad > o igual a 18 años

D) TAC cerebral sin evidencia de hemorragia

ANSWER: A

En relación a la hemorragia cerebral y sus factores de riesgo, señale el enunciado correcto:

A) La encefalopatía hipertensiva es una complicación de la hipertensión maligna o severa

B) La hemorragia hipertensiva resulta de ruptura espontánea de arterias de gran calibre

C) La cocaína o metanfetamina produce sangrado solo mientras el paciente está en abstinencia


D) La angiopatía amiloide es una afección de adultos jóvenes y medios

ANSWER: A

De los siguientes síntomas, señale el sitio de sangrado al que corresponden: afasia o apraxia
construccional, hemianopsia homónima.

A) Tálamo

B) Protuberancia

C) Cerebelo
D) Lóbulo occipital

ANSWER: A

En relación a la migraña, señale el enunciado correcto:

A) Es un síndrome recurrente de cefalea o dolor de cabeza

B) Un ataque de migraña tiene 3 fases: postdromo, fase dolorosa, pródromo

C) La mayor ocurrencia en mujeres jóvenes no se relaciona con aspectos hormonales

D) El paciente migrañoso no es susceptible ni tiene gatillantes de dolor

ANSWER: A

En relación a la cefalea tipo trueno o thunderclap, señale el enunciado verdadero:

A) Si se descarta lesiones orgánicas puede clasificarse como cefalea thunderclap primaria

B) La definición es dolor severo que llega a 10/10 en el lapso de 10 minutos

C) Las etiologías de la cefalea thunderclap secundaria son: tumores cerebrales, abscesos


D) El paciente requiere estudios de descarte basados en electroencefalograma

ANSWER: A

Un paciente que inicia con un estado confusional con agitación psicomotriz, cambios de la
conducta y personalidad llegando a un franco estado psicótico, además añade crisis convulsivas
tónico-clónicas generalizadas, la mejor posibilidad diagnóstica es:

A) Encefalitis

B) Meningitis

C) Absceso cerebral

D) Ventriculitis
ANSWER: A

La falta de producción de insulina en la Diabetes mellitus tipo I se produce por:

A) resistencia periférica a la insulina

B) hiperinsulinismo

C) destrucción de las células B del páncreas por reacción autoinmune

D) obesidad

ANSWER: C
Un paciente de 50 años masculino con índice de masa corporal de 30, sedentario, con hábito de
tabaquismo, acude a su consulta con un resultado de glicemia en ayunas de 118mg/dl (plasma
venoso), ¿cuál sería su siguiente acción en el estudio de diabetes?

A) Ordenar segunda muestra de sangre venosa para medir glicemia en ayunas

B) Prueba de tolerancia a la glucosa

C) HbA1c

D) Medir insulina

ANSWER: A

¿Qué tiempo de evolución natural de la enfermedad podría pasar antes de que la diabetes
mellitus tipo II presente síntomas clínicos?

A) 1 año

B) dos meses

C) 10 a 20 años

D) dos semanas

ANSWER: C

A qué edad es más frecuente el diagnóstico de Diabetes tipo I:

A) tercera década

B) 4 a 5 años

C) adulto mayor

D) antes del primer año

ANSWER: B

¿Qué células del cuerpo no requieren insulina para el ingreso de la glucosa a su interior?

A) adipocitos

B) sarcómeros

C) neuronas

D) hepatocitos

ANSWER: C

¿Cuál de los siguientes se considera un criterio para el diagnóstico de diabetes?


A) Glucosa en ayunas mayor a 100

B) Hemoglobina glicosilada mayor de 5.6

C) Paciente con síntomas de descompensación y glucosa al azar mayor a 200

D) Prueba de tolerancia a la glucosa con valores de 140 a 190 mg /dl

ANSWER: C

Con los siguientes valores de hormonas tiroideas: TSH: 7 y T4:1.3. ¿Qué diagnóstico es más
probable?

A) Hipotiroidismo primario .

B) Hipotiroidismo subclínico

C) Eutiroideo

D) Hipertiroidismo

ANSWER: B

Paciente que presenta en la escala de Burch y Watofsky un puntaje de 40 puntos nos indica cual
de las siguientes posibilidades:

A) Tormenta tiroidea

B) Tormenta tiroidea inminente

C) Tormenta tiroidea poco probable

D) Se excluye el diagnóstico de tormenta tiroidea .

ANSWER: B

Dentro de los tipos de cáncer de tiroides se encuentran los diferenciados, estos incluyen a cuál
de los siguientes:

A) Medular

B) Anaplásico

C) Linfoma

D) Papilar

ANSWER: D

Paciente que acude al hospital con lesiones hiperpigmentadas, sensación de debilidad, al


examen físico hipotensión, en la gasometría se evidencia hiponatremia, hipoglicemia .Usted
pensaría en:
A) Enfermedad de Addison .

B) Enfermedad de Cushing .

C) Hipotiroidismo .

D) Hipertiroidismo

ANSWER: A

Paciente que al examen físico se evidencia cara de luna llena, obesidad central, estrías rojo
vinoso, presencia de giba. Con estos hallazgos usted pensaría en:

A) Enfermedad de Addison

B) Hipotiroidismo

C) Enfermedad de Cushing

D) Hipertiroidismo

ANSWER: C

Paciente que presenta un valor de calcio de 7.5 y un valor de albumina de 2.8, ¿Cuál de los
siguientes seria el valor del calcio corregido?

A) 8.0

B) 8.4

C) 8.6

D) 8.9

ANSWER: C

La clasificación de hipoglicemia (según el diabetes Care 2022) el nivel 2 corresponde a cuál de


los siguientes valores de glucosa:

A) Menor de 70 mg/dl

B) Menor a 54 mg / dl

C) Menor a 60 mg / dl

D) Menor a 100 mg / dl

ANSWER: B

¿Los acrocordones y la acantosis nigricans nos sugiere cuál de las siguientes patologías?

A) Resistencia a la insulina

B) Hipotiroidismo
C) Enfermedad de Addison

D) Enferemedad de Cushing

ANSWER: A

Según el diabetes care 2022 ¿cuál sería el rango de glucosas preprandial que debería manejar
un paciente diabético ( no mujer embarazada )

A) Glucosa entre 60 -100 mg / dl

B) Glucosa entre 80 – 130 mg / dl

C) Glucosa entre 150 – 180 mg/ dl

D) Glucosa entre 180 – 200 mg/dl

ANSWER: B

Linfoma es una patología maligna, primaria de ganglio linfático, sus ganglios son no dolorosos,
blandos. ¿Qué es el signo de Hoster?

A) El ganglio en linfoma es doloroso con la ingestión de alcohol

B) Ganglios con abcesos

C) Cuando el ganglio hace fístula

D) Ganglios pétreos

ANSWER: A

El embarazo cursa con un estado de hipercoagulabilidad, por aumento de factores de


coagulación y de plaquetas, lo que constituye un alto riesgo de trombosis, por lo cual amerita
anticoagulación. ¿Responda cuál anticoagulante está contraindicado en el embarazo?

A) Heparina de bajo peso molecular (HBPM)

B) La warfarina sódica

C) Las nuevas heparinas

D) Heparina no fraccionada (HNF)

ANSWER: B

El índice de producción de reticulocitos (IPR), clasifica las anemias en arregenerativas y


regenerativas si el IPR es >2 o <2 ¿Cuál de estas anemias presenta IPR >2?

A) En una hemorragia aguda

B) Anemia de los procesos Crónico

C) Anemia por déficit de hierro


D) En Hipoplasia medular

ANSWER: A

Existen enfermedades como el Lupus, que pueden complicarse con AHAI. ¿Cuál es la prueba que
hace diagnóstico de anemia hemolítica autoinmune?

A) Electroforésis de hemoglobina

B) El frótis periférico

C) La prueba de coombs directa

D) Anticuerpos antinucleares

ANSWER: C

El plasma es la parte líquida de la sangre separada de la parte celular, contiene los factores de
coagulación, necesarios en manifestaciones hemorrágicas, de algunas enfermedades. Señale su
indicación en estas patologías.

A) Para ayudar a cicatrizar heridas

B) El niño con PTI con petequias

C) Hemofilia A

D) Hemofilia B

ANSWER:D

Qué antidepresivo causa disfunción del nódulo sinoauricular:

A) Fluoxetina

B) Sertralina

C) Amitriptilina

D) Midazolam

ANSWER: C

Cuánto es la dosis de dabigatrán con CCr >30 mL/min:

A) 150 mg cada 24 horas

B) 150 mg cada 8 horas

C) 150 mg cada 6 horas

D) 150 mg cada 12 horas

ANSWER: D
Cuántos Joules se usa en la fibrilación auricular con hipotensión profunda, edema pulmonar o
angina:

A) 200 Joules

B) 100 Joules

C) 150 Joules

D) 250 Joules

ANSWER: A

Qué familia de antibiótico no causa prolongación de QT y de taquicardia ventricular polimorfa:

A) Macrólidos

B) Quinolonas

C) Cefalosporinas

D) Antagonistas del folato


ANSWER: C

Qué endocrinopatía no causa prolongación de QT y de taquicardia ventricular polimorfa:

A) Hipotiroidismo

B) Hipogonadismo

C) Hiperparatiroidismo

D) Hiperladosteronismo

ANSWER: B

¿Qué valor define anuria?

A) 0 a 100 ml

B) 500 a 1000 ml

C) 100 a 400 ml

D) 1000 a 2000 ml

ANSWER: A

¿Cuál no se considera causa para azotemia prerrenal?

A) Hemorragia digestiva alta


B) Hiperplasia prostática

C) Vómito a repetición

D) Diarrea aguda

ANSWER: B

Según Harrison, los factores que predisponen a las mujeres a padecer cistitis, aumentan el
peligro de:

A) Absceso perirenal

B) Vaginitis

C) Pielonefritis

D) Uretritis

ANSWER: C

¿Cuál de las siguientes alternativas es el tratamiento sistémico de elección en impétigo


contagioso?

A) Eritromicina

B) Ampicilina

C) Dicloxacilina

D) Azitromicina

ANSWER: C

Paciente de sexo masculino, de 48 años de edad que presenta una tumefacción eritematosa de
bordes mal definidos, muy dolorosa a la palpación en tórax posterior de consistencia semidura,
con un pequeño orificio central a través del cual drena material purulento de mal olor de 8 días
de evolución. ¿Cuál de las siguientes alternativas corresponde al diagnóstico más probable?

A) Erisipela
B) Forúnculo

C) Foliculitis profunda

D) Celulitis

ANSWER: B

Paciente de sexo masculino, de 48 años de edad que presenta una tumefacción eritematosa de
bordes mal definidos, muy dolorosa a la palpación en tórax posterior de consistencia semidura,
con un pequeño orificio central a través del cual drena material purulento de mal olor de 8 días
de evolución. ¿Cuál de las siguientes alternativas indica las bacterias que con más frecuencia
ocasionan este cuadro?

A) Estreptococo beta hemolítico del grupo a, neumococo, eschericia coli

B) Estreptococo beta hemolítico del grupo a, pseudomona, h. Influenza

C) Estafilococo aureus, bacterias anaerobias, gramnegativos

D) Estafilococo aureus, clostridium, pseudomona

ANSWER: C

Paciente de sexo masculino, de 48 años de edad que presenta una tumefacción eritematosa de
bordes mal definidos, muy dolorosa a la palpación en tórax posterior de consistencia semidura,
con un pequeño orificio central a través del cual drena material purulento de mal olor de 8 días
de evolución. ¿Cuál de las siguientes alternativas indica los antibióticos de elección en este
caso?

A) Dicloxacilina y trimetroprim sulfametoxazol

B) Dicloxacilina y linezolid

C) Amoxicilina + ácido clavulanico

D) Ceftriaxona y gentamicina

ANSWER: A

Paciente de sexo femenino de 30 años de edad que presenta desde la infancia placas
eritematodescamativas de bordes bien definidos en codos, y rodilla con descamación intensa y
prurito moderado, además presenta eritema y descamación profusa en cuero cabelludo. ¿Cuál
de las siguientes alternativas corresponde al diagnóstico clínico más probable?

A) Dermatitis atópica + dermatitis seborreica de cuero cabeludo

B) Dermatitis seborreica + tiña corporis

C) Psoriasis

D) Psoriasis + dermatitis seborreica de cuero cabelludo

ANSWER: C

De los siguientes enunciados en relación a ictus isquémico, señale el verdadero:

A) El cigarrillo es un factor de riesgo para aterosclerosis por lo que el tratamiento de


deshabituación se recomienda en prevención secundaria únicamente

B) En estenosis carotidea sintomática en grado severo (> o igual al 70%) el tratamiento de


elección es quirúrgico (endarterectomía)

C) En aterosclerosis intracraneal sintomática se recomienda el tratamiento con stenting


intracraneal que es superior al tratamiento con aspirina
D) La fisiopatología de las causas cardioembólicas en ictus isquémico se basa en la trombosis
secundaria a lipohialinosis con afectación de vasos pequeños

ANSWER: B

En relación a la hemorragia cerebral, señale el enunciado verdadero:

A) Una hemorragia de gran tamaño puede deteriorar el estado de conciencia se un paciente e


inclusive llevarlo al coma

B) Las hemorragias talámicas producen hemiparesia ipsilateral y afasia si la afectación es del


lado no dominante

C) En la fisiopatología de hemorragia hipertensiva se destaca la afectación crónica de la pared


del vaso de gran calibre

D) La hemorragia produce déficit neurológico focal como la cefalea y el vómito lo que la hace
indistinguible del ictus isquémico

ANSWER: A

En relación a la hemorragia subaracnoidea, señale el enunciado verdadero:

A) La cefalea es un síntoma muy infrecuente en la presentación clínica

B) La etiología aneurismática es la causa más frecuente de hemorragia no traumática

C) De los principales factores de riesgo de mencionan la diabetes, dislipidemia y obesidad

D) El tratamiento es dirigido a resolver la causa del sangrado por lo que la cirugía se la debe
realizar en las primeras 48h

ANSWER: B

En relación a las infecciones del sistema nervioso central, señale el enunciado verdadero:

A) En las etiologías de meningitis crónica junto a la tuberculosis se debe considerar a la sífilis y a


la enfermedad de Lyme entre otras

B) Para el tratamiento de la meningitis tuberculosa se emplea fármacos antituberculosos


isoniazida, etambutol, pirazinamida, rifampicina por 1 mes

C) Los abscesos cerebrales se diseminan por sangre en un porcentaje bajo de pacientes,


contrario a los criptogénicos (fuente primaria desconocida) que son los más frecuentes

D) El tratamiento de todos los casos de absceso cerebral es quirúrgico y en casos puntuales se


adiciona tratamiento médico con antibióticos de amplio espectro

ANSWER: A

En un paciente masculino de 38 años de edad, que acude por presentar un cuadro de 2 días de
evolución de debilidad de las 4 extremidades iniciando en miembros inferiores y progresando
hasta miembros superiores, en su examen neurológico se encontró una cuadriparesia con
balance motor 2/5, con arreflexia y dísnea, cuál sería su actuación en la sala de urgencia:

A) Solicitar una resonancia de columna por la sospecha de mielopatía compresiva

B) Indicar plasmaféresis o inmunoglobulina IV con sospecha de sd de Guillain Barré

C) Realizar antes de la prescripción farmacológica gasometría, rx tórax para definir el diagnóstico

D) Solicitar Punción lumbar a su ingreso para confirmar el diagnóstico de sd de Guillain Barré y


luego administrar tratamiento

ANSWER: B

Paciente femenino de 67 años acude a la consulta por presentar un cuadro de distensión


abdominal progresiva en las últimas 3 semanas, por lo que ha aumentado de peso unos 7 kg en
su última medición. Niega haber aumentado el apetito, más bien, siente saciedad precoz. Como
antecedentes es diabética y tiene un IMC de 30. ¿Cuál cree usted la causa más probable de ese
aumento de peso?

A) METEORISMO

B) DIABETES DESCOMPENSADA

C) ASCITIS

D) INCREMENTO DE LAS CALORIAS EN LA DIETA

ANSWER: C

Paciente con cirrosis de 5 años de evolución, ha tenido en el pasado un episodio de sangrado


digestivo variceal. Se le trató con ligaduras y actualmente toma propanolol 40 mg por via oral
cada 12 horas. En el último control ecográfico se nota un nódulo de 3 cm en el segmento 5 del
hígado, con aparente vascularidad al Doppler. Se le pide una alfa fetoproteina que se encuentra
en 450. ¿Cuál cree usted sería el diagnóstico?

A) NÓDULO DE REGENERACIÓN

B) QUISTE SIMPLE

C) HEMANGIOMA

D) HEPATOCARCINOMA

ANSWER: D

Paciente masculino con cirrosis de etiología alcohólica activa a la fecha del ingreso, los familiares
indican que ha tomado alcohol hasta el día anterior al ingreso. Lo notan desorientado, no
reconoce a los familiares, asténico y con hiporexia. Al examen físico se encuentra somnoliento,
con temblor en las manos. ¿Cuál cree que sería el diagnóstico?

A) ENCEFALOPATIA URÉMICA
B) CETOACIDOSIS DIABÉTICA

C) ECV ISQUEMICO

D) ENCEFALOPATÍA HEPÁTICA

ANSWER: D

Paciente femenino de 55 años con ictericia y coluria. Cuadro de unos 2 meses de evolución,
niega antecedentes previos de anemia, tiene un diagnóstico de hemocromatosis, para lo cual se
ha tratado con sangrías para disminuir las concentraciones de hierro en niveles terapéuticos.
Tiene un laboratorio con albúmina de 1.5 y un INR en 1,7. Las bilirrubinas totales en 7 a
predominio de la directa en 5. Señale el diagnóstico más probable.

A) HEMÓLISIS

B) COLEDOCOLITIASIS
C) CIRROSIS

D) HEPATOXICIDAD

ANSWER: C

Paciente femenino de 57 años con cirrosis por NASH desde hace 7 años, tiene ascitis hace un
año, en este ingreso presenta aumento progresivo del perímetro abdominal a pesar de estar en
diuréticos a dosis de 160 mg de furosemida y 400 mg de espironolactona, los volúmenes
urinarios a menos de 500 cc diarios. Una paracentesis muestra 140 células y polimorfonucleares
del 40%. Señale el diagnóstico más probable.

A) PERITONITIS BACTERIANA ESPONTÁNEA

B) ASCITIS REFRACTARIA

C) SINDROME HEPATORENAL

D) SINDROME HEPATOPULMONAR

ANSWER: B

Paciente masculino de 65 años, refiere consumo de alcohol por varios años, cada semana y llega
a la embriaguez, por lo cual ha tenido problemas laborales y familiares. Nunca se ha hecho un
control, pero se siente preocupado por una masa que se palpa en el cuadrante superior
izquierdo desde hace unas semanas y acude a la consulta. Señale usted de acuerdo con la
historia cuál podría ser el diagnóstico más probable.

A) HEPATOMEGALIA

B) ESPLENOMEGALIA

C) ASCITIS

D) DILATACIÓN DEL COLON


ANSWER: B

Paciente femenina de 65 años, le diagnostican cirrosis hace una semana. Ella indica en su historia
médica que jamás ha bebido alcohol, no es hipertensa ni diabética, se le hicieron pruebas para
etiología viral, autoinmune, estudios del hierro, cobre que son todos negativos. Lo único
llamativo es que tiene un IMC de 31 y que toda su vida ha tenido problemas con el peso. Señale
usted que etiología podría estar relacionado con el desarrollo de su cirrosis.

A) ENFERMEDAD DEL HIGADO GRASO NO ALCOHÓLICO

B) CRIPTOGENÉTICA

C) FIBROSIS QUISTICA

D) CARDIACA

ANSWER: A

Paciente femenina de 25 años de edad con una historia de 3 meses con diarrea, refiere tener
distensión abdominal y abundantes gases. Ha bajado de peso unos 4 kg en esos meses. Dice que
ha notado que sus deposiciones son a veces blanquecinas y no tienen sangre. Señale de la
siguiente lista un examen de laboratorio que es importante para la evaluación de la etiología de
su diarrea crónica.

A) ANTICUERPOS CONTRA H PYLORI

B) DETERMINACIÓN DE P.C.R

C) ANTICUERPOS PARA ENFERMEDAD CELIACA

D) BIOMETRIA HEMÁTICA

ANSWER: C

Paciente masculino de 32 años, refiere una historia de al menos unos 10 años con deposiciones
diarreicas, intermitentes, no sangre, no baja de peso. Le han hecho unas 3 colonoscopias y
siempre le dicen que están normales. Una biopsia indica colitis crónica inespecífica.
Ocasionalmente relaciona las diarreas con eventos estresantes laborales y familiares. Señale el
diagnóstico más probable.

A) ENFERMEDAD CELIACA

B) COLITIS ULCERATIVA

C) ENFERMEDAD DE CRHON

D) SINDROME DE INTESTINO IRITABLE PATRÓN DIARREA

ANSWER: D
Paciente femenina de 26 años, tiene una historia de 6 meses con diarreas sanguinolentas con
moco. Refiere que le han tratado ya con antiparasitarios y antibióticos en varias ocasiones. Ha
bajado de peso unos 4 kg en estos meses. Le han propuesto que se haga una colonoscopia, pero
se ha negado en un par de ocasiones. En sus laboratorios hay anemia microcítica, tiene un PCR
elevada y una calprotectina fecal mayor a 200. Señale el diagnóstico más probable.

A) INSUFICIENCIA PANCREATICA EXÓCRINA

B) ENFERMEDAD INFLAMATORIA INTESTINAL

C) CÁNCER DE PANCREAS

D) CANCER GÁSTRICO

ANSWER: B

Paciente de 50 años que presenta un derrame pleural con las siguientes características: aspecto
pajizo, Ph 7.3, cociente de proteínas pleura/suero 0.8, cociente de LDH pleura/suero 0.9, Gram
y Ziehl negativos, lípidos totales, colesterol y triglicéridos normales, células mesoteliales <5%,
intensa linfocitosis sin atipias, ADA 64 U/l. ¿Qué diagnóstico le sugiere?

A) Empiema pleural.

B) Derrame pleural por insuficiencia cardiaca (trasudado).

C) Mesotelioma pleural.

D) Derrame pleural tuberculoso.

ANSWER: D

¿Cuáles son los factores de riesgo débiles para TEP?

A) Quimioterapia, embarazo

B) Fractura de cadera, cirugía mayor

C) Ancianos, obesidad, venas varicosas

D) Hemofilia, lesión medular

ANSWER: C

El mecanismo por el cual la infección por cólera produce una diarrea tan intensa es por:

A) Vibrios enteroagregativos.

B) Toxina Shiga-like

C) Toxina enterogénica

D) Vibrios entero hemorrágicos.

ANSWER: C
En cuanto a la profilaxis antitetánica en el tratamiento sistemático de las heridas, en una herida
pequeña limpia sin ningún antecedente de vacunación antitetánica Usted recomendaría:

A) Vacunación antitetánica de inmediato.

B) Solo limpiar la herida y dar indicaciones de cuidado al paciente, ofrecer la anti toxina.

C) Es indicativo de inmunoglobulina.

D) Dar antibiótico de manera profiláctica.

ANSWER: B

La ictericia en un paciente con Malaria es frecuente a partir:

A) Siempre existirá fiebre e ictericia debido a la destrucción de eritrocitos.

B) Ocasionalmente en niños debido a su alta susceptibilidad

C) La Ictericia en Malaria es frecuente a partir de los 15 años.

D) La ictericia no aparece en la malaria.


ANSWER: C

Una de las características principales por las cuales el dengue puede diseminarse es:

A) Debido a la gran capacidad de diseminación del vector Anopheles.

B) El vector es muy cercano a los asentamientos humanos.

C) Una característica precoz es la trombocitosis y neutrofilia.

D) Es sumamente frecuente la progresión de daño encefálico.

ANSWER: B

Tras la infección de un niño con el virus de la Influenza con la siguientes características: fiebre
intensa de inicio súbito, disnea y cianosis a la Rx de tórax se aprecia patrón asociado con
infiltrados intersticiales difusos e hipoxia intensa, usted sospecharía de:

A) Neumonía bacteriana secundaria.

B) Neumonía bacteriana primaria.

C) Neumonía viral primaría.

D) Neumonía viral secundaria.

ANSWER: C
Dentro de las manifestaciones tardías de la sífilis tenemos trastornos cardiovasculares de cual
podemos desprender el siguiente:

A) Insuficiencia cardiaca congestiva.

B) Aneurismas

C) Flebitis irritativa.

D) Trastornos del endotelio

ANSWER: B

En la clasificación de OMS de adultos con VIH – Sida en el estadio C3 tenemos a:

A) Mayor de 500 células CD4+ Asintomático.

B) Mayor de 400 células CD4+ con síntomas no definidores de Sida.

C) Menor de 200 células CD4+ con síntomas definidores de Sida.

D) Menor de 300 células CD4+ con síntomas definidores de Sida.

ANSWER: C

¿De qué depende la inmunidad contra Leptospira?

A) Producción de anticuerpos contra LPS inespecíficos.

B) Producción de anticuerpos circulantes contra LPS específicos de un serotipo.

C) Producción de anticuerpos contra los distintos serotipos.

D) Producción de anticuerpos contra proteínas inespecíficas de un serotipo.

ANSWER: B

El interferón es un medicamento útil para las siguientes infecciones excepto:

A) Hepatitis B

B) HPV

C) Sarcoma de Kaposi

D) VIH

ANSWER: D

Que caracteriza a una primoinfección por herpes tipo 1:

A) distribución en dermatoma en especial tórax

B) pápulas y pústulas intersticio entre piel y mucosa


C) faringitis o gingivoestomatitis

D) trastornos de córnea y conjuntiva

ANSWER: C

De los siguientes enunciados en relación a la diferencia entre vértigo central y periférico, señale
el correcto:

A) La fijación visual inhibe el nistagmus de una lesión periférica

B) El nistagmus en una lesión central aguda es unidireccional

C) El test de impulso cefálico ausente sugiere una lesión periférica

D) La pérdida auditiva sugiere una lesión central

ANSWER: A

En relación al delirio, señale el enunciado verdadero:

A) El déficit atencional es la característica clave de este trastorno


B) El electroencefalograma revela actividad rápida difusa

C) El déficit de dopamina es un factor clave en la fisiopatología del trastorno

D) Fármacos anticolinérgicos se usan en el tratamiento del trastorno

ANSWER: A

En relación a los diferentes tipos de crisis, señale el enunciado correcto:

A) Las crisis mioclónicas están comúnmente asociadas con desórdenes metabólicos, injuria
anóxica cerebral, etc

B) Las crisis atónicas tienen una breve pérdida del estado de conciencia con un estado
confusional post-ictal

C) Las crisis tónicas-clónicas generalizadas ocurren solo por procesos orgánico-estructurales


cerebrales

D) La alteración del estado de conciencia en las ausencias atípicas es más corto, que en las
ausencia típicas

ANSWER: A

Al producirse un infarto cerebral, señale la sintomatología relacionable con la cerebelosa


superior:

A) Ataxia ipsilateral al infarto

B) Afasia global
C) Apraxia para la vestimenta

D) Hemiparesia contralateral al infarto

ANSWER: A

El índíce de producción de reticulocitos (IPR) es un estudio que mide la capacidad de respuesta


de la medula. Si tenemos un IPR menor de 2. ¿Cuál de estas anemias presentan un IPR menor
de 2?

A) Anemia hemolítica

B) Hemorragia aguda

C) Hiperesplenismo

D) Hipoplasia medular.

ANSWER: D

Los productos de degradación de la fibrina (PDF) podemos medirlos a través de los Dímeros D,
su contaje elevado mayor de1000 (V.N 500) indicaría:

A) Tromboembolismo pulmonar

B) Derrame Pleural

C) Neumonia.

D) Hipertensión pulmonar

ANSWER: A

La Heparina no fraccionada (HNF) su uso puede causar algunas complicaciones por su acción
anticoagulante, sobre cuál factor de coagulación actúa principalmente?

A) Factor V

B) Factor VIII

C) Factor II Protrombina

D) Factor I

ANSWER: C

¿Cuál de estas patologías con manifestaciones de sangrado purpúrico, se identifica su frotis de


sangre periférica por la presencia de plaquetas gigantes?

A) Enfermedad de Von Willebrand

B) Síndrome urémico hemolítico SUH


C) Purpura trombocitopénica trombótica PTT

D) Enfermedad de Bernard Soulier

ANSWER: D

A) El cigarrillo es un factor de riesgo para aterosclerosis por lo que el tratamiento de


deshabituación se recomienda en prevención secundaria únicamente

B) En estenosis carotidea sintomática en grado severo (> o igual al 70%) el tratamiento de


elección es quirúrgico (endarterectomía)

C) En aterosclerosis intracraneal sintomática se recomienda el tratamiento con stenting


intracraneal que es superior al tratamiento con aspirina

D) La fisiopatología de las causas cardioembólicas en ictus isquémico se basa en la trombosis


secundaria a lipohialinosis con afectación de vasos pequeños
ANSWER: B

De la etiología de hemorragia cerebral, señale el enunciado verdadero:

A) La coagulopatía produce hemorragias se localización lobar

B) La angiopatía amiloide se debe sospechar en pacientes menores de 60 años

C) Las malformaciones arteriovenosas pueden causar sangrado intraventricular, subaracnoideo


e intraparenquimatoso

D) Las hemorragias por tumores cerebrales se producen tardíamente en la evolución luego de


tener otros síntomas neurológicos

ANSWER: C

Cuál de los siguientes es un signo característico de las vías respiratorias de las personas que
sufren asma:

A) Infiltración eosinofílica

B) Infiltración basófila

C) Adhesión de los glóbulos rojos al endotelio

D) Transformación de los basófilos en macrófagos

ANSWER: A

El hallazgo más frecuente en la radiografía de tórax en un paciente con asma es:

A) Hiperinsuflación pulmonar.

B) Condensaciones alveolares bilaterales y difusas.


C) Radiografía de tórax normal.

D) Engrosamiento de paredes bronquiales.

ANSWER: C

¿Cuál es la fisiopatología de la hemoptisis?

A) Hipervascularización de la circulación pulmonar, fiebre y regeneración alveolar

B) Hipervascularización de la circulación pulmonar, hipertensión pulmonar y remodelación.

C) Hipervascularización de la circulación brónquica, hipertensión pulmonar y neovascularización

D) Hipervascularización de la circulación brónquica, hipertensión pulmonar y disminución de


coagulabilidad

ANSWER: C

El tratamiento de las Bronquiectasias se basa en 3 pilares, excepto:

A) Eliminar la obstrucción bronquial.


B) Mejorar la eliminación de las secreciones, que se consigue con una adecuada hidratación, con
fisioterapia respiratoria y drenaje postural mantenidos.

C) Controlar las infecciones con el uso de antibióticos en las agudizaciones durante 10-15 días.

D) Revertir el remodelamiento bronquial

ANSWER: D

El concentrado plaquetario constituye el derivado con mayor riesgo de contaminación. ¿Cuál es


la causa?

A) Debido a mantener a -20 grados en congelación

B) Por estar a 26 grados de temperatura en el ambiente

C) Se mantiene a 6 grados en nevera

D) Se encuentra a 35 grados de temperatura.

ANSWER: B

La anemia hemolítica microangiopática es un cuadro clínico, que se presenta en patologías como


la coagulación intravascular diseminada, la presencia de esquitocitos en el frotis periférico es
una imagen característica. ¿Qué representa?

A) Buena respuesta de la médula

B) Hematíes fragmentados
C) Reticulocitos

D) Fibrina fragmentada

ANSWER: B

Un paciente con quemaduras de un 30% de su cuerpo, presenta además de sus lesiones un


edema importante. ¿Cuál de estos derivados pedirías para ayudar a resolver el edema?

A) Concentrado plaquetario

B) Albumina

C) Concentrado globular

D) Inmunoglobulina

ANSWER: B

Este paciente con quemaduras le van a cumplir posteriormente plasma, el paciente es grupo O
(-) Banco de sangre envía un plasma. ¿Cuál de estos podemos cumplir?

A) Plasma O +

B) Plasma A +

C) Plasma A B -

D) Plasma B +

ANSWER: C

En una infección pro Cándida en mucosa oral, indique la premisa correcta:

A) Invasión profunda con daño extenso de tejido.

B) El tratamiento de elección es la caspofungina vía parenteral.

C) Es importante saber el estado inmunitario del paciente para establecer el posible diagnóstico.
D) Constituye un hongo hifa, filamentoso de colonias secas.

ANSWER: C

Con respecto a la vacunación de la fiebre amarilla este tiene una inmunidad con un plazo de:

A) Plazo no mayor a 3 años.

B) Un plazo de 10 días a 10 años.

C) De por vida del huésped susceptible.

D) Únicamente 12 meses.
ANSWER: B

Paciente masculino de 65 años que fue sometido a una cirugía de Whiple por un cáncer
pancreático hace 8 meses. Refiere que luego de la cirugía empezó a tener deposiciones blandas
y en los últimos meses diarreicas, ocasionalmente blanquecinas y puede notar a veces partículas
de aceite o grasa en el agua del inodoro. Ha bajado de peso unos 7 kg. En un último rastreo con
PET scan indica nevatividad para neoplasia. Señale el diagnóstico más probable.

A) REACTIVACIÓN DEL CÁNCER PANCREÁTICO

B) METÁSTASIS PERITONEALES

C) DEPRESIÓN

D) INSUFICIENCIA PANCREÁTICA EXÓCRINA

ANSWER: D

Paciente de sexo femenino de 30 años de edad que presenta desde la infancia placas
eritematodescamativas de bordes bien definidos en codos, y rodilla con descamación intensa y
prurito moderado, además presenta eritema y descamación profusa en cuero cabelludo. ¿Cuál
de las siguientes alternativas corresponde al tratamiento tópico?

A) Inhibidores de la calcineurina asociados a calcipotriol

B) Corticoesteroides tópicos asociados a calcipotriol

C) Emolientes y corticoides topicos

D) Tacrolimus + tazaroteno

ANSWER: B

El dermatofito más común en la tiña pedis interdigital es, señale la alternativa correcta:

A) Epidermophyton

B) Tricophyton rubrum

C) Microsporun canis

D) Tricophyton mentagrophytes.

ANSWER: D

En la urticaria crónica resistente a antihistaminicos anti h1 se puede utilizar

A) Ciproheptadina

B) Ciclosporina

C) Infliximab
D) Rituximab

ANSWER: A

En la urticaria crónica autoinmune un medicamento de gran utilidad con pocos efectos


secundarios es:

A) Cimetidina

B) Levocetirizina

C) Ciclosporina

D) Loratadina

ANSWER: C

Dentro de la fisiopatogenia de la infección por VIH tenemos a los coreceptores principales en la


infección estos son:

A) Ácido siálico e integrasa.

B) Ácido hialurónico y CD59

C) CCR5 y gp 120

D) CCR5 y CXCR4

ANSWER: D

El tratamiento de las Bronquiectasias se basa en 3 pilares, excepto:

A) Eliminar la obstrucción bronquial.

B) Mejorar la eliminación de las secreciones, que se consigue con una adecuada hidratación, con
fisioterapia respiratoria y drenaje postural mantenidos.

C) Controlar las infecciones con el uso de antibióticos en las agudizaciones durante 10-15 días.

D) Revertir el remodelamiento bronquia

ANSWER: D

¿Cuál es la clasificación anatómica de las bronquiectasias?

A) Lobares, arteriales, globulares

B) Cubicas, cilíndricas, alargadas

C) Vasculares, semilunares saculares

D) Cilíndricas, varicosas, saculares

ANSWER: D
La causa principal de un fracaso en el tratamiento de la tuberculosis pulmonar es

A) Abandono del tratamiento por el paciente

B) Resistencia primaria a las drogas

C) Resistencia secundaria a las drogas

D) Toxicidad hepática

ANSWER: A

De los siguientes factores, cuál es el que debe estar alterado para que un paciente infectado de
tuberculosis se convierta en persona enferma:

A) Medio ambiente: Que haya elevada prevalencia de tuberculosis

B) Huésped: que genéticamente esté predispuesto a infectarse

C) Medio Ambiente: Que viva en un lugar frío

D) Huésped: Inmunidad celular deficiente, en especial CD4


ANSWER: D

La hipoplasia medular, consiste en un daño de la médula, lesionando célula madre mieloide.


¿Cuál de estas opciones es criterio de hipoplasia medular severa?

A) Petequias, las mucositis

B) Reticulocitos <de 1 corregido, neutrofilos < de 500 cifras absolutas

C) Epistaxis, sangrado por mucosas

D) La fiebre y anemia

ANSWER: B

Paciente adulto con una anemia sintomática. ¿Cuál es la cifra límite de hemoglobina aceptada
para ser transfundido?

A) Hemoglobina 8 gr/dl

B) No hay límite c.

C) Hemoglobina 10 gr/dl

D) Hemoglobina en 7 gr/ dl

ANSWER: D
Una familia (1) reclaman que su bebé que le entregaron en la maternidad no es su hijo, y que el
suyo es el bebé que tiene la familia (2), que lo niegan, Se estudian sus grupos sanguíneos, con
estos resultados. Familia 1: madre A, Padre O, bebé O, y la Familia 2: madre AB, padre O, y
bebé A. ¿Qué familia tiene la razón?

A) Hay un error en las pruebas

B) Familia 2

C) Familia 1

D) Ninguna

ANSWER: B

¿Cuál es el grupo sanguíneo que se identifica como receptor Universal?

A) El grupo AB +

B) Grupo O-

C) El grupo AB –

D) El grupo A

ANSWER: A

Los grupos sanguíneos tienen la importancia de identificar diferentes grupos de antígenos, en la


membrana de los glóbulos rojos, esto tiene la importancia para las transfusiones, y también
ayuda en la identificación de reclamo de paternidad. Una pareja tiene cuatro hijos, el padre es
grupo AB, la madre es grupo sanguíneo B, ¿Cuál no es su hijo?

A) Niño Grupo AB

B) Niño Grupo B

C) Niño Grupo O

D) Niño Grupo A

ANSWER: C

¿Cuál es la fisiopatología de la poliuria en la diabetes mellitus descompensada?

A) falta de hormona antidiurética

B) hiperfiltración glomerular

C) diuresis osmótica por hiperglicemia

D) uso de diuréticos

ANSWER: C
¿Cómo se encuentran los niveles de TSH en el hipotiroidismo primario sintomático?

A) Normal

B) Incrementada

C) Disminuida

D) No tiene importancia clínica

ANSWER: B

En qué edad es más frecuente el diagnóstico de hipotiroidismo?

A) adolescentes

B) infancia

C) 40 años

D) más de 60 años

ANSWER: D

El siguiente resultado de LCR: células blancas 200, de predominio polimorfonuclear, proteínas


600 mg/dl, hipoglucorraquia de 26 mg/dl, correspondería a qué enfermedad:

A) Encefalitis viral

B) Meningitis viral

C) Meningitis tuberculosa

D) Meningitis bacteriana

ANSWER: D

Paciente de 48 años sin APP, reside en una comunidad rural de la provincia de Cotopaxi,
agricultor, acude por 1 crisis convulsiva focal motora izquierda, en el trabajo diagnóstico que
requiere para averiguar la etiología de la crisis, usted señalaría que enunciado como verdadero:

A) El hallazgo en TAC cerebral de calcificaciones parenquimatosas denotan un criterio absoluto


de neurocisticercosis

B) La demostración histológica del parásito a través de una biopsia es considerado un criterio


menor de diagnóstico de neurocisticercosis

C) Se puede solicitar determinación de anticuerpos anticisticercosis o detección de antígenos


que son parte de los criterios diagnósticos de cisticercosis

D) La zona de residencia endémica para cisticercosis es considerado un criterio de exposición


pero no forma parte de los criterios diagnósticos
ANSWER: C

Paciente de 22 años de género femenino, con APP: Neuritis óptica 8 meses atrás, acude por un
cuadro de paraparesia de instauración aguda, se realizó un estudio de resonancia magnética de
la médula espinal encontrándose una lesión hiperintensa longitudinalmente extensa (4
segmentos vertebrales), usted consideraría como diagnóstico:

A) Mielitis transversa aguda

B) Mielitis secundaria a infección viral

C) Neuromielitis óptica

D) Malformación arteriovenosa medular

ANSWER: C

Si se necesita el aumento de plaquetas más efectivo en una trombocitopenia severa inmune.


¿Qué utilizas de primera línea?

A) Metilprednisolona

B) Inmunoglobulina

C) Eltrombopag

D) Rituximab anticuerpo monoclonal

ANSWER: B

Una reacción transfusional es una respuesta adversa a la transfusión de un derivado sanguíneo.


¿Cuál es la reacción más frecuente?
A) Reacción hemolítica aguda

B) Reacción hemolítica crónica

C) Reacción febril no hemolítica

D) Urticaria
ANSWER: C

Qué vitamina causa insuficiencia cardiaca con alto gasto:

A) B

B) D

C) E

D) A
ANSWER: A

Paciente que presenta cifras tensionales en el consultorio dentro de parámetros normales, pero
evidencia de retinopatía hipertensiva. Usted pensaría en

A) Hipertension de bata blanca

B) Hipertension enmascarada

C) Hipertension resistente

D) Hipertension refractaria

ANSWER: B

Paciente hipertenso con uso de 3 antihipertensivos uno de los cuales es un diurético que persiste
con presiones arteriales elevadas. Usted pensaría en
A) Hipertension de bata blanca

B) Hipertension enmascarada

C) Hipertension resistente

D) Hipertension refractaria

ANSWER: C

Un paciente con sospecha de encefalitis, debe tener, EXCEPTO:

A) Signos de focalidad neurológica

B) Disminución del estado de vigilia

C) Anormalidades conductuales

D) Deterioro cognitivo agudo


ANSWER: D

El siguiente resultado de LCR: células blancas 200, de predominio polimorfonuclear, proteínas


600 mg/dl, hipoglucorraquia de 26 mg/dl, correspondería a qué enfermedad:

A) Encefalitis viral

B) Meningitis viral

C) Meningitis tuberculosa

D) Meningitis bacteriana

ANSWER: D
Paciente de 48 años sin APP, reside en una comunidad rural de la provincia de Cotopaxi,
agricultor, acude por 1 crisis convulsiva focal motora izquierda, en el trabajo diagnóstico que
requiere para averiguar la etiología de la crisis, usted señalaría que enunciado como verdadero:

A) El hallazgo en TAC cerebral de calcificaciones parenquimatosas denotan un criterio absoluto


de neurocisticercosis

B) La demostración histológica del parásito a través de una biopsia es considerado un criterio


menor de diagnóstico de neurocisticercosis

C) Se puede solicitar determinación de anticuerpos anticisticercosis o detección de antígenos


que son parte de los criterios diagnósticos de cisticercosis

D) La zona de residencia endémica para cisticercosis es considerado un criterio de exposición


pero no forma parte de los criterios diagnósticos

ANSWER: C

Si se necesita el aumento de plaquetas más efectivo en una trombocitopenia severa inmune.


¿Qué utilizas de primera línea?

A) Metilprednisolona

B) Inmunoglobulina

C) Eltrombopag

D) Rituximab anticuerpo monoclonal

ANSWER: B

Una reacción transfusional es una respuesta adversa a la transfusión de un derivado sanguíneo.


¿Cuál es la reacción más frecuente?

A) Reacción hemolítica aguda

B) Reacción hemolítica crónica

C) Reacción febril no hemolítica

D) Urticaria

ANSWER: C

Qué vitamina causa insuficiencia cardiaca con alto gasto:

A) B

B) D
C) E

D) A

ANSWER: A

Paciente que presenta cifras tensionales en el consultorio dentro de parámetros normales, pero
evidencia de retinopatía hipertensiva. Usted pensaría en

A) Hipertension de bata blanca

B) Hipertension enmascarada

C) Hipertension resistente

D) Hipertension refractaria

ANSWER: B

Paciente hipertenso con uso de 3 antihipertensivos uno de los cuales es un diurético que persiste
con presiones arteriales elevadas. Usted pensaría en

A) Hipertension de bata blanca

B) Hipertension enmascarada

C) Hipertension resistente

D) Hipertension refractaria

ANSWER: C

El Síndrome de Alport, que nefropatía produce?

A) Glomerulonefritis membranoproliferativa

B) Granulomatosis de Wegener

C) Glomeruloesclerosis segmentaria focal


D) Nefropatía por IgA

ANSWER: C

Un paciente con sospecha de encefalitis, debe tener, EXCEPTO:

A) Signos de focalidad neurológica

B) Disminución del estado de vigilia

C) Anormalidades conductuales

D) Deterioro cognitivo agudo


ANSWER: D

El siguiente resultado de LCR: células blancas 200, de predominio polimorfonuclear, proteínas


600 mg/dl, hipoglucorraquia de 26 mg/dl, correspondería a qué enfermedad:

A) Encefalitis viral

B) Meningitis viral

C) Meningitis tuberculosa

D) Meningitis bacteriana

ANSWER: D

Paciente de 48 años sin APP, reside en una comunidad rural de la provincia de Cotopaxi,
agricultor, acude por 1 crisis convulsiva focal motora izquierda, en el trabajo diagnóstico que
requiere para averiguar la etiología de la crisis, usted señalaría que enunciado como verdadero:

A) El hallazgo en TAC cerebral de calcificaciones parenquimatosas denotan un criterio absoluto


de neurocisticercosis

B) La demostración histológica del parásito a través de una biopsia es considerado un criterio


menor de diagnóstico de neurocisticercosis

C) Se puede solicitar determinación de anticuerpos anticisticercosis o detección de antígenos


que son parte de los criterios diagnósticos de cisticercosis

D) La zona de residencia endémica para cisticercosis es considerado un criterio de exposición


pero no forma parte de los criterios diagnósticos

ANSWER: C

Paciente de 22 años de género femenino, con APP: Neuritis óptica 8 meses atrás, acude por un
cuadro de paraparesia de instauración aguda, se realizó un estudio de resonancia magnética de
la médula espinal encontrándose una lesión hiperintensa longitudinalmente extensa (4
segmentos vertebrales), usted consideraría como diagnóstico:

A) Mielitis transversa aguda

B) Mielitis secundaria a infección viral

C) Neuromielitis óptica

D) Malformación arteriovenosa medular

ANSWER: C

Si se necesita el aumento de plaquetas más efectivo en una trombocitopenia severa inmune.


¿Qué utilizas de primera línea?
A) Metilprednisolona

B) Inmunoglobulina

C) Eltrombopag

D) Rituximab anticuerpo monoclonal

ANSWER: B

Una reacción transfusional es una respuesta adversa a la transfusión de un derivado sanguíneo.


¿Cuál es la reacción más frecuente?

A) Reacción hemolítica aguda

B) Reacción hemolítica crónica

C) Reacción febril no hemolítica

D) Urticaria

ANSWER: C

El Síndrome de Alport, que nefropatía produce?

A) Glomerulonefritis membranoproliferativa

B) Granulomatosis de Wegener

C) Glomeruloesclerosis segmentaria focal

D) Nefropatía por IgA

ANSWER: C

Paciente de 48 años sin APP, reside en una comunidad rural de la provincia de Cotopaxi,
agricultor, acude por 1 crisis convulsiva focal motora izquierda, en el trabajo diagnóstico que
requiere para averiguar la etiología de la crisis, usted señalaría que enunciado como verdadero:

A) El hallazgo en TAC cerebral de calcificaciones parenquimatosas denotan un criterio absoluto


de neurocisticercosis

B) La demostración histológica del parásito a través de una biopsia es considerado un criterio


menor de diagnóstico de neurocisticercosis

C) Se puede solicitar determinación de anticuerpos anticisticercosis o detección de antígenos


que son parte de los criterios diagnósticos de cisticercosis

D) La zona de residencia endémica para cisticercosis es considerado un criterio de exposición


pero no forma parte de los criterios diagnósticos

ANSWER: C
Paciente de 22 años de género femenino, con APP: Neuritis óptica 8 meses atrás, acude por un
cuadro de paraparesia de instauración aguda, se realizó un estudio de resonancia magnética de
la médula espinal encontrándose una lesión hiperintensa longitudinalmente extensa (4
segmentos vertebrales), usted consideraría como diagnóstico:

A) Mielitis transversa aguda

B) Mielitis secundaria a infección viral

C) Neuromielitis óptica

D) Malformación arteriovenosa medular

ANSWER: C

El Síndrome de Alport, que nefropatía produce?

A) Glomerulonefritis membranoproliferativa

B) Granulomatosis de Wegener

C) Glomeruloesclerosis segmentaria focal

D) Nefropatía por IgA

ANSWER: C

La obstrucción ureteral bilateral crónica se caracteriza por:

A) Anuria

B) Dolor renal

C) Hipertensión

D) Diuresis posterior a la obstrucción

ANSWER: C

La obstrucción ureteral bilateral aguda se caracteriza por:

A) Disminución de prostaglandinas vasodilatadoras

B) Disminución de la producción de angiotensina

C) Disminución del flujo sanguíneo medular

D) Disminución de la liberación de óxido nítrico

ANSWER: D

La proteinuria sostenida es aquella que se expulsan más de:

A) 1 a 2 g/24 h
B) 3 a 4 g/24 h

C) 5 a 6 g/24 h

D) 7 a 8 g/24 h

ANSWER: A

Cuál no es una glomerulopatía de vasos pequeños ANCA:

A) Granulomatosis de Wegener

B) Púrpura de Henoch-Schönlein

C) Poliangitis microscópica

D) Síndrome de Churg-Strauss

ANSWER: B

Cuál glomerulopatía no produce daño vascular:

A) Nefritis lúpica
B) Crioglobulinemia

C) Granulomatosis de Wegener

D) Nefropatía por IgA

ANSWER: D

La clase VI de la nefritis lúpica se llama:

A) Nefritis esclerótica

B) Nefritis focal

C) Nefritis difusa

D) Proliferación mesangial
ANSWER: A

La clase IV de la nefritis lúpica se llama:

A) Nefritis esclerótica

B) Nefritis focal

C) Nefritis difusa

D) Proliferación mesangial

ANSWER: C
La hepatitis B que nefropatía produce:

A) Nefritis lúpica

B) Glomerulonefritis membranoproliferativa

C) Granulomatosis de Wegener

D) Nefropatía por IgA

ANSWER: B

Qué vitamina causa insuficiencia cardiaca con alto gasto:

A) B

B) D

C) E

D) A

ANSWER: A

Paciente que presenta cifras tensionales en el consultorio dentro de parámetros normales, pero
evidencia de retinopatía hipertensiva. Usted pensaría en

A) Hipertension de bata blanca

B) Hipertension enmascarada

C) Hipertension resistente

D) Hipertension refractaria

ANSWER: B

Paciente hipertenso con uso de 3 antihipertensivos uno de los cuales es un diurético que persiste
con presiones arteriales elevadas. Usted pensaría en

A) Hipertension de bata blanca

B) Hipertension enmascarada

C) Hipertension resistente

D) Hipertension refractaria

ANSWER: C
Paciente que en monitoreo ambulatorio de presión arterial presenta cifras tensionales en
parámetros adecuados, pero en el consultorio presenta cifras tensionales elevadas. Usted
pensaría en

A) Hipertension de bata blanca

B) Hipertension enmascarada

C) c)Hipertension resistente

D) d)Hipertension refractaria

ANSWER: A

La hipertensión esencial corresponde a

A) La minoriade paciente hipertensos

B) Mas frecuente en pacientes jóvenes


C) Se relaciona con alteración en la excreción de calcio

D) Se relaciona con alteración en la excreción de sodio

ANSWER: D

El siguiente resultado de LCR: células blancas 200, de predominio polimorfonuclear, proteínas


600 mg/dl, hipoglucorraquia de 26 mg/dl, correspondería a qué enfermedad:

A) Encefalitis viral

B) Meningitis viral

C) Meningitis tuberculosa

D) Meningitis bacteriana

ANSWER: D

Paciente de 48 años sin APP, reside en una comunidad rural de la provincia de Cotopaxi,
agricultor, acude por 1 crisis convulsiva focal motora izquierda, en el trabajo diagnóstico que
requiere para averiguar la etiología de la crisis, usted señalaría que enunciado como verdadero:
A) El hallazgo en TAC cerebral de calcificaciones parenquimatosas denotan un criterio absoluto
de neurocisticercosis

B) La demostración histológica del parásito a través de una biopsia es considerado un criterio


menor de diagnóstico de neurocisticercosis

C) Se puede solicitar determinación de anticuerpos anticisticercosis o detección de antígenos


que son parte de los criterios diagnósticos de cisticercosis

D) La zona de residencia endémica para cisticercosis es considerado un criterio de exposición


pero no forma parte de los criterios diagnósticos
ANSWER: C
Paciente de 22 años de género femenino, con APP: Neuritis óptica 8 meses atrás, acude por un
cuadro de paraparesia de instauración aguda, se realizó un estudio de resonancia magnética de
la médula espinal encontrándose una lesión hiperintensa longitudinalmente extensa (4
segmentos vertebrales), usted consideraría como diagnóstico:

A) Mielitis transversa aguda

B) Mielitis secundaria a infección viral

C) Neuromielitis óptica

D) Malformación arteriovenosa medular

ANSWER: C

Paciente que presenta en la escala de Burch y Watofsky un puntaje de 40 puntos nos indica cuál
de las siguientes posibilidades:

A) Tormenta tiroidea

B) Tormenta tiroidea inminente

C) Tormenta tiroidea poco probable

D) Se excluye el diagnóstico de tormenta tiroidea.

ANSWER: B
Pregunta 1 Correcta Se puntúa 0,20 sobre 0,20

Carmen de 40 años con obesidad mórbida, fue sometida a cirugía bariátrica. Durante la intervención quirúrgica
se realiza también una biopsia hepática, con hallazgo de acumulación de lípidos neutros en el citoplasma de lo
hepatocitos como en las células de Kupffer en forma vesículas que desplazan el núcleo celular a la periferia ¿A
qué tipo de lesión histológica hace referencia?:

Seleccione una:
a. Hepatitis crónica

b. Cirrosis

c. Esteatohepatitis

d. Esteatosis

La respuesta correcta es: Esteatosis

Pregunta 2 Correcta Se puntúa 0,20 sobre 0,20

Con respecto al reflejo ojos de muñeca, escoja la respuesta correcta:

Seleccione una:
a. Su presencia demuestra integralidad mesencefálica (III y IV par craneal) y protuberancial (VI y VIII par
craneal).
b. Es un reflejo patológico

c. Demuestra daño del VII par craneal.


d. Solo en algunas ocasiones está presente.

La respuesta correcta es: Su presencia demuestra integralidad mesencefálica (III y IV par craneal) y protuberancial (VI
y VIII par craneal).

Pregunta 3 Correcta Se puntúa 0,20 sobre 0,20

¿Cuál es el objetivo de la glucosa en ayunas en los pacientes con diabetes mellitus?

Seleccione una:
a. Menos de 180 mg/dL.

b. Entre 70 mg/dL y 130 mg/dL.


c. Entre 70 mg/dL y 120 mg/dL.

d. Entre 80 mg/dL y 130 mg/dL.

La respuesta correcta es: Entre 80 mg/dL y 130 mg/dL.


Pregunta 4 Correcta Se puntúa 0,20 sobre 0,20

¿Cuál es la causa más común de convulsiones en adolescentes entre 12 y 18 años?

Seleccione una:
a. Accidente cerebro vascular.
b. Enfermedades degenerativas.

c. Traumatismos.

d. Crisis febriles.

La respuesta correcta es: Traumatismos.

Pregunta 5 Incorrecta Se puntúa 0,00 sobre 0,20

En cuanto a la cefalea tensional episódica frecuente, el enunciado correcto es:

Seleccione una:
a. Dura de 30 minutos a 4 horas

b. Dura de 30 minutos a 7 días, durante más de 3 meses, es bilateral y no tiene irradiación.

c. Dura de 30 minutos a 7 días, es bilateral y no tiene irradiación.


d. Varios minutos de duración (entre 5 y 60 minutos)

La respuesta correcta es: Dura de 30 minutos a 7 días, durante más de 3 meses, es bilateral y no tiene irradiación.

Pregunta 6 Correcta Se puntúa 0,20 sobre 0,20

En la clasificación de las sulfonilureas, se menciona las de alto riesgo de hipoglicemias. Del siguiente listado,
cual es la de mayor riesgo de hipoglicemia.

Seleccione una:
a. Glicazida

b. Glibenclamida
c. Glimepririda

d. Glipizida

La respuesta correcta es: Glibenclamida


Pregunta 7 Correcta Se puntúa 0,20 sobre 0,20

En relación con la enfermedad celiaca, señale la respuesta correcta:

Seleccione una:
a. Las personas con síndrome de Prader-Willy tienen más riesgo de desarrollar una enfermedad celiaca que
población general.

b. La genética (DQ2/DQ8) tiene un elevado valor predictivo negativo.

c. Es necesaria la confirmación con biopsia intestinal para un diagnóstico definitivo.

d. Es frecuente adquirir tolerancia al gluten con la edad.

La respuesta correcta es: La genética (DQ2/DQ8) tiene un elevado valor predictivo negativo.

Pregunta 8 Correcta Se puntúa 0,20 sobre 0,20

En relación con los mecanismos normales de la digestión y absorción de los alimentos, una de las afirmacione
siguientes es Falsa:

Seleccione una:
a. La absorción de calcio esta facilitada por la vitamina D

b. Los ácidos grasos de cadena media son componentes constantes de la dieta vegetariana.

c. La vitamina B12 se absorbe en el íleon terminal


d. Las sales biliares facilitan la absorción de la grasa

La respuesta correcta es: Los ácidos grasos de cadena media son componentes constantes de la dieta vegetariana.

Pregunta 9 Incorrecta Se puntúa 0,00 sobre 0,20

En un hombre diabético de 60 años con muy alto riesgo cardiovascular. ¿Cuál es el objetivo terapéutico con
respecto a cifras de colesterol LDL y HbA1c?

Seleccione una:
a. LDL menor 55 mg/dL y Hb A1c menor7%.

b. LDL menor 115 mg/dL y Hb A1c menor 6.5%.


c. LDL menor 100 mg/dL y Hb A1c menor 7%.

d. LDL menor 150 mg/dL y Hb A1c menor 6.5%.

La respuesta correcta es: LDL menor 55 mg/dL y Hb A1c menor7%.


Pregunta 10 Incorrecta Se puntúa 0,00 sobre 0,20

Hombre de 87 años, con diabetes mellitus tipo 2, insuficiencia renal crónica, EPOC e insuficiencia cardiaca. Su
actividades de la vida diaria las realiza con apoyo de su cuidador. Esperanza de vida reducida. ¿Cuál sería el
objetivo terapéutico apropiado para el control glicémico en este paciente?

Seleccione una:
a. HbA1c entre 6.5% y 7%.
b. HbA1c entre 6% y 6.5%.

c. HbA1c menor de 8%.


d. HbA1c menor a 5%

La respuesta correcta es: HbA1c menor de 8%.

Pregunta 11 Correcta Se puntúa 0,20 sobre 0,20

Indique la situación clínica que, en relación con la infección por virus de la hepatitis B, presenta un paciente
joven, con exploración física normal y con la siguiente serología frente a hepatitis B: HBsAg + / ANTI-HBs – /
HbeAg – / ANTI-HBe + / ANTI-HBc IgM – / ANTI-HBc IgG + / DNA VHB +:

Seleccione una:
a. Infección aguda.
b. Paciente vacunado.

c. Infección crónica.
d. Portador asintomático.

La respuesta correcta es: Portador asintomático.

Pregunta 12 Correcta Se puntúa 0,20 sobre 0,20

La cefalea en racimos se clasifica como una:

Seleccione una:
a. Cefalea trigémino-autonómica
b. Otras cefaleas primarias
c. Cefalea tensional
d. Migraña

La respuesta correcta es: Cefalea trigémino-autonómica


Pregunta 13 Correcta Se puntúa 0,20 sobre 0,20

La cefalea más común y frecuente en mujeres es

Seleccione una:
a. Cefalea en racimos
b. Migraña androgénica

c. Cefalea tensional
d. Neuralgia del facial

La respuesta correcta es: Cefalea tensional

Pregunta 14 Correcta Se puntúa 0,20 sobre 0,20

La cetoacidosis diabética se define por:

Seleccione una:
a. Glucosa mayor a 600 mg/dl, pH mayor 7,30, bicarbonato menor a 18 mEq/l.

b. Glucosa mayor 150 mg/dl, pH menor a 7,30 con anion gap elevado mayor a 10 y disminución del
bicarbonato plasmático menor o igual 22 mEq/l.

c. Glucosa mayor 250 mg/dl, pH menor a 7,30 con anion gap elevado mayor a 10 y disminución del
bicarbonato plasmático menor o igual a 18 mEq/l.
d. Glucosa mayor 250 mg/dl, pH menor a 7,40 con anion gap elevado mayor a 10 y disminución del
bicarbonato plasmático menor o igual a 20 mEq/l.

La respuesta correcta es: Glucosa mayor 250 mg/dl, pH menor a 7,30 con anion gap elevado mayor a 10 y disminución
del bicarbonato plasmático menor o igual a 18 mEq/l.

Pregunta 15 Correcta Se puntúa 0,20 sobre 0,20

La diabetes mellitus tipo 1:

Seleccione una:
a. Puede asociarse a otras enfermedades endocrinas autoinmunes.
b. Es más frecuente que el tipo 2.
c. Suele controlarse inicialmente con antidiabéticos orales, aunque puede necesitar insulina con el tiempo.
d. Suele asociarse a obesidad.

La respuesta correcta es: Puede asociarse a otras enfermedades endocrinas autoinmunes.


Pregunta 16 Correcta Se puntúa 0,20 sobre 0,20

Los siguientes grupos de fármacos son utilizados en el tratamiento preventivo de la migraña, escoja la
agrupación correcta:

Seleccione una:
a. Propanolol, amitriptilina, acido valproico.

b. Ibuprofeno, propanolol, tramadol


c. Tramadol, ibuprofeno, paracetamol.

d. Propanolol, amitriptilina, paracetamol

La respuesta correcta es: Propanolol, amitriptilina, acido valproico.

Pregunta 17 Correcta Se puntúa 0,20 sobre 0,20

Los síntomas neurogénicos o autonómicos en la hipoglicemia aparecen por lo general cuando la glicemia se
encuentra entre:

Seleccione una:
a. 30 – 40 mg/dL.

b. Menor a 80 mg/dl

c. Menor a 40 mg/dl
d. 60 - 55 mg/dL.

La respuesta correcta es: 60 - 55 mg/dL.

Pregunta 18 Correcta Se puntúa 0,20 sobre 0,20

Mujer de 55 años con DM tipo 2 en tratamiento con metformina, mal controlada, e historia de infecciones de ví
urinarias recurrentes. ¿Cuál de los siguientes fármacos es el menos apropiado?

Seleccione una:
a. Insulina cristalina

b. Inhibidores SGLT2
c. Análogos GLP1
d. Insulina NPH

La respuesta correcta es: Inhibidores SGLT2


Pregunta 19 Correcta Se puntúa 0,20 sobre 0,20

¿Qué fármaco no es de elección en las crisis ausencia?

Seleccione una:
a. Carbamazepina

b. Lamotrigina.

c. Etosuximida
d. Ácido Valproico.

La respuesta correcta es: Carbamazepina

Pregunta 20 Correcta Se puntúa 0,20 sobre 0,20

Uno de los siguientes fármacos se utiliza en el tratamiento preventivo de la migraña:

Seleccione una:
a. Tramadol

b. Propanolol.

c. Paracetamol
d. Ibuprofeno

La respuesta correcta es: Propanolol.


23/2/23, 15:54 Tercer Examen Parcial PTitulación I: Revisión del intento

Acude a su consulta una paciente femenina de 64 años ella ha sido valorada previamente por fenómeno de
Raynaud, esofagitis y esclerodactilia. Usted sospecha en SD De CREST por lo cual al realizar el examen físico
debe hacer énfasis en buscar:

Seleccione una:
a. Calcinosis cutis y lesiones pustulosas
b. Únicamente calcinosis cutis

c. Calcinosis cutis y exantemas maculo papulares

d. Calcinosis cutis y telangiectasias

La respuesta correcta es: Calcinosis cutis y telangiectasias

Pregunta 2 Correcta Se puntúa 0,20 sobre 0,20

¿Cuál es el objetivo de la glucosa en ayunas en los pacientes con diabetes mellitus?

Seleccione una:
a. Entre 80 mg/dL y 130 mg/dL.
b. Entre 70 mg/dL y 130 mg/dL.
c. Menos de 180 mg/dL.

d. Entre 70 mg/dL y 120 mg/dL.

La respuesta correcta es: Entre 80 mg/dL y 130 mg/dL.

Pregunta 3 Correcta Se puntúa 0,20 sobre 0,20

El síndrome de Wallenberg se produce por infarto en:

Seleccione una:
a. Arterias para medianas basilares
b. Arteria cerebelosa anteroinferior
c. Arteria cerebelosa superior
d. Arteria cerebelosa posteroinferior

La respuesta correcta es: Arteria cerebelosa posteroinferior

https://lms.ute.edu.ec/mod/quiz/review.php?attempt=661252&cmid=1343990#question-741524-5 1/7
23/2/23, 15:54 Tercer Examen Parcial PTitulación I: Revisión del intento

Pregunta 4 Correcta Se puntúa 0,20 sobre 0,20

En la clasificación de las sulfonilureas, se menciona las de alto riesgo de hipoglicemias. Del siguiente listado,
cual es la de mayor riesgo de hipoglicemia.

Seleccione una:
a. Glicazida
b. Glipizida

c. Glibenclamida

d. Glimepririda

La respuesta correcta es: Glibenclamida

Pregunta 5 Incorrecta Se puntúa 0,00 sobre 0,20

En un hombre diabético de 60 años con muy alto riesgo cardiovascular. ¿Cuál es el objetivo terapéutico con
respecto a cifras de colesterol LDL y HbA1c?

Seleccione una:
a. LDL menor 100 mg/dL y Hb A1c menor 7%.

b. LDL menor 150 mg/dL y Hb A1c menor 6.5%.


c. LDL menor 115 mg/dL y Hb A1c menor 6.5%.

d. LDL menor 55 mg/dL y Hb A1c menor7%.

La respuesta correcta es: LDL menor 55 mg/dL y Hb A1c menor7%.

Pregunta 6 Correcta Se puntúa 0,20 sobre 0,20

Hombre de 87 años, con diabetes mellitus tipo 2, insuficiencia renal crónica, EPOC e insuficiencia cardiaca. Sus
actividades de la vida diaria las realiza con apoyo de su cuidador. Esperanza de vida reducida. ¿Cuál sería el
objetivo terapéutico apropiado para el control glicémico en este paciente?

Seleccione una:
a. HbA1c menor de 8%.

b. HbA1c menor a 5%
c. HbA1c entre 6% y 6.5%.

d. HbA1c entre 6.5% y 7%.

La respuesta correcta es: HbA1c menor de 8%.

https://lms.ute.edu.ec/mod/quiz/review.php?attempt=661252&cmid=1343990#question-741524-5 2/7
23/2/23, 15:54 Tercer Examen Parcial PTitulación I: Revisión del intento

Pregunta 7 Correcta Se puntúa 0,20 sobre 0,20

La cetoacidosis diabética se define por:

Seleccione una:
a. Glucosa mayor a 600 mg/dl, pH mayor 7,30, bicarbonato menor a 18 mEq/l.
b. Glucosa mayor 250 mg/dl, pH menor a 7,30 con anion gap elevado mayor a 10 y disminución del
bicarbonato plasmático menor o igual a 18 mEq/l.

c. Glucosa mayor 250 mg/dl, pH menor a 7,40 con anion gap elevado mayor a 10 y disminución del
bicarbonato plasmático menor o igual a 20 mEq/l.

d. Glucosa mayor 150 mg/dl, pH menor a 7,30 con anion gap elevado mayor a 10 y disminución del
bicarbonato plasmático menor o igual 22 mEq/l.

La respuesta correcta es: Glucosa mayor 250 mg/dl, pH menor a 7,30 con anion gap elevado mayor a 10 y disminución
del bicarbonato plasmático menor o igual a 18 mEq/l.

Pregunta 8 Correcta Se puntúa 0,20 sobre 0,20

La diabetes mellitus tipo 1:

Seleccione una:
a. Suele controlarse inicialmente con antidiabéticos orales, aunque puede necesitar insulina con el tiempo.

b. Es más frecuente que la tipo 2.

c. Suele asociarse a obesidad.


d. Puede asociarse a otras enfermedades endocrinas autoinmunes.

La respuesta correcta es: Puede asociarse a otras enfermedades endocrinas autoinmunes.

Pregunta 9 Correcta Se puntúa 0,20 sobre 0,20

La hipertensión esencial corresponde a

Seleccione una:
a. Se relaciona con alteración en la excreción de calcio
b. Se relaciona con alteración en la excreción de sodio
c. La minoría de paciente hipertensos
d. Mas frecuente en pacientes jóvenes

La respuesta correcta es: Se relaciona con alteración en la excreción de sodio

https://lms.ute.edu.ec/mod/quiz/review.php?attempt=661252&cmid=1343990#question-741524-5 3/7
23/2/23, 15:54 Tercer Examen Parcial PTitulación I: Revisión del intento

Pregunta 10 Correcta Se puntúa 0,20 sobre 0,20

Llega a consulta un paciente de sexo masculino de 60 años, acude con una biopsia particular de una lesión
pigmentada de 4 meses de evolución, la cual reporta Melanoma de extensión superficial con breslow de 1.2mm
sin ulceración, tomando en cuenta estos datos ¿cuál sería su estadiaje según la clasificación TNM?

Seleccione una:
a. 1b
b. 2b

c. 2a
d. 1a

La respuesta correcta es: 2a

Pregunta 11 Correcta Se puntúa 0,20 sobre 0,20

Los síntomas neurogénicos o autonómicos en la hipoglicemia aparecen por lo general cuando la glicemia se
encuentra entre:

Seleccione una:
a. Menor a 40 mg/dl
b. 60 - 55 mg/dL.

c. 30 – 40 mg/dL.
d. Menor a 80 mg/dl

La respuesta correcta es: 60 - 55 mg/dL.

Pregunta 12 Correcta Se puntúa 0,20 sobre 0,20

Mujer de 55 años con DM tipo 2 en tratamiento con metformina, mal controlada, e historia de infecciones de vías
urinarias recurrentes. ¿Cuál de los siguientes fármacos es el menos apropiado?

Seleccione una:
a. Insulina NPH
b. Análogos GLP1
c. Inhibidores SGLT2
d. Insulina cristalina

La respuesta correcta es: Inhibidores SGLT2

https://lms.ute.edu.ec/mod/quiz/review.php?attempt=661252&cmid=1343990#question-741524-5 4/7
23/2/23, 15:54 Tercer Examen Parcial PTitulación I: Revisión del intento

Pregunta 13 Correcta Se puntúa 0,20 sobre 0,20

Paciente acude por placas ovaladas circunscritas, de bordes regulares y definidos, eritemato- violáceas,
edematosas en dorso de manos y glande, indica haber presentado estas lesiones anteriormente en el mismo
sitio, sin embargo, no menciona causa aparente. ¿Cuál es su diagnóstico?

Seleccione una:
a. Eritema fijo pigmentario
b. impétigo ampolloso

c. Eritema anular centrífugo

La respuesta correcta es: Eritema fijo pigmentario

Pregunta 14 Correcta Se puntúa 0,20 sobre 0,20

Paciente hipertenso con uso de 3 antihipertensivos uno de los cuales es un diurético que persiste con presiones
arteriales elevadas. Usted pensaría en

Seleccione una:
a. hipertensión refractaria
b. hipertensión enmascarada

c. hipertensión resistente
d. hipertensión de bata blanca

La respuesta correcta es: hipertensión resistente

Pregunta 15 Correcta Se puntúa 0,20 sobre 0,20

Paciente masculino de 5 años acude al centro de salud por presentar cuadro de 4 días de evolución de fiebre,
cefalea, rinorrea y malestar general. Hace 24 horas presenta lesiones eritematosas en rostro que progresan y
diseminan a tronco y extremidades. Al examen físico el paciente presenta temperatura de 37,8 grados,
taquicardia, máculas y pápulas rosadas, adenopatías retroauriculares y occipitales dolorosas a la palpación y
manchas de Forchheimer en paladar blando. Con estos hallazgos, ¿cuál es su diagnóstico presuntivo?

Seleccione una:
a. Rubeola

b. Sarampión
c. Eritema infeccioso
d. Viruela del mono

La respuesta correcta es: Rubeola

https://lms.ute.edu.ec/mod/quiz/review.php?attempt=661252&cmid=1343990#question-741524-5 5/7
23/2/23, 15:54 Tercer Examen Parcial PTitulación I: Revisión del intento

Pregunta 16 Correcta Se puntúa 0,20 sobre 0,20

Paciente que en monitoreo ambulatorio de presión arterial presenta cifras tensionales en parámetros adecuados,
pero en el consultorio presenta cifras tensionales elevadas. Usted pensaría en

Seleccione una:
a. Hipertensión resistente

b. Hipertensión de bata blanca


c. Hipertensión enmascarada

d. Hipertensión refractaria

La respuesta correcta es: Hipertensión de bata blanca

Pregunta 17 Correcta Se puntúa 0,20 sobre 0,20

Paciente que presenta cifras tensionales en el consultorio dentro de parámetros normales, pero evidencia de
retinopatía hipertensiva. Usted pensaría en

Seleccione una:
a. hipertensión enmascarada

b. hipertensión de bata blanca

c. hipertensión refractaria
d. hipertensión resistente

La respuesta correcta es: hipertensión enmascarada

Pregunta 18 Correcta Se puntúa 0,20 sobre 0,20

Qué endocrinopatía no causa prolongación de QT y de taquicardia ventricular polimorfa:

Seleccione una:
a. Hiperaldosteronismo

b. Hipogonadismo
c. Hiperparatiroidismo
d. Hipotiroidismo

La respuesta correcta es: Hipogonadismo

https://lms.ute.edu.ec/mod/quiz/review.php?attempt=661252&cmid=1343990#question-741524-5 6/7
23/2/23, 15:54 Tercer Examen Parcial PTitulación I: Revisión del intento

Pregunta 19 Correcta Se puntúa 0,20 sobre 0,20

Qué familia de antibiótico no causa prolongación de QT y de taquicardia ventricular polimorfa:

Seleccione una:
a. Cefalosporinas

b. Quinolonas

c. Macrólidos
d. Antagonistas del folato

La respuesta correcta es: Cefalosporinas

Pregunta 20 Correcta Se puntúa 0,20 sobre 0,20

Qué vitamina causa insuficiencia cardiaca con alto gasto:

Seleccione una:
a. E

b. A

c. B
d. D

La respuesta correcta es: B

https://lms.ute.edu.ec/mod/quiz/review.php?attempt=661252&cmid=1343990#question-741524-5 7/7
Diego Martin Gomez Cadena

Área personal  Mis cursos  Quito  CIENCIAS DE LA SALUD  MEDICINA (R) - PRESENCIAL  OCT 2022 - FEB 2023  PLAN DE TITULACIÓN I - Prl: RDM A
Pen: 1572 Per:OCT 2022 - FEB 2023 Car:MEDICINA (R) - PRESENCIAL  General  Tercer Examen Parcial PTitulación I

Comenzado el jueves, 23 de febrero de 2023, 15:10


Estado Finalizado
Finalizado en jueves, 23 de febrero de 2023, 15:44
Tiempo empleado 34 minutos 9 segundos
Calificación 3,80 de 4,00 (95%)

Pregunta 1 Correcta Se puntúa 0,20 sobre 0,20

Con respecto a la fiebre de origen desconocido cual no es criterio para definirlo:

Seleccione una:

a. fiebre que dura 72 horas

b. fiebre mayor a 38.3 oc.

c. al menos una semana de investigación hospitalaria

d. persistencia por al menos tres semanas

La respuesta correcta es: fiebre que dura 72 horas


Pregunta 2 Correcta Se puntúa 0,20 sobre 0,20

Con respecto a la meningitis cual es el examen de elección para determinar si es bacteriana o viral

Seleccione una:

a. biometría hemática

b. procalcitonina

c. punción lumbar

d. tomografía axial de cráneo

La respuesta correcta es: punción lumbar

Pregunta 3 Correcta Se puntúa 0,20 sobre 0,20

Con respecto a la parasitosis cuales son los factores de riesgos:

Seleccione una:

a. lugares insalubres

b. climas fríos

c. nivel nutricional adecuado

d. edad

La respuesta correcta es: lugares insalubres


Pregunta 4 Correcta Se puntúa 0,20 sobre 0,20

Con respecto al reflejo ojos de muñeca, escoja la respuesta correcta:

Seleccione una:

a. Demuestra daño del VII par craneal.

b. Es un reflejo patológico

c. Su presencia demuestra integralidad mesencefálica (III y IV par craneal) y protuberancial (VI y VIII par craneal).

d. Solo en algunas ocasiones está presente.

La respuesta correcta es: Su presencia demuestra integralidad mesencefálica (III y IV par craneal) y protuberancial (VI y VIII par craneal).

Pregunta 5 Correcta Se puntúa 0,20 sobre 0,20

Cuál de los siguientes patógenos produce osteomielitis aguda

Seleccione una:

a. staphylococcus aureus

b. esherichia coli

c. staphylococcus epidermidis

d. pseudomonas aeruginosa

La respuesta correcta es: staphylococcus aureus


Pregunta 6 Correcta Se puntúa 0,20 sobre 0,20

Cuál es el vector causante de la enfermedad del dengue:

Seleccione una:

a. agua contaminada

b. mosquito aedes contaminado

c. mosquito aedes

d. mosquito anofeles

La respuesta correcta es: mosquito aedes contaminado

Pregunta 7 Correcta Se puntúa 0,20 sobre 0,20

¿Cuál es la causa más común de convulsiones en adolescentes entre 12 y 18 años?

Seleccione una:

a. Traumatismos.

b. Crisis febriles.

c. Accidente cerebro vascular.

d. Enfermedades degenerativas.

La respuesta correcta es: Traumatismos.


Pregunta 8 Correcta Se puntúa 0,20 sobre 0,20

¿Cuál NO es una causa de alcalosis respiratoria?

Seleccione una:

a. Exposición a grandes alturas.

b. Tratamiento con salicilatos

c. Aldosteronismo primario

d. Crisis asmática.

La respuesta correcta es: Aldosteronismo primario

Pregunta 9 Correcta Se puntúa 0,20 sobre 0,20

¿Cuál NO es un índice urinario en el diagnóstico del fracaso renal agudo prerrenal?

Seleccione una:

a. La excreción fraccional de Sodio (EFNa) es mayor al 1%

b. El sodio urinario es inferior a 20 mEq/l.

c. La osmolalidad urinaria es superior a 400 mOsm/Kg

d. El cociente entre la urea urinaria y la urea plasmática es superior a 10

La respuesta correcta es: La excreción fraccional de Sodio (EFNa) es mayor al 1%


Pregunta 10 Correcta Se puntúa 0,20 sobre 0,20

¿Cuál parámetro de la ecuación MDRD, NO se incluye para el cálculo del filtrado glomerular?

Seleccione una:

a. Talla

b. Sexo.

c. Edad.

d. Raza.

La respuesta correcta es: Talla

Pregunta 11 Correcta Se puntúa 0,20 sobre 0,20

El tiempo de incubación para la varicela es:

Seleccione una:

a. 7 a 14 días

b. 10 a 21 días

c. 2 a 8 días

d. 5 a 10 días

La respuesta correcta es: 10 a 21 días


Pregunta 12 Correcta Se puntúa 0,20 sobre 0,20

En cuanto a la cefalea tensional episódica frecuente, el enunciado correcto es:

Seleccione una:

a. Dura de 30 minutos a 4 horas

b. Dura de 30 minutos a 7 días, es bilateral y no tiene irradiación.

c. Dura de 30 minutos a 7 días, durante más de 3 meses, es bilateral y no tiene irradiación.

d. Varios minutos de duración (entre 5 y 60 minutos)

La respuesta correcta es: Dura de 30 minutos a 7 días, durante más de 3 meses, es bilateral y no tiene irradiación.

Pregunta 13 Correcta Se puntúa 0,20 sobre 0,20

En la fase primaria de la sífilis que tratamiento recomendaría

Seleccione una:

a. penicilina benzatínica 2400000 ui im semanal por 6 semanas

b. penicilina benzatínica 2400000 ui im semanal por 5 semanas

c. penicilina benzatínica 1200000 ui im semanal por 4 semanas

d. penicilina benzatínica 2400000 ui im semanal por 2 semanas

La respuesta correcta es: penicilina benzatínica 2400000 ui im semanal por 2 semanas


Pregunta 14 Correcta Se puntúa 0,20 sobre 0,20

En una nefrona, el 60% del cloruro de sodio es reabsorbido en?

Seleccione una:

a. Rama descendente del asa de Henle.

b. Túbulo contorneado distal.

c. Rama ascendente del asa de Henle.

d. Túbulo proximal

La respuesta correcta es: Túbulo proximal

Pregunta 15 Correcta Se puntúa 0,20 sobre 0,20

La brucelosis conocida también coma la:

Seleccione una:

a. fiebre de ondulante o malta

b. fiebre cuaternaria

c. fiebre terciaria

d. fiebre ondulante

La respuesta correcta es: fiebre de ondulante o malta


Pregunta 16 Correcta Se puntúa 0,20 sobre 0,20

La cefalea en racimos se clasifica como una:

Seleccione una:

a. Otras cefaleas primarias

b. Migraña

c. Cefalea trigémino-autonómica

d. Cefalea tensional

La respuesta correcta es: Cefalea trigémino-autonómica

Pregunta 17 Correcta Se puntúa 0,20 sobre 0,20

La cefalea más común y frecuente en mujeres es

Seleccione una:

a. Neuralgia del facial

b. Migraña androgénica

c. Cefalea en racimos

d. Cefalea tensional

La respuesta correcta es: Cefalea tensional


Pregunta 18 Incorrecta Se puntúa 0,00 sobre 0,20

Los siguientes grupos de fármacos son utilizados en el tratamiento preventivo de la migraña, escoja la agrupación correcta:

Seleccione una:

a. Ibuprofeno, propanolol, tramadol

b. Propanolol, amitriptilina, acido valproico.

c. Propanolol, amitriptilina, paracetamol

d. Tramadol, ibuprofeno, paracetamol.

La respuesta correcta es: Propanolol, amitriptilina, acido valproico.

Pregunta 19 Correcta Se puntúa 0,20 sobre 0,20

¿Qué fármaco no es de elección en las crisis ausencia?

Seleccione una:

a. Etosuximida

b. Lamotrigina.

c. Ácido Valproico.

d. Carbamazepina

La respuesta correcta es: Carbamazepina


Pregunta 20 Correcta Se puntúa 0,20 sobre 0,20

Uno de los siguientes fármacos se utiliza en el tratamiento preventivo de la migraña:

Seleccione una:

a. Propanolol.

b. Paracetamol

c. Ibuprofeno

d. Tramadol

La respuesta correcta es: Propanolol.

◄ Registro de asistencia Ir a...


Diana Carolina Salto Juca

Área personal  Mis cursos  Quito  CIENCIAS DE LA SALUD  MEDICINA (R) - PRESENCIAL  OCT 2022 - FEB 2023
 PLAN DE TITULACIÓN I - Prl: RDM C Pen: 1572 Per:OCT 2022 - FEB 2023 Car:MEDICINA (R) - PRESENCIAL  General
 Tercer Examen Parcial PTitulación I

Comenzado el jueves, 23 de febrero de 2023, 15:10


Estado Finalizado
Finalizado en jueves, 23 de febrero de 2023, 15:42
Tiempo empleado 31 minutos 57 segundos
Calificación 3,00 de 4,00 (75%)

Pregunta 1 Correcta Se puntúa 0,20 sobre 0,20

Carmen de 40 años con obesidad mórbida, fue sometida a cirugía bariátrica. Durante la intervención
quirúrgica se realiza también una biopsia hepática, con hallazgo de acumulación de lípidos neutros en el
citoplasma de los hepatocitos como en las células de Kupffer en forma vesículas que desplazan el núcleo
celular a la periferia ¿A qué tipo de lesión histológica hace referencia?:

Seleccione una:

a. Cirrosis

b. Esteatohepatitis

c. Hepatitis crónica

d. Esteatosis

La respuesta correcta es: Esteatosis


Pregunta 2 Correcta Se puntúa 0,20 sobre 0,20

¿Cuál NO es una causa de alcalosis respiratoria?

Seleccione una:

a. Exposición a grandes alturas.

b. Crisis asmática.

c. Aldosteronismo primario.

d. Tratamiento con salicilatos.

La respuesta correcta es: Aldosteronismo primario.

Pregunta 3 Correcta Se puntúa 0,20 sobre 0,20

¿Cuál NO es un índice urinario en el diagnóstico del fracaso renal agudo prerrenal?

Seleccione una:

a. El cociente entre la urea urinaria y la urea plasmática es superior a 10.

b. La excreción fraccional de Sodio (EFNa) es mayor al 1%.

c. La osmolalidad urinaria es superior a 400 mOsm/Kg.

d. El sodio urinario es inferior a 20 mEq/l.

La respuesta correcta es: La excreción fraccional de Sodio (EFNa) es mayor al 1%.


Pregunta 4 Correcta Se puntúa 0,20 sobre 0,20

¿Cuál parámetro de la ecuación MDRD, NO se incluye para el cálculo del filtrado glomerular?

Seleccione una:

a. Edad.

b. Sexo.

c. Talla.

d. Raza.

La respuesta correcta es: Talla.

Pregunta 5 Correcta Se puntúa 0,20 sobre 0,20

El síndrome de Wallenberg se produce por infarto en:

Seleccione una:

a. Arteria cerebelosa anteroinferior

b. Arteria cerebelosa posteroinferior

c. Arterias para medianas basilares

d. Arteria cerebelosa superior

La respuesta correcta es: Arteria cerebelosa posteroinferior


Pregunta 6 Correcta Se puntúa 0,20 sobre 0,20

En relación con el trasplante hepático, señale de las siguientes la respuesta CORRECTA:

Seleccione una:

a. El sistema MELD es un modelo pronóstico que permite la priorización para el trasplante hepático de los
enfermos en lista.

b. El trasplante hepático sería la mejor opción de tratamiento para un paciente de 45 años.

c. La indicación de trasplante hepático más frecuente en la actualidad es la cirrosis por infección del virus
B (VHB).

d. Las metástasis hepáticas por cáncer de colon constituyen una indicación aprobada para el trasplante
hepático.

La respuesta correcta es: El sistema MELD es un modelo pronóstico que permite la priorización para el trasplante
hepático de los enfermos en lista.

Pregunta 7 Correcta Se puntúa 0,20 sobre 0,20

En relación con la enfermedad celiaca, señale la respuesta correcta:

Seleccione una:

a. Las personas con síndrome de Prader-Willy tienen más riesgo de desarrollar una enfermedad celiaca
que la población general.

b. La genética (DQ2/DQ8) tiene un elevado valor predictivo negativo.

c. Es necesaria la confirmación con biopsia intestinal para un diagnóstico definitivo.

d. Es frecuente adquirir tolerancia al gluten con la edad.

La respuesta correcta es: La genética (DQ2/DQ8) tiene un elevado valor predictivo negativo.
Pregunta 8 Incorrecta Se puntúa 0,00 sobre 0,20

En relación con los mecanismos normales de la digestión y absorción de los alimentos, una de las
afirmaciones siguientes es Falsa:

Seleccione una:

a. Los ácidos grasos de cadena media son componentes constantes de la dieta vegetariana.

b. La absorción de calcio esta facilitada por la vitamina D

c. La vitamina B12 se absorbe en el íleon terminal

d. Las sales biliares facilitan la absorción de la grasa

La respuesta correcta es: Los ácidos grasos de cadena media son componentes constantes de la dieta
vegetariana.

Pregunta 9 Correcta Se puntúa 0,20 sobre 0,20

En una nefrona, el 60% del cloruro de sodio es reabsorbido en?

Seleccione una:

a. Túbulo proximal

b. Túbulo contorneado distal.

c. Rama ascendente del asa de Henle.

d. Rama descendente del asa de Henle.

La respuesta correcta es: Túbulo proximal


Pregunta 10 Incorrecta Se puntúa 0,00 sobre 0,20

Indique la situación clínica que, en relación con la infección por virus de la hepatitis B, presenta un paciente
joven, con exploración física normal y con la siguiente serología frente a hepatitis B: HBsAg + / ANTI-HBs – /
HbeAg – / ANTI-HBe + / ANTI-HBc IgM – / ANTI-HBc IgG + / DNA VHB +:

Seleccione una:

a. Infección aguda.

b. Paciente vacunado.

c. Infección crónica.

d. Portador asintomático.

La respuesta correcta es: Portador asintomático.

Pregunta 11 Incorrecta Se puntúa 0,00 sobre 0,20

Juan de 67 años, con diagnóstico de cirrosis hepática y bebedor activo, ingresa por cuadro de distensión
abdominal progresivo con malestar difuso, de dos semanas de evolución. No refiere fiebre ni otros síntomas.
A la exploración destaca matidez cambiante a la percusión abdominal, con ausencia de edemas. Se realiza
una paracentesis diagnóstica, encontrando un líquido ligeramente turbio, con 2.300 células/mL, de las cuales
30% son linfocitos, 60% polimorfonucleares y 10% hematíes. ¿Cuál es la primera medida terapéutica que
pautaría en este paciente de forma inmediata?

Seleccione una:

a. Restricción de sal y líquidos.

b. Paracentesis evacuadora.

c. Tratamiento con una cefalosporina de tercera generación.

d. Tratamiento diurético con espironolactona oral.

La respuesta correcta es: Tratamiento con una cefalosporina de tercera generación.


Pregunta 12 Correcta Se puntúa 0,20 sobre 0,20

La hipertensión esencial corresponde a

Seleccione una:

a. Se relaciona con alteración en la excreción de calcio

b. Mas frecuente en pacientes jóvenes

c. La minoría de paciente hipertensos

d. Se relaciona con alteración en la excreción de sodio

La respuesta correcta es: Se relaciona con alteración en la excreción de sodio

Pregunta 13 Incorrecta Se puntúa 0,00 sobre 0,20

Paciente de 78 años, previamente sano, que vive en una Residencia de Ancianos con insuficientes servicios
básicos, padece un cuadro diarreico desde hace 6 semanas. Refiere molestias abdominales tipo retortijón,
febrícula ocasional y 4-6 deposiciones diarias, alguna de ellas nocturna, con mucosidad y, en ocasiones, con
hebras de sangre. Entre los diagnósticos que se enumeran a continuación seleccione el que le parece
MENOS probable:

Seleccione una:

a. Cáncer de colon.

b. Salmonelosis.

c. Enfermedad de Crohn.

d. Infección por Clostridium Difficile.

La respuesta correcta es: Cáncer de colon.


Pregunta 14 Correcta Se puntúa 0,20 sobre 0,20

Paciente hipertenso con uso de 3 antihipertensivos uno de los cuales es un diurético que persiste con
presiones arteriales elevadas. Usted pensaría en

Seleccione una:

a. hipertensión enmascarada

b. hipertensión resistente

c. hipertensión de bata blanca

d. hipertensión refractaria

La respuesta correcta es: hipertensión resistente

Pregunta 15 Correcta Se puntúa 0,20 sobre 0,20

Paciente que en monitoreo ambulatorio de presión arterial presenta cifras tensionales en parámetros
adecuados, pero en el consultorio presenta cifras tensionales elevadas. Usted pensaría en

Seleccione una:

a. hipertensión de bata blanca

b. hipertensión enmascarada

c. hipertensión resistente

d. hipertensión refractaria

La respuesta correcta es: hipertensión de bata blanca


Pregunta 16 Correcta Se puntúa 0,20 sobre 0,20

Paciente que presenta cifras tensionales en el consultorio dentro de parámetros normales, pero evidencia de
retinopatía hipertensiva. Usted pensaría en

Seleccione una:

a. hipertensión de bata blanca

b. hipertensión resistente

c. hipertensión refractaria

d. hipertensión enmascarada

La respuesta correcta es: hipertensión enmascarada

Pregunta 17 Correcta Se puntúa 0,20 sobre 0,20

Qué endocrinopatía no causa prolongación de QT y de taquicardia ventricular polimorfa:

Seleccione una:

a. Hipogonadismo

b. Hiperparatiroidismo

c. Hipotiroidismo

d. Hiperaldosteronismo

La respuesta correcta es: Hipogonadismo


Pregunta 18 Correcta Se puntúa 0,20 sobre 0,20

Qué familia de antibiótico no causa prolongación de QT y de taquicardia ventricular polimorfa:

Seleccione una:

a. Macrólidos

b. Cefalosporinas

c. Antagonistas del folato

d. Quinolonas

La respuesta correcta es: Cefalosporinas

Pregunta 19 Correcta Se puntúa 0,20 sobre 0,20

Qué vitamina causa insuficiencia cardiaca con alto gasto:

Seleccione una:

a. A

b. B

c. E

d. D

La respuesta correcta es: B


Pregunta 20 Incorrecta Se puntúa 0,00 sobre 0,20

Señale la respuesta correcta, respecto al tratamiento de la enfermedad de Crohn:

Seleccione una:

a. En algunos casos es útil la asociación de metronidazol

b. En brotes leves no se recomienda usar como primera elección el infliximab.

c. Los esteroides se usan como tratamiento de mantenimiento, son útiles para evitar recidivas de la
enfermedad.

d. La azatioprina puede tardar varios días en iniciar su efecto.

La respuesta correcta es: En algunos casos es útil la asociación de metronidazol

◄ Registro de asistencia Ir a...


Página 1 -52

Paciente de sexo masculino, de 48 años de edad que presenta una tumefacción


eritematosa de bordes mal definidos, muy dolorosa a la palpación en tórax posterior
de consistencia semidura, con un pequeño orificio central a través del cual drena
material purulento de mal olor de 8 días de evolución. ¿Cuál de las siguientes
alternativas corresponde al diagnóstico más probable?
Seleccione una:
a. Forúnculo
b. Celulitis
c. Foliculitis profunda
d. Erisipela

Dentro de las manifestaciones tardías de la sífilis tenemos trastornos


cardiovasculares de cual podemos desprender el siguiente:
Seleccione una:
a. Aneurismas
b. Flebitis irritativa.
c. Trastornos del endotelio
d. Insuficiencia cardiaca congestiva

En caso que se enfrente a una situación de tétanos una medida de soporte importante
para el control de espasmos es:
Seleccione una:
a. Gabapentina
b. Metronidazol
c. Antitoxina
d. Diazepam
En la clasificación de OMS de adultos con VIH – Sida en el estadio C3 tenemos a:
Seleccione una:
a. Mayor de 400 células CD4+ con síntomas no definidores de Sida.
b. Menor de 200 células CD4+ con síntomas definidores de Sida.
c. Menor de 300 células CD4+ con síntomas definidores de Sida.
d. Mayor de 500 células CD4+ Asintomático.

Paciente de sexo masculino, de 48 años de edad que presenta una tumefacción


eritematosa de bordes mal definidos, muy dolorosa a la palpación en tórax posterior de
consistencia semidura, con un pequeño orificio central a través del cual drena material
purulento de mal olor de 8 días de evolución. ¿Cuál de las siguientes alternativas
indica los antibióticos de elección en este caso?
Seleccione una:
a. Amoxicilina + ácido clavulanico
b. Dicloxacilina y linezolid
c. Ceftriaxona y gentamicina
d. Dicloxacilina y trimetroprim sulfametoxazol

Qué infección causa bloqueo auriculoventricular:


Seleccione una:
a. Varicela
b. Tripanosomiasis
c. Sífilis
d. Sarampión

-
Cuál consideraría como primera opción para el diagnóstico de fiebre tifoidea ( fiebre
enterica):
Seleccione una:
a. biopsia intestinal
b. Reacción de Widal
c. rosa de bengala
d. hemocultivo

En cuanto a la profilaxis antitetánica en el tratamiento sistemático de las heridas, en


una herida pequeña limpia sin ningún antecedente de vacunación antitetánica Usted
recomendaría:
Seleccione una:
a. Es indicativo de inmunoglobulina.
b. Dar antibiótico de manera profiláctica.
c. Solo limpiar la herida y dar indicaciones de cuidado al paciente, ofrecer la antitoxina.
d. Vacunación antitetánica de inmediato.

Paciente de sexo masculino, de 48 años de edad que presenta una tumefacción


eritematosa de bordes mal definidos, muy dolorosa a la palpación en tórax posterior de
consistencia semidura, con un pequeño orificio central a través del cual drena material
purulento de mal olor de 8 días de evolución. ¿Cuál de las siguientes alternativas
indica las bacterias que con más frecuencia ocasionan este cuadro?
Seleccione una:
a. Estafilococo aureus, bacterias anaerobias, gram negativos
b. Estafilococo aureus, clostridium, pseudomona
c. Estreptococo beta hemolítico del grupo a, pseudomona, h. Influenza
d. Estreptococo beta hemolítico del grupo a, neumococo, eschericia coli

Tras la infección de un niño con el virus de la Influenza con la siguientes


características: fiebre intensa de inicio súbito, disnea y cianosis a la Rx de tórax se
aprecia patrón asociado con infiltrados intersticiales difusos e hipoxia intensa, usted
sospecharía de:
Seleccione una:
a. Neumonía bacteriana secundaria.
b. Neumonía viral secundaria.
c. Neumonía bacteriana primaria.
d. Neumonía viral primaría.

Cuál de las siguientes afirmaciones corresponden a Neumonía Adquirida en la


Comunidad.
Seleccione una:
a. Pacientes que viven en hogares de cuidados crónicos.
b. Pacientes que presentan síntomas durante las 48 primeras horas de la admisión al
hospital
c. Cuadro de presentación aguda.
d. Pacientes no hospitalizados en los 14 días anteriores

Cuál de las siguientes afirmaciones NO corresponde al diagnóstico diferencial de la


EPOC.
Seleccione una:
a. Obstrucción de vía aérea superior
b. Tromboembolismo pulmonar
c. Paro respiratorio
d. Infarto agudo de miocardio

¿Cuál de las siguientes alternativas indica el carcinoma que prevalece en el tercio


superior del esófago?
Seleccione una:
a. Carcinoma in situ
b. Adenocarcinoma
c. Carcinoma escamoso
d. Carcinoma atípico

¿Cuál de los siguientes hallazgos histológicos es necesario para establecer el


diagnóstico de cirrosis hepática?
Seleccione una:
a. Degeneración baloniforme de los hepatocitos.
b. Cuerpos de Mallory.
c. Presencia de fibrosis.
d. Destrucción de los conductos biliares.

¿Cuáles son los factores de riesgo débiles para TEP?


Seleccione una:
a. Quimioterapia, embarazo
b. Fractura de cadera, cirugía mayor
c. Hemofilia, lesión medular
d. Ancianos, obesidad, venas varicosas

El esófago de Barrett se caracteriza por, señale la alternativa correcta:


Seleccione una:
a. Se asocia con divertículo de Zenker
b. Metaplasia intestinal
c. Se lo considera carcinoma in situ de esófago distal
d. Se asocia con el anillo de Schatzki.

El siguiente resultado de LCR: células blancas 200, de predominio polimorfonuclear,


proteínas 600 mg/dl,hipoglucorraquia de 26 mg/dl, corresponde a qué enfermedad:
Seleccione una:
a. Meningitis viral
b. Meningitis tuberculosa
c. Encefalitis viral
d. Meningitis bacteriana

El tratamiento de elección en colitis pseudomembranosa que cursa con íleo paralítico


es, señale la alternativa correcta:
Seleccione una:
a. Metronidazol por vía oral.
b. Vancomicina por vía oral.
c. Metronidazol por vía intravenosa.
d. Vancomicina por vía intravenosa

Muchacha de 26 años hospitalizada por ictericia de instauración reciente, asociada a


dolor en hipocondrio derecho. Se detecta hepatomegalia sensible sin
esplenomegalia. Hay telangiectasias faciales. Reconoce antecedentes de
promiscuidad sexual, pero no consumo de droga intravenosa. AST 315 ;BT 16 mg/dl;
alt 110 ; GGT 680; FA 280 ; triglicéridos 600, colesterol 280. Ecografía con patrón
ecogénico del hígado. Cuál de las siguientes alternativas indica el diagnóstico más
probable:
Seleccione una:
a. Hepatopatía alcohólica
b. Hepatitis Autoinmune
c. Hepatitis tóxica
d. Hepatitis vírica

Paciente de 48 años sin APP, reside en una comunidad rural de la provincia de


Cotopaxi, agricultor, acude por 1 crisis convulsiva focal motora izquierda, en el
trabajo diagnóstico que requiere para averiguar la etiología de la crisis, usted
señalaría que enunciado como verdadero:
Seleccione una:
a. La zona de residencia endémica para cisticercosis es considerado un criterio de
exposición pero no
forma parte de los criterios diagnósticos
b. Se puede solicitar determinación de anticuerpos anticisticercosis o detección de
antígenos que son parte de los criterios diagnósticos de cisticercosis
c. La demostración histológica del parásito a través de una biopsia es considerado un
criterio menor de
diagnóstico de neurocisticercosis
d. El hallazgo en TAC cerebral de calcicaciones parenquimatosas denotan un criterio
absoluto de
neurocisticercosis

Paciente de 50 años que presenta un derrame pleural con las siguientes características:
aspecto pajizo, Ph 7.3, cociente de proteínas pleura/suero 0.8, cociente de LDH
pleura/suero 0.9, Gram y Ziehl negativos, flípidos totales, colesterol y triglicéridos normales,
células mesoteliales <5%, intensa linfocitosis sin atipias,
ADA 64 U/l. ¿Qué diagnóstico le sugiere?
Seleccione una:
a. Derrame pleural tuberculoso.
b. Derrame pleural por insuciencia cardiaca (trasudado).
c. Empiema pleural.
d. Mesotelioma pleural.

¿Qué evalúan los criterios de Wells para tromboembolia pulmonar?


Seleccione una:
a. Diagnóstico definitivo de tromboembolia pulmonar
b. Pronóstico de supervivencia para pacientes con tromboembolia pulmonar
c. Criterios radiológicos para diagnosticar tromboembolia pulmonar
d. La probabilidad clínica de tener tromboembolia pulmonar

Respecto a la cirrosis hepática, escoja la respuesta correcta.


Seleccione una:
a. Las complicaciones son consecuencia de la hipertensión portal y la insuficiencia hepática.
b. Es un proceso reversible.
c. Es un proceso localizado caracterizado por fibrosis y la conversión de la arquitectura
normal en nódulos de estructura alterada.
d. La cirrosis es la etapa nal de cualquier enfermedad aguda del hígado.

Respecto a la magnitud del daño causado por una tromboembolia pulmonar,


seleccione la respuesta
correcta:
Seleccione una:
a. Es únicamente importante la arteria que está siendo afectada (obstruida)
b. Es determinante el tamaño del émbolo y el diámetro de la arteria afectada (obstruida)
c. Lo principal es el tamaño, y son de importancia únicamente émbolos mayores a 2mm
d. Ningún factor es importante ya que el diagnóstico siempre es post mortem

Sobre la Enfermedad Inflamatoria Intestinal, escoja la respuesta INCORRECTA:


Seleccione una:
a. La mayor fuente de producción de estas interleucinas pro amatorias son los macrófagos
activados
de la lámina propia que estimulan las células inmunes.
b. El factor de necrosis tumoral y el interferón alfa contribuyen directamente al daño epitelial
del intestino.
c. Los niveles tisulares de IL 1 están elevados en la Enfermedad de Crohn y los niveles
séricos de IL 2 están igualmente elevados en la Colitis ulcerosa.
d. En la EII se encuentra aumentada la producción de las citokinas proinamatorias, IL 1, IL
6, IL 8, y además del FNT (factor de necrosis tumoral) alfa.
Una mujer de 25 años refiere una historia de 12 meses de cólicos abdominales recurrentes
en hemiabdomen inferior acompañado de distensión abdominal y diarreas. No presenta
fiebre, pérdida de peso ni anorexia ni deposición con sangre. Este paciente cumple con los
criterios de Roma IV para Sd de colon irritable con predominio de diarrea, no presenta
signos de alarma. ¿En qué otra patología Usted puede pensar?
Seleccione una:
a. Enfermedad inflamatoria intestinal.
b. Sobrecrecimiento bacteriano de intestino delgado.
c. Helicobacter pylori.
d. Enfermedad celiaca

Uno de los siguientes NO es síntoma general de la tuberculosis.


Seleccione una:
a. Pérdida de peso
b. Hematemesis
c. Síndrome febril
d. Sudoración nocturna

Un valor de > 4 en la escala de Wells indica que:


Seleccione una:
a. Clínicamente el paciente no tiene probabilidad de Embolismo Pulmonar
b. El paciente está inestable pero la probabilidad de que presente embolia pulmonar es baja
c. El paciente está estable y es poco probable que presente embolia pulmonar
d. Clínicamente el paciente tiene probabilidad de Embolismo Pulmonar

Con respecto a la fiebre de origen desconocido cual no es criterio para definirlo:


Seleccione una:
a. fiebre que dura 72 horas
b. persistencia por al menos tres semanas
c. al menos una semana de investigación hospitalaria
d. fiebre mayor a 38.3 oc.

Con respecto a la meningitis cual es el examen de elección para determinar si es


bacteriana o viral
Seleccione una:
a. biometría hemática
b. procalcitonina
c. punción lumbar
d. tomografía axial de cráneo
Con respecto a la parasitosis cuales son los factores de riesgos:
Seleccione una:
a. nivel nutricional adecuado
b. climas fríos
c. edad
d. lugares insalubres
cuál de los siguientes patógenos produce osteomielitis aguda
Seleccione una:
a. staphylococcus epidermidis
b. pseudomonas aeruginosa
c. esherichia coli
d. staphylococcus aureus

Cuál es el vector causante de la enfermedad del dengue:


Seleccione una:
a. mosquito aedes contaminado
b. agua contaminada
c. mosquito anofeles
d. mosquito aedes

¿Cuál NO es una causa de alcalosis respiratoria?


Seleccione una:
a. Crisis asmática.
b. Exposición a grandes alturas.
c. Tratamiento con salicilatos
d. Aldosteronismo primario

El tiempo de incubación para la varicela es:


Seleccione una:
a. 10 a 21 días
b. 5 a 10 días
c. 7 a 14 días
d. 2 a 8 días
En la fase primaria de la sífilis que tratamiento recomendaría
Seleccione una:
a. penicilina benzatínica 2400000 ui im semanal por 5 semanas
b. penicilina benzatínica 2400000 ui im semanal por 6 semanas
c. penicilina benzatínica 1200000 ui im semanal por 4 semanas
d. penicilina benzatínica 2400000 ui im semanal por 2 semanas

La brucelosis conocida también coma la:


Seleccione una:
a. fiebre terciaria
b. fiebre cuaternaria
c. fiebre de ondulante o malta
d. fiebre ondulante

La recomendación para el Manejo de primera línea, todo es verdad, Excepto:


Seleccione una:
a. Ausencia de varices esofágicas repetir endoscopia en 2-3 años
b. Varices esofágicas pequeñas – ausencia de hemorragia, repetir endoscopia en 1-2 años.
c. Hemorragia varicosa, tratamiento específico; fármaco vasoactivo sin ligadura
endoscópica de varice.
d. Varices esofágicas medianas / grandes – sin hemorragia, betabloqueantes o ligadura
endoscópica de
varices esofágicas si no tolera betabloqueantes.
Señale la respuesta FALSA de entre las siguientes en relación con el concepto de
gastritis:
Seleccione una:
a. Es un término que debe reservarse para definir la existencia de inflamación histológica en
la mucosa gástrica.
b. Existe un espectro de manifestaciones clínicas claramente definidas en relación con la
existencia de gastritis.
c. Existe una escasa correlación entre los datos histológicos, los síntomas del paciente y los
datos endoscópicos.
d. El Helicobacter pylori es una causa frecuente de gastritis.

Señale la respuesta correcta, respecto al tratamiento de la enfermedad de Crohn:


Seleccione una:
a. En brotes leves no se recomienda usar como primera elección el infliximab.
b. La azatioprina puede tardar varios días en iniciar su efecto.
c. En algunos casos es útil la asociación de metronidazol
d. Los esteroides se usan como tratamiento de mantenimiento, son útiles para evitar
recidivas de la
enfermedad.
Que infección intestinal, entre las siguientes puede confundirse con un brote de
enfermedad inflamatoria intestinal, por la similitud de hallazgos clínicos,
endoscópicos e incluso en los datos histopatológicos en la biopsia de colon:
Seleccione una:
a. Campylobacter yeyuni
b. Clostridium difficile
c. Absceso amebiano
d. Mycobacterium avium – complex

Son causas menos frecuentes de hemorragia digestiva alta aguda, Excepto:


Seleccione una:
a. Neoplasia
b. Desgarro de Mallory-Weiss
c. Ectasias vasculares
d. Esofagitis
Carmen de 40 años con obesidad mórbida, fue sometida a cirugía bariátrica. Durante
la intervención quirúrgica se realiza también una biopsia hepática, con hallazgo de
acumulación de lípidos neutros en el citoplasma de los hepatocitos como en las
células de Kupffer en forma vesículas que desplazan el núcleo celular a la periferia ¿A
qué tipo de lesión histológica hace referencia?:
Seleccione una:
a. Hepatitis crónica
b. Esteatohepatitis.
c. Cirrosis.
d. Esteatosis.
En relación al trasplante hepático, señale de las siguientes la respuesta CORRECTA:
Seleccione una:
a. El sistema MELD es un modelo pronóstico que permite la priorización para el trasplante
hepático de los enfermos en lista.
b. Las metástasis hepáticas por cáncer de colon constituyen una indicación aprobada para
el trasplante
hepático.
c. El trasplante hepático sería la mejor opción de tratamiento para un paciente de 45 años.
d. La indicación de trasplante hepático más frecuente en la actualidad es la cirrosis por
infección del virus
B. (VHB)

Indique la situación clínica que, en relación con la infección por virus de la hepatitis
B, presenta un paciente joven, con exploración física normal y con la siguiente
serología frente a hepatitis B: HBsAg + / ANTI-HBs – / HbeAg – / ANTI-HBe + /
ANTI-HBc IgM – / ANTI-HBc IgG + / DNA VHB +:Seleccione una:
a. Infección crónica.
b. Portador asintomático.
c. Infección aguda.
d. Paciente vacunado.

¿En qué lugar del tubo digestivo se absorbe el agua?


Seleccione una:
a. Esófago
b. Intestino grueso
c. Intestino delgado
d. Estómago

¿Cuál de las siguientes técnicas de endoscopia digestiva está asociada mayor riesgo de
complicaciones mayores?
Seleccione una:
a. Polipectomía de pólipo en colon descendente de morfología sésil.
b. Esclerosis de varices esofágicas.
c. Colonoscopia diagnóstica en paciente con dolicocolon.
d. Ecoendoscopia digestiva alta diagnóstica.

Hombre de 57 años que acude a urgencias por un dolor epigástrico de elevada


intensidad, irradiado a espalda y a hipocondrio derecho, junto a náuseas y vómitos
de 12 horas de evolución. En la exploración se encuentra afebril, estable
hemodinámicamente, lúcido y con buena perfusión. En la analítica destacan 18.000
leucocitos (80% neutrófilos), siendo el resto del hemograma normal. La bioquímica es
normal, incluyendo LDH y triglicéridos, excepto una amilasa de 3.000 U/L. Señale la
respuesta correcta:
Seleccione una:
a. No es necesario realizar prueba de imagen en urgencias o en todo caso una ecografía
abdominal.
b. Es preciso realizar una TC abdominal urgente.
c. El cuadro clínico sugiere una colangitis aguda.
d. El paciente tiene probablemente una isquemia mesentérica aguda.

Uno de los siguientes fármacos NO se utiliza en el tratamiento de la hepatitis crónica B:


Seleccione una:
a. Tenofovir.
b. Sofosbuvir.
c. Adefovir.
d. Entecavir.

En relación a la Peritonitis Bacteriana Espontánea, señale la respuesta Verdadera:


Seleccione una:
a. La PBE afecta 25% de los pacientes cirróticos con ascitis no internados.
b. La incidencia de PBE en cirróticos hospitalizados se encuentra en el 60%
c. La mortalidad durante la hospitalización en la que se diagnostica la PBE es de
aproximadamente el 30%
d. En ascitis hemorrágica se debe sustraer el 20% PMN cada 250 glóbulos rojos

Uno de los siguientes datos clínicos NO es sugerente del síndrome de colon irritable:
Seleccione una:
a. Dolor recurrente en hipogastrio.
b. Tenesmo rectal.
c. Pérdida de peso
d. Alternancia de diarrea/estreñimiento.

Juan de 67 años, con diagnóstico de cirrosis hepática y bebedor activo, ingresa por cuadro
de distensión abdominal progresivo con malestar difuso, de dos semanas de evolución. No
refiere fiebre ni otros síntomas. A la exploración destaca matidez cambiante a la percusión
abdominal, con ausencia de edemas. Se realiza una paracentesis diagnóstica, encontrando
un líquido ligeramente turbio, con 2.300 células/mL, de las cuales 30% son linfocitos, 60%
polimorfonucleares y 10% hematíes. ¿Cuál es la primera medida terapéutica que pautaría
en este paciente de forma inmediata?
Seleccione una:
a. Restricción de sal y líquidos.
b. Paracentesis evacuadora.
c. Tratamiento con una cefalosporina de tercera generación.
d. Tratamiento diurético con espironolactona oral.
Según la consistencia de las heces Bristol tipo 4 corresponde a:
Seleccione una:
a. Parecido a salchicha cuarteada
b. Parecido a salchicha blanda y lisa
c. Parecido a una salchicha, caprinas
d. Bolas duras y separadas
Varón de 52 años sin antecedentes patol ógicos de importancia, ingresa a Emergencias por presentar
melenas de 24 horas de evolución sin repercusión hemodinámica. Niega consumo de AINES . El
hematocrito es de 33% y el resto de la analítica es normal. La endos,c opia digestiva alta realizada de
forma urgente a las 6 horas del ingreso muestra un estómago normal, sin sangre ni restos hemáticos
y una úlcera excavada de 8 mm de diámetro en cara anterior del bulbo duodenal con «vaso visible» en
su base y sin sangrado activo. ¿Cuál de las siguientes afirmaciones es cierta?

Seleccione una:

a. En la e ndoscopia inicial está indicado aplicar una terapéutica endoscópica y posteriormente instaurar
tratamiento endovenoso con dosis altas de un inhibidor de la bomba de protones. Esta estrategia ha
demostrado reducir el riesgo de recidiva hemorrágica y la mortalidad . ..¡

b. En la e ndoscopia inicial, dada la ausencia de sangrado activo, no está indicado aplicar una terapéutica
endoscópica . Posteriormente, para redu cir el riesgo de recidiva hemorrágica , se deberá instaurar
tratamiento endovenoso con dosis altas de un inhibidor de la bomba de protones.

c. En la endoscopia inicial y dado que se trata de una úlcera complica.da (hemorragia) la mejor opción
terapéutica., una vez resuelto el episodio hemorrágico, es una vagotom ía y piloroplasti a.

d. En la e ndoscopia inicial está indicado aplicar una terapéutica endoscópica. No se ha podido demostrar
que instaurar posteriormente tratamiento con dosis altas de un inhibidor de la bomba de protones aporte
beneficio adiciona l alguno.

Juan de 55 años, padre de un hijo celiaco, cu rsa con anemia ferropén ica y deposiciones diarreicas con
frecuencia de 1-3 al dla de 5 semanas de evolución, se le ha realizado una determinación de los alelos HLA-
DQ2 y HLA-DQ8 que ha resultado negativa . ¿Qué estudio diagnóstico es el más apropiado en este caso?

Seleccione una:

a. Test de D-xilosa.

b. Determinación de anticuerpos antitransglutaminasa lgA. X


c. Endoscopia digestiva alta con toma de biopsias duodenales.

~ olonosco 1a

Respuesta incorrecta.

La respuesta correcta es: Colonoscopia.

Son causa de pancreatitis aguda las siguientes, Excepto:


Seleccione una:
a. Hipercalcemia.
b. Azatioprina.
c. CPRE.
d. Hipercolesterolemia

En relación al Esófago de Barrett, el segu imiento y conducta endoscópica todo es verdadero, Excepto:

Seleccione una:

a. Sin displasia se realiza cada 3 a 5 años. Considerar ablación endoscópica en casos seleccionados.

b. Displasia de bajo grado, valoración por un segundo patólogo experto, Repetir endoscopia en un año.

Considerar ablación o resección endoscópica. ~

c. Displasia de alto grado, valoración por un segundo patólogo experto. Repetir endoscopia en 3 meses.
Considerar ablación o resección endoscópica.

d. Indefinido para displasia, tratamiento con IBPs y repetir endoscopia y biopsia en ausencia de
inflamación.

Respuesta correcta

La respuesta correcta es: Displasia de bajo grado, valoración por un segundo patólogo experto, Repetir
endoscopia en un afio. Considerar ablación o resección endoscópica.

● El mecanismo por el cual la infección por cólera produce una diarrea tan intensa es
por: Toxina enterogénica
● En la clasiFICACIÓN DE FOD cual considera usted como una urgencia y debe ser
tratado de inmediato FOD neutropénico
● El interferón es un medicamento útil para las siguientes afecciones excepto VIH
● La infección de herpes en recién nacidos es principalmente dado por: VHS 2
● Dentro de las manifestaciones tardías de la sílis tenemos trastornos
cardiovasculares de cual podemos desprender el siguiente: Aneurismas
● ¿De qué depende la inmunidad contra Leptospira? Producción de anticuerpos
circulantes contra LPS especícos de un serotipo.
● En cuanto a la prolaxis antitetánica en el tratamiento sistemático de las heridas, en
una herida pequeña limpia sin ningún antecedente de vacunación antitetánica Usted
recomendaría: Solo limpiar la herida y dar indicaciones de cuidado al paciente, ofrecer la
anti toxina
● En la clasicación de OMS de adultos con VIH – Sida en el estadio C3 tenemos a:
Menor de 200 células CD4+ con síntomas definidores de Sida.
● Infeccion que causa un bloqueo AV Sifilis
● Tras la infección de un niño con el virus de la Influenza con la siguientes
características: ebre intensa de inicio súbito, disnea y cianosis a la Rx de tórax se
aprecia patrón asociado con inltrados intersticiales difusos e hipoxia intensa, usted
sospecharía de: Neumonía viral primaría.
● Desde que se quiere el virus del papiloma humano hasta que produzca verrugas o
condilomas un tiempo de de 3 a 4 meses
● Una de las características principales por las cuales el dengue puede diseminarse
es: El vector es muy cercano a los asentamientos humanos
● ¿A qué género pertenece el virus del dengue?
● ¿Cómo se denomina el vector transmisor de fiebre amarilla? Aedes aegypti
● ¿Cuál de las siguientes alternativas es una de las medidas para evitar la
diseminación del Dengue entre la población? Control vectorial
● ¿Cuántos días abarca el período de incubación de la leptospirosis? De 1 a 30 días
(aproximadamente 2 semanas)
● De las encefalitis virales seleccione la que se beneficia del tratamiento con Aciclovir:
Herpes virus
● Un paciente con sospecha de encefalitis debe tener excepto Deterioro cognitivo agudo
● Paciente de 48 años sin APP, reside en una comunidad rural de la provincia de Cotopaxi,
agricultor, acudepor 1 crisis convulsiva focal motora izquierda, en el trabajo diagnóstico que
requiere paraaveriguar laetiología de la crisis, usted señalaría que enunciado
como verdadero:Se puede solicitar determinación de anticuerpos
anticisticercosis o detección deantígenos que son parte de los criterios
diagnósticos de cisticercosis
● Los promastigotes son la forma de leishmania que se inyectan por medio de la
probóscide del ebótomo de sexo femenino en la piel del hospedador, ¿qué células
son las encargadas de fagocitar a estas formas parasitarias? Neutrófilos
● Forma de transmisión del virus de la hepatitis A: vía fecal- oral
● Paciente acude con trastornos osteomusculares crónicos, junto con déficit visual
inespecífico, pérdida de peso y ebre de larga evaluación, el paciente proviene de
una zona meramente ganadera, Usted sospecha de brucelosis cuya base de
tratamiento es: Dicloxacilina y Estreptomicina
● Paciente de 25 años que refiere viaje reciente a Filipinas presenta: fiebre, dolor
abdominal, diarrea, bradicardia relativa. ¿Cuál es el diagnóstico más probable?
Fiebre tifoidea
● Paciente masculino de 55 años de edad, sin antecedentes patológicos familiares de
relevancia. Acude por presentar desde hace 6 días dolor intenso y alodinia a nivel de
región lumbar derecha. Hoy en la mañana nota erupciones muy pruriginosas en la
piel. Antecedentes patológicos personales: varicela en su niñez. Al examen físico
presenta vesículas agrupadas sobre una placa eritematosa elevada que siguen un
mismo dermatoma. También se observa un desprendimiento de costras dejando
zonas expuestas de piel. ¿Cuál sería el diagnóstico probable y su terapéutica?
Herpes Zoster y su tratamiento más adecuado es Aciclovir 800 mg cada 4 horas
durante 7 días.
● Su paciente proviene de la amazonia del Ecuador y presenta aproximadamente 15
días con ebre alta con escalofríos, pero sin pérdida de peso considerable, solo con
ligera palidez, al examen físico se parecía muy levemente una hepato espleno
megalia, y Biometría sin cambios aparente, sería prudente realizar lo siguiente a su
criterio: Gota gruesa y extendido seriado
● En la malaria , la forma de plasmodio transmitida del mosquito al hombre es el
Esporozoito
● Mujer de 47 años de edad, nacida y residente en Orellana A mediados del año 2017
comienza conpalpitaciones, disnea y edemas progresivos en extremidades
inferiores. Es diagnosticada de insuficiencia cardiaca secundaria a miocardiopatía
por chagas cual es el agente Trypanosoma cruzi
● Paciente que acude a Urgencias con FIEBRE INTERMITENTE escalofrio cefalea
debilidad y abudante sudoración nocturna de 3 dias de evolución viaje a cayambe
consumo de leche cruda Prueba del Rosa de Bengala y toma de
hemocultivo

● Qué es el dengue? Una enfermedad vírica que puede producir una
fiebre hemorragica
● Un trabajador en el oleoducto en el Coca de 36 años de edad, sufre desde
hace dos meses un cuadroabdominal intermitente de náuseas, heces
pastosas, flatulencias perder tres kilos Giardia lamblia.
Recopilado RESPI
● Cuál de los siguientes es un signo característico de las vías respiratorias de las
personas que sufren asma: d. Inltración eosinofílica
● La causa más frecuente de hemoptisis leve-moderada es: Bronquiectasias.
● ¿Cuál es la clasicación anatómica de las bronquiectasias?: Cilíndricas, varicosas,
saculares
● qué grupo poblacional no restringiría la vacuna de la gripe Personas
alérgicas a las proteínas del huevo
● ¿Cuál es la siopatología de la hemoptisis? Hipervascularización de la circulación
brónquica, hipertensión pulmonar y neovascularización
● De los siguientes factores, cuál es el que debe estar alterado para que un paciente
infectado de tuberculosis se convierta en persona enferma: Huésped: Inmunidad
celular deciente, en especial CD4
● Cuál de las siguientes afirmaciones no corresponde al diagnostico
diferencial de EPOC Paro respiratorio
● Uno de los siguientes NO es síntoma general de la tuberculosis
Hematemesis
● A un joven asintomático de 14 años cuyo padre acaba de ser
diagnosticado de Tuberculosis Pulmonar se lerealiza un Mantoux,
con resultado negativo. ¿Qué actitud es la adecuada Iniciar
quimioprofilaxis y repetir la prueba cutanea a los 3 meses
● Con respecto a la infección por gripe indiqué la premisa verdadera
puede causar una neumonía viral primaria
● Paciente de 50 años que presenta un derrame pleural con las
siguientes características: aspecto pajizo, Ph7.3, cociente de
proteínas pleura/suero 0.8, cociente de LDH pleura/suero 0.9, Gram
y Ziehl negativos,lípidos totales, colesterol y triglicéridos normales,
células mesoteliales <5%, intensa linfocitosis sin atipias,ADA 64 U/l.
¿Qué diagnóstico le sugiere Derrame pleural tuberculoso
● El hallazgo más frecuente en la radiografía de tórax en un paciente con asma es:
Radiografía de tórax normal.
● El tratamiento de las Bronquiectasias se basa en 3 pilares, excepto:Revertir el
remodelamiento bronquial
● Respecto a la magnitud del daño causado por una tromboembolia
pulmonar, seleccione la respuesta Es determinante el tamaño del
émbolo y el diámetro de la arteria afectada (obstruida)
● La enfermedad pulmonar obstructiva crónica no tratada puede conducir a la
siguiente complicación cardíaca: Insuciencia cardíaca derecha
● La insuficiencia cardíaca izquierda puede provocar edema agudo de pulmón por
aumento de la Presión capilar pulmonar
● En cuanto al diagnóstico funcional del asma: Si la relación VEF1/CVF es menor a
0.7 (patrón obstructivo) y post broncodilatador obtenemos un aumento igual o
mayor al 12% en el VEF1, nos orienta hacia diagnóstico de asma
● La enfermedad pulmonar obstructiva crónica no tratada puede conducir a la
siguiente complicación cardíaca Insuficiencia cardíaca derecha
● Cuál de las siguientes afirmaciones corresponden a Neumonía adquirida en la
comunidad : Pacientes que viven en hogares de cuidados crónicos
● Ante un derrame pleural que esperaría encontrar a la percusión? . Matidez
● ¿Cuál de los siguientes componentes de la estructura de los virus gripales es el
principal responsable de su infecciosidad? Hemaglutinina
● ¿Cuáles son los factores de riesgo débiles para TEP? Ancianos,
obesidad, venas varicosas
● Señale la respuesta incorrecta con relación a la tuberculosis . La primoinfección
tuberculosa es aquella situación en la que el individuo entra en contacto con el
germen y desarrolla por primera vez la enfermedad
● ¿Cuál es la fisiopatología de la hemoptisis Hipervascularización de la
circulación brónquica, hipertensión pulmonar y neovascularización
● Cuál de las siguientes es una causa de hemoptisis? Estenosis Mitral
● Cuál de los siguientes enunciados corresponde a la definición de neumonía
nosocomial tardía Aparece después de 7 días del ingreso
● Durante un aviso nocturno usted acude al domicilio de una paciente de 47 años de
edad, diagnosticada de asma y que realiza habitualmente tratamiento con un
corticoide inhalado y un beta2 de larga duración. Sufamilia le explica que ha
presentado una crisis de asma que ha tratado con varias dosis de salbutamol
administradas a través de una cámara de inhalación sin observar mejoría. Al
inspeccionar a la paciente usted observa que realiza un importante trabajo
respiratorio con utilización de la musculatura respiratoria accesoria que usted
interpreta como una agudización grave de asma. ¿Cuál de las siguientes
asociaciones de hallazgos es más probable que usted pueda observar? Silencio
auscultatorio - espiración alargada - percusión timpánica.
● ¿De qué depende el mayor riesgo de ENFERMARSE de Tuberculosis? Inmunidad
celular, CD4
● El derrame pleural puede ser causado por diferentes causas. ¿En cuál de ellos usted
sugiere realizar una pleurodesis? Por Cáncer
● El dolor tipo pleurítico se caracteriza por ser Punzante intenso que aumenta con la
inspiración
● Recibe el reporte de una muestra de líquido pleural con los siguientes resultados:
exudado amarillento, recuento del 90% de linfocitos. Pertenece a un paciente que
presenta ebre y baja de peso, ¿en cuál de los siguientes diagnósticos usted
pensaría? Tuberculosis
● Señale las características del síndrome atípico de la Neumonía Adquirida en la
Comunidad. Disociación clínico radiológica
● La oligohemia distal a un vaso ocluido, se denomina: Westermark
● Cuál de las siguientes premisas es FALSA respecto del diagnóstico de
Tromboembolismo de Pulmón: El ecodoppler venoso es el estudio de mayor
sensibilidad y especificidad para el diagnóstico de Tromboembolismo de Pulmón.
● Son factores de riesgo para neumonía nosocomial Aspiración de contenido gástrico,
Reintubación
● Cuál es el cuadro clínico característico de los pacientes con Bronquitis crónica? .
llamados sopladores azules, cianóticos y caquécticos
● Cuáles son los principales factores de riesgo para el desarrollo de NAC?.. Extremos
de edad (ancianos y niños), tabaquismo, alcoholismo, diabetes, asma, insuficiencia
cardíaca, EPOC, cáncer, inmunodeprimidos.
● Cuáles son los 3 síntomas o signos más frecuentes de la EPOC Tos crónica, disnea
crónica y expectoración mucosa
● Con respecto a la EPOC, no es cierto: : Contrariamente al asma, no hay componente
inflamatorio
● En la exploración funcional de un paciente con enfermedad pulmonar obstructiva
crónica, son esperables todos los hallazgos MENOS uno: Volúmenes pulmonares
disminuidos
● Actualmente se utilizan 3 clases de broncodilatadores en pacientes con asma,
señale lo correcto: . Agonistas β2, anticolinérgicos, teofilina
● Cuál de las siguientes anomalías electrocardiográficas suele asociarse a
tromboembolia pulmonar? S1Q3T3, ondas T negativas en las derivaciones anteriores
de V1 a V4

● Qué técnica o espécimen ofrece el rendimiento más alto para el diagnóstico de la
tuberculosis pleural? Estudio directo de biopsia con histología de pleura
● Paciente femenino de 71 años con fractura de tobillo, presenta dolor en punta de
costado con una intensidad de 7/10, que se agrava al movimiento y la inspiración, se
acompaña de disnea de aparición súbita, tos y hemoptisis, FC: 114, FR: 26, T: 37.8
ºC. APP: Adenocarcinoma pulmonar con metástasis óseas en columna lumbar y
metástasis cerebral cosiderado etapa 4. Al valorarlo por escala de Weels usted
considera que: El paciente se encuentra en un riesgo elevado de presentar
tromboembolia pulmonar
● La acumulación de líquido en el espacio pleural, según Sahn se debe a:Aumento de
permeabilidad capilar
● Cuál de los siguientes signos radiográcos no corresponde a un hallazgo de
neumotórax? Joroba de Hampton.
● Cuál es la mejor opción al identicar un paciente sintomático respiratorio. 2 muestras
de BAAR en esputo
● Un paciente de 35 años, consulta por cuadro de 4 semanas de evolución, de tos,
con expectoración mucopurulenta, con estrías de sangre en algunas ocasiones,
asociado a compromiso del estado general, baja de peso, ebre intermitente y
sudoración nocturna. Ha tomado amoxicilina en varias oportunidades, sin respuesta.
Al examen físico destacan crépitos escasos, mayores en el ápice derecho. El
diagnóstico más probable es: Tuberculosis
● ¿En cuál sistema arterial se producen la mayoría de las hemoptisis y por qué?
Señale lo CORRECTO b. Las arterias bronquiales forman parte de la circulación sistémica,
tienen mayor presión y mucho menor flujo, de ellas depende la irrigación de los
bronquios y la pleura visceral
Página 53-104

1. En cuanto al manejo de la fibrosis pulmonar cual es el fármaco antifibrotico


a. Pirfenidona
2. La oxigenoterapia es importante en el manejo de la fibrosis pulmonar
porque elija la respuesta incorrecta
a. Detiene el daño pulmonar
3. Las vacunas frente a la gripe que se utilizan actualmente se caracterizan por
a. Variar su composición antigénica cada año, como consecuencia de
la deriva antigénica de los virusgripales A y B.
4. Paciente de 50 años que presenta un derrame pleural con las siguientes
características: aspecto pajizo, Ph7.3, cociente de proteínas pleura/suero 0.8,
cociente de LDH pleura/suero 0.9, Gram y Ziehl negativos,lípidos totales,
colesterol y triglicéridos normales, células mesoteliales <5%, intensa
linfocitosis sin atipias,ADA 64 U/l. ¿Qué diagnóstico le sugiere?
a. Derrame pleural tuberculoso
5. Qué evalúan los criterios de Wells para tromboembolia pulmonar
a. La probabilidad clínica de tener tromboembolia pulmonar
6. Un valor de > 4 en la escala de Wells indica que
a. Clínicamente el paciente tiene probabilidad de Embolismo
Pulmonar

1. A qué grupo poblacional no restringiría la vacuna de la gripe:


a. Personas alérgicas a las proteínas del huevo

b. Niños con antecedentes de síndrome de Reye

c. Personas con trastornos crónicos metabólicos

d. Personas con antecedente de Guillain Barré


2. ¿Cuál de los siguientes componentes de la estructura de los virus gripales es el principal
responsable de su infecciosidad?
a. Envoltura lipídica.
b. Hemaglutinina.
c. ARN polimerasa..
d. Neuraminidasa.

3. Mujer de 47 años de edad, nacida y residente en Orellana A mediados del año 2017
comienza con palpitaciones, disnea y edemas progresivos en extremidades inferiores.
Es diagnosticada de insuficiencia cardíaca secundaria a miocardiopatía por Chagas.
¿Cuál es el agente etiológico de esta enfermedad endémica?
a. Leishmania donovani.
b. Trypanosoma cruzi..
c. Giardia lamblia.
d. Trypanosoma brucei.
4. Para el tratamiento de Leishmania cutánea de primera línea se utiliza:
a. Dapsona
b. Anfotericina B
c. Antimonio pentavalente
d. Fluconazol
5. Joven de 23 años, que trabaja como DJ en un bar nocturno y tiene una historia de
promiscuidad bisexual e ingesta de drogas sintéticas, consulta por fiebre, malestar
general y dolor en glande. La exploración física muestra múltiples tatuajes, piercings
y la presencia de tres lesiones vesiculosas en glande y adenopatías inguinales
bilaterales. ¿Cuál es el diagnóstico más probable?
a. Infección por citomegalovirus.
b. Condilomas acuminados.
c. Primoinfección VIH.
d. Herpes genital.
6. La infección de herpes en recién nacidos es principalmente dado por:

-
a. VHH 8
b. VHS 2
c. VHS 1
d. HVZ
7. Con respecto al tétanos, ¿cuál de las siguientes frases es la correcta?
a. El tétanos está caracterizado por una rigidez generalizada junto a crisis de
espasmos musculares.
b. El tétanos aparece tras un periodo mínimo de incubación de 14 días.
c. La mortalidad del tétanos es todavía en la actualidad superior al 50%.
d. En el diagnóstico del tétanos, es fundamental haber efectuado
cultivos previos al inicio del tratamiento.
8. Lesión de Ghon es:
a. Diseminación miliar de tuberculosis.
b. linfadenopatia de cuello
c. pequeño nódulo calcificado
d. Ganglio de ápice de pulmón
9. Paciente diabética de 65 años de edad que consulta por un cuadro de dolor en la
cara, fiebre y aparición de una lesión inflamatoria bien delimitada que
diagnosticamos de erisipela. ¿Cuál es la etiología del proceso?
a. Streptococcus agalactiae.
b. Staphylococcus aureus.
c. Staphylococcus epidermidis.
d. Streptococcus pyogenes
10. ¿Qué es el dengue?
a. Una enfermedad vírica que ocasiona un eritema que evoluciona a
mácula y pápula afectando fundamentalmente a la población
infantil.
b. Una enfermedad causada por un poxvirus.
c. Una zoonosis que afecta al hombre ocasionalmente.
d. Una enfermedad vírica que puede producir una fiebre hemorrágica.
11. En la malaria, la forma de plasmodio transmitida del mosquito al hombre es el:
a. Hipnozoito.
b. Merozoito.
c. Gametocito.
d. Esporozoito.

-
12. Artritis reactiva es una complicación de:
a. Shigella
b. Giardiasis
c. Salmonelosis
d. Entamoeba histolytica
13. Tras la infección, ¿qué plasmodios parásitos humanos persisten como hipnozoitos
en el hígado y pueden causar recidivas?
a. P. falciparum.
b. P. cynmology y P. knowlesi.
c. P. malariae.
d. P. vivax y P. ovale.
14. La BCG es una vacuna atenuada derivada del:

-
a. M. leprae
b. M. bovis
c. M. tuberculosis
d. M. avium - intracellulare

-
15. Cuál es el grupo sanguíneo y con mayor susceptibilidad al contagio de cólera:
a. Grupo O
b. Grupo A
c. Ninguno
d. Grupo AB
16. Su paciente presenta las siguientes características fiebre alta prolongada dolor
abdominal diarrea y estreñimiento ocasionales esplenomegalia y roséola usted sospecha
de:
a. Shigella
b. Entamoeba histolytica
c. Salmonelosis
d. Giardiasis
17. Erisipela una infección de tejido blando es debido principalmente a:
a. Pseudomona aeruginosa
b. Enterobacterias
c. S. pyogenes
d. S. aureus
18. Con respecto a la infección por gripe indiqué la premisa verdadera:
a. puede causar una neumonía viral primaria
b. El virus solamente tiene mutaciones puntuales
c. la Clínica generalmente se asocia a trastornos respiratorios altos
d. casi siempre se asocia con febrícula, o sin fiebre sino con tos seca
19. Un varón de 19 años consulta por un cuadro de febrícula, mialgias y odinofagia,
acompañado de máculas cutáneas generalizadas no pruriginosas durante los
últimos 7 días. En el interrogatorio refiere, como único suceso médico, haber
presentado una úlcera anal que le causaba dolor a la defecación en el mes anterior
que desapareció progresivamente sin ninguna intervención. ¿Cuál es el germen
causante de este cuadro
a. Treponema pallidum.
b. Gonococo.
c. Chlamydia trachomatis.
d. Virus de herpes simple tipo 2.
20. Cuando la infección de la Tripanosoma cruzi ingresa por piel puede haber una
reacción inflamatoria en el sitio de la infección que dura hasta ocho semanas
denominado:
a. Complejo oftalmo ganglionar del Chagas.
b. Signo de Romaña - Mazza.
c. Reacción de Fernández.
d. Chagoma
21. ¿Cuál de los siguientes microorganismos que infectan el tracto intestinal causa
bacteriemia con mayor frecuencia?
a. Salmonella
b. Vibrio cholerae.
c. Shigella flexneri
d. Campylobacter jejuni.
22. En relación al tratamiento de la sífilis, ¿cuál de las siguientes es la respuesta correcta?
a. La penicilina es el tratamiento de elección sólo en la neurosífilis.
b. No hay ningún tratamiento efectivo.
c. La penicilina es el tratamiento de elección en cualquier estadio.
d. El tratamiento de elección son las tetraciclinas.
23. Qué patología de las siguientes es una condición define Sida:
a. displasia cervical
b. leucoplasia vellosa
c. herpes tipo 1: úlceras crónicas > 1 mes
d. candidiasis oral
24. Que caracteriza a una primoinfección por herpes tipo 1:
a. distribución en dermatoma en especial tórax
b. pápulas y pústulas intersticio entre piel y mucosa
c. trastornos de córnea y conjuntiva
d. faringitis o gingivoestomatitis
25. La tuberculosis osteoarticular ataca principalmente a columna a lo que se denomina:
a. Trastorno de Jacob
b. Osteofitosis
c. Mal de Pott
d. Enfermedad de Wegener
26. Paciente que acude a Urgencias con fiebre intermitente, escalofríos, cefalea, debilidad y
abundante sudoración nocturna de 3 días de evolución. Como antecedente
epidemiológico destaca un viaje a Cayambe hace 3 semanas donde consumieron leche
cruda y queso sin pasteurizar. El modo más rápido de diagnosticar la infección
sospechada es:
a. Prueba del Rosa de Bengala y toma de hemocultivo.
b. Gram directo del líquido cefalorraquídeo.
c. Tinciones de micobacterias en esputo u orina.
d. Test para detección de anticuerpos heterófilos (Paul-Bunnell).
27. Un trabajador en el oleoducto en el Coca de 36 años de edad, sufre desde hace dos
meses un cuadro abdominal intermitente de náuseas, heces pastosas, flatulencia,
meteorismo y que le han llevado a perder tres kg de peso. ¿Cuál de los siguientes
microorganismos sería con mayor probabilidad el responsable del cuadro?
a. Trichomonas hominis.
b. Entamoeba coli.
c. Giardia lamblia
d. Vibrio cholerae.
28. Si durante el examen físico en el área genital encuentra una úlcera lisa no purulenta bien

-
delimitado única superficial con linfadenopatía bilateral usted piensa en:
a. sífilis
b. Leishmania
c. chancroide
d. herpes tipo 2
29. En relación a la encefalitis, señale el enunciado FALSO:
a. El proceso infeccioso y respuesta inflamatoria se produce en el
parénquima cerebral, en algunos casos puede haber meningitis asociada
(meningoencefalitis)
b. La clínica puede expresarse como alucinaciones, agitación, cambios de
personalidad, trastornos conductuales y a veces un estado psicótico
franco
c. En adultos inmunocompetentes brotes episódicos de infección son causados por
herpesvirus (HSV, VZV, EBV), pero puede haber epidemias causadas por
arbovirus
d. En caso de comprobación de una encefalitis viral por arbovirus debe iniciarse
inmediatamente tratamiento con Aciclovir intravenoso por 14 a 21 días
30. En relación a la cisticercosis señale el enunciado FALSO::
a. La infección por cisticerco se deriva de la teniasis por Tenia saginata,
cuyo hospedario es el ser humano
b. El tratamiento se lo debe hacer con un fármaco cisticida como el praziquantel o
el albendazol
c. Las infecciones de cisticercosis pueden ser asintomáticas, pero también
presentarse con crisis convulsivas o no convulsivas, cefalea,
hidrocefalia, etc
d. La infección puede afectar a cualquier órgano de la economía, a nivel
cerebral puede invadir el parénquima, sistema ventricular o
subaracnoideo, etc.

Página 105-159 (Gabi)

Paciente que sufrió un cuadro de gastroenteritis aguda y 10 días después, desarrolla debilidad
de las 4 extremidades que evolucionó en el lapso de 4 días respiratoria, en el examen físico se
encontró cuadriparesia fláccida arrefléctica con respuesta plantar exora, sin trastorno sensitivo,
cuál sería el diagnóstico correcto y la conducta indicada:

A. Mielitis transversa, indicar inmunoglobulina IV


B. Sd Guillain Barré, indicar plasmaféresis
C. Sd Guillain Barré, indicar corticoterapia
D. Mielitis transversa, indicar corticoterapia

Cuál de los siguientes agentes es el causante de erisipela y celulitis:

A. Cándida
B. Estreptococo del grupo A
C. Estafilococo Aureus
D. Estreptococo del grupo B
El mejor tratamiento para el estómago en sandia (hipertensión Portal) es:

● a. Antrectomía
● b. Fármacos que promuevan la motilidad.
● c. Gastrectomía total.
● d. Agentes para reducir el acido

El tratamiento estándar de un tumor estromal gastrointestinal del estómago es:

Seleccione una:

● a. Gastrectomía total.
● b. Gastrectomía subtotal.
● c. Ablación endoscópica.
● d. Resección en cuña con márgenes limpios

El tratamiento para el síndrome de vaciamiento rápido temprano grave después de gastrectomía


es:

● a. Conversión quirúrgica a drenaje tipo Roux en Y.


● b. Tratamiento expectante.
● c. Octreotido
● d. Glucosa oral para los síntomas

La prueba diagnóstica más exacta para establecer el síndrome de Zollinger-Ellison es:

● a. Prueba de estimulación con secretina


● b. Gastrina sérica en ayuno
● c. Endoscopia
● d. Tomografía por computadora

Lo más frecuente es que la lesión de Dieulafoy derive en:

● a. Progresión a cáncer gástrico.


● b. Hemorragia gastrointestinal superior
● c. Gastroparesia
● d. Obstrucción de la salida gástrica.

Una de las siguientes afirmaciones es verdadera, respecto a la infección por c. dicile.

● a. Se debe esperar el cultivo mas antibiograma antes de iniciar tratamiento.


● b. Aproximadamente un 70% sufren una recaída al finalizar el tratamiento.
● c. Ante un caso de colitis fulminante, si pasados 3 días de tratamiento antibiótico no hay
respuesta, está indicado el tratamiento quirúrgico.
● d. La vía de contagio es oral-oral.

Una mujer con 65 años de edad con úlcera duodenal conocida se trata con dieta y un
bloqueador H2. Ingresa al hospital con hemorragia gastrointestinal superior con compromiso
hemodinámico. Junto a iniciar la reposición sanguínea, el próximo paso en el tratamiento sería

● a. Endoscopia y coagulación del vaso sangrante


● b. Iniciar omeprazol
● c. Iniciar bismuto, tetraciclina y metronidazol
● d. Piloroduodenotomía y cierre del vaso sangrante con sutura

Una mujer de 25 años refiere una historia de 12 meses de cólicos abdominales recurrentes en
hemiabdomen inferior acompañado de distensión abdominal y diarreas. No presenta ebre,
pérdida de peso ni anorexia ni deposición con sangre. Este paciente cumple con los criterios de
Roma IV para Sd de colon irritable con predominio de diarrea, no presenta signos de alarma. En
que otra patología usted puede pensar?.

● a. Hipertiroidismo.
● b. Helicobacter pylori.
● c. Enfermedad celiaca.
● d. Enfermedad inflamatoria intestinal.

Tras la infección de un niño con el virus de la Inuenza con la siguientes características: ebre
intensa de inicio súbito, disnea y cianosis a la Rx de tórax se aprecia patrón asociado con
inltrados intersticiales difusos e hipoxia intensa, usted sospecharía de:

● a. Neumonía viral secundaria.


● b. Neumonía viral primaría.
● c. Neumonía bacteriana primaria.
● d. Neumonía bacteriana secundaria.

Paginas 160-212

En la clasificación de OMS de adultos con VIH – Sida en el estadio C3 tenemos a: Seleccione


una:

a. Mayor de 400 células CD4+ con síntomas no definidores de Sida.

b. Menor de 200 células CD4+ con síntomas definidores de Sida.


c. Menor de 300 células CD4+ con síntomas definidores de Sida. d. Mayor de 500 células CD4+
Asintomático

En cuanto a la profilaxis antitetánica en el tratamiento sistemático de las heridas, en una herida


pequeña limpia sin ningún antecedente de vacunación antitetánica Usted recomendaría:
Seleccione una:

a. Es indicativo de inmunoglobulina.

b. Dar antibiótico de manera profiláctica.

c. Solo limpiar la herida y dar indicaciones de cuidado al paciente, ofrecer la anti toxina

. d. Vacunación antitetánica de inmediato.


Página 260-312
¿En cuál de los siguientes pacientes que acudió al departamento de urgencias
y señalan disnea aguda sería positiva una prueba de dímero D que obligara a
realizar estudios adicionales para identificar un émbolo pulmonar?
A. Mujer de 24 años con 32 semanas de gestación.
B. Varón de 48 años sin antecedentes personales que acudió por dolor en la
pantorrilla después de un viaje aeronáutico duradero; el gradiente de oxígeno
alveolo-arterial es normal.
C. Mujer de 56 años a quien se practica quimioterapia por cáncer de mama.
D. Varón de 62 años que fue sometido a reemplazo de cadera cuatro semanas
antes.
E. Varón de 72 años que dos semanas antes había tenido un infarto agudo del
miocardio
¿Cuál de los planteamientos siguientes sobre la embolia pulmonar es
verdadero?
A. La resistencia de vías respiratorias por lo común disminuye en casos de embolia
pulmonar aguda.
B. Prácticamente todos los pacientes con embolias pulmonares muestran signos de
trombosis de vena profunda en esa ocasión.
C. La hiperventilación alveolar es una anormalidad funcional típica en presencia de
embolia pulmonar.
D. La hipertensión en el marco de la embolia pulmonar aguda suele provenir de
disfunción sistólica aguda de ventrículo izquierdo.
E. La magnitud del gradiente arterial/alveolar guarda relación invariablemente con el
diámetro del émbolo pulmonar.
Ante un niño de 7 meses con fiebre e irritabilidad, fontanela abombada y un estudio
de líquido cefalorraquídeo con 110 células/ mm 3(75% linfocitos), proteínas 120
mg/dl y glucosa 28 mg/dl (glucemia sérica 89 mg/ dl), ¿cuál es la sospecha
diagnóstica más razonable?
a. Síndrome mononucleósico.
b. Meningitis bacteriana.
c. Meningitis vírica.
d. Meningitis tuberculosa.
Página 416-468

Cuál de los siguientes es un signo característico de las vías respiratorias de las personas
que sufren asma: Seleccione una:
a. Inltración basóla
b. Adhesión de los glóbulos rojos al endotelio
c. Transformación de los basólos en macrófagos
d. Inltración eosinofílica

¿Cuál es la siopatología de la hemoptisis? Seleccione una:


a. Hipervascularización de la circulación pulmonar, hipertensión pulmonar y remodelación.
b. Hipervascularización de la circulación brónquica, hipertensión pulmonar y
neovascularización
c. Hipervascularización de la circulación pulmonar, ebre y regeneración alveolar
d. Hipervascularización de la circulación brónquica, hipertensión pulmonar y disminución de
coagulabilidad

El virus de hepatitis B, puede ser causa de algunas enfermedades como agente biológico.
¿Cuál de estas enfermedades? Seleccione una:
a. Anemia por décit de hierro
b. Hipoplasia medular
c. Anemia megaloblástica
d. Anemia de proceso crónico

La causa principal de un fracaso en el tratamiento de la tuberculosis pulmonar es


Seleccione una:
a. Resistencia primaria a las drogas
b. Abandono del tratamiento por el paciente
c. Resistencia secundaria a las drogas
d. Toxicidad hepática

¿A qué género pertenece el virus del dengue? Seleccione una:


a. Poxvirus
b. Flaviviridae
c. Hantavirus
d. Coronavirus

¿Cómo se denomina el vector transmisor de fiebre amarilla? Seleccione una:


a. Anopheles
b. Culecoides
c. Lutzomya
d. Aedes aegypti
¿Cuál de las siguientes alternativas es una de las medidas para evitar la diseminación del
Dengue entre la población? Seleccione una:
a. Profilaxis con Cloroquina – Primaquina
b. Administración de la vacuna a poblaciones susceptibles.
c. Control vectorial
d. Introducción experimental de mosquitos transgénicos

Cuál de los siguientes enunciados corresponde a la denición de neumonía nosocomial


tardía Seleccione una:
a. Aparece después de 14 días del ingreso
b. Aparece después de 21 días del ingreso
c. Aparece después de 10 días del ingreso
d. Aparece después de 7 días del ingreso

¿Cuántos días abarca el período de incubación de la leptospirosis? Seleccione una:


a. De 10 a 20 días (aproximadamente 2 semanas)
b. De 1 a 15 días (aproximadamente 1 semana)
c. De 1 a 30 días (aproximadamente 2 semanas)
d. De 1 a 60 días (aproximadamente 3 semanas)

¿De qué depende el mayor riesgo de ENFERMARSE de Tuberculosis? Seleccione una:


a. Inmunidad celular, CD4
b. Estilo de vida
c. Prevalencia de Tuberculosis
d. Genètica

Forma de transmisión del virus de la hepatitis A Seleccione una:


a. Por animales
b. vía respiratoria
c. sexual
d. vía fecal- oral

La enfermedad pulmonar obstructiva crónica no tratada puede conducir a la siguiente


complicación cardíaca: Seleccione una:
a. Pericarditis
b. Bradicardia
c. Insuficiencia cardíaca derecha
d. Síndrome coronario agudo

Los promastigotes son la forma de leishmania que se inyectan por medio de la probóscide
del ebótomo de sexo femenino en la piel del hospedador, ¿qué células son las encargadas
de fagocitar a estas formas parasitarias? Seleccione una:
a. Neutrófilos
b. Eosinófilos
c. Monocitos
d. Macrófagos
Paciente masculino de 55 años de edad, sin antecedentes patológicos familiares de
relevancia. Acude por presentar desde hace 6 días dolor intenso y alodinia a nivel de región
lumbar derecha. Hoy en la mañana nota erupciones muy pruriginosas en la piel.
Antecedentes patológicos personales: varicela en su niñez. Al examen físico presenta
vesículas agrupadas sobre una placa eritematosa elevada que siguen un mismo
dermatoma. También se observa un desprendimiento de costras dejando zonas expuestas
de piel. ¿Cuál sería el diagnóstico probable y su terapéutica? Seleccione una:

-
a. Herpes Zoster y su tratamiento más adecuado es Aciclovir 800 mg cada 4 horas durante
7 días.
b. Infección por Herpes Zoster el mismo que necesitará tratamiento inmediato con
Fluconazol 250 mg VO cada día por 5 días.
c. Dermatitis de contacto y deberá averiguarse sobre factores de predisposición alérgica en
el paciente.
d. Herpes Zoster cuyo tratamiento ideal es Aciclovir, valaciclovir o famciclovir formando
parte estos fármacos de la familia de los inhibidores de la transcriptasa inversa

Recibe el reporte de una muestra de líquido pleural con los siguientes resultados: exudado
amarillento, recuento del 90% de linfocitos. Pertenece a un paciente que presenta ebre y
baja de peso, ¿en cuál de los siguientes diagnósticos usted pensaría? Seleccione una:
a. Empiema
b. Enfermedad del colágeno
c. Tuberculosis
d. Neoplasia

Señale las características del síndrome atípico de la Neumonía Adquirida en la Comunidad.


Seleccione una:
a. Clínica aguda.
b. Más evidente en personas ancianas
c. Se caracteriza por fiebre elevada, escalofríos, tos productiva y dolor pleurítico
d. Disociación clínico radiológica

Su paciente proviene de la amazonia del Ecuador y presenta aproximadamente 15 días con


ebre alta con escalofríos, pero sin pérdida de peso considerable, solo con ligera palidez, al
examen físico se parecía muy levemente una hepato espleno megalia, y Biometría sin
cambios aparente, sería prudente realizar lo siguiente a su criterio: Seleccione una:
a. Gota gruesa y extendido seriado
b. Ecografía y determinar si existe ascitis y derrame pleural (Extravasación)
c. Biopsia de Hígado y descartar procesos de ebre de larga evolución (FOD clásico)
d. Hemocultivo y esperar resultados

Uno de los criterios para considerar la probabilidad de muerte cerebral y proceder a realizar
el test de apnea es: Seleccione una:
a. temperatura corporal de 36 grados centígrados
b. TC cerebral con una lesión estructural cerebral compatible con muerte cerebral
c. pupilas mióticas de 2 mm débilmente reactivas a la luz
d. postura de descerebración luego de una reanimación cardiopulmonar de 20 minutos
Un paciente de 35 años, consulta por cuadro de 4 semanas de evolución, de tos, con
expectoración mucopurulenta, con estrías de sangre en algunas ocasiones, asociado a
compromiso del estado general, baja de peso, ebre intermitente y sudoración nocturna. Ha
tomado amoxicilina en varias oportunidades, sin respuesta. Al examen físico destacan
crépitos escasos, mayores en el ápice derecho. El diagnóstico más probable es: Seleccione
una:
a. Absceso pulmonar
b. Tuberculosis
c. Cáncer pulmonar
d. Bronquiectasias

¿Cuál es la clasificación anatómica de las bronquiectasias? Seleccione una:


a. Lobares, arteriales, globulares
b. Cilíndricas, varicosas, saculares
c. Vasculares, semilunares saculares
d. Cúbicas, cilíndricas, alargadas

Una de las características principales por las cuales el dengue Seleccione una:
a. Es sumamente frecuente la progresión de daño encefálico.
b. Una característica precoz es la trombocitosis y neutrofilia.
c. Debido a la gran capacidad de diseminación del vector Anopheles.
d. El vector es muy cercano a los asentamientos humanos.

La ictericia en un paciente con Malaria es frecuente a partir: Seleccione una:


a. La ictericia no aparece en la malaria
b. Ocasionalmente en niños debido a su alta susceptibilidad
c. Siempre existirá fiebre e ictericia debido a la destrucción de eritrocitos.
d. La Ictericia en Malaria es frecuente a partir de los 15 años.

574 - 626

Se valora a una mujer de 19 años en una clínica universitaria. Ella señala hinchazón no
dolorosa de la extremidad pélvica. En la exploración se advierte que la pierna muestra
engrosamiento de la piel y una contextura dura. La ecografía directa confirma que las venas
de la extremidad pélvica tienen buena circulación y no muestran trombos. Se sospecha la
presencia de linfedema y piensa que posiblemente proviene de la causa más común de este
trastorno secundario a nivel mundial. ¿Cuál es la causa que se sospecha?

A. Cáncer que abarca los ganglios inguinales.

B. Linfogranuloma venéreo.

C. Filariasis linfática.

D. Linfangitis bacteriana recurrente.

E. Tuberculosis
Enunciado de la pregunta Un varón de 19 años consulta por un cuadro de febrícula,
mialgias y odinofagia, acompañado de máculas cutáneas generalizadas no pruriginosas
durante los últimos 7 días. En el interrogatorio refiere, como único suceso médico, haber
presentado una úlcera anal que le causaba dolor a la defecación en el mes anterior que
desapareció progresivamente sin ninguna intervención. ¿Cuál es el germen causante de
este cuadro clínico? Seleccione una:

a. Virus de herpes simple tipo 2.

b. Chlamydia trachomatis.

c. Gonococo.

d. Treponema pallidum.

Retroalimentación La respuesta correcta es: Treponema pallidum

Enunciado de la pregunta ¿Qué es el dengue? Seleccione una:

a. Una zoonosis que afecta al hombre ocasionalmente.

b. Una enfermedad vírica que ocasiona un eritema que evoluciona a mácula y pápula
afectando fundamentalmente a la población infantil.

c. Una enfermedad causada por un poxvirus.

d. Una enfermedad vírica que puede producir una fiebre hemorrágica.

Retroalimentación La respuesta correcta es: Una enfermedad vírica que puede producir una
fiebre hemorrágica

Usted atiende a la persona que muestra linfedema crónico causada por linfangitis
estreptocócica recurrente desde niña. Para ella es antiestética la hinchazón de sus piernas
y solicita que le planteen opciones terapéuticas. Presentamos varias de ellas que son
razonables en caso de linfedema crónico, EXCEPTO:

A. Fisioterapia para descongestión.

B. Administración de diuréticos.

C. Elevación frecuente de las piernas.

D. Dispositivos de compresión neumática intermitente.

E. Liposucción.

Un varón de 19 años consulta por un cuadro de febrícula, mialgias y odinofagia,


acompañado de máculas cutáneas generalizadas no pruriginosas durante los últimos 7
días. En el interrogatorio refiere, como único suceso médico, haber presentado una úlcera
anal que le causaba dolor a la defecación en el mes anterior que desapareció
progresivamente sin ninguna intervención. ¿Cuál es el germen causante de este cuadro
clínico? Seleccione una:

a. Virus de herpes simple tipo 2.

b. Chlamydia trachomatis.

c. Gonococo.

d. Treponema pallidum.

Si durante el examen físico en el área genital encuentra una úlcera lisa no purulenta bien
delimitado única superficial con linfadenopatía bilateral usted piensa en: Seleccione una:

a. Leishmania

-
b. sífilis

c. cancroide

d. herpes tipo 2

¿Cuál de los siguientes componentes de la estructura de los virus gripales es el principal


responsable de su infecciosidad? Seleccione una:

a. Hemaglutinina.

b. ARN polimerasa.

c. Neuraminidasa.

d. Envoltura lipídica.

Retroalimentación La respuesta correcta es: Hemaglutinina

Lesión de Ghon es: Seleccione una:

a. linfadenopatia de cuello

b. pequeño nódulo calcificado

c. Ganglio de ápice de pulmón

d. Diseminación miliar de tuberculosis.

Retroalimentación La respuesta correcta es: pequeño nódulo calcificado

El interferón es un medicamento útil para las siguientes infecciones excepto:

Seleccione una:

-
a. VIH
b. Sarcoma de Kaposi

c. HPV

d. Hepatitis B

La tuberculosis osteoarticular ataca principalmente a columna a lo que se denomina:


Seleccione una:

a. Enfermedad de Wegener

b. Osteofitosis

c. Mal de Pott

d. Trastorno de Jacob

Cuál es el grupo sanguíneo y con mayor susceptibilidad al contagio de cólera:

Seleccione una:

a. Grupo AB

b. Grupo A

c. Ninguno

-
d. Grupo O

Retroalimentación La respuesta correcta es: Grupo O

Paciente diabética de 65 años de edad que consulta por un cuadro de dolor en la cara,
fiebre y aparición de una lesión inflamatoria bien delimitada que diagnosticamos de
erisipela. ¿Cuál es la etiología del proceso?

Seleccione una:

a. Staphylococcus aureus.

b. Staphylococcus epidermidis.

c. Streptococcus agalactiae.

d. Streptococcus pyogenes.

Retroalimentación La respuesta correcta es: Streptococcus pyogenes.

Artritis reactiva es una complicación de:

Seleccione una:

a. Salmonelosis
b. Entamoeba histolytica

-
c. Shigella

d. Giardiasis Retroalimentación

La respuesta correcta es: Shigella

Enunciado de la pregunta

Con respecto a la infección por gripe indiqué la premisa verdadera:

Seleccione una:

a. casi siempre se asocia con febrícula, o sin fiebre sino con tos seca

b. la Clínica generalmente se asocia a trastornos respiratorios altos

c. puede causar una neumonía viral primaria

d. El virus solamente tiene mutaciones puntuales

Retroalimentación La respuesta correcta es: puede causar una neumonía viral primaria

Enunciado de la pregunta

Que caracteriza a una primoinfección por herpes tipo 1:

Seleccione una:

a. trastornos de córnea y conjuntiva

b. faringitis o gingivoestomatitis

c. distribución en dermatoma en especial tórax

d. pápulas y pústulas intersticio entre piel y mucosa

Retroalimentación La respuesta correcta es: faringitis o gingivoestomatitis

En relación al tratamiento de la sífilis, ¿cuál de las siguientes es la respuesta correcta?


Seleccione una:

a. La penicilina es el tratamiento de elección en cualquier estadio.

b. La penicilina es el tratamiento de elección sólo en la neurosífilis.

c. No hay ningún tratamiento efectivo.

d. El tratamiento de elección son las tetraciclinas.


Retroalimentación La respuesta correcta es: La penicilina es el tratamiento de elección en
cualquier estadio.

Ante un niño de 7 meses con fiebre e irritabilidad, fontanela abombada y un estudio de


líquido cefalorraquídeo con 110 células/ mm 3(75% linfocitos), proteínas 120 mg/dl y
glucosa 28 mg/dl (glucemia sérica 89 mg/ dl), ¿cuál es la sospecha diagnóstica más
razonable?

Seleccione una:

a. Síndrome mononucleósico.

b. Meningitis bacteriana.

c. Meningitis vírica.

d. Meningitis tuberculosa.

Enunciado de la pregunta

Mujer de 47 años de edad, nacida y residente en Orellana A mediados del año 2017
comienza con palpitaciones, disnea y edemas progresivos en extremidades inferiores. Es
diagnosticada de insuficiencia cardíaca secundaria a miocardiopatía por Chagas. ¿Cuál es
el agente etiológico de esta enfermedad endémica?

Seleccione una:

a. Trypanosoma brucei.

b. Trypanosoma cruzi.

c. Leishmania donovani.

d. Giardia lamblia.

Retroalimentación La respuesta correcta es: Trypanosoma cruzi.

Enunciado de la pregunta

Su paciente presenta las siguientes características fiebre alta prolongada dolor abdominal
diarrea y estreñimiento ocasionales esplenomegalia y roséola usted sospecha de:

Seleccione una:

a. Giardiasis

b. Shigella

c. Salmonelosis

d. Entamoeba histolytica
Retroalimentación La respuesta correcta es: Salmonelosis

Enunciado de la pregunta

En el análisis de líquido cefalorraquídeo usted encuentra linfocitos aumentados, glucosa


disminuida, usted podría pensar en que la etiología es:

Seleccione una:

a. bacteriana

b. parasitaria

c. Micobacterias

d. viral

Retroalimentación La respuesta correcta es: Micobacterias

Enunciado de la pregunta

Erisipela una infección de tejido blando es debido principalmente a:

Seleccione una:

a. S. aureus

b. Enterobacterias

c. Pseudomona aeruginosa

d. S. pyogenes

Retroalimentación La respuesta correcta es: S. pyogenes

Enunciado de la pregunta

En la malaria, la forma de plasmodium transmitida del mosquito al hombre es el:

Seleccione una:

a. Esporozoito.

b. Hipnozoito.

c. Gametocito.

d. Merozoito.

Retroalimentación La respuesta correcta es: Esporozoito.

A qué grupo poblacional no restringiría la vacuna de la gripe:


Seleccione una:

a. Niños con antecedentes de síndrome de Reye

b. Personas con trastornos crónicos metabólicos

c. Personas con antecedente de Guillain Barré

d. Personas alérgicas a las proteínas del huevo

Retroalimentación La respuesta correcta es: Personas alérgicas a las proteínas del huevo

Enunciado de la pregunta

La infección de herpes en recién nacidos es principalmente dado por:

Seleccione una:

a. VHS 1

b. HVZ

c. VHH 8

-
d. VHS 2

Retroalimentación La respuesta correcta es: VHS 2

Cuál consideraría como primera opción para el diagnóstico de fiebre tifoidea ( fiebre
enterica): Seleccione una:

a. hemocultivo

b. rosa de bengala

c. biopsia intestinal

d. Reacción de Widal

Retroalimentación La respuesta correcta es: hemocultivo

En caso que se enfrente a una situación de tétanos una medida de soporte importante para
el control de espasmos es:

Seleccione una:

a. Diazepam

b. Antitoxina

c. Metronidazol
d. Gabapentina

Retroalimentación La respuesta correcta es: Diazepam

Joven de 23 años, que trabaja como DJ en un bar nocturno y tiene una historia de
promiscuidad bisexual e ingesta de drogas sintéticas, consulta por fiebre, malestar general y
dolor en glande. La exploración física muestra múltiples tatuajes, piercings y la presencia de
tres lesiones vesiculosas en glande y adenopatías inguinales bilaterales. ¿Cuál es el
diagnóstico más probable?

Seleccione una:

a. Condilomas acuminados.

b. Infección por citomegalovirus.

c. Primoinfección VIH.

d. Herpes genital.

Retroalimentación La respuesta correcta es: Herpes genital.

Enunciado de la pregunta

Un trabajador en el oleoducto en el Coca de 36 años de edad, sufre desde hace dos meses
un cuadro abdominal intermitente de náuseas, heces pastosas, flatulencia, meteorismo y
que le han llevado a perder tres kg de peso. ¿Cuál de los siguientes microorganismos sería
con mayor probabilidad el responsable del cuadro?

Seleccione una:

a. Trichomonas hominis.

b. Giardia lamblia

c. Entamoeba coli.

d. Vibrio cholerae.

Retroalimentación La respuesta correcta es: Giardia lamblia

Para el tratamiento de Leishmania cutánea de primera línea se utiliza:

Seleccione una:

a. Fluconazol

b. Antimonio pentavalente

c. Dapsona
d. Anfotericina B

Retroalimentación La respuesta correcta es: Antimonio pentavalente

Enunciado de la pregunta

Cuando la infección de la Tripanosoma cruzi ingresa por piel puede haber una reacción
inflamatoria en el sitio de la infección que dura hasta ocho semanas denominado:

Seleccione una:

a. Complejo oftalmo ganglionar del Chagas.

-
b. Chagoma

c. Signo de Romaña - Mazza.

d. Reacción de Fernández.

Retroalimentación La respuesta correcta es: Chagoma

Enunciado de la pregunta

Con respecto al tétanos, ¿cuál de las siguientes frases es la correcta?

Seleccione una:

-
a. El tétanos está caracterizado por una rigidez generalizada junto a crisis de espasmos
musculares.

b. La mortalidad del tétanos es todavía en la actualidad, superior al 50%.

c. En el diagnóstico del tétanos, es fundamental haber efectuado cultivos previos al inicio


del tratamiento.

d. El tétanos aparece tras un periodo mínimo de incubación de 14 días.

Retroalimentación La respuesta correcta es: El tétanos está caracterizado por una rigidez
generalizada junto a crisis de espasmos musculares.

Enunciado de la pregunta

¿Cuál de los siguientes microorganismos que infectan el tracto intestinal causa bacteriemia
con mayor frecuencia?

Seleccione una:

a. Campylobacter jejuni.

b. Shigella flexneri

c. Vibrio cholerae.
d. Salmonella

Retroalimentación La respuesta correcta es: Salmonella

Enunciado de la pregunta

Paciente que acude a Urgencias con fiebre intermitente, escalofríos, cefalea, debilidad y
abundante sudoración nocturna de 3 días de evolución. Como antecedente epidemiológico
destaca un viaje a Cayambe hace 3 semanas donde consumieron leche cruda y queso sin
pasteurizar. El modo más rápido de diagnosticar la infección sospechada es:

Seleccione una:

a. Prueba del Rosa de Bengala y toma de hemocultivo.

b. Test para detección de anticuerpos heterófilos (Paul-Bunnell).

c. Tinciones de micobacterias en esputo u orina.

d. Gram directo del líquido cefalorraquídeo.

Retroalimentación La respuesta correcta es: Prueba del Rosa de Bengala y toma de


hemocultivo.

Qué patología de las siguientes es una condición define Sida:

Seleccione una:

a. herpes tipo 1: úlceras crónicas > 1 mes

b. displasia cervical

c. candidiasis oral

d. leucoplasia vellosa

Retroalimentación La respuesta correcta es: herpes tipo 1: úlceras crónicas > 1 mes

Enunciado de la pregunta

Ante los hallazgos que cabe esperar del análisis del líquido cefalorraquídeo en un cuadro
de meningitis bacteriana, NO se encuentra:

Seleccione una:

a. Cultivo positivo en el 40% de los casos

b. Abundantes hematíes.

c. Glucosa < 40 mg/dl.

d. Proteínas > 45 mg/dl.


Retroalimentación La respuesta correcta es: Abundantes hematíes.

Enunciado de la pregunta

En la clasificación de FOD cuál considera usted como urgente y debe ser tratada de
inmediato: Seleccione una:

a. FOD Clásico

b. FOD asociado a HIV

c. FOD neutropénico

d. FOD Nosocomial Retroalimentación

La respuesta correcta es: FOD neutropénico

Enunciado de la pregunta

Tras la infección, ¿qué plasmodios parásitos humanos persisten como hipnozoitos en el


hígado y pueden causar recidivas?

Seleccione una:

a. P. falciparum.

b. P. cynmology y P. knowlesi.

c. P. malariae.

d. P. vivax y P. ovale.

Retroalimentación La respuesta correcta es: P. vivax y P. ovale.

Enunciado de la pregunta

La BCG es una vacuna atenuada derivada del:

Seleccione una:

a. M. tuberculosis

b. M. avium – intracellulare

c. M. leprae

-
d. M. bovis

Retroalimentación La respuesta correcta es: M. bovis

Se describen síntomas y signos más frecuentes en la Enfermedad por Coronavirus


COVID19 entre éstos cuál es el que se presenta con más frecuencia :
a) Tos seca (50,7%)

b) Fiebre (87,9%)

c) Astenia (99,1%)

d) Expectoración (33,4%)

¿Cuál es el receptor al que se une el virus para ingresar a las células pulmonares?

a. AT1

-
b. ECA

c.

d.
AT2

Bradicinina

Cuál es la respuesta a la que puede conducir la hiperactivación de macrofagos y


granulocitos que puede conllevar a la muerte del paciente:

A. Síndrome de liberación de inmunoglobulinas

B. Síndrome de liberación de linfocitos T

C. Síndrome de liberación de citoquinas

D. Síndrome de liberación de factores de crecimiento.

Seleccione la respuesta correcta.

Dentro de las especies del género Plasmodium que pueden causar infecciones palúdicas en
seres humanos se incluyen:

a) Falciparum, Vivax, Ovale,Malariae, Knowlesi.

b) Falciparum, Vivax, Marburg, Ovale

c) DENV-1, DENV-2, DENV-3, DENV-4

d) Cruzi, Gambiense, Rhodesiense

La malaria es una de las enfermedades endémicas en nuestro país. ¿Cuál de estos


enunciados es falsa?:

a) La gravedad de la infección se relaciona con el grado de parasitemia

b) La infección por Plasmodium falciparum es la presentación más grave.

c) La malaria cerebral es una de las manifestaciones típicas por Plasmodium


vivax
1111
d) El dato analítico más frecuente en la malaria es una anemia normocítica

Señale la respuesta correcta en relación a la Malaria:

a) Plasmodium ovale es el causante de la mayoría de las muertes por malaria

b) El examen de gota gruesa o de extendido de sangre periférica no sirven para el


diagnóstico.

c) El Anopheles infectado, al picar, inoculan los hipnozoitos, forma infectante del


parásito

d) Es una enfermedad causada por protozoarios del género Plasmodium

Cual es una característica principal para que el dengue pueda diseminarse?:

A. El vector es muy cercano a los asentamientos humanos.

B. Debido a la gran capacidad de diseminación del vector Anopheles.

C. Una característica precoz es la trombocitosis y neutrofilia.

D. Es sumamente frecuente la progresión de daño encefálico.

Uno de los criterios de DENGUE SEVERO es:

a) Dolor abdominal

b) Tos

c) Fiebre

d) Exantema

¿En cuánto tiempo se puede producir la muerte del paciente por dengue hemorrágico?

a) 7- 8 horas

b) 12 -24 horas

c) 10 - 12 horas

d) 24 - 48 horas

¿A qué familia y género pertenece el virus del sarampión?

a) familia Flaviviridae - género Flavivirus

b) familia Herpesviridae - género Varicellovirus

c) familia Paramyxoviridae - género Morbillivirus


d) familia Togaviridae - género Rubivirus

¿Cuál de los siguientes signos y síntomas son considerados el patognomónico del


sarampión?

a) Fiebre entre 39 y 40 °C

b) Conjuntivitis

c) lesiones de Köplik

d) Coriza y tos

El segundo microorganismo más frecuente causante de neumonía en la comunidad en


población adulta.

a) haemophilus influenzae

b) estafilococo weeks

c) estreptococo pneumoniae

d) mycobacterium tuberculosis

Respuesta: A

Respecto al esquema permanente de vacunación de sarampión y rubéola.

a) la primera dosis debe darse al primer mes de nacimiento.

b) la primera dosis debe darse entre los 6-9 meses de edad.

c) la primera dosis debe darse entre los 12 -23 meses de edad.

d) la primera dosis debe darse después de los 5 años de edad.

1.- Paciente de 25 años, presenta fiebre, tos, malestar general durante una semana, acude
el dia de hoy a consulta ,se le realizaron examen de esputo en el cual se detectó ?: según la
literatura cuál es el germen más común que causa neumonía:

1. Pneumocystis jiroveci

2. virus de la gripe

3. streptococcus pneumoniae

4. Pseudomona aeruginosa

Un paciente de 30 años adicto a drogas por vía parenteral, se presenta con un cuadro de 3
días de evolución, fiebre, dolor torácico y tos con expectoración verdosa con hilos de
sangre; en la radiografía de tórax presenta varios infiltrados pulmonares con cavitación
central en alguno de ellos. El diagnóstico más probable sería:

1. Neumonía neumocócica.
2. Tuberculosis pulmonar.
3. Neumonía por anaerobios.
4. Neumonía hematógena.
5. Neumonía por H. Influenzae.

Dentro de los factores de riesgo de neumonía extrahospitalaria (CAP) escoja la opción


correcta:

1. Alcoholismo, asma, inmunodepresión, hospitalización y tener ≥70 años.

2. Niños <5 años, asma, inmunosuprimidos

3. Tabaquismo, alcoholismo y asma

4. Edad >60 años, asma, uso de corticoides Harrison edicion 19 , capitulo 153, pag
805

¿En la infección tuberculosa cuál es el factor principal para adquirir dicha infección? Indique
la respuesta correcta:

1. Factor endógeno
2. Factor exógeno
3. Factor social
4. Factor ultravioleta

¿Cuál es el porcentaje de pacientes en los cuales se les reactiva la Tuberculosis dentro de


los 2 años posteriores, teniendo la enfermedad activa?

-
A. 50-80%

B. 2-10%

C. 30-50%

D. 99%

Es correcto en cuanto a la patogenia y transmisión de Tuberculosis. Excepto:

A. M. tuberculosis se transmite mediante partículas en suspensión en el aire, o núcleos de


gotitas, que pueden generarse cuando las personas tuberculosas estornudan, tosen o
hablan

B. La infección, que es generalmente asintomática, ocurre cuando una persona susceptible


inhala núcleos de gotitas que contienen M. tuberculosis y los microorganismos llegan a los
alvéolos pulmonares.
C. Una vez en el pulmón, los bacilos son fagocitados por los linfocitos T

D. Puede presentarse forma extrapulmonares como ganglionar, pleural, abdominal, renal,


pericárdica, entre otras

Señale cuál de estos enunciados respecto a la tuberculosis es incorrecto:

A. La transmisión ocurre por la diseminación aérea de núcleos de gotitas producidas por los
pacientes con tuberculosis.

B. El Mycobacterium tuberculosis es el agente causal que mide 0.5-3 um y en su pared


tiene lipoarabinomanano que interviene en la patogenia.

-C. La tuberculosis la enfermedad primaria es también conocida como tuberculosis de


reinfección.

D. Para el diagnóstico se realizan pruebas como baciloscopia,broncoscopia, radiografía de


tórax, prueba de tuberculina, cultivo, PCR.

- Cuál de los siguientes es un signo característico de las vías respiratorias de las


personas que sufren asma:

Seleccione una:

a. Infiltración basófila

b. Transformación de los basófilos en macrófagos

c. Adhesión de los glóbulos rojos al endotelio

d. Infiltración eosinofílica

¿Cuál es la clasificación anatómica de las bronquiectasias?

a. Lobares, arteriales, globulares

b. Cilíndricas, varicosas, saculares

c. Vasculares, semilunares, saculares

d. Cubicas, cilíndricas, alargadas

¿Cuál es la fisiopatología de la hemoptisis?

a. Hipervascularizacion de la circulación bronquica, hipertensión pulmonar y


neovascularización

b. Hipervascularizacion de la circulación pulmonar, fiebre y regeneración alveolar

c. Hipervascularizacion de la circulación pulmonar, hipertensión pulmonar y


remodelación
d. Hipervascularizacion de la circulación bronquica, hipertensión pulmonar y
disminución de coagulabilidad

De los siguientes factores, cuál es el que debe estar alterado para que un paciente
infectado de tuberculosis se convierta en persona enferma:

Seleccione una:

a. Huésped: Inmunidad celular deficiente, en especial CD4

b. Huésped: que genéticamente este predispuesto a infectarse

c. Medio ambiente: que haya elevada prevalencia de tuberculosis

d. Medio ambiente: que viva en un lugar frio

Dentro de las manifestaciones tardías de la sífilis tenemos trastornos cardiovasculares de


cual podemos desprender el siguiente:

a. Trastornos del endotelio

b. Insuficiencia cardiaca congestiva

c. Flebitis irritativa

d. Aneurismas

¿De qué depende la inmunidad contra Leptospira?

Seleccione una:

a. Producción de anticuerpos contra los distintos serotipos.

b. Producción de anticuerpos contra LPS inespecíficos

c. Producción de anticuerpos contra proteínas inespecíficas de un serotipo

d. Producción de anticuerpos circulantes contra LPS específicos de un serotipo

El hallazgo más frecuente en la radiografía de tórax en un paciente con asma es:


Seleccione una:

a. Engrosamiento de paredes bronquiales.

b. Radiografía de tórax normal.

c. Condensaciones alveolares bilaterales y difusas.

d. Hiperinsuflacion pulmonar

El mecanismo por el cual la infección por cólera produce una diarrea tan intensa es por:
Seleccione una:
a. Toxina Shiga-like

b. Vibrios entero hemorrágicos.

c. Vibrios enteroagregativos.

d. Toxina enterogénica

El tratamiento de las Bronquiectasias se basa en 3 pilares, excepto:

Seleccione una:

a. Revertir el remodelamiento bronquia

b. Mejorar la eliminación de las secreciones, que se consigue con una adecuada


hidratación, con fisioterapia respiratoria y drenaje postural mantenidos

c. Controlar las infecciones con el uso de antibióticos en las agudizaciones durante 10 – 15


dias

d. Eliminar la obstrucción bronquial

En cuanto a la profilaxis antitetánica en el tratamiento sistemáticos de las heridas, en una


herida pequeña limpia sin ningún antecedente de vacunación antitetánica usted
recomendaría:

a. Vacunación antitetánica de inmediato

b. Es indicativo de inmunoglobulina

d.
-
c. Solo limpiar la herida y dar indicaciones de cuidado al paciente, ofrecer la anti
toxina

Dar antibiótico de manera profiláctica

En cuanto al diagnóstico funcional del asma:

Seleccione una: a. Si la relación VEF1/CVF es menor

a 0.7 (patrón restrictivo) y post broncodilatador obtenemos un aumento igual o mayor al


12% en el VEF1, nos orienta hacia diagnóstico de asma

b. Si la relación VEF1/CVF es mayor 0.7 se considera patrón obstructivo, diagnosticamos


asma

c. Si la relación VEF1/CVF es menor a 0.7 (patrón obstructivo) y post broncodilatador


obtenemos un aumento igual o mayor al 12% en el VEF1, nos orienta hacia diagnóstico de
asma
1111
d. Valores espirométricos no tienen importancia en el diagnóstico del asma.
La respuesta correcta es: Si la relación VEF1/CVF es menor a 0.7 (patrón obstructivo) y
post broncodilatador obtenemos un aumento igual o mayor al 12% en el VEF1, nos orienta
hacia diagnóstico de asma

En la clasificación de OMS de adultos con VIH – Sida en el estadio C3 tenemos a:

Seleccione una:

a. Mayor de 400 células CD4+ con síntomas no definidores de Sida

b. Mayor de 400 células CD4+ con síntomas definidores de Sida

c. Menos de 300 células CD4+ con síntomas definidores de Sida

d. Mayor de 500 células CD4+ asintomático

La causa principal de un fracaso en el tratamiento de la tuberculosis pulmonar es

Seleccione una:

a. Resistencia secundaria a las drogas

b. Resistencia primaria a las drogas

c. Abandono del tratamiento por el paciente

d. Toxicidad hepática

La respuesta correcta es: Abandono del tratamiento por el paciente

La ictericia en un paciente con Malaria es frecuente a partir:

Seleccione una:

a. La ictericia no aparece en la malaria.

b. Siempre existirá fiebre e ictericia debido a la destrucción de eritrocitos

c. La ictericia en Malaria es frecuente a partir de los 15 años

d. Ocasionalmente en niños debido a su alta susceptibilidad

Tras la infección de un niño con el virus de la Influenza con las siguientes características:
fiebre intensa de inicio súbito, disnea y cianosis a la Rx de tórax se aprecia patrón asociado
con infiltrados intersticiales difusos e hipoxia intensa, usted sospecharía de:

a. Neumonía viral secundaria

b. Neumonía viral primaria

c. Neumonía bacteriana primaria


d. Neumonía bacteriana secundaria

Una de las características principales por las cuales el dengue puede diseminarse es:

Seleccione una:

a. Es sumamente frecuente la progresión de daño encefálico.

b. Una característica precoz es la trombocitosis y neutrofilia

c. Debido a la gran capacidad de diseminación del vector Anopheles

d. El vector es muy cercano a los asentamientos humanos

Qué técnica o espécimen ofrece el rendimiento más alto para el diagnóstico de la


tuberculosis pleural?

Seleccione una:

a. Estudio directo de biopsia con histología de pleura

b. Adenosin deaminasa en lìquido pleural

c. BAAR del lìquido pleural

d. Zhiel Nielsen y cultivo en Lowenstein del esputo

Respuesta incorrecta. La respuesta correcta es: Estudio directo de biopsia con histología de
pleura

Paciente femenino de 71 años con fractura de tobillo, presenta dolor en punta de costado
con una intensidad de 7/10, que se agrava al movimiento y la inspiración, se acompaña de
disnea de aparición súbita, tos y hemoptisis, FC: 114, FR: 26, T: 37.8 ºC. APP:
Adenocarcinoma pulmonar con metástasis óseas en columna lumbar y metástasis cerebral
cosiderado etapa 4. Al valorarlo por escala de Weels usted considera que:

Seleccione una:

a. El paciente se encuentra en un riesgo elevado de presentar tromboembolia pulmonar

b. El paciente se encuentra en un riesgo moderado de presentar tromboembolia pulmonar

c. El paciente se encuentra en un riesgo bajo de presentar tromboembolia pulmonar y no


debe considerarse pruebas de imagen

d. El paciente se encuentra en un riesgo bajo de presentar tromboembolia pulmonar por lo


que se debe considerar otros diagnósticos diferenciales

1. La acumulación de líquido en el espacio pleural, según Sahn se debe a:

Seleccione una:
a. Disminución de la presión negativa del espacio pleural

b. Aumento presión oncótica

c. Disminución presión hidrostática

d. Aumento de permeabilidad capilar

Cuál de los siguientes signos radiográficos no corresponde a un hallazgo de neumotórax:

a. Aumento de la radiolucidez en el área de acumulación de aire

b. Desplazamiento de estructuras del mediastino hacia el lado contrario del


afectado

c. Joroba de Hampton

d. Colapso pulmonar parcial o completo del pulmón del lado afectado

Cuál es la mejor opción al identificar un paciente sintomático respiratorio:

a. Realizar un PPD

b. Iniciar tratamiento contra tuberculosis

c. Pedir una Rx de tórax AP y lateral

d. 2 muestras de BAAR en esputo

En relación a las infecciones del SNC, señale el enunciado falso:

Seleccione una:

a. Una de las complicaciones graves en Meningitis bacteriana aguda puede serla


hipertensión intracraneal la misma que produce disminución del estado de conciencia

b. La otitis, mastoiditis y sinusitis sonentidades predisponentes para sufrir meningitis por


estreptococo, estafilococo o anaerobios gram negativos

c. La frecuencia de meningitis por haemofilus influenza tipo b en niños ha disminuido


gracias a la vacuna contra dicho gérmen

d. Una de las vías de ingresoal SNC por parte de S pneumoniae y N meningitidis es por
contigüidad desde el epitelio nasofaríngeo

En relación a la cisticercosis señale el enunciado FALSO:

Seleccione una:

a. La infección por cisticerco se deriva de la teniasis por Tenia saginata, cuyohospedario es


el ser humano
b. El tratamiento se lo debe hacer con un fármaco cisticida como el praziquantel o el
albendazol

c. La infección puede afectar a cualquier órgano de la economía, a nivel cerebral puede


invadir el parénquima, sistema ventricular o subaracnoideo, etc

d. Lasinfecciones de cisticercosis pueden ser asintomáticas, perotambién presentarse con


crisis convulsivas o no convulsivas, cefalea, hidrocefalia, etc

En relación a la neurocisticercosis, señale el enunciado falso:

Seleccione una:

a. Un individuo que viajó a una zona endémica tendría criterio epidemiológico y sería un
potencial foco de infección

b. La determinación de antígenos o anticuerpos por ELISA se los puede realizar, pero no


constan en los criterios diagnósticos

c. Para el diagnóstico semencionan criterios epidemiológicos: entre los que consta:residir


en un área endémica de cisticercosis

d. Uno de los criterios absolutos es: identificación neuroradiológica de lesiones quísticas


que contengan un escólex en su interior

Pg 624-676
En relación a la neurocisticercosis, señale el enunciado falso:

Seleccione una:

a. Un individuo que viajó a una zona endémica tendría criterio epldemlológlco y serla un potencial foco de
infección

b. La determinación de antígenos o anticuerpos por ELISAse los puede realizar, pero oo constan en los criterios
diagnósticos

c. Para el diagnóstico se mencionan criterios epidemiológicos: entre los que consta : residir en un área endémica de
cisticercosis

d. Uoo de los criterios absolt1.1os es: Identificación neuroradiológica de lesiones quíst icas que contengan un
escólex en su Interior

Respuesta correcta

La respuesta correcta es: La determinación de antígenos o anticuerpos por El ISA se los puede realizar. pero no constan en los
criterios diagnósticos

En relación a las encefalopatías, se~ale el enunciado falso:

Seleccione una:

a. la causa pulmonar expresada como insuficiencia pulmonar o respiratoria se basa en hlpoxemla ehipercapnla

b. El aporte diagnóstico del electroencefalograma se basa en el hallazgo de una lent itud difusa o simétrica que
traduce una disfunción cerebral difusa

c. Dent ro de la et iología o causa hepát ica se menciona al shunt porto-sistémico, así como a la cirrosis

d. Dentro de las causas hidroelectrolíticas se mencionan las hiperpotasemias y las hipermagnesemias

Respuesta correcta

La respuesta correcta es: Dentro de las causas hidroelectrolítlcas se mencionan las hiperpotasemias y las hipermagnesemias

¿A qué género pertenece el virus del dengue?

Seleccione una:

a. Flaviviridae .¡

b. Ha ntavirus

c. Cor onavirus

d. Poxvirus

La r espuesta correcta es: Flaviviridae


¿Cómo se denomina el vector transmisor de fiebre amarilla?

Seleccione una :

a. Anopheles X
b. Aedes aegypti

c. Culecoides

d. Lutzomya

La respuesta correcta es: Aedes a egypti

¿Cuál de las siguientes alternativas es una de las medidas para evitar la disem inación del Dengue entre la
población?

Seleccione una :
a. Introducción experimental de mosquitos transgénicos

b. Administración de la vacuna a poblaciones susceptibles.

c. Profilaxis con Cloroquina - Primaquina

d. Control vectorial .¡

La respuesta correcta es: Control vectorial

Cuál d e los siguientes enunciados corresponde a la definición de neu monía nosocomial tardía

Seleccione una :
a. Aparece después de 21 días del ingreso

b. Aparece después de 10 días del ingreso

c. Aparece después de 7 días del i ngreso .¡

d. Aparece después de 14 días del ingreso

La respuesta correcta es: Aparece después de 7 días del ingreso


¿Cuántos días abarca el período de incubación de la leptospirosis?

Seleccione una:

a. De 1O a 20 días (aproximadamente 2 semanas)

b. De 1 a 60 días [ap roximadamente 3 semanas)

c. De 1 a 30 días (aproximadamente 2 semanas) ..¡

d. De 1 a 15 días [ap roximadamente 1 semana)

La r espuesta correcta es: De 1 a 30 días (aproximadamernte 2 semanas)

De las encefalitis v irales seleccione la que se beneficia del tratamiento con Aciclovir:

Seleccione una:

a. Encefalitis equina

b. Citomega lovirus

c. Herpes virus ..¡

d. Enterovirus

La r espuesta correcta es: Her pes vir us

¿De qué depende el m ayor riesgo de ENFERMARSE de Tu berculosis?

Seleccione una:

a. Prevalencia de Tuberculosis

b. Estilo de vida

c. Genetica

d. Inmunidad celular, CD4 ..¡

La r espuesta correcta es: Inmunidad celular, CD4


Forma de t ransmisión del virus de la hepatitis A

Seleccione una :

a. vía fecal- oral ...¡

b. sexual

c. vía respiratoria

d. Por animales

La r espuesta correcta es: vía fecal- oral

La causa más frecuente de hemoptisis leve-moderada es:

Seleccione una :

a. Bronquiectasias. .¡

b. Infarto pu lmonar.

c. Ca rcinoma broncogénico.

d. Diátesis hemorrágica.

La r espuesta correcta es: Bronquiectasias.

Los promastigotes son la forma de leishmania que se inyectan por medio de la probóscide del flebótomo de
sexo femenino en la piel del hospedador, ¿qué células son las encargadas de fagocitar a estas formas
parasitarias?

Seleccione una :

a. Neutrófilos .¡

b. Macrófagos

c. Eosinófilos

d. Monocitos

La r espuesta correcta es: Neutrófilos


Paciente acude con t rastornos osteomusculares crónicos, junto con déficit visual inespecífico, pérdida de
peso y fiebre de larga evaluación, el paciente proviene de una zona meramente ganadera, Usted sospecha
de brucelosis cuya base de tratamiento es:

Seleccione una:

a. Penicilina y Met ronidazol

b. Dicloxacilina y Est reptomicina .¡

c. Gentamicina y Dapsona

d. lsoniacida y Estreptomicina

La r espuesta correcta es: Dicloxacili na y Estreptomicina

Paciente de 25 años que refiere viaj e reciente a Filipinas presenta: fiebre, dolor abdominal, diarrea,
bad icard ia relativa. ¿Cuá l es el diagnóstico más probable?

Seleccione una:

a. Fiebre tifoidea .¡

b. Paludismo

c. Gastroenteritis complicada

d. Micosos sistémica

La r espuesta correcta es: Fiebre tifoidea


Paciente m asculino de 55 años de edad, sin antecedentes patológicos familiares de relevancia. Acude por
presentar desde hace 6 días dolor intenso y alodinia a nivel de región lumba r derecha. Hoy en la ma ñana
nota erupciones muy pruriginosas en la piel. Antecedentes patológicos persona les: va ricela en su niñez. Al
exam en físico presenta vesículas agrupadas sobre una placa eritem atosa elevada que siguen un mismo
derm atom a. También se observa un desprend im iento de costras dejando zonas expuestas de piel. ¿Cuál
sería el diagnóstico proba ble y su terapéutica?

Seleccione una:

a. Herpes Zoster cuyo tratamiento ideal es Aciclovir, valaciclovir o famciclovir formando parte estos
fá rmacos de la familia de los inhibidores de la t ranscriptasa inversa.

b. Herpes Zoster y su tratamiento más adecuado es Aciclovir 800 mg cada 4 horas durante 7 días.

c. Infección por Herpes Zoster el mismo que necesitará tratamiento inmediato con Fluconazol 250 mg VO
cada día por 5 días.

d. Dermatit is de contacto y deberá averiguarse sobre factores de predisposición alérgica en el paciente.

La r espuesta correcta es: Herpes Zoster y su tratamiento más adecuado es Aciclovir 800 mg cada 4 horas durante
7 días.

Seña'l e las caracterísbicas del síndrome atípico de la Nleumonía Adqu irida en la Comu nidad.

Seleccione una:

a. Clínica aguda.

lb. Se caracteriza por fiebre elevada, escalofríos, tos productiva y dolor pleurítico X
c. Disociación clínico radiológica

d. Más evidente en personas ancianas

La r espuesta correcta es: Disociación clínico rad iológica


Un paciente de 35 años, consulta por cuadro de 4 sema nas de evolución, de tos, con expectoración
mucopurulenta, con estrías de sangre en algunas ocasiones, asociado a com promiso del estado general,
baj a de peso, fiebre interm itente y sudoración nocturna. Ha tomado amoxicilina en varias oportunidades,
sin respuesta. Al examen físico destacan crépitos escasos, mayores en el ápice derecho. El diagnóstico más
probable es:

Seleccione una:
a. Bronquiectasias

b. Tuberculosis .,¡

c. Cáncer pulmonar

d. Absceso pulmonar

La respuesta correcta es: Tube rculosis

Es la mejor prueba com plementaria para el diagnóstico de asma:

Seleccione una:

Seleccione una:

a. Determinación de inmunoglobulinas

b. Espirometría .,¡

c. Lavado bronquial

d. Broncoscopia

Respuesta correcta

La respuesta correcta es: Espirometría


Las tres alteraciones funcionales básicas del asma son:

Seleccione una:

Seleccione una :

a. Hipertensión arterial, hiperreactividad bronquial y obstrucción nasal

b. Obstrucción al flujo de aire, obstrucción al fluj o venoso e irreversibilidad de la vía aérea

c. Tos, obstrucció n fluj o aéreo y somnolencia

d. Obstrucción al flujo de aire, reversibilidad de la vía aérea e hiperreactividad bronquial .¡

Respuesta correcta

La r espuesta correcta es: Obstrucción al fluj o de aire, r ev ersibilidad de la vía aérea e hip er reactividad bronquial

En la exploraoión funcional de un paciente con enfermedad pulmonar obstructiva crónica, son esperables
todos los hallazgos MENOS uno:

Seleccione una:

Seleccione una :

a. Volúmenes pulmonares disminuidos .¡

b. FEV1 menor del 80%.

c. Prueba broncodilatadora negativa

d. Cociente FEV1/ FVC inferior al 0.7

Respuesta correcta

La r espuesta correcta es: Volúmenes pulmonares disminuidos


Cuáles son los 3 síntomas o signos más frecuentes de la EPOC

Seleccione una:

Seleccione una :
a. Tos crónica, disnea crónica y fiebre

b. Tos crónica, disnea crónica y expectoración mucosa ..¡

c. Tos crónica, disnea crónica y utilización de músculos accesorios

d. Tos crónica, disnea crónica y sibilancias

Respuesta correcta

La respuesta correcta es: Tos crónica, d isnea crónica y expectoración mucosa

Masculino de 65 años de edad se presenta con una historia de disnea progresiva y tos no productiva de 3
días de evolución. Había sido hospitalizado previamente hace 2 años por insuficiencia cardiaca congest iva.
Sus signos vitales: TA 90/55 mmHg, frecuenci a cardiaca de 11 O latidos por minuto, temperatura d e 37.6C, y
saturación de oxígeno al aire ambiente es de 86%. A la auscultación de tórax, se detectan estertores y
roncus bilaterales. La radiografía de tórax revela infiltrados bilaterales y un aumento discreto del tamaño de
la silueta cardiaca. ¿Cuál es el diagnóstico clínico más probable?
Seleccione una:

Seleccione una :

a. Ang ina estable

b. Disección de Aorta

c. Tromboembolia pulmonar

d. Edema agudo de pulmón ..¡

Respuesta correcta

La respuesta correcta es: Edem a agudo de pulmón


Con respeao a la EPOC, NO es cierto:

Seleccione una:

Seleccione una:

a. Espiromét ricamente, se detecta obstrucción por un cociente FEV1/FVC inferior a 0,70

b. El hábito tabáquico es el factor más importante para desarrollar EPOC

c. El factor genético para desarrollar EPOC mejor documentado es el déficit de alfa1 - antitripsina

d. Contrariamente al asma, no hay componente inflamatorio .¡

Respuesta correcta

La respuesta correcta es: Contrariamente al asma, no hay componente inflamatorio

Cuál de las siguientes anomalías electrocardiográficas sue le asociarse a tromboembolia pulmona r?

Seleccione u na:

Seleccione una:

a. Inversión de ondas T en la derivación de ondas laterales 1, AVL y VS- V6

b. Ondas Q en las derivaciones a nteriores de V1 - V4

c. 51Q3T3, ondas T negativas en las derivaciones anterio res de V1 a V4 .¡

d. Bloqueo A-V de primer grado

Respuesta correcta

La r espuesta correcta es: 51 Q3T3, ondas T negativas en las der ivaciones anteriores de V1 a V4
Actualmente se utilizan 3 clases de broncodilatadores en pacientes con asma, señale lo correcto:

Seleccione una:

Seleccione una:
a. Agon istas J32, anticolinérgicos, teofilina .¡

b. Agonistas J32, corticoesteroides inhalados, corticoesteroides sistémicos

c. Antileucotrienos, teofilina, agonistas J32

d. Anticoliné rgicos, antileucotrienos, agonistas J32

Respuesta correct a

La respuesta correcta es: Agonistas J32, anticolinérgicos, teofil ina

La decisión del ingreso hospitalario de un/a paciente con NAC depende de la capacidad del médico para
predecir la probabilidad de muerte. Según el índice CURB65, ¿cuándo se recomienda la hospitalización?

Seleccione una:

Seleccione una:

a. Cuando la puntuación sea de 1 puntos.

b. Cuando la puntuación sea de 1,5 puntos.

c. Cuando la puntuación sea de O puntos.

d. Cuando la puntuación sea > de 2 puntos . .¡

Respuesta correcta

La respuesta correcta es: Cuando la puntuación sea > de 2 puntos.


Criterios para derrame pleura l exudativo, excepto:

Seleccione una:

Seleccione una:

a. LDH del líquido pleural >66'11> del límite superior normal para el suero

b. LDH del líquido pleural/LDH sérica <0.4 ../

c. Proteínas de líquido pleural/proteínas séricas >0.5

d. LOH del líquido pleural/LOH sérica >0.6

Respuesta correcta

La respuesta correcta es: LOH del líquido pleural/LOH sérica <0.4

Señale la respuesta incorrecta con relación a la tub erculosis:

Seleccione una:

Seleccione una:

a. La enfermedad tuberculosa es aquella situación en la que el in dividuo presenta datos clínicos.

b. La primoinfección tuberculosa es aquella situación en la que el individuo entra en contacto con el


germen y desarrolla por primera vez la enfermedad . ..¡
c. La tuberculosis posprimaria es aquella situación en la que el individuo presenta una reactivación de la
enfermedad tuberculosa.

d. La infección tuberculosa es aquella situaciórn en la que el individuo entra en contacto con el germen

Respuesta correcta

La r espuesta correcta es: La primoinfección tuberculosa es aquella situación en la que el ind ividuo entra en
contacto con el germen y desarrolla por primera vez la enfermedad.
Cuál es el cuadro clínico característ ico de los pacientes con Bronquit is crón ica?

Seleccione una:

Seleccione una:

a. llamados tosedores rosados, hemoptisis sin tos productiva


b. llamados tosedores rosados, con espiración prolongada y aumento de peso

c. llamados sopladores azules, aumento de peso y coloración normal de la p iel

d. llamados sopladores azules, cianóticos y caquécticos .¡

Respuesta conrecta

La r espuesta correcta es: llamados sopladores azules, cianóticos y caquécticos

Cuáles son los principales factores de r iesgo para el desarrollo de NAC?

Seleccione una:

Seleccione una:

a. Alcoholismo, parejas sexuales múltiples, uso de anticonceptivos orales, antecedentes de cirugía


cardiaca o pulmonar.

b. Extremos de edad (ancianos y niños), tabaquismo, alcoholismo, diabetes, asma, insuficiencia cardíaca,
EPOC, cáncer, inmunodeprimidos. .¡

c. Múlt ip les pareja sexuales, EPOC, tabaqu ismo , lnmunodeprimidos, adultos mayores sanos, y personas
adultas j óvenes.

d. Enfermedad de Alzheimer, fibrosis quística, pacientes entre 30 y 50 años de edad, uso de


ant iconceptivos orales, antecedentes de cirugía cardiaca o pulmonar ..

Respuesta correcta

La r espuesta correcta es: Extremos de edad (ancianos: y niños), tabaquismo, alcoholismo, diabetes, asma,
insuficiencia ca rdíaca, EPOC, cá ncer, inmunodeprimidos.
Hombre de 68 años con antecedente de neoplasia de páncreas en curso de quimioterapia. Consulta en
Emergencias por dolor y edema de todo el miembro in ferior desde ingle. ¿Qué prueba d iagnóstica es más
coste-efectiva para confirmar la sospecha diagnóstica?

Seleccione una:

Seleccione una:
a. Ecografía doppler venosa .¡

b. Tomografía Axial Computarizada

c. Resonancia Magnética

d. Dímero D

Respuesta correcta

La respuesta correcta es: Ecografía doppler venosa

Señale el germen que no produce la tubercu losis pul monar:

Seleccione una:

Seleccione una:

a. Mycobacterium tuberculae

b. Mycobacterium a fricanum.

c. Mycobacterium bovis.

d. Mycobacterium leprae. .¡

Respuesta correcta

La respuesta correcta es: Mycobacterium leprae.


Muj er de 54 años es encontrada desorientada por unos transeúntes. Al llegar el equipo de emergencias la
encontraron con una saturación de oxígeno del 80% respirando aire ambiente y al examen físico pupilas
puntiformes. Se la traslada a la emergencia del hospital, donde la gasometría arterial basal muestra: pH
7,25, PaC02 60 mmHg, Pa02 56 mmHg, bicarbonato de 26 mEq/ I y exceso de bases de -1. En sangre el sodio
es 136 mEq/1 y el cloruro 100 m Eq/1. Desde el punto de vista gasométrico la paciente t iene: Seleccione una:

Seleccione una:

a. La gasometría solo puede ser de sangre venosa

b. Acidosis metabólica.

c. Acidosis respiratoria pura .¡


d. Insuficiencia respiratoria parcial

Respuesta correcta

La r espuesta correcta es: Acidosis respiratoria pura

En la gasometría rea lizada en un paciente con Insuficiencia Respiratoria Aguda esperaríamos encontrar:

Selecc ione una:

Seleccione una:

a. PaO2 menor de 60 mmHg-PaCO2 menor de 45 mmHg

b. PaO2 menor de 60 mmHg-PaCO2 mayor de 45 mmHg .¡

c. PaO2 mayor de 60 mmHg-PaCO2 menor de 45 m mHg

d. PaO2 mayor de 60 mmHg-PaCO2 mayor de 45 mmHg

Respuesta correcta

La r e,spuesta correcta es: Pa02 menor de 60 mmHg-PaC02 mayor de 45 mmHg


Cuál de las siguientes premisas es FALSA respecto del diagnóstico de Tromboembolia Pulmar:

Seleccione una:

Seleccione una :

a. La disnea y la taquipnea son las manifestacio nes clínicas más frecuentes

b. Los escores de Ginebra y de Wells permite estratificar la probabilidad diagnóstica de un


Troboembolismo de Pulmón.

c. El ecodoppler venoso es el estudio de mayor sensibilidad y especificidad para el diagnóstico de


Tromboembolismo de Pulmón . .¡

d. Ante una baja probabilidad clínica de Tromboembolismo de Pulmón un dímero-O de alta sensibilidad
negativo permite excluirlo.

Respuesta correcta

La r espuesta correcta es: El ecodoppler venoso es el estud io de mayor sensibilidad y especificidad para el
diagnóstico de Tromboembolismo de Pulmón.
Son factores de riesgo pa ra neumoníai nosocomial

Seleccione una:

Seleccione una:

a. Diabetes Mellitus

b. Edad y Etnia

c. Aspiración de contenido gástrico, Reintubación .¡


d. Extremos de la v ida

Respuesta correcta

La respuesta correcta es: Aspiración de contenido gástrico, Reintubación

El signo clínico más importante pa ra considerar el coma estructural más que meta bólico es:

Seleccione una:

A Respiración de Cheyne-Stokes X
B. Anisocoria

C. Crisis epilépticas

D. Postura extensora anormal

La respuesta correcta es: Anisocoria

676- 728

En relación al tratamiento de la sífilis, ¿cuál de las siguientes es la respuesta correcta?


Seleccione una:
a. La penicilina es el tratamiento de elección en cualquier estadio.
b. El tratamiento de elección son las tetraciclinas.
c. No hay ningún tratamiento efectivo.
d. La penicilina es el tratamiento de elección sólo en la neurosífilis.

En el análisis de líquido cefalorraquídeo usted encuentra linfocitos aumentados, glucosa


disminuida, usted podría pensar en que la etiología es: Seleccione una:
a. parasitaria
b. bacteriana
c. Micobacterias
d. viral

Qué patología de las siguientes es una condición dene Sida: Seleccione una:
a. leucoplasia vellosa
b. candidiasis oral
c. displasia cervical
d. herpes tipo 1: úlceras crónicas > 1 mes

Ante un niño de 7 meses con ebre e irritabilidad, fontanela abombada y un estudio de


líquido cefalorraquídeo con 110 células/ mm 3(75% linfocitos), proteínas 120 mg/dl y
glucosa 28 mg/dl (glucemia sérica 89 mg/ dl), ¿cuál es la sospecha diagnóstica más
razonable? Seleccione una:
a. Meningitis vírica.
b. Síndrome mononucleósico.
c. Meningitis tuberculosa.
d. Meningitis bacteriana.

El riesgo de contaminación Bacteriana debido a una transfusión ha aumentado conforme


disminuye el riesgo de infecciones virales, muchas bacterias no proliferan en frío. Cuál de
estos derivados representa el mayor riesgo de contaminación? Seleccione una:
a. Concentrado de plaquetas
b. Críoprecipitado
c. Plasma
d. Concentrado de glóbulos rojos

La hepatitis B que nefropatía produce: Seleccione una:


a. Nefritis lúpica
b. Nefropatía por IgA
c. Granulomatosis de Wegener
d. Glomerulonefritis membranoproliferativa

Pág 729 - 784


Masculino de 65 años de edad se presenta con una historia de disnea progresiva y
tos no productiva de 3 días de evolución. Había sido hospitalizado previamente hace
2 años por insuficiencia cardiaca congestiva. Sus signos v itales: TA 90/55 mmHg,
frecuencia cardiaca de 110 latidos por minuto, temperatura de 37.6C, y saturación de
oxígeno al aire ambiente es de 86%. A la auscultación de tórax, se detectan estertores
y roncus bilaterales. La radio¡¡rafía de tórax revela infiltrados bilaterales y un aumento
discreto del tamaño de la silueta cardiaca. ¿Cuál es el diagnóstico clínico más
probable?

Seleccione una:

r a. Disección de Aorta
r b. Angina estable
r c. Tromboembolia pulmonar
r

d. Edema agudo de pulmón
Retroalimentación
Respuesta correcta
La respuesta correcta es: Edema agudo de pulmón

Las tres alteraciones funcionales básicas del asma son:


Seleccione una:
1 a. Obstrucción al fh.~o de aire, obstrucción al flujo venoso e irreversibilidad de la vfa aérea
r

b Obstrucción al fll4Q de aire, reversibi ldad de la vla aérea e hiperreactividad bronquial
r c. Hipertensión arterlal, hiperreactlvldad bronquial y obstrucción nasal
r d. Tos, obstrucción flujo aéreo y somnolencia

Retroalimentación
Respuesta correcta
La respuesta correcta es Obstrucción al flujo de aire, reversibilidad de la vía aérea e
h1perreactiv1dad bronquial

Es la mejor prueba complementaria para el diagnóstico de asma:

Seleccione una:
r a. Broncoscopia
r b. Lavado bronquial
r c. Detem,inación de inmuno¡¡lobuhnas
r

d Esp1rometrla

Retroalimentación

Respuesta correcta
La respuesta correcta es. Esp1rometría
Señale la respuesta incorrecta con relación a la tuberculosis:
Seleccione una:

r a. La enfermedad tuberculosa es aquella situación en la que el individuo presenta datos


clínicos.

r b. La infección tuberculosa es aquella situación en la que el individuo entra en contacto con el


germen
r- e. La primoinfección tuberculosa es aquella situación en la que el individuo entra en contacto

con el germen y desarrolla por primera vez la enfermedad.


r d. La tuberculosis posprimaria es aquella situación en la que el individuo presenta una
reactivación de la enfermedad tuberculosa.

Retroalimentación

Respuesta correcta
La respuesta correcta es: La primoinfección tuberculosa es aquella situación en la que el
individuo entra en contacto con el germen y desarrolla por primera vez la enfermedad .

Cuál de las siguientes premisas es FALSA respecto del diagnóstico de


Tromboembolismo de Pulmón:

Seleccione una:
r a. Los escores de Ginebra y de Wells permite estratif ,car la probabilidad d 1agnóst1ca de un
Troboembolismo de Pulmón.
r b. Ante una baja probabilidad clínica de Tromboembohsmo de Pulmón un dlmero-0 de alta
sensibilidad negativo permile excluirlo.
r

c. El ecodopplervenoso es el estudio de mayor sensibilidad y especificidad para el diagnóstico

de Tromboembohsmo de Pulmón.

(' d. La disnea y la taquipnea son las manifestaciones clínicas más frecuentes

Retroalimentación
Respuesta correcta
La respuesta correcta es: El ecodoppler venoso es el estudio de mayor sensibilidad y
espec1fic1dad para el diaanóstoco de Tromboembollsmo de Pulmón
Son factores de riesgo para neumonía nosocomial
Seleccione una:
r- a.

Aspiración de contenido gástrico, Reintubación


r b.
Diabetes Mellitus
r c. Edad y Etnia
r d.
Extremos de la vida

Retroalimentación

Respuesta correcta
La respuesta correcta es:
Aspiración de contenido gástrico, Reintubación

Cuál es el cuadro clínico característico de los pacientes con Bronquitis crónica?


Seleccione una:

r a. llamad os sopladores azules, aumento de peso y coloración normal de la piel

r b. llamad os tosedores rosados, con espiración prolongada y aumento de peso

r- e. llamados sopladores azules, cianóticos y caquécticos


r d. llamad os tosedores rosados, hemopt isis sin tos productiva

Retroalimentación

Respuesta correcta
La respuesta correcta es: llamados sopladores azules, cianóticos y caquécticos
Cuáles son los principales factores de riesgo para el desarrollo de NAC?.

Seleccione una:
r a. Alcoholismo, parejas sexuales múltiples, uso de anticonceptivos orales, antecedentes de
cirugía cardiaca o pulmonar

r b. Enfermedad de Alzheimer, fibrosis quística, pacientes entre 30 y 50 años de edad, uso de


anticonceptivos orales, antecedentes de cirugía cardiaca o pulmonar.

r c. Múltiples pareja sexuales, EPOC, tabaquismo, lnmunodeprimidos, adultos mayores sanos, y


personas adultas jóvenes
r- d. Extremos die edad (ancianos y niños), tabaquismo, alcoholismo, diabetes, asma,

insuficiencia cardíaca, EPOC, cáncer, inmunodeprimidos.

Retroalimentación

Respuesta correcta
La respuesta correcta es: Extremos de edad (ancianos y niños), tabaquismo, alcoholismo ,
diabetes, asma, insuficiencia cardíaca, EPOC, cáncer, inmunodeprimidos.

Criterios para derrame pleural exudativo, excepto:

Seleccione una:

l a. Proteínas de liquido pleural/proteínas séricas >0 ..5

r b. LDH del liquido pleural >66% del limite superior normal para el suero

r- e. LDH del liqu ido pleural/LDH sérica <0.4


r d. LDH del liquido pleural/LDH sérica >0.6

Retroalimentación
Respuesta correcta
La respuesta correcta es: LDH del liquido pleural/LDH sérica <0.4
Cuáles son los 3 síntomas o signos más frecuentes de la EPOC

Seleccione una:
r a . Tos crónica, disnea crónica y sibilancías
r b. Tos crónica, disnea crónica y utilización de músculos accesonos

r- e. Tos cróruca, disnea crónica y expectoración mucosa


r d . Tos crónica, disnea crónica y fiebre

Retroalimentación

Respuesta correcta
La respuesta correcta es, Tos crónica disnea crómca y expectoración mucosa

C on respecto a la EPOC, no es cierto:


Seleccione una:

r a . Espirométricamente, se detecta obstrucción por un cociente FEV1/FVC inferior a 0,70

r- b. Contrariamente al asma, no hay componente inflamatorio

r c . El factor genético para desarro llar EPOC mejo r docu mentado es el déficit de alfa1.
antitripsina

r d . El hábito tabáquico es el factor más importante para desarrollar EPOC

Retroalimentación

Respuesta correcta
La respuesta correcta es: Contrariamente al asma, no hay componente inflamatorio

En la exploración funcional de un paciente con enf ermedad pulmonar obstructiva


crónica, son esperables todos los hallazgos MENOS uno:

Seleccione una:

(" a. Cociente FEV1/FVC inferio r al O. 7

(" b. FEV1 menor del 80% .


("
c. Prueba broncodilatadora negativa

(-
d. Volúmenes pulmonares d isminuidos

Retroalimentación

Respuesta correcta
La respuesta correcta es: Volúmenes pulmonares disminuidos
Actualmente se utilizan 3 clases de broncodilatadores en pacientes con asma, señale
lo correcto:

Seleccione una:

("' a. Agonistas ¡32, corticoesteroides inhalados, corticoesteroides sistémicos


(i'
b. Agonistas ¡32, anbcolinérg,cos, teof,ltna
("' c. Anticohnéfgicos, antileucotnenos, agonistas 132

("' d. Antileucotrienos, teofilina, agonistas 132


Retroalimentación

Respuesta correcta
La respuesta correcta es: Agonistas ¡32, anticohnérgicos teofilina

Cuál de las siquientes anomalías electrocardiOQráficas suele asociarse a


tromboembolia pulmonar?

Seleccione una:
("'
a. Inversión deondas T en la derivación de ondas laterales 1, AVL yV5-V6
("' b. Ondas Q en las derivaciones anteriores de V1· V4
("'
c. BloQueo A-V de primer ¡¡rad o
(i'
d. S1Q3T3, ondas T negativas en las derivaciones anteriores de V 1 a V4
Retroalimentación

Respuesta correcta
La respuesta correcta es: S1Q3T3, ondas T negativas en las derivaciones anteriores de V1 a
V4
¿Patrón electrocardiográfico?

Respuest a: S1 Q3T3 onda Q Negat ivo

Factores de riesgo de desarrollo de neumonía:

RP : Niño o ancianos (edad),comorbilidades (OM, HTA), Asinam ient o, inmunosupresión,


institucionalización, demencia, exceso de peso.

Alteraciones básicas del asma:

RP : Obstrucción al fluj o de la vía aérea, reversibilidad de la vía área, hiperactiv idad bronq uial.

EPOC no lo que no es cierto:

RP : No hay component e inflamat orio cont rario al asma

Tratamientos broncodilatadores de l asma:

RP : Beta agonist a, leucot rienos (t eofilina), ant icolin érgico.

Mujer 54 años encontrada desorientada, saturaci,ón 80% al ambiente, pupilas puntiformes, pH: 7,25,
pCO2: 60, pO2: 56 Na 136 CI 100, Según la gasometría:

RP : (oj o leer) Acidosis respirat oria pura .

TB Cual no es la correcta:

RP : la prim oinfección es cuando el pacient e ent ra en cont act o y desarro lla la infección .

TEP cual es LA INCORRECTA:

RP : Ecodopleres el mas sensibles: no es muy sen,sible (es la angiografía pulmonar eso la correcta
angioTAC)

Gasometría lo verdadero de

RP: Patrón invertido PCO2 aumentado ( + 45) pO2 dismin uida (-60).

Masculino de 65 años disnea progresiva, con tos no productiva de 3 días de evolución. Signos vitales:
Pa 90/55 p 110, t 37,6 saturación 86%. Diagnostico probable. Estertores, aumento de la silueta
cardiaca, infiltrados.

RP : Edema de pulmón ,cardiogénico.


TEP 68años edad, neoplasia de páncreas más quimioterapia viene a consulta por dolor y edema de
miembros inferiores.

Rp : eco Ooppler venoso. (t rombosis venosa prof unda).

Hemoptisis masiva:

Rp : de400 a 600 mi

3 signos del EPOC:


Rp : Tos crón ica expectoración productiva mucosa, disnea crón ica .

Exploración funciona con EPOC son esperables todos los hallazgos meno s a uno

Pp: Disminución de volúmenes pulmonares.

De l o siguiente cual es i ncorrecta: Germen que no prod uc.e tube rcul osi s pul monar

RP: Mcleprae.

Definición de ingreso NAC, Hospital ización

RP : más de 2 hospitalización.

Cual cuadro clínico del paciente con bronquitis crónica

RP : Soplador azules, caquéctico, cianótico (flaco azulado).

Mejor prueba de Asma

Espirometría.

Factores de riesgo neumonía nosocomi al :

Rp : Re intubación, Aspiración de contenido gástrico.

Criterios de derrame pleural exudativo excepto:

Rp : LDH menor a 0,4.

Cuál de los siguientes agentes es el causante de erisipela y celulitis:

Seleccione una:

a. Cándida

b. Estreptococo del grupo A.¡

c. Estafilococo Aureus

d. Estreptococo del grupo B

La respuesta correcta es: Estreptococo del grupo A

PAG 784-835 ROMI

El mejor tratamiento ara el estómago en sandia (hipertensión Portal) es:

a. Antrectomía

b. Fármacos que promuevan la motilidad.

c. Gastrectomía total.
d. Agentes para reducir el acido

El tratamiento estándar de un tumor estromal gastrointestinal del estómago es:

a. Gastrectomía total.

b. Gastrectomía subtotal.

c. Ablación endoscópica.

d. Resección en cuña con márgenes limpios.

El tratamiento para el síndrome de vaciamiento rápido temprano grave después de


gastrectomía es:

a. Conversión quirúrgica a drenaje tipo Roux en Y.

b. Tratamiento expectante.

c. Octreotido

d. Glucosa oral para los síntomas

La prueba diagnóstica más exacta para establecer el síndrome de Zollinger-Ellison


es:

a. Prueba de estimulación con secretina

b. Gastrina sérica en ayuno

c. Endoscopia

d. Tomografía por computadora

Lo más frecuente es que la lesión de Dieulafoy derive en:

Seleccione una:

a. Progresión a cáncer gástrico.

b. Hemorragia gastrointestinal superior

c. Gastroparesia

d. Obstrucción de la salida gástrica.

Una de las siguientes afirmaciones es verdadera, respecto a la infección por c.


di􀃞cile.

a. Se debe esperar el cultivo mas antibiograma antes de iniciar tratamiento.

b. Aproximadamente un 70% sufren una recaída al _nalizar el tratamiento.


c. Ante un caso de colitis fulminante, si pasados 3 días de tratamiento antibiótico no
hay respuesta,

está indicado el tratamiento quirúrgico.

d. La vía de contagio es oral-oral.

Una mujer con 65 años de edad con ulcera duodenal conocida se trata con dieta y un
bloqueador H2. Ingresa al hospital con hemorragia gastrointestinal superior de con
compromiso hemodinámico. Junto a iniciar la reposición sanguínea, el próximo paso
en el tratamiento sería

Seleccione una:

a. Endoscopia y coagulación del vaso sangrante

b. Iniciar omeprazol

c. Iniciar bismuto, tetraciclina y metronidazol

d. Piloroduodenotomía y cierre del vaso sangrante con sutura.

Una mujer de 25 años re􀃕ere una historia de 12 meses de cólicos abdominales


recurrentes en hemiabdomen inferior acompañado de distensión abdominal y
diarreas. No presenta 􀃕ebre, pérdida de peso ni anorexia ni deposición con sangre.
Este paciente cumple con los criterios de Roma IV para Sd de colon irritable con
predominio de diarrea, no presenta signos de alarma. En que otra patología usted
puede pensar?. Seleccione una:

a. Hipertiroidismo.

b. Helicobacter pylori.

c. Enfermedad celiaca.

d. Enfermedad inflamatoria intestina

Una de las características principales por las cuales el dengue puede diseminarse es:

a. Es sumamente frecuente la progresión de daño encefálico.

b. Una característica precoz es la trombocitosis y neutrofilia.

c. Debido a la gran capacidad de diseminación del vector Anopheles.

d. El vector es muy cercano a los asentamientos humanos.

¿De qué depende la inmunidad contra Leptospira? Seleccione una:

a. Producción de anticuerpos contra LPS inespecíficos.


b. Producción de anticuerpos circulantes contra LPS específicos de un serotipo.

c. Producción de anticuerpos contra proteínas inespecíficas de un serotipo.

d. Producción de anticuerpos contra los distintos serotipos

El mecanismo por el cual la infección por cólera produce una diarrea tan intensa es
por. Seleccione una:

a. Vibrios enteroagregativos.

b. Toxina enterogénica

c. Toxina Shiga-like

d. Vibrios entero hemorrágicos.

La ictericia en un paciente con Malaria es frecuente a partir:

a. La ictericia no aparece en la malaria.

b. Ocasionalmente en niños debido a su alta susceptibilidad

c. Siempre existirá fiebre e ictericia debido a la destrucción de eritrocitos.

d. La Ictericia en Malaria es frecuente a partir de los 15 años.

Tras la infección de un niño con el virus de la Influenza con la siguientes

características: fiebre intensa de inicio súbito, disnea y cianosis a la Rx de tórax se


aprecia patrón asociado con infiltrados intersticiales difusos e hipoxia intensa, usted
sospecharía de:

a. Neumonía bacteriana secundaria.

b. Neumonía viral primaría.

c. Neumonía bacteriana primaria.

d. Neumonía viral secundaria

En cuanto a la profilaxis antitetánica en el tratamiento sistemático de las heridas, en


una herida pequeña limpia sin ningún antecedente de vacunación antitetánica Usted
recomendaría:

a. Es indicativo de inmunoglobulina.

b. Solo limpiar la herida y dar indicaciones de cuidado al paciente, ofrecer la antitoxina.

c. Dar antibiótico de manera profiláctica.

d. Vacunación antitetánica de inmediato.


Dentro de las manifestaciones tardías de la sífilis tenemos trastornos
cardiovasculares de cual podemos desprender el siguiente:

a. Insuficiencia cardiaca congestiva.

b. Aneurismas

c. Trastornos del endotelio

d. Flebitis irritativa.

En la clasificación de OMS de adultos con VIH – Sida en el estadio C3 tenemos a::

a. Mayor de 500 células CD4+ Asintomático.

b. Menor de 300 células CD4+ con síntomas definidores de Sida.

c. Menor de 200 células CD4+ con síntomas definidores de Sida.

d. Mayor de 400 células CD4+ con síntomas no definidores de Sida.

Qué infección causa bloqueo auriculoventricular:

a. Varicela

b. Tripanosomiasis

-
c. Sífilis

d. Sarampión

En caso que se enfrente a una situación de tétanos una medida de soporte


importantepara el control de espasmos es:

a. Gabapentina

b. Metronidazol

c. Antitoxina

d. Diazepam

Cuál consideraría como primera opción para el diagnóstico de fiebre tifoidea ( fiebre
enterica):

a. biopsia intestinal

b. Reacción de Widal

c. rosa de bengala

d. hemocultivo
Cuál de los siguientes es un signo característico de las vías respiratorias de las
personas que sufren asma:

a. Adhesión de los glóbulos rojos al endotelio

b. Infiltración basófila

c. Transformación de los basófilos en macrófagos

d. Infiltración eosinofílica

¿Cuál es la clasificación anatómica de las bronquiectasias?

a. Cubicas, cilíndricas, alargadas

b. Lobares, arteriales, globulares

c. Vasculares, semilunares saculares

d. Cilíndricas, varicosas, saculares

¿Cuál es la fisiopatología de la hemoptisis?

a. Hipervascularización de la circulación brónquica, hipertensión pulmonar y


neovascularización

b. Hipervascularización de la circulación pulmonar, _ebre y regeneración alveolar

c. Hipervascularización de la circulación pulmonar, hipertensión pulmonar y remodelación.

d. Hipervascularización de la circulación brónquica, hipertensión pulmonar y disminución de


coagulabilidad

De los siguientes factores, cuál es el que debe estar alterado para que un paciente
infectado de tuberculosis se convierta en persona enferma:

a. Medio Ambiente: Que viva en un lugar frío

b. Huésped: que genéticamente esté predispuesto a infectarse

c. Medio ambiente: Que haya elevada prevalencia de tuberculosis

d. Huésped: Inmunidad celular deficiente, en especial CD4

El hallazgo más frecuente en la radiografía de tórax en un paciente con asma es:

a. Engrosamiento de paredes bronquiales.

b. Condensaciones alveolares bilaterales y difusas.

c. Radiografía de tórax normal.


d. Hiperinsuflación pulmonar.

El tratamiento de las Bronquiectasias se basa en 3 pilares, excepto:

a. Eliminar la obstrucción bronquial.

b. Controlar las infecciones con el uso de antibióticos en las agudizaciones durante 10- 15
días.

c. Revertir el remodelamiento bronquia

d. Mejorar la eliminación de las secreciones, que se consigue con una adecuada


hidratación, con fisioterapia respiratoria y drenaje postural mantenidos.

Pag 677 - 726 Erick }

En caso que se enfrente a una situación de tétanos una medida de soporte importante
para el control de espasmos es: Seleccione una:
a. Diazepam
b. Antitoxina
c. Gabapentina d
. Metronidazol

Su paciente presenta las siguientes características ebre alta prolongada dolor


abdominal diarrea y estreñimiento ocasionales esplenomegalia y roséola usted
sospecha de: Seleccione una:
a. Salmonelosis
b. Shigella
c. Entamoeba histolytica
d. Giardiasis

Paciente que acude a Urgencias con ebre intermitente, escalofríos, cefalea, debilidad
y abundante sudoración nocturna de 3 días de evolución. Como antecedente
epidemiológico destaca un viaje a Cayambe hace 3 semanas donde consumieron
leche cruda y queso sin pasteurizar. El modo más rápido de diagnosticar la infección
sospechada es: Seleccione una:
a. Gram directo del líquido cefalorraquídeo.
b. Test para detección de anticuerpos heterólos (Paul-Bunnell).
c. Tinciones de micobacterias en esputo u orina.
d. Prueba del Rosa de Bengala y toma de hemocultivo.
En la clasicación de FOD cuál considera usted como urgente y debe ser tratada de
inmediato: Seleccione una:
a. FOD neutropénico
b. FOD Clásico
c. FOD Nosocomial
d. FOD asociado a HIV

El interferón es un medicamento útil para las siguientes infecciones excepto:

-
Seleccione una:
a. VIH
b. HPV
c. Hepatitis B
d. Sarcoma de Kaposi

Para el tratamiento de Leishmania cutánea de primera línea se utiliza: Seleccione una:


a. Anfotericina B
b. Dapsona
c. Fluconazol
d. Antimonio pentavalente

En relación al tratamiento de la sílis, ¿cuál de las siguientes es la respuesta correcta?


Seleccione una:
a. La penicilina es el tratamiento de elección en cualquier estadio.
b. El tratamiento de elección son las tetraciclinas.
c. No hay ningún tratamiento efectivo.
d. La penicilina es el tratamiento de elección sólo en la neurosílis.

Si durante el examen físico en el área genital encuentra una úlcera lisa no purulenta
bien delimitado única supercial con linfadenopatía bilateral usted piensa en:
Seleccione una:
a. Leishmania
b. herpes tipo 2

-
c. chancroide
d. sífilis

De los siguientes factores, cuál es el que debe estar alterado para que un paciente
infectado de tuberculosis se convierta en persona enferma: Seleccione una:
a. Medio Ambiente: Que viva en un lugar frío
b. Huésped: que genéticamente esté predispuesto a infectarse
c. Medio ambiente: Que haya elevada prevalencia de tuberculosis
d. Huésped: Inmunidad celular deciente, en especial CD4
HEMOPTISIS
1. ¿La hemoptisis tiene como fisiopatología 3 cambios importantes?
a. Hipervascularización, hipertensión y neovascularización
b. Hipervascularización, fibrosis y neovascularización
c. Fibrosis, ruptura de vasos sanguíneos, aumento de la presión vascular
d. Ninguna de las anteriores
2. ¿Cuál de las siguientes no corresponde a la clasificación de la SEPAR?
a. Expectoración hemoptóica
b. Hemoptisis franca
c. Hemoptisis exanguinante
d. Hemoptisis amenazante
3. La causa más frecuente de hemoptisis son excepto:
a. Neoplasias
b. Bronquitis crónica
c. Neumonías o obsesos pulmonares
d. Vasculitis
4. Etiológicamente existe hemoptisis de origen pulmonar y extrapulmonar. Dentro de
las hemoptisis de origen pulmonar tenemos las siguientes excepto:
a. Neumonía
b. Tuberculosis
c. Tromboembolismo pulmonar
d. Absceso de pulmón
5. Etiológicamente existe hemoptisis de origen pulmonar y extrapulmonar. Dentro de
las hemoptisis de origen extrapulmonar tenemos las siguientes excepto:
a. Enfermedad mitral
b. Vasculitis
c. Enfermedad de Rendu-Osler
d. Alteración de la coagulación
6. De acuerdoa la clasificación según el volumen de sangre expulsado en la
hemoptisis en determinado tiempo señale cual no corresponde:
a. Esputo hemoptoico
b. Hemoptisis leve
c. Hemoptisis mayor
d. Hemoptisis exanguinante
7. El volumen de sangre que se encuentra en la hemoptisis franca es de:
a. Menor a 15ml/d
b. Más de 15ml – 200ml/d
c. Más de 200ml/d – 600ml/d
d. Más de 600ml/d/16hrs
8. El volumen de sangre que se encuentra en la hemoptisis exanguinante es de:
a. Menor a 15ml/d
b. Más de 15ml – 200ml/d
c. Más de 200ml/d – 600ml/d
d. 1000ml/d – 150ml/h
9. Entre las recomendaciones para hemoptisis leve señale la incorrecta:
a. Reposo absoluto decúbito ipsilateral
b. Abstención tabáquica
c. Antitusígenos
d. Terapia antibiótica en sospecha de infección
10. En la hemoptisis masiva o amenazante los objetivos primordiales del tratamiento
son los siguientes excepto:
a. Detener la tos persistente del paciente
b. Asegurar la permeabilidad de la vía aérea y oxigenación
c. Lograr estabilidad hemodinámica
d. Localizar y detener el sangrado

BRONQUIECTASIAS
1. La definición de bronquiectasia es:
a. Dilataciones de capilares pequeños y de los vasos superficiales.
b. Dilataciones anormales e irreversibles de los bronquios
c. Contracciones anormales e irreversibles de los bronquios
d. Dilataciones fisiológicas, reversibles de los bronquios
2. Cuál es la estructura que se ve afectada en el bronquio?
a. Parénquima pulmonar causando fibrosis
b. Paredes alveolares
c. Destrucción del componente elástico y muscular de la pared bronquial
d. Epitelio cilíndrico pseudoestratificado y cilios
3. Según la etiología de las bronquiectasias, cuál de las siguientes es de origen
congénito?
a. Aspiración de contenido gástrico
b. Sd. Bronquítico crónico
c. Fibrosis quística
d. Infección por adenovirus
4. Las bronquiectasias tienen como fisiopatología lo siguiente excepto:
a. Alteración de la motilidad ciliar y el aclaramiento
b. Respuesta inflamatoria local
c. Hipertensión vascular
d. Daño epitelial
5. La manifestación clínica más importante en la bronquiectasia es:
a. Tos crónica con expectoración matutina de tipo mucopurulento
b. Pérdida de peso
c. Fiebre
d. Disnea de aparición súbita
6. En el examen físico durante la inspección un signo que nos puede orientar es:
a. Acropaquia
b. Tiraje intercostal
c. Temblor fino
d. Cianosis periférica

7. Según la clasificación de Reid las bronquiectasias son :


a. Cilíndricas o fusiformes
b. Varicosas
c. Saculares o quísticas
d. Todas las anteriores
8. El diagnostico de bronquiectasias se le realiza con excepto:
a. Espirometría
b. Resonancia magnética
c. Tomografía computarizada
d. Radiografía de tórax
9. Dentro de la valoración de la función pulmonar durante la espirometria en
bronquiectasia se encuentra:
a. Una CVF normal o levemente reducida
b. En VEF1 está disminuido en fumadores y no fumadores,
c. La relación VEF1/CVF está disminuida en hombres y mujeres
d. Todas las anteriores
10. El tratamiento de las bronquiectasia se basa en 3 pilares excepto:
a. Eliminar la obstrucción bronquial
b. Mejorar la eliminación de las secreciones
c. Controlar las infecciones
d. Revertir el daño pulmonar

EPID
1. La evidencia demuestra que la fibrosis pulmonar idiopática afectan a personas
mayores de:
a. 12 años.
b. 8 años
c. 20 años
d. 50 años
2. Cuál es el Gold estándar de la EPID?
a. Rx de Torax
b. TAC de torax
c. Resonancia Magnetica
d. Ecografia.
3. En relación a la EPID la biopsia estaría indicada en cuál de los siguientes casos?
a. En el caso que la TAC no ponga de manifiesto los hallazgos típicos de
neumonía intersticial usual.
b. Cuando hablamos de una hipertrofia de las células del intersticio, que no
logran ser visualizadas a través de Rx de Torax
c. Nunca se realiza Biopsia porque este examen esta descontinuado para esta
patología.
d. En la presencia de hipertrofia sin exudado.
4. A la exploración física de un paciente con dg de EPID, cual de los siguientes signos o
síntomas usd esperaría encontrar, SEÑALE EL INCORRECTO:
a. Estertores
b. Crepitantes.
c. Acropaquia.
d. Alopecia
5. Los factores que influyen en el pronostico de EPID son, excepto:
a. Hora del diagnostico (Deteccion neonatal)
b. Poblacion de Patogenos resistentes
c. Funcion pulmonar
d. El alelo CFTR se transmite del padre al feto
6. Los dos diagnosticos de EPI mas comunes son:
a. FPI y Sarcoidosis.
b. TB y TVP
c. Bronquiectasias y Hemoptisis
d. TB y Bronquiectasias.
7. Una nueva clasificación de EPID habla de 3 grupos estos son, excepto:
a. Fibrosis Pulmonar idiopática.
b. Neumonitis intersticial inespecífica.
c. Neumonitis intersticial aguda
d. Sarcoidosis
8. La neumonía intersticial idiopática presenta el siguiente cuadro clínico:
a. Inicio incidioso y se caracteriza por disnea de esfuerzo progresiva y tos
secante persistente.
b. Tos por mas de 15 dias de evolución y puede ser productiva o no productiva
c. Restriccion de la respiración, fiebre, disnea de pequeños esfuerzos.
d. Lipotimia, astenia, hemoptisis.
9. La sarcoidosis afecta predominantemente a adultos jóvenes entre:
a. 20 a 40 años
b. 12 a 15 años
c. 30 a 50 años
d. 10 a 20 años
10. El cuadro clínico de la sarcoidosis incluye, excepto:
a. Tos
b. Disnea
c. Hiperreactividad bronquial
d. Dolor abdominal y toraxico.
DERRAME PLEURAL
1. El movimiento neto de líquido desde pleura visceral hacia la cavidad pleural es de:
a) 0,04 ml / kg / h
b) 0,20 ml / kg / h
c) 0,01 ml / kg / h
d) 0,10 ml / kg / h
2. La acumulación en de líquido en el espacio pleural se debe a, excepto:
a) Disminución de la presión oncótica
b) Aumento de la presión hidrostática
c) Alteración del drenaje linfático
d) Aumento de la Presión positiva de espacio pleural
3. Son complicaciones de la toracocentesis, excepto:
a) Neumotórax
b) Hemotórax
c) Trastorno de la coagulación
d) Punción de órganos abdominales
4. ¿Cuál de las siguientes etiologías es la que más produce incidencia de derrame pleural
en un paciente?
a) Insuficiencia cardíaca
b) Tuberculosis
c) Neumonía
d) Ateroesclerosis
5. ¿Cuál de los siguientes signos radiológicos está presentes en la RX de un paciente con
derrame pleural?
a) Signo del Menisco de Damoisseau
b) Opacidad apical bilateral
c) Signo de la S de Golden
d) Signo del Diafragma Continuo
6. ¿La relación que existe entre proteínas pleura / proteínas de plasma en el análisis de
líquido pleural para caracterizarlo como exudado es de?
a) > 0,3
b) > 0,5
c) > 0,7
d) > 0,9
7. ¿Son causas de derrame pleural exudativo, excepto?
a) Insuficiencia cardiaca congestiva
b) Cirrosis
c) tuberculosis
d) Embolia pulmonar
8. ¿Cuál de las siguientes es una complicación al momento de realizar una biopsia
pleural?
a) Neumotórax
b) Hipotensión
c) Infecciones
d) Broncoespasmo
9. ¿Cuál es el gold estándar para diagnosticar derrame pleural maligno?
a) Tomografia computarizada
b) Toracoscopia
c) Resonancia Magnetica
d) RX torax
10. ¿El tratamiento de eleccion en pacientes que presentan derrame pleural con
características de trasudado es?
a) Quimioterapia
b) Drenaje
c) Antibióticos
d) Diuréticos

NEUMOTÓRAX

1. De acuerdo a las causas de neumotórax. ¿Cuál de las siguientes correspondería a un


neumotórax espontáneo primario?
a) Asma
b) Bula subpleural congénita
c) EPOC
d) Fibrosis quística

2. ¿Cuál es la complicación más frecuente del neumotórax?


a) Hemoneumotórax
b) Neumotórax a tensión
c) Pioneumotórax
d) Neumotórax bilateral

3. Paciente de 27 años, apuñalado con arma cortopunzante en el tórax en 4to espacio


intercostal línea axilar anterior , presenta cianosis , taquicardia, taquipnea, disnea ,
regurgitación yugular, y al instante sufre un paro cardiorespiratorio. ¿Cuál es el
diagnóstico más probable?
a) TEP
b) Neumotórax a tensión
c) Infarto del miocardio
d) Derrame pleural
4. ¿Cuál de los siguientes signos radiográficos no corresponde a un hallazgo de
neumotórax?
a. Desplazamiento de las estructuras del mediastino hacia el lado contrario del
afectado.
b. Aumento de la radiolucidez en el área de acumulación del aire.
c. Colapso pulmonar parcial o completo del pulmón del lado afectado.
d. Joroba de Hampton.

5. ¿Cómo se diagnostica un neumotórax?


a. Historia clínica, examen físico y una Rx.
b. La TAC es el gold standard para el diagnóstico de neumotórax.
c. Se realiza una radiografía y la medición del dimero D.
d. Historia clínica, examen físico, espirometría y Rx.

6. ¿Dónde se acumula el aire en el neumotórax?


a. Entre la pleura parietal y la pared torácica.
b. Entre la pleura visceral y el pulmón.
c. Entre los alvéolos y el parénquima pulmonar.
d. En el espacio pleural.

7. Paciente inestable de 34 años de edad, es llevado a emergencias por trauma con


arma blanca. ¿Qué es lo primero que debe hacer?
a. Observación
b. Drenaje
c. Corregir los trastornos electrolíticos.
d. Tratamiento antibiótico

8. ¿Qué tipo de neumotórax puede tener un paciente con antecedente de EPOC?


a. Adquirido
b. Iatrogénico
c. Barotrauma
d. Espontáneo-secundario
9. En el manejo de un neumotórax solo se hace observación en:
a. Paciente inestable
b. Neumotórax pequeño <3cm
c. Patología relevante
d. Sometido a ventilación mecánica
10. Paciente con antecedentes de bronquitis crónica grave es llevado a emergencias por
presentar disnea y dolor pleurítico. ¿Cuál examen le haría al sospechar de un
neumotórax?
a. RX de tórax
b. Biometría hemática
c. Espirometría
d. Química sanguínea

ASMA

1. Clasificación GINA del Asma


a) Leve persistente, moderada intermitente y severa persistente
b) Intermitente, leve persistente, moderada persistente y severa persistente
c) Leve intermitente, persistente, moderada y severa
d) Intermitente, moderada intermitente y persistente severa

2. Gold Standard para Diagnóstico de Asma


a) Rx de tórax
b) Pruebas cutáneas
c) Óxido nítrico exhalado
d) Espirometría

3. Para el diagnóstico del asma post broncodilatador, debemos encontrar un valor:


a) VEF1 mayor 20%
b) VEF1 menor 20%
c) VEF1 mayor 12%
d) VEF1 menor 15%

4. Patrón característico de asma en espirometría que confirma el diagnóstico:


a) Patrón Obstructivo Reversible
b) Patrón Restrictivo Reversible
c) Patrón Mixto

5. Medicamentos indicados en el tratamiento leve persistente del asma


a) B2 antagonista de larga acción más corticoide inhalado mayor a 400 ug
b) B2 agonista de corta acción más corticoide inhalado mayor a 400 ug más corticoide
sistémico
c) B2 agonista de acción corta más corticoide inhalado dosis menores a 400 ug
d) Corticoide inhalado, teofilina más un B2 agonista de corta acción
6. Medicamento indicado para el tratamiento de rescate en el asma
a) Salbutamol
b) Teofilina
c) B2 agonista de acción larga
d) Anti IgE

7. Tratamiento recomendado para Asma Leve Persistente


a) Teofilina y AntiL más corticoide inhalado >500 ug
b) B2 agonista de larga acción más corticoide inhalado >400 ug
c) B2 agonistas de acción corta más corticoide inhalado <400 ug.
d) Teofilina y AntiL más corticoide inhalado >400 ug.

8. Señale lo correcto con respecto al manejo de asma en emergencia


a) Administración de oxígeno contraindicada, broncodilatadores B2 agonistas de
acción larga y corticoide VO.
b) Corregir hipoxemia administrando oxígeno, broncodilatadores B2 de acción larga
y corticoide VO.
c) Corregir la hipoxemia administrando oxígeno, broncodilatadores agonistas B2 de
acción corta y corticoide sistémico.
d) Administración de oxígeno contraindicada, Teofilina y corticoides sistémico.

9. Criterios para hospitalizar al paciente


a) Paciente asintomático después del tratamiento, con necesidad de O2 <90%
b) Paciente con FEM >40% y con necesidad de O2 <90%
c) Paciente sintomático a pesar del tratamiento en las primeras 2 a 3 horas, con
necesidad de O2 >90%.
d) Paciente sintomático a pesar de tratamiento en la primera hora con FEM >40%.

10. De acuerdo al test de control del Asma señale lo correcto:


a) <20 puntos significan control total del Asma
b) 20 – 24 puntos significan buen control del Asma
c) 25 puntos significan Asma no controlado
d) 20-24 puntos significan control total del Asma

TB

1. El bacilo tuberculoso cuánto tarda en dividirse?


a. 16-24horas
b. 24-48horas
c. 12-16horas
d. 12-24horas
2. Sintomático respiratorio se define como
a. Tos por más de 15 días
b. Tos y expectoración por más de 15 días
c. Tos y expectoración por más de 7 días
d. Tos no expectorante por más de 15 días
3. Cuál es el esquema de tratamiento para la tuberculosis?
a. Isoniacida+etambutol
b. Isoniacida+ ritonavir + estreptomicina
c. Isoniacida + rifampicina + pirazinamida + etambutol
d. Isoniacida + estreptomicina + etambutol + rifampicina
4. Quienes son los mayores transmisores de tuberculosis?
a. Personas que menos tosen
b. Con BK- de esputo
c. Pacientes con tratamiento
d. Enfermos que acaban de iniciar el tratamiento
5. Señale la respuesta correcta sobre la INFECCIÓN de la TB:
a) bacilos se replican activamente en el cuerpo
b) Esputo y cultivo negativos
c) Rx Anormal
d) bacilos no se replican activamente en el cuerpo
6. Según el mecanismo de transmisión de la tuberculosis con la tos, cuantos bacilos
expulsamos?
a) 100 – 200 bacilos
b) 4.500 – 1.000.000 bacilos
c) 0 – 3.500 bacilos
d) 1000 – 3000
7. En la Tuberculosis pulmonar la tos en un comienzo es:
a) Con expectoración purulenta
b) Con expectoración mucosa
c) Con hemoptisis
d) Seca e irritativa

8. Señale la respuesta INCORRECTA. Los factores de riesgo para adquirir TB son:


a) Infección reciente
b) Silicosis
c) Hipertensión pulmonar
d) VIH

9. Señale la respuesta correcta. El patrón radiológico típico de TB pulmonar es:


a) Ninguno
b) Condensación pulmonar
c) Intersticial
d) Alveolar

10. Señale la respuesta correcta. Qué porcentaje de enfermos de TB curan


espontáneamente, sin tratamiento:
a) 70%
b) 10%
c) 85%
d) 30%
TEP
1. Conceptualmente la TEP es:
a. Enclavamiento en las arterias pulmonares de un émbolo venoso profundo
b. Enclavamiento en las arterias coronarias de un émbolo arterial
c. Enclavamiento de las arterias pulmonares de un émbolo venoso superficial
de miembros inferiores.
d. Enclavamiento de las arterias pulmonares de un trombo arterial procedente
del corazón.
2. La respuesta hemodinamica a la TEP depende de:
a. Tamaño del émbolo, origen del émbolo y status cardíaco previo.
b. Tamaño del émbolo, status cardiologico previo y respuesta neurohumoral.
c. Tamaño del émbolo, luz del vaso ocluido y respuesta neurohumoral.
d. Tamaño del émbolo, con tenido de fibrina del émbolo y territorio vascular del
que proviene
3. Los dos mecanismos fisiopatológicos de la TEP son:
a. Obstrucción funcional pulmonar y obstruccion mecánica
b. Obstrucción funcional y aumento de mediadores químicos.
c. Obstrucción mecánica y aumento de mediadores químicos.
d. Obstrucción mecánica y colapso de la vía aérea
4. Cuales son los 3 síntomas más frecuentes de la TEP?
a. Disnea, tos y dolor pleuritico.
b. Tos, síncope y disnea.
c. Disnea, tos y hemoptisis
d. Dolor pleuritico, síncope y hemoptisis.
5. Pc Femenino de 71 años con fractura de toblillo, presenta dolor en punta de costado
de instauración súbita con una intensidad de 7/10, que se agrava al movimiento y la
inspiración, se acompaña de disnea, tos y hemoptisis, FC: 114, FR: 26, T: 37.8 ºC.
APP: Adenocarcinoma pulmonar con metástasis óseas en columna lumbar y
metástasis cerebral cosiderado etapa 4. Al valorarlo por escala de Weels usted
considera que:
a) El paciente se encuentra en un riesgo elevado de presentar tromboembolia
pulmonar
b) El paciente se encuentra en un riesgo moderado de presentar tromboembolia
pulmonar
c) El paciente se encuentra en un riesgo bajo de presentar tromboembolia pulmonar y
no debe considerarse pruebas de imagen
d) El paciente se encuentra en un riesgo bajo de presentar tromboembolia pulmonar
por lo que se debe considerar otros diagnósticos diferenciales.
6. Paciente femenino de 24 años usuaria de anticonceptivos orales combinados, con
fractura del pilón tibial en su 4 día de recuperación post quirúrgico presenta dolor en
punta de costado de instauración súbita que aumenta la inspiración acompañado de
tos. Usted solicita Rx de tórax esperando encontrar signos radiográficos de
tromboembolia pulmonar, señale la respuesta incorrecta
a) Presencia de Signo de Westmark y Hilio reducido.
b) Presencia de Joroba de Hampton y Agrandamiento cardíaco
c) Elevación hemidiafragmatica y Atelectasias
d) Disminución del diámetro de la arteria pulmonar y disminución del índice
cardiotorácico
7. La muerte súbita es la manifestación clínica inicial en
a. 14%
b. 25%
c. 32%
d. 45%
8. Un paciente según la escala para predecir la probabilidad de embolismo pulmonar es
de 5.5 es:
a. Nula
b. Baja
c. Intermedia
d. Alta
9. Los valores de la prueba del dimero D para la sospecha de TEP es de:
a. Mayor a 500 mg/dl
b. Mayor a 500 u/dl
c. Mayor a 400 mg/dl
d. Mayor a 400 u/dl
10. Ante la sospecha de un paciente con TEP la conducta adecuada es:
a. Conducta expectante
b. Administrar warfarina
c. Administrar heparina
d. No hacer referencia
3
IVAN FERNANDO PILLAJO TENELEMA

Área personal  Mis cursos  Quito  CIENCIAS DE LA SALUD  MEDICINA - PRESENCIAL  ABR 2021 - AGO 2021
 INVESTIGACION I-HEMATO-TEORIA - Prl: MD NVA Pen: 961 Per:ABR 2021 - AGO 2021 Car:MEDICINA - PRESENCIAL  28
de junio - 4 de julio  Evaluación 1 Investigación I

Comenzado el miércoles, 30 de junio de 2021, 10:45


Estado Finalizado
Finalizado en miércoles, 30 de junio de 2021, 10:59
Tiempo empleado 14 minutos 44 segundos
Puntos 10,00/10,00
Calificación 4,00 de 4,00 (100%)

Pregunta 1 Correcta Puntúa 1,00 sobre 1,00

Un hombre de grupo AB tiene un hijo con una mujer del grupo O. ¿Qué grupo sanguíneo puede tener el
hijo de esta pareja?

Seleccione una:

a. Grupo AB

b. Grupo O

c. Grupo A

d. Grupo O-

Respuesta correcta

La respuesta correcta es: Grupo A


Pregunta 2 Correcta Puntúa 1,00 sobre 1,00

¿Cuál es la reacción más frecuente que se presenta en una transfusión sanguínea?

Seleccione una:

a. La hemolítica crónica

b. La reacción febril no hemolítica

c. La alérgica, urticaria

d. La hemolítica aguda

Respuesta correcta

La respuesta correcta es: La reacción febril no hemolítica

Pregunta 3 Correcta Puntúa 1,00 sobre 1,00

El plasma es la parte líquida de la sangre separada de la parte celular, señale su indicación en estas
patologías.

Seleccione una:

a. Hemofilia B

b. El niño con PTI con petequias

c. Hemofilia A

d. Para ayudar a cicatrizar heridas

Respuesta correcta

La respuesta correcta es: Hemofilia B


Pregunta 4 Correcta Puntúa 1,00 sobre 1,00

Un paciente grupo A (+) sufrió quemaduras en un 30 % de su cuerpo amerita una transfusión de Plasma.
¿Cuál de estos derivados plasmático puede recibir?

Seleccione una:

a. Plasma grupo B

b. Plasma grupo O (+)

c. Plasma grupo AB

d. Plasma grupo O (-)

Respuesta correcta

La respuesta correcta es: Plasma grupo AB

Pregunta 5 Correcta Puntúa 1,00 sobre 1,00

Cuando cumplimos un concentrado globular para tratar una anemia, ¿Cuánto aumenta la hemoglobina
por 1 unidad de concentrado transfundido?

Seleccione una:

a. Aumenta un 8% Hto y 2 g/dl

b. Aumenta 1.5 g/dl y 4% del Hto

c. Aumenta un 3% Hto y 2.5 g/dl

d. Aumenta un 5% Hto y 1 g/dl

Respuesta correcta

La respuesta correcta es: Aumenta 1.5 g/dl y 4% del Hto


Pregunta 6 Correcta Puntúa 1,00 sobre 1,00

La hipersensibilidad tipo III o enfermedad del suero, es una reacción que se da por una transfusión o
por antibióticos, vacunas. ¿Como se diferencia clínicamente de la Enfermedad injerto contra huesped?

Seleccione una:

a. Por la presencia de adenomegalia, glomerulonefrítis, vasculitis

b. Por la hemólisis que se presenta

c. Por la fiebre y la diarrea.

d. Por la erupción cutánea, urticaria

Respuesta correcta

La respuesta correcta es: Por la presencia de adenomegalia, glomerulonefrítis, vasculitis

Pregunta 7 Correcta Puntúa 1,00 sobre 1,00

Dos hombres reclaman en un juzgado la paternidad de un niño de grupo O. La madre es grupo A,


mientras que el posible padre 1 es grupo B, y el padre 2 es de grupo AB. ¿Cuál de los dos es el padre?

Seleccione una:

a. Ninguno de los dos

b. Se cometió un error

c. Padre 2 (Grupo AB)

d. Padre 1 (grupo B)

Respuesta correcta

La respuesta correcta es: Padre 1 (grupo B)


Pregunta 8 Correcta Puntúa 1,00 sobre 1,00

¿Cuál es el grupo sanguíneo Donante Universal, pero no puede recibir de ningún otro grupo?

Seleccione una:

a. Grupo O +

b. Grupo AB –

c. Grupo AB+

d. Grupo O -

Respuesta correcta

La respuesta correcta es: Grupo O -

Pregunta 9 Correcta Puntúa 1,00 sobre 1,00

Una familia (1) reclaman que su bebé que le entregaron en la maternidad no es su hijo, y que el suyo es
el bebé que tiene la familia (2), que lo niegan, Se estudian sus grupos sanguíneos, con estos resultados.
Familia 1: madre A, Padre O, bebé O, y la Familia 2: madre AB, padre O, y bebé A. ¿Qué familia tiene la
razón?

Seleccione una:

a. Familia 2

b. Familia 1

c. Hay un error en las dos familias.

d. Ninguna

Respuesta correcta

La respuesta correcta es: Familia 2


Pregunta 10 Correcta Puntúa 1,00 sobre 1,00

Una pareja ambos del grupo O . ¿Qué posibilidad tiene de tener un hijo con este grupo?

Seleccione una:

a. 25% de probabilidad de tener un grupo B

b. La posibilidad 100% un hijo grupo O

c. 25% de posibilidad de tener un hijo grupo A

d. Otro 25% del grupo AB

Respuesta correcta

La respuesta correcta es: La posibilidad 100% un hijo grupo O

◄ Trombosis Ir a...
3
IVAN FERNANDO PILLAJO TENELEMA

Área personal  Mis cursos  Quito  CIENCIAS DE LA SALUD  MEDICINA - PRESENCIAL  ABR 2021 - AGO 2021
 INVESTIGACION I-HEMATO-TEORIA - Prl: MD NVA Pen: 961 Per:ABR 2021 - AGO 2021 Car:MEDICINA - PRESENCIAL  28
de junio - 4 de julio  Evaluación 2 Investigación

Comenzado el viernes, 2 de julio de 2021, 10:45


Estado Finalizado
Finalizado en viernes, 2 de julio de 2021, 11:00
Tiempo empleado 14 minutos 19 segundos
Puntos 8,00/10,00
Calificación 3,20 de 4,00 (80%)

Pregunta 1 Correcta Puntúa 1,00 sobre 1,00

Un joven de 14 años con signos de varicela. ¿Cuál de estos medicamentos está contraindicado su uso?

Seleccione una:

a. Paracetamol

b. Ibuprofeno

c. Aciclovir

d. Ácido acetíl salicílico

Respuesta correcta

La respuesta correcta es: Ácido acetíl salicílico


Pregunta 2 Correcta Puntúa 1,00 sobre 1,00

Después de una transfusión, puede ocurrir una reacción adversa, y estas pueden ser inmediatas o
tardías. ¿Cuál de estas es una reacción transfusional tardía?

Seleccione una:

a. Enfermedad del suero

b. Fiebre y escalofrío

c. Reacción alérgica

d. Reacción hemolítica aguda

Respuesta correcta

La respuesta correcta es: Enfermedad del suero

Pregunta 3 Correcta Puntúa 1,00 sobre 1,00

La éstasis según la Triada de Virchow, produce disfunción endotelial, porque ocasiona un flujo
sanguíneo lento y no llegan los factores anticoagulante y reparadores de tejidos. ¿A quién favorece esto
en favor a las trombosis?

Seleccione una:

a. A los trombos venosos

b. A los trombos arteriales

c. A la fibrinólisis

d. A los anticoagulantes

Respuesta correcta

La respuesta correcta es: A los trombos venosos


Pregunta 4 Incorrecta Puntúa 0,00 sobre 1,00

¿En cuál de las enfermedades siguientes sería más probable encontrar un tiempo de protrombina (TP)
anormal?

Seleccione una:

a. Enfermedad de Von Willebrand

b. Coagulación intravascular diseminada CID

c. Hemofilia A

d. Hemofilia B

Respuesta incorrecta.

La respuesta correcta es: Coagulación intravascular diseminada CID

Pregunta 5 Incorrecta Puntúa 0,00 sobre 1,00

Un paciente con una angina inestable por un trombo, en una arteria coronaria. ¿Cuál de estos
medicamentos utlilizarías de forma profiláctica?

Seleccione una:

a. Fibrinolítico

b. Heparina de bajo peso Molecular.

c. Aspirina

d. Warfarina Sódica

Respuesta incorrecta.

La respuesta correcta es: Aspirina


Pregunta 6 Correcta Puntúa 1,00 sobre 1,00

Las indicaciones de concentrado plaquetario, se definen por la clínica de sangrado. ¿Cuál de estas
condiciones tiene indicación profilactica de concentrado plaquetario?

Seleccione una:

a. Esplenectomía en PTI

b. Leucemias agudas post quimioterápia

c. Hipoplasia medular severa

d. Leucemias agudas post radiación

Respuesta correcta

La respuesta correcta es: Esplenectomía en PTI

Pregunta 7 Correcta Puntúa 1,00 sobre 1,00

Anticoagulante, administrado por vía subcutánea, tanto en la profilaxis como en el tratamiento de las
enfermedades tromboembólicas. ¿Cuál anticoagulante?

Seleccione una:

a. HBPM

b. Fibrinolítico

c. Warfarina Sódica

d. Aspirina

Respuesta correcta

La respuesta correcta es: HBPM


Pregunta 8 Correcta Puntúa 1,00 sobre 1,00

Un paciente alcoholico crónico con una desnutrición severa, presenta edema generalizado tipo
anasarca, se le indica cumplir Albumina. ¿Porqué?

Seleccione una:

a. Estimula la célula endotelial en sus funciones.

b. Hace que las plaquetas actúen más activas

c. .Para reponer las proteínas y presión coloidosmótica

d. Mejora la función hepática

Respuesta correcta

La respuesta correcta es: .Para reponer las proteínas y presión coloidosmótica

Pregunta 9 Correcta Puntúa 1,00 sobre 1,00

¿Cuál de estos fármacos tiene acción potenciadora de los anticoagulantes?

Seleccione una:

a. Rifampicina

b. Vitamina K

c. Clopidogrel

d. Barbitúrico

Respuesta correcta

La respuesta correcta es: Clopidogrel


Pregunta 10 Correcta Puntúa 1,00 sobre 1,00

¿Responda cuál anticoagulante está contraindicado en el embarazo?

Seleccione una:

a. Las nuevas heparinas

b. Heparina no fraccionada (HNF)

c. La warfarina sódica

d. Heparina de bajo peso molecular (HBPM)

Respuesta correcta

La respuesta correcta es: La warfarina sódica

◄ Evaluación 1 Investigación I Ir a...


Alan Chapi

Área personal  Mis cursos  Quito  CIENCIAS DE LA SALUD  MEDICINA - PRESENCIAL  OCT 2020 - FEB 2021
 INVESTIGACION I-HEMATO-TEORIA - Prl: MD NVA Pen: 961 Per:OCT 2020 - FEB 2021 Car:MEDICINA - PRESENCIAL  30
de noviembre - 6 de diciembre  Evaluación 1 Investigación 1/12/20

Comenzado el martes, 1 de diciembre de 2020, 11:40


Estado Finalizado
Finalizado en martes, 1 de diciembre de 2020, 11:51
Tiempo empleado 11 minutos 28 segundos
Cali cación 10,00 de 10,00 (100%)

Pregunta 1 Correcta Puntúa 1,00 sobre 1,00

Un niño con pancitopenia, alteraciones genéticas, presencia de malformaciones esqueléticas,


hipogonadismo. ¿Cuál diagnóstico se plantea?

Seleccione una:

a. Anemia de Fanconi

b. Anemia congénita por defecto de membrana

c. Anemia megaloblastica por dé cit de vitamina B12

d. Anemia Perniciosa

Respuesta correcta

La respuesta correcta es: Anemia de Fanconi

/
Pregunta 2 Correcta Puntúa 1,00 sobre 1,00

¿Cuál es la reacción Transfusional más frecuente que se presenta durante una transfusión sanguínea?

Seleccione una:

a. La hemolítica crónica

b. La alérgica, urticaria

c. La reacción febril no hemolítica

d. La hemolítica aguda

Respuesta correcta

La respuesta correcta es: La reacción febril no hemolítica

Pregunta 3 Correcta Puntúa 1,00 sobre 1,00

. Paciente adulto con una anemia sintomática. ¿Cuál es la cifra límite de hemoglobina aceptada para ser
transfundido?

Seleccione una:

a. Hemoglobina 10 gr/dl

b. No hay límite

c. Hemoglobina 8 gr/dl

d. Hemoglobina en 7 gr/ dl

Respuesta correcta

La respuesta correcta es: Hemoglobina en 7 gr/ dl

/
Pregunta 4 Correcta Puntúa 1,00 sobre 1,00

Cuál es el grupo sanguíneo que se identi ca como receptor Universal?

Seleccione una:

a. Grupo O +

b. Grupo O -

c. El grupo AB –

d. El grupo AB +

Respuesta correcta

La respuesta correcta es: El grupo AB +

Pregunta 5 Correcta Puntúa 1,00 sobre 1,00

Unas reacciones alérgicas tipo urticaria o ana lácticas que reincide. ¿Qué estudio debemos pedir a ese
paciente?

Seleccione una:

a. Anticuerpo Ig A por de ciencia.

b. Fosfatasas alcalina

c. Deshidrogenasa Láctica (DHL)

d. Proteína C in amatoria

Respuesta correcta

La respuesta correcta es: Anticuerpo Ig A por de ciencia.

/
Pregunta 6 Correcta Puntúa 1,00 sobre 1,00

.¿Cuándo se habla de reacción transfusional aguda?

Seleccione una:

a. Aquella que ocurre durante la transfusión hasta 24 horas

b. Después de las 24 hora

c. A los 7 días después

d. La que ocurre durante la transfusión

Respuesta correcta

La respuesta correcta es: La que ocurre durante la transfusión

Pregunta 7 Correcta Puntúa 1,00 sobre 1,00

¿Qué estudios hacen diferencia entre las hemólisis intravascular de las hemólisis extravasculares?

Seleccione una:

a. La hemoglobinuria y haptoglobina ausente

b. Bilirrubina directa aumentada

c. La ictericia con sus pigmentos biliares aumentados

d. Bilirrubina indirecta aumentada

Respuesta correcta

La respuesta correcta es: La hemoglobinuria y haptoglobina ausente

/
Pregunta 8 Correcta Puntúa 1,00 sobre 1,00

Dos hombres reclaman en un juzgado la paternidad de un niño de grupo O. La madre es grupo A,


mientras que el posible padre 1 es grupo B, y el padre 2 es de grupo AB. ¿Cuál de ellos es el padre?

Seleccione una:

a. Se cometió un error

b. Padre 2

c. Ninguno de los dos

d. Padre 1

Respuesta correcta

La respuesta correcta es: Padre 1

Pregunta 9 Correcta Puntúa 1,00 sobre 1,00

. La hemólisis intravascular se presenta en algunas anemias graves de evolución agudas. ¿En cuál de
estos cuadros clínico se presenta?

Seleccione una:

a. En las anemias de proceso crónico

b. Síndrome urémico hemolítico

c. En la anemia perniciosa

d. En todas las anemias congénitas

Respuesta correcta

La respuesta correcta es: Síndrome urémico hemolítico

/
Pregunta 10 Correcta Puntúa 1,00 sobre 1,00

Un paciente con heridas por quemaduras en 40% de su área corporal, presenta un edema marcado, se
le cumple albumina. ¿Cuál es la indicación de la albumina?

Seleccione una:

a. . Para reponer las proteínas y la presión coloidosmótica, mejorando el edema

b. Para cicatrizar mejor las heridas

c. Oxigenar mejor el tejido

d. Para mejorar las infecciones de las heridas

Respuesta correcta

La respuesta correcta es: . Para reponer las proteínas y la presión coloidosmótica, mejorando el edema

◄ Linfomas Ir a...

/
Investigación Hematología lección 1 y 2
Lección 1
Pregunta 1
Un niño con pancitopenia, alteraciones genéticas, presencia de malformaciones
esqueléticas, hipogonadismo. ¿Cuál diagnóstico se plantea?
Seleccione una:

a. Anemia de Fanconi

b. Anemia congénita por defecto de membrana

c. Anemia megaloblastica por déficit de vitamina B12

d. Anemia Perniciosa

Pregunta 2
¿Cuál es la reacción Transfusional más frecuente que se presenta durante
una transfusión sanguínea?
Seleccione una:

a. La hemolítica crónica

b. La alérgica, urticaria

c. La reacción febril no hemolítica

d. La hemolítica aguda

Pregunta 3
Paciente adulto con una anemia sintomática. ¿Cuál es la cifra límite de hemoglobina
aceptada para ser transfundido?
Seleccione una:

a. Hemoglobina 10 gr/dl

b. No hay límite

c. Hemoglobina 8 gr/dl

d. Hemoglobina en 7 gr/ dl

Pregunta 4
Enunciado de la pregunta
Cuál es el grupo sanguíneo que se identifica como receptor Universal?
Seleccione una:

a. Grupo O +

b. Grupo O -

c. El grupo AB –

d. El grupo AB +
Pregunta 5
Unas reacciones alérgicas tipo urticaria o anafilácticas que reincide. ¿Qué estudio
debemos pedir a ese paciente?
Seleccione una:

a. Anticuerpo Ig A por deficiencia.

b. Fosfatasas alcalina

c. Deshidrogenasa Láctica (DHL)

d. Proteína C inflamatoria

Pregunta 6
Enunciado de la pregunta
.¿Cuándo se habla de reacción transfusional aguda?
Seleccione una:

a. Aquella que ocurre durante la transfusión hasta 24 horas

b. Después de las 24 hora

c. A los 7 días después

d. La que ocurre durante la transfusión


Retroalimentación
Respuesta correcta
La respuesta correcta es: Aquella que ocurre durante la transfusión hasta 24 horas

Pregunta 7
¿Qué estudios hacen diferencia entre las hemólisis intravascular de las hemólisis
extravasculares?
Seleccione una:

a. La hemoglobinuria y haptoglobina ausente

b. Bilirrubina directa aumentada

c. La ictericia con sus pigmentos biliares aumentados

d. Bilirrubina indirecta aumentada

Pregunta 8
Dos hombres reclaman en un juzgado la paternidad de un niño de grupo O. La madre
es grupo A, mientras que el posible padre 1 es grupo B, y el padre 2 es de grupo AB.
¿Cuál de ellos es el padre?
Seleccione una:

a. Se cometió un error

b. Padre 2

c. Ninguno de los dos

d. Padre 1
Pregunta 9
La hemólisis intravascular se presenta en algunas anemias graves de evolución
agudas. ¿En cuál de estos cuadros clínico se presenta?
Seleccione una:

a. En las anemias de proceso crónico

b. Síndrome urémico hemolítico

c. En la anemia perniciosa

d. En todas las anemias congénitas

Pregunta 10
Enunciado de la pregunta
Un paciente con heridas por quemaduras en 40% de su área corporal, presenta un
edema marcado, se le cumple albumina. ¿Cuál es la indicación de la albumina?
Seleccione una:

a. . Para reponer las proteínas y la presión coloidosmótica, mejorando el edema

b. Para cicatrizar mejor las heridas

c. Oxigenar mejor el tejido

d. Para mejorar las infecciones de las heridas

Lección 2
Pregunta 1
. La coluria es un signo urinario por la presencia de hemoglobinuria. ¿En cuál de estas
patologías se puede
ver?
Seleccione una:

a. Talasemia

b. Síndrome Urémico Hemolítico (SUH).

c. Esferocitosis

d. Anemia Falciforme

Pregunta 2
Un paciente adulto con anemia normocítica, normocrómica y reticulocitos elevados,
¿Cuál de estas patologías podrían presentar este cuadro clínico?
Seleccione una:

a. Pielonefritis

b. Lupus Eritematoso Sistémico

c. Hemorragia digestiva superior

d. Insuficiencia Renal Crónica


Retroalimentación
Respuesta incorrecta.
La respuesta correcta es: Hemorragia digestiva superior

Pregunta 3
. Pre escolar de 5 años, diarrea con sangre, vómitos. Presenta cuadro de anemia
aguda, ictericia, hepatoesplenomegalia, y disminución de la filtración glomerular, con
coluria. ¿Cuál es su posibilidad diagnóstica?
Seleccione una:

a. Coagulación Intravascular diseminada (CID).

b. Síndrome urémico hemolítico

c. Insuficiencia Renal Crónica

d. Síndrome de mala absorción

Pregunta 4
. Paciente con Anemia Reticulocitos bajo, IPR < de 1, además presenta
trombocitopenia y leucopenia es decir tiene una Pancitopenia. ¿Qué diagnóstico
tendría este paciente?
Seleccione una:

a. Síndrome Urémico Hemolítico (SUH).

b. Hemorragia digestiva

c. Hipoplasia medular

d. Anemia Falciforme

Pregunta 5
. La hipoplasia medular, es una patología donde se produce un daño estructural de la
médula ósea, por diferentes causas y se traduce por una disminución de las tres líneas
celulares. ¿Cuál de estas opciones nos orienta a una hipoplasia severa?
Seleccione una:

a. Neutrófilos < de 0.5 x10n9/, reticulocitos corregidos <de 1%

b. Neutrófilos > de 1000, y reticulocitos en 2%

c. La presencia de esplenomegalia y hepatomegalia

d. Presencia de anemia, fiebre y petequias

Pregunta 6
Paciente con Anemia Reticulocitos bajo, IPR < de 1, además presenta trombocitopenia
y leucopenia es decir tiene una Pancitopenia. ¿Qué conducta seguir para llegar al
diagnóstico?
Seleccione una:

a. Hierro sérico

b. Ferritina

c. Aspirado- biopsia de Médula Ósea


d. Transferrina

Pregunta 7
.La hipostenuria, es la incapacidad del riñón de concentrar la orina, ¿En cuál de estas
patología se presenta?
Seleccione una:

a. Talasemia minor

b. Talasemia mayor

c. Anemia Falciforme

d. Esferocitosis

Pregunta 8
. La Anemia de Cooley, es una anemia congénita, con las características de: palidez,
ictericia, baja talla, hemocromatosis, esplenomegalia. ¿En cuál anemia se presenta?
Seleccione una:

a. Talasemia minor

b. Talasemia mayor

c. Esferocitosis

d. Enfermedad de Gilbert (déficit enzimático)

Pregunta 9
Cuál de las siguientes anemias, tienen estas características: ¿Congénitas,
intracorpuscular, crónica?
Seleccione una:

a. Anemia esferocitica

b. Anemia de proceso crónico.

c. Aplasia medular

d. Anemia megaloblastica,

Pregunta 10
Las anemias arregenerativas, son anemias con baja respuesta de la médula, por tener
un IPR < de 2. ¿Cuál de estas anemias es arregenerativas?
Seleccione una:

a. Hiperesplenismo.

b. Hemorragia aguda

c. Anemia Falciforme

d. Anemia Ferropénica
4
IVAN FERNANDO PILLAJO TENELEMA

Área personal  Mis cursos  Quito  CIENCIAS DE LA SALUD  MEDICINA - PRESENCIAL  ABR 2021 - AGO 2021
 INVESTIGACIÓN I - Prl: MD NVA Pen: 961 Per:ABR 2021 - AGO 2021 Car:MEDICINA - PRESENCIAL  23 de agosto - 29 de
agosto  PRIMER EXAMEN PARCIAL INVESTIGACIÓN I

Comenzado el jueves, 26 de agosto de 2021, 09:10


Estado Finalizado
Finalizado en jueves, 26 de agosto de 2021, 09:53
Tiempo empleado 43 minutos 42 segundos
Calificación 3,68 de 4,00 (92%)

Pregunta 1 Correcta Puntúa 0,16 sobre 0,16

¿Cuál de estas patologías con manifestaciones de sangrado purpúrico, se identifica su frotis de sangre
periférica por la presencia de plaquetas gigantes?

Seleccione una:

a. Enfermedad de Von Willebrand

b. Enfermedad de Bernard Soulier

c. Purpura trombocitopénica trombótica PTT

d. Síndrome urémico hemolítico SUH

La respuesta correcta es: Enfermedad de Bernard Soulier


Pregunta 2 Correcta Puntúa 0,16 sobre 0,16

¿Cuál de las siguientes alternativas es el tratamiento sistémico de elección en impétigo contagioso?

Seleccione una:

a. Ampicilina

b. Azitromicina

c. Dicloxacilina

d. Eritromicina

La respuesta correcta es: Dicloxacilina

Pregunta 3 Correcta Puntúa 0,16 sobre 0,16

¿Cuál no se considera causa para azotemia prerrenal?

Seleccione una:

a. Vómito a repetición

b. Hemorragia digestiva alta

c. Hiperplasia prostática

d. Diarrea aguda

La respuesta correcta es: Hiperplasia prostática


Pregunta 4 Correcta Puntúa 0,16 sobre 0,16

¿Cuál no se considera un tratamiento para hiperpotasemia?

Seleccione una:

a. Furosemida

b. Solución polarizante

c. Gluconato de calcio

d. Bicarbonato de calcio

La respuesta correcta es: Bicarbonato de calcio

Pregunta 5 Correcta Puntúa 0,16 sobre 0,16

Cuántos mEq de Na contiene 500 ml de Cloruro de sodio al 0.09%?

Seleccione una:

a. 250 mEq/L

b. 130 mEq/L

c. 154 mEq/L

d. 132 mEq/L

La respuesta correcta es: 154 mEq/L


Pregunta 6 Correcta Puntúa 0,16 sobre 0,16

El concentrado plaquetario constituye el derivado con mayor riesgo de contaminación. ¿Cuál es la


causa?

Seleccione una:

a. Debido a mantener a -20 grados en congelación

b. Se mantiene a 6 grados en nevera

c. Por estar a 26 grados de temperatura en el ambiente

d. Se encuentra a 35 grados de temperatura.

La respuesta correcta es: Por estar a 26 grados de temperatura en el ambiente

Pregunta 7 Correcta Puntúa 0,16 sobre 0,16

El índíce de producción de reticulocitos (IPR) es un estudio que mide la capacidad de respuesta de la


medula. Si tenemos un IPR menor de 2. ¿Cuál de estas anemias presentan un IPR menor de 2?

Seleccione una:

a. Anemia hemolítica

b. Hemorragia aguda

c. Hiperesplenismo

d. Hipoplasia medular.

La respuesta correcta es: Hipoplasia medular.


Pregunta 8 Correcta Puntúa 0,16 sobre 0,16

El Síndrome de Alport, que nefropatía produce?

Seleccione una:

a. Glomeruloesclerosis segmentaria focal

b. Glomerulonefritis membranoproliferativa

c. Granulomatosis de Wegener

d. Nefropatía por IgA

La respuesta correcta es: Glomeruloesclerosis segmentaria focal

Pregunta 9 Correcta Puntúa 0,16 sobre 0,16

En que valores se encuentra una hiponatremia leve?

Seleccione una:

a. 105 a 115 mEq/l

b. 115 a 125 mEq/l

c. 95 a 105 mEq/l

d. 125 a 135 mEq/l

La respuesta correcta es: 125 a 135 mEq/l


Pregunta 10 Incorrecta Puntúa 0,00 sobre 0,16

Este paciente con quemaduras le van a cumplir posteriormente plasma, el paciente es grupo O (-) Banco
de sangre envía un plasma. ¿Cuál de estos podemos cumplir?

Seleccione una:

a. Plasma O +

b. Plasma A +

c. Plasma B +

d. Plasma A B -

La respuesta correcta es: Plasma A B -

Pregunta 11 Correcta Puntúa 0,16 sobre 0,16

La anemia hemolítica microangiopática es un cuadro clínico, que se presenta en patologías como la


coagulación intravascular diseminada, la presencia de esquitocitos en el frotis periférico es una imagen
característica. ¿Qué representa?

Seleccione una:

a. Hematíes fragmentados

b. Fibrina fragmentada

c. Reticulocitos

d. Buena respuesta de la médula

La respuesta correcta es: Hematíes fragmentados


Pregunta 12 Correcta Puntúa 0,16 sobre 0,16

La furosemida ¿qué categoría de riesgo en gestantes tiene?

Seleccione una:

a. A

b. B

c. C

d. D

La respuesta correcta es: C

Pregunta 13 Correcta Puntúa 0,16 sobre 0,16

La Heparina no fraccionada (HNF) su uso puede causar algunas complicaciones por su acción
anticoagulante, sobre cuál factor de coagulación actúa principalmente?

Seleccione una:

a. Factor II Protrombina

b. Factor V

c. Factor I

d. Factor VIII

La respuesta correcta es: Factor II Protrombina


Pregunta 14 Correcta Puntúa 0,16 sobre 0,16

La obstrucción ureteral bilateral crónica se caracteriza por?

Seleccione una:

a. Anuria

b. Diuresis posterior a la obstrucción

c. Dolor renal

d. Hipertensión

La respuesta correcta es: Hipertensión

Pregunta 15 Correcta Puntúa 0,16 sobre 0,16

Los productos de degradación de la fibrina (PDF) podemos medirlos a través de los Dímeros D, su
contaje elevado mayor de1000 (V.N 500) indicaría:

Seleccione una:

a. Neumonia.

b. Derrame Pleural

c. Tromboembolismo pulmonar

d. Hipertensión pulmonar

La respuesta correcta es: Tromboembolismo pulmonar


Pregunta 16 Incorrecta Puntúa 0,00 sobre 0,16

Paciente de sexo femenino de 30 años de edad que presenta desde la infancia placas
eritematodescamativas de bordes bien definidos en codos, y rodilla con descamación intensa y prurito
moderado, además presenta eritema y descamación profusa en cuero cabelludo. ¿Cuál de las siguientes
alternativas corresponde al diagnóstico clínico más probable?

Seleccione una:

a. Dermatitis atópica + dermatitis seborreica de cuero cabeludo

b. Psoriasis + dermatitis seborreica de cuero cabelludo

c. Dermatitis seborreica + tiña corporis

d. Psoriasis

La respuesta correcta es: Psoriasis

Pregunta 17 Correcta Puntúa 0,16 sobre 0,16

Paciente de sexo masculino, de 48 años de edad que presenta una tumefacción eritematosa de bordes
mal definidos, muy dolorosa a la palpación en tórax posterior de consistencia semidura, con un
pequeño orificio central a través del cual drena material purulento de mal olor de 8 días de evolución.
¿Cuál de las siguientes alternativas indica las bacterias que con más frecuencia ocasionan este cuadro?

Seleccione una:

a. Estafilococo aureus, clostridium, pseudomona

b. Estafilococo aureus, bacterias anaerobias, gramnegativos

c. Estreptococo beta hemolítico del grupo a, neumococo, eschericia coli

d. Estreptococo beta hemolítico del grupo a, pseudomona, h. Influenza

La respuesta correcta es: Estafilococo aureus, bacterias anaerobias, gramnegativos


Pregunta 18 Correcta Puntúa 0,16 sobre 0,16

Paciente de sexo masculino, de 48 años de edad que presenta una tumefacción eritematosa de bordes
mal definidos, muy dolorosa a la palpación en tórax posterior de consistencia semidura, con un
pequeño orificio central a través del cual drena material purulento de mal olor de 8 días de evolución.
¿Cuál de las siguientes alternativas corresponde al diagnóstico más probable?

Seleccione una:

a. Celulitis

b. Forúnculo

c. Foliculitis profunda

d. Erisipela

La respuesta correcta es: Forúnculo

Pregunta 19 Correcta Puntúa 0,16 sobre 0,16

Paciente de sexo masculino, de 48 años de edad que presenta una tumefacción eritematosa de bordes
mal definidos, muy dolorosa a la palpación en tórax posterior de consistencia semidura, con un
pequeño orificio central a través del cual drena material purulento de mal olor de 8 días de evolución.
¿Cuál de las siguientes alternativas indica los antibióticos de elección en este caso?

Seleccione una:

a. Dicloxacilina y linezolid

b. Dicloxacilina y trimetroprim sulfametoxazol

c. Ceftriaxona y gentamicina

d. Amoxicilina + ácido clavulanico

La respuesta correcta es: Dicloxacilina y trimetroprim sulfametoxazol


Pregunta 20 Correcta Puntúa 0,16 sobre 0,16

¿Qué valor define anuria?

Seleccione una:

a. 0 a 100 ml

b. 1000 a 2000 ml

c. 500 a 1000 ml

d. 100 a 400 ml

La respuesta correcta es: 0 a 100 ml

Pregunta 21 Correcta Puntúa 0,16 sobre 0,16

Según Harrison, los factores que predisponen a las mujeres a padecer cistitis, aumentan el peligro de:

Seleccione una:

a. Uretritis

b. Absceso perirenal

c. Vaginitis

d. Pielonefritis

La respuesta correcta es: Pielonefritis


Pregunta 22 Correcta Puntúa 0,16 sobre 0,16

Si el cálculo se ubica en la porción inferior del uréter, el dolor puede irradiarse en mujeres hacia?

Seleccione una:

a. Útero

b. Pubis

c. Labio mayor ipsilateral

d. Miometrio

La respuesta correcta es: Labio mayor ipsilateral

Pregunta 23 Correcta Puntúa 0,16 sobre 0,16

Si se necesita el aumento de plaquetas más efectivo en una trombocitopenia severa inmune. ¿Qué
utilizas de primera línea?

Seleccione una:

a. Metilprednisolona

b. Rituximab anticuerpo monoclonal

c. Eltrombopag

d. Inmunoglobulina

La respuesta correcta es: Inmunoglobulina


Pregunta 24 Correcta Puntúa 0,16 sobre 0,16

Una reacción transfusional es una respuesta adversa a la transfusión de un derivado sanguíneo. ¿Cuál
es la reacción más frecuente?

Seleccione una:

a. Reacción febril no hemolítica

b. Reacción hemolítica crónica

c. Urticaria

d. Reacción hemolítica aguda

La respuesta correcta es: Reacción febril no hemolítica

Pregunta 25 Correcta Puntúa 0,16 sobre 0,16

Un paciente con quemaduras de un 30% de su cuerpo, presenta además de sus lesiones un edema
importante. ¿Cuál de estos derivados pedirías para ayudar a resolver el edema?

Seleccione una:

a. Inmunoglobulina

b. Albumina

c. Concentrado plaquetario

d. Concentrado globular

La respuesta correcta es: Albumina

◄ Registro de asistencia Ir a...


4
IVAN FERNANDO PILLAJO TENELEMA

Área personal  Mis cursos  Quito  CIENCIAS DE LA SALUD  MEDICINA - PRESENCIAL  ABR 2021 - AGO 2021
 INVESTIGACIÓN I - Prl: MD NVA Pen: 961 Per:ABR 2021 - AGO 2021 Car:MEDICINA - PRESENCIAL  06 al 11 de
septiembre  SEGUNDO EXAMEN PARCIAL INVESTIGACIÓN I

Comenzado el martes, 7 de septiembre de 2021, 13:10


Estado Finalizado
Finalizado en martes, 7 de septiembre de 2021, 13:39
Tiempo empleado 28 minutos 41 segundos
Calificación 4,00 de 4,00 (100%)

Pregunta 1 Correcta Puntúa 0,16 sobre 0,16

A qué grupo poblacional no restringiría la vacuna de la gripe:

Seleccione una:

a. Personas con trastornos crónicos metabólicos

b. Personas alérgicas a las proteínas del huevo

c. Personas con antecedente de Guillain Barré

d. Niños con antecedentes de síndrome de Reye

La respuesta correcta es: Personas alérgicas a las proteínas del huevo


Pregunta 2 Correcta Puntúa 0,16 sobre 0,16

A un joven asintomático de 14 años cuyo padre acaba de ser diagnosticado de Tuberculosis Pulmonar se
le realiza un Mantoux, con resultado negativo. ¿Qué actitud es la adecuada?

Seleccione una:

a. Iniciar quimioprofilaxis y repetir la prueba cutánea a los 3 meses.

b. Repetir la prueba cutánea al mes.

c. Realizarle una radiografía de tórax.

d. Tranquilizarle, ya que no tiene ningún riesgo.

La respuesta correcta es: Iniciar quimioprofilaxis y repetir la prueba cutánea a los 3 meses.

Pregunta 3 Correcta Puntúa 0,16 sobre 0,16

Con respecto a la infección por gripe indiqué la premisa verdadera:

Seleccione una:

a. puede causar una neumonía viral primaria

b. El virus solamente tiene mutaciones puntuales

c. casi siempre se asocia con febrícula, o sin fiebre sino con tos seca

d. la Clínica generalmente se asocia a trastornos respiratorios altos

La respuesta correcta es: puede causar una neumonía viral primaria


Pregunta 4 Correcta Puntúa 0,16 sobre 0,16

¿Cuál arritmia es frecuente en pacientes con hipertiroidismo no controlado?

Seleccione una:

a. bloqueo de rama izquierda del Haz de Hiss

b. bradicardia sinusal

c. fibrilación auricular

d. bloqueo aurículo-ventricular de I grado

La respuesta correcta es: fibrilación auricular

Pregunta 5 Correcta Puntúa 0,16 sobre 0,16

¿Cuál de los siguientes exámenes es útil para el diagnóstico de feocromocitoma?

Seleccione una:

a. TSH

b. glicemia

c. metanefrinas en orina de 24 horas

d. Prueba de supresión con dexametasona

La respuesta correcta es: metanefrinas en orina de 24 horas


Pregunta 6 Correcta Puntúa 0,16 sobre 0,16

¿Cuál de los siguientes fármacos puede producir hiperprolactinemia?

Seleccione una:

a. metoclopramida

b. levortiroxina

c. furosemida

d. metformina

La respuesta correcta es: metoclopramida

Pregunta 7 Correcta Puntúa 0,16 sobre 0,16

Cuál síndrome genético es común en la insuficiencia aórtica:

Seleccione una:

a. Síndrome de Rastelli

b. Anomalía de Ebstein

c. Síndrome de Marfán

d. Síndrome de Eisenmenger

La respuesta correcta es: Síndrome de Marfán


Pregunta 8 Correcta Puntúa 0,16 sobre 0,16

Cuánto es la dosis máxima de dinitrato de isosorbide de liberación prolongada en angina de pecho:

Seleccione una:

a. 160 mg

b. 300 mg

c. 120 mg

d. 40 mg

La respuesta correcta es: 300 mg

Pregunta 9 Correcta Puntúa 0,16 sobre 0,16

Cuánto es la dosis sublingual de nitroglicerina en angina de pecho:

Seleccione una:

a. 3 a 6 mg

b. 1.3 a 1.6 mg

c. 0.3 a 0.6 mg

d. 0.8 a 1.0 mg

La respuesta correcta es: 0.3 a 0.6 mg


Pregunta 10 Correcta Puntúa 0,16 sobre 0,16

El interferón es un medicamento útil para las siguientes infecciones excepto:

Seleccione una:

a. Hepatitis B

b. HPV

c. VIH

d. Sarcoma de Kaposi

La respuesta correcta es: VIH

Pregunta 11 Correcta Puntúa 0,16 sobre 0,16

En el análisis de líquido cefalorraquídeo usted encuentra linfocitos aumentados, glucosa disminuida,


usted podría pensar en que la etiología es:

Seleccione una:

a. viral

b. micobacterias

c. bacteriana

d. parasitaria

La respuesta correcta es: micobacterias


Pregunta 12 Correcta Puntúa 0,16 sobre 0,16

En el síndrome de Cushing cómo se encuentran los valores de glicemia?

Seleccione una:

a. incrementados

b. no tiene relación la glicemia con el Síndrome de Cushing

c. disminuidos

d. normales

La respuesta correcta es: incrementados

Pregunta 13 Correcta Puntúa 0,16 sobre 0,16

En el Síndrome de Secreción inadecuada de hormona antidiurética existe:

Seleccione una:

a. hiperpotasemia

b. hipernatremia

c. aumento de cortisol

d. hiponatremia y reabsorción excesiva de agua a nivel renal

La respuesta correcta es: hiponatremia y reabsorción excesiva de agua a nivel renal


Pregunta 14 Correcta Puntúa 0,16 sobre 0,16

En la clasificación de FOD cuál considera usted como urgente y debe ser tratada de inmediato:

Seleccione una:

a. FOD Clásico

b. FOD neutropénico

c. FOD Nosocomial

d. FOD asociado a HIV

La respuesta correcta es: FOD neutropénico

Pregunta 15 Correcta Puntúa 0,16 sobre 0,16

En pacientes con enfermedad de Addison no controlada. ¿Cómo están los niveles de sodio?

Seleccione una:

a. aumentados

b. Normales

c. Disminuidos

d. no tiene relación la natremia con la Enfermedad de Addison

La respuesta correcta es: Disminuidos


Pregunta 16 Correcta Puntúa 0,16 sobre 0,16

En qué edad es más frecuente el diagnóstico de hipotiroidismo?

Seleccione una:

a. adolescentes

b. más de 60 años

c. 40 años

d. infancia

La respuesta correcta es: más de 60 años

Pregunta 17 Correcta Puntúa 0,16 sobre 0,16

La tuberculosis osteoarticular ataca principalmente a columna a lo que se denomina:

Seleccione una:

a. Mal de Pott

b. Trastorno de Jacob

c. Osteofitosis

d. Enfermedad de Wegener

La respuesta correcta es: Mal de Pott


Pregunta 18 Correcta Puntúa 0,16 sobre 0,16

Paciente masculino de 45 años quien desde hace varios años presenta, cambio en las facciones con
prominencia frontal, aumento de espacios interdentales, incremento de talla de calzado y guantes, voz
ronca, fue diagnosticado de Diabetes mellitus hace 6 meses. Qué otra patología debería ser investigada?

Seleccione una:

a. Acromegalia

b. Tumores óseos

c. hipotiroidismo

d. Síndrome de Cushing

La respuesta correcta es: Acromegalia

Pregunta 19 Correcta Puntúa 0,16 sobre 0,16

Que caracteriza a una primoinfección por herpes tipo 1:

Seleccione una:

a. trastornos de córnea y conjuntiva

b. distribución en dermatoma en especial tórax

c. pápulas y pústulas intersticio entre piel y mucosa

d. faringitis o gingivoestomatitis

La respuesta correcta es: faringitis o gingivoestomatitis


Pregunta 20 Correcta Puntúa 0,16 sobre 0,16

Qué enfermedad reumatológica causa insuficiencia aórtica:

Seleccione una:

a. Artritis reumatoidea

b. Lupus eritematoso sistémico

c. Vasculitis leucocitoclástica

d. Espondilitis anquilosante

La respuesta correcta es: Espondilitis anquilosante

Pregunta 21 Correcta Puntúa 0,16 sobre 0,16

Qué patología de las siguientes es una condición Que Define sida:

Seleccione una:

a. leucoplasia vellosa

b. candidiasis oral

c. herpes tipo 1: úlceras crónicas &gt; 1 mes

d. displasia cervical

La respuesta correcta es: herpes tipo 1: úlceras crónicas &gt; 1 mes


Pregunta 22 Correcta Puntúa 0,16 sobre 0,16

Si durante el examen físico en el área genital encuentra una úlcera lisa no purulenta bien delimitado
única superficial con linfadenopatía bilateral usted piensa en:

Seleccione una:

a. herpes tipo 2

b. sífilis

c. chancroide

d. leishmania

La respuesta correcta es: sífilis

Pregunta 23 Correcta Puntúa 0,16 sobre 0,16

Una paciente femenina de 40 años con cefalea, hemianopsia y galactorrea, debe ser investigada para
confirmar:

Seleccione una:

a. Adenoma hipofisario productor de prolactina

b. Diabetes

c. Síndrome de Cushing

d. Acromegalia

La respuesta correcta es: Adenoma hipofisario productor de prolactina


Pregunta 24 Correcta Puntúa 0,16 sobre 0,16

Un paciente con antecedentes de Trauma cráneo encefálico severo, presenta signos de hipertensión
intracraneal, poliuria mayor a 3cc/kg/h, hipernatremia e hipotensión. Cuál su diagnóstico preguntivo?

Seleccione una:

a. Síndrome de secreción inadecuada de hormona antidiurética

b. Diabetes insípida de origen central

c. Diabetes insípida nefrogénica

d. Diabetes mellitus

La respuesta correcta es: Diabetes insípida de origen central

Pregunta 25 Correcta Puntúa 0,16 sobre 0,16

Un paciente hipertenso con descompensación aguda “típica” de insuficiencia cardiaca sin sobrecarga de
volumen, que fármaco usaría:

Seleccione una:

a. Furosemida

b. Dobutamina

c. Clortalidona

d. Nitroprusiato

La respuesta correcta es: Nitroprusiato

◄ Registro de asistencia Ir a...


Cuál de los siguientes es un signo característico de las vías respiratorias de las personas que
sufren asma:
Seleccione una:

a. Infiltración eosinofílica
b. Transformación de los basófilos en macrófagos
c. Adhesión de los glóbulos rojos al endotelio
d. Infiltración basófila
Retroalimentación
La respuesta correcta es: Infiltración eosinofílica

Pregunta 2
Correcta
Puntúa 0,16 sobre 0,16

Marcar pregunta
Enunciado de la pregunta

¿Cuál es la clasificación anatómica de las bronquiectasias?


Seleccione una:
a. Cubicas, cilíndricas, alargadas
b. Lobares, arteriales, globulares
c. Vasculares, semilunares saculares

d. Cilíndricas, varicosas, saculares


Retroalimentación
La respuesta correcta es: Cilíndricas, varicosas, saculares

Pregunta 3
Correcta
Puntúa 0,16 sobre 0,16

Marcar pregunta
Enunciado de la pregunta

¿Cuál es la fisiopatología de la hemoptisis?


Seleccione una:
a. Hipervascularización de la circulación brónquica, hipertensión pulmonar y disminución de
coagulabilidad
b. Hipervascularización de la circulación brónquica, hipertensión pulmonar y neovascularización
c. Hipervascularización de la circulación pulmonar, hipertensión pulmonar y remodelación.
d. Hipervascularización de la circulación pulmonar, fiebre y regeneración alveolar
Retroalimentación
La respuesta correcta es: Hipervascularización de la circulación brónquica, hipertensión pulmonar y
neovascularización

Pregunta 4
Correcta
Puntúa 0,16 sobre 0,16

Marcar pregunta
Enunciado de la pregunta

Cuál es una causa de taquicardia sinusal fisiológica:


Seleccione una:

a. Feocromocitoma
b. Sedenterismo
c. Hipotiroidismo
d. Hipertensión pulmonar
Retroalimentación
La respuesta correcta es: Feocromocitoma

Pregunta 5
Correcta
Puntúa 0,16 sobre 0,16

Marcar pregunta
Enunciado de la pregunta

Cuál síndrome genético es común en la insuficiencia aórtica:


Seleccione una:
a. Síndrome de Rastelli
b. Síndrome de Eisenmenger

c. Síndrome de Marfán
d. Anomalía de Ebstein
Retroalimentación
La respuesta correcta es: Síndrome de Marfán

Pregunta 6
Correcta
Puntúa 0,16 sobre 0,16

Marcar pregunta
Enunciado de la pregunta

Cuánto es la dosis intravenosa de sostén para digoxina en arritmias:


Seleccione una:
a. 0.200-0.250 mg/día

b. 0.125-0.250 mg/día
c. 0.145-0.250 mg/día
d. 0.175-0.250 mg/día
Retroalimentación
La respuesta correcta es: 0.125-0.250 mg/día

Pregunta 7
Correcta
Puntúa 0,16 sobre 0,16

Marcar pregunta
Enunciado de la pregunta

Cuánto es la dosis sublingual de nitroglicerina en angina de pecho:


Seleccione una:
a. 1.3 a 1.6 mg

b. 0.3 a 0.6 mg
c. 3 a 6 mg
d. 0.8 a 1.0 mg
Retroalimentación
La respuesta correcta es: 0.3 a 0.6 mg

Pregunta 8
Incorrecta
Puntúa 0,00 sobre 0,16
Marcar pregunta
Enunciado de la pregunta

Cuánto es la frecuencia de administración de atenolol en cardiopatía isquémica:


Seleccione una:
a. Cada 8 horas

b. Cada 12 horas
c. Cada 24 horas
d. Cada 6 horas
Retroalimentación
La respuesta correcta es: Cada 24 horas

Pregunta 9
Correcta
Puntúa 0,16 sobre 0,16

Marcar pregunta
Enunciado de la pregunta

De los siguientes factores, cuál es el que debe estar alterado para que un paciente infectado de
tuberculosis se convierta en persona enferma:
Seleccione una:

a. Huésped: Inmunidad celular deficiente, en especial CD4


b. Medio Ambiente: Que viva en un lugar frío
c. Medio ambiente: Que haya elevada prevalencia de tuberculosis
d. Huésped: que genéticamente esté predispuesto a infectarse
Retroalimentación
La respuesta correcta es: Huésped: Inmunidad celular deficiente, en especial CD4

Pregunta 10
Correcta
Puntúa 0,16 sobre 0,16

Marcar pregunta
Enunciado de la pregunta

El hallazgo más frecuente en la radiografía de tórax en un paciente con asma es:


Seleccione una:

a. Radiografía de tórax normal.


b. Condensaciones alveolares bilaterales y difusas.
c. Engrosamiento de paredes bronquiales.
d. Hiperinsuflación pulmonar.
Retroalimentación
La respuesta correcta es: Radiografía de tórax normal.

Pregunta 11
Correcta
Puntúa 0,16 sobre 0,16

Marcar pregunta
Enunciado de la pregunta

El tratamiento de las Bronquiectasias se basa en 3 pilares, excepto:


Seleccione una:
a. Eliminar la obstrucción bronquial.

b. Revertir el remodelamiento bronquia


c. Mejorar la eliminación de las secreciones, que se consigue con una adecuada hidratación, con
fisioterapia respiratoria y drenaje postural mantenidos.
d. Controlar las infecciones con el uso de antibióticos en las agudizaciones durante 10-15 días.
Retroalimentación
La respuesta correcta es: Revertir el remodelamiento bronquia

Pregunta 12
Correcta
Puntúa 0,16 sobre 0,16

Marcar pregunta
Enunciado de la pregunta

El valor de oliguria define?


Seleccione una:
a. 0 a 50 ml
b. 500 a 1000 ml

c. 100 a 400 ml
d. 1000 a 1500 ml
Retroalimentación
La respuesta correcta es: 100 a 400 ml

Pregunta 13
Incorrecta
Puntúa 0,00 sobre 0,16

Marcar pregunta
Enunciado de la pregunta

El virus de hepatitis B, puede ser causa de algunas enfermedades como agente biológico. ¿Cuál
de estas enfermedades?
Seleccione una:
a. Anemia por déficit de hierro
b. Hipoplasia medular
c. Anemia megaloblástica

d. Anemia de proceso crónico


Retroalimentación
La respuesta correcta es: Hipoplasia medular

Pregunta 14
Correcta
Puntúa 0,16 sobre 0,16

Marcar pregunta
Enunciado de la pregunta

En cuanto al diagnóstico funcional del asma:


Seleccione una:
a. Si la relación VEF1/CVF es menor a 0.7 (patrón obstructivo) y post broncodilatador obtenemos un

aumento igual o mayor al 12% en el VEF1, nos orienta hacia diagnóstico de asma
b. Si la relación VEF1/CVF es menor a 0.7 (patrón restrictivo) y post broncodilatador obtenemos un
aumento igual o mayor al 12% en el VEF1, nos orienta hacia diagnóstico de asma
c. Si la relación VEF1/CVF es mayor 0.7 se considera patrón obstructivo, diagnosticamos asma
d. Valores espirométricos no tienen importancia en el diagnóstico del asma.
Retroalimentación
La respuesta correcta es: Si la relación VEF1/CVF es menor a 0.7 (patrón obstructivo) y post
broncodilatador obtenemos un aumento igual o mayor al 12% en el VEF1, nos orienta hacia diagnóstico
de asma

Pregunta 15
Incorrecta
Puntúa 0,00 sobre 0,16

Marcar pregunta
Enunciado de la pregunta

Existen fármacos que aumentan o disminuyen la acción de los anticoagulantes ¿Cuál de estos
fármacos aumenta su acción?
Seleccione una:
a. Aspirina
b. Rifampicina

c. Vitamina K
d. Anticonceptivo
Retroalimentación
La respuesta correcta es: Aspirina

Pregunta 16
Correcta
Puntúa 0,16 sobre 0,16

Marcar pregunta
Enunciado de la pregunta

La causa principal de un fracaso en el tratamiento de la tuberculosis pulmonar es


Seleccione una:

a. Abandono del tratamiento por el paciente


b. Resistencia secundaria a las drogas
c. Toxicidad hepática
d. Resistencia primaria a las drogas
Retroalimentación
La respuesta correcta es: Abandono del tratamiento por el paciente
Pregunta 17
Correcta
Puntúa 0,16 sobre 0,16

Marcar pregunta
Enunciado de la pregunta

La fiebre por meningitis que trastorno ácido básico produce?


Seleccione una:
a. Acidosis respiratoria
b. Acidosis metabólica
c. Alcalosis metabólica

d. Alcalosis respiratoria
Retroalimentación
La respuesta correcta es: Alcalosis respiratoria

Pregunta 18
Incorrecta
Puntúa 0,00 sobre 0,16

Marcar pregunta
Enunciado de la pregunta

La triada de Virchow está definida por lesión endotelial, hipercoagulabilidad y la inamovilidad.


¿Cuál de estas es un riesgo de hipercoagulabilidad?
Seleccione una:
a. Apoplejía lesión de columna
b. Edad avanzada

c. Cirugía
d. Estrógenos
Retroalimentación
La respuesta correcta es: Estrógenos

Pregunta 19
Correcta
Puntúa 0,16 sobre 0,16
Marcar pregunta
Enunciado de la pregunta

La trombosis venosa tiene sus características por estar constituidos por mucha fibrina y
hematíes (trombo rojo) ¿Cuál de estos factores es de riesgo para una trombosis venosa?
Seleccione una:
a. Dislipidemia
b. Diabetes

c. Estasis, inamovilidad
d. Hipertensión
Retroalimentación
La respuesta correcta es: Estasis, inamovilidad

Pregunta 20
Correcta
Puntúa 0,16 sobre 0,16

Marcar pregunta
Enunciado de la pregunta

Qué infección causa bloqueo auriculoventricular:


Seleccione una:
a. Sarampión
b. Tripanosomiasis
c. Varicela

d. Sífilis
Retroalimentación
La respuesta correcta es: Sífilis

Pregunta 21
Correcta
Puntúa 0,16 sobre 0,16

Marcar pregunta
Enunciado de la pregunta

Según Harrison, cuál es un microorganismo virulento para ITU por propagación hematógena:
Seleccione una:

a. Estafilococo aureus
b. Acinetobacter Baumani
c. Estreptococo aureus
d. Proteus mirabilis
Retroalimentación
La respuesta correcta es: Estafilococo aureus

Pregunta 22
Correcta
Puntúa 0,16 sobre 0,16

Marcar pregunta
Enunciado de la pregunta

Según Harrison, cuántas semanas se requieren para catalogar cistitis recurrente?


Seleccione una:
a. 8 semanas
b. 6 semanas
c. 4 semanas

d. 2 semanas
Retroalimentación
La respuesta correcta es: 2 semanas

Pregunta 23
Correcta
Puntúa 0,16 sobre 0,16

Marcar pregunta
Enunciado de la pregunta

Si existen una intoxicacion crónica por Sal Andrews que trastorno ácido básico se produce?
Seleccione una:
a. Acidosis metabólica
b. Alcalosis respiratoria
c. Alcalosis metabólica
d. Acidosis respiratoria
Retroalimentación
La respuesta correcta es: Alcalosis metabólica

Pregunta 24
Incorrecta
Puntúa 0,00 sobre 0,16

Marcar pregunta
Enunciado de la pregunta

Una reacción transfusional aguda o inmediata, se define como la que ocurre durante la
transfusión o en las 24 horas. ¿Cuál de estas opciones es una reacción aguda?
Seleccione una:

a. Enfermedad del Suero


b. Purpura post transfusional
c. Reacción Alérgica Urticaria
d. Transmisión de un HIV
Retroalimentación
La respuesta correcta es: Reacción Alérgica Urticaria

Pregunta 25
Correcta
Puntúa 0,16 sobre 0,16

Marcar pregunta
Enunciado de la pregunta

Un paciente hipertenso con descompensación aguda “típica” de insuficiencia cardiaca sin


sobrecarga de volumen, que fármaco usaría:
Seleccione una:
a. Clortalidona

b. Nitroprusiato
c. Dobutamina
d. Furosemida
Retroalimentación
La respuesta correcta es: Nitroprusiato
Pregunta 1
Correcta
Puntúa 0,16 sobre 0,16

Marcar pregunta
Enunciado de la pregunta

Con respecto a la infección por Virus del Papiloma Humano los tipos (oncogénicos) e
alto riesgo tenemos a:
Seleccione una:

a. 13, 15 y 17.

b. 16, 19, 31 y 22

c. 13, 12,23 y 21

d. 16, 18, 31, 33 y 45.


Retroalimentación
La respuesta correcta es: 16, 18, 31, 33 y 45.

Pregunta 2
Correcta
Puntúa 0,16 sobre 0,16

Marcar pregunta
Enunciado de la pregunta

Con respecto a la vacunación de la fiebre amarilla este tiene una inmunidad con un
plazo de:
Seleccione una:

a. Plazo no mayor a 3 años.

b. Únicamente 12 meses.

c. De por vida del huésped susceptible.

d. Un plazo de 10 días a 10 años.


Retroalimentación
La respuesta correcta es: Un plazo de 10 días a 10 años.

Pregunta 3
Correcta
Puntúa 0,16 sobre 0,16
Marcar pregunta
Enunciado de la pregunta

¿Cuál de las siguientes alternativas indica el carcinoma que prevalece en el tercio


superior del esófago?
Seleccione una:

a. Carcinoma in situ

b. Carcinoma escamoso

c. Adenocarcinoma

d. Carcinoma atípico
Retroalimentación
La respuesta correcta es: Carcinoma escamoso

Pregunta 4
Correcta
Puntúa 0,16 sobre 0,16

Marcar pregunta
Enunciado de la pregunta

¿Cuál de los siguientes hallazgos histológicos es necesario para establecer el


diagnóstico de cirrosis hepática?
Seleccione una:

a. Degeneración baloniforme de los hepatocitos.

b. Destrucción de los conductos biliares.

c. Cuerpos de Mallory.

d. Presencia de fibrosis.
Retroalimentación
La respuesta correcta es: Presencia de fibrosis.

Pregunta 5
Correcta
Puntúa 0,16 sobre 0,16
Marcar pregunta
Enunciado de la pregunta

Dentro de la fisiopatogenia de la infección por VIH tenemos a los coreceptores


principales en la infección estos son:
Seleccione una:

a. CCR5 y CXCR4

b. CCR5 y gp 120

c. Ácido siálico e integrasa.

d. Ácido hialurónico y CD59


Retroalimentación
La respuesta correcta es: CCR5 y CXCR4

Pregunta 6
Correcta
Puntúa 0,16 sobre 0,16

Marcar pregunta
Enunciado de la pregunta

Dentro de las complicaciones extra pulmonares en influenza se han descrito:


Seleccione una:

a. Síndrome de Reye

b. Trombocitopenia y trastornos de la coagulación.

c. Síndrome urémico.

d. Coagulación intravascular diseminada.


Retroalimentación
La respuesta correcta es: Síndrome de Reye

Pregunta 7
Correcta
Puntúa 0,16 sobre 0,16

Marcar pregunta
Enunciado de la pregunta

Dentro de las manifestaciones tardías de la sífilis tenemos trastornos cardiovasculares


de cual podemos desprender el siguiente:
Seleccione una:

a. Insuficiencia cardiaca congestiva.

b. Trastornos del endotelio

c. Flebitis irritativa.

d. Aneurismas
Retroalimentación
La respuesta correcta es: Aneurismas

Pregunta 8
Correcta
Puntúa 0,16 sobre 0,16

Marcar pregunta
Enunciado de la pregunta

El dermatofito más común en la tiña pedis interdigital es, señale la alternativa correcta:
Seleccione una:

a. Tricophyton rubrum

b. Epidermophyton

c. Tricophyton mentagrophytes.

d. Microsporun canis
Retroalimentación
La respuesta correcta es: Tricophyton mentagrophytes.

Pregunta 9
Correcta
Puntúa 0,16 sobre 0,16

Marcar pregunta
Enunciado de la pregunta

El esófago de Barret se caracteriza por, señale la alternativa correcta:


Seleccione una:
a. Se lo considera carcinoma in situ de esófago distal
b. Metaplasia intestinal

c. Se asocia con el anillo de Schatzki.

d. Se asocia con divertículo de Zenker


Retroalimentación
La respuesta correcta es: Metaplasia intestinal

Pregunta 10
Correcta
Puntúa 0,16 sobre 0,16

Marcar pregunta
Enunciado de la pregunta

El tratamiento de elección en colitis pseudomembranosa que cursa con íleo paralítico


es, señale la alternativa correcta:
Seleccione una:

a. Metronidazol por vía intravenosa.

b. Vancomicina por vía intravenosa.

c. Vancomicina por vía oral.

d. Metronidazol por vía oral.


Retroalimentación
La respuesta correcta es: Metronidazol por vía intravenosa.

Pregunta 11
Correcta
Puntúa 0,16 sobre 0,16

Marcar pregunta
Enunciado de la pregunta

En la clasificación de OMS de adultos con VIH – Sida en el estadio C3 tenemos a:


Seleccione una:

a. Mayor de 500 células CD4+ Asintomático.

b. Menor de 200 células CD4+ con síntomas definidores de Sida.

c. Menor de 300 células CD4+ con síntomas definidores de Sida.

d. Mayor de 400 células CD4+ con síntomas no definidores de Sida.


Retroalimentación
La respuesta correcta es: Menor de 200 células CD4+ con síntomas definidores de Sida.

Pregunta 12
Incorrecta
Puntúa 0,00 sobre 0,16

Marcar pregunta
Enunciado de la pregunta

En la urticaria crónica autoinmune un medicamento de gran utilidad con pocos efectos


secundarios es:
Seleccione una:

a. Loratadina

b. Cimetidina

c. Levocetirizina

d. Ciclosporina
Retroalimentación
La respuesta correcta es: Ciclosporina

Pregunta 13
Incorrecta
Puntúa 0,00 sobre 0,16

Marcar pregunta
Enunciado de la pregunta

En la urticaria crónica resistente a antihistaminicos anti h1 se puede utilizar


Seleccione una:

a. Infliximab

b. Rituximab

c. Ciproheptadina

d. Ciclosporina
Retroalimentación
La respuesta correcta es: Ciproheptadina

Pregunta 14
Correcta
Puntúa 0,16 sobre 0,16
Marcar pregunta
Enunciado de la pregunta

En una infección pro Cándida en mucosa oral, indique la premisa correcta:


Seleccione una:

a. Invasión profunda con daño extenso de tejido.

b. El tratamiento de elección es la caspofungina vía parenteral.

c. Constituye un hongo hifa, filamentoso de colonias secas.

d. Es importante saber el estado inmunitario del paciente para establecer el posible

diagnóstico.
Retroalimentación
La respuesta correcta es: Es importante saber el estado inmunitario del paciente para
establecer el posible diagnóstico.

Pregunta 15
Correcta
Puntúa 0,16 sobre 0,16

Marcar pregunta
Enunciado de la pregunta

La primoinfeccion de VHS 1 se basa principalmente es cuales aspectos clínicos:


Seleccione una:

a. Herpes labial exfoliativo.

b. Irritabilidad y llanto fácil del lactante.

c. Panadizo herpético.

d. Gingivoestomatitis y faringitis.
Retroalimentación
La respuesta correcta es: Gingivoestomatitis y faringitis.

Pregunta 16
Correcta
Puntúa 0,16 sobre 0,16
Marcar pregunta
Enunciado de la pregunta

Muchacha de 26 años hospitalizada por ictericia de instauración reciente, asociada a


dolor en hipocondrio derecho. Se detecta hepatomegalia sensible sin esplenomegalia.
Hay telangiectasias faciales. Reconoce antecedentes de promiscuidad sexual, pero no
consumo de droga intravenosa. AST 315 ;BT 16 mg/dl; alt 110 ; GGT 680; FA 280 ;
triglicéridos 600, colesterol 280. Ecografía con patrón ecogénico del hígado. Cuál de
las siguientes alternativas indica el diagnóstico más probable:
Seleccione una:

a. Hepatopatía alcoholica

b. Hepatitis Autoinmune

c. Hepatitis tóxica

d. Hepatitis vírica
Retroalimentación
La respuesta correcta es: Hepatitis vírica

Pregunta 17
Correcta
Puntúa 0,16 sobre 0,16

Marcar pregunta
Enunciado de la pregunta

Paciente de sexo femenino de 30 años de edad que presenta desde la infancia placas
eritematodescamativas de bordes bien definidos en codos, y rodilla con descamación
intensa y prurito moderado, además presenta eritema y descamación profusa en cuero
cabelludo. ¿Cuál de las siguientes alternativas corresponde al tratamiento tópico?
Seleccione una:

a. Tacrolimus + tazaroteno

b. Emolientes y corticoides topicos

c. Inhibidores de la calcineurina asociados a calcipotriol

d. Corticoesteroides tópicos asociados a calcipotriol


Retroalimentación
La respuesta correcta es: Corticoesteroides tópicos asociados a calcipotriol

Pregunta 18
Correcta
Puntúa 0,16 sobre 0,16

Marcar pregunta
Enunciado de la pregunta

Para el manejo de infecciones dermatológicas la Penicilina G es uno de los


betalactámicos más eficientes por su gran actividad sobre Pseudomonas, responda
verdadero o falso
Seleccione una:

a. Falso

b. Verdadero
Retroalimentación
La respuesta correcta es: Falso

Pregunta 19
Correcta
Puntúa 0,16 sobre 0,16

Marcar pregunta
Enunciado de la pregunta

Respecto a la cirrosis hepática, escoja la respuesta correcta.


Seleccione una:

a. La cirrosis es la etapa final de cualquier enfermedad aguda del hígado.

b. Es un proceso reversible.

c. Es un proceso localizado caracterizado por fibrosis y la conversión de la arquitectura normal


en nódulos de estructura alterada.

d. Las complicaciones son consecuencia de la hipertensión portal y la insuficiencia

hepática.
Retroalimentación
La respuesta correcta es: Las complicaciones son consecuencia de la hipertensión portal y la
insuficiencia hepática.

Pregunta 20
Correcta
Puntúa 0,16 sobre 0,16
Marcar pregunta
Enunciado de la pregunta

Sobra la Enfermedad Inflamatoria Intestinal, escoja la respuesta INCORRECTA:


Seleccione una:

a. El factor de necrosis tumoral y el interferón alfa contribuyen directamente al daño epitelial del
intestino.

b. Los niveles tisulares de IL 1 están elevados en la Enfermedad de Crohn y los niveles séricos

de IL 2 están igualmente elevados en la Colitis ulcerosa.

c. En la EII se encuentra aumentada la producción de las citokinas proinflamatorias, IL 1, IL 6,


IL 8, y además del FNT (factor de necrosis tumoral) alfa.

d. La mayor fuente de producción de estas interleukinas proinflamatorias son los macrófagos


activados de la lámina propia que estimulan las células inmunes.
Retroalimentación
La respuesta correcta es: Los niveles tisulares de IL 1 están elevados en la Enfermedad de
Crohn y los niveles séricos de IL 2 están igualmente elevados en la Colitis ulcerosa.

Pregunta 21
Correcta
Puntúa 0,16 sobre 0,16

Marcar pregunta
Enunciado de la pregunta

Tras la infección de un niño con el virus de la Influenza con la siguientes


características: fiebre intensa de inicio súbito, disnea y cianosis a la Rx de tórax se
aprecia patrón asociado con infiltrados intersticiales difusos e hipoxia intensa, usted
sospecharía de:
Seleccione una:

a. Neumonía viral secundaria.

b. Neumonía bacteriana primaria.

c. Neumonía viral primaría.

d. Neumonía bacteriana secundaria.


Retroalimentación
La respuesta correcta es: Neumonía viral primaría.

Pregunta 22
Correcta
Puntúa 0,16 sobre 0,16

Marcar pregunta
Enunciado de la pregunta

Una de las características principales por las cuales el dengue puede diseminarse es:
Seleccione una:

a. Debido a la gran capacidad de diseminación del vector Anopheles.

b. Es sumamente frecuente la progresión de daño encefálico.

c. El vector es muy cercano a los asentamientos humanos.

d. Una característica precoz es la trombocitosis y neutrofilia.


Retroalimentación
La respuesta correcta es: El vector es muy cercano a los asentamientos humanos.

Pregunta 23
Correcta
Puntúa 0,16 sobre 0,16

Marcar pregunta
Enunciado de la pregunta

Una mujer de 25 años refiere una historia de 12 meses de cólicos abdominales


recurrentes en hemi-abdomen inferior acompañado de distensión abdominal y
diarreas. No presenta fiebre, pérdida de peso ni anorexia ni deposición con sangre.
Este paciente cumple con los criterios de Roma IV para Sd de colon irritable con
predominio de diarrea, no presenta signos de alarma. ¿En qué otra patología Usted
puede pensar?
Seleccione una:

a. Sobrecrecimiento bacteriano de intestino delgado.

b. Helicobacter pylori.

c. Enfermedad celiaca.

d. Enfermedad inflamatoria intestinal.


Retroalimentación
La respuesta correcta es: Enfermedad celiaca.

Pregunta 24
Correcta
Puntúa 0,16 sobre 0,16
Marcar pregunta
Enunciado de la pregunta

Un ejecutivo de 55 años de edad se atiende por dolor epigástrico intenso; en la


endoscopia gastroduodenal se encuentra una ulcera grande en el bulbo duodenal.
Se indican una dieta e inhibidor de bomba de protones, pero los síntomas persisten.
En este momento lo más apropiado seria sugerir, señale la opción correcta:
Seleccione una:

a. Tratamiento de erradicación de helicobacter pylori.

b. Vagotomía altamente selectiva.

c. Vagotomía troncal y antrectomia

d. Vagotomía troncal y piloroplastia.


Retroalimentación
La respuesta correcta es: Tratamiento de erradicación de helicobacter pylori.

Pregunta 25
Correcta
Puntúa 0,16 sobre 0,16

Marcar pregunta
Enunciado de la pregunta

Un hombre de 69 años con hepatitis crónica por virus C y cirrosis sin antecedentes
previos de hemorragia varicial, acude por el hallazgo endoscópico de várices
esofágicas grado III. ¿Cuál es el agente farmacológico de preferencia para la
prevención de sangrado varicial?
Seleccione una:
a. Atenolol.

b. Pantoprazol.

c. Carvedilol.

d. 5 mononitrato de isosorbide.
Retroalimentación
La respuesta correcta es: Carvedilol.
Pregunta 1
Correcta
Puntúa 0,16 sobre 0,16

Marcar pregunta
Enunciado de la pregunta

Con respecto a DM2 señale lo correcto:


Seleccione una:

a. Se asocia con una fuerte predisposición genética o antecedentes familiares en familiares de

primer grado.

b. Existe una destrucción de las células beta del páncreas con un déficit absoluto de insulina

c. Un de los criterios para su diagnóstico es PTOG a las dos horas con un valor de 140-199
mg/dl

d. Representa 2– 4% de todas las diabetes


Retroalimentación
La respuesta correcta es: Se asocia con una fuerte predisposición genética o antecedentes
familiares en familiares de primer grado.

Pregunta 2
Correcta
Puntúa 0,16 sobre 0,16

Marcar pregunta
Enunciado de la pregunta

¿Cuál de las siguientes es una causa secundaria de Diabetes?


Seleccione una:
a. Hipoparatiroidismo, hipertiroidismo, pancreatitis

b. Acromegalia, embarazo, enfermedad de Addison

c. Síndrome de ovario poliquístico, Síndrome de Cushing, acromegalia.

d. Hipotiroidismo, embarazo, enfermedad de Addison


Retroalimentación
La respuesta correcta es: Síndrome de ovario poliquístico, Síndrome de Cushing, acromegalia.

Pregunta 3
Correcta
Puntúa 0,16 sobre 0,16
Marcar pregunta
Enunciado de la pregunta

¿Cuál de las siguientes es una complicación aguda?


Seleccione una:

a. Cetoacidosis.

b. Nefropatía (grado I)

c. Cardiopatía isquémica

d. Retinopatía proliferativa
Retroalimentación
La respuesta correcta es: Cetoacidosis.

Pregunta 4
Correcta
Puntúa 0,16 sobre 0,16

Marcar pregunta
Enunciado de la pregunta

¿Cuál de los siguientes corresponde a un criterio de diagnóstico en DM2?


Seleccione una:

a. Glucosa al azar (2 tomas ≥ 200mg/dl).

b. Glucosas en ayunas 100-125mg/dl

c. Glucosa al azar ≥ 126 mg/dl junto con las tres P (poliuria, polidipsia, polifagia)

d. HbAc1 de 5,7 – 6,4%


Retroalimentación
La respuesta correcta es: Glucosa al azar (2 tomas ≥ 200mg/dl).

Pregunta 5
Correcta
Puntúa 0,16 sobre 0,16

Marcar pregunta
Enunciado de la pregunta
De los siguientes enunciados señale el verdadero:
Seleccione una:

a. El falx divide al cerebro en hemisferio derecho e izquierdo

b. La herniación se refiere al desplazamiento de un compartimento de menor a mayor presión

c. El tentorio divide al cerebral en región infratentorial de región medular

d. El desplazamiento del mesencéfalo con compresión del pedúnculo origina midriasis


ipsilateral
Retroalimentación
La respuesta correcta es: El falx divide al cerebro en hemisferio derecho e izquierdo

Pregunta 6
Correcta
Puntúa 0,16 sobre 0,16

Marcar pregunta
Enunciado de la pregunta

El dato clínico más importante para considerar un evento cerebrovascular como


hemorrágico más que como isquémico es:
Seleccione una:

a. antecedente de hipertensión arterial

b. glicemia en urgencias mayor de 300 mg/dL

c. hemiparesia de instauración súbita

d. afectación temprana del estado de conciencia


Retroalimentación
La respuesta correcta es: afectación temprana del estado de conciencia

Pregunta 7
Incorrecta
Puntúa 0,00 sobre 0,16

Marcar pregunta
Enunciado de la pregunta

En relación a los factores de riesgo en infarto cerebral, señale el enunciado verdadero:


Seleccione una:

a. El tratamiento de prevención secundaria evita la posibilidad de recurrencia de un infarto


b. La presencia de fibrilación auricular valvular requiere anticoagulación total crónicamente

c. Factores no modificables como el sedentarismo no requiere manejo específico

d. Los niveles de control de lípidos son: HDL >130 mg/dl, colesterol total >250 mg/dl
Retroalimentación
La respuesta correcta es: El tratamiento de prevención secundaria evita la posibilidad de
recurrencia de un infarto

Pregunta 8
Correcta
Puntúa 0,16 sobre 0,16

Marcar pregunta
Enunciado de la pregunta

En relación al tratamiento de Encefalitis viral, señale el enunciado verdadero:


Seleccione una:

a. El tratamiento antiviral debe ser iniciado cuando se tenga el resultado del cultivo

b. Aciclovir debe administrarse en vía IV a dosis de 30 mg/kg/día

c. Se debe añadir al antiviral tratamiento a largo plazo con prednisona oral

d. El aciclovir en vía oral tiene igual efectividad que el IV


Retroalimentación
La respuesta correcta es: Aciclovir debe administrarse en vía IV a dosis de 30 mg/kg/día

Pregunta 9
Correcta
Puntúa 0,16 sobre 0,16

Marcar pregunta
Enunciado de la pregunta

En relación al tratamiento de meningitis bacteriana aguda, señale el enunciado


verdadero:
Seleccione una:

a. En Haemophilus influenzae se prescribiría ceftriaxona

b. En estafilococo meticilino resistente se prescribiría penicilina G

c. Si hay resistencia a la penicilina se prescribiría Amikacina

d. En Neisseria Meningitidis se prescribiría metronidazol


Retroalimentación
La respuesta correcta es: En Haemophilus influenzae se prescribiría ceftriaxona

Pregunta 10
Correcta
Puntúa 0,16 sobre 0,16

Marcar pregunta
Enunciado de la pregunta

Entre los factores de riesgo de enfermedad de Graves tenemos escoja la opción


correcta
Seleccione una:

a. no se relaciona con la herencia familiar , el tabaquismo es un factor de riesgo intermedio , se


ve en gemelos idénticos

b. historia familiar de enfermedad de Hashimoto, presencia de otras enfermedades

autoinmunes , más común en mujeres que en hombres , tabaquismo.

c. se ve en misma proporción como factor de riesgo en mujeres y en hombres, el tabaquismo y


se asocia con enfermedades autoinmunes

d. presencia de otras enfermedades autoinmunes , tabaquismo más frecuente en hombres que


el mujeres , antecedente de enfermedad de Graves .
Retroalimentación
La respuesta correcta es: historia familiar de enfermedad de Hashimoto, presencia de otras
enfermedades autoinmunes , más común en mujeres que en hombres , tabaquismo.

Pregunta 11
Correcta
Puntúa 0,16 sobre 0,16

Marcar pregunta
Enunciado de la pregunta

En un paciente con síntomas de diplopía, con antecedentes de una infección


gastrointestinal 10 días atrás y que al examen se lo encuentra con oftalmoparesia y
arreflexia, al estudiarlo se encuentra disociación albúmino-citológica, cuál sería su
proceder:
Seleccione una:

a. Se debe solicitar determinación de anticuerpos contra los lípidos del axón neuronal

b. Le prescribiría corticoides en vía IV durante 5 días

c. Si lleva 3 días de evolución tendría que realizarse una electromiografía


d. Le diagnosticaría Sd de Miller-Fisher como variante del Sd de Guillain-Barré
Retroalimentación
La respuesta correcta es: Le diagnosticaría Sd de Miller-Fisher como variante del Sd de
Guillain-Barré

Pregunta 12
Correcta
Puntúa 0,16 sobre 0,16

Marcar pregunta
Enunciado de la pregunta

En un paciente de 77 años con antecedentes de fumador, estenosis carotídea,


diabetes mellitus y dislipidemia, presentó un episodio de pérdida del estado de
conciencia al parecer con crisis convulsivas, sin testigos directos, se sospecha en
crisis convulsivas, con qué entidad debería hacer diagnóstico diferencial:
Seleccione una:
a. Migraña basilar

b. Hipotensión ortostática

c. Crisis psicogénica

d. Ataques de pánico
Retroalimentación
La respuesta correcta es: Hipotensión ortostática

Pregunta 13
Incorrecta
Puntúa 0,00 sobre 0,16

Marcar pregunta
Enunciado de la pregunta

En un paciente que acude a urgencias por presentar un cuadro de cefalea de inicio


súbito con deterioro del estado de conciencia y una crisis tónico clónica generalizada,
qué estudio requeriría con mayor urgencia:
Seleccione una:

a. Biometría hemática

b. Electrolitos

c. Electroencefalograma
d. TAC cerebral simple
Retroalimentación
La respuesta correcta es: TAC cerebral simple

Pregunta 14
Correcta
Puntúa 0,16 sobre 0,16

Marcar pregunta
Enunciado de la pregunta

Escoja la respuesta CORRECTA sobre la tiroiditis crónica o de Hashimoto


Seleccione una:

a. Produce una glándula blanda, de gran crecimiento, dolorosa y con consistencia lobulada

b. Es una enfermedad autoinmune donde actúan los linfocitos T CD4 que libran IL-2 y FNT y

los linfocitos T citotóxicos que produce apoptosis de tirocitos

c. Es más frecuente en hombres (1:10) vs mujeres (1:20).

d. No tiene factores desencadenantes como estrés, déficit de selenio, infección por Yersinia
Enterocolítica o hepatitis C
Retroalimentación
La respuesta correcta es: Es una enfermedad autoinmune donde actúan los linfocitos T CD4
que libran IL-2 y FNT y los linfocitos T citotóxicos que produce apoptosis de tirocitos

Pregunta 15
Correcta
Puntúa 0,16 sobre 0,16

Marcar pregunta
Enunciado de la pregunta

Hablando de la fisiopatología de la oftalmopatía de Graves .Escoja la opción correcta


Seleccione una:

a. la disminución de las células T juegan un papel importante en la generación de la


oftalmopatía .

b. las células T dismunuidas , los fibroblastos producen ácido hialuronico y glucosamino


glucanos y esto aumenta el tejido retro ocular .

c. el volumen de los músculos retro oculares aumentan debido a la disminución de los


fibroblastos .
d. la presencia aumentada d e células T juegan un papel importantes en la oftalmopatía , y los

fibroblastos retro oculares secretan glucosaminoglucanos


Retroalimentación
La respuesta correcta es: la presencia aumentada d e células T juegan un papel importantes en
la oftalmopatía , y los fibroblastos retro oculares secretan glucosaminoglucanos

Pregunta 16
Correcta
Puntúa 0,16 sobre 0,16

Marcar pregunta
Enunciado de la pregunta

Hay que hacer una historia clínica minuciosa en el hipotiroidismo, tomando especial
importancia a ciertos fármacos que producen disfunción de hormona tiroidea. ¿Cuál de
los siguientes NO está asociado con hipotiroidismo?
Seleccione una:

a. Yodo

b. Cefalosporinas.

c. Amiodarona

d. Interferon alfa
Retroalimentación
La respuesta correcta es: Cefalosporinas.

Pregunta 17
Incorrecta
Puntúa 0,00 sobre 0,16

Marcar pregunta
Enunciado de la pregunta

La hemorragia cerebral primaria o por hipertensión arterial se localiza en:


Seleccione una:

a. Bulbo raquídeo

b. Región lobar parietal

c. Núcleo putaminal

d. Ventrículos
Retroalimentación
La respuesta correcta es: Núcleo putaminal

Pregunta 18
Correcta
Puntúa 0,16 sobre 0,16

Marcar pregunta
Enunciado de la pregunta

Paciente de 4 0 años de sexo femenino que se ha pedido exámenes por control


médico anual y en el Examen Físico se encontró BOCIO regresa con resultados: Eco
tiroideo: reporta imagen nodular solida de 1 cm hipo eco génica con vascularidad
central y periférica con margen irregular . TSH 4 . 2 u/ IU/ ml T 4 1.6 ng / dl. Escoja la
opción correcta.
Seleccione una:

a. el reporte ecográfico no hace sospechar de malignidad se debe hacer seguimiento


únicamente en 6 meses

b. le derivo al endocrinólogo y al cirujano de cuello para que le le opren inmediatamente , y no


necesita de otros exámenes .

c. es una paciente con sospecha de malignidad mas de un 70 % se debe solicitar punción con

aguja fina y se solicita marcadores tumorales.

d. es una paciente con poca sospecha de malignidad como de 30 % y se debe pedir un nuevo
control en 3 meses
Retroalimentación
La respuesta correcta es: es una paciente con sospecha de malignidad mas de un 70 % se
debe solicitar punción con aguja fina y se solicita marcadores tumorales.

Pregunta 19
Correcta
Puntúa 0,16 sobre 0,16

Marcar pregunta
Enunciado de la pregunta

Paciente de 45 años acude a consulta por presentar 6 meses de evolución de caída de


cabello y no refiere otros síntomas, Al examen físico se encuentra escaso cabello y
hay piel seca lo demás de examen físico esta normal , se solicita exámenes los
mismos que reportan : TSH5.17 u/IU/ml , T3 3 ,3 n mol/ L- T 4 1.0 ng / dl. Escoja la
opción correcta.
Seleccione una:
a. el paciente tiene diagnóstico de hipotiroidismo subclínico

b. el paciente tiene diagnóstico de eutiroidismo

c. el paciente tiene diagnóstico de hipertiroidismo

d. paciente tiene diagnóstico de hipotiroidismo central


Retroalimentación
La respuesta correcta es: el paciente tiene diagnóstico de hipotiroidismo subclínico

Pregunta 20
Correcta
Puntúa 0,16 sobre 0,16

Marcar pregunta
Enunciado de la pregunta

Paciente de 65 años con antecedente de cáncer de estómago, tiene una dieta


fraccionada con 5 comidas ha bajado de peso mucho como 10 kilogramos y refiere
dolor de epigastrio en forma continua ,además refiere que se ha sentido con polidipsia
y poliuria por lo que se solicita exámenes los mismos que reportan: Glicemia 103 mg /
dl Hemoglobina Glicosilada 6.4 mg y curva de tolerancia a la glicemia reporta a la 99
mg , 30 minutos, 108 mg, 1 horas 199 mg , 2 horas 155 mg. Escoja la opción correcta
Seleccione una:

a. la paciente tiene valores normales de laboratorio y no necesita recomendaciones

b. la paciente tiene diagnóstico de prediabetes tiene que remitirse a nutrición y debe regresar a

un control en 3 meses con glicemia y nueva hemoglobina glicosilada.

c. tiene cáncer gástrico y tiene diabetes y debe recibir ya tratamiento con metformina de 1000
mg cada 12 horas

d. la paciente tiene gastrectomía por cáncer gástrico y al momento cursa además con diabetes
y se le recomienda valoración nutricional , ejercicio y control en 1 mes con nueva glicemia
Retroalimentación
La respuesta correcta es: la paciente tiene diagnóstico de prediabetes tiene que remitirse a
nutrición y debe regresar a un control en 3 meses con glicemia y nueva hemoglobina
glicosilada.

Pregunta 21
Incorrecta
Puntúa 0,00 sobre 0,16

Marcar pregunta
Enunciado de la pregunta
Señale la respuesta CORRECTA sobre las complicaciones del hipotiroidismo
Seleccione una:

a. En el hipotiroidismo severo se puede generar hipernatremia dilucional.

b. En el hipotiroidismo leve puede haber apnea del sueño y síndrome del túnel carpiano

c. Puede producir hipertensión, cardiomegalia y disfunción diastólica

d. Algunas personas desarrollan incremento de memoria, estado de ánimo eufórica


Retroalimentación
La respuesta correcta es: Puede producir hipertensión, cardiomegalia y disfunción diastólica

Pregunta 22
Incorrecta
Puntúa 0,00 sobre 0,16

Marcar pregunta
Enunciado de la pregunta

Si un paciente es detectado con HbA1c: > 9%, la acción correcta para su tratamiento
sería:
Seleccione una:

a. Se debe realizar a triple terapia de ADO

b. Administrar metformina junto con un ADO

c. Iniciar inmediatamente con insulina.

d. La primera línea indicada es la metformina en busca de HbAc1 de < 7%


Retroalimentación
La respuesta correcta es: Iniciar inmediatamente con insulina.

Pregunta 23
Incorrecta
Puntúa 0,00 sobre 0,16

Marcar pregunta
Enunciado de la pregunta

Un hallazgo clínico sugestivo de infarto cerebral en la circulación carotídea es:


Seleccione una:

a. parálisis facial ipsilateral a la lesión con hemiplejia contralateral

b. alteración del estado de conciencia


c. afasia de Broca sin hemiparesia

d. parálisis oculomotora con tetraplejia


Retroalimentación
La respuesta correcta es: afasia de Broca sin hemiparesia

Pregunta 24
Incorrecta
Puntúa 0,00 sobre 0,16

Marcar pregunta
Enunciado de la pregunta

Un paciente joven es encontrado en coma, con pupilas isocóricas de 2 mm reactivas al


reflejo fotomotor, reflejo oculocefálico normal, al dolor la respuesta era de flexión
parcial en 4 extremidades, con estos datos qué estudio hubiera solicitado con mayor
urgencia:
Seleccione una:

a. Electrocardiograma

b. Tiempos de coagulación

c. Pruebas toxicológicas

d. Electroencefalograma
Retroalimentación
La respuesta correcta es: Pruebas toxicológicas

Pregunta 25
Incorrecta
Puntúa 0,00 sobre 0,16

Marcar pregunta
Enunciado de la pregunta

Un paciente que ingresa con un cuadro de amaurosis fugax del ojo derecho que
revirtió en 10 minutos, usted sospecharía en:
Seleccione una:

a. Neuritis óptica secundario a vasculitis

b. Infarto occipital agudo secundario a aterosclerosis vertebral

c. Compresión del quiasma óptico por macroadenoma hipofisario

d. Accidente isquémico transitorio secundario a aterosclerosis carotídea


Retroalimentación
La respuesta correcta es: Accidente isquémico transitorio secundario a aterosclerosis carotídea
Con respecto a DM2 señale lo correcto:
Seleccione una:

a. Se asocia con una fuerte predisposición genética o antecedentes familiares en familiares de

primer grado.

b. Existe una destrucción de las células beta del páncreas con un déficit absoluto de insulina

c. Representa 2– 4% de todas las diabetes

d. Un de los criterios para su diagnóstico es PTOG a las dos horas con un valor de 140-199
mg/dl
Retroalimentación
La respuesta correcta es: Se asocia con una fuerte predisposición genética o antecedentes
familiares en familiares de primer grado.

Pregunta 2
Correcta
Puntúa 0,16 sobre 0,16

Desmarcar
Enunciado de la pregunta

¿Cuál de las siguientes es una causa secundaria de Diabetes?


Seleccione una:

a. Acromegalia, embarazo, enfermedad de Addison

b. Hipotiroidismo, embarazo, enfermedad de Addison

c. Hipoparatiroidismo, hipertiroidismo, pancreatitis

d. Síndrome de ovario poliquístico, Síndrome de Cushing, acromegalia.


Retroalimentación
La respuesta correcta es: Síndrome de ovario poliquístico, Síndrome de Cushing, acromegalia.

Pregunta 3
Correcta
Puntúa 0,16 sobre 0,16

Desmarcar
Enunciado de la pregunta

¿Cuál de las siguientes es una complicación aguda?


Seleccione una:
a. Nefropatía (grado I)

b. Cardiopatía isquémica

c. Cetoacidosis.

d. Retinopatía proliferativa
Retroalimentación
La respuesta correcta es: Cetoacidosis.

Pregunta 4
Correcta
Puntúa 0,16 sobre 0,16

Desmarcar
Enunciado de la pregunta

¿Cuál de los siguientes corresponde a un criterio de diagnóstico en DM2?


Seleccione una:

a. Glucosa al azar (2 tomas ≥ 200mg/dl).

b. Glucosa al azar ≥ 126 mg/dl junto con las tres P (poliuria, polidipsia, polifagia)

c. HbAc1 de 5,7 – 6,4%

d. Glucosas en ayunas 100-125mg/dl


Retroalimentación
La respuesta correcta es: Glucosa al azar (2 tomas ≥ 200mg/dl).

Pregunta 5
Correcta
Puntúa 0,16 sobre 0,16

Desmarcar
Enunciado de la pregunta

Cuál de los siguientes no es un factor de riesgo para el desarrollo de cáncer gástrico


Seleccione una:

a. Infección por h pylori.

b. Poliposis gástrica

c. Hipotiroidismo
d. Familiar con antecedente de cáncer digestivo
Retroalimentación
La respuesta correcta es: Hipotiroidismo

Pregunta 6
Correcta
Puntúa 0,16 sobre 0,16

Desmarcar
Enunciado de la pregunta

El tratamiento de elección para un hombre de 40 años de edad con diagnóstico de


linfoma MALT después de endoscopia y biopsia es:
Seleccione una:

a. Radioterapia

b. Tratamiento de erradicación de Helicobacter Pylori.

c. Gastrectomía subtotal

d. Quimioterapia
Retroalimentación
La respuesta correcta es: Tratamiento de erradicación de Helicobacter Pylori.

Pregunta 7
Incorrecta
Puntúa 0,00 sobre 0,16

Desmarcar
Enunciado de la pregunta

El tratamiento que se utiliza para reducir la IGE en la urticaria crónica es:


Seleccione una:

a. Rituximab

b. Ciproheptadina

c. Ciclosporina

d. Omalizumab
Retroalimentación
La respuesta correcta es: Omalizumab

Pregunta 8
Correcta
Puntúa 0,16 sobre 0,16
Desmarcar
Enunciado de la pregunta

En que situaciones es mandatorio dar tratamiento para erradicación de helicobacter


pylori
Seleccione una:

a. En síndrome de malabsorción

b. Anemia ferropenica

c. En cirugía intestinal

d. Cuando hay episodios de constipación


Retroalimentación
La respuesta correcta es: Anemia ferropenica

Pregunta 9
Correcta
Puntúa 0,16 sobre 0,16

Desmarcar
Enunciado de la pregunta

Entre los factores de riesgo de enfermedad de Graves tenemos escoja la opción


correcta
Seleccione una:

a. presencia de otras enfermedades autoinmunes , tabaquismo más frecuente en hombres que


el mujeres , antecedente de enfermedad de Graves .

b. historia familiar de enfermedad de Hashimoto, presencia de otras enfermedades

autoinmunes , más común en mujeres que en hombres , tabaquismo.

c. no se relaciona con la herencia familiar , el tabaquismo es un factor de riesgo intermedio , se


ve en gemelos idénticos

d. se ve en misma proporción como factor de riesgo en mujeres y en hombres, el tabaquismo y


se asocia con enfermedades autoinmunes
Retroalimentación
La respuesta correcta es: historia familiar de enfermedad de Hashimoto, presencia de otras
enfermedades autoinmunes , más común en mujeres que en hombres , tabaquismo.

Pregunta 10
Correcta
Puntúa 0,16 sobre 0,16
Desmarcar
Enunciado de la pregunta

En un paciente con disfagia esofágica que refiere además regurgitación y pirosis


frecuente de predominio nocturno que diagnostico consideraría
Seleccione una:

a. Diverticulo de zenker

b. Enfermedad por reflujo gastroesofagico

c. Disfagia orofaringea

d. Esofagitis infecciosa
Retroalimentación
La respuesta correcta es: Enfermedad por reflujo gastroesofagico

Pregunta 11
Correcta
Puntúa 0,16 sobre 0,16

Desmarcar
Enunciado de la pregunta

Erisipela una infección de tejido blando es debido principalmente a:


Seleccione una:

a. S. pyogenes

b. Enterobacterias

c. S. aureus

d. Pseudomona aeruginosa
Retroalimentación
La respuesta correcta es: S. pyogenes

Pregunta 12
Correcta
Puntúa 0,16 sobre 0,16

Desmarcar
Enunciado de la pregunta

Hablando de la fisiopatología de la oftalmopatía de Graves .Escoja la opción correcta


Seleccione una:

a. el volumen de los músculos retro oculares aumentan debido a la disminución de los


fibroblastos .

b. la disminución de las células T juegan un papel importante en la generación de la


oftalmopatía .

c. las células T dismunuidas , los fibroblastos producen ácido hialuronico y glucosamino


glucanos y esto aumenta el tejido retro ocular .

d. la presencia aumentada d e células T juegan un papel importantes en la oftalmopatía , y los

fibroblastos retro oculares secretan glucosaminoglucanos


Retroalimentación
La respuesta correcta es: la presencia aumentada d e células T juegan un papel importantes en
la oftalmopatía , y los fibroblastos retro oculares secretan glucosaminoglucanos

Pregunta 13
Correcta
Puntúa 0,16 sobre 0,16

Desmarcar
Enunciado de la pregunta

La causa mas frecuente de gastritis es


Seleccione una:

a. Aines

b. Helicobacter pylori

c. Alcohol

d. Radiacion
Retroalimentación
La respuesta correcta es: Helicobacter pylori

Pregunta 14
Correcta
Puntúa 0,16 sobre 0,16

Desmarcar
Enunciado de la pregunta
La fase inicial del melanoma maligno es:
Seleccione una:

a. Fase de crecimiento vertical

b. Fase de crecimiento radial

c. Fase de crecimiento acral

d. Fase de crecimiento lentiginoso


Retroalimentación
La respuesta correcta es: Fase de crecimiento radial

Pregunta 15
Correcta
Puntúa 0,16 sobre 0,16

Desmarcar
Enunciado de la pregunta

La infección por estafilococo aureus a nivel epidérmico ocasiona ampollas por


Seleccione una:

a. Lesión de los desmosomas basales

b. Lesión de los desmosomas granulosos

c. Lesión del estrato espinoso

d. Lesión del estrato basal


Retroalimentación
La respuesta correcta es: Lesión de los desmosomas granulosos

Pregunta 16
Correcta
Puntúa 0,16 sobre 0,16

Desmarcar
Enunciado de la pregunta

La presencia de superficie perlada, telangiectasias arboriformes y nódulos con


pigmento son característicos de
Seleccione una:

a. Carcinoma escamocelular

b. Melanoma maligno
c. Carcinoma basocelular

d. Tumor de celulas de merkel


Retroalimentación
La respuesta correcta es: Carcinoma basocelular

Pregunta 17
Correcta
Puntúa 0,16 sobre 0,16

Desmarcar
Enunciado de la pregunta

Paciente de 4 0 años de sexo femenino que se ha pedido exámenes por control


médico anual y en el Examen Físico se encontró BOCIO regresa con resultados: Eco
tiroideo: reporta imagen nodular solida de 1 cm hipo eco génica con vascularidad
central y periférica con margen irregular . TSH 4 . 2 u/ IU/ ml T 4 1.6 ng / dl. Escoja la
opción correcta.
Seleccione una:

a. es una paciente con sospecha de malignidad mas de un 70 % se debe solicitar punción con

aguja fina y se solicita marcadores tumorales.

b. es una paciente con poca sospecha de malignidad como de 30 % y se debe pedir un nuevo
control en 3 meses

c. el reporte ecográfico no hace sospechar de malignidad se debe hacer seguimiento


únicamente en 6 meses

d. le derivo al endocrinólogo y al cirujano de cuello para que le le opren inmediatamente , y no


necesita de otros exámenes .
Retroalimentación
La respuesta correcta es: es una paciente con sospecha de malignidad mas de un 70 % se
debe solicitar punción con aguja fina y se solicita marcadores tumorales.

Pregunta 18
Correcta
Puntúa 0,16 sobre 0,16

Desmarcar
Enunciado de la pregunta

Paciente de 45 años acude a consulta por presentar 6 meses de evolución de caída de


cabello y no refiere otros síntomas, Al examen físico se encuentra escaso cabello y
hay piel seca lo demás de examen físico esta normal , se solicita exámenes los
mismos que reportan : TSH5.17 u/IU/ml , T3 3 ,3 n mol/ L- T 4 1.0 ng / dl. Escoja la
opción correcta.
Seleccione una:

a. paciente tiene diagnóstico de hipotiroidismo central

b. el paciente tiene diagnóstico de hipotiroidismo subclínico

c. el paciente tiene diagnóstico de eutiroidismo

d. el paciente tiene diagnóstico de hipertiroidismo


Retroalimentación
La respuesta correcta es: el paciente tiene diagnóstico de hipotiroidismo subclínico

Pregunta 19
Correcta
Puntúa 0,16 sobre 0,16

Desmarcar
Enunciado de la pregunta

Paciente de 65 años con antecedente de cáncer de estómago, tiene una dieta


fraccionada con 5 comidas ha bajado de peso mucho como 10 kilogramos y refiere
dolor de epigastrio en forma continua ,además refiere que se ha sentido con polidipsia
y poliuria por lo que se solicita exámenes los mismos que reportan: Glicemia 103 mg /
dl Hemoglobina Glicosilada 6.4 mg y curva de tolerancia a la glicemia reporta a la 99
mg , 30 minutos, 108 mg, 1 horas 199 mg , 2 horas 155 mg. Escoja la opción correcta
Seleccione una:

a. tiene cáncer gástrico y tiene diabetes y debe recibir ya tratamiento con metformina de 1000
mg cada 12 horas

b. la paciente tiene gastrectomía por cáncer gástrico y al momento cursa además con diabetes
y se le recomienda valoración nutricional , ejercicio y control en 1 mes con nueva glicemia

c. la paciente tiene diagnóstico de prediabetes tiene que remitirse a nutrición y debe regresar a

un control en 3 meses con glicemia y nueva hemoglobina glicosilada.

d. la paciente tiene valores normales de laboratorio y no necesita recomendaciones


Retroalimentación
La respuesta correcta es: la paciente tiene diagnóstico de prediabetes tiene que remitirse a
nutrición y debe regresar a un control en 3 meses con glicemia y nueva hemoglobina
glicosilada.

Pregunta 20
Correcta
Puntúa 0,16 sobre 0,16
Desmarcar
Enunciado de la pregunta

Que de lo siguiente usted consideraría como tratamiento inicial ante un diagnóstico de


disfagia orofaringea
Seleccione una:

a. La disfagia orofaringea es indicacion absoluta de gastrostomia endoscopica percutanea

b. Colocacion de sonda de alimentacion duodenal ya que es necesario prevenir la


deshidratacion

c. Debido a la facil descompensacion de los pacientes durante los procedimientos


endoscopicos se prefiere una gastrostomia quirurgica

d. Corregir la enfermedad de base, terapia deglutoria, espesantes alimenticios


Retroalimentación
La respuesta correcta es: Corregir la enfermedad de base, terapia deglutoria, espesantes
alimenticios

Pregunta 21
Incorrecta
Puntúa 0,00 sobre 0,16

Desmarcar
Enunciado de la pregunta

Que examen considera el más adecuado para evaluar disfagia orofaringea en un


paciente con antecedente de enfermedad de parkinson
Seleccione una:

a. Endoscopia digestiva alta

b. Videodeglucion

c. Manometria esofagica de alta resolución

d. Rx detorax
Retroalimentación
La respuesta correcta es: Videodeglucion

Pregunta 22
Incorrecta
Puntúa 0,00 sobre 0,16
Desmarcar
Enunciado de la pregunta

Qué parámetros se usan para valorar la severidad en la evolución de pancreatitis


aguda.
Seleccione una:

a. Amilasa

b. Pcr

c. Bisap

d. Apache 2
Retroalimentación
La respuesta correcta es: Amilasa

Pregunta 23
Correcta
Puntúa 0,16 sobre 0,16

Desmarcar
Enunciado de la pregunta

Señale en vesícula biliar el tipo de cálculo más frecuente


Seleccione una:

a. colesterol

b. pigmento

c. ácido úrico

d. calcico
Retroalimentación
La respuesta correcta es: colesterol

Pregunta 24
Correcta
Puntúa 0,16 sobre 0,16

Desmarcar
Enunciado de la pregunta

Señale lo falso sobre el helicobacter pylori


Seleccione una:

a. La erradicación de helicobacter pylori mejora los síntomas dispépticos

b. Es un bacilo gram positivo

c. Es un bacilo gram negativo

d. El tratamiento de erradicación evita la progresión a lesiones premalignas


Retroalimentación
La respuesta correcta es: Es un bacilo gram positivo

Pregunta 25
Correcta
Puntúa 0,16 sobre 0,16

Desmarcar
Enunciado de la pregunta

Si un paciente es detectado con HbA1c: > 9%, la acción correcta para su tratamiento
sería:
Seleccione una:

a. La primera línea indicada es la metformina en busca de HbAc1 de <7%

b. Administrar metformina junto con un ADO

c. Se debe realizar a triple terapia de ADO

d. Iniciar inmediatamente con insulina.


Pregunta 1

Enunciado de la pregunta

ANALICE EL SIGUIENTE CASO CLÍNICO, PACIENTE MASCULINO DE 40


AÑOS, SIN ANTECEDENTES PERSONALES DE IMPORTANCIA, NACE Y
RESIDE EN PORTOVIEJO, PESCADOR, ACUDIÓ A CENTRO DE SALUD POR
PRESENTAR DESDE HACE 5 DÍAS UNA PLACA BIEN DELIMITADA,
ERITEMATOVIOLÁCEA EN DEDO MEDIO DE MANO IZQUIERDA, SEGÚN
LAS CARACTERÍSTICAS A QUE INFECCIÓN BACTERIANA
CORRESPONDERÍA:
Seleccione una:
a. FOLICULITIS
b. ERISIPELOIDE
c. CARBUNCO
d. ERITRASMA

Retroalimentación

La respuesta correcta es: ERISIPELOIDE

Pregunta 2

Enunciado de la pregunta

CUALES DE ESTAS LESIONES FORMAN PARTE DE LAS LESIONES


DERMATOLÓGICAS PRIMARIAS
Seleccione una:
a. VEGETACIÓN
b. LIQUENIFICACION
c. ESCAMAS
d. COSTRAS

Retroalimentación

La respuesta correcta es: LIQUENIFICACION (está mal)

Pregunta 3

Enunciado de la pregunta

Paciente con reacción transfusional Anafiláctica a repetición. Debe ser estudiado para
descartar:
Seleccione una:
a. Déficit de Ig M
b. Déficit de Ig A
c. Déficit de Ig E
d. Déficit de Ig G

Retroalimentación

La respuesta correcta es: Déficit de Ig A

Pregunta 4

Enunciado de la pregunta

Una anemia con un Índice de Producción de Reticulocítos ( IPR ) mayor de 2 % es una


anemia hiperproliferativa . Cuál sería la causa de anemia?
Seleccione una:
a. Hemorragia y Hemólisis
b. Hipoplasia medular
c. Disfunción Renal
d. Déficit de hierro

Retroalimentación

La respuesta correcta es: Hemorragia y Hemólisis

Pregunta 5

Enunciado de la pregunta

¿Con cuál de los siguientes fármacos NO interacciona la rifampicina?:


Seleccione una:
a. Fenitoína
b. Metadona
c. Cloranfenicol
d. Hidróxido de aluminio

Retroalimentación

La respuesta correcta es: Hidróxido de aluminio

Pregunta 6

Enunciado de la pregunta

¿CUÁL DE LOS SIGUIENTES NO ES CRITERIO MAYOR DE DERMATITIS


ATÓPICA?
Seleccione una:
a. PLIEGUE DE DENNIE MORGAN
b. PRURITO NOCTURNO
c. TOPOGRAFÍA DE LA LESIÓN
d. PLACAS ERITEMATOSAS DESCAMATIVAS MAL DEFINIDAS

Retroalimentación

La respuesta correcta es: PLACAS ERITEMATOSAS DESCAMATIVAS MAL


DEFINIDAS (está mal)

Pregunta 7

Enunciado de la pregunta

Causa de riesgo de Trombosis adquirida tanto venosa como arterial, la cual amerita
tratamiento anticoagulante?
Seleccione una:
a. Embarazo
b. Síndrome Antifosfolipídico
c. Inmovilización
d. Cirugía mayor

Retroalimentación

La respuesta correcta es: Síndrome Antifosfolipídico

Pregunta 8

Enunciado de la pregunta

El riesgo de contaminación Bacteriana debido a una transfusión ha aumentado conforme


disminuye el riesgo de infecciones virales, muchas bacterias no proliferan en frío. Cuál
de estos derivados representa el mayor riesgo de contaminación?
Seleccione una:
a. Plasma
b. Concentrado de glóbulos rojos
c. Concentrado de plaquetas
d. Críoprecipitado

Retroalimentación

La respuesta correcta es: Concentrado de plaquetas

Pregunta 9
Enunciado de la pregunta

Paciente de 50 años que presenta un derrame pleural con las siguientes características:
aspecto pajizo, Ph 7.3, cociente de proteínas pleura/suero 0.8, cociente de LDH
pleura/suero 0.9, Gram y Ziehl negativos, lípidos totales, colesterol y triglicéridos
normales, células mesoteliales <5%, intensa linfocitosis sin atipias, ADA 64 U/l. ¿Qué
diagnóstico le sugiere?
Seleccione una:
a. Mesotelioma pleural.
b. Empiema pleural.
c. Derrame pleural tuberculoso.
d. Derrame pleural por insuficiencia cardiaca (trasudado).

Retroalimentación

La respuesta correcta es: Derrame pleural tuberculoso.

Pregunta 10

Enunciado de la pregunta

RESPECTO AL ABSCESO SUBCUTÁNEO, SEÑALE EL LITERAL CORRECTO:


Seleccione una:
a. LAS LOCALIZACIONES MENOS FRECUENTES EN NIÑOS SON: MAMA,
ZONA PERIRECTAL, Y CUERO CABELLUDO.
b. EL TRATAMIENTO NO DEBE CONSIDERAR EL DRENAJE LOCAL
c. LAS BACTERIAS MÁS FRECUENTEMENTE IMPLICADAS SON
ANAEROBIAS
d. SE MANIFIESTA COMO UN NÓDULO FIRME, ERITEMATOSO Y
DOLOROSO

Retroalimentación

La respuesta correcta es: SE MANIFIESTA COMO UN NÓDULO FIRME,


ERITEMATOSO Y DOLOROSO

Pregunta 11

Enunciado de la pregunta

Los Eritrocitos jóvenes que con la coloración de azul de metileno observamos restos de
ARN y su aumento indica buena respuesta de la médula ante una anemia. Cuál célula
nos referimos?
Seleccione una:
a. Células en Diana
b. Reticulocitos
c. Cuerpos de Howel Jolly
d. Esquitocitos

Retroalimentación

La respuesta correcta es: Reticulocitos

Pregunta 12

Enunciado de la pregunta

¿Cuáles son los factores de riesgo débiles para TEP?


Seleccione una:
a. Quimioterapia, embarazo
b. Fractura de cadera, cirugía mayor
c. Hemofilia, lesión medular
d. Ancianos, obesidad, venas varicosas

Retroalimentación

La respuesta correcta es: Ancianos, obesidad, venas varicosas

Pregunta 13

Enunciado de la pregunta

Un valor de > 4 en la escala de Wells indica que:


Seleccione una:
a. El paciente está estable y es poco probable que presente embolia pulmonar
b. Clínicamente el paciente no tiene probabilidad de Embolismo Pulmonar
c. El paciente está inestable pero la probabilidad de que presente embolia pulmonar
es baja
d. Clínicamente el paciente tiene probabilidad de Embolismo Pulmonar

Retroalimentación

La respuesta correcta es: Clínicamente el paciente tiene probabilidad de Embolismo


Pulmonar

Pregunta 14

Enunciado de la pregunta

Cuál de las siguientes afirmaciones corresponden a Neumonía Adquirida en la


Comunidad.
Seleccione una:
a. Pacientes que viven en hogares de cuidados crónicos.
b. Pacientes que presentan síntomas durante las 48 primeras horas de la admisión al
hospital
c. Cuadro de presentación aguda.
d. Pacientes no hospitalizados en los 14 días anteriores

Retroalimentación

La respuesta correcta es: Pacientes que viven en hogares de cuidados crónicos.

Pregunta 15

Enunciado de la pregunta

CUANTO A LA DERMATITIS DE CONTACTO SEÑALE LA INCORRECTA:


Seleccione una:
a. EL 80 PORCIENTO DE LAS DERMATITIS POR CONTACTO SE DEBEN A
LA ACCIÓN DE LOS IRRITANTES.
b. LA DERMATITIS DE CONTACTO POR IRRITANTES PRODUCE UNA
RESPUESTA INFLAMATORIA DE GRADO VARIABLE.
c. EL 20 PORCIENTO DE LAS DERMATITIS POR CONTACTO
OCUPACIONALES ESTÁN PRODUCIDAS POR SENSIBILIZACIÓN.
d. LA DERMATITIS DE CONTACTO POR IRRITANTES ES EL RESULTADO
DE UNA REACCIÓN ALÉRGICA.

Retroalimentación

La respuesta correcta es: LA DERMATITIS DE CONTACTO POR IRRITANTES ES


EL RESULTADO DE UNA REACCIÓN ALÉRGICA.

Pregunta 16

Enunciado de la pregunta

¿Cuál es la reacción transfusional más frecuente?


Seleccione una:
a. Reacción Anafiláctica
b. Reacción alérgica
c. Reacción febril no hemolítica
d. Reacción Hemolítica

Retroalimentación

La respuesta correcta es: Reacción febril no hemolítica


Pregunta 17

Enunciado de la pregunta

La fragmentación de Eritrocitos (Esquitocitos) imagen en sangre periférica que


observamos en pacientes con:
Seleccione una:
a. Anemia por trastorno de maduración (A. megaloblástica)
b. Daño medular. Hipoplasia
c. Déficit de hierro
d. Válvulas Cardíacas Mecánicas

Retroalimentación

La respuesta correcta es: Válvulas Cardíacas Mecánicas

Pregunta 18

Enunciado de la pregunta

PACIENTE DE 35 AÑOS QUE ACUDE A CENTRO MEDICO PORQUE DESDE


HACE 4 DÍAS PRESENTA PRURITO NOCTURNO Y HA NOTADO EN SU
ANTEBRAZO IZQUIERDO PÁPULAS Y VESÍCULAS QUE FORMA UN
TRAYECTO SERPIGINOSO. ¿CUÁL SERÍA EL DIAGNÓSTICO Y
TRATAMIENTO A SEGUIR?
Seleccione una:
a. DERMATITIS SEBORREICA; KETOCONAZOL, ITRACONAZOL
b. ESCABIOSIS; PERMETRINA, IVERMECTINA TÓPICA
c. PSORIASIS; RETINOIDES Y QUERATOLITICOS
d. DERMATITIS ATÓPICA; RETINOIDES Y ANTIBIÓTICOS TÓPICOS

Retroalimentación

La respuesta correcta es: ESCABIOSIS; PERMETRINA, IVERMECTINA TÓPICA

Pregunta 19

Enunciado de la pregunta

SEÑALE LO VERDADERO SOBRE LA INFLAMACION:


Seleccione una:
a. EL RUBOR DURANTE LA INFLAMACIÓN ES PRODUCTO DE LA
VASOCONSTRICCIÓN PERIFÉRICA.
b. EL CALOR DURANTE LA INFLAMACIÓN ES PRODUCTO DE LA
VASOCONSTRICCIÓN LOCAL.
c. EL CALOR DURANTE LA INFLAMACIÓN ES PRODUCTO DE LA
INFECCIÓN.
d. EL RUBOR DURANTE LA INFLAMACIÓN ES PRODUCTO DE LA
VASODILATACIÓN LOCAL.

Retroalimentación

La respuesta correcta es: EL RUBOR DURANTE LA INFLAMACIÓN ES


PRODUCTO DE LA VASODILATACIÓN LOCAL.

Pregunta 20

Enunciado de la pregunta

Respecto a la magnitud del daño causado por una tromboembolia pulmonar, seleccione
la respuesta correcta:
Seleccione una:
a. Es determinante el tamaño del émbolo y el diámetro de la arteria afectada
(obstruida)
b. Ningún factor es importante ya que el diagnóstico siempre es post mortem
c. Es únicamente importante la arteria que está siendo afectada (obstruida)
d. Lo principal es el tamaño, y son de importancia únicamente émbolos mayores a
2mm

Retroalimentación

La respuesta correcta es: Es determinante el tamaño del émbolo y el diámetro de la


arteria afectada (obstruida)

Pregunta 21

Enunciado de la pregunta

¿Qué evalúan los criterios de Wells para tromboembolia pulmonar?


Seleccione una:
a. La probabilidad clínica de tener tromboembolia pulmonar
b. Pronóstico de supervivencia para pacientes con tromboembolia pulmonar
c. Diagnóstico definitivo de tromboembolia pulmonar
d. Criterios radiológicos para diagnosticar tromboembolia pulmonar

Retroalimentación

La respuesta correcta es: La probabilidad clínica de tener tromboembolia pulmonar

Pregunta 22
Enunciado de la pregunta

LA FASCITIS NECROTIZANTE, SE CARACTERIZA POR:


Seleccione una:
a. INFECCIÓN AGUDA Y RÁPIDAMENTE PROGRESIVA DEL TEJIDO
CELULAR SUBCUTÁNEO
b. AFECTAR A LA EPIDERMIS
c. NO POSEE UNA ELEVADA MORBIMORTALIDAD
d. LA LOCALIZACIÓN MÁS FRECUENTE ES EL TÓRAX

Retroalimentación

La respuesta correcta es: INFECCIÓN AGUDA Y RÁPIDAMENTE PROGRESIVA


DEL TEJIDO CELULAR SUBCUTÁNEO

Pregunta 23

Enunciado de la pregunta

LAS LESIONES DE LA DERMATITIS ATÓPICA EN EL LACTANTE, SEÑALE LO


INCORRECTO:
Seleccione una:
a. PREDOMINIO EN ZONAS EXTENSORAS COMO CODOS Y RODILLAS.
b. SON MÁS HABITUALES ES EN LA CARA, FRENTE Y MEJILLAS Y
RETROAURICULAR
c. AFECTA EL TRONCO
d. LA DERMATITIS ATÓPICA NO SE PRESENTA EN LACTANTES.

Retroalimentación

La respuesta correcta es: LA DERMATITIS ATÓPICA NO SE PRESENTA EN


LACTANTES.

Pregunta 24

Enunciado de la pregunta

¿Ante un derrame pleural que esperaría encontrar a la percusión?


Seleccione una:
a. Hiperresonante
b. Resonante
c. Matidez
d. Timpánico

Retroalimentación
La respuesta correcta es: Matidez

Pregunta 25

Enunciado de la pregunta

CUÁLES SON LAS PRIMERAS CÉLULAS EN MIGRAR DURANTE EL PROCESO


DE LA INFLAMACIÓN:
Seleccione una:
a. PMN Y MACRÓFAGOS.
b. EOSINOFILOS Y BASÓFILOS.
c. LINFOCITOS Y MONOCITOS.
d. ERITROCITOS Y PLAQUETAS.

Retroalimentación

La respuesta correcta es: PMN Y MACRÓFAGOS.


Pregunta 1 Correaa Pumúa O.16 sobre O,16 'f Marcar pregunta

¿Cual de estas patologías con manitestaciones de sangrado purpurico, se identitica su trotis de sangre peritérica por la
presencia de plaquetas gigantes?

Seleccione una:

a. Síndrome urémico l1emolítico SUH

b. Enfermedad de Bernard Soulier j

c. Enfermedad de Von Willebra1d

d. Purpura trombocitopénica trombótica PTT

La respuesta correcta es: Enfermedad de Bernard Soulier

Pregunta 2 Incorrecta Pumúa 0,00 sobre 0, 16 'f Marcar pregunta

¿Cuál de las sigu ientes alternativas es el tratamiento sistémico de elección en impétigo contagioso?

Seleccione una:

a. Eritromicina X
u. ALilr umiUr Id
c. Dicloxacilina

cl .Ampicilina

La respuesta correcta es: Oícloxacñina


Pregunta 3 Incorrecta Puntúa 0,00 sobre O, 16 f' Marcar pregunta

Cuá ' es una causa de taquicardia sinusa l fisiológica:

Seleccione una:

a. Hipotiroidismo

b. Hipertensión pulmonar

c. Feocromocitoma

d. Sedenterismo X

La respuesta correcta es: Feocromocitoma

Pregunta 4 Correaa Pumúa O, 16 'Sobre O, 16 f' Marcar pregunta

¿Cuál no se considera causa para azotemia prerrenal?

Seleccione una:

a. Hiperplasia prostática

b. Diarrea aguda

c. Hemorragia digestiva alta

Pregunta 5 Correcta Puntúa O, 16sobre0,1 6 'f' M arcar pregunt a

¿Cuál no se considera un tratamiento para hiperpotasem ia?

Seleccione una:

a. Furosemida

b. Bicarbonato de calcio ~

c. Solución polartzante

d. Gluconato de calcio

La respuesta correcta es: Bicarbonato de calcio

Pregunta 6 Correcta Puntúa O. 16 sobre O,16 'f' Marca r pregu nta

Cuál síndrome genético es común en la insuficiencia aórtica:

Selecci one una:

a. Síndrome de Eisenmenger

b. Síndrome de Marfán

c. Anomalía de Ebstem

d. Síndrome de Rastelll

La respuesta correcta es: Síndrome de Marfán


Pregunta 7 Correcta Puntúa O, 16 sobre O, 16 f Marcar pregunta

Cuánto es la dosis intravenosa de sostén para digoxina en arritmias:

Seleccione una:

a. 0.125-0.250 mg/día

b. 0.145-0.250 mg/día

c. 0.175-0.250 mg/día
d. 0.200-0.250 mg/día

Pregunta8 lncorrec~a Puntúa 0,00 sobre O, 16 'f Marcar pregunta

Cuánto es la dosis máxima de dinitrato de isosorbide de liberación prolongaoa en angina de ¡:echo:

Seleccione una:
a. 300 mg
b.40mg X

c. 16C mg

d.120mg

La respuesta correcta es: 300 mg

Pregunta 9 Correcta Puntúa O,16 sobre O, 16 f Marcar pregunta

Cuánto es le dosis sublingual de nitroglicerina en angina de pecho:

Seleccione una:
a. 0.8 a 1.0 mg
b. 0.3 a0.6 mg ,

c. 3 a 6 mg
d.1.3a1 .6mg

La respuesta correcta es: 0.3 a O6 mg


Pregunta 10 lncorrec~a Puntúa 0,00 sobre 0,16 f Marcar pregunta

Cuánto es la frecuencia de administración de atenolol en cardiopatía isquémica:

Seleccione una:

a. Cada 6 horas

b. Cada 24 horas

c. Cada 12 horas X
d. Cada 8 horas

La respuesta correcta es: Cada 24 horas

Pregunta 11 lncorrttta Puntúa O, 00 sobre O, 16 f' Marcar pregunta

Cuánto es la frecuencia d e administración d e propanolol en cardiopatía isquémica:

Seleccione una:

a Cada 6 horas X

b. Cada 8 horas

c. Cada 12 horas
d. Cada 24 horas

La respuesta correcta es: Cad a 12 horas

Pregunta 12 Correcta Puntúa O,16 sobre O, 16 f Marcar pregunta

El índice de producción de reticulocrtos (IPR) es un estudio que mide la capacidad de respuesta d e la medula. Si t enemos u n IPR
menor de 2. ¿Cuál de estas anemias presentan un IPR menor de 2?

Seleccione una:

a. Hiperesplenísmo

b. Hípoplasia medular ✓

c. Anemia hemol1t1ca

d. Hemorragia aguda

La respuesta correcta es: Hlpoplas,a medular.


Pregunta 13 Correcta Puntúa O, 16 sobre O, 16 'f Marcar pregunta

El Síndro me de Alpor t que nefropatía produce?

Seleccione una:

a. Glomeruloesclerosis segmentaría focal ,


b. Glomerulonefritís membranoprolíferatíva

c. Nefropatía por lgA


d. Granulomatosís de Wegener

La respuesta correcta es: Glomeruloesclerosis segmentaría focal

Pregunta 14 \r.rn=•ru Punr(LA 0,1fi ~nhn•· fl, 1fi 'f Marcar pregunta

La Heparína no fraccionada (HNF) su uso puede causar algunas compl icac iones por su acción anticoagulante, sobre cuál factor
de coagulación actúa principalmente?

seleccione ura:

a. Facto· V

b. Factor 1

c. Factor VI 11
d. Factor II Protrombína

La respuesta correcta es: Factor II Pr:>trombina

Pregunta 15 Correcta Puntúa O 16 sobre O, 16 f' Marcar pregunla

Los productos de degradación de la fibrina (PDF) pooemos medirlos a t ravés d? los Dímeros o, su contaje elevado mcyor
de1000 (V.N 500) indicaría:

SglgccionQ u n a:

a. Hipertensión pulmo1ar

b. Neumonía.

c. -romboembolismo pulmonar ¿

d. Derrame Pleual

La respuesta correcta es: Trcmboembolismo pulmonar


Pregunta 16 Incorrecta Puntúa 0,00 sobre 0,16 'f Marcar pregunta

Paciente de sexo femen ino de 30 años de edad que presenta desde la infancia placas eritematodescamativas de bordes bien
definidos en codos. y rod illa con descamación intensa y prurito moderado. además presenta eritema y descamación profusa en
cuero cabelludo. ¿Cuál de las siguientes alternativas corresponde al diagnóstico clínico más probable?

Seleccione una:

a. Dermatitis atópica + dermatitis seborreica ole cuero cabelludo X


b. Psoriasís + dermatitis seborreica de cuero cabelludo

c. Psoriasis

d. Dermatitis seborreica + tiña corporis

La respuesta correcta es: Psoriasis

Pregunta 17 Correcta: Punrúa O, 16 sobre O,16 'f Marcar pregunta

Paciente de sexo masculino. ele 48 años de edad que presenta una tu mefacción eritematosa de bordes mal definidos. muy
dolorosa a la palpación en tórax post erior de consistencia semidura, con un pequeño orificio centra l a través del cual drena
material purulento ce mal olor de 8 días d e evoluc ón. ¿Cual de las sigu ientes alternativas corresponde al diagn5stico mas
probable?

Seleccione una:

a. Forúnculo
b. Erisipela

c. Celulitis

d. Follculltls pro'unda

La respuesta correcta es: ForGnculo


Pregunta 18 Incorrecta Puntúa 0,00 sobre 0,16 'f Marcar pregunta

Paciente de sexo masculino, de 48 años de edad que presenta una tumefacción eritematosa de bordes mal definidos, muy
dolorosa a la palpación en tórax posterior de consistencia semidura, con un pequeño orificio central a través del cual drena
material purulento de mal olor de 8 días de evolución. ¿Cuál de las siguientes alternativas indica las bacterias que con más
frecuencia ocasionan este cuadro?

Seleccione una:

a. Estreptococo beta hemolítico del grupo a, neumococo, eschericía coli

b. Estreptococo beta hemolítico del grupo a, pseudomona, h. Influenza X


c. Estafilococo aureus, bacterias anaerobias, gramnegativos
d. Estafilococo aureus, clostridium, pseudomona

La respuesta correcta es: Estafilococo aureus, bacterias anaerobias, gramnegativos

Pregunta 19 lncorreaa Puntúa 0,00 sobre O, 16 'f Marcar pregunta

Paciente de sexo masculino, de -18 años de edad que presenta una tumefacción eritemato;a de bordes mal defiridos, muy
dolorosa a a palpación en tórax posterior de consistenc a semidura, con un pequeño orificio ce1tral a través del cual drena
material purulento de mal olor de 8 día; de evolución. ¿Cuál de las siguientes alternctivas ndica los a1t ibióticos de ele:ción en
este caso?

Selecc one una:

a. Oicloxacilina y lhezolid X
b. Ceftriaxona y gentamicina

c. Oicloxacílína y trimetrJprim sulfametoY.azol

d. Amcxicilina + ácido cl;vulanico

La respuesta correcta es: Diclcxacilir a y tri11etr0Jrim sultametoxazJI

Pregunta 20 Correcta Puntúa 0.16 sobre 0.16 'f Marcar pregunta

Qué enfermedad reumatológica causa insuficiencia aórtica:

a. Vasculitis leucocitoclástíca

b. E,pondilitis anquilosante

c. Lupus er tematoso sistémico


d. Artrit is reumato dea

_a respuesta correcta es: tspond1lrus anqu11osante


Pregunta 21 Correcta Pu ntúa 0,16 sobre 0,16 f Marcar pregunta

Qué infección causa bloqueo auriculoventricular:

Seleccione una:

a. Sarampión

b. Varicela

c. Sífilis

d. Tripanosomiasis

La respuesta correcta es: Sífilis

Pregunta 22 Corre cta Puntúa 0, 15 sobre O, 16 f Marcar pregunta

¿Qué valor define anuria?

Seleccione una:

a. 1000 a 2000 mi

b. 100 a 400 mi

c. o a 100 mi

d. 500 a 1000 mi

La respuesta correcta es: Oa 100 mi

Pregunta 23 Correcta Puntúa 0,16 sobre O, 16 'f Marcar pregunta

Según Harrison. los factores que predisponen a las mujeres a padecer cistitis, aumentan el peligro de:

Seleccione una:

a. Uretntis
b. Vaginitis

c. Plelonefritis ,.

d. Absceso p erirenal

La respuesta correcta es: Pielonefntis


Pregunta 24 Correcta Puntúa 0, 16 sobre O, 16 'f Marca r pregunta

Si se necesita el aumento de plaquetas más efect ivo en una trombocitopenia severa inm une. ¿Qué utilizas de primera línea?

Seleccione una:

a. Metilprednisolo na

b. lnmunoglobulina

c. Rituximab anticuerpo monoclona l


d. Eltrombopag

La respuesta correcta es: lnmunoglobuli na

Pregunta 25 Correcta Puntúa 0,16 sobre O, 16 'f Marca r pregunta

Un paciente hipertenso con descompensación aguda "t ípica" de insuficiencia card iaca sin sobrecarga d e volumen, que fármaco
usaría :

Seleccione una:

a. Nitroprusiato

b. Furosem ida

c. Dobutam ina
d. Clortalidona

La respuesta correcta es: Nitroprusiato


2
ARIEL GUSTAVO TORO AGUILAR

Área personal  Mis cursos  Quito  CIENCIAS DE LA SALUD  MEDICINA - PRESENCIAL  ABR 2020 - AGO 2020
 INVESTIGACIÓN I - Prl: MD NVD Pen: 961  13 de junio: Examen Primer Parcial Investigación I  Examen Primer Parcial
Investigación I

Comenzado el sábado, 13 de junio de 2020, 10:10


Estado Finalizado
Finalizado en sábado, 13 de junio de 2020, 10:59
Tiempo empleado 49 minutos 46 segundos
Calificación 3,04 de 4,00 (76%)

Pregunta 1 Correcta Puntúa 0,16 sobre 0,16

Cuál de los siguientes es un signo característico de las vías respiratorias de las personas que sufren
asma:

Seleccione una:

o a. In ltración basó la

o b. Transformación de los basó los en macrófagos

o c. Adhesión de los glóbulos rojos al endotelio

@ d. In ltración eosinofílica

La respuesta correcta es: In ltración eosinofílica

/
Pregunta 2 Correcta Puntúa 0,16 sobre 0,16

¿Cuál es la clasi cación anatómica de las bronquiectasias?

Seleccione una:

o a. Lobares, arteriales, globulares

@ b. Cilíndricas, varicosas, saculares

o c. Vasculares, semilunares saculares

o d. Cubicas, cilíndricas, alargadas

La respuesta correcta es: Cilíndricas, varicosas, saculares

Pregunta 3 Correcta Puntúa 0,16 sobre 0,16

¿Cuál es la siopatología de la hemoptisis?

Seleccione una:

@ a. Hipervascularización de la circulación brónquica, hipertensión pulmonar y neovascularización

o b. Hipervascularización de la circulación pulmonar, ebre y regeneración alveolar

o c. Hipervascularización de la circulación pulmonar, hipertensión pulmonar y remodelación.

o d. Hipervascularización de la circulación brónquica, hipertensión pulmonar y disminución de


coagulabilidad

La respuesta correcta es: Hipervascularización de la circulación brónquica, hipertensión pulmonar y


neovascularización

/
Pregunta 4 Correcta Puntúa 0,16 sobre 0,16

De los siguientes factores, cuál es el que debe estar alterado para que un paciente infectado de
tuberculosis se convierta en persona enferma:

Seleccione una:

@ a. Huésped: Inmunidad celular de ciente, en especial CD4

o b. Huésped: que genéticamente esté predispuesto a infectarse

o c. Medio ambiente: Que haya elevada prevalencia de tuberculosis

o d. Medio Ambiente: Que viva en un lugar frío

La respuesta correcta es: Huésped: Inmunidad celular de ciente, en especial CD4

Pregunta 5 Correcta Puntúa 0,16 sobre 0,16

Dentro de las manifestaciones tardías de la sí lis tenemos trastornos cardiovasculares de cual podemos
desprender el siguiente:

Seleccione una:

o a. Trastornos del endotelio

o b. Insu ciencia cardiaca congestiva.

o c. Flebitis irritativa.

@ d. Aneurismas

La respuesta correcta es: Aneurismas

/
Pregunta 6 Correcta Puntúa 0,16 sobre 0,16

¿De qué depende la inmunidad contra Leptospira?

Seleccione una:

o a. Producción de anticuerpos contra los distintos serotipos.

o b. Producción de anticuerpos contra LPS inespecí cos.

o c. Producción de anticuerpos contra proteínas inespecí cas de un serotipo.

@ d. Producción de anticuerpos circulantes contra LPS especí cos de un serotipo.

La respuesta correcta es: Producción de anticuerpos circulantes contra LPS especí cos de un serotipo.

Pregunta 7 Correcta Puntúa 0,16 sobre 0,16

El hallazgo más frecuente en la radiografía de tórax en un paciente con asma es:

Seleccione una:

o a. Engrosamiento de paredes bronquiales.

@ b. Radiografía de tórax normal.

o c. Condensaciones alveolares bilaterales y difusas.

o d. Hiperinsu ación pulmonar.

La respuesta correcta es: Radiografía de tórax normal.

/
Pregunta 8 Correcta Puntúa 0,16 sobre 0,16

El mecanismo por el cual la infección por cólera produce una diarrea tan intensa es por:

Seleccione una:

o a. Toxina Shiga-like

o b. Vibrios entero hemorrágicos.

o c. Vibrios enteroagregativos.

@ d. Toxina enterogénica

La respuesta correcta es: Toxina enterogénica

Pregunta 9 Correcta Puntúa 0,16 sobre 0,16

El siguiente enunciado: Un estado parecido al sueño profundo en el que el paciente permanece con los
ojos cerrados y el paciente no puede ser despertado, se re ere a:

Seleccione una:

@ a. Estado de coma

o b. Estado vegetativo

o c. Somnolencia

o d. Estupor

La respuesta correcta es: Estado de coma

/
Pregunta 10 Correcta Puntúa 0,16 sobre 0,16

El tratamiento de las Bronquiectasias se basa en 3 pilares, excepto:

Seleccione una:

@ a. Revertir el remodelamiento bronquia

o b. Mejorar la eliminación de las secreciones, que se consigue con una adecuada hidratación, con
sioterapia respiratoria y drenaje postural mantenidos.

o c. Controlar las infecciones con el uso de antibióticos en las agudizaciones durante 10-15 días.

o d. Eliminar la obstrucción bronquial.

La respuesta correcta es: Revertir el remodelamiento bronquia

Pregunta 11 Incorrecta Puntúa 0,00 sobre 0,16

En cuanto a la pro laxis antitetánica en el tratamiento sistemático de las heridas, en una herida
pequeña limpia sin ningún antecedente de vacunación antitetánica Usted recomendaría:

Seleccione una:

@ a. Vacunación antitetánica de inmediato. X


o b. Es indicativo de inmunoglobulina.

o c. Solo limpiar la herida y dar indicaciones de cuidado al paciente, ofrecer la anti toxina.

o d. Dar antibiótico de manera pro láctica.

La respuesta correcta es: Solo limpiar la herida y dar indicaciones de cuidado al paciente, ofrecer la anti
toxina.

/
Pregunta 12 Correcta Puntúa 0,16 sobre 0,16

En cuanto al diagnóstico funcional del asma:

Seleccione una:

o a. Si la relación VEF1/CVF es menor a 0.7 (patrón restrictivo) y post broncodilatador obtenemos un


aumento igual o mayor al 12% en el VEF1, nos orienta hacia diagnóstico de asma

o b. Si la relación VEF1/CVF es mayor 0.7 se considera patrón obstructivo, diagnosticamos asma

@ c. Si la relación VEF1/CVF es menor a 0.7 (patrón obstructivo) y post broncodilatador obtenemos un


aumento igual o mayor al 12% en el VEF1, nos orienta hacia diagnóstico de asma

o d. Valores espirométricos no tienen importancia en el diagnóstico del asma.

La respuesta correcta es: Si la relación VEF1/CVF es menor a 0.7 (patrón obstructivo) y post broncodilatador
obtenemos un aumento igual o mayor al 12% en el VEF1, nos orienta hacia diagnóstico de asma

Pregunta 13 Correcta Puntúa 0,16 sobre 0,16

En el manejo de un paciente con delirium agitado (o hiperactivo), de las siguientes opciones, señale la
que NO incluiría en su prescripción médica:

Seleccione una:

o a. Evitar uso de restricciones físicas (amarras) para controlar el movimiento del paciente

o b. Antipsicóticos atípicos como la quetiapina en vía oral

@ c. Benzodiacepina como el clonazepam en vía oral

o d. Manejo no farmacológico como: colocar un reloj visible para el paciente

La respuesta correcta es: Benzodiacepina como el clonazepam en vía oral

/
Pregunta 14 Correcta Puntúa 0,16 sobre 0,16

En la clasi cación de OMS de adultos con VIH – Sida en el estadio C3 tenemos a:

Seleccione una:

o a. Mayor de 400 células CD4+ con síntomas no de nidores de Sida.

@ b. Menor de 200 células CD4+ con síntomas de nidores de Sida.

o c. Menor de 300 células CD4+ con síntomas de nidores de Sida.

o d. Mayor de 500 células CD4+ Asintomático.

La respuesta correcta es: Menor de 200 células CD4+ con síntomas de nidores de Sida.

Pregunta 15 Correcta Puntúa 0,16 sobre 0,16

En pacientes con catatonía, usted sospecharía:

Seleccione una:

o a. Lesión de la región orbitaria frontal

o b. Secuela del coma

o c. Herniación central

@ d. Trastorno psiquiátrico

La respuesta correcta es: Trastorno psiquiátrico

/
Pregunta 16 Incorrecta Puntúa 0,00 sobre 0,16

En relación a los siguientes enunciados acerca del ictus isquémico o enfermedad cerebrovascular
isquémica, señale el verdadero:

Seleccione una:

o a. El uso de estatinas reduce el riesgo de ictus isquémico inclusive con niveles normales de LDL o
niveles bajos de HDL

o b. Cualquier fármaco antiplaquetario (aspirina, clopidogrel, ticlopidina) son e caces y aprobados para
el uso en fase aguda de un ictus isquémico

c. Un paciente con brilación auricular valvular (ej enfermedad valvular reumática) requiere uso de
anticoagulantes como prevención primaria o secundaria X
o d. El tratamiento de HTA como prevención primaria con cifras de TA sistólica &lt;120 mm Hg reduce
en 43% la presencia de ictus y ataques del corazón

La respuesta correcta es: Cualquier fármaco antiplaquetario (aspirina, clopidogrel, ticlopidina) son e caces y
aprobados para el uso en fase aguda de un ictus isquémico

Pregunta 17 Incorrecta Puntúa 0,00 sobre 0,16

En un paciente de 68 años de edad, con APP: HTA, que ingresa con un cuadro de crisis focales motoras
sin pérdida del estado de conciencia y limitadas al miembro superior derecho, que se han repetido 1
crisis cada 3 días desde hace 2 semanas, además se acompaña de cefalea desde hace 3 meses con
características de empeoramiento progresivo, en los estudios de imagen se encontró una lesión
expansiva frontal izquierda; en relación al tratamiento farmacológico de elección, señale el que
escogería:

Seleccione una:

(!) a. Acido valproico X


o b. Lamotrigina

o c. Fenobarbital

o d. Clonazepam

La respuesta correcta es: Lamotrigina

/
Pregunta 18 Correcta Puntúa 0,16 sobre 0,16

La causa principal de un fracaso en el tratamiento de la tuberculosis pulmonar es

Seleccione una:

o a. Resistencia secundaria a las drogas

o b. Resistencia primaria a las drogas

@ c. Abandono del tratamiento por el paciente

o d. Toxicidad hepática

La respuesta correcta es: Abandono del tratamiento por el paciente

Pregunta 19 Incorrecta Puntúa 0,00 sobre 0,16

La ictericia en un paciente con Malaria es frecuente a partir:

Seleccione una:

o a. La ictericia no aparece en la malaria.

@ b. Siempre existirá ebre e ictericia debido a la destrucción de eritrocitos. X


o c. La Ictericia en Malaria es frecuente a partir de los 15 años.

o d. Ocasionalmente en niños debido a su alta susceptibilidad

La respuesta correcta es: La Ictericia en Malaria es frecuente a partir de los 15 años.

/
Pregunta 20 Correcta Puntúa 0,16 sobre 0,16

Paciente de 16 años de género masculino, sin APP, empieza con crisis convulsivas tónico clónicas
generalizadas hace 1 año, 1 crisis cada 2 meses, su examen neurológico es normal al igual que la
Resonancia Magnética cerebral, el electroencefalograma demostró actividad epileptiforme, cuál de las
siguientes etiologías consideraría en el diagnóstico

Seleccione una:

o a. Epilepsia por enfermedad degenerativa

@ b. Epilepsia por desorden genético

o c. Epilepsia secundaria a disturbios hidroelectrolíticos

o d. Epilepsia secundaria a autoanticuerpos

La respuesta correcta es: Epilepsia por desorden genético

Pregunta 21 Correcta Puntúa 0,16 sobre 0,16

Paciente de 22 años, género femenino, tiene desde hace 1 año dolor de cabeza de forma episódica (2
episodios al mes), hemicránea derecha o izquierda, pulsátil, intensidad moderada, se acompaña de
náusea y fotofobia, el dolor es precedido por escotomas centellantes que duran 90 minutos
aproximadamente, en los últimos 3 meses el dolor se ha tornado más frecuente (3 episodios semanales)
y al toser exacerba en intensidad a dolor severo, de los siguientes enunciados cuál NO considera
signo/síntoma de alarma:

Seleccione una:

@ a. Localización del dolor

o b. Valsalva positivo

o c. Aura de duración larga

o d. Empeoramiento del dolor (más frecuente)

La respuesta correcta es: Localización del dolor

/
Pregunta 22 Incorrecta Puntúa 0,00 sobre 0,16

Paciente de 38 años de género femenino, con APP diagnóstico de fenómeno de Raynaud en estudio de
un posible Lupus eritematoso sistémico, también diagnóstico de migraña con aura desde la juventud,
acude por un ataque agudo de cefalea, usted qué fármaco NO recomendaría:

Seleccione una:

o a. AINES

o b. Aspirina + metoclopramida

@ c. Clorpromazina X
o d. Sumatriptán

La respuesta correcta es: Sumatriptán

Pregunta 23 Correcta Puntúa 0,16 sobre 0,16

Paciente masculino de 66 años de edad, con antecedentes de deterioro cognitivo leve, HTA, ingresa para
una cirugía electiva de resección prostática por una hipertro a prostática benigna, en su postoperatorio
se encuentra con sondaje (cateterización), vesical permanente, dolor pélvico, y un cuadro de delirium (o
sd confusional agudo), se reinició el enalapril en el postoperatorio, de las siguientes opciones señale
cuál NO es un factor de riesgo para el aparecimiento de delirium

Seleccione una:

o a. Dolor en el postoperatorio

o b. Antecedentes de deterioro cognitivo leve

@ c. Administración de enalapril

o d. Sondaje vesical

La respuesta correcta es: Administración de enalapril

/
Pregunta 24 Incorrecta Puntúa 0,00 sobre 0,16

Tras la infección de un niño con el virus de la In uenza con la siguientes características: ebre intensa de
inicio súbito, disnea y cianosis a la Rx de tórax se aprecia patrón asociado con in ltrados intersticiales
difusos e hipoxia intensa, usted sospecharía de:

Seleccione una:

@ a. Neumonía viral secundaria. X


o b. Neumonía viral primaría.

o c. Neumonía bacteriana primaria.

o d. Neumonía bacteriana secundaria.

La respuesta correcta es: Neumonía viral primaría.

Pregunta 25 Correcta Puntúa 0,16 sobre 0,16

Una de las características principales por las cuales el dengue puede diseminarse es:

Seleccione una:

o a. Es sumamente frecuente la progresión de daño encefálico.

o b. Una característica precoz es la trombocitosis y neutro lia.

o c. Debido a la gran capacidad de diseminación del vector Anopheles.

@ d. El vector es muy cercano a los asentamientos humanos.

La respuesta correcta es: El vector es muy cercano a los asentamientos humanos.

◄ Avisos Ir a... V Link sesión zoom examen primer parcial ►

/
JOSELYN SOFIA BARAJA CASTILLO

Área personal  Mis cursos  Quito  CIENCIAS DE LA SALUD  MEDICINA - PRESENCIAL  ABR 2020 - AGO 2020
 INVESTIGACIÓN I - Prl: MD NVA Pen: 961  Sábado 4 de julio 8:10 am. Investigación I: Examen nal 1  Examen nal 1:
Investigación I

Comenzado el sábado, 4 de julio de 2020, 08:20


Estado Finalizado
Finalizado en sábado, 4 de julio de 2020, 09:27
Tiempo empleado 1 hora 7 minutos
Cali cación 4,40 de 4,67 (94%)

[Pregunta 1 Correcta Puntúa 0,13 sobre 0,13

¿Cuál de estos fármacos tiene acción potenciadora de los anticoagulantes?

Seleccione una:

@ a. Ácido Acetil Salicílico

o b. Vitamina K

o c. Rifampicina

o d. Barbitúricos

La respuesta correcta es: Ácido Acetil Salicílico


/
Pregunta 2 Correcta Puntúa 0,13 sobre 0,13

¿Cuál es la acción de los nitratos en el síndrome coronario agudo?

Seleccione una:

o a. sedante

@ b. vasodilatador coronario

o c. antiagregante

o d. revascularizador

La respuesta correcta es: vasodilatador coronario

Pregunta 3 Correcta Puntúa 0,13 sobre 0,13

¿Cuál es la causa más frecuente de Trombo lia congénita?

Seleccione una:

o a. Dé cit de antitrombina III

@ b. Resistencia a la proteína C

o c. Dé cit de proteína S

o d. Dé cit de proteína C

La respuesta correcta es: Resistencia a la proteína C


/
Pregunta 4 Correcta Puntúa 0,13 sobre 0,13

¿Cuál es la función de la aspirina en el síndrome coronario agudo?

Seleccione una:

@ a. Evitar el incremento de la agregación plaquetaria en la placa ateromatosa accidentada

o b. Anticoagulantes

o c. Analgésico antin amatorio

o d. Trombolisis

La respuesta correcta es: Evitar el incremento de la agregación plaquetaria en la placa ateromatosa


accidentada

Pregunta 5 Correcta Puntúa 0,13 sobre 0,13

¿Cuáles son signos de insu ciencia cardíaca derecha?

Seleccione una:

@ a. Edema de miembros inferiores, hepatomegalia

o b. Dolor abdominal, astenia

o c. Disnea, estertores crepitantes

o d. Ortópnea, dolor precordial

La respuesta correcta es: Edema de miembros inferiores, hepatomegalia


/
Pregunta 6 Correcta Puntúa 0,13 sobre 0,13

Después de una transfusión, puede ocurrir una reacción adversa, y estas pueden ser inmediatas o
tardías. ¿Cuál de estas es una reacción transfusional tardía?

Seleccione una:

o a. sobrecarga circulatoria

o b. Fiebre escalofríos

o c. Reacción alérgica

@ d. Transmisión de enfermedad

La respuesta correcta es: Transmisión de enfermedad

Pregunta 7 Correcta Puntúa 0,13 sobre 0,13

El 10 % de las neoplasias malignas del esófago aparecen en:

Seleccione una:

o a. Es indiferente

@ b. Tercio superior

o c. Tercio medio

o d. Tercio distal

La respuesta correcta es: Tercio superior


/
Pregunta 8 Correcta Puntúa 0,13 sobre 0,13

El primer estudio de imagen que se debe solicitar ante la sospecha clínica de cirrosis hepática es:

Seleccione una:

o a. Ecografía abdomen inferior

o b. Ecografía abdomen.

o c. Ecografía doppler portal

@ d. Ecografía abdomen superior

La respuesta correcta es: Ecografía abdomen superior

Pregunta 9 Correcta Puntúa 0,13 sobre 0,13

El uso de betabloqueantes debe ser analizado en el paciente con insu ciencia cardiaca ya que este
grupo farmacológico posee ciertas características importantes. Señale lo correcto respecto a estos
productos:

Seleccione una:

o a. Pueden utilizarse con seguridad en combinación con verapamilo

o b. Pueden utilizarse con seguridad en pacientes con hipotensión sintomático

@ c. Pueden utilizarse con seguridad en pacientes con insu ciencia cardiaca sistólica y angina de
esfuerzo

o d. Puede utilizarse con seguridad en pacientes con asma

La respuesta correcta es: Pueden utilizarse con seguridad en pacientes con insu ciencia cardiaca sistólica y
angina de esfuerzo


/
Pregunta 10 Correcta Puntúa 0,13 sobre 0,13

En el manejo de la erisipela se plantea como primera elección los betalactámicos. Señale lo correcto
respecto a este grupo:

Seleccione una:

o a. Su mecanismo de acción consiste en el bloqueo ribosomal de la síntesis de peptidoglicano

o b. No presenta efecto antibiótico en los gram negativos por la presencia de su cápsula bacteriana

o c. Afecta la síntesis de mureina desde su fase 2 en las bacterias gram positivas

@ d. Su acoplamiento irreversible a las PBP es característico en los gérmenes gram positivos

La respuesta correcta es: Su acoplamiento irreversible a las PBP es característico en los gérmenes gram
positivos

Pregunta 11 Correcta Puntúa 0,13 sobre 0,13

En la cirrosis biliar primaria ¿cuál de los siguientes anticuerpos se presentan en cerca del 90 % de las
personas que padecen esta entidad?

Seleccione una:

o a. AntI KLM2

@ b. AMA

o c. Anti DNA

o d. Asma

La respuesta correcta es: AMA


/
Pregunta 12 Correcta Puntúa 0,13 sobre 0,13

¿En qué caso complementamos el electrocardiograma con derivaciones derechas?

Seleccione una:

o a. Ritmo de galope

o b. Taquicardia ventricular monomorfa

o c. Sospecha de edema agudo de pulmón

@ d. Sospecha de infarto de ventrículo derecho

La respuesta correcta es: Sospecha de infarto de ventrículo derecho

Pregunta 13 Correcta Puntúa 0,13 sobre 0,13

La anemia de proceso crónico tiene problema de movilización del hierro. Por tener hierro sérico bajo se
confunde con la de ciencia de hierro. ¿Cuál estudio hace la diferencia?

Seleccione una:

o a. Hierro sérico bajo

@ b. Ferritina normal o aumentada.

o c. TIBC capacidad de combinación total

o d. Hemoglobina baja

La respuesta correcta es: Ferritina normal o aumentada.


/
Pregunta 14 Correcta Puntúa 0,13 sobre 0,13

La endoscopia ha sustituido al esofagograma para el estudio inicial de la patología esofágica EXCEPTO


en:

Seleccione una:

@ a. Trastorno motor esofágico.

o b. Neoplasia.

o c. Varices esofágicas.

o d. Enfermedad por re ujo.

La respuesta correcta es: Trastorno motor esofágico.

Pregunta 15 Correcta Puntúa 0,13 sobre 0,13

La enfermedad pulmonar obstructiva crónica no tratada puede conducir a la siguiente complicación


cardíaca:

Seleccione una:

o a. Pericarditis

o b. Bradicardia

@ c. Insu ciencia cardíaca derecha

o d. Síndrome coronario agudo

La respuesta correcta es: Insu ciencia cardíaca derecha


/
Pregunta 16 Correcta Puntúa 0,13 sobre 0,13

La insu ciencia cardíaca izquierda puede provocar edema agudo de pulmón por aumento de la:

Seleccione una:

@ a. Presión capilar pulmonar

o b. Presión arterial

o c. Presión venosa central

o d. Presión sistólica del ventrículo derecho

La respuesta correcta es: Presión capilar pulmonar

Pregunta 17 Correcta Puntúa 0,13 sobre 0,13

La principal forma de acceso de microorganismos a las vías urinarias es:

Seleccione una:

o a. Hematógena

o b. Por contigüidad

@ c. Ascendente

o d. Linfática

La respuesta correcta es: Ascendente


/
Pregunta 18 Correcta Puntúa 0,13 sobre 0,13

La principal lesión que debe descartarse por imagen en el hígado cirrótico es:

Seleccione una:

@ a. Hepatocarcinoma.

o b. Hemangioma hepático

o c. Colangiocarcinoma

o d. Hiperplasia nodular focal

La respuesta correcta es: Hepatocarcinoma.

Pregunta 19 Correcta Puntúa 0,13 sobre 0,13

Las infecciones cutáneas provocadas por los virus del herpes son tratadas, como primera opción, con
Aciclovir. Señale lo correcto respecto a este producto:

Seleccione una:

o a. La a nidad de timidincinasa al Aciclovir es similar a la de las células del huésped

o b. El efecto farmacológico de Aciclovir inicia luego del efecto del primer paso hepático

@ c. Su acción selectiva es a virus que poseen timidincinasas

o d. Su acción puede desarrollarse en virus en replicación y en reposo

La respuesta correcta es: Su acción selectiva es a virus que poseen timidincinasas


/
Pregunta 20 Correcta Puntúa 0,13 sobre 0,13

Los fármacos de primera elección contra la pielonefritis aguda no complicada son:

Seleccione una:

o a. Aminoglucósidos

o b. Cefalosporinas

o c. Betalactámicos

@ d. Fluoroquinolonas

La respuesta correcta es: Fluoroquinolonas

Pregunta 21 Correcta Puntúa 0,13 sobre 0,13

Los Inhibidores de la bomba de protones son el pilar terapéutico fundamental para el manejo de la
enfermedad ácido péptico. El mecanismo de acción de esta droga sostiene que:

Seleccione una:

o a. El mecanismo de acción provoca una importante hipersecreción de rebote

o b. El medio ácido gástrico incrementa la liposolubilidad del principio activo

o c. Al pH siológico Omeprazol posee mayor actividad farmacológica

@ d. El fármaco activo queda atrapado en el canalículo de la célula parietal para desarrollar su


mecanismo de acción

La respuesta correcta es: El fármaco activo queda atrapado en el canalículo de la célula parietal para
desarrollar su mecanismo de acción


/
Pregunta 22 Incorrecta Puntúa 0,00 sobre 0,13

María, 65 años, acude a la consulta re riendo cansancio a los moderados esfuerzos, re ere haber tenido
ebre reumática, al examen físico presenta PA 130/80, FC 88lpm, soplo sistólico en quinto espacio
intercostal línea media clavicular izquierda III/VI, con irradiación a la axila. ¿Cuál es el diagnóstico más
probable en esta paciente?

Seleccione una:

@ a. Insu ciencia aortica X


o b. Estenosis aortica

o c. Insu ciencia mitral

o d. Estenosis mitral

La respuesta correcta es: Insu ciencia mitral

Pregunta 23 Correcta Puntúa 0,13 sobre 0,13

Paciente con albuminuria de 30 a 300 mg en 24 horas en orina ¿con qué estadio de la nefropatía
diabética se corresponde?

Seleccione una:

o a. Estadio 2

o b. Estadio 5

o c. Estadio 1

@ d. Estadio 3

La respuesta correcta es: Estadio 3


/
Pregunta 24 Correcta Puntúa 0,13 sobre 0,13

Paciente masculino acude a la consulta re riendo secreción uretral o disuria, por lo general sin
polaquiuria, se realiza un cultivo de la secreción que reporta C. trachomatis, ¿qué diagnóstico usted
sospecha?

Seleccione una:

@ a. Uretritis

o b. Pielonefritis

o c. Prostatitis

o d. Cistitis

La respuesta correcta es: Uretritis

Pregunta 25 Correcta Puntúa 0,13 sobre 0,13

Paciente masculino, adulto medio, que acude a la consulta re riendo dolor de cabeza, malestar general,
anorexia, dolor en el costado, ha notado disminución del volumen urinario, al examen físico se constata
cifras elevadas de tensión arterial y leve edema de miembros inferiores, se le realiza un examen de
orina evidenciándose hematuria con cilindros eritrociticos, piuria, además se realiza creatinina sérica la
cual está elevada con relación a valores previos, ¿en este caso usted considera que este paciente
presenta?:

Seleccione una:

o a. Síndrome de la membrana basal

o b. Síndrome nefrótico

@ c. Síndrome nefrítico

o d. Síndrome por enfermedades infecciosas

La respuesta correcta es: Síndrome nefrítico


/
Pregunta 26 Incorrecta Puntúa 0,00 sobre 0,13

Qué antihipertensivo se debe usar con cautela en un paciente con arteriopatia coronaria grave:

Seleccione una:

@ a. Clonidina X
o b. Espironolactona

o c. Atenolol

o d. Hidralazina

La respuesta correcta es: Hidralazina

Pregunta 27 Correcta Puntúa 0,13 sobre 0,13

Qué endocrinopatía es causa secundaria de hipertensión arterial:

Seleccione una:

o a. Hipogonadismo

o b. Hipocalcemia

@ c. Síndrome de Cushing

o d. Hiperprolactinemia

La respuesta correcta es: Síndrome de Cushing


/
Pregunta 28 Correcta Puntúa 0,13 sobre 0,13

Señale lo verdadero de la ESCABIOSIS

Seleccione una:

o a. Una característica es la afectación de la cara en adultos

o b. Se acompaña de prurito diurno

o c. La bacteria causal es la Borrelia

@ d. La lesión elemental son pápulas discretas pareadas

La respuesta correcta es: La lesión elemental son pápulas discretas pareadas

Pregunta 29 Correcta Puntúa 0,13 sobre 0,13

Señale lo verdadero del HERPES ZOSTER

Seleccione una:

o a. Es causado por el virus del papiloma

o b. La lesión elemental son vesículas umbilicadas agrupadas

o c. Nunca se acompaña de dolor o neuralgia

@ d. Las lesiones siguen el trayecto del dermatoma

La respuesta correcta es: Las lesiones siguen el trayecto del dermatoma


/
Pregunta 30 Correcta Puntúa 0,13 sobre 0,13

Señale lo verdadero del IMPÉTIGO CONTAGIOSO

Seleccione una:

o a. Se acompaña de ebre

@ b. Una característica es la costra melicérica

o c. La bacteria causal es la pseudomona

o d. La lesión elemental es el nódulo

La respuesta correcta es: Una característica es la costra melicérica

Pregunta 31 Correcta Puntúa 0,13 sobre 0,13

Una de las complicaciones características del paciente con Hepatopatía crónica es la ascitis. Su manejo
farmacológico plantea que:

Seleccione una:

o a. Es necesario iniciar siempre con dosis elevadas de los diuréticos para obtener la reversión de la
ascitis

b. La potencia natriurética de Espironolactona es mayor que la de furosemida en el paciente cirrótico

o c. Es necesario asociar la terapia diurética con espironolactona y furosemida a un aporte de sales de


potasio

o d. El uso de furosemida como monoterapia está indicado

La respuesta correcta es: La potencia natriurética de Espironolactona es mayor que la de furosemida en el


paciente cirrótico


/
Pregunta 32 Correcta Puntúa 0,13 sobre 0,13

Un ejecutivo de 55 años de edad se atiende por dolor epigástrico intenso. En la endoscopia


gastroduodenal se encuentra una ulcera grande en el bulbo duodenal. Se indican una dieta e inhibidor
de bomba de protones, pero los síntomas persisten. ¿Cuál de las siguientes sería la alternativa de
tratamiento más apropiada en este momento?

Seleccione una:

o a. Tratamiento con domperidona y antiácidos.

@ b. Tratamiento de erradicación de helicobacter pylori.

o c. Tratamiento con ranitidina.

o d. Tratamiento con omeprazol.

La respuesta correcta es: Tratamiento de erradicación de helicobacter pylori.

Pregunta 33 Correcta Puntúa 0,13 sobre 0,13

Un hombre de 69 años con hepatitis crónica por virus C y cirrosis sin antecedentes previos de
hemorragia varicial, acude por el hallazgo endoscópico de várices esofágicas grado III. ¿Cuál es el agente
farmacológico de preferencia para la prevención de sangrado varicial?

Seleccione una:

o a. Pantoprazol.

o b. Cimeidina.

o c. 5 mononitrato de isosorbide.

@ d. Carvedilol.

La respuesta correcta es: Carvedilol.


/
Pregunta 34 Correcta Puntúa 0,13 sobre 0,13

Un paciente de 52 años de edad con antecedente de diarrea ocasionado por un trastorno intestinal que
afecta la porción terminal de íleon acude a consulta. ¿Cuál de estos nutrientes está probablemente más
afectado?

Seleccione una:

o a. Hierro

o b. Aminoácidos

@ c. Cobalamina

o d. Ácido fólico

La respuesta correcta es: Cobalamina

Pregunta 35 Correcta Puntúa 0,13 sobre 0,13

Un paciente varón de 55 años acude a su consulta por diarrea de 8 días de evolución luego de la toma
de antibióticos para una neumonía. ¿Cuál sería el examen inicial de su elección?

Seleccione una:

o a. Colonoscopía con biopsias.

@ b. Coproparasitario con investigación de polimorfonucleares. Toxina A y B de Clostridium di cile.

o c. Test de sudan III.

o d. Anticuerpos para enfermedad celiaca.

La respuesta correcta es: Coproparasitario con investigación de polimorfonucleares. Toxina A y B de


Clostridium di cile.

◄ Link sesión zoom examen primer parcial Ir a... V

Link sesión zoom examen nal 1: Investigación II ►


/
JOSELYN SOFIA BARAJA CASTILLO

Área personal  Mis cursos  Quito  CIENCIAS DE LA SALUD  MEDICINA - PRESENCIAL  ABR 2020 - AGO 2020
 INVESTIGACIÓN I - Prl: MD NVA Pen: 961  22 de agosto Dra Nilda Iriarte  Examen nal 2

Comenzado el sábado, 22 de agosto de 2020, 09:10


Estado Finalizado
Finalizado en sábado, 22 de agosto de 2020, 10:29
Tiempo empleado 1 hora 19 minutos
Cali cación 4,40 de 5,33 (83%)

Pregunta 1 Incorrecta Puntúa 0,00 sobre 0,13

Al medir la TSH, si sospechamos de un nódulo tiroideo, y el resultado de la misma es baja. ¿Cuál de las
opciones sería adecuada de seguir?

Seleccione una:

o a. Con rmar el hipotiroidismo o considere el hipertiroidismo central

o b. Mida la tiroxina libre (FT4) y la triyodotironina total o libre (FT3)

o c. Hacer RMN cervical

(!) d. La TSH baja es una indicación inmediata de biopsia con aguja na. X

La respuesta correcta es: Mida la tiroxina libre (FT4) y la triyodotironina total o libre (FT3)


/
Pregunta 2 Correcta Puntúa 0,13 sobre 0,13

Ante un derrame pleural que esperaría encontrar a la percusión?

Seleccione una:

o a. Hiperresonante

o b. Timpánico

@ c. Matidez

o d. Resonante

La respuesta correcta es: Matidez

Pregunta 3 Correcta Puntúa 0,13 sobre 0,13

¿Cuál de las siguientes complicaciones es la más frecuente en una tiroidectomía total?

Seleccione una:

@ a. Hipoparatiroidismo transitorio.

o b. Hematoma cervical

o c. Hipoparatiroidismo permanente

o d. Lesión del nervio laríngeo recurrente

La respuesta correcta es: Hipoparatiroidismo transitorio.


/
Pregunta 4 Correcta Puntúa 0,13 sobre 0,13

Cuál de las siguientes opciones es una contraindicación para usar Metformina

Seleccione una:

o a. Insu ciencia respiratoria compensada

o b. Estado hipoosmolar hipoglucémico

@ c. Aclaramiento de creatinina menor a 30 mL/min/1.73m2.

o d. Insu ciencia cardíaca con fracción de eyección > 50%

La respuesta correcta es: Aclaramiento de creatinina menor a 30 mL/min/1.73m2.

Pregunta 5 Incorrecta Puntúa 0,00 sobre 0,13

Cuál de las siguientes opciones se considera como factor de riesgo para Diabetes tipo 2

Seleccione una:

o a. Peso al nacer < 300gr

@ b. Perímetro de cintura > 90cm mujeres X


o c. Perímetro de cintura > 80cm hombres

o d. IMC > 25kg/m2.

La respuesta correcta es: IMC > 25kg/m2.


/
Pregunta 6 Correcta Puntúa 0,13 sobre 0,13

¿Cuál de los siguientes componentes de la estructura de los virus gripales es el principal responsable de
su infecciosidad?

Seleccione una:

o a. Envoltura lipídica.

o b. Neuraminidasa.

@ c. Hemaglutinina.

o d. ARN polimerasa.

La respuesta correcta es: Hemaglutinina.

Pregunta 7 Correcta Puntúa 0,13 sobre 0,13

Cual de los siguientes es un signo característico de las vías respiratorias de las personas que sufren
asma:

Seleccione una:

o a. Transformación de los basó los en macrófagos

o b. Adhesión de los glóbulos rojos al endotelio

@ c. In ltración eosinofílica

o d. In ltración basó la

La respuesta correcta es: In ltración eosinofílica


/
Pregunta 8 Correcta Puntúa 0,13 sobre 0,13

¿Cuál es la siopatología de la hemoptisis?

Seleccione una:

o a. Hipervascularización de la circulación brónquica, hipertensión pulmonar y disminución de


coagulabilidad

o b. Hipervascularización de la circulación pulmonar, hipertensión pulmonar y remodelación.

o c. Hipervascularización de la circulación pulmonar, ebre y regeneración alveolar

@ d. Hipervascularización de la circulación brónquica, hipertensión pulmonar y neovascularización

La respuesta correcta es: Hipervascularización de la circulación brónquica, hipertensión pulmonar y


neovascularización

Pregunta 9 Correcta Puntúa 0,13 sobre 0,13

Cuál NO es un uso del ultrasonido en el diagnóstico de un nódulo tiroideo:

Seleccione una:

o a. Valora el tamaño del nódulo

@ b. Estudia si el nódulo es hipofuncionante o hiperfuncionante.

o c. Ubica nódulos que se encuentran en la parte posterior o nódulos predominantemente quísticos

o d. Identi ca estructuras adyacentes en el cuello

La respuesta correcta es: Estudia si el nódulo es hipofuncionante o hiperfuncionante.


/
Pregunta 10 Incorrecta Puntúa 0,00 sobre 0,13

De acuerdo a la ATA, un nódulo con un patrón de sospecha muy baja. ¿Cuál es el riesgo de malignidad y
a qué clase según la ACE pertenece?

Seleccione una:

o a. Malignidad < 3%. Lesión clase 1.

o b. Malignidad 10 - 20%. Lesión clase 2

@ c. Malignidad < 3%. Lesión clase 2 X


o d. Malgnidad 10 – 20%. Lesión clase 1

La respuesta correcta es: Malignidad < 3%. Lesión clase 1.

Pregunta 11 Correcta Puntúa 0,13 sobre 0,13

De los siguiente factores, cuál es el que debe estar alterado para que un paciente infectado de
tuberculosis se convierta en persona enferma:

Seleccione una:

o a. Medio ambiente: Que haya elevada prevalencia de tuberculosis

o b. Huésped: que genéticamente esté predispuesto a infectarse

@ c. Huésped: Inmunidad celular de ciente, en especial CD4

o d. Medio Ambiente: Que viva en un lugar frío

La respuesta correcta es: Huésped: Inmunidad celular de ciente, en especial CD4


/
Pregunta 12 Correcta Puntúa 0,13 sobre 0,13

De los siguientes enunciados en relación al vértigo señale el correcto:

Seleccione una:

@ a. Una de las causas neurológicas no estructurales de vértigo episódico en la migraña vestibular

o b. En la enfermedad de Meniere el aqtaque de vértigo se acompaña de vómito pero nunca de pérdida


auditiva

o c. El nistagmus producido por una lesión periférica es multidireccional y cambia de dirección

o d. Una de las causas de vértigo persistente y crónico es el vértigo postural paroxístico benigno

La respuesta correcta es: Una de las causas neurológicas no estructurales de vértigo episódico en la migraña
vestibular

Pregunta 13 Incorrecta Puntúa 0,00 sobre 0,13

Durante un aviso nocturno usted acude al domicilio de una paciente de 47 años de edad, diagnosticada
de asma y que realiza habitualmente tratamiento con un corticoide inhalado y un beta2 de larga
duración. Su familia le explica que ha presentado una crisis de asma que ha tratado con varias dosis de
salbutamol administradas a través de una cámara de inhalación sin observar mejoría. Al inspeccionar a
la paciente usted observa que realiza un importante trabajo respiratorio con utilización de la
musculatura respiratoria accesoria que usted interpreta como una agudización grave de asma. ¿Cuál de
las siguientes asociaciones de hallazgos es más probable que usted pueda observar?

Seleccione una:

@ a. Sibilancias - inspiración alargada - percusión timpánica. X


o b. Silencio auscultatorio - espiración alargada - percusión mate.

o c. Roncus - espiración alargada - percusión mate.

o d. Silencio auscultatorio - espiración alargada - percusión timpánica.

La respuesta correcta es: Silencio auscultatorio - espiración alargada - percusión timpánica.


/
Pregunta 14 Correcta Puntúa 0,13 sobre 0,13

El cáncer de tiroides papilar, es el tipo de cáncer de tiroides más frecuente. En relación a lo dicho, señale
la respuesta más adecuada.

Seleccione una:

o a. Todos los subtipos del cáncer de tiroides papilar son malignos

@ b. Representa aproximadamente el 80% de todos los cánceres de tiroides.

o c. Más común en regiones con un bajo consumo de yodo en la dieta

o d. Se asocia con un mal pronóstico, mayor al cáncer folicular de tiroides

La respuesta correcta es: Representa aproximadamente el 80% de todos los cánceres de tiroides.

Pregunta 15 Correcta Puntúa 0,13 sobre 0,13

El diagnóstico de Guillain-Barré se puede considerar en uno de los siguientes hallazgos:

Seleccione una:

o a. nivel sensitivo algésico en el torax

@ b. disociación albúmino citológica

o c. re ejos normales en las extremidades débiles

o d. estudio de conducción nerviosa alterado en una sola extremidad

La respuesta correcta es: disociación albúmino citológica


/
Pregunta 16 Correcta Puntúa 0,13 sobre 0,13

El fármaco de primera elección para el tratamiento de las crisis de ausencia es:

Seleccione una:

o a. lamotrigina

@ b. etosuximida

o c. carbamazepina

o d. fenitoína sódica

La respuesta correcta es: etosuximida

Pregunta 17 Correcta Puntúa 0,13 sobre 0,13

El tratamiento de las Bronquiectasias se basa en 3 pilares, EXCEPTO:

Seleccione una:

o a. Mejorar la eliminación de las secreciones, que se consigue con una adecuada hidratación, con
sioterapia respiratoria y drenaje postural mantenidos.

o b. Controlar las infecciones con el uso de antibióticos en las agudizaciones durante 10-15 días.

@ c. Revertir el remodelamiento bronquial

o d. Eliminar la obstrucción bronquial.

La respuesta correcta es: Revertir el remodelamiento bronquial


/
Pregunta 18 Correcta Puntúa 0,13 sobre 0,13

¿En cuanto al manejo de la brosis pulmonar cual es un fármaco anti brótico?

Seleccione una:

o a. Azatioprina

@ b. Pirfenidona

o c. Micofenolato

o d. Ciclofosfamida

La respuesta correcta es: Pirfenidona

Pregunta 19 Correcta Puntúa 0,13 sobre 0,13

En la malaria, la forma de plasmodio transmitida del mosquito al hombre es el:

Seleccione una:

@ a. Esporozoito.

o b. Gametocito.

o c. Hipnozoito.

o d. Merozoito.

La respuesta correcta es: Esporozoito.


/
Pregunta 20 Correcta Puntúa 0,13 sobre 0,13

¿En qué período de tiempo se hace evidente la DM2 al tener ya resistencia a la insulina?

Seleccione una:

@ a. 5-10 años.

o b. 12-24 meses

o c. 2 años

o d. 5 años

La respuesta correcta es: 5-10 años.

Pregunta 21 Incorrecta Puntúa 0,00 sobre 0,13

En relación a iniciar el tratamiento frente a una crisis epiléptica única, señale la a rmación correcta:

Seleccione una:

o a. la neuroimagen no es importante para decidir si se inicia o no el tratamiento

@ b. deben ser tratatadas independientemente del resultado del EEG X


o c. la crisis epiléptica que ocurre durante el sueño aumenta el riesgo de recurrencia

o d. deben ser tratadas aún si el examen neurológicoes normal

La respuesta correcta es: la crisis epiléptica que ocurre durante el sueño aumenta el riesgo de recurrencia


/
Pregunta 22 Correcta Puntúa 0,13 sobre 0,13

La arteria cerebral responsable de un embolismo arteria-arteria, en un paciente con antecedente de


episodios de amaurosis fugaz es:

Seleccione una:

o a. cerebral media

@ b. carótida interna

o c. basilar

o d. cerebral posterior

La respuesta correcta es: carótida interna

Pregunta 23 Correcta Puntúa 0,13 sobre 0,13

La criptosporidiosis, que está causada por Cryptosporidium parvum, cursa como una enteritis benigna
en personas previamente sanas, pero en personas con inmunosupresión causa una enfermedad grave.
En estas formas graves su diagnóstico convencional se efectúa por:

Seleccione una:

o a. Serología.

o b. Coprocultivo.

@ c. Examen microscópico de las heces (T. de Ziehl-Neelsen modi cada).

o d. Hemocultivo.

La respuesta correcta es: Examen microscópico de las heces (T. de Ziehl-Neelsen modi cada).


/
Pregunta 24 Correcta Puntúa 0,13 sobre 0,13

La oxigenoterapia es importante en el manejo de la brosis pulmonar porque: (escoja la opción


incorrecta):

Seleccione una:

o a. Reduce la presión arterial del lado derecho del corazón

o b. Evita o disminuye las complicaciones de los niveles bajos de oxígeno en la sangre

@ c. Detiene el daño pulmonar

o d. Facilita la respiración y por consiguiente la actividad física

La respuesta correcta es: Detiene el daño pulmonar

Pregunta 25 Correcta Puntúa 0,13 sobre 0,13

La primera medida terapéutica a realizar en un paciente con una puntuación de 4 en la escala de


Glasgow alteración, con anisocoria pupilar y mirada desconjugada, una vez intubado es :

Seleccione una:

o a. corticosteroides y anticonvulsivantes

o b. naloxona ante la sospecha de sobredosis de opiodes

o c. administración de tiamina y glucosa al 50%

@ d. manitol intravenoso

La respuesta correcta es: manitol intravenoso


/
Pregunta 26 Correcta Puntúa 0,13 sobre 0,13

Las vacunas frente a la gripe que se utilizan actualmente se caracterizan por:

Seleccione una:

o a. Impedir, mediante la inmunidad que generan, que aparezca una pandemia.

@ b. Variar su composición antigénica cada año, como consecuencia de la deriva antigénica de los virus
gripales A y B.

o c. Disponer de tres subtipos del virus de la gripe A (H3N2, HIN1 y H5N1)

o d. Precisar sólo la administración de una dosis a lo largo de la vida.

La respuesta correcta es: Variar su composición antigénica cada año, como consecuencia de la deriva
antigénica de los virus gripales A y B.

Pregunta 27 Correcta Puntúa 0,13 sobre 0,13

Mujer de 47 años de edad, nacida y residente en Orellana A mediados del año 2017 comienza con
palpitaciones, disnea y edemas progresivos en extremidades inferiores. Es diagnosticada de
insu ciencia cardíaca secundaria a miocardiopatía por Chagas. ¿Cuál es el agente etiológico de esta
enfermedad endémica?

Seleccione una:

o a. Trypanosoma brucei.

@ b. Trypanosoma cruzi.

o c. Giardia lamblia.

o d. Leishmania donovani.

La respuesta correcta es: Trypanosoma cruzi.


/
Pregunta 28 Correcta Puntúa 0,13 sobre 0,13

Paciente de 50 años que presenta un derrame pleural con las siguientes características: aspecto pajizo,
Ph 7.3, cociente de proteínas pleura/suero 0.8, cociente de LDH pleura/suero 0.9, Gram y Ziehl
negativos, lípidos totales, colesterol y triglicéridos normales, células mesoteliales <5%, intensa
linfocitosis sin atipias, ADA 64 U/l. ¿Qué diagnóstico le sugiere?

Seleccione una:

@ a. Derrame pleural tuberculoso.

o b. Empiema pleural.

o c. Derrame pleural por insu ciencia cardiaca (trasudado).

o d. Mesotelioma pleural.

La respuesta correcta es: Derrame pleural tuberculoso.

Pregunta 29 Correcta Puntúa 0,13 sobre 0,13

Paciente que acude a Urgencias con ebre intermitente, escalofríos, cefalea, debilidad y abundante
sudoración nocturna de 3 días de evolución. Como antecedente epidemiológico destaca un viaje a
Cayambe hace 3 semanas donde consumieron leche cruda y queso sin pasteurizar. El modo más rápido
de diagnosticar la infección sospechada es:

Seleccione una:

o a. Gram directo del líquido cefalorraquídeo.

o b. Test para detección de anticuerpos heteró los (Paul-Bunnell).

@ c. Prueba del Rosa de Bengala y toma de hemocultivo.

o d. Tinciones de micobacterias en esputo u orina.

La respuesta correcta es: Prueba del Rosa de Bengala y toma de hemocultivo.


/
Pregunta 30 Correcta Puntúa 0,13 sobre 0,13

¿Qué es el dengue?

Seleccione una:

o a. Una enfermedad vírica que ocasiona un eritema que evoluciona a mácula y pápula afectando
fundamentalmente a la población infantil.

@ b. Una enfermedad vírica que puede producir una ebre hemorrágica.

o c. Una enfermedad causada por un poxvirus.

o d. Una zoonosis que afecta al hombre ocasionalmente.

La respuesta correcta es: Una enfermedad vírica que puede producir una ebre hemorrágica.

Pregunta 31 Correcta Puntúa 0,13 sobre 0,13

¿Qué evalúan los criterios de Wells para tromboembolia pulmonar?

Seleccione una:

o a. Diagnóstico de nitivo de tromboembolia pulmonar

o b. Criterios radiológicos para diagnosticar tromboembolia pulmonar

@ c. La probabilidad clínica de tener tromboembolia pulmonar

o d. Pronóstico de supervivencia para pacientes con tromboembolia pulmonar

La respuesta correcta es: La probabilidad clínica de tener tromboembolia pulmonar


/
Pregunta 32 Correcta Puntúa 0,13 sobre 0,13

Respecto al tratamiento NO farmacológico que se emplea en Diabetes tipo II elija la correcta

Seleccione una:

o a. Grasa Mono insaturados < 10 %

o b. No se recomienda el uso de edulcorantes

o c. Proteínas: >50 %

(!) d. Actividad física 150 minutos semanales.

La respuesta correcta es: Actividad física 150 minutos semanales.

Pregunta 33 Incorrecta Puntúa 0,00 sobre 0,13

Seleccione la respuesta correcta con respecto al fondo de ojo en un paciente con diabetes.

Seleccione una:

(!) a. Si este se encuentra normal se debe repetir el examen en 6-12 meses X


o b. Se realiza en el momento del diagnóstico de DM II.

o c. El último signo en presentarse son las microhemorragias

o d. Este debe realizarse a los 5 años del diagnóstico de DM

La respuesta correcta es: Se realiza en el momento del diagnóstico de DM II.


/
Pregunta 34 Correcta Puntúa 0,13 sobre 0,13

Staphylococcus aureus puede producir una de las siguientes enfermedades:

Seleccione una:

o a. Brucelosis

o b. Escarlatina

o c. Botulismo

@ d. Osteomielitis

La respuesta correcta es: Osteomielitis

Pregunta 35 Correcta Puntúa 0,13 sobre 0,13

Tras la infección, ¿qué plasmodios parásitos humanos persisten como hipnozoitos en el hígado y
pueden causar recidivas?

Seleccione una:

@ a. P. vivax y P. ovale.

o b. P. malariae.

o c. P. falciparum.

o d. P. cynmology y P. knowlesi.

La respuesta correcta es: P. vivax y P. ovale.


/
Pregunta 36 Correcta Puntúa 0,13 sobre 0,13

Una causa común de muerte cerebral:

Seleccione una:

@ a. hemorragia subaracnoidea

o b. encefalitis herpética

o c. hematoma epidural

o d. infarto de la arteria cerebral anterior

La respuesta correcta es: hemorragia subaracnoidea

Pregunta 37 Incorrecta Puntúa 0,00 sobre 0,13

Una de las siguientes consideraciones puede excluir un diagnóstico de Guillain-Barré:

Seleccione una:

@ a. estudio de conducción nerviosa normal en las extremidades débiles X


o b. arre exia en las extremidades débiles

o c. nivel sensitivo algésico espinal

o d. disociación albúmino citológica

La respuesta correcta es: nivel sensitivo algésico espinal


/
Pregunta 38 Correcta Puntúa 0,13 sobre 0,13

Uno de los criterios para el diagnóstico clínico de muerte cerebral es:

Seleccione una:

o a. postura de decerebración bilateral

@ b. ausencia de re ejos del tronco cerebral

o c. hipotermia de 31 grados centígrados

o d. puntuación en la escala de Glasgow 4

La respuesta correcta es: ausencia de re ejos del tronco cerebral

Pregunta 39 Correcta Puntúa 0,13 sobre 0,13

Un signo clínico importante que oriente a considerar un coma como de origen metabólico más que
estructural en un paciente en coma es:

Seleccione una:

@ a. pupilas mióticas, con re ejo fotomotor presente

o b. puntuación de 7 en la escala de Glasgow

o c. postura de decorticación

o d. respiracion ataxica

La respuesta correcta es: pupilas mióticas, con re ejo fotomotor presente


/
Pregunta 40 Correcta Puntúa 0,13 sobre 0,13

Un trabajador en el oleoducto en el Coca de 36 años de edad, sufre desde hace dos meses un cuadro
abdominal intermitente de náuseas, heces pastosas, atulencia, meteorismo y que le han llevado a
perder tres kg de peso. ¿Cuál de los siguientes microorganismos sería con mayor probabilidad el
responsable del cuadro?

Seleccione una:

@ a. Giardia lamblia.

o b. Vibrio cholerae.

o c. Entamoeba coli.

o d. Trichomonas hominis.

La respuesta correcta es: Giardia lamblia.

◄ Link sesión zoom examen nal 2 Ir a... V


/
JOSELYN SOFIA BARAJA CASTILLO

Área personal  Mis cursos  Quito  CIENCIAS DE LA SALUD  MEDICINA - PRESENCIAL  ABR 2020 - AGO 2020
 INVESTIGACIÓN I - Prl: MD NVA Pen: 961  18 de julio: 12:00H: Examen Segundo Parcial Investigación I  Examen
Segundo Parcial: Investigación I

Comenzado el sábado, 18 de julio de 2020, 12:10


Estado Finalizado
Finalizado en sábado, 18 de julio de 2020, 12:59
Tiempo empleado 49 minutos 52 segundos
Cali cación 3,04 de 4,00 (76%)

[Pregunta 1 Correcta Puntúa 0,16 sobre 0,16

¿Con cuál de los siguientes fármacos NO interacciona la rifampicina?:

Seleccione una:

o a. Fenitoína

@ b. Hidróxido de aluminio

o c. Metadona

o d. Cloranfenicol

La respuesta correcta es: Hidróxido de aluminio

/
Pregunta 2 Incorrecta Puntúa 0,00 sobre 0,16

Cuál de las siguientes a rmaciones corresponden a Neumonía Adquirida en la Comunidad.

Seleccione una:

o a. Pacientes que viven en hogares de cuidados crónicos.

o b. Pacientes que presentan síntomas durante las 48 primeras horas de la admisión al hospital

@ c. Pacientes no hospitalizados en los 14 días anteriores X


o d. Cuadro de presentación aguda.

La respuesta correcta es: Pacientes que viven en hogares de cuidados crónicos.

Pregunta 3 Correcta Puntúa 0,16 sobre 0,16

Cuál de las siguientes a rmaciones NO corresponde al diagnóstico diferencial de la EPOC.

Seleccione una:

o a. Obstrucción de vía aérea superior

o b. Trombo embolismo pulmonar

o c. Infarto agudo de miocardio

@ d. Paro respiratorio

La respuesta correcta es: Paro respiratorio

/
Pregunta 4 Correcta Puntúa 0,16 sobre 0,16

¿Cuál de las siguientes alternativas indica el carcinoma que prevalece en el tercio superior del esófago?

Seleccione una:

o a. Carcinoma atípico

@ b. Carcinoma escamoso

o c. Carcinoma in situ

o d. Adenocarcinoma

La respuesta correcta es: Carcinoma escamoso

Pregunta 5 Correcta Puntúa 0,16 sobre 0,16

¿Cuál de los siguientes hallazgos histológicos es necesario para establecer el diagnóstico de cirrosis
hepática?

Seleccione una:

o a. Destrucción de los conductos biliares.

o b. Cuerpos de Mallory.

o c. Degeneración baloniforme de los hepatocitos.

@ d. Presencia de brosis.

La respuesta correcta es: Presencia de brosis.

/
Pregunta 6 Correcta Puntúa 0,16 sobre 0,16

¿Cuáles son los factores de riesgo débiles para TEP?

Seleccione una:

o a. Hemo lia, lesión medular

o b. Quimioterapia, embarazo

o c. Fractura de cadera, cirugía mayor

@ d. Ancianos, obesidad, venas varicosas

La respuesta correcta es: Ancianos, obesidad, venas varicosas

Pregunta 7 Incorrecta Puntúa 0,00 sobre 0,16

De la etiología de hemorragia cerebral, señale el enunciado verdadero:

Seleccione una:

@ a. La coagulopatía produce hemorragias se localización lobar X


o b. La angiopatía amiloide se debe sospechar en pacientes menores de 60 años

o c. Las hemorragias por tumores cerebrales se producen tardíamente en la evolución luego de tener
otros síntomas neurológicos

o d. Las malformaciones arteriovenosas pueden causar sangrado intraventricular, subaracnoideo e


intraparenquimatoso

La respuesta correcta es: Las malformaciones arteriovenosas pueden causar sangrado intraventricular,
subaracnoideo e intraparenquimatoso

/
Pregunta 8 Correcta Puntúa 0,16 sobre 0,16

De los siguientes enunciados en relación a ictus isquémico, señale el verdadero:

Seleccione una:

a. En estenosis carotidea sintomática en grado severo (&gt; o igual al 70%) el tratamiento de elección
es quirúrgico (endarterectomía)

o b. En aterosclerosis intracraneal sintomática se recomienda el tratamiento con stenting intracraneal


que es superior al tratamiento con aspirina

o c. La siopatología de las causas cardioembólicas en ictus isquémico se basa en la trombosis


secundaria a lipohialinosis con afectación de vasos pequeños

o d. El cigarrillo es un factor de riesgo para aterosclerosis por lo que el tratamiento de deshabituación


se recomienda en prevención secundaria únicamente

La respuesta correcta es: En estenosis carotidea sintomática en grado severo (&gt; o igual al 70%) el
tratamiento de elección es quirúrgico (endarterectomía)

Pregunta 9 Correcta Puntúa 0,16 sobre 0,16

El esófago de Barret se caracteriza por, señale la alternativa correcta:

Seleccione una:

o a. Se asocia con el anillo de Schatzki.

@ b. Metaplasia intestinal

o c. Se lo considera carcinoma in situ de esófago distal

o d. Se asocia con divertículo de Zenker

La respuesta correcta es: Metaplasia intestinal

/
Pregunta 10 Correcta Puntúa 0,16 sobre 0,16

El siguiente resultado de LCR: células blancas 200, de predominio polimorfonuclear, proteínas 600
mg/dl, hipoglucorraquia de 26 mg/dl, correspondería a qué enfermedad:

Seleccione una:

o a. Meningitis viral

@ b. Meningitis bacteriana

o c. Encefalitis viral

o d. Meningitis tuberculosa

La respuesta correcta es: Meningitis bacteriana

Pregunta 11 Correcta Puntúa 0,16 sobre 0,16

El tratamiento de elección en colitis pseudomembranosa que cursa con íleo paralítico es, señale la
alternativa correcta:

Seleccione una:

o a. Vancomicina por vía oral.

o b. Vancomicina por vía intravenosa.

@ c. Metronidazol por vía intravenosa.

o d. Metronidazol por vía oral.

La respuesta correcta es: Metronidazol por vía intravenosa.

/
Pregunta 12 Correcta Puntúa 0,16 sobre 0,16

En relación a la hemorragia cerebral, señale el enunciado verdadero:

Seleccione una:

a. Una hemorragia de gran tamaño puede deteriorar el estado de conciencia se un paciente e


inclusive llevarlo al coma

o b. En la siopatología de hemorragia hipertensiva se destaca la afectación crónica de la pared del


vaso de gran calibre

o c. La hemorragia produce dé cit neurológico focal como la cefalea y el vómito lo que la hace
indistinguible del ictus isquémico

o d. Las hemorragias talámicas producen hemiparesia ipsilateral y afasia si la afectación es del lado no
dominante

La respuesta correcta es: Una hemorragia de gran tamaño puede deteriorar el estado de conciencia se un
paciente e inclusive llevarlo al coma

Pregunta 13 Correcta Puntúa 0,16 sobre 0,16

En relación a la hemorragia subaracnoidea, señale el enunciado verdadero:

Seleccione una:

@ a. La etiología aneurismática es la causa más frecuente de hemorragia no traumática

o b. De los principales factores de riesgo de mencionan la diabetes, dislipidemia y obesidad

o c. El tratamiento es dirigido a resolver la causa del sangrado por lo que la cirugía se la debe realizar en
las primeras 48h

o d. La cefalea es un síntoma muy infrecuente en la presentación clínica

La respuesta correcta es: La etiología aneurismática es la causa más frecuente de hemorragia no traumática

/
Pregunta 14 Correcta Puntúa 0,16 sobre 0,16

En relación a las infecciones del sistema nervioso central, señale el enunciado verdadero:

Seleccione una:

o a. El tratamiento de todos los casos de absceso cerebral es quirúrgico y en casos puntuales se


adiciona tratamiento médico con antibióticos de amplio espectro

o b. Los abscesos cerebrales se diseminan por sangre en un porcentaje bajo de pacientes, contrario a
los criptogénicos (fuente primaria desconocida) que son los más frecuentes

c. En las etiologías de meningitis crónica junto a la tuberculosis se debe considerar a la sí lis y a la


enfermedad de Lyme entre otras

o d. Para el tratamiento de la meningitis tuberculosa se emplea fármacos antituberculosos isoniazida,


etambutol, pirazinamida, rifampicina por 1 mes

La respuesta correcta es: En las etiologías de meningitis crónica junto a la tuberculosis se debe considerar a la
sí lis y a la enfermedad de Lyme entre otras

Pregunta 15 Correcta Puntúa 0,16 sobre 0,16

Muchacha de 26 años hospitalizada por ictericia de instauración reciente, asociada a dolor en


hipocondrio derecho. Se detecta hepatomegalia sensible sin esplenomegalia. Hay telangiectasias
faciales. Reconoce antecedentes de promiscuidad sexual, pero no consumo de droga intravenosa. AST
315 ;BT 16 mg/dl; alt 110 ; GGT 680; FA 280 ; triglicéridos 600, colesterol 280. Ecografía con patrón
ecogénico del hígado. Cuál de las siguientes alternativas indica el diagnóstico más probable:

Seleccione una:

o a. Hepatitis tóxica

@ b. Hepatitis vírica

o c. Hepatopatía alcoholica

o d. Hepatitis Autoinmune

La respuesta correcta es: Hepatitis vírica

/
Pregunta 16 Correcta Puntúa 0,16 sobre 0,16

Paciente de 48 años sin APP, reside en una comunidad rural de la provincia de Cotopaxi, agricultor,
acude por 1 crisis convulsiva focal motora izquierda, en el trabajo diagnóstico que requiere para
averiguar la etiología de la crisis, usted señalaría que enunciado como verdadero:

Seleccione una:

o a. La zona de residencia endémica para cisticercosis es considerado un criterio de exposición pero no


forma parte de los criterios diagnósticos

o b. El hallazgo en TAC cerebral de calci caciones parenquimatosas denotan un criterio absoluto de


neurocisticercosis

c. Se puede solicitar determinación de anticuerpos anticisticercosis o detección de antígenos que son


parte de los criterios diagnósticos de cisticercosis

o d. La demostración histológica del parásito a través de una biopsia es considerado un criterio menor
de diagnóstico de neurocisticercosis

La respuesta correcta es: Se puede solicitar determinación de anticuerpos anticisticercosis o detección de


antígenos que son parte de los criterios diagnósticos de cisticercosis

Pregunta 17 Incorrecta Puntúa 0,00 sobre 0,16

Paciente de 50 años que presenta un derrame pleural con las siguientes características: aspecto pajizo,
Ph 7.3, cociente de proteínas pleura/suero 0.8, cociente de LDH pleura/suero 0.9, Gram y Ziehl
negativos, lípidos totales, colesterol y triglicéridos normales, células mesoteliales <5%, intensa
linfocitosis sin atipias, ADA 64 U/l. ¿Qué diagnóstico le sugiere?

Seleccione una:

o a. Empiema pleural.

o b. Derrame pleural tuberculoso.

@ c. Derrame pleural por insu ciencia cardiaca (trasudado). X


o d. Mesotelioma pleural.

La respuesta correcta es: Derrame pleural tuberculoso.

/
Pregunta 18 Correcta Puntúa 0,16 sobre 0,16

¿Qué evalúan los criterios de Wells para tromboembolia pulmonar?

Seleccione una:

o a. Diagnóstico de nitivo de tromboembolia pulmonar

o b. Criterios radiológicos para diagnosticar tromboembolia pulmonar

@ c. La probabilidad clínica de tener tromboembolia pulmonar

o d. Pronóstico de supervivencia para pacientes con tromboembolia pulmonar

La respuesta correcta es: La probabilidad clínica de tener tromboembolia pulmonar

Pregunta 19 Correcta Puntúa 0,16 sobre 0,16

Respecto a la cirrosis hepática, escoja la respuesta correcta.

Seleccione una:

@ a. Las complicaciones son consecuencia de la hipertensión portal y la insu ciencia hepática.

o b. La cirrosis es la etapa nal de cualquier enfermedad aguda del hígado.

o c. Es un proceso localizado caracterizado por brosis y la conversión de la arquitectura normal en


nódulos de estructura alterada.

o d. Es un proceso reversible.

La respuesta correcta es: Las complicaciones son consecuencia de la hipertensión portal y la insu ciencia
hepática.

/
Pregunta 20 Correcta Puntúa 0,16 sobre 0,16

Respecto a la magnitud del daño causado por una tromboembolia pulmonar, seleccione la respuesta
correcta:

Seleccione una:

o a. Ningún factor es importante ya que el diagnóstico siempre es post mortem

o b. Lo principal es el tamaño, y son de importancia únicamente émbolos mayores a 2mm

o c. Es únicamente importante la arteria que está siendo afectada (obstruida)

(j) d. Es determinante el tamaño del émbolo y el diámetro de la arteria afectada (obstruida)

La respuesta correcta es: Es determinante el tamaño del émbolo y el diámetro de la arteria afectada
(obstruida)

Pregunta 21 Incorrecta Puntúa 0,00 sobre 0,16

Sobra la Enfermedad In amatoria Intestinal, escoja la respuesta INCORRECTA:

Seleccione una:

o a. La mayor fuente de producción de estas interleukinas proin amatorias son los macrófagos
activados de la lámina propia que estimulan las células inmunes.

o b. Los niveles tisulares de IL 1 están elevados en la Enfermedad de Crohn y los niveles séricos de IL 2
están igualmente elevados en la Colitis ulcerosa.

(j) c. En la EII se encuentra aumentada la producción de las citokinas proin amatorias, IL 1, IL 6, IL 8, y


además del FNT (factor de necrosis tumoral) alfa. X
o d. El factor de necrosis tumoral y el interferón alfa contribuyen directamente al daño epitelial del
intestino.

La respuesta correcta es: Los niveles tisulares de IL 1 están elevados en la Enfermedad de Crohn y los niveles
séricos de IL 2 están igualmente elevados en la Colitis ulcerosa.

/
Pregunta 22 Correcta Puntúa 0,16 sobre 0,16

Una mujer de 25 años re ere una historia de 12 meses de cólicos abdominales recurrentes en hemi-
abdomen inferior acompañado de distensión abdominal y diarreas. No presenta ebre, pérdida de peso
ni anorexia ni deposición con sangre. Este paciente cumple con los criterios de Roma IV para Sd de colon
irritable con predominio de diarrea, no presenta signos de alarma. ¿En qué otra patología Usted puede
pensar?

Seleccione una:

o a. Enfermedad in amatoria intestinal.

@ b. Enfermedad celiaca.

o c. Helicobacter pylori.

o d. Sobrecrecimiento bacteriano de intestino delgado.

La respuesta correcta es: Enfermedad celiaca.

Pregunta 23 Correcta Puntúa 0,16 sobre 0,16

Uno de los siguientes NO es síntoma general de la tuberculosis.

Seleccione una:

o a. Pérdida de peso

o b. Síndrome febril

o c. Sudoración nocturna

@ d. Hematemesis

La respuesta correcta es: Hematemesis

/
Pregunta 24 Incorrecta Puntúa 0,00 sobre 0,16

Un paciente con sospecha de encefalitis, debe tener, EXCEPTO:

Seleccione una:

o a. Deterioro cognitivo agudo

o b. Anormalidades conductuales

@ c. Signos de focalidad neurológica X


o d. Disminución del estado de vigilia

La respuesta correcta es: Deterioro cognitivo agudo

Pregunta 25 Incorrecta Puntúa 0,00 sobre 0,16

Un valor de > 4 en la escala de Wells indica que:

Seleccione una:

@ a. El paciente está estable y es poco probable que presente embolia pulmonar X


o b. Clínicamente el paciente tiene probabilidad de Embolismo Pulmonar

o c. El paciente está inestable pero la probabilidad de que presente embolia pulmonar es baja

o d. Clínicamente el paciente no tiene probabilidad de Embolismo Pulmonar

La respuesta correcta es: Clínicamente el paciente tiene probabilidad de Embolismo Pulmonar

◄ Link sesión zoom examen nal 1: Investigación II

Ir a... V Link de sesión zoom examen segundo parcial ►

/
JOSELYN SOFIA BARAJA CASTILLO

Área personal  Mis cursos  Quito  CIENCIAS DE LA SALUD  MEDICINA - PRESENCIAL  ABR 2020 - AGO 2020
 INVESTIGACIÓN I - Prl: MD NVA Pen: 961  14 de agosto 18:00H Dr Hugo Miranda  Examen tercer parcial

Comenzado el viernes, 14 de agosto de 2020, 18:10


Estado Finalizado
Finalizado en viernes, 14 de agosto de 2020, 18:59
Tiempo empleado 49 minutos 14 segundos
Cali cación 2,88 de 4,00 (72%)

Pregunta 1 Incorrecta Puntúa 0,00 sobre 0,16

A qué grupo poblacional no restringiría la vacuna de la gripe:

Seleccione una:

o a. Personas con antecedente de Guillain Barré

@ b. Personas con trastornos crónicos metabólicos

o c. Niños con antecedentes de síndrome de Reye

o d. Personas alérgicas a las proteínas del huevo

La respuesta correcta es: Personas alérgicas a las proteínas del huevo


/
Pregunta 2 Correcta Puntúa 0,16 sobre 0,16

A un joven asintomático de 14 años cuyo padre acaba de ser diagnosticado de Tuberculosis Pulmonar se
le realiza un Mantoux, con resultado negativo. ¿Qué actitud es la adecuada?

Seleccione una:

o a. Realizarle una radiografía de tórax.

o b. Repetir la prueba cutánea al mes.

@ c. Iniciar quimiopro laxis y repetir la prueba cutánea a los 3 meses.

o d. Tranquilizarle, ya que no tiene ningún riesgo.

La respuesta correcta es: Iniciar quimiopro laxis y repetir la prueba cutánea a los 3 meses.

Pregunta 3 Correcta Puntúa 0,16 sobre 0,16

Con respecto a DM2 señale lo correcto:

Seleccione una:

o a. Representa 2– 4% de todas las diabetes

@ b. Se asocia con una fuerte predisposición genética o antecedentes familiares en familiares de primer
grado.

o c. Existe una destrucción de las células beta del páncreas con un dé cit absoluto de insulina

o d. Un de los criterios para su diagnóstico es PTOG a las dos horas con un valor de 140-199 mg/dl

La respuesta correcta es: Se asocia con una fuerte predisposición genética o antecedentes familiares en
familiares de primer grado.


/
Pregunta 4 Incorrecta Puntúa 0,00 sobre 0,16

Con respecto a la infección por gripe indiqué la premisa verdadera:

Seleccione una:

o a. El virus solamente tiene mutaciones puntuales

o b. la Clínica generalmente se asocia a trastornos respiratorios altos

o c. puede causar una neumonía viral primaria

@ d. casi siempre se asocia con febrícula, o sin ebre sino con tos seca X

La respuesta correcta es: puede causar una neumonía viral primaria

Pregunta 5 Correcta Puntúa 0,16 sobre 0,16

¿Cuál de las siguientes es una causa secundaria de Diabetes?

Seleccione una:

o a. Hipoparatiroidismo, hipertiroidismo, pancreatitis

o b. Acromegalia, embarazo, enfermedad de Addison

@ c. Síndrome de ovario poliquístico, Síndrome de Cushing, acromegalia.

o d. Hipotiroidismo, embarazo, enfermedad de Addison

La respuesta correcta es: Síndrome de ovario poliquístico, Síndrome de Cushing, acromegalia.


/
Pregunta 6 Correcta Puntúa 0,16 sobre 0,16

¿Cuál de las siguientes es una complicación aguda?

Seleccione una:

o a. Retinopatía proliferativa

o b. Nefropatía (grado I)

o c. Cardiopatía isquémica

@ d. Cetoacidosis.

La respuesta correcta es: Cetoacidosis.

Pregunta 7 Incorrecta Puntúa 0,00 sobre 0,16

¿Cuál de los siguientes corresponde a un criterio de diagnóstico en DM2?

Seleccione una:

o a. Glucosa al azar (2 tomas ≥ 200mg/dl).

o b. Glucosas en ayunas 100-125mg/dl

o c. HbAc1 de 5,7 – 6,4%

@ d. Glucosa al azar ≥ 126 mg/dl junto con las tres P (poliuria, polidipsia, polifagia) X

La respuesta correcta es: Glucosa al azar (2 tomas ≥ 200mg/dl).


/
Pregunta 8 Correcta Puntúa 0,16 sobre 0,16

Desde que se quiere el virus del papiloma humano hasta que produzca verrugas o condilomas un
tiempo de:

Seleccione una:

o a. pocas semanas (<2)

@ b. de 3 a 4 meses

o c. muchos años

o d. menos de 1 mes.

La respuesta correcta es: de 3 a 4 meses

Pregunta 9 Correcta Puntúa 0,16 sobre 0,16

El interferón es un medicamento útil para las siguientes infecciones excepto:

Seleccione una:

o a. Sarcoma de Kaposi

o b. HPV

@ c. VIH

o d. Hepatitis B

La respuesta correcta es: VIH


/
Pregunta 10 Incorrecta Puntúa 0,00 sobre 0,16

En el análisis de líquido cefalorraquídeo usted encuentra linfocitos aumentados, glucosa disminuida,


usted podría pensar en que la etiología es:

Seleccione una:

@ a. bacteriana X
o b. parasitaria

o c. micobacterias

o d. viral

La respuesta correcta es: micobacterias

Pregunta 11 Correcta Puntúa 0,16 sobre 0,16

En la clasi cación de FOD cuál considera usted como urgente y debe ser tratada de inmediato:

Seleccione una:

o a. FOD asociado a HIV

o b. FOD Clásico

o c. FOD Nosocomial

(!) d. FOD neutropénico

La respuesta correcta es: FOD neutropénico


/
Pregunta 12 Correcta Puntúa 0,16 sobre 0,16

Entre los factores de riesgo de enfermedad de Graves tenemos escoja la opción correcta

Seleccione una:

o a. se ve en misma proporción como factor de riesgo en mujeres y en hombres, el tabaquismo y se


asocia con enfermedades autoinmunes

o b. presencia de otras enfermedades autoinmunes , tabaquismo más frecuente en hombres que el


mujeres , antecedente de enfermedad de Graves .

o c. no se relaciona con la herencia familiar , el tabaquismo es un factor de riesgo intermedio , se ve en


gemelos idénticos

d. historia familiar de enfermedad de Hashimoto, presencia de otras enfermedades autoinmunes ,


más común en mujeres que en hombres , tabaquismo.

La respuesta correcta es: historia familiar de enfermedad de Hashimoto, presencia de otras enfermedades
autoinmunes , más común en mujeres que en hombres , tabaquismo.

Pregunta 13 Incorrecta Puntúa 0,00 sobre 0,16

Escoja la respuesta CORRECTA sobre la tiroiditis crónica o de Hashimoto

Seleccione una:

@ a. Es más frecuente en hombres (1:10) vs mujeres (1:20). X


o b. Produce una glándula blanda, de gran crecimiento, dolorosa y con consistencia lobulada

o c. No tiene factores desencadenantes como estrés, dé cit de selenio, infección por Yersinia
Enterocolítica o hepatitis C

o d. Es una enfermedad autoinmune donde actúan los linfocitos T CD4 que libran IL-2 y FNT y los
linfocitos T citotóxicos que produce apoptosis de tirocitos

La respuesta correcta es: Es una enfermedad autoinmune donde actúan los linfocitos T CD4 que libran IL-2 y
FNT y los linfocitos T citotóxicos que produce apoptosis de tirocitos


/
Pregunta 14 Correcta Puntúa 0,16 sobre 0,16

Hablando de la siopatología de la oftalmopatía de Graves .Escoja la opción correcta

Seleccione una:

o a. la disminución de las células T juegan un papel importante en la generación de la oftalmopatía .

o b. el volumen de los músculos retro oculares aumentan debido a la disminución de los broblastos .

o c. las células T dismunuidas , los broblastos producen ácido hialuronico y glucosamino glucanos y
esto aumenta el tejido retro ocular .

d. la presencia aumentada d e células T juegan un papel importantes en la oftalmopatía , y los


broblastos retro oculares secretan glucosaminoglucanos

La respuesta correcta es: la presencia aumentada d e células T juegan un papel importantes en la


oftalmopatía , y los broblastos retro oculares secretan glucosaminoglucanos

Pregunta 15 Correcta Puntúa 0,16 sobre 0,16

La BCG es una vacuna atenuada derivada del:

Seleccione una:

@ a. M. bovis

o b. M. tuberculosis

o c. M. leprae

o d. M. avium - intracellulare

La respuesta correcta es: M. bovis


/
Pregunta 16 Correcta Puntúa 0,16 sobre 0,16

La infección de herpes en recién nacidos es principalmente dado por:

Seleccione una:

o a. HVZ

o b. VHH 8

o c. VHS 1

@ d. VHS 2

La respuesta correcta es: VHS 2

Pregunta 17 Correcta Puntúa 0,16 sobre 0,16

La tuberculosis osteoarticular ataca principalmente a columna a lo que se denomina:

Seleccione una:

o a. Trastorno de Jacob

o b. Osteo tosis

@ c. Mal de Pott

o d. Enfermedad de Wegener

La respuesta correcta es: Mal de Pott


/
Pregunta 18 Correcta Puntúa 0,16 sobre 0,16

Paciente de 4 0 años de sexo femenino que se ha pedido exámenes por control médico anual y en el
Examen Físico se encontró BOCIO regresa con resultados: Eco tiroideo: reporta imagen nodular solida
de 1 cm hipo eco génica con vascularidad central y periférica con margen irregular . TSH 4 . 2 u/ IU/ ml T
4 1.6 ng / dl. Escoja la opción correcta.

Seleccione una:

o a. el reporte ecográ co no hace sospechar de malignidad se debe hacer seguimiento únicamente en 6


meses

o b. le derivo al endocrinólogo y al cirujano de cuello para que le le opren inmediatamente , y no


necesita de otros exámenes .

c. es una paciente con sospecha de malignidad mas de un 70 % se debe solicitar punción con aguja
na y se solicita marcadores tumorales.

o d. es una paciente con poca sospecha de malignidad como de 30 % y se debe pedir un nuevo control
en 3 meses

La respuesta correcta es: es una paciente con sospecha de malignidad mas de un 70 % se debe solicitar
punción con aguja na y se solicita marcadores tumorales.

Pregunta 19 Correcta Puntúa 0,16 sobre 0,16

Paciente de 45 años acude a consulta por presentar 6 meses de evolución de caída de cabello y no
re ere otros síntomas, Al examen físico se encuentra escaso cabello y hay piel seca lo demás de examen
físico esta normal , se solicita exámenes los mismos que reportan : TSH5.17 u/IU/ml , T3 3 ,3 n mol/ L- T
4 1.0 ng / dl. Escoja la opción correcta.

Seleccione una:

o a. paciente tiene diagnóstico de hipotiroidismo central

o b. el paciente tiene diagnóstico de hipertiroidismo

o c. el paciente tiene diagnóstico de eutiroidismo

@ d. el paciente tiene diagnóstico de hipotiroidismo subclínico

La respuesta correcta es: el paciente tiene diagnóstico de hipotiroidismo subclínico


/
Pregunta 20 Correcta Puntúa 0,16 sobre 0,16

Paciente de 65 años con antecedente de cáncer de estómago, tiene una dieta fraccionada con 5 comidas
ha bajado de peso mucho como 10 kilogramos y re ere dolor de epigastrio en forma continua ,además
re ere que se ha sentido con polidipsia y poliuria por lo que se solicita exámenes los mismos que
reportan: Glicemia 103 mg / dl Hemoglobina Glicosilada 6.4 mg y curva de tolerancia a la glicemia
reporta a la 99 mg , 30 minutos, 108 mg, 1 horas 199 mg , 2 horas 155 mg. Escoja la opción correcta

Seleccione una:

a. la paciente tiene diagnóstico de prediabetes tiene que remitirse a nutrición y debe regresar a un
control en 3 meses con glicemia y nueva hemoglobina glicosilada.

o b. la paciente tiene valores normales de laboratorio y no necesita recomendaciones

o c. la paciente tiene gastrectomía por cáncer gástrico y al momento cursa además con diabetes y se le
recomienda valoración nutricional , ejercicio y control en 1 mes con nueva glicemia

o d. tiene cáncer gástrico y tiene diabetes y debe recibir ya tratamiento con metformina de 1000 mg
cada 12 horas

La respuesta correcta es: la paciente tiene diagnóstico de prediabetes tiene que remitirse a nutrición y debe
regresar a un control en 3 meses con glicemia y nueva hemoglobina glicosilada.

Pregunta 21 Incorrecta Puntúa 0,00 sobre 0,16

Que caracteriza a una primoinfección por herpes tipo 1:

Seleccione una:

o a. distribución en dermatoma en especial tórax

o b. faringitis o gingivoestomatitis

o c. trastornos de córnea y conjuntiva

@ d. pápulas y pústulas intersticio entre piel y mucosa X

La respuesta correcta es: faringitis o gingivoestomatitis


/
Pregunta 22 Incorrecta Puntúa 0,00 sobre 0,16

Qué patología de las siguientes es una condición Que De ne sida:

Seleccione una:

@ a. leucoplasia vellosa X
o b. displasia cervical

o c. candidiasis oral

o d. herpes tipo 1: úlceras crónicas > 1 mes

La respuesta correcta es: herpes tipo 1: úlceras crónicas > 1 mes

Pregunta 23 Correcta Puntúa 0,16 sobre 0,16

Señale la respuesta CORRECTA sobre las complicaciones del hipotiroidismo

Seleccione una:

o a. En el hipotiroidismo leve puede haber apnea del sueño y síndrome del túnel carpiano

o b. En el hipotiroidismo severo se puede generar hipernatremia dilucional.

@ c. Puede producir hipertensión, cardiomegalia y disfunción diastólica

o d. Algunas personas desarrollan incremento de memoria, estado de ánimo eufórica

La respuesta correcta es: Puede producir hipertensión, cardiomegalia y disfunción diastólica


/
Pregunta 24 Correcta Puntúa 0,16 sobre 0,16

Si durante el examen físico en el área genital encuentra una úlcera lisa no purulenta bien delimitado
única super cial con linfadenopatía bilateral usted piensa en:

Seleccione una:

o a. chancroide

@ b. sí lis

o c. herpes tipo 2

o d. leishmania

La respuesta correcta es: sí lis

Pregunta 25 Correcta Puntúa 0,16 sobre 0,16

Si un paciente es detectado con HbA1c: > 9%, la acción correcta para su tratamiento sería:

Seleccione una:

o a. Se debe realizar a triple terapia de ADO

o b. La primera línea indicada es la metformina en busca de HbAc1 de < 7%

@ c. Iniciar inmediatamente con insulina.

o d. Administrar metformina junto con un ADO

La respuesta correcta es: Iniciar inmediatamente con insulina.

◄ Link de sesión zoom examen segundo parcial Ir a... V

Link de sesión examen tercer parcial ►


/
FERNANDA MARISOL SANDOVAL FARINANGO

Área personal  Mis cursos  Quito  CIENCIAS DE LA SALUD  MEDICINA - PRESENCIAL  ABR 2020 - AGO 2020
 INVESTIGACION I-NEURO-TEORIA - Prl: MD NVA Pen: 961  General  INVESTIGACIÓN I

Comenzado el viernes, 26 de junio de 2020, 14:15


Estado Finalizado
Finalizado en viernes, 26 de junio de 2020, 14:40
Tiempo empleado 24 minutos 52 segundos
Cali cación 1,50 de 4,00 (38%)

Pregunta 1 Incorrecta Puntúa 0,00 sobre 0,25

En la hemorragia subaracnoidea por ruptura de un aneurisma cerebral:

Seleccione una:

@ a. Alteración del estado de conciencia es inusual al inicio del evento X


o b. Una punción lumbar está indicada cuando la neuroimagen es normal

o c. El vasoespasmo es una complicación frecuente dentro de los 3 primeros días del evento

o d. La ruptura de aneurismas de la circulación posterior es la causa más frecuente

La respuesta correcta es: Una punción lumbar está indicada cuando la neuroimagen es normal

/
Pregunta 2 Incorrecta Puntúa 0,00 sobre 0,25

La localización cortical es una característica de la hemorragia secundaria a:

Seleccione una:

o a. Fármacos simpaticomiméticos

o b. Activador del plasminógeno tisular

o c. Cocaina

@ d. Hipertensión arterial X

La respuesta correcta es: Activador del plasminógeno tisular

Pregunta 3 Incorrecta Puntúa 0,00 sobre 0,25

En el trabajo diagnóstico de un hombre de 55 años que presenta crisis parciales motoras con generalización
secundaria, además de una adecuada historia clínica es necesario:

Seleccione una:

@ a. Electroencefalograma con privación de sueño. Opcional tomografía cerebral contrastada X


o b. Resonancia magnética cerebral simple, contrastada y electroencefalograma con privación sueño

o c. Resonancia magnética cerebral simple, contrastada y electroencefalograma en vigilia

o d. Estudio del líquido cefalorraquídeo. Opcional electroencefalograma con privación de sueño

La respuesta correcta es: Resonancia magnética cerebral simple, contrastada y electroencefalograma con
privación sueño

/
Pregunta 4 Correcta Puntúa 0,25 sobre 0,25

Una característica en común de las ausencias y de las crisis parciales complejas es:

Seleccione una:

o a. Automatismos oromandibulares

o b. Electroencefalograma con punta onda a 3 Hz por segundo

@ c. Afectación de la conciencia

o d. Duración del evento

La respuesta correcta es: Afectación de la conciencia

Pregunta 5 Correcta Puntúa 0,25 sobre 0,25

De los criterios diagnósticos de la Sociedad Internacional de la Cefalea para el diagnóstico de migraña sin
aura, considera el más importante

Seleccione una:

o a. Que la cefalea debe ser siempre pulsátil y unilateral

o b. Dolor de intensidad moderada o severa

@ c. El número de episodios de la cefalea

o d. Los síntomas acompañantes como nausea y/o vómito o fotofobia y fonofobia

La respuesta correcta es: El número de episodios de la cefalea

/
Pregunta 6 Correcta Puntúa 0,25 sobre 0,25

Un signo clínico sugestivo de coma metabólico es:

Seleccione una:

o a. Patrón respiratorio normal

o b. Mirada desconjugada

o c. Postura de decorticación bilateral

@ d. Pupilas isocóricas y reactivas

La respuesta correcta es: Pupilas isocóricas y reactivas

Pregunta 7 Correcta Puntúa 0,25 sobre 0,25

El dato clínico más importante para considerar un evento cerebrovascular como hemorrágico más que
isquémico es:

Seleccione una:

o a. Hemiparesia de instauración súbita

@ b. Alteración temprana del estado de conciencia

o c. Edad mayor de 70 años

o d. Antecedente personal de hipertensión arterial

La respuesta correcta es: Alteración temprana del estado de conciencia

/
Pregunta 8 Correcta Puntúa 0,25 sobre 0,25

Una mujer de 40 años es trasladada inconsciente a la emergencia del hospital. En el examen clínico no abre
los ojos a estímulos dolorosos; emite palabras incompresibles y realiza una extensión y pronación de la
extremidad superior izquierda, con exión plantar izquierda. La puntuación en la escala de Glasgow sería:

Seleccione una:

o a. 5

o b. 4

o c. 7

@ d. 6

La respuesta correcta es: 6

Pregunta 9 Incorrecta Puntúa 0,00 sobre 0,25

En orden de mayor a menor importancia los factores de riesgo para enfermedad cerebrovascular
trombótica se consideran:

Seleccione una:

@ a. Hipertensión arterial, diabetes, obesidad, hipercolesterolemia X


o b. Diabetes, hipercolesterolemia, soplo carotídeo, poliglobulia

o c. Hipertensión arterial, edad, brilación auricular, sedentarismo

o d. Edad, prótesis valvular mecánica cardíaca, diabetes, tabaquismo

La respuesta correcta es: Edad, prótesis valvular mecánica cardíaca, diabetes, tabaquismo

/
Pregunta 10 Incorrecta Puntúa 0,00 sobre 0,25

Un hombre de 79 años, hipertenso y con brilación auricular crónica en tratamiento con betabloqueantes y
warfarina 2.5 mg diarios, presenta un infarto cerebral en el territorio carotídeo de 3.5 horas de evolución.
La valoración inicial objetiva: presión arterial 180/100 mmHg, NIHSS 18. Glucosa 300 mg/dL, plaquetas
150000, INR normal. TC cerebral sin evidencia de hemorragia cerebral.

El médico neurólogo de emergencia no considera oportuno la administración de activador del


plasminógeno tisular (rTPA) debido a:

Seleccione una:

o a. 3.5 horas de evolución del evento

o b. Presión arterial 188/100 mmHg

o c. Tratamiento con warfarina 2.5 mg diarios con INR normal

@ d. NIHSS 18 X

La respuesta correcta es: Tratamiento con warfarina 2.5 mg diarios con INR normal

Pregunta 11 Incorrecta Puntúa 0,00 sobre 0,25

Un criterio para el diagnóstico de neurocisticercosis parenquimatosa de nitiva es:

Seleccione una:

o a. Test inmunológico positivo en sangre o líquido cefalorraquídeo

o b. Quistes parenquimatosos en diferentes estadios de evolución

@ c. Una o más calci caciones en el parenquima cerebral X


o d. Quiste coloidal con test inmunológico positivo en liquido cefalorraquídeo

La respuesta correcta es: Quistes parenquimatosos en diferentes estadios de evolución

/
Pregunta 12 Incorrecta Puntúa 0,00 sobre 0,25

Una mujer de 28 años con historia de migraña desde los 10 años acude con cefalea holocraneal, pulsátil,
acompañada de nausea, vómito y fotofobia, de presentación diaria los últimos 15 días. Ha tomado
medicación para la migraña de venta libre, consiguiendo un alivio parcial. Al momento la cefalea es de 3/10,
se incrementa con la actividad física. El examen neurológico no muestra focalidad, no hay papiledema en el
examen de fondo de ojo. La TC cerebral simple es normal. Consideraría como posibilidad diagnóstica:

Seleccione una:

o a. Cefalea tensional episódica

o b. Migraña sin aura

@ c. Migraña crónica X
o d. Cefalea por abuso de medicación

La respuesta correcta es: Migraña sin aura

Pregunta 13 Correcta Puntúa 0,25 sobre 0,25

Un hombre de 66 años en tratamiento con anticoagulantes sufre un accidente de tránsito violento, lo que le
ocasiona una pérdida de la conciencia. La valoración neurológica inicial objetiva una herida en el cuero
cabelludo en la región fronto-temporal derecha, Glasgow 5 (O1V1M3) con postura de decorticación en las
extremidades izquierdas, anisocoria pupilar (derecha 5 mm no reactiva, izquierda 3 mm reactiva.

La primera hipótesis diagnóstica a considerar es:

Seleccione una:

o a. Laceración

@ b. Hematoma epidural

o c. Hematoma subdural

o d. Contusión

La respuesta correcta es: Hematoma epidural

/
Pregunta 14 Sin contestar Puntúa como 0,25

Una mujer de 58 años presenta una parálisis facial bilateral con tetraparesia arre éctica sin trastorno
sensitivo y di cultad para respirar. 7 días antes presentó varias deposiciones diarréicas. El coprocultivo fue
positivo para campylobacter jejuni. El líquido cefalorraquídeo con 10 células monocleadas por mm3. Las
velocidades de conducción motora muy alteradas en las 4 extremidades.

Recomendaría para su tratamiento:

Seleccione una:

o a. Esteroides 1 mg Kg diario por 30 días

o b. Inmunoglobulina 0.4 g/Kg por 5 días y esteroides 1 mg/Kg

o c. Plasmaféresis 5 sesiones

o d. Plasmaferesis 5 sesiones e inmunoglobulina 0.4 g/kg diario por 5 días

La respuesta correcta es: Plasmaféresis 5 sesiones

Pregunta 15 Incorrecta Puntúa 0,00 sobre 0,25

Con un test de apnea con hipotensión arterial o arritmia cardíaca y pCO2 mayor de 60 mmHg, una vez
desconectado del respirador se debe considerar:

Seleccione una:

o a. Solicitar un Doppler transcraneal

o b. Diagnóstico clínico de muerte cerebral

o c. Repetir el procedimiento después de 10 minutos

@ d. Solicitar una angiografía cerebral X

La respuesta correcta es: Diagnóstico clínico de muerte cerebral

/
Pregunta 16 Incorrecta Puntúa 0,00 sobre 0,25

El tratamiento pro láctico de elección para la neuralgia del trigémino clásica es:

Seleccione una:

o a. Opioides

o b. Agonistas de los receptores 5HT/1B-1D

@ c. Anti-in amatorios no esteroidales X


o d. Bloqueantes de los canales de sodio

La respuesta correcta es: Bloqueantes de los canales de sodio

◄ CEFALEA Ir a... V EPILEPSIA ►

/
GLORIA JUDITH LOYA ANDRADE

Área personal  Mis cursos  Quito  CIENCIAS DE LA SALUD  MEDICINA - PRESENCIAL  ABR 2020 - AGO 2020
 INVESTIGACIÓN I - Prl: MD NVC Pen: 961  13 de junio: Examen Primer Parcial Investigación I  Examen Primer Parcial
Investigación I

Comenzado el sábado, 13 de junioprimer


de 2020, 10:10
parcial inves
Estado Finalizado
Finalizado en sábado, 13 de junio de 2020, 10:52
Tiempo empleado 42 minutos 16 segundos
Cali cación 3,68 de 4,00 (92%)

Pregunta 1 Correcta Puntúa 0,16 sobre 0,16

Cuál de los siguientes es un signo característico de las vías respiratorias de las personas que sufren
asma:

Seleccione una:

o a. In ltración basó la

o b. Adhesión de los glóbulos rojos al endotelio

o c. Transformación de los basó los en macrófagos

@ d. In ltración eosinofílica

La respuesta correcta es: In ltración eosinofílica


/
Pregunta 2 Correcta Puntúa 0,16 sobre 0,16

¿Cuál es la clasi cación anatómica de las bronquiectasias?

Seleccione una:

o a. Cubicas, cilíndricas, alargadas

@ b. Cilíndricas, varicosas, saculares

o c. Vasculares, semilunares saculares

o d. Lobares, arteriales, globulares

La respuesta correcta es: Cilíndricas, varicosas, saculares

Pregunta 3 Correcta Puntúa 0,16 sobre 0,16

¿Cuál es la siopatología de la hemoptisis?

Seleccione una:

o a. Hipervascularización de la circulación pulmonar, hipertensión pulmonar y remodelación.

@ b. Hipervascularización de la circulación brónquica, hipertensión pulmonar y neovascularización

o c. Hipervascularización de la circulación pulmonar, ebre y regeneración alveolar

o d. Hipervascularización de la circulación brónquica, hipertensión pulmonar y disminución de


coagulabilidad

La respuesta correcta es: Hipervascularización de la circulación brónquica, hipertensión pulmonar y


neovascularización


/
Pregunta 4 Correcta Puntúa 0,16 sobre 0,16

Cuál es una causa de taquicardia sinusal siológica:

Seleccione una:

o a. Hipertensión pulmonar

@ b. Feocromocitoma

o c. Hipotiroidismo

o d. Sedenterismo

La respuesta correcta es: Feocromocitoma

Pregunta 5 Correcta Puntúa 0,16 sobre 0,16

Cuál síndrome genético es común en la insu ciencia aórtica:

Seleccione una:

o a. Síndrome de Rastelli

o b. Síndrome de Eisenmenger

o c. Anomalía de Ebstein

@ d. Síndrome de Marfán

La respuesta correcta es: Síndrome de Marfán


/
Pregunta 6 Correcta Puntúa 0,16 sobre 0,16

Cuánto es la dosis intravenosa de sostén para digoxina en arritmias:

Seleccione una:

o a. 0.145-0.250 mg/día

@ b. 0.125-0.250 mg/día

o c. 0.200-0.250 mg/día

o d. 0.175-0.250 mg/día

La respuesta correcta es: 0.125-0.250 mg/día

Pregunta 7 Correcta Puntúa 0,16 sobre 0,16

Cuánto es la dosis sublingual de nitroglicerina en angina de pecho:

Seleccione una:

o a. 0.8 a 1.0 mg

@ b. 0.3 a 0.6 mg

o c. 1.3 a 1.6 mg

o d. 3 a 6 mg

La respuesta correcta es: 0.3 a 0.6 mg


/
Pregunta 8 Correcta Puntúa 0,16 sobre 0,16

Cuánto es la frecuencia de administración de atenolol en cardiopatía isquémica:

Seleccione una:

o a. Cada 8 horas

o b. Cada 6 horas

@ c. Cada 24 horas

o d. Cada 12 horas

La respuesta correcta es: Cada 24 horas

Pregunta 9 Correcta Puntúa 0,16 sobre 0,16

De los siguientes factores, cuál es el que debe estar alterado para que un paciente infectado de
tuberculosis se convierta en persona enferma:

Seleccione una:

o a. Huésped: que genéticamente esté predispuesto a infectarse

o b. Medio Ambiente: Que viva en un lugar frío

@ c. Huésped: Inmunidad celular de ciente, en especial CD4

o d. Medio ambiente: Que haya elevada prevalencia de tuberculosis

La respuesta correcta es: Huésped: Inmunidad celular de ciente, en especial CD4


/
Pregunta 10 Incorrecta Puntúa 0,00 sobre 0,16

El hallazgo más frecuente en la radiografía de tórax en un paciente con asma es:

Seleccione una:

o a. Hiperinsu ación pulmonar.

o b. Condensaciones alveolares bilaterales y difusas.

@ c. Engrosamiento de paredes bronquiales. X


o d. Radiografía de tórax normal.

La respuesta correcta es: Radiografía de tórax normal.

Pregunta 11 Correcta Puntúa 0,16 sobre 0,16

El tratamiento de las Bronquiectasias se basa en 3 pilares, excepto:

Seleccione una:

o a. Mejorar la eliminación de las secreciones, que se consigue con una adecuada hidratación, con
sioterapia respiratoria y drenaje postural mantenidos.

@ b. Revertir el remodelamiento bronquia

o c. Controlar las infecciones con el uso de antibióticos en las agudizaciones durante 10-15 días.

o d. Eliminar la obstrucción bronquial.

La respuesta correcta es: Revertir el remodelamiento bronquia


/
Pregunta 12 Correcta Puntúa 0,16 sobre 0,16

El valor de oliguria de ne?

Seleccione una:

o a. 500 a 1000 ml

o b. 0 a 50 ml

@ c. 100 a 400 ml

o d. 1000 a 1500 ml

La respuesta correcta es: 100 a 400 ml

Pregunta 13 Correcta Puntúa 0,16 sobre 0,16

El virus de hepatitis B, puede ser causa de algunas enfermedades como agente biológico. ¿Cuál de estas
enfermedades?

Seleccione una:

o a. Anemia por dé cit de hierro

@ b. Hipoplasia medular

o c. Anemia megaloblástica

o d. Anemia de proceso crónico

La respuesta correcta es: Hipoplasia medular


/
Pregunta 14 Correcta Puntúa 0,16 sobre 0,16

En cuanto al diagnóstico funcional del asma:

Seleccione una:

o a. Si la relación VEF1/CVF es mayor 0.7 se considera patrón obstructivo, diagnosticamos asma

o b. Valores espirométricos no tienen importancia en el diagnóstico del asma.

@ c. Si la relación VEF1/CVF es menor a 0.7 (patrón obstructivo) y post broncodilatador obtenemos un


aumento igual o mayor al 12% en el VEF1, nos orienta hacia diagnóstico de asma

o d. Si la relación VEF1/CVF es menor a 0.7 (patrón restrictivo) y post broncodilatador obtenemos un


aumento igual o mayor al 12% en el VEF1, nos orienta hacia diagnóstico de asma

La respuesta correcta es: Si la relación VEF1/CVF es menor a 0.7 (patrón obstructivo) y post broncodilatador
obtenemos un aumento igual o mayor al 12% en el VEF1, nos orienta hacia diagnóstico de asma

Pregunta 15 Incorrecta Puntúa 0,00 sobre 0,16

Existen fármacos que aumentan o disminuyen la acción de los anticoagulantes ¿Cuál de estos fármacos
aumenta su acción?

Seleccione una:

o a. Anticonceptivo

o b. Rifampicina

o c. Aspirina

@ d. Vitamina K X

La respuesta correcta es: Aspirina


/
Pregunta 16 Correcta Puntúa 0,16 sobre 0,16

La causa principal de un fracaso en el tratamiento de la tuberculosis pulmonar es

Seleccione una:

o a. Resistencia primaria a las drogas

@ b. Abandono del tratamiento por el paciente

o c. Resistencia secundaria a las drogas

o d. Toxicidad hepática

La respuesta correcta es: Abandono del tratamiento por el paciente

Pregunta 17 Correcta Puntúa 0,16 sobre 0,16

La ebre por meningitis que trastorno ácido básico produce?

Seleccione una:

o a. Acidosis metabólica

o b. Alcalosis metabólica

@ c. Alcalosis respiratoria

o d. Acidosis respiratoria

La respuesta correcta es: Alcalosis respiratoria


/
Pregunta 18 Correcta Puntúa 0,16 sobre 0,16

La triada de Virchow está de nida por lesión endotelial, hipercoagulabilidad y la inamovilidad. ¿Cuál de
estas es un riesgo de hipercoagulabilidad?

Seleccione una:

@ a. Estrógenos

o b. Edad avanzada

o c. Apoplejía lesión de columna

o d. Cirugía

La respuesta correcta es: Estrógenos

Pregunta 19 Correcta Puntúa 0,16 sobre 0,16

La trombosis venosa tiene sus características por estar constituidos por mucha brina y hematíes
(trombo rojo) ¿Cuál de estos factores es de riesgo para una trombosis venosa?

Seleccione una:

o a. Dislipidemia

o b. Diabetes

o c. Hipertensión

@ d. Estasis, inamovilidad

La respuesta correcta es: Estasis, inamovilidad


/
Pregunta 20 Correcta Puntúa 0,16 sobre 0,16

Qué infección causa bloqueo auriculoventricular:

Seleccione una:

@ a. Sí lis

o b. Tripanosomiasis

o c. Varicela

o d. Sarampión

La respuesta correcta es: Sí lis

Pregunta 21 Correcta Puntúa 0,16 sobre 0,16

Según Harrison, cuál es un microorganismo virulento para ITU por propagación hematógena:

Seleccione una:

o a. Acinetobacter Baumani

o b. Estreptococo aureus

o c. Proteus mirabilis

@ d. Esta lococo aureus

La respuesta correcta es: Esta lococo aureus


/
Pregunta 22 Correcta Puntúa 0,16 sobre 0,16

Según Harrison, cuántas semanas se requieren para catalogar cistitis recurrente?

Seleccione una:

o a. 8 semanas

@ b. 2 semanas

o c. 4 semanas

o d. 6 semanas

La respuesta correcta es: 2 semanas

Pregunta 23 Correcta Puntúa 0,16 sobre 0,16

Si existen una intoxicacion crónica por Sal Andrews que trastorno ácido básico se produce?

Seleccione una:

o a. Acidosis metabólica

@ b. Alcalosis metabólica

o c. Acidosis respiratoria

o d. Alcalosis respiratoria

La respuesta correcta es: Alcalosis metabólica


/
Pregunta 24 Correcta Puntúa 0,16 sobre 0,16

Una reacción transfusional aguda o inmediata, se de ne como la que ocurre durante la transfusión o en
las 24 horas. ¿Cuál de estas opciones es una reacción aguda?

Seleccione una:

@ a. Reacción Alérgica Urticaria

o b. Transmisión de un HIV

o c. Purpura post transfusional

o d. Enfermedad del Suero

La respuesta correcta es: Reacción Alérgica Urticaria

Pregunta 25 Correcta Puntúa 0,16 sobre 0,16

Un paciente hipertenso con descompensación aguda “típica” de insu ciencia cardiaca sin sobrecarga de
volumen, que fármaco usaría:

Seleccione una:

o a. Clortalidona

@ b. Nitroprusiato

o c. Dobutamina

o d. Furosemida

La respuesta correcta es: Nitroprusiato

◄ Avisos Ir a... V Link sesión zoom examen primer parcial ►


/
2
JORGE PAUL GUAILLAS GUAILLAS

Área personal  Mis cursos  Quito  CIENCIAS DE LA SALUD  MEDICINA - PRESENCIAL  ABR 2020 - AGO 2020
 INVESTIGACIÓN I - Prl: MD NVD Pen: 961  Sábado 4 de julio 8:00 am. Investigación I: Examen nal 1  Examen nal 1:
Investigación I

Comenzado el sábado, 4 de julio de 2020, 08:10


Estado Finalizado
Finalizado en sábado, 4 de julio de 2020, 09:30
Tiempo empleado 1 hora 19 minutos
Cali cación 4,93 de 5,33 (93%)

Pregunta 1 Correcta Puntúa 0,13 sobre 0,13

Ante un paciente con infarto cerebral en fase aguda, seleccione la respuesta correcta en cuanto al
tratamiento.

Seleccione una:

o a. Se debe valorar tratamiento trombolítico intravenoso si llega antes de 6 horas del inicio de los
síntomas.

o b. Debe reducirse la presión arterial siempre.

@ c. Se debe utilizar manitol cuando el edema cerebral produce disminución de la conciencia o se


observa desplazamiento de la línea media en la TC.

o d. Se debe colocar solución de Dextrosa 5% para hidratación.

La respuesta correcta es: Se debe utilizar manitol cuando el edema cerebral produce disminución de la
conciencia o se observa desplazamiento de la línea media en la TC.


/
Pregunta 2 Correcta Puntúa 0,13 sobre 0,13

¿A qué género pertenece el virus del dengue?

Seleccione una:

o a. Poxvirus

@ b. Flaviviridae

o c. Hantavirus

o d. Coronavirus

La respuesta correcta es: Flaviviridae

Pregunta 3 Correcta Puntúa 0,13 sobre 0,13

¿Cómo se denomina el vector transmisor de ebre amarilla?

Seleccione una:

o a. Anopheles

o b. Culecoides

o c. Lutzomya

@ d. Aedes aegypti

La respuesta correcta es: Aedes aegypti


/
Pregunta 4 Correcta Puntúa 0,13 sobre 0,13

¿Cuál de las siguientes alternativas es una de las medidas para evitar la diseminación del Dengue entre
la población?

Seleccione una:

o a. Pro laxis con Cloroquina – Primaquina

o b. Administración de la vacuna a poblaciones susceptibles.

@ c. Control vectorial

o d. Introducción experimental de mosquitos transgénicos

La respuesta correcta es: Control vectorial

Pregunta 5 Correcta Puntúa 0,13 sobre 0,13

¿Cuál de las siguientes es una causa de hemoptisis?

Seleccione una:

o a. Vàrices esofágicas

o b. Edema agudo de Pulmòn

o c. Fibrosis Pulmonar

@ d. Estenosis Mitral

La respuesta correcta es: Estenosis Mitral


/
Pregunta 6 Correcta Puntúa 0,13 sobre 0,13

Cuál de los siguientes enunciados corresponde a la de nición de neumonía nosocomial tardía

Seleccione una:

o a. Aparece después de 14 días del ingreso

o b. Aparece después de 21 días del ingreso

o c. Aparece después de 10 días del ingreso

@ d. Aparece después de 7 días del ingreso

La respuesta correcta es: Aparece después de 7 días del ingreso

Pregunta 7 Correcta Puntúa 0,13 sobre 0,13

¿Cuál es la acción de los nitratos en el síndrome coronario agudo?

Seleccione una:

@ a. vasodilatador coronario

o b. revascularizador

o c. sedante

o d. antiagregante

La respuesta correcta es: vasodilatador coronario


/
Pregunta 8 Correcta Puntúa 0,13 sobre 0,13

¿Cuál es la característica de la cefalea en racimos?

Seleccione una:

o a. Síntomas bilaterales

@ b. Síntomas ipsilaterales

o c. Dolor en la región occipital del cráneo

o d. Dolor leve

La respuesta correcta es: Síntomas ipsilaterales

Pregunta 9 Correcta Puntúa 0,13 sobre 0,13

¿Cuál es la diferencia entre una crisis focal y una crisis generalizada?

Seleccione una:

a. Una crisis focal se limita a un solo hemisferio mientras que una crisis generalizada se distribuye en
ambos hemisferios.

o b. Una crisis focal se origina en el interior conectándose con las redes de ambos hemisferios mientras
que una crisis generalizada se limita a un solo hemisferio.

o c. Una crisis focal se origina en un lóbulo del cerebro mientras que una generalizada se origina en las
redes límites de todo un hemisferio.

o d. No existe diferencia

La respuesta correcta es: Una crisis focal se limita a un solo hemisferio mientras que una crisis generalizada
se distribuye en ambos hemisferios.


/
Pregunta 10 Correcta Puntúa 0,13 sobre 0,13

¿Cuál es la función de la aspirina en el síndrome coronario agudo?

Seleccione una:

o a. Analgésico antin amatorio

@ b. Evitar el incremento de la agregación plaquetaria en la placa ateromatosa accidentada

o c. Anticoagulantes

o d. Trombolisis

La respuesta correcta es: Evitar el incremento de la agregación plaquetaria en la placa ateromatosa


accidentada

Pregunta 11 Correcta Puntúa 0,13 sobre 0,13

¿Cuáles son signos de insu ciencia cardíaca derecha?

Seleccione una:

o a. Ortópnea, dolor precordial

o b. Dolor abdominal, astenia

@ c. Edema de miembros inferiores, hepatomegalia

o d. Disnea, estertores crepitantes

La respuesta correcta es: Edema de miembros inferiores, hepatomegalia


/
Pregunta 12 Correcta Puntúa 0,13 sobre 0,13

¿Cuántos días abarca el período de incubación de la leptospirosis?

Seleccione una:

o a. De 10 a 20 días (aproximadamente 2 semanas)

o b. De 1 a 15 días (aproximadamente 1 semana)

@ c. De 1 a 30 días (aproximadamente 2 semanas)

o d. De 1 a 60 días (aproximadamente 3 semanas)

La respuesta correcta es: De 1 a 30 días (aproximadamente 2 semanas)

Pregunta 13 Correcta Puntúa 0,13 sobre 0,13

De las encefalitis virales seleccione la que se bene cia del tratamiento con Aciclovir:

Seleccione una:

o a. Citomegalovirus

o b. Encefalitis equina

@ c. Herpes virus

o d. Enterovirus

La respuesta correcta es: Herpes virus


/
Pregunta 14 Correcta Puntúa 0,13 sobre 0,13

¿De qué depende el mayor riesgo de ENFERMARSE de Tuberculosis?

Seleccione una:

@ a. Inmunidad celular, CD4

o b. Estilo de vida

o c. Prevalencia de Tuberculosis

o d. Genètica

La respuesta correcta es: Inmunidad celular, CD4

Pregunta 15 Correcta Puntúa 0,13 sobre 0,13

El derrame pleural puede ser causado por diferentes causas. ¿En cuál de ellos usted sugiere realizar una
pleurodesis?

Seleccione una:

o a. Por Insu ciencia Cardíaca

o b. Por Tuberculosis

o c. Por Neumonía

@ d. Por Cáncer

La respuesta correcta es: Por Cáncer


/
Pregunta 16 Correcta Puntúa 0,13 sobre 0,13

El dolor tipo pleurítico se caracteriza por ser:

Seleccione una:

o a. Opresivo intenso que disminuye con la inspiración

o b. Opresivo intenso que aumenta con la inspiraciòn

o c. Punzante intenso que disminuye con la inspiración

@ d. Punzante intenso que aumenta con la inspiración

La respuesta correcta es: Punzante intenso que aumenta con la inspiración

Pregunta 17 Correcta Puntúa 0,13 sobre 0,13

El uso de betabloqueantes debe ser analizado en el paciente con insu ciencia cardiaca ya que este
grupo farmacológico posee ciertas características importantes. Señale lo correcto respecto a estos
productos:

Seleccione una:

o a. Puede utilizarse con seguridad en pacientes con asma

o b. Pueden utilizarse con seguridad en combinación con verapamilo

@ c. Pueden utilizarse con seguridad en pacientes con insu ciencia cardiaca sistólica y angina de
esfuerzo

o d. Pueden utilizarse con seguridad en pacientes con hipotensión sintomático

La respuesta correcta es: Pueden utilizarse con seguridad en pacientes con insu ciencia cardiaca sistólica y
angina de esfuerzo


/
Pregunta 18 Correcta Puntúa 0,13 sobre 0,13

¿En cuál sistema arterial se producen la mayoría de las hemoptisis y por qué? Señale lo CORRECTO

Seleccione una:

o a. Las arterias pulmonares son un sistema de alta presión por el que circula todo el gasto cardíaco

o b. Las arterias pulmonares son un sistema de baja presión por el que circula todo el gasto cardiaco y
son las responsables del intercambio gaseoso

o c. Las arterias bronquiales forman parte de la circulación sistémica, tienen menor presión y mucho
mayor ujo, de ellas depende la irrigación de los bronquios y la pleura visceral.

d. Las arterias bronquiales forman parte de la circulación sistémica, tienen mayor presión y mucho
menor ujo, de ellas depende la irrigación de los bronquios y la pleura visceral.

La respuesta correcta es: Las arterias bronquiales forman parte de la circulación sistémica, tienen mayor
presión y mucho menor ujo, de ellas depende la irrigación de los bronquios y la pleura visceral.

Pregunta 19 Correcta Puntúa 0,13 sobre 0,13

¿En qué caso complementamos el electrocardiograma con derivaciones derechas?

Seleccione una:

@ a. Sospecha de infarto de ventrículo derecho

o b. Ritmo de galope

o c. Sospecha de edema agudo de pulmón

o d. Taquicardia ventricular monomorfa

La respuesta correcta es: Sospecha de infarto de ventrículo derecho


/
Pregunta 20 Correcta Puntúa 0,13 sobre 0,13

En una paciente de 30 años es llevada inconsciente a la emergencia de un hospital. Uno de los signos
clínicos que excluye el diagnóstico de muerte cerebral es:

Seleccione una:

o a. Glasgow 3

@ b. postura de decerebración bilateral

o c. pupilas midriáticas arreactivas

o d. ausencia de re ejo nauseoso y tusígeno

La respuesta correcta es: postura de decerebración bilateral

Pregunta 21 Correcta Puntúa 0,13 sobre 0,13

Forma de transmisión del virus de la hepatitis A

Seleccione una:

o a. Por animales

o b. vía respiratoria

o c. sexual

@ d. vía fecal- oral

La respuesta correcta es: vía fecal- oral


/
Pregunta 22 Correcta Puntúa 0,13 sobre 0,13

La causa más frecuente de hemoptisis leve-moderada es:

Seleccione una:

o a. Diátesis hemorrágica.

o b. Infarto pulmonar.

o c. Carcinoma broncogénico.

@ d. Bronquiectasias.

La respuesta correcta es: Bronquiectasias.

Pregunta 23 Correcta Puntúa 0,13 sobre 0,13

La enfermedad pulmonar obstructiva crónica no tratada puede conducir a la siguiente complicación


cardíaca:

Seleccione una:

o a. Pericarditis

o b. Bradicardia

@ c. Insu ciencia cardíaca derecha

o d. Síndrome coronario agudo

La respuesta correcta es: Insu ciencia cardíaca derecha


/
Pregunta 24 Correcta Puntúa 0,13 sobre 0,13

La insu ciencia cardíaca izquierda puede provocar edema agudo de pulmón por aumento de la:

Seleccione una:

@ a. Presión capilar pulmonar

o b. Presión arterial

o c. Presión venosa central

o d. Presión sistólica del ventrículo derecho

La respuesta correcta es: Presión capilar pulmonar

Pregunta 25 Correcta Puntúa 0,13 sobre 0,13

Los promastigotes son la forma de leishmania que se inyectan por medio de la probóscide del
ebótomo de sexo femenino en la piel del hospedador, ¿qué células son las encargadas de fagocitar a
estas formas parasitarias?

Seleccione una:

@ a. Neutró los

o b. Eosinó los

o c. Monocitos

o d. Macrófagos

La respuesta correcta es: Neutró los


/
Pregunta 26 Correcta Puntúa 0,13 sobre 0,13

María, 65 años, acude a la consulta re riendo cansancio a los moderados esfuerzos, re ere haber tenido
ebre reumática, al examen físico presenta PA 130/80, FC 88lpm, soplo sistólico en quinto espacio
intercostal línea media clavicular izquierda III/VI, con irradiación a la axila. ¿Cuál es el diagnóstico más
probable en esta paciente?

Seleccione una:

o a. Estenosis mitral

o b. Insu ciencia aortica

@ c. Insu ciencia mitral

o d. Estenosis aortica

La respuesta correcta es: Insu ciencia mitral

Pregunta 27 Correcta Puntúa 0,13 sobre 0,13

Paciente acude con trastornos osteomusculares crónicos, junto con dé cit visual inespecí co, pérdida de
peso y ebre de larga evaluación, el paciente proviene de una zona meramente ganadera, Usted
sospecha de brucelosis cuya base de tratamiento es:

Seleccione una:

o a. Penicilina y Metronidazol

@ b. Dicloxacilina y Estreptomicina

o c. Gentamicina y Dapsona

o d. Isoniacida y Estreptomicina

La respuesta correcta es: Dicloxacilina y Estreptomicina


/
Pregunta 28 Correcta Puntúa 0,13 sobre 0,13

Paciente de 25 años que re ere viaje reciente a Filipinas presenta: ebre, dolor abdominal, diarrea,
badicardia relativa. ¿Cuál es el diagnóstico más probable?

Seleccione una:

@ a. Fiebre tifoidea

o b. Micosos sistémica

o c. Paludismo

o d. Gastroenteritis complicada

La respuesta correcta es: Fiebre tifoidea

Pregunta 29 Correcta Puntúa 0,13 sobre 0,13

Paciente masculino de 55 años de edad, sin antecedentes patológicos familiares de relevancia. Acude
por presentar desde hace 6 días dolor intenso y alodinia a nivel de región lumbar derecha. Hoy en la
mañana nota erupciones muy pruriginosas en la piel. Antecedentes patológicos personales: varicela en
su niñez. Al examen físico presenta vesículas agrupadas sobre una placa eritematosa elevada que
siguen un mismo dermatoma. También se observa un desprendimiento de costras dejando zonas
expuestas de piel. ¿Cuál sería el diagnóstico probable y su terapéutica?

Seleccione una:

@ a. Herpes Zoster y su tratamiento más adecuado es Aciclovir 800 mg cada 4 horas durante 7 días.

o b. Infección por Herpes Zoster el mismo que necesitará tratamiento inmediato con Fluconazol 250 mg
VO cada día por 5 días.

o c. Dermatitis de contacto y deberá averiguarse sobre factores de predisposición alérgica en el


paciente.

o d. Herpes Zoster cuyo tratamiento ideal es Aciclovir, valaciclovir o famciclovir formando parte estos
fármacos de la familia de los inhibidores de la transcriptasa inversa.

La respuesta correcta es: Herpes Zoster y su tratamiento más adecuado es Aciclovir 800 mg cada 4 horas
durante 7 días.


/
Pregunta 30 Correcta Puntúa 0,13 sobre 0,13

Qué antihipertensivo se debe usar con cautela en un paciente con arteriopatia coronaria grave:

Seleccione una:

o a. Atenolol

o b. Clonidina

o c. Espironolactona

@ d. Hidralazina

La respuesta correcta es: Hidralazina

Pregunta 31 Correcta Puntúa 0,13 sobre 0,13

Qué endocrinopatía es causa secundaria de hipertensión arterial:

Seleccione una:

o a. Hiperprolactinemia

o b. Hipogonadismo

o c. Hipocalcemia

@ d. Síndrome de Cushing

La respuesta correcta es: Síndrome de Cushing


/
Pregunta 32 Correcta Puntúa 0,13 sobre 0,13

¿Qué es una convulsión?

Seleccione una:

a. Episodio paroxístico producido por descargas anormales excesivas o actividad neuronal sincrónica
en el cerebro

o b. Episodio paroxístico producido por descargas anormales disminuidas o actividad neuronal no


sincrónica en el cerebro

o c. Trastorno en el que una persona tiene convulsiones o crisis recurrentes debido a proceso crónico
subyacente

o d. Ninguna de las anteriores

La respuesta correcta es: Episodio paroxístico producido por descargas anormales excesivas o actividad
neuronal sincrónica en el cerebro

Pregunta 33 Correcta Puntúa 0,13 sobre 0,13

¿Qué parámetros se consideran para tener migraña crónica?

Seleccione una:

o a. Episodios de migraña cada semana

o b. Episodios de migraña con alteraciones visuales

o c. Episodios de migraña por más de tres años

@ d. Episodios de migraña todos los días o casi a diario

La respuesta correcta es: Episodios de migraña todos los días o casi a diario


/
Pregunta 34 Correcta Puntúa 0,13 sobre 0,13

Recibe el reporte de una muestra de líquido pleural con los siguientes resultados: exudado amarillento,
recuento del 90% de linfocitos. Pertenece a un paciente que presenta ebre y baja de peso, ¿en cuál de
los siguientes diagnósticos usted pensaría?

Seleccione una:

o a. Empiema

o b. Enfermedad del colágeno

@ c. Tuberculosis

o d. Neoplasia

La respuesta correcta es: Tuberculosis

Pregunta 35 Incorrecta Puntúa 0,00 sobre 0,13

Señale las características del síndrome atípico de la Neumonía Adquirida en la Comunidad.

Seleccione una:

o a. Clínica aguda.

@ b. Más evidente en personas ancianas X


o c. Se caracteriza por ebre elevada, escalofríos, tos productiva y dolor pleurítico

o d. Disociación clínico radiológica

La respuesta correcta es: Disociación clínico radiológica


/
Pregunta 36 Correcta Puntúa 0,13 sobre 0,13

Su paciente proviene de la amazonia del Ecuador y presenta aproximadamente 15 días con ebre alta
con escalofríos, pero sin pérdida de peso considerable, solo con ligera palidez, al examen físico se
parecía muy levemente una hepato espleno megalia, y Biometría sin cambios aparente, sería prudente
realizar lo siguiente a su criterio:

Seleccione una:

@ a. Gota gruesa y extendido seriado

o b. Ecografía y determinar si existe ascitis y derrame pleural (Extravasación)

o c. Biopsia de Hígado y descartar procesos de ebre de larga evolución (FOD clásico)

o d. Hemocultivo y esperar resultados

La respuesta correcta es: Gota gruesa y extendido seriado

Pregunta 37 Incorrecta Puntúa 0,00 sobre 0,13

Una mujer de 40 años presenta debilidad progresiva y dolor en las piernas después de una semana de
haber presentado un episodio gripal. El examen físico demuestra tensión arterial 80/40 mm Hg, pulso
150 por minuto, temperatura bucal 37 grados C. Fuerza normal en la cara. Paresia 2/5 en la extremidad
inferior derecha y 3/5 en las extremidad inferior izquierda. Arre exia global con respuestas plantares en
exión. Uno de los signos o síntomas que podría hacer dudar en el diagnóstico de un síndrome de
Guillain-Barré es:

Seleccione una:

o a. ausencia de paresia facial

o b. disfunción autonómica

o c. debilidad asimétrica en las extremidades inferiores

@ d. dolor en las piernas X

La respuesta correcta es: debilidad asimétrica en las extremidades inferiores


/
Pregunta 38 Correcta Puntúa 0,13 sobre 0,13

Uno de los criterios para considerar la probabilidad de muerte cerebral y proceder a realizar el test de
apnea es:

Seleccione una:

o a. temperatura corporal de 36 grados centígrados

@ b. TC cerebral con una lesión estructural cerebral compatible con muerte cerebral

o c. pupilas mióticas de 2 mm débilmente reactivas a la luz

o d. postura de descerebración luego de una reanimación cardiopulmonar de 20 minutos

La respuesta correcta es: TC cerebral con una lesión estructural cerebral compatible con muerte cerebral

Pregunta 39 Correcta Puntúa 0,13 sobre 0,13

Un paciente de 35 años, consulta por cuadro de 4 semanas de evolución, de tos, con expectoración
mucopurulenta, con estrías de sangre en algunas ocasiones, asociado a compromiso del estado general,
baja de peso, ebre intermitente y sudoración nocturna. Ha tomado amoxicilina en varias
oportunidades, sin respuesta. Al examen físico destacan crépitos escasos, mayores en el ápice derecho.
El diagnóstico más probable es:

Seleccione una:

o a. Absceso pulmonar

@ b. Tuberculosis

o c. Cáncer pulmonar

o d. Bronquiectasias

La respuesta correcta es: Tuberculosis


/
Pregunta 40 Incorrecta Puntúa 0,00 sobre 0,13

Un paciente de 50 años, es trasladado inconsciente a la emergencia del hospital. Se procede a intubarlo


para protección de la vía aérea. Al estímulo doloroso no abre por los ojos, presenta exión de la
extremidad superior derecha con extensión plantar del pie derecho. La puntuación en la escala de
Glasgow sería:

Seleccione una:

o a. 4

@ b. 6 X
o c. 7

o d. 5

La respuesta correcta es: 5

◄ Link sesión zoom examen primer parcial Ir a... V

Link sesión zoom examen nal 1: Investigación I ►


/
Ana Villamarín Rodríguez

Área personal  Mis cursos  Quito  CIENCIAS DE LA SALUD  MEDICINA - PRESENCIAL  ABR 2020 - AGO 2020
 INVESTIGACIÓN I - Prl: MD NVB Pen: 961  13 de junio: Examen Primer Parcial Investigación I  Examen Primer Parcial
Investigación I

Comenzado el sábado, 13 de junio de 2020, 10:10


Estado Finalizado
Finalizado en sábado, 13 de junio de 2020, 10:59
Tiempo empleado 49 minutos 49 segundos
Cali cación 2,72 de 4,00 (68%)

Pregunta 1 Correcta Puntúa 0,16 sobre 0,16

Cuál de los siguientes es un signo característico de las vías respiratorias de las personas que sufren asma:

Seleccione una:

o a. In ltración basó la

o b. Transformación de los basó los en macrófagos

o c. Adhesión de los glóbulos rojos al endotelio

@ d. In ltración eosinofílica

La respuesta correcta es: In ltración eosinofílica


Pregunta 2 Correcta Puntúa 0,16 sobre 0,16

¿Cuál es la clasi cación anatómica de las bronquiectasias?

Seleccione una:

o a. Lobares, arteriales, globulares

@ b. Cilíndricas, varicosas, saculares

o c. Vasculares, semilunares saculares

o d. Cubicas, cilíndricas, alargadas

La respuesta correcta es: Cilíndricas, varicosas, saculares

Pregunta 3 Correcta Puntúa 0,16 sobre 0,16

¿Cuál es la siopatología de la hemoptisis?

Seleccione una:

o a. Hipervascularización de la circulación pulmonar, hipertensión pulmonar y remodelación.

o b. Hipervascularización de la circulación brónquica, hipertensión pulmonar y disminución de coagulabilidad

@ c. Hipervascularización de la circulación brónquica, hipertensión pulmonar y neovascularización

o d. Hipervascularización de la circulación pulmonar, ebre y regeneración alveolar

La respuesta correcta es: Hipervascularización de la circulación brónquica, hipertensión pulmonar y


neovascularización

Pregunta 4 Correcta Puntúa 0,16 sobre 0,16

Cuál síndrome genético es común en la insu ciencia aórtica:

Seleccione una:

@ a. Síndrome de Marfán

o b. Síndrome de Rastelli

o c. Anomalía de Ebstein

o d. Síndrome de Eisenmenger

La respuesta correcta es: Síndrome de Marfán 


Pregunta 5 Incorrecta Puntúa 0,00 sobre 0,16

Cuánto es la dosis máxima de dinitrato de isosorbide de liberación prolongada en angina de pecho:

Seleccione una:

o a. 300 mg

o b. 120 mg

@ c. 40 mg X
o d. 160 mg

La respuesta correcta es: 300 mg

Pregunta 6 Correcta Puntúa 0,16 sobre 0,16

Cuánto es la dosis sublingual de nitroglicerina en angina de pecho:

Seleccione una:

o a. 1.3 a 1.6 mg

o b. 0.8 a 1.0 mg

@ c. 0.3 a 0.6 mg

o d. 3 a 6 mg

La respuesta correcta es: 0.3 a 0.6 mg

Pregunta 7 Incorrecta Puntúa 0,00 sobre 0,16

Cuánto es la frecuencia de administración de atenolol en cardiopatía isquémica:

Seleccione una:

o a. Cada 8 horas

@ b. Cada 12 horas X
o c. Cada 6 horas

o d. Cada 24 horas


La respuesta correcta es: Cada 24 horas
Pregunta 8 Incorrecta Puntúa 0,00 sobre 0,16

Cuánto es la frecuencia de administración de propanolol en cardiopatía isquémica:

Seleccione una:

o a. Cada 8 horas

o b. Cada 12 horas

o c. Cada 24 horas

@ d. Cada 6 horas X

La respuesta correcta es: Cada 12 horas

Pregunta 9 Correcta Puntúa 0,16 sobre 0,16

Cuántos mEq de Na contiene 500 ml de Cloruro de sodio al 0.09%?

Seleccione una:

o a. 250 mEq/L

o b. 130 mEq/L

o c. 132 mEq/L

@ d. 154 mEq/L

La respuesta correcta es: 154 mEq/L

Pregunta 10 Correcta Puntúa 0,16 sobre 0,16

De los siguientes factores, cuál es el que debe estar alterado para que un paciente infectado de tuberculosis
se convierta en persona enferma:

Seleccione una:

o a. Medio Ambiente: Que viva en un lugar frío

@ b. Huésped: Inmunidad celular de ciente, en especial CD4

o c. Medio ambiente: Que haya elevada prevalencia de tuberculosis

o d. Huésped: que genéticamente esté predispuesto a infectarse

La respuesta correcta es: Huésped: Inmunidad celular de ciente, en especial CD4 


Pregunta 11 Incorrecta Puntúa 0,00 sobre 0,16

El concentrado plaquetario constituye el derivado con mayor riesgo de contaminación. ¿Cuál es la causa?

Seleccione una:

@ a. Debido a mantener a -20 grados en congelación X


o b. Se mantiene a 6 grados en nevera

o c. Se encuentra a 35 grados de temperatura.

o d. Por estar a 26 grados de temperatura en el ambiente

La respuesta correcta es: Por estar a 26 grados de temperatura en el ambiente

Pregunta 12 Incorrecta Puntúa 0,00 sobre 0,16

El hallazgo más frecuente en la radiografía de tórax en un paciente con asma es:

Seleccione una:

o a. Radiografía de tórax normal.

o b. Engrosamiento de paredes bronquiales.

@ c. Hiperinsu ación pulmonar. X


o d. Condensaciones alveolares bilaterales y difusas.

La respuesta correcta es: Radiografía de tórax normal.

Pregunta 13 Correcta Puntúa 0,16 sobre 0,16

El tratamiento de las Bronquiectasias se basa en 3 pilares, excepto:

Seleccione una:

o a. Mejorar la eliminación de las secreciones, que se consigue con una adecuada hidratación, con
sioterapia respiratoria y drenaje postural mantenidos.

o b. Eliminar la obstrucción bronquial.

o c. Controlar las infecciones con el uso de antibióticos en las agudizaciones durante 10-15 días.

@ d. Revertir el remodelamiento bronquia

La respuesta correcta es: Revertir el remodelamiento bronquia 


Pregunta 14 Correcta Puntúa 0,16 sobre 0,16

En cuanto al diagnóstico funcional del asma:

Seleccione una:

o a. Si la relación VEF1/CVF es menor a 0.7 (patrón restrictivo) y post broncodilatador obtenemos un aumento
igual o mayor al 12% en el VEF1, nos orienta hacia diagnóstico de asma

o b. Si la relación VEF1/CVF es mayor 0.7 se considera patrón obstructivo, diagnosticamos asma

@ c. Si la relación VEF1/CVF es menor a 0.7 (patrón obstructivo) y post broncodilatador obtenemos un


aumento igual o mayor al 12% en el VEF1, nos orienta hacia diagnóstico de asma

o d. Valores espirométricos no tienen importancia en el diagnóstico del asma.

La respuesta correcta es: Si la relación VEF1/CVF es menor a 0.7 (patrón obstructivo) y post broncodilatador
obtenemos un aumento igual o mayor al 12% en el VEF1, nos orienta hacia diagnóstico de asma

Pregunta 15 Correcta Puntúa 0,16 sobre 0,16

En que valores se encuentra una hiponatremia leve?

Seleccione una:

@ a. 125 a 135 mEq/l

o b. 115 a 125 mEq/l

o c. 105 a 115 mEq/l

o d. 95 a 105 mEq/l

La respuesta correcta es: 125 a 135 mEq/l


Pregunta 16 Incorrecta Puntúa 0,00 sobre 0,16

Este paciente con quemaduras le van a cumplir posteriormente plasma, el paciente es grupo O (-) Banco de
sangre envía un plasma. ¿Cuál de estos podemos cumplir?

Seleccione una:

o a. Plasma B +

o b. Plasma A B -

@ c. Plasma O + X
o d. Plasma A +

La respuesta correcta es: Plasma A B -

Pregunta 17 Correcta Puntúa 0,16 sobre 0,16

La anemia hemolítica microangiopática es un cuadro clínico, que se presenta en patologías como la


coagulación intravascular diseminada, la presencia de esquitocitos en el frotis periférico es una imagen
característica. ¿Qué representa?

Seleccione una:

o a. Buena respuesta de la médula

o b. Fibrina fragmentada

@ c. Hematíes fragmentados

o d. Reticulocitos

La respuesta correcta es: Hematíes fragmentados


Pregunta 18 Correcta Puntúa 0,16 sobre 0,16

La causa principal de un fracaso en el tratamiento de la tuberculosis pulmonar es

Seleccione una:

o a. Resistencia secundaria a las drogas

o b. Toxicidad hepática

o c. Resistencia primaria a las drogas

@ d. Abandono del tratamiento por el paciente

La respuesta correcta es: Abandono del tratamiento por el paciente

Pregunta 19 Correcta Puntúa 0,16 sobre 0,16

La furosemida ¿qué categoría de riesgo en gestantes tiene?

Seleccione una:

o a. A

o b. B

@ c. C

o d. D

La respuesta correcta es: C

Pregunta 20 Incorrecta Puntúa 0,00 sobre 0,16

La obstrucción ureteral bilateral crónica se caracteriza por?

Seleccione una:

o a. Dolor renal

o b. Hipertensión

@ c. Anuria X
o d. Diuresis posterior a la obstrucción


La respuesta correcta es: Hipertensión
Pregunta 21 Incorrecta Puntúa 0,00 sobre 0,16

Qué enfermedad reumatológica causa insu ciencia aórtica:

Seleccione una:

@ a. Artritis reumatoidea X
o b. Espondilitis anquilosante

o c. Vasculitis leucocitoclástica

o d. Lupus eritematoso sistémico

La respuesta correcta es: Espondilitis anquilosante

Pregunta 22 Correcta Puntúa 0,16 sobre 0,16

Si el cálculo se ubica en la porción inferior del uréter, el dolor puede irradiarse en mujeres hacia?

Seleccione una:

o a. Útero

o b. Pubis

o c. Miometrio

@ d. Labio mayor ipsilateral

La respuesta correcta es: Labio mayor ipsilateral

Pregunta 23 Correcta Puntúa 0,16 sobre 0,16

Una reacción transfusional es una respuesta adversa a la transfusión de un derivado sanguíneo. ¿Cuál es la
reacción más frecuente?

Seleccione una:

@ a. Reacción febril no hemolítica

o b. Reacción hemolítica crónica

o c. Urticaria

o d. Reacción hemolítica aguda

La respuesta correcta es: Reacción febril no hemolítica 


Pregunta 24 Correcta Puntúa 0,16 sobre 0,16

Un paciente con quemaduras de un 30% de su cuerpo, presenta además de sus lesiones un edema
importante. ¿Cuál de estos derivados pedirías para ayudar a resolver el edema?

Seleccione una:

(!) a. Albumina

o b. Inmunoglobulina

o c. Concentrado plaquetario

o d. Concentrado globular

La respuesta correcta es: Albumina

Pregunta 25 Correcta Puntúa 0,16 sobre 0,16

Un paciente hipertenso con descompensación aguda “típica” de insu ciencia cardiaca sin sobrecarga de
volumen, que fármaco usaría:

Seleccione una:

o a. Furosemida

(!) b. Nitroprusiato

o c. Dobutamina

o d. Clortalidona

La respuesta correcta es: Nitroprusiato

◄ Avisos Ir a... V Link sesión zoom examen primer parcial ►


ADRIAN GABRIEL JARAMILLO PEREZ

Área personal  Mis cursos  Quito  CIENCIAS DE LA SALUD  MEDICINA - PRESENCIAL  ABR 2020 - AGO 2020
 INVESTIGACION I-RESPI-TEORIA - Prl: MD NVB Pen: 961  1 de junio - 7 de junio  EXAMEN

Comenzado el viernes, 5 de junio de 2020, 16:45


Estado Finalizado
Finalizado en viernes, 5 de junio de 2020, 16:56
Tiempo empleado 11 minutos 14 segundos

Pregunta 1 Finalizado Puntúa como 1,00

La maniobra de Hommans positiva, me indica:

Seleccione una:

o a. Paquipleuritis

o b. TromboEmbolia Pulmonar

o c. Derrame Pleural

@ d. Trombosis Venosa Profunda

Pregunta 2 Finalizado Puntúa como 1,00

Respecto a la siopatología cardiaca en la TEP cuál de las siguientes opciones es verdadera:

Seleccione una:

@ a. La obstrucción en la arteria pulmonar aumenta la poscarga del VD

o b. El VI se dilata y aumenta la tensión en su pared.

o c. La estenosis valvular aórtica genera una obstrucción a nivel de las arterias pulmonares

o d. Disminución de la resistencia vascular periférica debido a la obstrucción vascular y a los agentes


neuro-hormonales.


/
Pregunta 3 Finalizado Puntúa como 1,00

Cuál es la de nición de bronquiectasias?

Seleccione una:

o a. Dilatación anormal y reversible de los bronquios

o b. Constricción anormal e irreversible de los bronquios

o c. Constricción anormal y reversibles de los bronquios

@ d. Dilatación anormal e irreversible de los bronquios

Pregunta 4 Finalizado Puntúa como 1,00

Que de lo siquiente puede producir Hemoptisis

Seleccione una:

@ a. Edema agudo de Pulmòn

o b. Fibrosis Pulmonar

o c. Estenosis Mitral

o d. Vàrices esofágicas

Pregunta 5 Finalizado Puntúa como 1,00

Cual es el principal medicamento controlador para el Asma

Seleccione una:

o a. Beta 2 agonista de acción rápida

@ b. Glucocorticoide Inhalado

o c. Glucocorticoide sistémico

o d. Anticolinérgico


/
Pregunta 6 Finalizado Puntúa como 1,00

En que nivel del Asma el FEV1 esta entre el 80 y el 60%

Seleccione una:

o a. Persistente leve

o b. Intermitente

o c. Persistente severa

@ d. Persistente moderada

Pregunta 7 Finalizado Puntúa como 1,00

La Hemoptisis en un paciente con Tuberculosis se presenta en que porcentaje

Seleccione una:

o a. 14%

o b. 34%

o c. 44%

@ d. 24%

Pregunta 8 Finalizado Puntúa como 1,00

Con cual de los siguientes criterios usted puede diferenciar un paciente infectado de un paciente
enfermo de Tuberculosis.

Seleccione una:

@ a. Esputo y cultivo positivo

o b. El examen físico es negativo

o c. Mantoux (PPD) usualmente positivo

o d. Rx usualmente normal


/
Pregunta 9 Finalizado Puntúa como 1,00

¿Cuál de los siguientes signos radiográ cos no corresponde a un hallazgo de neumotórax?

Seleccione una:

o a. Desplazamiento de las estructuras del mediastino hacia el lado contrario del afectado.

o b. Aumento de la radiolucidez en el área de acumulación del aire.

o c. Colapso pulmonar parcial o completo del pulmón del lado afectado.

(!) d. Joroba de Hampton.

Pregunta 10 Finalizado Puntúa como 1,00

La acumulación de líquido en el espacio pleural, según Sahn se debe a:

Seleccione una:

o a. Aumento presión oncótica

o b. Disminución de la presión negativa del espacio pleural

o c. Disminución presión hidrostática

(!) d. Aumento de permeabilidad capilar

◄ FIBROSIS PULMONAR Ir a... V


/
PRIMER PARCIAL

Pregunta 1

Enunciado de la pregunta

Carlos paciente de 50 años, fumador, con obesidad, que presenta desde hace 2 semanas
polidipsia polifagia, poliurea, pérdida de peso, que examen como parte de los criterios
de la ADA nos daría el diagnóstico de diabetes
Seleccione una:
J
a. Glucosa en ayunas mayor de 100 mg/dl
r:il
b. Glucosa mayor de 200 tomada al azar.
J
c. Glucosa postprandial mayor de 150 mg/dl
J
d. Glucosa mayor a 126 mg/dl durante prueba oral de tolerancia a la glucosa

Retroalimentación

La respuesta correcta es: Glucosa mayor de 200 tomada al azar.

Pregunta 2

Enunciado de la pregunta

Una de las características principales por las cuales el dengue puede diseminarse es:
Seleccione una:
J
a. Es sumamente frecuente la progresión de daño encefálico.
J
b. Una característica precoz es la trombocitosis y neutrofilia.
J
c. Debido a la gran capacidad de diseminación del vector Anopheles.
r:il
d. El vector es muy cercano a los asentamientos humanos.

Retroalimentación

La respuesta correcta es: El vector es muy cercano a los asentamientos humanos.

Pregunta 3

Enunciado de la pregunta

Paciente masculino de 66 años de edad, con antecedentes de deterioro cognitivo leve,


HTA, ingresa para una cirugía electiva de resección prostática por una hipertrofia
prostática benigna, en su postoperatorio se encuentra con sondaje (cateterización),
vesical permanente, dolor pélvico, y un cuadro de delirium (o sd confusional agudo), se
reinició el enalapril en el postoperatorio, de las siguientes opciones señale cuál NO es
un factor de riesgo para el aparecimiento de delirium
Seleccione una:
J
a. Dolor en el postoperatorio
J
b. Sondaje vesical
00
c. Administración de enalapril
J
d. Antecedentes de deterioro cognitivo leve

Retroalimentación

La respuesta correcta es: Administración de enalapril

Pregunta 4

Enunciado de la pregunta

Paciente de 38 años de género femenino, con APP diagnóstico de fenómeno de


Raynaud en estudio de un posible Lupus eritematoso sistémico, también diagnóstico de
migraña con aura desde la juventud, acude por un ataque agudo de cefalea, usted qué
fármaco NO recomendaría:
Seleccione una:
J
a. Clorpromazina
J
b. AINES
00
c. Sumatriptán
J
d. Aspirina + metoclopramida

Retroalimentación

La respuesta correcta es: Sumatriptán

Pregunta 5

Enunciado de la pregunta

Hombre de 54 años que acude a chequeo general. Se detecta un índice de masa corporal
de 32,8 kg/m2 y glucemia en ayunas 138 mg/l. Un mes después, glucemia 123 mg/dl.
¿Qué recomendación terapéutica efectuaría en primer lugar?
Seleccione una:
J
a. Prescribir una sulfonilurea.
J
b. Administrar metformina.
00
c. Cambios conductuales, dieta y ejercicio físico.
J
d. Insulina antes de cada comida.

Retroalimentación

La respuesta correcta es: Cambios conductuales, dieta y ejercicio físico.

Pregunta 6
Enunciado de la pregunta

Paciente de 16 años de género masculino, sin APP, empieza con crisis convulsivas
tónico clónicas generalizadas hace 1 año, 1 crisis cada 2 meses, su examen neurológico
es normal al igual que la Resonancia Magnética cerebral, el electroencefalograma
demostró actividad epileptiforme, cuál de las siguientes etiologías consideraría en el
diagnóstico
Seleccione una:
J
a. Epilepsia por enfermedad degenerativa
00
b. Epilepsia por desorden genético
J
c. Epilepsia secundaria a disturbios hidroelectrolíticos
J
d. Epilepsia secundaria a autoanticuerpos

Retroalimentación

La respuesta correcta es: Epilepsia por desorden genético

Pregunta 7

Enunciado de la pregunta

Mujer de 68 años de edad en estado de coma. Al examen: PA: 90/50 mmHg; Urea: 56
mg%, Creatinina: 2,5 mg%; Glucosa: 1150 mg%; Na: 130 mEq/L; Bicarbonato: 23
mEq/L; pH: 7,32. Volumen urinario de 24 horas: 450 mL. El diagnóstico MÁS probable
es:
Seleccione una:
J
a. Coma por hiponatremia.
J
b. Coma por cetoacidosis diabética.
J
c. Coma urémico.
00
d. Coma hiperosmolar no cetosico.

Retroalimentación

La respuesta correcta es: Coma hiperosmolar no cetosico.

Pregunta 8

Enunciado de la pregunta

¿De qué depende la inmunidad contra Leptospira?


Seleccione una:
J
a. Producción de anticuerpos contra LPS inespecíficos.
00
b. Producción de anticuerpos circulantes contra LPS específicos de un serotipo.
J
c. Producción de anticuerpos contra proteínas inespecíficas de un serotipo.
J
d. Producción de anticuerpos contra los distintos serotipos.
Retroalimentación

La respuesta correcta es: Producción de anticuerpos circulantes contra LPS específicos


de un serotipo.

Pregunta 9

Enunciado de la pregunta

El mecanismo por el cual la infección por cólera produce una diarrea tan intensa es por:
Seleccione una:
r a. Vibrios enteroagregativos.
~ b. Toxina enterogénica
r c. Toxina Shiga-like
r d. Vibrios entero hemorrágicos.

Retroalimentación

La respuesta correcta es: Toxina enterogénica

Pregunta 10

Enunciado de la pregunta

Un paciente con una glucemia de 300mg/dl puede referir los siguientes signos y
síntomas: señale lo correcto:
Seleccione una:
a. Cataratas, cetosis, uremia
b. Polidipsia, astenia, calambres musculares
r c. Amputación de miembro inferior, HTA, IVU
r d. Rinorrea, glucosuria, Ceguera

Retroalimentación

La respuesta correcta es: Polidipsia, astenia, calambres musculares

Pregunta 11

Enunciado de la pregunta

En un paciente con diabetes mellitus tipo 2, índice de masa corporal mayor de 30 cuál
sería el antidiabético oral más indicado:
Seleccione una:
a. Metformina
r b. Tiazolidinedionas
J
c. Ninguna de las anteriores
J
d. Glibenclamida

Retroalimentación

La respuesta correcta es: Metformina

Pregunta 12

Enunciado de la pregunta

La ictericia en un paciente con Malaria es frecuente a partir:


Seleccione una:
J
a. La ictericia no aparece en la malaria.
J
b. Ocasionalmente en niños debido a su alta susceptibilidad
J
c. Siempre existirá fiebre e ictericia debido a la destrucción de eritrocitos.
r:il
d. La Ictericia en Malaria es frecuente a partir de los 15 años.

Retroalimentación

La respuesta correcta es: La Ictericia en Malaria es frecuente a partir de los 15 años.

Pregunta 13

Enunciado de la pregunta

En pacientes con catatonía, usted sospecharía:


Seleccione una:
J
a. Lesión de la región orbitaria frontal
J
b. Secuela del coma
J
c. Herniación central
r:il
d. Trastorno psiquiátrico

Retroalimentación

La respuesta correcta es: Trastorno psiquiátrico

Pregunta 14

Enunciado de la pregunta

Tras la infección de un niño con el virus de la Influenza con la siguientes características:


fiebre intensa de inicio súbito, disnea y cianosis a la Rx de tórax se aprecia patrón
asociado con infiltrados intersticiales difusos e hipoxia intensa, usted sospecharía de:
Seleccione una:
a. Neumonía bacteriana secundaria.
b. Neumonía viral primaría.
c. Neumonía bacteriana primaria.
d. Neumonía viral secundaria.

Retroalimentación

La respuesta correcta es: Neumonía viral primaría.

Pregunta 15

Enunciado de la pregunta

En cuanto a la profilaxis antitetánica en el tratamiento sistemático de las heridas, en una


herida pequeña limpia sin ningún antecedente de vacunación antitetánica Usted
recomendaría:
Seleccione una:
a. Es indicativo de inmunoglobulina.
b. Solo limpiar la herida y dar indicaciones de cuidado al paciente, ofrecer la anti
toxina.
r c. Dar antibiótico de manera profiláctica.
r d. Vacunación antitetánica de inmediato.

Retroalimentación

La respuesta correcta es: Solo limpiar la herida y dar indicaciones de cuidado al


paciente, ofrecer la anti toxina.

Pregunta 16

Enunciado de la pregunta

Carlos paciente de 18 años sin antecedentes de importancia acude a la sala de


Emergencia con aliento de manzana, somnoliento, se le realiza una glicemia y se
encuentra glucosa de 300 mg / dl, se le realiza una gasometría y se encuentra un PH de
7 . Bicarbonato de 9, presencia de cetonas en orina ++, cuál sería su apreciación
diagnostica
Seleccione una:
a. Estado hiperosmolar con debut diabético
b. Hiperglucemia sostenida con debut diabético
c. Cetoacidosis diabética con debut diabético
r d. Descompensación simple

Retroalimentación
La respuesta correcta es: Cetoacidosis diabética con debut diabético

Pregunta 17

Enunciado de la pregunta

Masculino de 52 años consulta por poliuria, polidipsia intensa y pérdida involuntaria de


10 kg de peso; es diagnosticado de diabetes mellitus por una glucemia plasmática de
322 mg/dL y una HbA1c de 9,8%. Se le recomienda dieta, ejercicio físico, e iniciar
tratamiento con metformina 850 mg cada 12 horas y glimepirida 6 mg/dia. En las
semanas siguientes los controles glucémicos se van reduciendo progresivamente. A los
4 meses la glucemia es de 94 mg/dL y la HbA1c de 5,9%. El paciente se queja de
episodios frecuentes de pérdida del equilibrio “mareo”, dolor epigástrico, visión
borrosa, sudoración y temblor, que mejoran con la ingesta de alimentos y que ocurren
sobre todo al final de la mañana y al final de la tarde. ¿Qué modificación propondría en
su tratamiento?
Seleccione una:
r a. Suspender la metformina.
r b. Sustituir la metformina por un inhibidor de la DPP4
~ c. Suspender la sulfonilurea
r d. Revisar la distribución de hidratos de carbono de su dieta.

Retroalimentación

La respuesta correcta es: Suspender la sulfonilurea

Pregunta 18

Enunciado de la pregunta

Un hombre de 62 años con una diabetes mellitus tipo 2, de 10 años de evolución realiza
tratamiento con metformina y sitagliptina. Hace ejercicio físico escaso y realiza una
dieta adecuada. En los últimos 6 meses ha perdido peso y tiene más astenia. Sus
controles glucémicos se han deteriorado pasando de glucemias basales de 110-140
mg/dl a glucemias de 170-200 mg/dl, así como su hemoglobina glicosilada que ha
pasado de 7,1 a 8,5%. La medida terapéutica más adecuada a realizar es, señale la
alternativa correcta:
Seleccione una:
a. Sustituir metformina por glimepirida
b. Asociar al tratamiento ascarbosa.
c. Sustituir sitagliptina por pioglitazona
d. Asociar al tratamiento una dosis de insulina basal.

Retroalimentación

La respuesta correcta es: Asociar al tratamiento una dosis de insulina basal.


Pregunta 19

Enunciado de la pregunta

El siguiente enunciado: Un estado parecido al sueño profundo en el que el paciente


permanece con los ojos cerrados y el paciente no puede ser despertado, se refiere a:
Seleccione una:
00
a. Estado de coma
J
b. Estupor
J
c. Somnolencia
J
d. Estado vegetativo

Retroalimentación

La respuesta correcta es: Estado de coma

Pregunta 20

Enunciado de la pregunta

Dentro de las manifestaciones tardías de la sífilis tenemos trastornos cardiovasculares


de cual podemos desprender el siguiente:
Seleccione una:
J
a. Insuficiencia cardiaca congestiva.
00
b. Aneurismas
J
c. Trastornos del endotelio
J
d. Flebitis irritativa.

Retroalimentación

La respuesta correcta es: Aneurismas

Pregunta 21

Enunciado de la pregunta

Niña de 11 años es hospitalizada por presentar nerviosismo y palpitaciones de 24 horas


de evolución. El apetito ha sido bueno pero registra pérdida de peso. Al examen físico:
FC 100lpm, temperatura 37.5°C, transpiración excesiva y ROT rápidos. Los ojos
presentaban una apariencia vidriosa. El diagnóstico MÁS probable es:
Seleccione una:
J
a. Disautonomía familiar.
J
b. Tirotoxicosis juvenil.
00
c. Diabetes mellitus tipo 1.
J
d. Psicosis juvenil.
Retroalimentación

La respuesta correcta es: Diabetes mellitus tipo 1.

Pregunta 22

Enunciado de la pregunta

En el manejo de un paciente con delirium agitado (o hiperactivo), de las siguientes


opciones, señale la que NO incluiría en su prescripción médica:
Seleccione una:
r a. Manejo no farmacológico como: colocar un reloj visible para el paciente
r b. Evitar uso de restricciones físicas (amarras) para controlar el movimiento del
paciente
r c. Antipsicóticos atípicos como la quetiapina en vía oral
~ d. Benzodiacepina como el clonazepam en vía oral

Retroalimentación

La respuesta correcta es: Benzodiacepina como el clonazepam en vía oral

Pregunta 23

Enunciado de la pregunta

En un paciente de 68 años de edad, con APP: HTA, que ingresa con un cuadro de crisis
focales motoras sin pérdida del estado de conciencia y limitadas al miembro superior
derecho, que se han repetido 1 crisis cada 3 días desde hace 2 semanas, además se
acompaña de cefalea desde hace 3 meses con características de empeoramiento
progresivo, en los estudios de imagen se encontró una lesión expansiva frontal
izquierda; en relación al tratamiento farmacológico de elección, señale el que escogería:
Seleccione una:
a. Clonazepam
b. Lamotrigina
c. Acido valproico
d. Fenobarbital

Retroalimentación

La respuesta correcta es: Lamotrigina

Pregunta 24

Enunciado de la pregunta
Paciente de 22 años, género femenino, tiene desde hace 1 año dolor de cabeza de forma
episódica (2 episodios al mes), hemicránea derecha o izquierda, pulsátil, intensidad
moderada, se acompaña de náusea y fotofobia, el dolor es precedido por escotomas
centellantes que duran 90 minutos aproximadamente, en los últimos 3 meses el dolor se
ha tornado más frecuente (3 episodios semanales) y al toser exacerba en intensidad a
dolor severo, de los siguientes enunciados cuál NO considera signo/síntoma de alarma:
Seleccione una:
r a. Aura de duración larga
~ b. Localización del dolor
r c. Empeoramiento del dolor (más frecuente)
r d. Valsalva positivo

Retroalimentación

La respuesta correcta es: Localización del dolor

Pregunta 25

Enunciado de la pregunta

En la clasificación de OMS de adultos con VIH – Sida en el estadio C3 tenemos a:


Seleccione una:
a. Mayor de 500 células CD4+ Asintomático.
b. Menor de 300 células CD4+ con síntomas definidores de Sida.
c. Menor de 200 células CD4+ con síntomas definidores de Sida.
r d. Mayor de 400 células CD4+ con síntomas no definidores de Sida.

Retroalimentación

La respuesta correcta es: Menor de 200 células CD4+ con síntomas definidores de Sida.
Pregunta 1 Correcta Puntúa 1,00 sobre 1,00
Cuánto es la dosis sublingual de nitroglicerina en angina de pecho:
Seleccione una:
a. 1.3 a 1.6 mg
b. 0.8 a 1.0 mg
c. 3 a 6 mg
d. 0.3 a 0.6 mg
La respuesta correcta es: 0.3 a 0.6 mg

Pregunta 2 Incorrecta Puntúa 0,00 sobre 1,00


Mujer de 68 años de edad en estado de coma. Al examen: PA: 90/50 mmHg; Urea: 56 mg%,
Creatinina: 2,5 mg%; Glucosa: 1150 mg%; Na: 130 mEq/L; Bicarbonato: 23 mEq/L; pH: 7,32.
Volumen urinario de 24 horas: 450 mL. El diagnóstico MÁS probable es:
Seleccione una:
a. Coma por cetoacidosis diabética.
b. Coma por hiponatremia.
c. Coma hiperosmolar no cetosico.
d. Coma urémico.
La respuesta correcta es: Coma hiperosmolar no cetosico.

Pregunta 3 Correcta Puntúa 1,00 sobre 1,00


Cuánto es la dosis máxima de dinitrato de isosorbide de liberación prolongada en angina de pecho:
Seleccione una:

-
a. 40 mg
b. 300 mg
c. 120 mg
d. 160 mg
La respuesta correcta es: 300 mg

Pregunta 4 Correcta Puntúa 1,00 sobre 1,00


Un paciente con una glucemia de 300mg/dl puede referir los siguientes signos y síntomas: señale lo
correcto: Seleccione una:
a. Rinorrea, glucosuria, Ceguera
b. Cataratas, cetosis, uremia
c. Amputación de miembro inferior, HTA, IVU
d. Polidipsia, astenia, calambres musculares
La respuesta correcta es: Polidipsia, astenia, calambres musculares

Pregunta 5 Correcta Puntúa 1,00 sobre 1,00


El mecanismo por el cual la infección por cólera produce una diarrea tan intensa es por:
Seleccione una:
a. Toxina Shiga-like
b. Toxina enterogénica
c. Vibrios enteroagregativos.
d. Vibrios entero hemorrágicos.
La respuesta correcta es: Toxina enterogénica

Pregunta 6 Incorrecta Puntúa 0,00 sobre 1,00


Niña de 11 años es hospitalizada por presentar nerviosismo y palpitaciones de 24 horas de
evolución. El apetito ha sido bueno pero registra pérdida de peso. Al examen físico: FC 100lpm,
temperatura 37.5°C, transpiración excesiva y ROT rápidos. Los ojos presentaban una apariencia
vidriosa. El diagnóstico MÁS probable es:
Seleccione una:
a. Diabetes mellitus tipo 1.
b. Disautonomía familiar.
c. Psicosis juvenil.
d. Tirotoxicosis juvenil.
La respuesta correcta es: Diabetes mellitus tipo 1.

Pregunta 7 Correcta Puntúa 1,00 sobre 1,00


Carlos paciente de 18 años sin antecedentes de importancia acude a la sala de Emergencia con
aliento de manzana, somnoliento, se le realiza una glicemia y se encuentra glucosa de 300 mg / dl,
se le realiza una gasometría y se encuentra un PH de 7 . Bicarbonato de 9, presencia de cetonas en
orina ++, cuál sería su apreciación diagnostica
Seleccione una:
a. Hiperglucemia sostenida con debut diabético
b. Descompensación simple
c. Estado hiperosmolar con debut diabético
d. Cetoacidosis diabética con debut diabético
La respuesta correcta es: Cetoacidosis diabética con debut diabético

Pregunta 8 Correcta Puntúa 1,00 sobre 1,00


¿De qué depende la inmunidad contra Leptospira?
Seleccione una:
a. Producción de anticuerpos circulantes contra LPS específicos de un serotipo.
b. Producción de anticuerpos contra proteínas inespecíficas de un serotipo.
c. Producción de anticuerpos contra LPS inespecíficos.
d. Producción de anticuerpos contra los distintos serotipos.
La respuesta correcta es: Producción de anticuerpos circulantes contra LPS específicos de un serotipo.

Pregunta 9 Correcta Puntúa 1,00 sobre 1,00


Una de las características principales por las cuales el dengue puede diseminarse es:
Seleccione una:
a. El vector es muy cercano a los asentamientos humanos.
b. Una característica precoz es la trombocitosis y neutro􀃕lia.
c. Es sumamente frecuente la progresión de daño encefálico.
d. Debido a la gran capacidad de diseminación del vector Anopheles.
La respuesta correcta es: El vector es muy cercano a los asentamientos humanos.

Pregunta 10 Correcta Puntúa 1,00 sobre 1,00


Un hombre de 62 años con una diabetes mellitus tipo 2, de 10 años de evolución realiza
tratamiento con metformina y sitagliptina. Hace ejercicio físico escaso y realiza una dieta adecuada.
En los últimos 6 meses ha perdido peso y tiene más astenia. Sus controles glucémicos se han
deteriorado pasando de glucemias basales de 110-140 mg/dl a glucemias de 170-200 mg/dl, así
como su hemoglobina glicosilada que ha pasado de 7,1 a 8,5%. La medida terapéutica más
adecuada a realizar es, señale la alternativa correcta:
Seleccione una:
a. Sustituir metformina por glimepirida
b. Sustituir sitagliptina por pioglitazona
c. Asociar al tratamiento ascarbosa.
d. Asociar al tratamiento una dosis de insulina basal.
La respuesta correcta es: Asociar al tratamiento una dosis de insulina basal.

Pregunta 11 Correcta Puntúa 1,00 sobre 1,00


Dentro de las manifestaciones tardías de la sí􀃕lis tenemos trastornos cardiovasculares de cual
podemos desprender el siguiente:

-
Seleccione una:
a. Aneurismas
b. Trastornos del endotelio
c. Insu􀃕ciencia cardiaca congestiva.
d. Flebitis irritativa.
La respuesta correcta es: Aneurismas

Pregunta 12 Correcta Puntúa 1,00 sobre 1,00


Cuál síndrome genético es común en la insuficiencia aórtica:
Seleccione una:
a. Síndrome de Marfán
b. Anomalía de Ebstein
c. Síndrome de Eisenmenger
d. Síndrome de Rastelli
La respuesta correcta es: Síndrome de Marfán

Pregunta 13 Correcta Puntúa 1,00 sobre 1,00


Qué infección causa bloqueo auriculoventricular:
Seleccione una:
a. Varicela

-
b. Tripanosomiasis
c. Sífilis
d. Sarampión
La respuesta correcta es: Sífilis

Pregunta 14 Correcta Puntúa 1,00 sobre 1,00


La ictericia en un paciente con Malaria es frecuente a partir:
Seleccione una:
a. Ocasionalmente en niños debido a su alta susceptibilidad
b. La ictericia no aparece en la malaria.
c. La Ictericia en Malaria es frecuente a partir de los 15 años.
d. Siempre existirá fiebre e ictericia debido a la destrucción de eritrocitos.
La respuesta correcta es: La Ictericia en Malaria es frecuente a partir de los 15 años.

Pregunta 15 Correcta Puntúa 1,00 sobre 1,00


Masculino de 52 años consulta por poliuria, polidipsia intensa y pérdida involuntaria de 10 kg de
peso; es diagnosticado de diabetes mellitus por una glucemia plasmática de 322 mg/dL y una
HbA1c de 9,8%. Se le recomienda dieta, ejercicio físico, e iniciar tratamiento con metformina 850
mg cada 12 horas y glimepirida 6 mg/dia. En las semanas siguientes los controles glucémicos se van
reduciendo progresivamente. A los 4 meses la glucemia es de 94 mg/dL y la HbA1c de 5,9%. El
paciente se queja de episodios frecuentes de pérdida del equilibrio “mareo”, dolor epigástrico,
visión borrosa, sudoración y temblor, que mejoran con la ingesta de alimentos y que ocurren sobre
todo al final de la mañana y al final de la tarde. ¿Qué modificación propondría en su tratamiento?
Seleccione una:
a. Sustituir la metformina por un inhibidor de la DPP4
b. Suspender la metformina.
c. Revisar la distribución de hidratos de carbono de su dieta.
d. Suspender la sulfonilurea
La respuesta correcta es: Suspender la sulfonilurea
Pregunta 16 Correcta Puntúa 1,00 sobre 1,00
Cuánto es la frecuencia de administración de propanolol en cardiopatía isquémica:
Seleccione una:
a. Cada 8 horas
b. Cada 6 horas
c. Cada 24 horas
d. Cada 12 horas
La respuesta correcta es: Cada 12 horas

Pregunta 17 Correcta Puntúa 1,00 sobre 1,00


Hombre de 54 años que acude a chequeo general. Se detecta un índice de masa corporal de 32,8
kg/m2 y glucemia en ayunas 138 mg/l. Un mes después, glucemia 123 mg/dl. ¿Qué recomendación
terapéutica efectuaría en primer lugar?
Seleccione una:
a. Administrar metformina.
b. Prescribir una sulfonilurea.
c. Cambios conductuales, dieta y ejercicio físico.
d. Insulina antes de cada comida.
La respuesta correcta es: Cambios conductuales, dieta y ejercicio físico.

Pregunta 18 Correcta Puntúa 1,00 sobre 1,00


Tras la infección de un niño con el virus de la In􀃕uenza con la siguientes características: 􀃕ebre
intensa de inicio súbito, disnea y cianosis a la Rx de tórax se aprecia patrón asociado con in􀃕ltrados
intersticiales difusos e hipoxia intensa, usted sospecharía de:
Seleccione una:
a. Neumonía bacteriana secundaria.
b. Neumonía viral secundaria.
c. Neumonía viral primaría.
d. Neumonía bacteriana primaria.
La respuesta correcta es: Neumonía viral primaría.
Pregunta 19 Correcta Puntúa 1,00 sobre 1,00
Cuál es una causa de taquicardia sinusal fisiológica:
Seleccione una:
a. Hipotiroidismo
b. Hipertensión pulmonar
c. Sedenterismo
d. Feocromocitoma
La respuesta correcta es: Feocromocitoma

Pregunta 20 Correcta Puntúa 1,00 sobre 1,00


En cuanto a la profilaxis antitetánica en el tratamiento sistemático de las heridas, en una herida
pequeña limpia sin ningún antecedente de vacunación antitetánica Usted recomendaría:
Seleccione una:
a. Vacunación antitetánica de inmediato.
b. Dar antibiótico de manera pro􀃕láctica.
c. Solo limpiar la herida y dar indicaciones de cuidado al paciente, ofrecer la anti toxina.
d. Es indicativo de inmunoglobulina.
La respuesta correcta es: Solo limpiar la herida y dar indicaciones de cuidado al paciente, ofrecer la anti
toxina.

Pregunta 21 Correcta Puntúa 1,00 sobre 1,00


Cuánto es la frecuencia de administración de atenolol en cardiopatía isquémica:
Seleccione una:
a. Cada 12 horas
b. Cada 6 horas
c. Cada 8 horas
d. Cada 24 horas
La respuesta correcta es: Cada 24 horas

Pregunta 22 Correcta Puntúa 1,00 sobre 1,00


En un paciente con diabetes mellitus tipo 2, índice de masa corporal mayor de 30 cuál sería el
antidiabético oral más indicado:
Seleccione una:
a. Metformina
b. Tiazolidinedionas
c. Glibenclamida
d. Ninguna de las anteriores
La respuesta correcta es: Metformina

Pregunta 23 Correcta Puntúa 1,00 sobre 1,00


Un paciente hipertenso con descompensación aguda “típica” de insu􀃕ciencia cardiaca sin
sobrecarga de volumen, que fármaco usaría:
Seleccione una:
a. Clortalidona
b. Dobutamina
c. Furosemida
d. Nitroprusiato
La respuesta correcta es: Nitroprusiato

Pregunta 24 Correcta Puntúa 1,00 sobre 1,00


Qué enfermedad reumatológica causa insu􀃕ciencia aórtica:
Seleccione una:
a. Espondilitis anquilosante
b. Vasculitis leucocitoclástica
c. Lupus eritematoso sistémico
d. Artritis reumatoidea
La respuesta correcta es: Espondilitis anquilosante

Pregunta 25 Incorrecta Puntúa 0,00 sobre 1,00


Paciente diabético que acude a Emergencia obnubilado con los siguientes resultados de
laboratorio: glucemia 340 mg/dL, pH 7.20 mg/dL, Na+ 136rnEq/L, K+ 4,5 mEq/L, y Bicarbonato
estándar 11,2mMol/L, con cetonuria. En relación con el tratamiento y la evolución ¿cuál de los
siguientes enunciados NO es correcto?:
Seleccione una:
a. Proporcionaremos aporte de suero glucosado dextrosa a 5% cuando la glucemia sea igual o
inferior a 250 mg/dL.
b. La infusión endovenosa con insulina debe iniciarse dos horas después de la fluidoterapia de
resucitación con solución salina.
c. Puede observarse respiración de Kussmaul.
d. Son frecuentes el dolor abdominal, náuseas y vómitos; también puede alterarse el estado de
conciencia.
La respuesta correcta es: La infusión endovenosa con insulina debe iniciarse dos horas después de la
fluidoterapia de resucitación con solución salina.
2
AYLEN MARILYN VERGARA TITO

Área personal  Mis cursos  Quito  CIENCIAS DE LA SALUD  MEDICINA - PRESENCIAL  ABR 2020 - AGO 2020
 INVESTIGACIÓN I - Prl: MD NVD Pen: 961  13 de junio: Examen Primer Parcial Investigación I  Examen Primer Parcial
Investigación I

Comenzado el sábado, 13 de junio de 2020, 10:10


Estado Finalizado
Finalizado en sábado, 13 de junio de 2020, 10:55
Tiempo empleado 44 minutos 48 segundos
Cali cación 2,88 de 4,00 (72%)

Pregunta 1 Correcta Puntúa 0,16 sobre 0,16

Cuál de los siguientes es un signo característico de las vías respiratorias de las personas que sufren
asma:

Seleccione una:

o a. Adhesión de los glóbulos rojos al endotelio

o b. In ltración basó la

o c. Transformación de los basó los en macrófagos

d. In ltración eosinofílica

La respuesta correcta es: In ltración eosinofílica

/
Pregunta 2 Correcta Puntúa 0,16 sobre 0,16

¿Cuál es la clasi cación anatómica de las bronquiectasias?

Seleccione una:

o a. Cubicas, cilíndricas, alargadas

o b. Lobares, arteriales, globulares

o c. Vasculares, semilunares saculares

d. Cilíndricas, varicosas, saculares

La respuesta correcta es: Cilíndricas, varicosas, saculares

Pregunta 3 Correcta Puntúa 0,16 sobre 0,16

¿Cuál es la siopatología de la hemoptisis?

Seleccione una:

a. Hipervascularización de la circulación brónquica, hipertensión pulmonar y neovascularización

o b. Hipervascularización de la circulación pulmonar, ebre y regeneración alveolar

o c. Hipervascularización de la circulación pulmonar, hipertensión pulmonar y remodelación.

o d. Hipervascularización de la circulación brónquica, hipertensión pulmonar y disminución de


coagulabilidad

La respuesta correcta es: Hipervascularización de la circulación brónquica, hipertensión pulmonar y


neovascularización

/
Pregunta 4 Correcta Puntúa 0,16 sobre 0,16

De los siguientes factores, cuál es el que debe estar alterado para que un paciente infectado de
tuberculosis se convierta en persona enferma:

Seleccione una:

o a. Medio Ambiente: Que viva en un lugar frío

o b. Huésped: que genéticamente esté predispuesto a infectarse

o c. Medio ambiente: Que haya elevada prevalencia de tuberculosis

d. Huésped: Inmunidad celular de ciente, en especial CD4

La respuesta correcta es: Huésped: Inmunidad celular de ciente, en especial CD4

Pregunta 5 Correcta Puntúa 0,16 sobre 0,16

Dentro de las manifestaciones tardías de la sí lis tenemos trastornos cardiovasculares de cual podemos
desprender el siguiente:

Seleccione una:

a. Aneurismas

o b. Insu ciencia cardiaca congestiva.

o c. Flebitis irritativa.

o d. Trastornos del endotelio

La respuesta correcta es: Aneurismas

/
Pregunta 6 Correcta Puntúa 0,16 sobre 0,16

¿De qué depende la inmunidad contra Leptospira?

Seleccione una:

o a. Producción de anticuerpos contra LPS inespecí cos.

o b. Producción de anticuerpos contra proteínas inespecí cas de un serotipo.

o c. Producción de anticuerpos contra los distintos serotipos.

@ d. Producción de anticuerpos circulantes contra LPS especí cos de un serotipo.

La respuesta correcta es: Producción de anticuerpos circulantes contra LPS especí cos de un serotipo.

Pregunta 7 Incorrecta Puntúa 0,00 sobre 0,16

El hallazgo más frecuente en la radiografía de tórax en un paciente con asma es:

Seleccione una:

o a. Engrosamiento de paredes bronquiales.

o b. Condensaciones alveolares bilaterales y difusas.

o c. Radiografía de tórax normal.

@ d. Hiperinsu ación pulmonar. X

La respuesta correcta es: Radiografía de tórax normal.

/
Pregunta 8 Correcta Puntúa 0,16 sobre 0,16

El mecanismo por el cual la infección por cólera produce una diarrea tan intensa es por:

Seleccione una:

o a. Vibrios entero hemorrágicos.

o b. Toxina Shiga-like

o c. Vibrios enteroagregativos.

d. Toxina enterogénica

La respuesta correcta es: Toxina enterogénica

Pregunta 9 Correcta Puntúa 0,16 sobre 0,16

El siguiente enunciado: Un estado parecido al sueño profundo en el que el paciente permanece con los
ojos cerrados y el paciente no puede ser despertado, se re ere a:

Seleccione una:

o a. Estado vegetativo

b. Estado de coma

o c. Somnolencia

o d. Estupor

La respuesta correcta es: Estado de coma

/
Pregunta 10 Correcta Puntúa 0,16 sobre 0,16

El tratamiento de las Bronquiectasias se basa en 3 pilares, excepto:

Seleccione una:

o a. Eliminar la obstrucción bronquial.

o b. Controlar las infecciones con el uso de antibióticos en las agudizaciones durante 10-15 días.

@ c. Revertir el remodelamiento bronquia

o d. Mejorar la eliminación de las secreciones, que se consigue con una adecuada hidratación, con
sioterapia respiratoria y drenaje postural mantenidos.

La respuesta correcta es: Revertir el remodelamiento bronquia

Pregunta 11 Correcta Puntúa 0,16 sobre 0,16

En cuanto a la pro laxis antitetánica en el tratamiento sistemático de las heridas, en una herida
pequeña limpia sin ningún antecedente de vacunación antitetánica Usted recomendaría:

Seleccione una:

a. Solo limpiar la herida y dar indicaciones de cuidado al paciente, ofrecer la anti toxina.

o b. Dar antibiótico de manera pro láctica.

o c. Vacunación antitetánica de inmediato.

o d. Es indicativo de inmunoglobulina.

La respuesta correcta es: Solo limpiar la herida y dar indicaciones de cuidado al paciente, ofrecer la anti
toxina.

/
Pregunta 12 Correcta Puntúa 0,16 sobre 0,16

En cuanto al diagnóstico funcional del asma:

Seleccione una:

a. Si la relación VEF1/CVF es menor a 0.7 (patrón obstructivo) y post broncodilatador obtenemos un


aumento igual o mayor al 12% en el VEF1, nos orienta hacia diagnóstico de asma

o b. Valores espirométricos no tienen importancia en el diagnóstico del asma.

o c. Si la relación VEF1/CVF es menor a 0.7 (patrón restrictivo) y post broncodilatador obtenemos un


aumento igual o mayor al 12% en el VEF1, nos orienta hacia diagnóstico de asma

o d. Si la relación VEF1/CVF es mayor 0.7 se considera patrón obstructivo, diagnosticamos asma

La respuesta correcta es: Si la relación VEF1/CVF es menor a 0.7 (patrón obstructivo) y post broncodilatador
obtenemos un aumento igual o mayor al 12% en el VEF1, nos orienta hacia diagnóstico de asma

Pregunta 13 Incorrecta Puntúa 0,00 sobre 0,16

En el manejo de un paciente con delirium agitado (o hiperactivo), de las siguientes opciones, señale la
que NO incluiría en su prescripción médica:

Seleccione una:

o a. Manejo no farmacológico como: colocar un reloj visible para el paciente

o b. Evitar uso de restricciones físicas (amarras) para controlar el movimiento del paciente

c. Antipsicóticos atípicos como la quetiapina en vía oral X


o d. Benzodiacepina como el clonazepam en vía oral

La respuesta correcta es: Benzodiacepina como el clonazepam en vía oral

/
Pregunta 14 Correcta Puntúa 0,16 sobre 0,16

En la clasi cación de OMS de adultos con VIH – Sida en el estadio C3 tenemos a:

Seleccione una:

o a. Menor de 300 células CD4+ con síntomas de nidores de Sida.

o b. Mayor de 500 células CD4+ Asintomático.

c. Menor de 200 células CD4+ con síntomas de nidores de Sida.

o d. Mayor de 400 células CD4+ con síntomas no de nidores de Sida.

La respuesta correcta es: Menor de 200 células CD4+ con síntomas de nidores de Sida.

Pregunta 15 Correcta Puntúa 0,16 sobre 0,16

En pacientes con catatonía, usted sospecharía:

Seleccione una:

a. Trastorno psiquiátrico

o b. Lesión de la región orbitaria frontal

o c. Secuela del coma

o d. Herniación central

La respuesta correcta es: Trastorno psiquiátrico

/
Pregunta 16 Incorrecta Puntúa 0,00 sobre 0,16

En relación a los siguientes enunciados acerca del ictus isquémico o enfermedad cerebrovascular
isquémica, señale el verdadero:

Seleccione una:

a. El tratamiento de HTA como prevención primaria con cifras de TA sistólica &lt;120 mm Hg reduce
en 43% la presencia de ictus y ataques del corazón X
o b. Cualquier fármaco antiplaquetario (aspirina, clopidogrel, ticlopidina) son e caces y aprobados para
el uso en fase aguda de un ictus isquémico

o c. Un paciente con brilación auricular valvular (ej enfermedad valvular reumática) requiere uso de
anticoagulantes como prevención primaria o secundaria

o d. El uso de estatinas reduce el riesgo de ictus isquémico inclusive con niveles normales de LDL o
niveles bajos de HDL

La respuesta correcta es: Cualquier fármaco antiplaquetario (aspirina, clopidogrel, ticlopidina) son e caces y
aprobados para el uso en fase aguda de un ictus isquémico

Pregunta 17 Correcta Puntúa 0,16 sobre 0,16

En un paciente de 68 años de edad, con APP: HTA, que ingresa con un cuadro de crisis focales motoras
sin pérdida del estado de conciencia y limitadas al miembro superior derecho, que se han repetido 1
crisis cada 3 días desde hace 2 semanas, además se acompaña de cefalea desde hace 3 meses con
características de empeoramiento progresivo, en los estudios de imagen se encontró una lesión
expansiva frontal izquierda; en relación al tratamiento farmacológico de elección, señale el que
escogería:

Seleccione una:

o a. Fenobarbital

o b. Clonazepam

® c. Lamotrigina

o d. Acido valproico

La respuesta correcta es: Lamotrigina

/
Pregunta 18 Correcta Puntúa 0,16 sobre 0,16

La causa principal de un fracaso en el tratamiento de la tuberculosis pulmonar es

Seleccione una:

o a. Toxicidad hepática

o b. Resistencia secundaria a las drogas

o c. Resistencia primaria a las drogas

d. Abandono del tratamiento por el paciente

La respuesta correcta es: Abandono del tratamiento por el paciente

Pregunta 19 Incorrecta Puntúa 0,00 sobre 0,16

La ictericia en un paciente con Malaria es frecuente a partir:

Seleccione una:

o a. La ictericia no aparece en la malaria.

o b. La Ictericia en Malaria es frecuente a partir de los 15 años.

@ c. Siempre existirá ebre e ictericia debido a la destrucción de eritrocitos. X


o d. Ocasionalmente en niños debido a su alta susceptibilidad

La respuesta correcta es: La Ictericia en Malaria es frecuente a partir de los 15 años.

/
Pregunta 20 Correcta Puntúa 0,16 sobre 0,16

Paciente de 16 años de género masculino, sin APP, empieza con crisis convulsivas tónico clónicas
generalizadas hace 1 año, 1 crisis cada 2 meses, su examen neurológico es normal al igual que la
Resonancia Magnética cerebral, el electroencefalograma demostró actividad epileptiforme, cuál de las
siguientes etiologías consideraría en el diagnóstico

Seleccione una:

@ a. Epilepsia por desorden genético

o b. Epilepsia por enfermedad degenerativa

o c. Epilepsia secundaria a autoanticuerpos

o d. Epilepsia secundaria a disturbios hidroelectrolíticos

La respuesta correcta es: Epilepsia por desorden genético

Pregunta 21 Incorrecta Puntúa 0,00 sobre 0,16

Paciente de 22 años, género femenino, tiene desde hace 1 año dolor de cabeza de forma episódica (2
episodios al mes), hemicránea derecha o izquierda, pulsátil, intensidad moderada, se acompaña de
náusea y fotofobia, el dolor es precedido por escotomas centellantes que duran 90 minutos
aproximadamente, en los últimos 3 meses el dolor se ha tornado más frecuente (3 episodios semanales)
y al toser exacerba en intensidad a dolor severo, de los siguientes enunciados cuál NO considera
signo/síntoma de alarma:

Seleccione una:

o a. Localización del dolor

o b. Empeoramiento del dolor (más frecuente)

o c. Valsalva positivo

d. Aura de duración larga X

La respuesta correcta es: Localización del dolor

/
Pregunta 22 Incorrecta Puntúa 0,00 sobre 0,16

Paciente de 38 años de género femenino, con APP diagnóstico de fenómeno de Raynaud en estudio de
un posible Lupus eritematoso sistémico, también diagnóstico de migraña con aura desde la juventud,
acude por un ataque agudo de cefalea, usted qué fármaco NO recomendaría:

Seleccione una:

@ a. Clorpromazina X
o b. AINES

o c. Aspirina + metoclopramida

o d. Sumatriptán

La respuesta correcta es: Sumatriptán

Pregunta 23 Correcta Puntúa 0,16 sobre 0,16

Paciente masculino de 66 años de edad, con antecedentes de deterioro cognitivo leve, HTA, ingresa para
una cirugía electiva de resección prostática por una hipertro a prostática benigna, en su postoperatorio
se encuentra con sondaje (cateterización), vesical permanente, dolor pélvico, y un cuadro de delirium (o
sd confusional agudo), se reinició el enalapril en el postoperatorio, de las siguientes opciones señale
cuál NO es un factor de riesgo para el aparecimiento de delirium

Seleccione una:

o a. Dolor en el postoperatorio

b. Administración de enalapril

o c. Antecedentes de deterioro cognitivo leve

o d. Sondaje vesical

La respuesta correcta es: Administración de enalapril

/
Pregunta 24 Incorrecta Puntúa 0,00 sobre 0,16

Tras la infección de un niño con el virus de la In uenza con la siguientes características: ebre intensa de
inicio súbito, disnea y cianosis a la Rx de tórax se aprecia patrón asociado con in ltrados intersticiales
difusos e hipoxia intensa, usted sospecharía de:

Seleccione una:

@ a. Neumonía bacteriana secundaria. X


o b. Neumonía viral primaría.

o c. Neumonía viral secundaria.

o d. Neumonía bacteriana primaria.

La respuesta correcta es: Neumonía viral primaría.

Pregunta 25 Correcta Puntúa 0,16 sobre 0,16

Una de las características principales por las cuales el dengue puede diseminarse es:

Seleccione una:

o a. Una característica precoz es la trombocitosis y neutro lia.

o b. Debido a la gran capacidad de diseminación del vector Anopheles.

@ c. El vector es muy cercano a los asentamientos humanos.

o d. Es sumamente frecuente la progresión de daño encefálico.

La respuesta correcta es: El vector es muy cercano a los asentamientos humanos.

◄ Avisos Ir a... Link sesión zoom examen primer parcial ►

/
2
AYLEN MARILYN VERGARA TITO

Área personal  Mis cursos  Quito  CIENCIAS DE LA SALUD  MEDICINA - PRESENCIAL  ABR 2020 - AGO 2020
 INVESTIGACIÓN I - Prl: MD NVD Pen: 961  18 de julio: 12:00 H: Examen Segundo parcial Investigación I  Examen Segundo
Parcial Investigación I

Comenzado el sábado, 18 de julio de 2020, 12:10


Estado Finalizado
Finalizado en sábado, 18 de julio de 2020, 12:52
Tiempo empleado 42 minutos
Cali cación 3,68 de 4,00 (92%)

Pregunta 1 Correcta Puntúa 0,16 sobre 0,16

Causa de riesgo de Trombosis adquirida tanto venosa como arterial, la cual amerita tratamiento
anticoagulante?

Seleccione una:

o a. Inmovilización

o b. Cirugía mayor

@ c. Síndrome Antifosfolipídico

o d. Embarazo

La respuesta correcta es: Síndrome Antifosfolipídico


/
Pregunta 2 Correcta Puntúa 0,16 sobre 0,16

¿Cuál es la reacción transfusional más frecuente?

Seleccione una:

@ a. Reacción febril no hemolítica

o b. Reacción alérgica

o c. Reacción Hemolítica

o d. Reacción Ana láctica

La respuesta correcta es: Reacción febril no hemolítica

Pregunta 3 Correcta Puntúa 0,16 sobre 0,16

Cuál glomerulopatía no produce daño vascular:

Seleccione una:

o a. Crioglobulinemia

o b. Nefritis lúpica

o c. Granulomatosis de Wegener

@ d. Nefropatía por IgA

La respuesta correcta es: Nefropatía por IgA


/
Pregunta 4 Correcta Puntúa 0,16 sobre 0,16

Cuál no es una glomerulopatía de vasos pequeños ANCA:

Seleccione una:

o a. Síndrome de Churg-Strauss

@ b. Púrpura de Henoch-Schönlein

o c. Granulomatosis de Wegener

o d. Poliangitis microscópica

La respuesta correcta es: Púrpura de Henoch-Schönlein

Pregunta 5 Correcta Puntúa 0,16 sobre 0,16

Cuánto es la dosis de dabigatrán con CCr >30 mL/min:

Seleccione una:

o a. 150 mg cada 24 horas

@ b. 150 mg cada 12 horas

o c. 150 mg cada 6 horas

o d. 150 mg cada 8 horas

La respuesta correcta es: 150 mg cada 12 horas


/
Pregunta 6 Correcta Puntúa 0,16 sobre 0,16

Cuántos Joules se usa en la brilación auricular con hipotensión profunda, edema pulmonar o angina:

Seleccione una:

o a. 150 Joules

o b. 100 Joules

o c. 250 Joules

@ d. 200 Joules

La respuesta correcta es: 200 Joules

Pregunta 7 Incorrecta Puntúa 0,00 sobre 0,16

El riesgo de contaminación Bacteriana debido a una transfusión ha aumentado conforme disminuye el


riesgo de infecciones virales, muchas bacterias no proliferan en frío. Cuál de estos derivados representa el
mayor riesgo de contaminación?

Seleccione una:

o a. Concentrado de plaquetas

o b. Críoprecipitado

o c. Plasma

@ d. Concentrado de glóbulos rojos X

La respuesta correcta es: Concentrado de plaquetas


/
Pregunta 8 Correcta Puntúa 0,16 sobre 0,16

La clase IV de la nefritis lúpica se llama:

Seleccione una:

@ a. Nefritis difusa

o b. Nefritis esclerótica

o c. Nefritis focal

o d. Proliferación mesangial

La respuesta correcta es: Nefritis difusa

Pregunta 9 Correcta Puntúa 0,16 sobre 0,16

La clase VI de la nefritis lúpica se llama:

Seleccione una:

o a. Nefritis difusa

o b. Proliferación mesangial

@ c. Nefritis esclerótica

o d. Nefritis focal

La respuesta correcta es: Nefritis esclerótica


/
Pregunta 10 Correcta Puntúa 0,16 sobre 0,16

La fragmentación de Eritrocitos (Esquitocitos) imagen en sangre periférica que observamos en pacientes


con:

Seleccione una:

@ a. Válvulas Cardíacas Mecánicas

o b. Anemia por trastorno de maduración (A. megaloblástica)

o c. Daño medular. Hipoplasia

o d. Dé cit de hierro

La respuesta correcta es: Válvulas Cardíacas Mecánicas

Pregunta 11 Correcta Puntúa 0,16 sobre 0,16

La hepatitis B que nefropatía produce:

Seleccione una:

o a. Nefritis lúpica

o b. Nefropatía por IgA

o c. Granulomatosis de Wegener

@ d. Glomerulonefritis membranoproliferativa

La respuesta correcta es: Glomerulonefritis membranoproliferativa


/
Pregunta 12 Correcta Puntúa 0,16 sobre 0,16

La hipertensión esencial corresponde a

Seleccione una:

o a. Mas frecuente en pacientes jóvenes

@ b. Se relaciona con alteración en la excreción de sodio

o c. La minoriade paciente hipertensos

o d. Se relaciona con alteración en la excreción de calcio

La respuesta correcta es: Se relaciona con alteración en la excreción de sodio

Pregunta 13 Incorrecta Puntúa 0,00 sobre 0,16

La obstrucción ureteral bilateral aguda se caracteriza por:

Seleccione una:

o a. Disminución de la liberación de óxido nítrico

o b. Disminución de la producción de angiotensina

@ c. Disminución del ujo sanguíneo medular X


o d. Disminución de prostaglandinas vasodilatadoras

La respuesta correcta es: Disminución de la liberación de óxido nítrico


/
Pregunta 14 Correcta Puntúa 0,16 sobre 0,16

La obstrucción ureteral bilateral crónica se caracteriza por:

Seleccione una:

o a. Diuresis posterior a la obstrucción

@ b. Hipertensión

o c. Dolor renal

o d. Anuria

La respuesta correcta es: Hipertensión

Pregunta 15 Correcta Puntúa 0,16 sobre 0,16

La proteinuria sostenida es aquella que se expulsan más de:

Seleccione una:

o a. 7 a 8 g/24 h

o b. 5 a 6 g/24 h

o c. 3 a 4 g/24 h

@ d. 1 a 2 g/24 h

La respuesta correcta es: 1 a 2 g/24 h


/
Pregunta 16 Correcta Puntúa 0,16 sobre 0,16

Los Eritrocitos jóvenes que con la coloración de azul de metileno observamos restos de ARN y su aumento
indica buena respuesta de la médula ante una anemia. Cuál célula nos referimos?

Seleccione una:

o a. Células en Diana

o b. Esquitocitos

o c. Cuerpos de Howel Jolly

@ d. Reticulocitos

La respuesta correcta es: Reticulocitos

Pregunta 17 Correcta Puntúa 0,16 sobre 0,16

Paciente con reacción transfusional Ana láctica a repetición. Debe ser estudiado para descartar:

Seleccione una:

@ a. Dé cit de Ig A

o b. Dé cit de Ig M

o c. Dé cit de Ig E

o d. Dé cit de Ig G

La respuesta correcta es: Dé cit de Ig A


/
Pregunta 18 Correcta Puntúa 0,16 sobre 0,16

Paciente hipertenso con uso de 3 antihipertensivos uno de los cuales es un diurético que persiste con
presiones arteriales elevadas. Usted pensaría en

Seleccione una:

@ a. Hipertension resistente

o b. Hipertension de bata blanca

o c. Hipertension enmascarada

o d. Hipertension refractaria

La respuesta correcta es: Hipertension resistente

Pregunta 19 Correcta Puntúa 0,16 sobre 0,16

Paciente que en monitoreo ambulatorio de presión arterial presenta cifras tensionales en parámetros
adecuados, pero en el consultorio presenta cifras tensionales elevadas. Usted pensaría en

Seleccione una:

@ a. Hipertension de bata blanca

o b. d)Hipertension refractaria

o c. c)Hipertension resistente

o d. Hipertension enmascarada

La respuesta correcta es: Hipertension de bata blanca


/
Pregunta 20 Correcta Puntúa 0,16 sobre 0,16

Paciente que presenta cifras tensionales en el consultorio dentro de parámetros normales, pero evidencia
de retinopatía hipertensiva. Usted pensaría en

Seleccione una:

o a. Hipertension resistente

o b. Hipertension refractaria

o c. Hipertension de bata blanca

@ d. Hipertension enmascarada

La respuesta correcta es: Hipertension enmascarada

Pregunta 21 Correcta Puntúa 0,16 sobre 0,16

Qué antidepresivo causa disfunción del nódulo sinoauricular:

Seleccione una:

@ a. Amitriptilina

o b. Sertralina

o c. Fluoxetina

o d. Midazolam

La respuesta correcta es: Amitriptilina


/
Pregunta 22 Correcta Puntúa 0,16 sobre 0,16

Qué endocrinopatía no causa prolongación de QT y de taquicardia ventricular polimorfa:

Seleccione una:

o a. Hipotiroidismo

@ b. Hipogonadismo

o c. Hiperladosteronismo

o d. Hiperparatiroidismo

La respuesta correcta es: Hipogonadismo

Pregunta 23 Correcta Puntúa 0,16 sobre 0,16

Qué familia de antibiótico no causa prolongación de QT y de taquicardia ventricular polimorfa:

Seleccione una:

@ a. Cefalosporinas

o b. Macrólidos

o c. Quinolonas

o d. Antagonistas del folato

La respuesta correcta es: Cefalosporinas


/
Pregunta 24 Correcta Puntúa 0,16 sobre 0,16

Qué vitamina causa insu ciencia cardiaca con alto gasto:

Seleccione una:

@ a. B

o b. D

o c. E

o d. A

La respuesta correcta es: B

Pregunta 25 Correcta Puntúa 0,16 sobre 0,16

Una anemia con un Índice de Producción de Reticulocítos ( IPR ) mayor de 2 % es una anemia
hiperproliferativa . Cuál sería la causa de anemia?

Seleccione una:

o a. Hipoplasia medular

o b. Disfunción Renal

o c. Dé cit de hierro

@ d. Hemorragia y Hemólisis

La respuesta correcta es: Hemorragia y Hemólisis

◄ Link sesión zoom examen nal 1: Investigación I Ir a... V

Link sesión zoom examen segundo parcial ►


/
2
AYLEN MARILYN VERGARA TITO

Área personal  Mis cursos  Quito  CIENCIAS DE LA SALUD  MEDICINA - PRESENCIAL  ABR 2020 - AGO 2020
 INVESTIGACIÓN I - Prl: MD NVD Pen: 961  Sábado 4 de julio 8:00 am. Investigación I: Examen nal 1  Examen nal 1:
Investigación I

Comenzado el sábado, 4 de julio de 2020, 08:10


Estado Finalizado
Finalizado en sábado, 4 de julio de 2020, 09:27
Tiempo empleado 1 hora 16 minutos
Cali cación 4,13 de 5,33 (78%)

Pregunta 1 Correcta Puntúa 0,13 sobre 0,13

Ante un paciente con infarto cerebral en fase aguda, seleccione la respuesta correcta en cuanto al
tratamiento.

Seleccione una:

o a. Debe reducirse la presión arterial siempre.

o b. Se debe valorar tratamiento trombolítico intravenoso si llega antes de 6 horas del inicio de los
síntomas.

c. Se debe utilizar manitol cuando el edema cerebral produce disminución de la conciencia o se observa
desplazamiento de la línea media en la TC.

o d. Se debe colocar solución de Dextrosa 5% para hidratación.

La respuesta correcta es: Se debe utilizar manitol cuando el edema cerebral produce disminución de la conciencia
o se observa desplazamiento de la línea media en la TC.

/
Pregunta 2 Correcta Puntúa 0,13 sobre 0,13

¿A qué género pertenece el virus del dengue?

Seleccione una:

@ a. Flaviviridae

o b. Hantavirus

o c. Coronavirus

o d. Poxvirus

La respuesta correcta es: Flaviviridae

Pregunta 3 Incorrecta Puntúa 0,00 sobre 0,13

¿Cómo se denomina el vector transmisor de ebre amarilla?

Seleccione una:

@ a. Anopheles X
o b. Aedes aegypti

o c. Culecoides

o d. Lutzomya

La respuesta correcta es: Aedes aegypti

/
Pregunta 4 Correcta Puntúa 0,13 sobre 0,13

¿Cuál de las siguientes alternativas es una de las medidas para evitar la diseminación del Dengue entre la
población?

Seleccione una:

o a. Introducción experimental de mosquitos transgénicos

o b. Administración de la vacuna a poblaciones susceptibles.

o c. Pro laxis con Cloroquina – Primaquina

@ d. Control vectorial

La respuesta correcta es: Control vectorial

Pregunta 5 Incorrecta Puntúa 0,00 sobre 0,13

¿Cuál de las siguientes es una causa de hemoptisis?

Seleccione una:

o a. Vàrices esofágicas

o b. Estenosis Mitral

@ c. Edema agudo de Pulmòn X


o d. Fibrosis Pulmonar

La respuesta correcta es: Estenosis Mitral

/
Pregunta 6 Correcta Puntúa 0,13 sobre 0,13

Cuál de los siguientes enunciados corresponde a la de nición de neumonía nosocomial tardía

Seleccione una:

o a. Aparece después de 21 días del ingreso

o b. Aparece después de 10 días del ingreso

@ c. Aparece después de 7 días del ingreso

o d. Aparece después de 14 días del ingreso

La respuesta correcta es: Aparece después de 7 días del ingreso

Pregunta 7 Correcta Puntúa 0,13 sobre 0,13

¿Cuál es la acción de los nitratos en el síndrome coronario agudo?

Seleccione una:

@ a. vasodilatador coronario

o b. antiagregante

o c. revascularizador

o d. sedante

La respuesta correcta es: vasodilatador coronario

/
Pregunta 8 Correcta Puntúa 0,13 sobre 0,13

¿Cuál es la característica de la cefalea en racimos?

Seleccione una:

o a. Síntomas bilaterales

o b. Dolor leve

@ c. Síntomas ipsilaterales

o d. Dolor en la región occipital del cráneo

La respuesta correcta es: Síntomas ipsilaterales

Pregunta 9 Correcta Puntúa 0,13 sobre 0,13

¿Cuál es la diferencia entre una crisis focal y una crisis generalizada?

Seleccione una:

a. Una crisis focal se limita a un solo hemisferio mientras que una crisis generalizada se distribuye en
ambos hemisferios.

o b. No existe diferencia

o c. Una crisis focal se origina en el interior conectándose con las redes de ambos hemisferios mientras
que una crisis generalizada se limita a un solo hemisferio.

o d. Una crisis focal se origina en un lóbulo del cerebro mientras que una generalizada se origina en las
redes límites de todo un hemisferio.

La respuesta correcta es: Una crisis focal se limita a un solo hemisferio mientras que una crisis generalizada se
distribuye en ambos hemisferios.

/
Pregunta 10 Correcta Puntúa 0,13 sobre 0,13

¿Cuál es la función de la aspirina en el síndrome coronario agudo?

Seleccione una:

o a. Trombolisis

o b. Anticoagulantes

@ c. Evitar el incremento de la agregación plaquetaria en la placa ateromatosa accidentada

o d. Analgésico antin amatorio

La respuesta correcta es: Evitar el incremento de la agregación plaquetaria en la placa ateromatosa accidentada

Pregunta 11 Correcta Puntúa 0,13 sobre 0,13

¿Cuáles son signos de insu ciencia cardíaca derecha?

Seleccione una:

o a. Dolor abdominal, astenia

o b. Ortópnea, dolor precordial

@ c. Edema de miembros inferiores, hepatomegalia

o d. Disnea, estertores crepitantes

La respuesta correcta es: Edema de miembros inferiores, hepatomegalia

/
Pregunta 12 Correcta Puntúa 0,13 sobre 0,13

¿Cuántos días abarca el período de incubación de la leptospirosis?

Seleccione una:

o a. De 10 a 20 días (aproximadamente 2 semanas)

o b. De 1 a 60 días (aproximadamente 3 semanas)

@ c. De 1 a 30 días (aproximadamente 2 semanas)

o d. De 1 a 15 días (aproximadamente 1 semana)

La respuesta correcta es: De 1 a 30 días (aproximadamente 2 semanas)

Pregunta 13 Correcta Puntúa 0,13 sobre 0,13

De las encefalitis virales seleccione la que se bene cia del tratamiento con Aciclovir:

Seleccione una:

o a. Encefalitis equina

o b. Citomegalovirus

@ c. Herpes virus

o d. Enterovirus

La respuesta correcta es: Herpes virus

/
Pregunta 14 Correcta Puntúa 0,13 sobre 0,13

¿De qué depende el mayor riesgo de ENFERMARSE de Tuberculosis?

Seleccione una:

o a. Prevalencia de Tuberculosis

o b. Estilo de vida

o c. Genètica

@ d. Inmunidad celular, CD4

La respuesta correcta es: Inmunidad celular, CD4

Pregunta 15 Incorrecta Puntúa 0,00 sobre 0,13

El derrame pleural puede ser causado por diferentes causas. ¿En cuál de ellos usted sugiere realizar una
pleurodesis?

Seleccione una:

o a. Por Insu ciencia Cardíaca

o b. Por Tuberculosis

@ c. Por Neumonía X
o d. Por Cáncer

La respuesta correcta es: Por Cáncer

/
Pregunta 16 Correcta Puntúa 0,13 sobre 0,13

El dolor tipo pleurítico se caracteriza por ser:

Seleccione una:

o a. Opresivo intenso que disminuye con la inspiración

o b. Punzante intenso que disminuye con la inspiración

@ c. Punzante intenso que aumenta con la inspiración

o d. Opresivo intenso que aumenta con la inspiraciòn

La respuesta correcta es: Punzante intenso que aumenta con la inspiración

Pregunta 17 Correcta Puntúa 0,13 sobre 0,13

El uso de betabloqueantes debe ser analizado en el paciente con insu ciencia cardiaca ya que este grupo
farmacológico posee ciertas características importantes. Señale lo correcto respecto a estos productos:

Seleccione una:

o a. Puede utilizarse con seguridad en pacientes con asma

o b. Pueden utilizarse con seguridad en combinación con verapamilo

o c. Pueden utilizarse con seguridad en pacientes con hipotensión sintomático

@ d. Pueden utilizarse con seguridad en pacientes con insu ciencia cardiaca sistólica y angina de esfuerzo

La respuesta correcta es: Pueden utilizarse con seguridad en pacientes con insu ciencia cardiaca sistólica y
angina de esfuerzo

/
Pregunta 18 Correcta Puntúa 0,13 sobre 0,13

¿En cuál sistema arterial se producen la mayoría de las hemoptisis y por qué? Señale lo CORRECTO

Seleccione una:

o a. Las arterias bronquiales forman parte de la circulación sistémica, tienen menor presión y mucho
mayor ujo, de ellas depende la irrigación de los bronquios y la pleura visceral.

b. Las arterias bronquiales forman parte de la circulación sistémica, tienen mayor presión y mucho
menor ujo, de ellas depende la irrigación de los bronquios y la pleura visceral.

o c. Las arterias pulmonares son un sistema de alta presión por el que circula todo el gasto cardíaco

o d. Las arterias pulmonares son un sistema de baja presión por el que circula todo el gasto cardiaco y son
las responsables del intercambio gaseoso

La respuesta correcta es: Las arterias bronquiales forman parte de la circulación sistémica, tienen mayor presión
y mucho menor ujo, de ellas depende la irrigación de los bronquios y la pleura visceral.

Pregunta 19 Correcta Puntúa 0,13 sobre 0,13

¿En qué caso complementamos el electrocardiograma con derivaciones derechas?

Seleccione una:

o a. Sospecha de edema agudo de pulmón

@ b. Sospecha de infarto de ventrículo derecho

o c. Taquicardia ventricular monomorfa

o d. Ritmo de galope

La respuesta correcta es: Sospecha de infarto de ventrículo derecho

/
Pregunta 20 Correcta Puntúa 0,13 sobre 0,13

En una paciente de 30 años es llevada inconsciente a la emergencia de un hospital. Uno de los signos
clínicos que excluye el diagnóstico de muerte cerebral es:

Seleccione una:

o a. ausencia de re ejo nauseoso y tusígeno

@ b. postura de decerebración bilateral

o c. Glasgow 3

o d. pupilas midriáticas arreactivas

La respuesta correcta es: postura de decerebración bilateral

Pregunta 21 Correcta Puntúa 0,13 sobre 0,13

Forma de transmisión del virus de la hepatitis A

Seleccione una:

@ a. vía fecal- oral

o b. sexual

o c. vía respiratoria

o d. Por animales

La respuesta correcta es: vía fecal- oral

/
Pregunta 22 Correcta Puntúa 0,13 sobre 0,13

La causa más frecuente de hemoptisis leve-moderada es:

Seleccione una:

@ a. Bronquiectasias.

o b. Infarto pulmonar.

o c. Carcinoma broncogénico.

o d. Diátesis hemorrágica.

La respuesta correcta es: Bronquiectasias.

Pregunta 23 Correcta Puntúa 0,13 sobre 0,13

La enfermedad pulmonar obstructiva crónica no tratada puede conducir a la siguiente complicación


cardíaca:

Seleccione una:

o a. Síndrome coronario agudo

o b. Bradicardia

o c. Pericarditis

@ d. Insu ciencia cardíaca derecha

La respuesta correcta es: Insu ciencia cardíaca derecha

/
Pregunta 24 Correcta Puntúa 0,13 sobre 0,13

La insu ciencia cardíaca izquierda puede provocar edema agudo de pulmón por aumento de la:

Seleccione una:

o a. Presión sistólica del ventrículo derecho

o b. Presión venosa central

@ c. Presión capilar pulmonar

o d. Presión arterial

La respuesta correcta es: Presión capilar pulmonar

Pregunta 25 Correcta Puntúa 0,13 sobre 0,13

Los promastigotes son la forma de leishmania que se inyectan por medio de la probóscide del ebótomo de
sexo femenino en la piel del hospedador, ¿qué células son las encargadas de fagocitar a estas formas
parasitarias?

Seleccione una:

@ a. Neutró los

o b. Macrófagos

o c. Eosinó los

o d. Monocitos

La respuesta correcta es: Neutró los

/
Pregunta 26 Correcta Puntúa 0,13 sobre 0,13

María, 65 años, acude a la consulta re riendo cansancio a los moderados esfuerzos, re ere haber tenido
ebre reumática, al examen físico presenta PA 130/80, FC 88lpm, soplo sistólico en quinto espacio
intercostal línea media clavicular izquierda III/VI, con irradiación a la axila. ¿Cuál es el diagnóstico más
probable en esta paciente?

Seleccione una:

o a. Insu ciencia aortica

o b. Estenosis mitral

@ c. Insu ciencia mitral

o d. Estenosis aortica

La respuesta correcta es: Insu ciencia mitral

Pregunta 27 Correcta Puntúa 0,13 sobre 0,13

Paciente acude con trastornos osteomusculares crónicos, junto con dé cit visual inespecí co, pérdida de
peso y ebre de larga evaluación, el paciente proviene de una zona meramente ganadera, Usted sospecha
de brucelosis cuya base de tratamiento es:

Seleccione una:

o a. Penicilina y Metronidazol

@ b. Dicloxacilina y Estreptomicina

o c. Gentamicina y Dapsona

o d. Isoniacida y Estreptomicina

La respuesta correcta es: Dicloxacilina y Estreptomicina

/
Pregunta 28 Correcta Puntúa 0,13 sobre 0,13

Paciente de 25 años que re ere viaje reciente a Filipinas presenta: ebre, dolor abdominal, diarrea,
badicardia relativa. ¿Cuál es el diagnóstico más probable?

Seleccione una:

@ a. Fiebre tifoidea

o b. Paludismo

o c. Gastroenteritis complicada

o d. Micosos sistémica

La respuesta correcta es: Fiebre tifoidea

Pregunta 29 Sin contestar Puntúa como 0,13

Paciente masculino de 55 años de edad, sin antecedentes patológicos familiares de relevancia. Acude por
presentar desde hace 6 días dolor intenso y alodinia a nivel de región lumbar derecha. Hoy en la mañana
nota erupciones muy pruriginosas en la piel. Antecedentes patológicos personales: varicela en su niñez. Al
examen físico presenta vesículas agrupadas sobre una placa eritematosa elevada que siguen un mismo
dermatoma. También se observa un desprendimiento de costras dejando zonas expuestas de piel. ¿Cuál
sería el diagnóstico probable y su terapéutica?

Seleccione una:

o a. Herpes Zoster cuyo tratamiento ideal es Aciclovir, valaciclovir o famciclovir formando parte estos
fármacos de la familia de los inhibidores de la transcriptasa inversa.

o b. Herpes Zoster y su tratamiento más adecuado es Aciclovir 800 mg cada 4 horas durante 7 días.

o c. Infección por Herpes Zoster el mismo que necesitará tratamiento inmediato con Fluconazol 250 mg VO
cada día por 5 días.

o d. Dermatitis de contacto y deberá averiguarse sobre factores de predisposición alérgica en el paciente.

La respuesta correcta es: Herpes Zoster y su tratamiento más adecuado es Aciclovir 800 mg cada 4 horas durante
7 días.

/
Pregunta 30 Incorrecta Puntúa 0,00 sobre 0,13

Qué antihipertensivo se debe usar con cautela en un paciente con arteriopatia coronaria grave:

Seleccione una:

o a. Hidralazina

@ b. Espironolactona X
o c. Atenolol

o d. Clonidina

La respuesta correcta es: Hidralazina

Pregunta 31 Correcta Puntúa 0,13 sobre 0,13

Qué endocrinopatía es causa secundaria de hipertensión arterial:

Seleccione una:

o a. Hipogonadismo

@ b. Síndrome de Cushing

o c. Hipocalcemia

o d. Hiperprolactinemia

La respuesta correcta es: Síndrome de Cushing

/
Pregunta 32 Correcta Puntúa 0,13 sobre 0,13

¿Qué es una convulsión?

Seleccione una:

o a. Trastorno en el que una persona tiene convulsiones o crisis recurrentes debido a proceso crónico
subyacente

o b. Episodio paroxístico producido por descargas anormales disminuidas o actividad neuronal no


sincrónica en el cerebro

o c. Ninguna de las anteriores

@ d. Episodio paroxístico producido por descargas anormales excesivas o actividad neuronal sincrónica en
el cerebro

La respuesta correcta es: Episodio paroxístico producido por descargas anormales excesivas o actividad neuronal
sincrónica en el cerebro

Pregunta 33 Incorrecta Puntúa 0,00 sobre 0,13

¿Qué parámetros se consideran para tener migraña crónica?

Seleccione una:

o a. Episodios de migraña todos los días o casi a diario

@ b. Episodios de migraña cada semana X


o c. Episodios de migraña con alteraciones visuales

o d. Episodios de migraña por más de tres años

La respuesta correcta es: Episodios de migraña todos los días o casi a diario

/
Pregunta 34 Incorrecta Puntúa 0,00 sobre 0,13

Recibe el reporte de una muestra de líquido pleural con los siguientes resultados: exudado amarillento,
recuento del 90% de linfocitos. Pertenece a un paciente que presenta ebre y baja de peso, ¿en cuál de los
siguientes diagnósticos usted pensaría?

Seleccione una:

o a. Empiema

@ b. Neoplasia X
o c. Enfermedad del colágeno

o d. Tuberculosis

La respuesta correcta es: Tuberculosis

Pregunta 35 Incorrecta Puntúa 0,00 sobre 0,13

Señale las características del síndrome atípico de la Neumonía Adquirida en la Comunidad.

Seleccione una:

o a. Clínica aguda.

@ b. Se caracteriza por ebre elevada, escalofríos, tos productiva y dolor pleurítico X


o c. Disociación clínico radiológica

o d. Más evidente en personas ancianas

La respuesta correcta es: Disociación clínico radiológica

/
Pregunta 36 Correcta Puntúa 0,13 sobre 0,13

Su paciente proviene de la amazonia del Ecuador y presenta aproximadamente 15 días con ebre alta con
escalofríos, pero sin pérdida de peso considerable, solo con ligera palidez, al examen físico se parecía muy
levemente una hepato espleno megalia, y Biometría sin cambios aparente, sería prudente realizar lo
siguiente a su criterio:

Seleccione una:

@ a. Gota gruesa y extendido seriado

o b. Ecografía y determinar si existe ascitis y derrame pleural (Extravasación)

o c. Hemocultivo y esperar resultados

o d. Biopsia de Hígado y descartar procesos de ebre de larga evolución (FOD clásico)

La respuesta correcta es: Gota gruesa y extendido seriado

Pregunta 37 Correcta Puntúa 0,13 sobre 0,13

Una mujer de 40 años presenta debilidad progresiva y dolor en las piernas después de una semana de
haber presentado un episodio gripal. El examen físico demuestra tensión arterial 80/40 mm Hg, pulso 150
por minuto, temperatura bucal 37 grados C. Fuerza normal en la cara. Paresia 2/5 en la extremidad inferior
derecha y 3/5 en las extremidad inferior izquierda. Arre exia global con respuestas plantares en exión.
Uno de los signos o síntomas que podría hacer dudar en el diagnóstico de un síndrome de Guillain-Barré es:

Seleccione una:

o a. ausencia de paresia facial

@ b. debilidad asimétrica en las extremidades inferiores

o c. dolor en las piernas

o d. disfunción autonómica

La respuesta correcta es: debilidad asimétrica en las extremidades inferiores

/
Pregunta 38 Correcta Puntúa 0,13 sobre 0,13

Uno de los criterios para considerar la probabilidad de muerte cerebral y proceder a realizar el test de
apnea es:

Seleccione una:

o a. postura de descerebración luego de una reanimación cardiopulmonar de 20 minutos

o b. pupilas mióticas de 2 mm débilmente reactivas a la luz

o c. temperatura corporal de 36 grados centígrados

@ d. TC cerebral con una lesión estructural cerebral compatible con muerte cerebral

La respuesta correcta es: TC cerebral con una lesión estructural cerebral compatible con muerte cerebral

Pregunta 39 Correcta Puntúa 0,13 sobre 0,13

Un paciente de 35 años, consulta por cuadro de 4 semanas de evolución, de tos, con expectoración
mucopurulenta, con estrías de sangre en algunas ocasiones, asociado a compromiso del estado general,
baja de peso, ebre intermitente y sudoración nocturna. Ha tomado amoxicilina en varias oportunidades,
sin respuesta. Al examen físico destacan crépitos escasos, mayores en el ápice derecho. El diagnóstico más
probable es:

Seleccione una:

o a. Bronquiectasias

@ b. Tuberculosis

o c. Cáncer pulmonar

o d. Absceso pulmonar

La respuesta correcta es: Tuberculosis

/
Pregunta 40 Incorrecta Puntúa 0,00 sobre 0,13

Un paciente de 50 años, es trasladado inconsciente a la emergencia del hospital. Se procede a intubarlo


para protección de la vía aérea. Al estímulo doloroso no abre por los ojos, presenta exión de la extremidad
superior derecha con extensión plantar del pie derecho. La puntuación en la escala de Glasgow sería:

Seleccione una:

o a. 6

o b. 7

@ c. 4 X
o d. 5

La respuesta correcta es: 5

◄ Link sesión zoom examen primer parcial Ir a... V

Link sesión zoom examen nal 1: Investigación I ►

/
-
DERMATO

1. ¿CUÁL DE LAS SIGUIENTES REGIONES CORPORALES SON MÁS


SUSCEPTIBLES A LA INFECCIÓN POR CÁNDIDA SSP?

Seleccione una:
a. CARA
b. PIES
c. ÁREAS INTERTRIGINOSAS Y MUCOCUTÁNEAS
d. CABEZA
e. TORAX

2. Señale lo incorrecto en el diagnóstico de la dermatitis atópica

Seleccione una:
a. El diagnóstico de la dermatitis atópica se basa en la clínica
b. Los datos de laboratorio son constantes y específicos para su diagnóstico
c. Existen diversas pautas de diagnóstico con criterios mayores y menores, que son
muy empleadas en estudios epidemiológicos y ensayos clínicos
d. La histopatología muestra una dermatitis espongiforme (eccema), la mayoría de
las veces en un estadio subagudo

3. En la dermatitis atópica señale lo correcto

Seleccione una:
a. Los eccemas por contacto no suelen limitarse a los puntos de actuación del
irritante o del alérgeno
b. Los diagnósticos diferenciales no varían mucho ni dependen de la edad del
paciente ni del tipo de lesiones que presente
c. En el lactante es típica la transición desde la dermatitis seborreica a la atópica
d. La sarna debe tenerse en cuenta en todos los niños con prurito y lesiones
cutáneas, sobre todo si son de inicio reciente.

4. ANTE LA SOSPECHA DE UNA INFECCIÓN FÚNGICA SE DEBE


CONFIRMAR EL DIAGNÓSTICO CLÍNICO, ¿QUÉ EXPLORACIÓN
COMPLEMENTARIA REALIZARÍA?

Seleccione una:
a. BIOPSIA
b. AFECTAN ESTRUCTURAS QUE CONTIENEN QUERATINA
EXAMEN CON LÁMPARA DE WOOD EN PIEL LAMPIÑA
c.EXAMEN DIRECTO CON KOH Y CULTIVO EN SABOURAUD
d.TODAS LAS ANTERIORES
e.ANALÍTICA DE SANGRE
5. Señale lo verdadero de los PIOJOS

Seleccione una:
a. Se acompaña de adenopatía cervical
b. El agente causal es pediculus corporis
c. No tiene carácter epidémico
d. El tratamiento de elección es la Ivermectina tópica y/o sistémica

6. TODOS SON FACTORES QUE PUEDEN ALTERAR EL EQUILIBRIO


ENTRE MICROORGANISMO Y HUÉSPED Y FAVORECER INFECCIONES
CUTÁNEAS, EXCEPTO:

Seleccione una:
a. NINGUNA DE LAS ANTERIORES
b. AUMENTO DE TEMPERATURA
c. INMUNOSUPRESIÓN O ENFERMEDADES
d. HUMEDAD

7. EN UNA URTICARIA CRONICA LA RONCHA O HABON DESAPARECE A


LOS :

Seleccione una:
a. EN OCASIONES A LOS SEIS MESES
b. AL MES
c. MINUTOS U HORAS
d. A LOS 4 DIAS

8. ¿EN QUE EDAD PREDOMINA EL IMPÉTIGO?

Seleccione una:
a. LACTANCIA
b. INFANCIA
c. MADUREZ
d. JUVENTUD

9. Señale lo falso de la DERMATITIS ATOPICA

Seleccione una:
a. LA PREVALENCIA ALCANZA SU MÁXIMO EN LA EDAD ADULTA
b. ASOCIADA A DISFUNCIÓN DE LA BARRERA CUTÁNEA
c. SE CARACTERIZA POR RESEQUEDAD CUTÁNEA Y PRURITO
d. EL DIAGNÓSTICO SE BASA EN LAS MANIFESTACIONES CLÍNICAS

10. LA FASCITIS NECROTIZANTE, SE CARACTERIZA POR:


Seleccione una:
a. NO POSEE UNA ELEVADA MORBIMORTALIDAD
b. AFECTAR A LA EPIDERMIS
c. INFECCIÓN AGUDA Y RÁPIDAMENTE PROGRESIVA DEL TEJIDO CELULAR
SUBCUTÁNEO
d. LA LOCALIZACIÓN MÁS FRECUENTE ES EL TÓRAX

11. Señale lo verdadero del IMPÉTIGO CONTAGIOSO


a) La bacteria causal es la pseudomona
b) se acompaña de fiebre
c) la lesión elemental es el nódulo
d) una característica es la costra melisérica

12. Señale la respuesta incorrecta


a) En un cuadro eritrodérmico es raro encontrar manifestaciones generales,
como fiebre, escalofríos, trastornos digestivos y adenopatías
b) a Eritrodermia es la forma más grave de la dermatitis atópica
c) La eritrodermia se acompaña de prurito, exudación y descamación
d) El cuadro eritrodérmico se presenta con un enrojecimiento generalizado de la
piel.

13. En cuanto a las manifestaciones clínicas de la Dermatitis atópica señale lo


incorrecto
a) La piel seca o xerosis es un hallazgo casi constante en los pacientes con
dermatitis atópica de todas las edades
b) El prurigo es una erupción de pápulas muy pruriginosas, que como
consecuencia del rascamiento evolucionan a excoriaciones y ulceraciones
con distribución predominante en cara
c) La xerosis causa también el pliegue o línea de Dennie-Morgan, un pliegue
doble y a veces varios paralelos al pliegue palpebral inferior
d) En los niños mayores y los adolescentes se observa xerosis acompañada de
hiperqueratosis en la desembocadura de los folículos pilosos (hiperqueratosis
folicular).

14. SEÑALE LO VERDADERO SOBRE LA DERMATITIS DE CONTACTO


IRRITATIVA
a) NO ES LA FORMA MÁS COMÚN DE ENFERMEDAD CUTÁNEA
OCUPACIONAL
b) DENTRO DEL TRATAMIENTO NO SE DEBERÍA EVITAR LA
REEXPOSICIÓN
c) SON IRRITANTES: JABONES, DETERGENTES, ÁCIDOS, PLANTAS
d) LOS PIES SON A MENUDO LOS MÁS AFECTADOS
15. SELECCIONE LA OPCIÓN QUE MEJOR DEFINA EL SIGUIENTE CUADRO
CLÍNICO: PACIENTE DE 5 AÑOS DE EDAD, PRESENTA EN EL TERCIO DISTAL
DE LAS PLANTAS Y LA CARA PLANTAR DE LOS DEDOS ERITEMA, DE
ASPECTO BRILLANTE Y APERGAMINADO, NO EXUDATIVO, CON ALGUNAS
FISURAS.
a) NINGUNA DE LAS ANTERIORES
b) TIÑA PEDIS
c) PSORIASIS
d) DERMATITIS PLANTAR JUVENIL

16. SEÑALE LO FALSO


a) EL VIRUS HPV TIENE POTENCIAL ONCOGÉNICO EN EL CÉRVIX
b) LAS PICADURAS DE PULGA SE LOCALIZAN EN ÁREAS EXPUESTAS
c) EL SÍNTOMA MÁS FRECUENTE DEL HERPES ZOSTER ES EL DOLOR
d) LA IVERMECTINA ES UN ANTIPARASITICIDA

17. SON MANIFESTACIONES DE LA DERMATITIS ATÓPICA


a) QUERATOTOSIS FOLICULAR
b) PITIRIASIS ALBA
c) XEROSIS EXTREMA
d) TODAS LAS ANTERIORES

18. LA TÉCNICA DE KOH ES ESPECIALMENTE ÚTIL EN EL DIAGNÓSTICO DE


a) LUPUS CUTANEO
b) ALOPECIA AREATA
c) TIÑA CAPITIS
d) TRICOTILOMANIA

19. Señale lo CORRECTO DE LAS ENFERMEDADES CAUSADAS POR VIRUS


a) Las verrugas son de origen bacteriano
b) EL HERPES GENITAL SIEMPRE ES DE TRANSMISIÓN SEXUAL
c) El papiloma virus no afecta a los genitales
d) EL TRATAMIENTO DE ELECCIÓN DEL HERPES ZOSTER ES EL
VALACICLOVIR

20. EL PARÁSITO CUYAS LESIONES SE LOCALIZAN PREFERENTEMENTE EN


LOS ESPACIOS INTERDIGITALES DE LOS PIES, REGIONES SUB Y PERI
UNGUEAL, DORSO DE PIE Y TOBILLO ES:

a) GARRAPATA
b) DEMODEX FOLICULORUM
c) IXODIDAE
d) TUNGA PENETRANS
21. NIÑO DE 11 AÑOS QUE CONSULTA POR UNA LESIÓN ERITEMATO
EXUDATIVA CUBIERTA DE COSTRA MELISÉRICA POSTERIOR A SÍNDROME
GRIPAL CUAL ES EL DIAGNÓSTICO MÁS PROBABLE DE ESTA LESIÓN
Seleccione una:
a. IMPETIGO CONTAGIOSO
b. DERMATITIS ATOPICA
c. ERISIPELA
d. ECTIMA

22. CUAL DE LOS SIGUIENTES NO ES NECESARIO PARA EL DIAGNÓSTICO DE


DERMATITIS ATÓPICA
Seleccione una:
a. CRITERIOS CLÍNICOS MENORES
b. PRUEBAS ALERGICAS
c. BIOPSIA DE PIEL
d. CRITERIOS CLÍNICOS MAYORES

23. Señale lo verdadero de la ESCABIOSIS


Seleccione una:
a.Se acompaña de prurito diurno
b. La lesión elemental son papulas discretas pareadas
c. La bacteria causal es la borelia
d.Una característica es la afectación de la cara en adultos
24. La Dermatitis atópica Infantil se caracteriza por: Señale lo correcto
Seleccione una:
a. En esta fase es más fácil ver lesiones eccematosas con vesículas, pero el

-intenso prurito hace que enseguida se transformen en erosiones, con


exudación
y formación de costras.
b. Es poco frecuente encontrar lesiones en las 􀃖exuras, en especial en los
codos
y las rodillas.
c. Este periodo suele considerarse con un inicio hacia los cuatro años y un
􀃖nal
entre los siete años y la pubertad.
d. Las lesiones características se observan sobre todo en la cara, respetando
las
zonas alrededor de los ojos, la nariz y la boca

25. ¿CUÁL DE LOS SIGUIENTES FÁRMACOS NO ESTÁ INDICADO EN EL


TRATAMIENTO DE LA TIÑA CAPITIS?
Seleccione una:
a. FLUCONAZOL
b. NISTATINA
c. GRISEOFULVINA
d. ITRACONAZOL
e. TERBINAFINA
RESPECTO AL ABSCESO SUBCUTÁNEO, SEÑALE EL LITERAL
CORRECTO:
Seleccione una:
a. SE MANIFIESTA COMO UN NÓDULO FIRME, ERITEMATOSO Y
DOLOROSO
b. LAS LOCALIZACIONES MENOS FRECUENTES EN NIÑOS SON: MAMA,
ZONA PERIRECTAL, GLÁNDULAS SUDORÍPARAS Y CUERO
CABELLUDO.
c. LAS BACTERIAS MÁS FRECUENTEMENTE IMPLICADAS SON
ANAEROBIAS
d. EL TRATAMIENTO NO DEBE CONSIDERAR EL DRENAJE LOCAL

LAS INFECCIONES DE PIEL Y TEJIDOS BLANDOS SE DEFINEN SEGÚN


LA LOCALIZACIÓN DE LAS MISMAS, DEL SIGUIENTE MODO:
Seleccione una:
a. INFECCIONES DE PIEL: AFECTAN A LA EPIDERMIS, DERMIS O TCS.
b. NINGNA DE LAS ANTERIORES
c. A y B SON CORRECTAS
d. INFECCIONES DE PARTES BLANDAS AFECTAN A: FASCIA
PROFUNDA O AL MÚSCULO

SEÑALE LO CORRECTO DEL ACNE

a. LOS CORTICOIDES VIA ORAL SE USAN EN TODOS LOS CASOS


b.
EL ACNÉ SE CONTAGIA POR MANIPULACIÓN DE LAS LESIONES

- c. EL TRATAMIENTO INCLUYE QUERATOLÍTICOS Y ANTIBIÓTICOS


TÓPICOS
d. EL ACNÉ SE CARACTERIZA POR UNA PIEL SECA POR DISMINUCIÓN
DEL SEBO
Respuesta correcta

LA PITIRIASIS VERSICOLOR ES PRODUCIDA POR:


Seleccione una:
a. HONGO LEVADURIFORME Y LIPOFÍLICO
b. HONGOS FILAMENTOSOS
c. HONGO LEVADURIFORME Y QUERATINOFÍLICO
d. HONGOS DE LAS ESPÉCIES DE CÁNDIDA
e. POR MOHOS.
La Dermatitis atópica Infantil se caracteriza por: Señale lo correcto

a. Es poco frecuente encontrar lesiones en las flexuras, en especial en los


codos y las rodillas.
b. En esta fase es más fácil ver lesiones eccematosas con vesículas, pero el
intenso prurito hace que enseguida se transformen en erosiones, con exudación y
formación de costras.
c. Este periodo suele considerarse con un inicio hacia los cuatro años y un
􀃖nal entre los siete años y la pubertad.
d. Las lesiones características se observan sobre todo en la cara, respetando
las zonas alrededor de los ojos, la nariz y la boca

¿CUÁL DE LOS SIGUIENTES HONGOS SE CONSIDERAN COMO


PATÓGENOS PRIMARIOS EN LAS INFECCIONES FÚNGICAS DE LAS UÑAS?
Seleccione una:
a. DERMATOFITOS
b. ALTERNARIA
c. ASPERGILLUS
d. MOHOS
e. CÂNDIDAS

TODOS SON FACTORES QUE PUEDEN ALTERAR EL EQUILIBRIO


ENTRE MICROORGANISMO Y HUÉSPED Y FAVORECER
INFECCIONESCUTÁNEAS, EXCEPTO:
Seleccione una:
a. INMUNOSUPRESIÓN O ENFERMEDADES
b. NINGUNA DE LAS ANTERIORES
c. AUMENTO DE TEMPERATURA
d. HUMEDAD
Respuesta incorrecta.
La respuesta correcta es:
NINGUNA DE LAS ANTERIORES

VARÓN 34 AÑOS PRESENTA LESIONES GENERALIZADAS POSTERIOR


A INFECCIÓN HERPÉTICA, INDIQUE QUE LESIÓN CLÍNICA SUGIERE
ELDIAGNÓSTICO DE ERITEMA POLIMORFO, TAMBIÉN LLAMADO ERITEMA
EXUDATIVO MULTIFORME

a. LESIONES BLANQUECINAS LINEALES EN FORMA DE RED EN


MUCOSA
b. MANCHAS ERITEMATOSAS Y PIGMENTADAS DE DISPOSICIÓN
IRREGULAR
c. LESIONES ERITEMATO-ESCAMOSAS DE LOCALIZACIÓN EN
GRANDES PLIEGUE

-
DIANA
d. MACULAS REDONDEADAS ERITEMATO-VIOLÁCEAS, EN FORMA DE

SON INFECCIONES LOCALES O PRIMARIAS:

a. IMPÉTIGO
b. HIDROSADENITIS
c. FOLICULITIS
d. TODAS LAS ANTERIORES

Señale lo verdadero del IMPÉTIGO CONTAGIOSO

a. La lesión elemental es el nódulo


b. Se acompaña de fiebre
c. Una característica es la costra melisérica
d. La bacteria causal es la pseudomona
Respuesta correcta

' R,.,,b óo, t,,: X f.) (,ooq,(Jr X • l l()Ull)()~Y X @ Wh..,sApp X ;' !ARM,,(O'l' X QI PRUEBA Df.lW X

ID ii https.://lms-fcsee.ute.edu.ec/mod/quil/review.php' attempt :- 126787&cmid :- 127496 ... 8 * :!: 11\ ID 9

---------
Navegación por el cuestionario
Jinal lu,do.,. ~28de,_,.,bt•de2'020,12:12
flolmpo t mpk,Mo \1flWIUtOSS6M:Cl,H'l(los

Pregunta 1
c,lilkKl6n 2.«I de ,t,00 (IOli)

- - . " ' - - 0 . 0 0 ........ ,,00


•"
RESPECTO Al ABSCESO SUBCUTÁNEO, StJ.lAL.E El UTUW.. CORRECTO:

-- .. lJtS 1.0CAUV.CIONES MENOS FRECUENTES EN NIÑOS SON: MAMA. ZONA PERI RECTAL GLÁNOUU.S SUOOftlPAAAS VOJEAOCA8ELLUOO.
b. SE MANIAESTA COMO UN NÓDULO FIRME. ERITtMATOSOYOOLOROSO
c. lAS BACTERIAS MÁS FRt:CUENTEMENTE IMPLICADAS SON ANAEROBIAS X
d. El TAATAMIENTO NOOEBECONSIOtJWl:fl DRENAJE LOCAL

~lnCorrec..

L1I rnpunt, co,re,;u, ne SE MAHIFl(STACOl.40UN NÓOUl.0 FIRME. El!ITEIAAT050Y DOt.OltOSO

Pregunta 2 f MlrUrpr. .un l ..

--
EN RELACIÓN CON EL CARCINOMA 8ASOCf.LULAA. lCUÁL OE lAS StGU IENTES .-SIRMACIONES ES FALSA?:

•. PUEDE APARECER EN AAfAS NO FOTOEX.PUESlAS

b, NUNCAOCASIONAMETASTA.SIS
A vrWnows
C. PUEDf: SERPIGMENTAOO X

d. NO SE DESARROllA EN lAS MUCOSAS

■ P Escribe aquí para buscar O 1:11


" Mi 121
G) PRUEBADEAA· X

e Gt ID Q ... * ! 11\ ('[) 8

---
h1tps://1ms-k~ ute.edu.ec/mod/quil/1evil'W.php?at1empt- 126787&cm1d. 1274% ... 0

Urnpul'<UIC!ll'l'«uK:kUNCAOCA.SIO~l,,ln-'Sl"'5"5

Pregunta3

MODELO DE 27 AÑOS DE [OAO QUE PRESENTA PlACAS UUTEMATO ESCAMOSAS EH ZONA OfLAAETf V~ 0[8AI() OH OM8llGO. ESTA UlTIMA EN FORMA CIACUlAR DE MONEDA.

--
H ALERGtNO MÁS PR06A8t.E ES

•- NIQUEV.OOS YCROMADOS ./
b.FAAG.UICV.S

d.1.Attx

Pragunt.a4

VARÓN 34 AÑOS PRESENTA LESIONES Gft.ERAUZAOA.S POSTUUOR A INFECCIÓN HE~TICA. INOIQUE QUE LESIÓN CLINtCA SUGIERE El DIAGNÓSTICO DE ERITEMA POUMOIIFO.
TAMBltN UAMAOO EJUl[MA EXUOATIVO MULTlroAME

•· lfSIONES 81..ANQUEONAS UP..EAUS EN FOflMA DE Rf.D EN MUCOSA

b. us,ot,¡ES ERITEMATo-ESCAMOSAS DE LOCALIZACIÓN EN GRANDES PUEGUE

t. MA.NCW.S EAITIMATOSAS Y PIGM[NT"°"5 DE DISPOSICIÓN IRREGIJIJ.R )(

d MACUI.AS Rf.OONDEADM fRITIMATO-VIOIJ.cfAS. EN FORMA DE DIANA

---
... ,.., ~e r«utt::M.tCVI.AS l)ONl)(AO,t,,S(lt'l(MATOv,oL,l,clAS,INFOlt\V.l)(Dl,,t,NA
A W

o
• P Escribe aquí para buscar Ql
(¡il PRUEBA DEIW X

ID ii https://Tms-fcsee.ute.edu.ec/mod/quil/review.php?attempt .· 126787&cmid: 127496 ... 0 * ! 11\ ('[) 8


U , t ~ l ( O t t ~ t ' f MA(Vt.ASlll:OONOCAOA,SEIUT(MAT().Vl()l).(EAS, (NFQAAl,\.OE ~

Pregunu1S f Man:;o,pr....,.11

--
Sfofia!tLilrtsJJU~incorrtcta

• . Enuncuac1rot1itroc,ffm,¡c:0Hrarol'f'l(onttl!Jilmanlf~8ff'M!falH.comofltbrt.Ho:alofrlM. tr.ntOl'T'I01ICltSHlMMyadfflOpa!IM

b.L.aEmroclfflni,atslafotm.m&sarawMladttmalltlillópK.a

(.Elcuaodro«iU~o~prl!Vrucot1unMrOj,ttilniHKO¡Mll'allladodflaptfl. X

d. L.aeritrod"'1Tlll~Komp,t'widfprur1to,exOO,,Cl6nydKUlm«iOn

~ -r~.
U•~corr~K:Enun~roerftrocH-nnoc:oinr•roen:ontrMrTWn,t~i.oonncer,er.in.cornoflotbre.~lnos. tr•~nosdoentM>s'f~

Pregunta6

SE..iALE LO VEROADERO SOBRE LA OERMAffllS OE CONTACTO IRRITATIVA

Sttt,;:oor,e...,..·

._ LOS PIES SON A MENUDO LOS MÁS AFECTADOS


b. NO ES LA fORMA MÁS COMÚN CE fNr:t:llMEOAOCVTÁNEA OCUPAOONAl
(. SON IRRITANTES:JA80NES. DETE RGENTES. ÁCIOOS. PLAHTAS

d. DENTRO OfL TRATAMIENTO NO SE OEBERlA fVITAA LA RE EXPOSICIÓN

---
UftipUtit,ICOfT~K:S<m IIIRITNHIS.:~B, DlTUtGE"'1"B,ÁCIDOS, PI.ANTAS

AvrWnow
Pregunta7
o
■ P Escribe aquí para buscar O 1:11 " lii,j¡ IIl ra¡¡
' Re< O d<, 6- X 8 boo<jlt ( JI<' X • l l (}Ul[)(J', V X (9 w.-.., s,',.¡_,p X ? fAl{MA( O 'l' X (w PRUEBADEAA' X

(9 a https://lms-fcsee ute.edu.ectmod/quiztreview.php?auemp1, 126787&cmid, 127496 ... 0 * ! 11\ I![) 8

Prvgunt.1 7

MUJER DE 45 AÑOS QUE PRESENTA FOTOSENSIBIUOAO. AATAALGIAS. ULCERAS ORALES EL DIAGNÓSTICO MÁS PROBABtE ES

._ ESCLfwot:RMIA

b.UQUENPI.ANO

º""""""'
d. LUPUS ERITEMATOSO SISTEMICO .,/

llflpu,KYCotHCUI

U~correcun:lUPVS(Rff(IAATOSOSIST(i.1,CO

Pregunt.1 8

SE~AlE LO CORRECTO DEL ACNE

A. EL TRATAMIENTO INCUJVE QUERATOÚTlCOS Y ANTlSIÓTICOS TÓPKOS V

EL ACN( !>E CONTAGIA POR MANJPUI.ACIÓN Of: LAS LESIONES

,.
EL ACN( !>E CARACTERIZA POR UNA PIEL SECA POfl DISMINUCIÓN DEL !>EBO

-·~
u respue-n, correcu n: Et TII.ATAAllENTO •NCUJV( QU(ltATOl.iTICOS V ANTIBIÓTICOS TÓPICOS AvrWnow

o
PrH unt.1 9
• P Escribe aquí para buscar 111
• ¡.,:,.,,o úo, t~ X Ea G<xx¡ t (J.. X • ll(}Ull)()', V X (9 WMJ sApp X ? !Al{M,;( () 'l' X (w PRUEBADEAA' X

(9 a h1tps://lms-fcsee.ute.edu.ec/mod/quiZ/review.php?attemp1 .· 126787&cmid: 127496 ... 0 * ! 11\ I![) 8

Pregunt.19 f Maturpnaunta

El SINDfl:OME DE RAMSAV-HUNT. OESCRJBE LA ASOCIACIÓN DE

Selecoone,..,.·
._pAAAl.JslSFA.CIAl.VHERPESSIMPUOAAL

b. PAAALJSIS FACIAL Y HERPES ZOSTEA AURICUI..AA .

c. HEMlPARESIA COMPLETA Y HERPES20STER


d. PAIW.JSIS FACIAL Y HERPES ZOSTER ORAL

U~Cotrl!CUH"PAA,Ó,USISF...CW.Y!-IEltPES20STER.-.uRIC\.II.AR

Pregunt• 10 y warcar.........,u

¿CUÁL ES LA INF'ECCIÓN OE LA EPIDERMIS MÁS FRECUENTE7

._ERISIPELA

b.lMf>tnGo

dFASC1TISNECAOTIZANTE

AvrWnow

o
■ P Escribe aquí para buscar O 1:11

CARDIO

Usted se desempeña como epidemiólogo en cardiología en un comité internacional


que plantea normas de salud global. El comité le asigna a usted hacer análisis de las
tendencias sanitarias en una nación del Este Asiático y Pacífico, con una población y
un entorno muy similares a los de China. En este punto de la historia, la nación está
por cambiar de la primera fase clásica en la transición epidemiológica (“peste y
hambre”) a la segunda etapa clásica (“retroceso de las pandemias”). En lo que se
refiere a los perfiles previstos de enfermedad cardiovascular (CVD) en esa nación:
¿cuál de los siguientes planteamientos es verdadero?
C. En la nación en cuestión, cuando alcanza el máximo de mortalidad por CVD, sería
de esperarse que la mortalidad por apoplejía sea más elevada que la correspondiente
a cardiopatías coronarias.

Usted labora en una clínica sanitaria rural del Norte de la India. Se valora a un niño
de ocho años que nunca ha sido atendido por un médico. Su madre indica que él no
puede desempeñar la misma actividad física de sus coetáneos. En la exploración
inicial de la piel usted detecta hipocratismo digital y cianosis en los pies, pero las
manos tienen aspecto normal. Sin más exploraciones: ¿cuál de las siguientes
anomalías congénitas sospecha usted en el niño?

C. Persistencia del conducto arterioso con hipertensión secundaria de la pulmonar.

Una mujer afroestadounidense de 55 años sin antecedentes patológicos de


importancia, fue llevada al departamento de urgencias por un síncope; en la figura V-
3A se muestra el electrocardiográficos (ECG). También se observan nódulos
subcutáneos dolorosos en sus piernas (semejantes a los que se muestran en la Figura
V-3B). Se sospecha que las anomalías del corazón y de la piel se deben
predominantemente a:

C. Sarcoidosis.

Se valoró al Sr. Estevez de 67 años, propietario de una cadena de restaurantes en


que se sirve sushi con que ha tenido mucho éxito. Señala falta de aire con el ejercicio
(disnea), edema de extremidades inferiores e informó que despertaba por la noche
con la sensación apremiante de falta de aire. Se intentó valorar su estado volumétrico
y se sabe que el índice de la exploración física más importante para facilitar al médico
el conocimiento de este componente es la medición del pulso venoso yugular (JVP).
De los planteamientos siguientes respecto a la medición de JVP: ¿cuál es verdadero?

E. Las pulsaciones venosas por arriba de la clavícula con la persona en posición


sedente son anormales.

De los planteamientos siguientes respecto a las mediciones de la presión arterial:


¿cuál es verdadero?

A. La tensión sistólica aumenta y la diastólica disminuye cuando se le mide en las


zonas más distales de arterias.

Un varón de 75 años acudió al departamento de urgencias, por un padecimiento


grave. Sus familiares indicaron que los últimos seis meses perdió su energía normal
y que ha estado confuso y letárgico los últimos dos días. Conforme se recaban datos
del interrogatorio entre los parientes, se palpa el pulso radial del enfermo y se percibe
una variabilidad de latido a latido constante, en la amplitud del pulso, aunque el ritmo
es regular. En consecuencia, más tarde usted mide la presión arterial y advierte que
es audible únicamente el tercer ruido de Korotko de fase I (sistólico) conforme
disminuye la tensión en el esfigmomanómetro y el manguito y que constituye un
fenómeno independiente del ciclo respiratorio. Con base en tal situación: ¿cuál de las
entidades patológicas siguientes tiene el paciente?

E. Disfunción grave de ventrículo izquierdo.

Un varón de 78 años fue internado en la unidad de cuidados intensivos por


insuficiencia cardiaca descompensada. Desde tiempo atrás había tenido
miocardiopatía isquémica. Los trazos ECG indican fibrilación auricular y bloqueo de
rama izquierda del haz de His. En la radiografía de tórax se detectan cardiomegalia e
infiltrados alveolares bilaterales con línea B de Kerley. De los signos: ¿cuál es el que
menos posibilidad tiene de aparecer en la exploración física?

A. Cuarto ruido cardiaco.

Un varón de 24 años fue referido al cardiólogo después de una crisis sincopal mientras
jugaba basquetbol. No recuerda lo que ocurrió, pero se le dijo que presentó un colapso
mientras corría. Recuperó el conocimiento acostado en el piso y con múltiples
contusiones como consecuencia de la caída. Siempre fue una persona activa, pero
en fecha reciente presentó moderado dolor retroesternal con el ejercicio, que le obligó
a restringir sus actividades. Su padre falleció a los 44 años mientras practicaba
alpinismo. Piensa que la causa de la muerte de su progenitor fue repentina y de origen
cardiaco y recuerda que él le indicó que tenía el corazón muy grande. En la
exploración, el paciente tiene un soplo III/VI en crescendo-decrescendo mesosistólico.
Los trazos ECG indican signos de hipertrofia de ventrículo izquierdo. Se sospecha
miocardiopatía hipertrófica. De las maniobras siguientes: ¿cuál sería la que causa el
incremento en la intensidad del soplo?

A. Ejercicio de cerrar el puño.


B. Posición en cuclillas.
C. Bipedestación.
D. Maniobra de Valsalva (pujar).

-
E. A y B.
F. C y D.

Una mujer de 75 años con metástasis de cáncer pulmonar no microcítico fue internada
en la unidad de cuidados intensivos con tensión sistólica de 73/52 mmHg. Señaló al
acudir a solicitar atención, que en los últimos 3-5 días tuvo fatiga y disnea cada vez
peores. En la exploración física se advirtió distensión de las venas del cuello. En la
radiografía de tórax se detectó que la sombra cardiaca es masiva y tiene la forma de
un “botellón de agua”, pero no tiene nuevos infiltrados pulmonares. De los hallazgos
adicionales siguientes: ¿cuál tiene mayor posibilidad de ser detectado en la
exploración física?

A. Disminución de la tensión sistólica >10 mmHg con la inspiración.

De los tratamientos siguientes respecto a los perfiles de despolarización normales del


corazón: ¿cuál o cuáles son verdaderos?

B. El orden normal de despolarización es el siguiente: nudo sinoauricular (SA) -


miocardio auricular - nudo AV - haz de His - fibras de Purkinje - miocardio ventricular.

Todas las ondas ECG siguientes concuerdan correctamente con el ciclo cardiaco que
representan, EXCEPTO:

A. Onda P - repolarización auricular.

De los vectores QRS medios siguientes en el plano electrocardiográfico frontal: ¿cuál


concuerda apropiadamente con su designación?

A. -20 grados - eje normal.

De las anomalías siguientes: ¿cuál está representada en los trazos ECG de la Figura
V-13?

27/4/2020

g
1 11 111 VR VL VF

-' ._
i
I+
i

1 1 1
I 1'
y ¡
• l 1
V
- ,. 1

1
' >-
1 t
1
1 1
1'
1
1 1
'
1
1 1
-+--- 1 -,-
~- 1
i .
l'\ . """" 1 1
'' 1 - 1
......
• 1
1 1 1 .
~ : Oennia L. ~per~ Anthony S- Fauci, Stephen L. Haw,er, Dan L.. 1.0ns,o, J. L.any meaon, Josc ph ~ :
41.1toev.Ju.oón y~~ 1 ~ HDrnsot\. Pn,nap;as d e ~ 1~ OeN!!Chca O McGnrw--H,11 Education..
Oere.chu ■ Resenada..
View FtJU S,ze Favorite Fi

+- -
1 1 - - - -- VR VL VF
--- r-- -t- H,_ -
y
1
Ff' - 1
'
1
'--- -

1

1
,l. ' --,--¡ --
11\
1 - ,r-,...
,-
1
1
- ~¡...._-
1
-- ,- 1-1-
......
1
1 . i
'
D. Bloqueo de la rama derecha del haz de His.
De las opciones siguientes: ¿cuál describe el hallazgo primario en el trazo ECG que
se muestra en la Figura V-14?

Fvente O . . . L ~, ~ S. FIIUO,~Ltt-...,;0.11 L."°"90,l. L . - ' I V ~ - . ~:


4utoevM.l«lldrny~ l9e. HMnsOff.. ~ d e ~ l m w - . ~ O ~·Htll !duc:atioft. DeredlmltHtt-,,.doa.,
\l\ew HJII Siiej Favoote F1gurelOownload Slide (.pptl

B. ECG normal.

GUISHO
Un trabajador de la construcción de 56 años, con hipertensión y antecedente de abuso
de tabaco acudió al departamento de urgencias porque 30 minutos antes presentó
náusea aguda, disnea y sensación de opresión precordial. Sus trazos ECG iniciales
se presentan en la Figura V-15. Todos los trazos siguientes aparecen en este ECG,
EXCEPTO:
••-•• • ........ •--1 • .........._ • 'b".. ~I .............................. _ ,.,.-,.. .. ,
1 ■VA V

A. Isquemia miocárdica en la cara inferior.


B. Ondas P.
C. Isquemia miocárdica en la cara posterior.
D. Taquicardia sinusal.
E. Taquicardia ventricular

Un trabajador de la construcción de 56 años, con hipertensión y antecedente de abuso


de tabaco acudió al departamento de urgencias porque 30 minutos antes presentó
náusea aguda, disnea y sensación de opresión precordial. Sus trazos ECG iniciales
se presentan en la Figura V-15.

¿Cuál de las siguientes arterias coronarias muestra la mayor posibilidad de estar


ocluida?
A. Primera rama perforante septal.
B. Segmento proximal de la arteria descendente anterior izquierda.
C. Segmento proximal de la arteria circunfleja izquierda.
D. Segmento proximal de las arterias descendente anterior y circunfleja izquierdas.
E. Arteria coronaria derecha.

Mujer de 48 años que acudió a la clínica de atención primaria en que usted trabaja
para que se le practique una valoración inicial después de haber viajado por todo el
país. No tiene registros médicos aunque insiste que los envió por correo una semana
antes. Indica que ha tenido algunos “problemas del corazón”, pero no explica detalles.
Así mismo, comenta que consume tabletas contra el “colesterol y la presión arterial”.
El trazo ECG inicial se presenta en la Figura V-17. De los planteamientos siguientes
respecto a su ECG: ¿cuál es verdadero?
A. Es posible que antes haya tenido un infarto del miocardio.

Se le solicita a usted que valore a un residente de medicina interna de 27 años, que


señala durante una semana haber tenido tos, coriza y febrícula. En el día de hoy ha
presentado una molestia retroesternal cada vez más intensa en su permanencia en
la clínica. Percibe que el dolor se intensifica más cuando respira hondamente. Usted
realiza un ECG estándar de 12 derivaciones (Figura V-18). En la exploración física,
su presión arterial es normal, están afebril y no hay elevación del pulso venoso
yugular. Sin embargo, muestra incomodidad leve por el dolor del tórax. ¿Cuál de las
medidas siguientes sería la más apropiada?
D. Administrar ibuprofeno y colchicina

Una gran tormenta de nieve azotó la zona en que usted vive el martes y fue imposible
transitar de las carreteras hasta el lunes siguiente. En la mañana de ese día un varón
de 48 años fue llevado por los servicios médicos (EMS) al departamento de urgencias
después de que un vecino le halló con alteración del estado mental. Estaba levemente
obnubilado y no podía aportar datos. Usted observó que la arteria humeral izquierda
tenía una fístula de hemodiálisis. Su ECG inicial se muestra en la Figura V-19. De las
anomalías siguientes de electrólitos: ¿cuáles probablemente presentaría este
paciente?
B. Hiperpotasemia.

Varón de 66 años internado en un hospital por disnea de esfuerzo progresiva y fatiga.


Entre sus antecedentes tiene el consumo de tabaco, que viajaba extensamente y que
en fecha reciente volvió de visitar múltiples países en América del Sur. Al ser atendido
por primera vez su frecuencia cardiaca era de 104 lpm (lpm), totalmente irregulares.
Su presión arterial es de 96/76 mmHg. Se destaca pulso venoso yugular amplio y
notable edema de extremidades inferiores. En el ecocardiograma se detecta una
fracción de expulsión ventricular izquierda de 55% y en la Figura V-20 se incluyen
imágenes fijas de esa modalidad diagnóstica. Para dilucidar el origen de su
insuficiencia cardiaca: ¿cuál de los estudios diagnósticos es el apropiado para
realizarse a continuación?
E. Electroforesis de proteínas en suero y orina y cuantificación de cadenas ligeras.

La Sra. Jackson es una estadounidense de raza negra de 45 años que abusaba del
tabaco, tenía cáncer de mama (estado actual: posoperatorio de mastectomía y
radioterapia) y alergia a mariscos. Dos semanas antes acudió a la consulta de
cardiología en que usted trabaja y señaló que en los últimos seis meses tuvo disnea
mientras ascendía una colina cerca de su casa. Nunca había tenido dificultad con tal
ascenso antes. Junto con la disnea, presentó náusea vaga y abundante diaforesis.
Los trazos de ECG con ella en reposo y la exploración física aportaron datos sin
trascendencia. Usted la refirió para la práctica de tomografía computarizada por
emisión de fotón único durante ejercicio (SPECT; single-photon emission computed
tomography) y una gammagrafía de riego del miocardio con 99m tecnecio sestamibi
(99mTc) y recibió los resultados que se muestran en la Figura V-21. Inmediatamente
usted se percata de que la imagen indica isquemia reversible. ¿Qué territorio arterial
muy probablemente intervino en el gammagrama del estudio?
A. Arteria descendente anterior izquierda.

El nudo SA actúa como un marcapaso dominante del corazón y genera ritmo sinusal
normal. ¿Qué propiedad de dicho nudo permite a las células actuar con esa función
primaria?
E. Despolarización espontánea durante la fase 4 del potencial de acción, con mucha
mayor rapidez que la observada con otros miocardiocitos.

El Sr. Enríquez es un sommelier de 21 años en un conocido restorán de la ciudad. Su


diversión primaria es el ciclismo competitivo y la semana próxima espera participar de
una carrera de 150 km, para la cual estuvo entrenando en los últimos seis meses. Los
coordinadores de ese evento exigen a todos los competidores someterse a una
valoración cardiovascular integral antes de la competencia y con ese motivo acude a
la clínica en que usted trabaja. En la valoración que usted hace, percibe que la
frecuencia cardiaca con el sujeto en reposo es de 45 lpm con una pausa ocasional de
dos segundos, incluso. Su presión arterial es de 108/72 mmHg. Se siente bien y no
señala episodios sincopales o presincopales en el reposo o durante el ejercicio. Salvo
la bradicardia, usted no detecta otras anomalías en su exploración. Su trazo ECG
indica ritmo sinusal con un intervalo PR de 128 mseg, duración de QRS, 80 mseg y
pausas ocasionales que duran incluso 2.2 segundos. De las recomendaciones
siguientes: ¿cuál sería la más apropiada para el Sr. Enríquez?
C. No se necesita más estudios de vigilancia y que tenga buena suerte con la carrera.

Un varón de 60 años es sometido a un estudio electrofisiológico por el antecedente


de síncope. Después de canulación cuidadosa de una vena y colocación de catéteres
de conductancia y estimulación y después de administrar 0.2 mg de propranolol/kg de
peso y 0.04 mg de atropina/kg de peso, su frecuencia cardiaca es de 65 lpm. Después
de interrumpir el uso de los fármacos y que transcurra tiempo suficiente para su
eliminación, se estimula las porciones superior/lateral de la aurícula derecha a razón
de 140 lpm. Una vez que se interrumpe la estimulación externa, el siguiente latido
sinusal aparece 1 800 mseg después. Con base en las observaciones anteriores:
¿cuál de los diagnósticos siguientes se puede asignar al paciente?
D. Enfermedad del nudo SA.

Todas las siguientes entidades son causas reversibles de disfunción del nudo SA,
EXCEPTO:
E. Radioterapia.
¿Cuál de las siguientes entidades constituye un factor de riesgo para que surja
tromboembolia en pacientes con la variante de taquicardia/bradicardia y síndrome de
seno enfermo?
B. Auriculomegalia.
Las células normales dentro del nudo AV presentan una propiedad conocida como
conducción en decremento. Si usted desea demostrar esta propiedad durante un
estudio electrofisiológico: ¿qué maniobra practicaría?
A. Estimularía la aurícula derecha con disminución seriada de la duración del ciclo y
mediría el tiempo de conducción de estímulo-haz de His.

Un varón de 87 años con el antecedente de hipertensión, tratado satisfactoriamente


de ese problema y de estenosis aórtica, comenzó a mostrar síntomas de la estenosis
en los últimos dos meses. Un día antes fue sometido a reemplazo quirúrgico de la
válvula aórtica y colocación de una válvula bioprotésica de 25 mm, con excelentes
resultados transoperatorios. Rápidamente se le separó de la circulación extracorporal
y se extubó en término de 24 horas. Con base en el protocolo quirúrgico
temporalmente se le dejaron los electrodos de estimulación epicárdica en la superficie
ventricular y recibió estímulos a razón de 90 lpm. Al revisar su estado en la mañana,
usted hace una pausa breve de su estimulación ventricular para cuantificar el ritmo
básico. Advierte una frecuencia auricular de 80 lpm pero la frecuencia ventricular es
de 32 lpm con complejo QRS amplio. No se advierte relación entre las ondas P y los
complejos QRS. ¿A cuál de los siguientes factores muy probablemente se debe la
bradicardia ventricular del paciente?
. E. Lesión quirúrgica del nudo AV.

La Sra. Hellwig es una mujer de 25 años con lupus eritematoso sistémico (SLE)
complicado por nefropatía, anemia hemolítica y pleuritis. Su enfermedad ha estado
controlada en forma satisfactoria con tratamiento. En fecha reciente se percató que
estaba embarazada y acudió el día de hoy para orientación prenatal. Le interesa
específicamente el efecto que ejerza su enfermedad autoinmunitaria en el producto
de la concepción. Usted indica que la complicación cardiaca más común en los hijos
de madres con lupus es: A. Bloqueo AV

El Sr. Homan, un antiguo equilibrista, acude a su consultorio y señala que tuvo un


síncope. Afirma que en dos ocasiones de la semana pasada, espontáneamente se
desmayó, sin síntomas premonitorios. Una vez se golpeó la cara y usted advierte que
en la exploración hay equimosis periorbitaria. Salvo ese signo, no hay nada anormal
en la exploración. Solicita la práctica de un ECG y se detiene en la sala adjunta para
comenzar la documentación. Minutos más tarde un asistente le solicita su presencia
urgente en la sala clínica en que está el paciente. Tuvo otro “desvanecimiento”
durante el ECG y quedó inconsciente. Como un hecho casual, el asistente captó el
desvanecimiento en el ECG y se muestra en la Figura V-30. ¿Qué tipo de bloqueo AV
apareció y cuál es la concordancia con el tratamiento o el método diagnóstico
apropiado?
+

E. Bloqueo de nudo AV Mobitz tipo II de segundo grado - Colocación de marcapaso


permanente.

Un corredor de fondo de 19 años, que acabó en el décimo lugar en el maratón local


el año pasado, acudió para someterse a valoración cardiaca por parte de su médico
general y usted ordena la vigilancia con un monitor de Holter con fines de detección
sistemática. En el señalamiento de esa técnica, se advierte que surgieron varios
episodios de bloqueo AV (Wenckebach) Mobitz I de segundo grado, todos durante el
sueño. El paciente no señala síntomas, pero piensa que tuvo un abuelo que tenía un
marcapaso implantado a edad avanzada. ¿Cuál es la medida siguiente más
apropiada?
C. Tranquilización verbal.

Mujer de 47 años con antecedente de tabaquismo intenso y colitis ulcerosa, que es


sometida a valoración para estudiar palpitaciones intermitentes. Indica que los últimos
seis meses cada dos a cuatro días percibía una sensación de que el corazón le
“golpeaba” en el tórax, unos cinco minutos. No percibió factores desencadenantes ni
sintió obnubilación y dolores retroesternales con esos episodios. Los datos de la
exploración física son normales. El ECG de reposo indicó ritmo sinusal y ninguna
anormalidad. Además de medir los electrólitos séricos: ¿cuál es el estudio más
adecuado entre los que mencionaremos?
B. Monitor de crisis

Mujer de 47 años con antecedente de tabaquismo intenso y colitis ulcerosa, que es


sometida a valoración para estudiar palpitaciones intermitentes. Indica que los últimos
seis meses cada dos a cuatro días percibía una sensación de que el corazón le
“golpeaba” en el tórax, unos cinco minutos. No percibió factores desencadenantes ni
sintió obnubilación y dolores retroesternales con esos episodios. Los datos de la
exploración física son normales. El ECG de reposo indicó ritmo sinusal y ninguna
anormalidad. Después de nuevas pruebas esta paciente presentó varios episodios de
extrasístoles auriculares. De los planteamientos siguientes respecto a la arritmia de
la paciente: ¿cuál es verdadero?
D. Se tranquilizará verbalmente a la paciente de que no se trata de un problema
peligroso y no necesita ahondar en las valoraciones.

Mila es una joven de 18 años que está en estudios preparatorios y es estrella del
volibol, con una beca en deportes en la universidad local. Como parte de su proceso
de admisión se le solicita que se someta a una valoración médica completa antes de
participar en deportes en su universidad. En la exploración física no se advierten
anomalías, aunque ella señala ocasionalmente presentar palpitaciones y obnubilación
leve. En el ECG se detectó un intervalo PR de 0.06 mseg, duración de QRS, 140
mseg y un “empastamiento” y onda delta en el segmento inicial de QRS. Usted
diagnostica con certeza que se trata de un perfil de Wol-Parkinson-White. ¿Cuál de
los hallazgos siguientes brinda tranquilización de que Mila no padecerá efectos
nocivos o que necesitará ablación por catéter a causa de su anomalía?
D. Estudio dinámico en banda sin fin que demuestra desaparición de la onda delta y
QRS ancho con la frecuencia cardiaca de 120 lpm.

Mujer de 85 años sin antecedentes de cardiopatía que acudió al departamento de


urgencias por haber mostrado palpitaciones durante dos horas. La presión arterial,
saturación de oxígeno y frecuencia cardiaca son normales, aunque usted percibe un
ritmo muy irregular en la exploración. En el ECG se advierte una onda QRS angosta
y fuertemente irregular y ondas P discernibles, con frecuencia de 75 lpm. En el
ecocardiograma no se identifican cardiopatías estructurales. A pesar de la frecuencia
cardiaca normal, la paciente está consciente de su fibrilación auricular sintomática e
intenta recuperar el ritmo sinusal. Todas las intervenciones siguientes pueden ser
beneficiosas, EXCEPTO: A. Adenosina por vía intravenosa.

Un varón de 79 años con el antecedente de arteriopatía coronaria, miocardiopatía


isquémica con una fracción de expulsión de ventrículo izquierdo (LV) de 30% e
hipertensión, acude al consultorio de usted, sin nuevos síntomas. Su presión arterial
es de 108/56 mmHg, su frecuencia cardiaca es de 88 lpm y la saturación de oxígeno
en sangre arterial es de 98%. La tira del ritmo se señala en la Figura V-36. Con base
en los trazos ECG en la actualidad se percibe que el paciente tiene una indicación
definida (clase I) por alguno de los siguientes tratamientos. Señale el más indicado:

-
D. Anticoagulantes con acción sistémica como la warfarina o un nuevo anticoagulante
ingerible.

Mujer de 43 años atendida en el departamento de urgencias por palpitaciones de


comienzo repentino 30 minutos antes de su consulta. Estaba sentada en su ordenador
cuando comenzaron los síntomas. Salvo la lumbalgia, por lo demás su estado es
satisfactorio. En el área de clasificación del servicio de urgencias, su frecuencia
cardiaca es de 178 lpm y la presión arterial, 98/56 mmHg con saturación normal de
oxígeno. En la exploración física se percibe el “signo de la rana” en su cuello y
taquicardia, pero por lo demás su estado es normal. En el ECG se advierte una
taquicardia con complejo angosto sin onda P identificable. ¿Cuál es la primera medida
más adecuada entre las siguientes para tratar su taquicardia?
D. Masaje del seno carotídeo.

De los planteamientos siguientes respecto a la restauración del ritmo sinusal después


de fibrilación auricular: ¿cuál es verdadero?
B. En personas tratadas con farmacoterapia y en las que se advierte ritmo sinusal,
habrá que utilizar por largo tiempo el monitor Holter para saber si es posible
interrumpir sin consecuencias graves el uso de anticoagulantes.

Mujer de 76 años con antecedente de hipertensión, neumopatía obstructiva crónica


(EPOC), diabetes mellitus y osteoporosis que acudió al departamento de urgencias
después de presentar una caída en su hogar, a la que siguió inmediatamente dolor
intenso en la cadera izquierda. Después de varias horas la descubrió un vecino. Ella
no recuerda si perdió el conocimiento. Presenta un dolor intensísimo a la palpación
en la cadera izquierda y la pierna muestra acortamiento y está rotada hacia afuera.
Sus mucosas están secas y es fácil de estirar la piel “a manera de una tienda de
campaña”. La presión arterial es de 170/80 mmHg y la frecuencia cardiaca es de 130
lpm. La tira del ritmo se presenta en la Figura V-39. ¿Cuál es la primera medida más
importante contra su taquicardia?
E. Control del dolor e hidratación por vía IV.

Una joven es llevada al departamento de urgencias después que algunos testigos


observaron que repentinamente perdió el conocimiento mientras trotaba en un parque
cercano. Presenta fractura de huesos de la nariz y dientes rotos por la caída. No tiene
identificación y reacciona a estímulos dolorosos. Su presión arterial es de
50/palpación y la frecuencia cardiaca se acerca a 280 lpm. En la Figura V-40 se
incluyen los trazos de ECG de 12 derivaciones. ¿Cuál es la siguiente medida más
apropiada?
D. Desfibrilación con DC

Una joven es llevada al departamento de urgencias después que algunos testigos


observaron que repentinamente perdió el conocimiento mientras trotaba en un parque
cercano. Presenta fractura de huesos de la nariz y dientes rotos por la caída. No tiene
identificación y reacciona a estímulos dolorosos. Su presión arterial es de
50/palpación y la frecuencia cardiaca se acerca a 280 lpm. En la Figura V-40 se
incluyen los trazos de ECG de 12 derivaciones.
A. Fibrilación auricular con conducción anterógrada por vías accesorias.

Un varón de 67 años con antecedente de hipertensión e hiperlipidemia acudió 3 h


antes al departamento de urgencias con dolor retroesternal opresivo y disnea de
comienzo agudo. En ese sitio, se observaron en el ECG elevaciones del segmento
ST en las derivaciones anterior y lateral y se administraron trombolíticos. Se le internó
en la unidad de cuidados intensivos cardiacos. La enfermera le pide a usted acudir al
cuarto del paciente, por haber detectado trazos inusuales en el monitor. En la Figura
V-42 se incluye una muestra del ritmo captado en la derivación V6. ¿Qué ritmo
representa?
C. Ritmo idioventricular.

Usted trabaja un turno tranquilo en el departamento de urgencias local. En los últimos


cuatro días en la localidad ha habido una gran tormenta de nieve que ha paralizado
el tránsito y las personas por lo común han permanecido en sus casas, para evitar
accidentes. En forma súbita, el personal médico de urgencias llega con una mujer de
edad madura que muestra obnubilación. Sus vecinos de la planta baja advirtieron que
no la habían visto en los últimos cuatro días (desde que comenzó la tormenta) y
avisaron a la policía, que la halló inconsciente en el piso de su casa. En la valoración
que usted hace apenas si se percibe el pulso radial. Observa usted que hay un injerto
de fístula para diálisis en el brazo izquierdo. Su presión arterial es de 60/palpación y
los trazos ECG se presentan en la Figura V-43. Con rapidez usted dispone lo
necesario para desfibrilación y solicita que se practique una química sanguínea
completa. ¿Qué anormalidad electrolítica espera usted?
B. Hiperpotasemia.

Una mujer de 71 años con miocardiopatía isquémica y fracción de expulsión


ventricular izquierda de 38% estuvo hospitalizada la semana anterior por insuficiencia
cardiaca aguda descompensada. Después de usar diuréticos y optimizar los fármacos
la paciente se siente muchísimo mejor. Recibe las dosis máximas toleradas de
inhibidor de la enzima convertidora de angiotensina (ACE; angiotensin-converting
enzyme), un antagonista β y una dosis apropiada de diurético. Usted planea darla de
alta el día de hoy. En las visitas de revisión la enfermera percibe que la paciente tiene
“trenes” breves (5-10 latidos) de taquicardia ventricular no sostenida (NSVT) y
múltiples extrasístoles ventriculares (PVC) durante la noche, aunque ella permaneció
asintomática. Una estudiante de medicina de los servicios escolares pregunta si la
NSVT es de importancia diagnóstica y si se necesita alguna intervención. ¿Cuál es la
respuesta más apropiada?
E. “NSVT se acompaña de mayor mortalidad en pacientes en insuficiencia cardiaca.
Sin embargo, la supresión de las extrasístoles ventriculares y NSVT con antiarrítmicos
no cambia el pronóstico”

Usted atiende al Sr. Justino en la unidad de cuidados intensivos cardiacos. Se trata


de un varón de 62 años con hipertensión, hiperlipidemia y tabaquismo intenso que
cuatro días antes había padecido un infarto masivo en la porción anterior del
miocardio. Acudió tardíamente al departamento de urgencias y por ello, a pesar de la
intervención coronaria primaria expedita en que se realizaron angioplastia y
colocación de una endoprótesis en arteria descendente izquierda, el ecocardiograma
del día presente señala una fracción de expulsión de 25%. No ha tenido arritmias
ventriculares. Aún persiste levemente la disnea y en la exploración física se advierten
estertores pulmonares bilaterales y elevación del pulso venoso yugular. No recibe
más fármacos que ácido acetilsalicílico, clopidogrel y atorvastatina. Con los
tratamientos y fármacos siguientes aminorará la cifra de mortalidad en los 40 días
siguientes, EXCEPTO:
A. Desfibrilador implantable automatizado.

Una joven universitaria de 21 años estudia educación musical en la institución local y


en fecha reciente se unió al coro. Sin embargo, siempre que se pone de pie para
cantar un solo percibe comienzo de palpitaciones y obnubilación. Nunca ha perdido
el conocimiento, pero a veces ha tenido que sentarse por el mareo. Acude a la clínica
en que usted trabaja para ser valorada. Los datos de la exploración física son
totalmente normales, incluido el ECG basal en reposo. Le solicita usted a ella que
cante un trozo en solo para el personal asistencial mientras simultáneamente registra
un ECG de 12 derivaciones (Figura V-46). Su voz es bella, pero la joven rápidamente
presenta síntomas y tiene que interrumpir su canto. Usted la refiere para la práctica
de ecocardiografía y MRI cardiaco, pero ambas son normales. Se sospecha que las
palpitaciones de la paciente se deben a algunas de las entidades que expondremos.
Señale la más idónea.

E. Taquicardia ventricular del infundíbulo ventricular derecho

Una mujer de 47 años que recibe como fármaco de mantenimiento metadona por el
antecedente de abuso de narcóticos, en fecha reciente contrajo una infección de vías
respiratorias altas y un amigo le ofreció algunos comprimidos de eritromicina
sobrantes. Hoy percibió múltiples episodios de palpitaciones y obnubilación, que le
obligaron a acudir al departamento de urgencias. En ese sitio, en la tira de ritmo ECG
se identificaron “trenes” no sostenidos y múltiples de arritmias que se muestran en la
Figura V-47. Se administraron 2 mg de sulfato de magnesio por vía IV aunque no
ceden con tal maniobra los episodios de arritmia no sostenida. En los estudios de
laboratorio se señala que el potasio es normal. ¿Cuál es la medida siguiente más
adecuada?
D. Goteo IV de isoproterenol ajustado a un ritmo de 100-120 lpm.
V-48. En un ECG que tiene taquicardia con complejo ancho: ¿cuál de los elementos
siguientes refuerza con mayor peso el diagnóstico de taquicardia ventricular?
A. Disociación auriculoventricular

ALGUIEN MÁS
V-74. ¿Cuál de los planteamientos siguientes sobre la epidemiología de cardiopatías
congénitas (CHD) en los Estados Unidos es verdadera?

b) Ante los progresos en las técnicas operatorias y de la atención prenatal y posnatal,


la supervivencia de un recién nacido que tiene CHD se acerca actualmente a 90%.
V-75. Se valoró a un varón de 62 años valorado para la sustitución de válvula mitral
por insuficiencia mitral grave. Como parte de la valoración se le practicó un
ecocardiograma transesofágico en que se demuestra un pequeño chorro del flujo
Doppler de derecha a izquierda durante la sístole a través del tabique interauricular.
El chorro está situado aproximadamente en la mitad del tabique y aparece cuando un
pequeño colgajo de tejido oscila y está abierto <1 mm. No se advierte flujo diastólico
ni hay abertura visible de cualquier zona del tabique durante la diástole. ¿Cuál de los
planteamientos siguientes explica dicho hallazgo en la ecocardiografía?

d) Persistencia del agujero oval.

V-76. Se valora en la clínica a un joven de 21 años. Indica que en ocasiones se siente


cansado en el último año de su carrera universitaria, pero por lo demás está
asintomático. Lo atendió usted por primera vez seis semanas antes y percibió un soplo
holosistólico áspero en la mitad inferior del borde esternal izquierdo que se
intensificaba cuando cerraba el puño de la mano de ese lado. Ante la sospecha de
una comunicación interventricular (VSD) usted lo refirió al departamento de
ecocardiografía, en la cual se confirmó la sospecha clínica y se visualizó una VSD de
5 mm en el tabique interventricular en su porción muscular. El cateterismo de las
cavidades derechas del corazón para valoración hemodinámica que se realizó en la
fase siguiente indicó que la prsión media en la arteria pulmonar era de 20 mmHg, la
resistencia venosa pulmonar, de 2 unidades Wood y la resistencia vascular sistémica,
de 6 unidades Wood. Por medio de mediciones seriadas y meticulosas de la
saturación de oxígeno en sangre venosa en venas centrales y cavidades derechas
del corazón se pudo calcular que el gasto cardiaco derecho era de 7.5 l/minuto y el
gasto sistémico de 6 l/minuto. Ante los hallazgos anteriores: ¿qué estrategia
terapéutica recomienda usted?

d) Ninguna intervención en la actualidad; cada año valoraciones cardiológicas y


nuevos estudios imagenológicos en caso de surgir síntomas nuevos

V-77. Se valora a un paciente de 52 años que acudió al departamento de urgencias


por dolor retroesternal (torácico) y un nivel mayor de enzimas cardiacas. Al auscultarlo
se percibe fácilmente un soplo continuo y superficial a nivel mesoesternal que no
había corroborado anteriormente. Los estudios ecocardiográficos son limitados, pero
con ellos se puede visualizar las cuatro válvulas cardiacas y su aspecto es totalmente
normal. Se practica cateterismo de las cavidades derechas del corazón. Más adelante
se presenta la cifra de saturación de oxígeno. ¿Cuál es la anormalidad que existe en
los síntomas de este enfermo?

Vena cava superior 58%


Aurícula derecha 60%

Seno coronario medio 91%

Ventrículo derecho 70%

Arteria pulmonar 70%

c) Fístula arteriovenosa coronaria, del seno coronario.

V-78. El Sr. Jiménez es un varón de 40 años con válvula aórtica bicúspide congénita
que usted ha atendido por más de 10 años. Cada tercer año usted repite un
ecocardiograma para vigilar la evolución de la enfermedad, en el entendido de que
las válvulas bicúspides a menudo presentan estenosis o insuficiencia que obliga a su
sustitución en la edad madura. Ante la anomalía congénita específica del paciente:
¿cuál otra estructura anatómica es importante para vigilar los ecocardiogramas
bienales?

a) Diámetro de la base de la aorta.

V-79.. Todos los signos de definición clásica de la tetralogía de Fallot se exponen en


este apartado, EXCEPTO:

b) Cabalgamiento de la aorta

V-80. Un varón de 78 años fue valorado por disnea de esfuerzo. Tiene antecedentes
de tabaquismo, obesidad y diabetes mellitus, todos de larga evolución. Los fármacos
que recibe incluyen metformina, ácido acetilsalicílico y ocasionalmente ibuprofeno. En
la exploración física se advierte que sus pulsos periféricos tienen un pico tardío y
sobresale la onda ventricular izquierda. Muestra regularidad en su ritmo con un soplo
mesosistólico IV/VI más intenso en la base del corazón y que irradia a las arterias
carótidas. También se identifica un cuarto ruido cardiaco. Los trazos
ecocardiográficos confirman la estenosis aórtica intensa sin otra valvulopatías. De las
entidades siguientes: ¿cuál muy probablemente contribuyó a la aparición de la lesión
en su corazón?

b) Diabetes mellitus.

V-81. Un varón de 63 años acudió por primera vez por haber mostrado sincope con
esfuerzo y se le detecta estenosis aórtica. Al aconsejar al paciente usted le indica que
las recomendaciones terapéuticas que le hará se basan en la observación de que las
personas no tratadas que muestran el cuadro inicial que él tuvo tienen un promedio
de vida anticipable de:

-
c) 3 años.

V-82. El señor Belliard es un anciano de 82 años que usted ha atendido en sus visitas
de vigilancia a la clínica. Lo atendió por última vez tres años antes, cuando el paciente
mencionó que tenía síncopes ocasionales. Las investigaciones ulteriores revelaron
estenosis intensa y calcificación de la válvula aórtica con un área valvular de 0.7 cm2
y un gradiente medio de 45 mmHg. En esa ocasión se recomendó la sustitución
quirúrgica de la válvula aórtica. Sin embargo, el día de la operación el paciente indicó
que se sentía lo suficientemente enfermo como para no soportar la cirugía a corazón
abierto, no aceptaba el procedimiento quirúrgico y que prefería seguir en contacto
para cuando estuviera listo. No acudió a ninguna de las citas de vigilancia ulteriores.
En esta ocasión acudió al departamento de urgencias después de varias semanas de
letargo y disnea. Su presión arterial es de 82/68 mmHg y su frecuencia cardiaca de
110 lpm y en el ECG se identifica ritmo sinusal. Su pulso carotídeo es débil y tardío y
al tacto sus extremidades estaban frías. De nuevo se identifica un cuarto ruido en la
auscultación y un soplo sistólico que alcanza su máximo tardíamente, de intensidad
III/VI. En la actualidad son funcionales el riñón y el hígado y tiene oliguria. El cirujano
cardiotorácico solicitado prefiere diferir la sustitución de la válvula, como cirugía de
suma urgencia y ello se debe al daño renal y hepático agudo. De los planteamientos
que mostramos: ¿cuál sería una opción terapéutica razonable para mejorar el riego a
corto plazo del paciente al grado de permitir la sustitución de la válvula aórtica?

d) Valvuloplastia con globo aórtico por vía percutánea.

V-83. Una ex abogada de 85 años solicita atención porque durante varios meses ha
tenido disnea cada vez más intensa con el ejercicio y edema de extremidades
inferiores. En la exploración usted percibe que el choque de punta (PMI) se desplazó
hacia afuera y hay un ritmo de galope con cuarto ruido. Presenta un soplo sistólico
III/VI en la base, que irradia a las arterias carótidas. El ecocardiograma transtorácico
indica que la fracción de expulsión del ventrículo izquierdo es de 25%, con hipocinesia
global. El área calculada de la válvula aórtica es de 0.8 cm2 y un gradiente medio de
25 mmHg. ¿Cuál sería la medida siguiente más razonable para saber si la paciente
beneficiaría de la sustitución de la válvula aórtica?

d) Ecocardiografía de esfuerzo con dobutamina.

V-84. De los parámetros siguientes: ¿cuál mostraría disminución típicamente crónica


e intensa en la insuficiencia aórtica?

a) Tensión arterial diastólica.

V-85. Usted está a cargo de los internamientos nuevos en la unidad de cuidados


intensivos cardiológicos y atiende a una mujer de 21 años con una conjuntivopatía.
Su cuadro inicial incluyó disnea aguda y los signos de la radiografía de tórax indicaron
edema pulmonar difuso. En la exploración física se detectó un soplo protodiastólico
breve y suave en la mitad superior del borde esternal derecho y la ecocardiografía de
urgencia demostró insuficiencia aórtica grave con avulsión de la valva coronaria
derecha. En la CT de tórax no se detectó disección aórtica alguna. Al llegar a la unidad
de cuidados coronarios estaba ya intubada y sedada. Su presión arterial era de
110/50 mmHg y la frecuencia cardiaca, de 115 lpm. Era escasa su micción y las
extremidades estaban frías. El personal quirúrgico cardiotorácico estaba reunido para
que se practicara un trasplante de corazón, pero no se responsabilizaba de atender a
esta paciente como mínimo unas cuatro horas más. ¿Cuál intervención muy
probablemente permitirá conservar el riego terminal hasta la intervención quirúrgica?

c) Nitroprusiato por vía IV.

V-86. ¿Cuál es la causa más común de obstrucción del infundíbulo ventricular


izquierdo?

e) Enfermedad reumática mitral.

V-87. En casos de estenosis mitral intensa: ¿cuál de los parámetros siguientes


aumenta en forma típica?

b) Presión en la aurícula izquierda.

V-88. Se valora a una mujer de 42 años con el antecedente de fiebre reumática y su


consecuencia, estenosis mitral. Su valvulopatía actual es moderada. Usted sabe que
la estenosis mitral hace que aumente la presión en la aurícula izquierda, lo cual con
el paso del tiempo originará edema pulmonar cardiógeno e hipertensión pulmonar.
Todos los planteamientos siguientes harán que aumente la presión en la aurícula
izquierda con posible empeoramiento de la función pulmonar, EXCEPTO:

c) Metoprolol.

V-89. La Sra. Lezama acude al departamento de urgencias porque se agravó en


forma aguda su falta de aire (disnea). Tiene 84 años, estenosis mitral intensa y está
programada para que se le practique valvulotomía percutánea con globo en la mitral,
en un lapso de tres días. Sin embargo, el día de hoy mientras preparaba una ensalada
de pollo advirtió que sentía debilidad y disnea abrumadoras. Durante la exploración
se le encontró con disnea y angustia leve. La saturación de oxígeno con respiración
de aire ambiental fue de 91%; su frecuencia cardiaca es de 55 lpm y la presión arterial,
110/80 mmHg. En ambos campos pulmonares a nivel medio tiene estertores. Se
observa que su ritmo cardiaco es totalmente irregular y por medio de ECG se confirma
que surgió como trastorno nuevo, fibrilación auricular. Sospecha usted que tiene
incremento notable de la presión auricular izquierda que origina edema pulmonar. Las
intervenciones terapéuticas siguientes serán útiles para disminuir su presión auricular
izquierda, EXCEPTO:
-
d) Colocación transvenosa de marcapaso y estimulación externa con frecuencia
de 90 lpm.

V-90. Un varón de 34 años con estenosis mitral reumática fue referido a valoración.
Disfruta de la práctica de fútbol recreativo y no tiene limitaciones ni síntomas. Su
frecuencia cardiaca es de 65 lpm en reposo. En el ecocardiograma transtorácico se
identifica que el ventrículo izquierdo tiene tamaño y función normales; hay dilatación
leve de la aurícula de ese lado y un área de válvula mitral de 1.7 cm2 con valvas
relativamente delgadas no calcificadas. En el trazo ECG se advierte auriculomegalia
izquierda y ritmo sinusal. En la prueba de esfuerzo, su presión sistólica calculada de
la arteria pulmonar con el punto máximo de esfuerzo es de 40 mmHg. De los planes
terapéuticos siguientes: ¿cuál o cuáles recomendaría usted?

c) Valoraciones de cardiología periódicas y vigilancia seriada por ecocardiografía.

V-91. Se valora a un varón de 65 años que durante 15 años ha presentado una


miocardiopatía no isquémica con dilatación del ventrículo izquierdo y una fracción de
expulsión de 15%. En los últimos cinco años los ecocardiogramas practicados
anualmente demostraron insuficiencia mitral grave. Recibe tratamiento médico óptimo
y el paciente tiene síntomas catalogados dentro de la clase II de NYHA. El día de hoy
solicitó específicamente la sustitución de la válvula para mejorar su supervivencia.

b) “En pacientes como usted, la reparación valvular nunca ha mejorado la


supervivencia”.

V-92. Se realiza un cateterismo de derecha a izquierda al paciente de estenosis, que


abarca las dos valvas. Con base en las cifras de presión que señalamos: ¿cuáles de
las dos válvulas intervendrían?

d) Mitral y tricúspide.

V-93. Se valora a una mujer de 50 años con hipertensión pulmonar. En el último


ecocardiograma transtorácico se identificó insuficiencia tricuspídea intensa además
de dilatación e hipocinesia del ventrículo derecho y presiones sistólicas calculadas en
la arteria pulmonar que rebasaban 70 mmHg. En la exploración se advierte edema de
extremidades inferiores, hepatomegalia con hígado pulsátil, incremento del pulso
venoso yugular que llega al maxilar inferior con notables ondas c-v y un descenso y
notable así como choque de punta del ventrículo derecho. Indica que con el ejercicio
moderado “le falta el aire”. ¿Cuál es el mejor tratamiento de su insuficiencia
tricuspídea graves

a) Diuréticos y restricción de sodio acompañado de tratamiento médico orientado


a controlar la hipertensión pulmonar.

V-94. Se valora por primera vez a una mujer de 21 años en la clínica de medicina
familiar. Antes de la sesión la paciente nunca acudió a un médico porque sus padres
no confiaban en la medicina occidental. En el interrogatorio indicó que en ocasiones
se siente cansada y que “no podía seguirles el ritmo a sus compañeros en las clases
de educación física de la universidad. En la exploración física se detecta la existencia
de un soplo sistólico en el segundo espacio intercostal izquierdo antecedido de un
chasquido presistólico. Los trazos del ecocardiograma transtorácico confirman la
presencia de estenosis de la válvula pulmonar con un gradiente máximo de 60 mmHg
y “curvatura” de la válvula pulmonar sin reflujo pulmonar. ¿Cuál es la mejor opción
terapéutica?

c) Valvulotomía con globo por vía percutánea.

V-95. Se atiende a un paciente de 77 años con estenosis aórtica intensa en la unidad


de cuidados intensivos cardiológicos. Se planea para el día siguiente la sustitución
quirúrgica de la válvula aórtica. Sin embargo, de manera repentina el paciente
presenta disnea aguda y tiene signos de edema pulmonar agudo. En la auscultación
usted percibe un soplo suave y corto apical sistólico (además del identificado
anteriormente, de la estenosis aórtica) que no estaba presente. Sospecha usted que
se rompió alguna cuerda tendinosa de la válvula mitral y que tiene insuficiencia mitral
aguda y grave. De los parámetros siguientes: ¿cuál posiblemente aumentará a causa
del nuevo cuadro de insuficiencia mitral grave?

d) Fracción de expulsión.

V-96. El Sr. Martínez es uno de los pacientes con el antecedente de cardiopatía


reumática que por mayor tiempo ha sido atendido en la clínica. En el último
ecocardiograma se identificó un gradiente medio de válvula mitral de 11 mmHg, con
un área calculada valvular de 1.3 cm2, con frecuencia cardiaca de 60 lpm. El día de
hoy acude y señala que se empeoró su falta de aire y en los trazos ECG se advierte
fibrilación auricular con un ritmo de 60 lpm. Nunca ha presentado episodios
hemorrágicos y en la última revisión hecha dos semanas antes sus recuentos
hematológicos fueron normales. De las opciones siguientes para la profilaxia de
tromboémbolos: ¿cuál es la más apropiada?

d) Warfarina

V-97. Los siguientes elementos constituyen factores de riesgo para que surja
miocardiopatía periparto, EXCEPTO:

c) Primípara.

V-98. Se valoró a una nueva paciente en la clínica. Se trata de una persona de 25


años en la que se hizo el diagnóstico de “insuficiencia cardiaca” en otra población y
desde esa fecha ha cambiado de domicilio. Tiene síntomas de clase funcional II de
NYHA pero niega haber tenido angina. Acudió para valoración y tratamiento. Durante
la anamnesis, se observó que la paciente solamente puede desplazarse en silla de
ruedas y que tiene escoliosis intensa. No tiene antecedentes familiares de
hiperlipidemia. En la exploración física destacan signos como estertores crepitantes
en ambos campos pulmonares, tercer ruido y ausencia de cianosis. En la clínica se
practica ECG y en él se advierte que las ondas R son altas en V1 y V2, con QS
profundas en V5 y V6. El ecocardiograma señala intensa disfunción global del
ventrículo izquierdo con disminución de la fracción de expulsión. ¿Cuál es el
diagnóstico más probable?

d) Distrofia muscular de Duchenne.

V-99. ¿Cuál de los siguientes enunciados corresponde al perfil de herencia más


común respecto a miocardiopatías familiares?

b) Autosómico dominante impulsado por mutaciones no codificantes.

V-100. Varón de 45 años con el antecedente de obesidad, que acudió al


departamento de urgencias a causa de disnea, fatiga y tos nocturna que en los últimos
meses empeoró. Niega haber mostrado dolor o sensación de opresión retroesternal
en reposo o con el ejercicio. En la valoración muestra signos de cardiomegalia con
desplazamiento del choque de punta e incremento de las presiones de llenado con
estertores en ambos campos pulmonares y elevación del pulso venoso yugular. En la
ecocardiografía se identifica depresión global de 25% de la fracción de expulsión del
ventrículo izquierdo con dilatación de esa cavidad cardiaca. De los métodos
siguientes: ¿cuál es una recomendación de nivel I para nuevas investigaciones?

e) Nivel de hormona tiroestimulante.

V-101. Un estudiante universitario de 22 años sin antecedentes personales


patológicos fue atendido tres días antes en la clínica de urgencias por coriza, mialgias,
tos y fiebre, cuadro típico de una infección viral de vías respiratorias altas, que ha
contagiado a muchas personas en las instalaciones de la institución. Se le recetaron
un antitusígeno y antipiréticos y se le indicó que estuviera bien hidratado. El día de
hoy acudió al departamento de urgencias por letargo y fatiga. Mostraba obnubilación,
con una frecuencia cardiaca de 120 lpm y presión arterial de 78/62 mmHg. Presentaba
frialdad de extremidades y elevación del pulso venoso y yugular hasta un punto
cercano al maxilar inferior. En la auscultación del área precordial se identificaron
ruidos cardiacos “lejanos”, galope S3 y un soplo suave de reflujo mitral. En el
ecocardiograma transtorácico de urgencia no se identificó derrame pericárdico, no
había dilatación del ventrículo izquierdo, tenía una fracción de expulsión de 30% y
había insuficiencia mitral leve. En la biopsia endomiocárdica se detectó miocarditis
linfocítica. De los planteamientos siguientes respecto al pronóstico y consecuencias
del tratamiento: ¿cuál o cuáles son verdaderos?

a) Sus posibilidades de sobrevivir son <10% sin trasplante de corazón. Está


justificado incluirlo en la lista de espera de trasplantes de urgencia.
V-102. Se valora a una mujer de 42 años con el antecedente de tiroiditis de Hashimoto
muchos años antes de que fuera tratada satisfactoriamente con yodo radiactivo.
Acude al departamento de urgencias después de varias lipotimias e indica que mostró
días en que la letargia empeoró y también las molestias retroesternales. Muestra
disnea y su frecuencia cardiaca con la persona en reposo es de 110 lpm y la presión
arterial, de 77/62 mmHg. También tiene las extremidades frías y su aspecto es
somnoliento. Hay incremento del nivel de lactato en sangre completa y es mínima su
micción en las primeras dos horas de su permanencia en la sala. En la vigilancia
cardiaca por monitor repetidamente tiene “salvas” de taquicardia ventricular no
sostenida. El ecocardiograma revela una fracción de expulsión de ventrículo izquierdo
de 15% y en la biopsia endomiocárdica de urgencia indica lesiones granulomatosas
difusas, rodeadas de infiltrado inflamatorio extenso. De los planteamientos siguientes:
¿cuál es el verdadero en cuanto a su diagnóstico?

c) La evolución de esta miocardiopatía es mala, a menudo con deterioro rápido que


obliga a trasplante urgente de corazón

V-103. De los factores que se mencionan a continuación, todos ellos se asocian con
incremento en el riesgo de miocardiopatía por quimioterapia con antraciclinas,
EXCEPTO:

e) Todos los planteamientos anteriores conllevan un mayor riesgo de miocardiopatía


causada por antraciclinas

V104. l Sr. Kia es un bajista-guitarrista de 32 años sin antecedentes personales.


Acude a que se le atienda por un dolor retroesternal después de presentar coriza, tos,
fiebre y mialgias en las últimas semanas. Su dolor, según el paciente, es constante y
se irradia al hombro izquierdo; se exacerba en el decúbito dorsal y durante las
respiraciones en el sueño. En la exploración se advierte un ruido extracardiaco
raspante en los tres componentes por cada latido. Los trazos ECG se muestran en la
Figura V-104. En la visita inicial no se detectan los niveles de troponina I y seis horas
más tarde. Las cifras de presión arterial, frecuencia cardiaca y oxigenación son
normales. ¿Cuál es la siguiente medida más adecuada?

b) Administrar 1 g de ácido acetilsalicílico cada ocho horas con 20 mg de


omeprazol diariamente.

V-105. Se valora a un varón de 45 años en la unidad de cuidados intensivos


cardiológicos. Acudió por dolor retroesternal y se pensó en el comienzo que tenía un
síndrome coronario agudo, lo que obligó a administrar inicialmente fármacos
antiplaquetarios y heparina por vía IV. Después de la valoración completa y la
negativización de las enzimas cardiacas seriadas se pudo advertir que en vez de lo
que se pensaba, tenía pericarditis aguda. Poco después de su internamiento en la
unidad de cuidados intensivos cardiológicos mostró hipotensión con distensión de las
venas del cuello. En la auscultación los campos pulmonares estaban limpios. Sus
extremidades estaban frías y se advirtió que sólo durante la espiración se podía palpar
el pulso humeral. ¿Cuál es el diagnóstico más probable?

b) Taponamiento cardiaco

V-106. Se valora a un varón de 45 años en la unidad de cuidados intensivos


cardiológicos. Acudió por dolor retroesternal y se pensó en el comienzo que tenía un
síndrome coronario agudo, lo que obligó a administrar inicialmente fármacos
antiplaquetarios y heparina por vía IV. Después de la valoración completa y la
negativización de las enzimas cardiacas seriadas se pudo advertir que en vez de lo
que se pensaba, tenía pericarditis aguda. Poco después de su internamiento en la
unidad de cuidados intensivos cardiológicos mostró hipotensión con distensión de las
venas del cuello. En la auscultación los campos pulmonares estaban limpios. Sus
extremidades estaban frías y se advirtió que sólo durante la espiración se podía palpar
el pulso humeral. Usted solicita con carácter de urgencia la ecocardiografía de este
paciente. En este caso: ¿cuál de los siguientes elementos muy probablemente se
intensificará con la inspiración, en comparación con la espiración?

b) Volumen sistólico de ventrículo izquierdo.

V-107. Todas las características ECG serán útiles para diferenciar entre la pericarditis
aguda y el infarto agudo del miocardio, EXCEPTO:
d) Elevación del segmento ST en V2

V-108. Mujer de 35 años hospitalizada por malestar general, incremento ponderal,


mayor circunferencia abdominal y edema. Los síntomas comenzaron unos tres meses
antes y poco a poco evolucionaron. La paciente refiere un aumento cercano a 15
centímetros en la circunferencia abdominal. La hinchazón de las piernas ha
empeorado cada vez más al grado de que percibe a los muslos como hinchados
también. Muestra disnea del ejercicio y ortopnea con uso de dos almohadas al dormir.
Tiene el antecedente de enfermedad de Hodgkin diagnosticada 12 o 13 años y en esa
ocasión recibió quimioterapia y radiación mediastínica. En la exploración física
muestra consunción de los músculos temporales y su aspecto es de enfermedad
crónica. Su peso actual es de 96 kg y en los últimos tres meses hubo un aumento de
11 kg. Sus signos vitales son normales. Su presión venosa yugular se acerca a 16 cm
y las venas del cuello no muestran colapso durante la inspiración. Los ruidos
cardiacos se escuchan amortiguados. Se produce un tercer ruido cardiaco poco
después del cierre de la válvula aórtica. El ruido es breve y repentino y se percibe
mejor en la punta. Hay hepatomegalia e hígado pulsatil, así como ascitis. Muestra
edema blando en todas las extremidades inferiores y parte de la pared abdominal. El
ecocardiograma indica engrosamiento pericárdico, dilatación de la vena cava inferior
y de las venas hepáticas, e interrupción repentina del llenado ventricular en la
protodiástole. Su fracción de expulsión es de 65%. ¿Cuál es la mejor estrategia
terapéutica en esta paciente?
d) Ablación pericárdica

V-109. Un contador de 45 años sin antecedentes personales, presentó en los últimos


tres meses ocasionalmente fiebre, artralgias simétricas y fatiga. Dos semanas antes
presentó debilidad repentina de la mano izquierda que mostró resolución en término
de una hora. Múltiples cultivos de sangre no han arrojado resultados positivos. La
valoración en el servicio de urgencias incluyó tomografía computarizada de la cabeza,
en la que no se advirtieron signos anormales. La semana anterior se le practicó un
ecocardiograma transtorácico en el cual se identificó una masa solitaria de 2 cm en la
aurícula izquierda que provenía del tabique interauricular, muy cerca de la fosa oval
y al parecer era pediculada y pendía de un tallo fibrovascular. Fue valorado
nuevamente en la clínica. ¿Cuál es la medida siguiente más adecuada?

a) Extirpación quirúrgica cardiotorácica de la masa.

Se valora a un paciente de dolor retroesternal en el departamento de urgencias. En


su ECG no presenta cambios en las ondas Q o en el segmento ST. Se intentó saber
si la persona tiene un infarto del miocardio. De los siguientes biomarcadores: ¿cuál
es el de tipo bioquímico preferido en casos de infarto del miocardio?
A. Creatina cinasa.
B. Banda de CK-MB.
C. Velocidad de eritrosedimentación.
D. Lactato deshidrogenasa.
E. Troponina I.

Usted está realizando una angiografía coronaria diagnóstica del Sr. Hayes. En las
imágenes angiográficas iniciales no hay segmentos obstruidos en sus arterias. Sin
embargo, durante un angiograma selectivo de la arteria descendente anterior
izquierda percibe que se formó un trombo en la punta del catéter que usted utiliza.
Durante la inyección de medio de contraste el émbolo viajó por la arteria descendente
anterior izquierda y obstruyó totalmente el flujo sanguíneo. Por fortuna, se puede
extraer el trombo por medio de trombectomía por aspiración, casi inmediatamente. En
caso de haber sido imposible extraer el trombo: ¿qué momento habría usted esperado
para que se detectara por primera vez la isoforma CK-MB en un método sérico
corriente?

A. 5-10 minutos.
B.
C.
D.
E.
-
1-2 horas.
4-8 horas.
12-24 horas.
24-48 horas

¿En qué periodo se produce el máximo retraso entre el comienzo de los síntomas y
el del tratamiento definitivo en personas con infarto del miocardio con elevación del
segmento ST?
A. Entre el comienzo del dolor y la decisión del enfermo de solicitar auxilio.
B. Entre la llamada del paciente y la llegada del personal médico de urgencia.
C. Entre la llegada del personal médico de urgencia y la llegada al hospital.
D. Entre la llegada al hospital y la decisión de comenzar el tratamiento de
reperfusión.
E. Entre la decisión de emprender el tratamiento por reperfusión y el comienzo
real de dicho método.

Todos los medicamentos que señalaremos utilizados en el tratamiento inmediato


(agudo) del infarto del miocardio con elevación del segmento ST, concuerdan
adecuadamente con su mecanismo de acción, EXCEPTO:
A. Ácido acetilsalicílico - disminución del nivel de tromboxano A2
B. Abciximab - inhibición del receptor de la glucoproteína IIb/IIIa.
C. Antagonistas β-adrenérgicos - disminución del consumo de oxígeno por el
miocardio.
D. Clopidogrel - inhibición del receptor de difosfato de adenosina (ADP) por
plaquetas.
E. Nitroglicerina - disminución de la poscarga cardiaca

Un varón de 61 años acudió al departamento de urgencias 2 h antes, con dolor


retroesternal anginoso y elevaciones del segmento ST en derivaciones I, aVL y V1-
V3. Al mismo tiempo, el cardiólogo intervencionista que estaba en funciones,
participaba en el caso complejo y no podía emprender una intervención coronaria
percutánea primaria en forma inmediata; en tal situación se administró al paciente el
activador de plasminógeno hístico (tPA). Sin embargo, 120 min después de
administrar tPA el paciente siguió quejándose de dolor retroesternal y en su ECG no
se identificó resolución de las elevaciones del segmentoST. ¿Cuál es la medida
siguiente más apropiada?
A. Administrar una segunda dosis de tPA.
B. Administrar 1.5 millones de unidades de estreptocinasa en un lapso de una
hora.
C. Emprender el injerto por derivación de arteria coronaria con carácter urgente.
D. Emprender la intervención coronaria percutánea.
E. Esperar 60 min antes de intentar otras opciones terapéuticas

Al Sr. Cooper se le administró tPA por mostrar en forma aguda un infarto del miocardio
con elevación del segmento ST, una hora antes. Había mostrado una respuesta
excelente con resolución de dicha elevación y del dolor retroesternal. Sin embargo, la
enfermera por medio del altavoz solicita a usted que valore el cambio de ritmo
cardiaco. Usted percibe un ritmo ventricular con complejos amplios con frecuencia de
75 lpm. El paciente dice sentirse bien sin nuevos síntomas y su presión arterial es de
120/80 mmHg. De las siguientes maniobras terapéuticas por practicar: ¿cuál será la
más apropiada?
A. Administrar 150 mg de amiodarona por vía IV en un lapso de 10 minutos.
B. Administrar 400 mg de flecainida por vía oral.
C. Administrar 5 mg de metoprolol por vía IV cada cinco minutos tres veces.
D. Masaje del seno carotídeo.
E. Continuar la observación.

Mujer de 84 años con antecedente de diabetes e hiperlipidemia que acudió desde


hace tres semanas al departamento de urgencias con infarto del miocardio con
elevación del segmento ST. Se le practicó con carácter de urgencia una intervención
coronaria percutánea primaria por oclusión aguda de la arteria descendente anterior
izquierda. Se le colocó una endoprótesis impregnada con everolimús de 2.5 × 24 mm,
con resultado angiográfico excelente y resolución completa de sus síntomas. Desde
esa fecha se sintió extraordinariamente bien. Sus fármacos al momento del alta fueron
metoprolol, ácido acetilsalicílico, clopidogrel, rosuvastatina, lisinopril e insulina con
una escala móvil. En fecha reciente acudió a su oalmólogo quien le identificó una
catarata en el ojo derecho. Dicho médico habló al consultorio de usted solicitando su
opinión para interrumpir el uso de ácido acetilsalicílico, clopidogrel o ambos fármacos
y así aminorar el riesgo de hemorragia durante la cirugía de extracción de catarata
que había sido programada para la semana siguiente. La enferma piensa que el
riesgo de hemorragia con ambos fármacos es demasiado grande como para
emprender la operación oalmológica. ¿Cuál de las siguientes opiniones daría usted?

A. “Interrumpa el uso de ácido acetilsalicílico, continúe el del clopidogrel y siga


adelante con la cirugía de la catarata”.
B. “Interrumpa el uso de clopidogrel, continúe el ácido acetilsalicílico y siga
adelante con la cirugía de catarata”.
C. “Interrumpa el uso de ácido acetilsalicílico y clopidogrel y siga adelante con la
cirugía de catarata”.
D. “Difiera cuando menos seis meses la cirugía planeada y de preferencia 12
meses y para esa fecha interrumpa el uso de clopidogrel”.
E. “Le recomiendo practicar la cirugía incluso si recibe el ácido acetilsalicílico y
el clopidogrel, porque nunca podrá interrumpir el uso de los dos fármacos

La Sra. Constance tiene 65 años, es una bibliotecaria jubilada recientemente y tiene


antecedente de hipertensión. Después de su jubilación comenzó un programa de
marcha por ejercicio, pero advirtió que le faltaba extraordinariamente el aire y percibía
una sensación de opresión torácica moderada después de caminar sólo dos o tres
cuadras. Hace dos semanas acudió al consultorio. Ante la preocupación de que sus
síntomas provinieran de alguna arteriopatía coronaria, fue enviada al cardiólogo para
la práctica de angiografía coronaria diagnóstica, que se realizó hace una semana y
en la cual se identificaron estenosis focales graves en las arterias: descendente
anterior izquierda en su porción proximal, circunfleja izquierda y coronaria derecha.
Salvo las lesiones mencionadas, no tiene la paciente alguna enfermedad que limite el
flujo sanguíneo. El cardiólogo no realizó la intervención coronaria percutánea (PCI) y
recomendó que analizara con el cirujano cardiotorácico local la posibilidad de colocar
un injerto de derivación en arteria coronaria (CABG). La paciente retornó al hospital y
solicita consejo. Durante la reunión se logra que participen el cardiólogo y el cirujano
cardiotorácico. Al aconsejar a la paciente respecto a la estrategia de revascularización
más adecuada: ¿cuál de los planteamientos siguiente es verdadero?

A. CABG tiene una cifra menor de mortalidad al año, en comparación con PCI
en pacientes como la Sra. Constance.
B. Al año, pacientes como la Sra. Constance a quien se practicaría CABG,
tienen mayor riesgo de necesitar otras técnicas de revascularización, en
comparación con personas a quienes se practica PCI.
C. PCI conlleva un mayor riesgo de accidente cerebrovascular que CABG.
D. Deben incorporarse a la decisión las características de la anatomía de
coronarias y la previsión de buenos resultados por parte del cardiólogo.
E. PCI tiene una frecuencia menor de ocasionar infarto de miocardio en término
de 12 meses, en comparación con CABG para pacientes como la presente

El Sr. Gruentzig acude a la consulta de cardiología por disnea y sensación de opresión


retroesternal leve con el esfuerzo. Después de un interrogatorio y una exploración
física hechos cuidadosamente y un comentario minucioso con el paciente sobre los
riesgos y beneficios, usted lo refiere para que se le practique un gammagrama con
99m-tecnecio en el cual se observa isquemia infero-apico-posterior. Inmediatamente
refiere usted al paciente a su colega para la práctica de angiografía coronaria. En ella,
se advierte que el paciente tiene un sistema dominante izquierdo con oclusión total
de la arteria circunfleja izquierda. La porción distal de esta última arteria recibe sangre
por medio de vasos colaterales de la arteria descendente anterior de ese lado. El
cardiólogo intervencionista es muy hábil, pero no pudo pasar a través de la oclusión
total en la arteria circunfleja izquierda por medio de una guía hidrófila y la vía de
acceso anterógrada. De los planteamientos siguientes en cuanto a la intervención en
este caso de oclusión crónica total (CTO): ¿cuál es verdadero?

A. Si es imposible pasar a través de la oclusión total con cualquier guía por la vía
anterógrada, el operador debe abandonar la reparación percutánea y pensar
en el referimiento de la paciente para cirugía de derivación coronaria.
B. El intento para tener acceso a la oclusión coronaria total por medio de la vía
retrógrada, a través de un vaso colateral, es una opción cuando se intenta la
revascularización percutánea.
C. A pesar de que la presencia de oclusión coronaria total se acompaña de una
cifra mayor de mortalidad, nunca se ha demostrado que los buenos resultados
de PCI de una oclusión coronaria total disminuyeran la frecuencia de
mortalidad.
D. Rara vez se obtienen buenos resultados con la cirugía de derivación de arteria
coronaria en casos de oclusión coronaria total, a causa de la naturaleza
atrésica del vaso en sentido distal al segmento ocluido en forma crónica.
E. En los estudios, la revascularización lograda en caso de oclusiones coronarias
totales crónicas se acompaña de alivio sintomático, pero no de mejoría en la
función del ventrículo izquierdo.

Un conductor de camión de 67 años, 12 años antes fue sometido a CABG de cuatro


vasos con injerto de arteria mamaria interna (LIMA) unida a la arteria descendente
anterior izquierda (LAD) y tres injertos de vena safena (SVG) a las arterias primera
diagonal, primera marginal obtusa y descendente posterior derecha. Al atenderlo en
la visita en esta ocasión, señala que hace un mes, aproximadamente, mostró dolor
retroesternal cada vez más intenso y disnea con grados cada vez menores de
ejercicio. Usted lo refiere para la práctica de angiografía coronaria en la cual se
detecta estenosis notable en la porción principal de SVG con la primera rama marginal
obtusa. ¿Cuál de los planteamientos siguientes es verdadero?

A. La ineficacia del injerto de SVG casi siempre proviene de la embolia por un


trombo originado en el sitio de anastomosis aórtica.
B. La PCI al injerto de vena safena necesitará la colocación de un dispositivo
protector distal.
C. La PCI unida al injerto de vena safena es demasiado peligroso como para
justificar su práctica. Habrá que intentar tratamiento intensivo con fármacos
antianginosos.
D. El riesgo de PCI en caso de SVG es idéntico al PCI con el vaso original.
E. Los conductos de injerto de vena y arteria (p. ej., los de SVG y LIMA) tienen
una cifra idéntica de libre tránsito en los primeros cinco años después la
colocación de la derivación

La Sra. Edwards tiene 87 años y antecedente de tabaquismo intenso y como


resultado, COPD con volumen espiratorio forzado al segundo (FEV1) de 0.76 L;
diabetes mal controlada y nefropatía crónica en etapa III. En etapas anteriores usted
la había atendido, pero no ha sido valorada en los últimos ocho años. Sin embargo,
acude a la clínica el día de hoy y señala sensación de opresión retroesternal con el
esfuerzo, hinchazón de piernas, disnea y fatiga intensa incluso con el mínimo
esfuerzo. En la exploración se percibe un soplo sistólico áspero que alcanza su
máximo en etapa tardía en la mitad superior del borde esternal derecho, que irradia a
las arterias carótidas y tono musical en la punta. Usted no identifica un segundo ruido
y no se ausculta el pulso carotídeo, detectándose retraso en la aparición del mismo
en comparación con el choque de punta del ventrículo izquierdo, detectado a la
palpación. Se sospechó estenosis extensa de la válvula aórtica y en consecuencia, el
ecocardiograma confirmó la presencia de estenosis aórtica con un área de 0.59 cm2.
Usted refiere a la paciente a un cirujano cardiotorácico, quien indicó que la paciente
estaba expuesta a un riesgo muchísimo mayor de reemplazo quirúrgico de válvula
aórtica y cita una cifra de riesgo por mortalidad intrahospitalaria, mayor de 15%. Usted
tiene cita con la paciente en la mañana siguiente. De los planteamientos siguientes:
¿cuál sería el más razonable?

A. “En el caso de esta enfermedad se ha demostrado que los fármacos


aminoran el riesgo de mortalidad”.
B. “Indudablemente usted no constituye una paciente elegible para cirugía. Su
problema conlleva un riesgo pequeño de mortalidad anual y quinquenal y
podemos controlar los síntomas con medicamentos”.
C. “La valvulotomía con globo aórtico por vía percutánea es una opción
satisfactoria como tratamiento no quirúrgico y con ella se obtienen resultados
excelentes a corto y largo plazo”.
D. “Usted no es elegible para cirugía y por esta razón no existe otra técnica para
el reemplazo de válvula aórtica en este caso. Debemos considerar las
opciones de cuidado en una institución para cuidados de enfermos a largo
plazo”.
E. “Debemos pensar en el envío para estudios si fuera elegible para reemplazo
de válvula aórtica a través de catéter

Los tratamientos siguientes en cuanto a la epidemiología de hipertensión son


ciertos, EXCEPTO:
A. En personas de 60 años o mayores la tensión sistólica promedio es mayor en
mujeres.
B. La tensión diastólica aumenta poco a poco con la edad durante toda la vida.
C. La prevalencia de hipertensión es menor en mexicoestadounidenses que en
caucásicos no latinoamericanos.
D. En países industrializados, la presión arterial aumenta gradualmente en los
primeros 20 años de vida.
E. En los Estados Unidos, en promedio, 30% de los adultos tienen hipertensión

¿La enfermedad de cuál órgano es la causa más común de mortalidad en


hipertensos?

A.
B.
C.
D.
E.
-
Encéfalo.
Corazón.
Riñones.
Hígado.
Pulmones
Una mujer de 28 años ha tenido hipertensión difícil de controlar. Se le diagnosticó a
los 26 años de vida y desde esa fecha ha recibido cantidades cada vez mayores de
fármacos. Su régimen actual consiste en 1 000 mg de labetalol cada 12 h (bid), 40 mg
de lisinopril cada 24 h (qd), 0.1 mg de clonidina cada 12 h y 5 mg de amlodipino cada
24 h. En la exploración física su aspecto es por lo demás normal. La presión arterial
es de 168/100 mmHg y su frecuencia cardiaca de 84 lpm. Los datos de la exploración
cardiovascular no tienen importancia, pues no hay frotes, galopes ni soplos. Sus
pulsos periféricos son satisfactorios y no tiene edema. En su aspecto físico no se
identifican signos como hirsutismo, distribución anormal de la grasa o anomalías de
genitales. Los datos de estudios de laboratorio indican un nivel de potasio de 2.8
meq/100 ml y de bicarbonato sérico de 32 meq/100 ml. La glucemia con la paciente
en ayuno es de 114 mg/100 ml. ¿Cuál es la entidad diagnóstica más probable?

A. Hiperplasia suprarrenal congénita.


B. Displasia fibromuscular.
C. Síndrome de Cushing.
D. Síndrome de Conn.
E. Feocromocitoma

Una mujer de 28 años ha tenido hipertensión difícil de controlar. Se le diagnosticó a


los 26 años de vida y desde esa fecha ha recibido cantidades cada vez mayores de
fármacos. Su régimen actual consiste en 1 000 mg de labetalol cada 12 h (bid), 40 mg
de lisinopril cada 24 h (qd), 0.1 mg de clonidina cada 12 h y 5 mg de amlodipino cada
24 h. En la exploración física su aspecto es por lo demás normal. La presión arterial
es de 168/100 mmHg y su frecuencia cardiaca de 84 lpm. Los datos de la exploración
cardiovascular no tienen importancia, pues no hay frotes, galopes ni soplos. Sus
pulsos periféricos son satisfactorios y no tiene edema. En su aspecto físico no se
identifican signos como hirsutismo, distribución anormal de la grasa o anomalías de
genitales. Los datos de estudios de laboratorio indican un nivel de potasio de 2.8
meq/100 ml y de bicarbonato sérico de 32 meq/100 ml. La glucemia con la paciente
en ayuno es de 114 mg/100 ml. ¿Cuál es la mejor estrategia para diagnosticar la
enfermedad de esta paciente?

A. Medir los niveles de renina en vena renal.


B. Medir metanefrinas en orina de 24 horas.
C. MRI de arterias renales.
D. Medir cortisol en orina de 24 horas.
E. Medir la razón aldosterona/renina en plasma
22-29
El nudo SA actúa como un marcapaso dominante del corazón y genera ritmo sinusal
normal. ¿Qué propiedad de dicho nudo permite a las células actuar con esa función
primaria?

A. Situación cerca del área lateral superior de la aurícula derecha.

B. Numerosos discos intercalares en comparación con otras zonas del tejido


miocárdico.

C. Despolarización de fase 0 más rápida.

D. Cuenta sólo con células con la capacidad de despolarización espontánea.

E. Despolarización espontánea durante la fase 4 del potencial de acción, con mucha


mayor rapidez que la observada con otros miocardiocitos.

El Sr. Enríquez es un sommelier de 21 años en un conocido restorán de la ciudad. Su


diversión primaria es el ciclismo competitivo y la semana próxima espera participar de
una carrera de 150 km, para la cual estuvo entrenando en los últimos seis meses. Los
coordinadores de ese evento exigen a todos los competidores someterse a una
valoración cardiovascular integral antes de la competencia y con ese motivo acude a
la clínica en que usted trabaja. En la valoración que usted hace, percibe que la
frecuencia cardiaca con el sujeto en reposo es de 45 lpm con una pausa ocasional de
dos segundos, incluso. Su presión arterial es de 108/72 mmHg. Se siente bien y no
señala episodios sincopales o presincopales en el reposo o durante el ejercicio. Salvo
la bradicardia, usted no detecta otras anomalías en su exploración. Su trazo ECG
indica ritmo sinusal con un intervalo PR de 128 mseg, duración de QRS, 80 mseg y
pausas ocasionales que duran incluso 2.2 segundos. De las recomendaciones
siguientes: ¿cuál sería la más apropiada para el Sr. Enríquez?

A. Practicar una valoración electrofisiológica ante la posibilidad de colocar un


marcapaso.

B. Él no podrá competir en la carrera próxima por el problema de la bradicardia y se


necesita someterlo a la prueba de “basculación” en una mesa.

C. No se necesita más estudios de vigilancia y que tenga buena suerte con la carrera.

D. Practicar una vigilancia con el monitor Holter durante 48 horas.

E. Es necesario practicar ECG de esfuerzo en una banda sin fin para dilucidar la
presencia de suficiencia cronotrópica.

Un varón de 60 años es sometido a un estudio electrofisiológico por el antecedente


de síncope. Después de canulación cuidadosa de una vena y colocación de catéteres
de conductancia y estimulación y después de administrar 0.2 mg de propranolol/kg de
peso y 0.04 mg de atropina/kg de peso, su frecuencia cardiaca es de 65 lpm. Después
de interrumpir el uso de los fármacos y que transcurra tiempo suficiente para su
eliminación, se estimula las porciones superior/lateral de la aurícula derecha a razón
de 140 lpm. Una vez que se interrumpe la estimulación externa, el siguiente latido
sinusal aparece 1 800 mseg después. Con base en las observaciones anteriores:
¿cuál de los diagnósticos siguientes se puede asignar al paciente?
A. Miocardiopatía por amiloidosis.
B. Enfermedad de nudo AV.
C. Fibrilación auricular paroxística.
D. Enfermedad del nudo SA.
E. Síndrome de taquicardia/bradicardia.

Todas las siguientes entidades son causas reversibles de disfunción del nudo SA,
EXCEPTO:
A. Hipotermia.
B. Hipotiroidismo.
C. Hipertensión intracraneal.
D. Toxicidad por litio.
E. Radioterapia.
¿Cuál de las siguientes entidades constituye un factor de riesgo para que surja
tromboembolia en pacientes con la variante de taquicardia/bradicardia y síndrome de
seno enfermo?
A. Tener >50 años de vida.
B. Auriculomegalia.
C. Diabetes mellitus.
D. Mutación 20210 de protrombina.
E. Ninguno de los anteriores; no aumentó el riesgo de tromboembolia con la variante
de taquicardia/bradicardia del síndrome del seno patológico.
Las células normales dentro del nudo AV presentan una propiedad conocida como
conducción en decremento. Si usted desea demostrar esta propiedad durante un
estudio electrofisiológico: ¿qué maniobra practicaría?
A. Estimularía la aurícula derecha con disminución seriada de la duración del ciclo y
mediría el tiempo de conducción de estímulo-haz de His.
B. Estimularía el ventrículo y registraría los potenciales auriculares derechos.
C. Administraría 0.04 mg de atropina/kg de peso y registraría el tiempo de conducción
del nudo AV al medir el tiempo que tarda la estimulación auricular para llegar al haz
de His.
D. Administraría 10 mg de metoprolol por vía endovenosa y registraría el tiempo de
conducción del nudo AV al medir el tiempo que necesita el latido de estimulación
auricular para alcanzar el haz de His.
E. Administraría 12 mg de adenosina por vía intravenosa y registraría el tiempo de
recuperación del nudo AV.

Un varón de 87 años con el antecedente de hipertensión, tratado satisfactoriamente


de ese problema y de estenosis aórtica, comenzó a mostrar síntomas de la estenosis
en los últimos dos meses. Un día antes fue sometido a reemplazo quirúrgico de la
válvula aórtica y colocación de una válvula bioprotésica de 25 mm, con excelentes
resultados transoperatorios. Rápidamente se le separó de la circulación extracorporal
y se extubó en término de 24 horas. Con base en el protocolo quirúrgico
temporalmente se le dejaron los electrodos de estimulación epicárdica en la superficie
ventricular y recibió estímulos a razón de 90 lpm. Al revisar su estado en la mañana,
usted hace una pausa breve de su estimulación ventricular para cuantificar el ritmo
básico. Advierte una frecuencia auricular de 80 lpm pero la frecuencia ventricular es
de 32 lpm con complejo QRS amplio. No se advierte relación entre las ondas P y los
complejos QRS. ¿A cuál de los siguientes factores muy probablemente se debe la
bradicardia ventricular del paciente?

A. Aparición de una enfermedad sistémica como la sarcoidosis o la enfermedad de


Lyme que ocasionó disfunción del nudo AV.

B. Endocarditis de la nueva bioprótesis aórtica que ocasionó bloqueo del nudo AV.

C. Enfermedad del nudo SA.

D. Retraso de la recuperación del nudo AV después de la estimulación supraumbral.

E. Lesión quirúrgica del nudo AV.

La Sra. Hellwig es una mujer de 25 años con lupus eritematoso sistémico (SLE)
complicado por nefropatía, anemia hemolítica y pleuritis. Su enfermedad ha estado
controlada en forma satisfactoria con tratamiento. En fecha reciente se percató que
estaba embarazada y acudió el día de hoy para orientación prenatal. Le interesa
específicamente el efecto que ejerza su enfermedad autoinmunitaria en el producto
de la concepción. Usted indica que la complicación cardiaca más común en los hijos
de madres con lupus es:
A. Bloqueo AV.

B. Arteriopatía coronaria.

C. Miocardiopatía dilatada.

D. Hipertensión pulmonar con insuficiencia ventricular derecha (RV).

E. Endocarditis estéril de Libman-Sacks.

El Sr. Ho_man, un antiguo equilibrista, acude a su consultorio y señala que tuvo un


síncope. Afirma que en dos ocasiones de la semana pasada, espontáneamente se
desmayó, sin síntomas premonitorios. Una vez se golpeó la cara y usted advierte que
en la exploración hay equimosis periorbitaria. Salvo ese signo, no hay nada anormal
en la exploración. Solicita la práctica de un ECG y se detiene en la sala adjunta para
comenzar la documentación. Minutos más tarde un asistente le solicita su presencia
urgente en la sala clínica en que está el paciente. Tuvo otro “desvanecimiento”
durante el ECG y quedó inconsciente. Como un hecho casual, el asistente captó el
desvanecimiento en el ECG y se muestra en la ¿Qué tipo de bloqueo AV apareció y
cuál es la concordancia con el tratamiento o el método diagnóstico apropiado?

A. Bloqueo completo del nudo SA - Colocación de un marcapaso permanente.

B. Bloqueo AV de primer grado - Administrar atropina.

C. Bloqueo de nudo SA de tipo Mobitz II de segundo grado - Estudio ergométrico en


banda sin fin.

D. Bloqueo AV de tipo I Mobitz de segundo grado - No se necesitan intervenciones.

E. Bloqueo de nudo AV Mobitz tipo II de segundo grado - Colocación de marcapaso


permanente.

El Sr. Wilkins es un ingeniero aeronáutico de 65 años que nunca ha acudido a


un médico y que no recibe fármaco alguno. Fue llevado al departamento de
urgencias por su esposa, pues se quejó de cefalea cada vez más intensa en
las últimas 24 horas y confusión que comenzó una hora antes. En la primera
visita su presión arterial fue de 230/140 mmHg, su frecuencia cardiaca, de 90
lpm y su saturación de oxígeno arterial de 95%. En la exploración, mueve por
igual las cuatro extremidades y no hay alteraciones en los pares craneales,
pero muestra delirio. En la exploración cardiaca se identifica choque de punta
intenso y galope S4; los campos pulmonares están limpios a la auscultación.
Los datos de estudios de laboratorio indican creatinina de 2.4 mg/100 ml;
proteína 2+ en su orina con hematuria y valor hematócrito de 32% con
recuento normal de plaquetas. Se revisa un frotis de sangre periférica y se
detectó la presencia de esquistocitos. La MRI del cerebro de urgencia indicó
antiguos cambios microvasculares, pero no infarto ni hemorragia agudos. En
lo que toca al control de su presión arterial: ¿cuál de los siguientes
planteamientos podría ser el tratamiento más razonable?

A. 0.1 mg de clonidina por vía oral.

B. 20 mg de lisinopril por vía oral.

C. 20 mg de labetalol por vía IV y continuar el goteo intravenoso continuo para


alcanzar como objetivo una presión arterial media de 125 mmHg en la primera

hora.

D. 90 mg de nifedipino ingerible de liberación inmediata.

E. Plasmaféresis de urgencia.

¿Cuál de los planteamientos siguientes en cuanto a los aspectos fisiológicos


de riego por riñones es cierto?

A. Por la estricta regulación precapilar el endotelio capilar glomerular está


relativamente “protegido” del daño por la presión.

B. El flujo a la corteza renal es menor que el que llega a la médula y junto con las
grandes necesidades metabólicas de la corteza de ese órgano, deja a esta

región “al borde” de la hipoxemia.

C. El riego de riñones está regulado estrictamente con arreglo a las necesidades


metabólicas de dichos órganos.

D. La excreción de albúmina por orina permite pronosticar fenómenos


ateroescleróticos sistémicos y puede surgir muchos años antes de la enfermedad

ateroesclerótica clínica.

E. La sangre venosa que vuelve de la corteza renal tiene un menor contenido de


oxígeno que la misma sangre que vuelve de la médula de los riñones.

De los planteamientos siguientes respecto a la estenosis de arteria renal:


¿cuál es verdadero?

A. La disminución anormal de la velocidad de flujo a través de la arteria renal,


medida por ecografía Doppler permite anticipar que existe estenosis
hemodinámicamente importante.

B. En el comienzo de la estenosis de arteria renal es útil conservar mayores niveles


de renina a nivel general.

C. En la población general la presencia de displasia fibromuscular es rara (<1% de


prevalencia).

D. Los niveles de actividad de renina observados en la estenosis de arteria renal


permiten conocer anticipadamente la respuesta al tratamiento médico.

E. Por lo común la primera manifestación clínica de la estenosis renal por displasia


fibromuscular es la disminución de la función

Varón de 65 años a quien se practicó angiografía coronaria con fines


diagnósticos después de anomalías en una prueba de esfuerzo. Fue fácil tener
acceso a la arteria femoral derecha y se completó la angiografía con la
introducción de 35 ml de medio yodado de contraste. Por fortuna, no se
identificaron estenosis coronarias notables. Siete días más tarde el paciente
acudió al departamento de urgencias con dolor abdominal y náuseas. Indicó
que la orina que generaba había sido muy escasa recientemente. En la
exploración se detectó febrícula (38.3°C) y livedo reticularis en sus
extremidades inferiores. Los datos de estudios de laboratorio señalaron
creatinina de 2.7 mg/100 ml (en fecha previa, 1.1 mg/100 ml), recuento de 10
500 leucocitos/ml, de los cuales 21% eran eosinófilos y velocidad de
eritrosedimentación de 92 mm/h. ¿Cuál es la entidad diagnóstica más
probable?

A. Nefritis intersticial aguda.

B. Nefropatía ateroembólica.

C. Síndrome de Churg-Strauss.

D. Nefropatía inducida por medio de contraste.

E. Síndrome hipereosinófilo.

¿En cuál de los siguientes pacientes que acudió al departamento de urgencias


y señalan disnea aguda sería positiva una prueba de dímero D que obligara a
realizar estudios adicionales para identificar un émbolo pulmonar?

A. Mujer de 24 años con 32 semanas de gestación.


B. Varón de 48 años sin antecedentes personales que acudió por dolor en la
pantorrilla después de un viaje aeronáutico duradero; el gradiente de oxígeno
alveolo-arterial es normal.

C. Mujer de 56 años a quien se practica quimioterapia por cáncer de mama.

D. Varón de 62 años que fue sometido a reemplazo de cadera cuatro semanas


antes.

E. Varón de 72 años que dos semanas antes había tenido un infarto agudo del
miocardio

Mujer de 42 años que acudió al departamento de urgencias por disnea, de comienzo


agudo. En fecha reciente acudió a visitar a sus padres fuera de su Estado y condujo
un automóvil unas nueve horas de ida y de vuelta. Hace dos días presentó dolor
leve e hinchazón de la pantorrilla, pero se le comentó era un signo usual después de
haber estado sentada con las piernas en posición declive en su viaje reciente. Al
llegar al departamento de urgencias, se detectó taquipnea. Sus signos vitales son:
presión arterial, 98/60 mmHg; frecuencia cardiaca, 114 lpms, 28
respiraciones/minuto; saturación de oxígeno, 92% con respiración de aire ambiental
y 89 kg de peso. Los campos pulmonares están limpios. Hay dolor en la pantorrilla
derecha con la dorsiflexión del pie y la pierna derecha está más hinchada que la
izquierda. La medición de gases en sangre arterial indicó pH de 7.52; PCO2 de 25
mmHg y PO2 de 68 mmHg. La función hepática y renal son normales. Los datos de
CT helicoidal confirman la presencia de un émbolo pulmonar. Todos los productos
siguientes pueden utilizarse solos como fármaco inicial en esta paciente EXCEPTO:

A. Enoxaparina, 1 mg/kg de peso por vía subcutánea (SC) cada 12 h.

B. Fondaparinux, 7.5 mg por vía SC cada 24 h.

C. Tinzaparina, 175 unidades/kg por vía SC cada 24 h.

D. Heparina no fraccionada por vía IV ajustada para que el tiempo de tromboplastina


parcial activada (aPTT) sea dos a tres veces mayor que el límite normal

superior.

E. Warfarina, 7.5 mg ingeridos cada 24 h para que la razón internacional


normalizada sea de 2 a 3.

¿Cuál de los planteamientos siguientes sobre la embolia pulmonar es


verdadero?
A. La resistencia de vías respiratorias por lo común disminuye en casos de embolia
pulmonar aguda.

B. Prácticamente todos los pacientes con embolias pulmonares muestran signos de


trombosis de vena profunda en esa ocasión.

C. La hiperventilación alveolar es una anormalidad funcional típica en presencia de


embolia pulmonar.

D. La hipertensión en el marco de la embolia pulmonar aguda suele provenir de


disfunción sistólica aguda de ventrículo izquierdo.

E. La magnitud del gradiente arterial/alveolar guarda relación invariablemente con el


diámetro del émbolo pulmonar.

Un varón de 57 años con antecedente de hipertensión fue hospitalizado por


embolia pulmonar después de que de forma repentina comenzó a sentir
molestias retroesternales. Su presión arterial es de 132/62 mmHg; su
frecuencia cardiaca es de 85 lpm y la saturación de oxígeno es de 95% con
respiración de aire ambiental. En la ecografía se observa tamaño y función
normales de las cavidades derechas del corazón y no se detecta troponina I
cardiaca. La ecografía Doppler de las extremidades inferiores señala una
trombosis profunda extensa, que afecta en la vena femoropoplítea derecha. Se
comienza la administración concomitante de heparina de bajo peso molecular
y warfarina. Al revisar a la paciente el día siguiente él indica si sería apropiada
la colocación de un filtro en vena cava inferior (IVC) ¿cuál sería la respuesta
más apropiada?

A. “No, porque en este momento no obtendría usted beneficio de dicho filtro”.

B. “No, no existen por ahora indicaciones para colocar el filtro en el marco de


trombosis venosa profunda o embolia pulmonar”.

C. “Es necesario revisar de nuevo las imágenes obtenidas del volumen máximo del
émbolo pulmonar antes de decidir si es adecuada o no la colocación de

un filtro en la vena cava inferior”.

D. “Sí, dada la presencia de un trombo residual en extremidades inferiores se le


programará para colocación de filtro en la vena cava inferior”.

E. “Sí, dada la edad y el género de usted, lo programaremos para que se le coloque


un filtro en la vena cava inferior”.

La Sra. Tupulo tiene 45 años de edad y antecedente de obesidad mórbida (140


kg de peso), hipertensión y diabetes. Estuvo internada en el hospital durante
cuatro días para que se le hiciera una operación de derivación gástrica abierta.
El cuarto día del posoperatorio presentó repentinamente disnea, taquicardia,
hipoxia y una sensación abrumadora de muerte. En la angiografía por CT
urgente de tórax se detectó un émbolo pulmonar “a horcajadas”. Su presión
arterial es de 70/45 mmHg y hay oliguria. Se administraron por vía IV 500 ml de
solución salina normal y comenzó la administración de 5 μg de dobutamina/kg
de peso/minuto; la presión arterial después de iniciado este tratamiento fue de
82/55 mmHg. ¿cuál es la medida siguiente más apropiada?

A. Administrar enoxaparina.

B. Colocar un filtro en IVC.

C. Embolectomía pulmonar quirúrgica.

D. tPA.

E. Urocinasa.

Un varón de 67 años, con antecedente de tabaquismo intenso, está en el cuarto día


posoperatorio después de una operación de derivación aortobifemoral abierta por
claudicación intensa. En la CT sistemática en el posoperatorio antes del alta se
identificó un seudoaneurisma en sentido proximal al nacimiento de los injertos de
derivación en la aorta. De los planteamientos siguientes: ¿cuál define mejor a un
seudoaneurisma?

A. Dilatación focal en un vaso en que hay rotura de las capas íntima y media y el
segmento dilatado está cubierto únicamente por la capa adventicia.

B. Una dilatación focal de un vaso que abarca sólo una parte de su circunferencia.

C. Dilatación de un vaso por estrechamiento intrínseco en sentido proximal y distal


al punto de estenosis manifiesta.

D. Dilatación de un vaso, aunque no hasta el tamaño necesario como para ser


diagnosticada como aneurisma verdadero.

E. Aparición de una dilatación aneurismática de un vaso en estudios imagenológicos


causada por la angulación de la arteria y la técnica imagenológica.

Los factores siguientes se acompañan de un mayor riesgo de que surja un


aneurisma aórtico, EXCEPTO:

A. Envejecimiento.

B. Tabaquismo (cigarrillos).

C. Género femenino.
D. Hipercolesterolemia.

E. Hipertensión.

Se atiende al Sr. Walker en la clínica, en su cita de vigilancia. Es un varón de


22 años con el antecedente familiar de muerte repentina y aneurismas
arteriales. Es una persona muy alta con ojos muy separados y úvula bífida. En
su primera visita usted ordenó la práctica de un ecocardiograma transtorácico
por la presencia de un soplo diastólico y se detectó un aneurisma de 4.9 cm
en la base de la aorta. ¿En cuál de los genes siguientes muy probablemente
muestre una mutación?

A. Fibrillina-1

B. SMAD3.

C. α-actinina específica de músculo liso.

D. Receptor β de TGF.

E. Procolágeno de tipo III.

De los siguientes pacientes con disección o hematoma aórticos: ¿cuál será


tratado mejor sin medidas quirúrgicas?

A. Mujer de 45 años con disección que abarca la aorta en sentido distal al


nacimiento de dicho gran vaso, pero en sentido superior a las arterias renales.

B. Varón de 74 años con disección que abarca la base de la aorta.

C. Varón de 58 años con disección que abarca la zona distal de la aorta y ambas
arterias renales.

D. Varón de 69 años con hematoma intramural dentro de la base de la aorta.

E. Todos los pacientes mencionados necesitan tratamiento quirúrgico de su


aortopatía.

Una mujer de 32 años es atendida en el departamento de urgencias por disnea


aguda. En la CT helicoidal no se detectaron signos de embolia pulmonar,
peroaccidentalmente el operador se percata de dilatación de aorta ascendente
hasta un nivel de 4.3 cm. Todos los planteamientos siguientes guardan
relación con tal hallazgo, EXCEPTO:

A. Arteritis de células gigantes.

B. Artritis reumatoide.
C. Sífilis.

D. Lupus eritematoso sistémico.

E. Arteritis de Takayasu.

El Sr. Tomazelli tiene 75 años de vida, diabetes mellitus e hipertensión. A


pesar de los consejos ininterrumpidos sigue fumando cigarrillos sin filtro,
importados. En la visita del día de hoy señaló dolor ardoroso en ambas
pantorrillas con la ambulación, que típicamente aparece después de haber
caminado dos cuadras de la ciudad en terreno plano, pero que mejora con el
reposo. También se percató de molestias en la pantorrilla durante la noche,
que mejoraban cuando se sentaba en la cama. De los planteamientos
siguientes: ¿cuál posiblemente sea el verdadero en este paciente?

A. Posiblemente tiene una estenosis arterial aortoiliaca crítica.

B. La amplitud del control volumétrico de su pulso será nítido y con picos.

C. La causa más probable de sus síntomas es la displasia fibromuscular de arteria


femoral.

D. El método diagnóstico más razonable por practicar es la angiografía por


resonancia magnética.

E. La razón de la presión arterial sistólica en el tarso, en comparación con la medida


en la arteria humeral, posiblemente sea menor de 0.9

El Sr. Tomazelli tiene 75 años de vida, diabetes mellitus e hipertensión. A pesar de


los consejos ininterrumpidos sigue fumando cigarrillos sin filtro, importados. En la
visita del día de hoy señaló dolor ardoroso en ambas pantorrillas con la ambulación,
que típicamente aparece después de haber caminado dos cuadras de la ciudad en
terreno plano, pero que mejora con el reposo. También se percató de molestias en
la pantorrilla durante la noche, que mejoraban cuando se sentaba en la cama. El
diagnóstico de este paciente es el de arteriopatía periférica sintomática (PAD), con
índices bilaterales
tarsianos/humerales de 0.82. Él solicita que se le comunique el pronóstico y también
cuál será el siguiente paso en el tratamiento. De los planteamientos siguientes en
cuanto a su pronóstico y tratamiento: ¿cuál es verdadero?

A. La anticoagulación con warfarina es mejor que el uso de agentes antiplaquetarios


para evitar trastornos cardiovasculares adversos en personas con PAD.
B. Se ha demostrado que los antagonistas β-adrenérgicos empeoran la isquemia de
extremidades y no deben utilizarse en PAD.

C. El paciente debe ejercitarse todos los días, caminar hasta el punto de


claudicación máxima antes de interrumpir su actividad y reposar y permitir que
muestren resolución sus síntomas.

D. El máximo riesgo en cuanto a morbilidad y mortalidad en los cinco años


próximos, entraña la evolución hasta llegar a la isquemia crítica de extremidades.

E. Los vasodilatadores constituyen los fármacos de primera línea en caso de PAD


sintomática.

Una mujer de 37 años sin antecedentes personales patológicos de importancia,


salvo un soplo en la niñez, fue sometida a valoración por dolor intenso de comienzo
repentino en su extremidad inferior derecha. En la exploración se advierte que se
trata de una persona joven con fascies de dolor, con signos vitales normales,
excepto frecuencia cardiaca de 110 lpm. En su pierna derecha hay palidez en
sentido distal hasta la rodilla, está fría al tacto y no se palpan los pulsos dorsales del
pie. El miembro izquierdo es normal. De los estudios siguientes: ¿con cuál muy
probablemente se diagnosticará la causa primaria del cuadro inicial de la paciente?
A. Angiografía de la extremidad inferior derecha.
B. Cultivos de sangre.
C. Ecocardiograma con contraste con burbujas.
D. Medición de anticuerpos séricos contra citoplasma de neutrófilos (c-ANCA).
E. Ecografía del sistema venoso de la extremidad superior derecha.

Un capataz de construcción de 32 años de edad, acudió por dolor con el esfuerzo


con ambos antebrazos y manos. Fuma una cajetilla de cigarrillos todos los días,
pero por lo demás no tiene antecedentes personales patológicos de importancia. Su
pulso humeral se palpa fácilmente, pero los pulsos radiales y cubitales son débiles.
En la Figura V-162 se incluye una imagen de la mano. La angiografía de la
extremidad superior indica lesiones segmentarias con estrechamiento cada vez
mayor en la zona distal de arteriolas. De los tratamientos siguientes: ¿cuál tiene las
mayores posibilidades de lograr buenos
resultados?

A. Cilostazol.
B. Enoxaparina.
C. Prednisona.
D. Dejar el cigarrillo.
E. Warfarina
De los planteamientos siguientes respecto a enfermedades venosas crónicas:
¿cuál es verdadero?

A. Un número mayor de varones que el de mujeres tienen venas varicosas.


B. Un número mayor de varones que el de mujeres tiene insuficiencia venosa
crónica con edema.
C. Más de 50% de los pacientes mayores de 70 años de edad tienen insuficiencia
venosa crónica.
D. Casi todos los pacientes de insuficiencia venosa crónica terminarán por mostrar
úlceras venosas.
E. Todos los planteamientos anteriores son verdaderos.

De las siguientes venas del cuerpo: ¿cuál es la más larga?


A. Vena ázigos.
B. Vena safena interna.
C. Vena cava inferior.
D. Vena ilíaca primitiva izquierda.
E. Vena femoral derecha.

Se valoró a una mujer de 77 años de Ohio con antecedente de insuficiencia


cardiaca, aunque con conservación de la fracción de expulsión; también tiene
diabetes mellitus y tiene el antecedente de trombosis venosa profunda en la
extremidad inferior izquierda, después de un ciclo con anticoagulantes. En los
últimos meses señaló presentar calambres y ardor en las piernas. En esta ocasión
acudió por la úlcera cutánea que se muestra en la Figura V-165. En la exploración
física se advierte que hay una úlcera cutánea en el maléolo interno y edema duro
bilateral en el tarso y zonas tibiales. La piel también está más oscura. De los
planteamientos siguientes: ¿cuál tiene mayor probabilidad de causar la úlcera?

A. Picadura de una araña venenosa.


B. Infección por Bacillus anthracis.
C. Insuficiencia venosa crónica.
D. Úlcera de pie de diabético.
E. Arteriopatía periférica.

Se valora a una mujer de 19 años en una clínica universitaria. Ella señala hinchazón
no dolorosa de la extremidad pélvica. En la exploración se advierte que la pierna
muestra engrosamiento de la piel y una con textura dura. La ecografía directa
confirma que las venas de la extremidad pélvica tienen buena circulación y no
muestran trombos. Se sospecha la presencia de linfedema y piensa que
posiblemente proviene de la causa más común de este trastorno secundario a nivel
mundial. ¿Cuál es la causa que se sospecha?
A. Cáncer que abarca los ganglios inguinales.
B. Linfogranuloma venéreo.
C. Filariasis linfática.
D. Linfangitis bacteriana recurrente.
E. Tuberculosis.
Usted atiende a la persona que muestra linfedema crónico causada por linfangitis
estreptocócica recurrente desde niña. Para ella es antiestética la hinchazón de sus
piernas y solicita que le planteen opciones terapéuticas. Presentamos varias de
ellas que son razonables en caso de linfedema crónico, EXCEPTO:
A. Fisioterapia para descongestión.
B. Administración de diuréticos.
C. Elevación frecuente de las piernas.
D. Dispositivos de compresión neumática intermitente.
E. Liposucción.

INFECTO SÁNCHEZ

Pregunta 1

Correcta

Puntúa 1,00 sobre 1,00

Marcar pregunta
Enunciado de la pregunta

Un varón de 19 años consulta por un cuadro de febrícula, mialgias y odinofagia,


acompañado de máculas cutáneas generalizadas no pruriginosas durante los
últimos 7 días. En el interrogatorio refiere, como único suceso médico, haber
presentado una úlcera anal que le causaba dolor a la defecación en el mes anterior
que desapareció progresivamente sin ninguna intervención. ¿Cuál es el germen
causante de este cuadro clínico?

Seleccione una:

a. Virus de herpes simple tipo 2.

b. Chlamydia trachomatis.

c. Gonococo.

d. Treponema pallidum.

Retroalimentación

La respuesta correcta es: Treponema pallidum.


Pregunta 2

Correcta

Puntúa 1,00 sobre 1,00

Marcar pregunta
Enunciado de la pregunta

¿Qué es el dengue?

Seleccione una:

a. Una zoonosis que afecta al hombre ocasionalmente.

b. Una enfermedad vírica que ocasiona un eritema que evoluciona a mácula y


pápula afectando fundamentalmente a la población infantil.

c. Una enfermedad causada por un poxvirus.

d. Una enfermedad vírica que puede producir una fiebre hemorrágica.

Retroalimentación

La respuesta correcta es: Una enfermedad vírica que puede producir una fiebre
hemorrágica.

Pregunta 3

Incorrecta

Puntúa 0,00 sobre 1,00

Marcar pregunta

Enunciado de la pregunta

Si durante el examen físico en el área genital encuentra una úlcera lisa no purulenta
bien delimitado única superficial con linfadenopatía bilateral usted piensa en:

Seleccione una:

a. Leishmania

b. sífilis

c. chancroide

d. herpes tipo 2
Retroalimentación

La respuesta correcta es: sífilis

Pregunta 4

Correcta

Puntúa 1,00 sobre 1,00

Marcar pregunta
Enunciado de la pregunta

¿Cuál de los siguientes componentes de la estructura de los virus gripales es el


principal responsable de su infecciosidad?

Seleccione una:

a. Hemaglutinina.

b. ARN polimerasa.

c. Neuraminidasa.

d. Envoltura lipídica.

Retroalimentación

La respuesta correcta es: Hemaglutinina.

Pregunta 5

Correcta

Puntúa 1,00 sobre 1,00

Marcar pregunta
Enunciado de la pregunta

Lesión de Ghon es:

Seleccione una:

a. linfadenopatia de cuello

b. pequeño nódulo calcificado

c. Ganglio de ápice de pulmón


d. Diseminación miliar de tuberculosis.

Retroalimentación

La respuesta correcta es: pequeño nódulo calcificado

Pregunta 6

Correcta

Puntúa 1,00 sobre 1,00

Marcar pregunta
Enunciado de la pregunta

El interferón es un medicamento útil para las siguientes infecciones excepto:

Seleccione una:

a. VIH

b. Sarcoma de Kaposi

c. HPV

d. Hepatitis B

Retroalimentación

La respuesta correcta es: VIH

Pregunta 7

Correcta

Puntúa 1,00 sobre 1,00

Marcar pregunta
Enunciado de la pregunta

La tuberculosis osteoarticular ataca principalmente a columna a lo que se denomina:

Seleccione una:

a. Enfermedad de Wegener

b. Osteofitosis
c. Mal de Pott

d. Trastorno de Jacob

Retroalimentación

La respuesta correcta es: Mal de Pott

Pregunta 8

Correcta

Puntúa 1,00 sobre 1,00

Marcar pregunta

Enunciado de la pregunta

Cuál es el grupo sanguíneo y con mayor susceptibilidad al contagio de cólera:

Seleccione una:

a. Grupo AB

b. Grupo A

c. Ninguno

d. Grupo O

Retroalimentación

La respuesta correcta es: Grupo O

Pregunta 9

Correcta

Puntúa 1,00 sobre 1,00

Marcar pregunta
Enunciado de la pregunta

Paciente diabética de 65 años de edad que consulta por un cuadro de dolor en la


cara, fiebre y aparición de una lesión inflamatoria bien delimitada que
diagnosticamos de erisipela. ¿Cuál es la etiología del proceso?

Seleccione una:
a. Staphylococcus aureus.

b. Staphylococcus epidermidis.

c. Streptococcus agalactiae.

d. Streptococcus pyogenes.

Retroalimentación

La respuesta correcta es: Streptococcus pyogenes.

Pregunta 10

Incorrecta

Puntúa 0,00 sobre 1,00

Marcar pregunta
Enunciado de la pregunta

Artritis reactiva es una complicación de:

Seleccione una:

a. Salmonelosis

b. Entamoeba histolytica

c. Shigella

d. Giardiasis

Retroalimentación

La respuesta correcta es: Shigella

Pregunta 11

Incorrecta

Puntúa 0,00 sobre 1,00

Marcar pregunta
Enunciado de la pregunta

Con respecto a la infección por gripe indiqué la premisa verdadera:


Seleccione una:

a. casi siempre se asocia con febrícula, o sin fiebre sino con tos seca

b. la Clínica generalmente se asocia a trastornos respiratorios altos

c. puede causar una neumonía viral primaria

d. El virus solamente tiene mutaciones puntuales

Retroalimentación

La respuesta correcta es: puede causar una neumonía viral primaria

Pregunta 12

Correcta

Puntúa 1,00 sobre 1,00

Marcar pregunta

Enunciado de la pregunta

Que caracteriza a una primoinfección por herpes tipo 1:

Seleccione una:

a. trastornos de córnea y conjuntiva

b. faringitis o gingivoestomatitis

c. distribución en dermatoma en especial tórax

d. pápulas y pústulas intersticio entre piel y mucosa

Retroalimentación

La respuesta correcta es: faringitis o gingivoestomatitis

Pregunta 13

Correcta

Puntúa 1,00 sobre 1,00

Marcar pregunta
Enunciado de la pregunta
En relación al tratamiento de la sífilis, ¿cuál de las siguientes es la respuesta
correcta?

Seleccione una:

a. La penicilina es el tratamiento de elección en cualquier estadio.

b. La penicilina es el tratamiento de elección sólo en la neurosífilis.

c. No hay ningún tratamiento efectivo.

d. El tratamiento de elección son las tetraciclinas.

Retroalimentación

La respuesta correcta es: La penicilina es el tratamiento de elección en cualquier


estadio.

Pregunta 14

Correcta

Puntúa 1,00 sobre 1,00

Marcar pregunta

Enunciado de la pregunta

Ante un niño de 7 meses con fiebre e irritabilidad, fontanela abombada y un estudio


de líquido cefalorraquídeo con 110 células/ mm 3(75% linfocitos), proteínas 120
mg/dl y glucosa 28 mg/dl (glucemia sérica 89 mg/ dl), ¿cuál es la sospecha
diagnóstica más razonable?

Seleccione una:

a. Síndrome mononucleósico.

b. Meningitis bacteriana.

c. Meningitis vírica.

d. Meningitis tuberculosa.

Retroalimentación

La respuesta correcta es: Meningitis tuberculosa.

Pregunta 15
Correcta

Puntúa 1,00 sobre 1,00

Marcar pregunta
Enunciado de la pregunta

Mujer de 47 años de edad, nacida y residente en Orellana A mediados del año 2017
comienza con palpitaciones, disnea y edemas progresivos en extremidades
inferiores. Es diagnosticada de insuficiencia cardíaca secundaria a miocardiopatía
por Chagas. ¿Cuál es el agente etiológico de esta enfermedad endémica?

Seleccione una:

a. Trypanosoma brucei.

b. Trypanosoma cruzi.

c. Leishmania donovani.

d. Giardia lamblia.

Retroalimentación

La respuesta correcta es: Trypanosoma cruzi.

Pregunta 16

Correcta

Puntúa 1,00 sobre 1,00

Marcar pregunta

Enunciado de la pregunta

Su paciente presenta las siguientes características fiebre alta prolongada dolor


abdominal diarrea y estreñimiento ocasionales esplenomegalia y roséola usted
sospecha de:

Seleccione una:

a. Giardiasis

b. Shigella

c. Salmonelosis
d. Entamoeba histolytica

Retroalimentación

La respuesta correcta es: Salmonelosis

Pregunta 17

Incorrecta

Puntúa 0,00 sobre 1,00

Marcar pregunta
Enunciado de la pregunta

En el análisis de líquido cefalorraquídeo usted encuentra linfocitos aumentados,


glucosa disminuida, usted podría pensar en que la etiología es:

Seleccione una:

a. bacteriana

b. parasitaria

c. Micobacterias

d. viral

Retroalimentación

La respuesta correcta es: Micobacterias

Pregunta 18

Correcta

Puntúa 1,00 sobre 1,00

Marcar pregunta
Enunciado de la pregunta

Erisipela una infección de tejido blando es debido principalmente a:

Seleccione una:

a. S. aureus

b. Enterobacterias
c. Pseudomona aeruginosa

d. S. pyogenes

Retroalimentación

La respuesta correcta es: S. pyogenes

Pregunta 19

Correcta

Puntúa 1,00 sobre 1,00

Marcar pregunta

Enunciado de la pregunta

En la malaria, la forma de plasmodio transmitida del mosquito al hombre es el:

Seleccione una:

a. Esporozoito.

b. Hipnozoito.

c. Gametocito.

d. Merozoito.

Retroalimentación

La respuesta correcta es: Esporozoito.

Pregunta 20

Incorrecta

Puntúa 0,00 sobre 1,00

Marcar pregunta
Enunciado de la pregunta

A qué grupo poblacional no restringiría la vacuna de la gripe:

Seleccione una:

a. Niños con antecedentes de síndrome de Reye


b. Personas con trastornos crónicos metabólicos

c. Personas con antecedente de Guillain Barré

d. Personas alérgicas a las proteínas del huevo

Retroalimentación

La respuesta correcta es: Personas alérgicas a las proteínas del huevo

Pregunta 21

Correcta

Puntúa 1,00 sobre 1,00

Marcar pregunta
Enunciado de la pregunta

La infección de herpes en recién nacidos es principalmente dado por:

Seleccione una:

a. VHS 1

b. HVZ

c. VHH 8

d. VHS 2

Retroalimentación

La respuesta correcta es: VHS 2

Pregunta 22

Correcta

Puntúa 1,00 sobre 1,00

Marcar pregunta
Enunciado de la pregunta

Cuál consideraría como primera opción para el diagnóstico de fiebre tifoidea ( fiebre
enterica):

Seleccione una:
a. hemocultivo

b. rosa de bengala

c. biopsia intestinal

d. Reacción de Widal

Retroalimentación

La respuesta correcta es: hemocultivo

Pregunta 23

Correcta

Puntúa 1,00 sobre 1,00

Marcar pregunta
Enunciado de la pregunta

En caso que se enfrente a una situación de tétanos una medida de soporte


importante para el control de espasmos es:

Seleccione una:

a. Diazepam

b. Antitoxina

c. Metronidazol

d. Gabapentina

Retroalimentación

La respuesta correcta es: Diazepam

Pregunta 24

Correcta

Puntúa 1,00 sobre 1,00

Marcar pregunta
Enunciado de la pregunta
Joven de 23 años, que trabaja como DJ en un bar nocturno y tiene una historia de
promiscuidad bisexual e ingesta de drogas sintéticas, consulta por fiebre, malestar
general y dolor en glande. La exploración física muestra múltiples tatuajes, piercings
y la presencia de tres lesiones vesiculosas en glande y adenopatías inguinales
bilaterales. ¿Cuál es el diagnóstico más probable?

Seleccione una:

a. Condilomas acuminados.

b. Infección por citomegalovirus.

c. Primoinfección VIH.

d. Herpes genital.

Retroalimentación

La respuesta correcta es: Herpes genital.

Pregunta 25

Correcta

Puntúa 1,00 sobre 1,00

Marcar pregunta
Enunciado de la pregunta

Un trabajador en el oleoducto en el Coca de 36 años de edad, sufre desde hace dos


meses un cuadro abdominal intermitente de náuseas, heces pastosas, flatulencia,
meteorismo y que le han llevado a perder tres kg de peso. ¿Cuál de los siguientes
microorganismos sería con mayor probabilidad el responsable del cuadro?

Seleccione una:

a. Trichomonas hominis.

b. Giardia lamblia

c. Entamoeba coli.

d. Vibrio cholerae.

Retroalimentación

La respuesta correcta es: Giardia lamblia


Pregunta 26

Correcta

Puntúa 1,00 sobre 1,00

Marcar pregunta
Enunciado de la pregunta

Para el tratamiento de Leishmania cutánea de primera línea se utiliza:

Seleccione una:

a. Fluconazol

b. Antimonio pentavalente

c. Dapsona

d. Anfotericina B

Retroalimentación

La respuesta correcta es: Antimonio pentavalente

Pregunta 27

Correcta

Puntúa 1,00 sobre 1,00

Marcar pregunta

Enunciado de la pregunta

Cuando la infección de la Tripanosoma cruzi ingresa por piel puede haber una
reacción inflamatoria en el sitio de la infección que dura hasta ocho semanas
denominado:

Seleccione una:

a. Complejo oftalmo ganglionar del Chagas.

b. Chagoma

c. Signo de Romaña - Mazza.

d. Reacción de Fernández.
Retroalimentación

La respuesta correcta es: Chagoma

Pregunta 28

Correcta

Puntúa 1,00 sobre 1,00

Marcar pregunta
Enunciado de la pregunta

Con respecto al tétanos, ¿cuál de las siguientes frases es la correcta?

Seleccione una:

a. El tétanos está caracterizado por una rigidez generalizada junto a crisis de


espasmos musculares.

b. La mortalidad del tétanos es todavía en la actualidad, superior al 50%.

c. En el diagnóstico del tétanos, es fundamental haber efectuado cultivos previos


al inicio del tratamiento.

d. El tétanos aparece tras un periodo mínimo de incubación de 14 días.

Retroalimentación

La respuesta correcta es: El tétanos está caracterizado por una rigidez generalizada
junto a crisis de espasmos musculares.

Pregunta 29

Correcta

Puntúa 1,00 sobre 1,00

Marcar pregunta

Enunciado de la pregunta

¿Cuál de los siguientes microorganismos que infectan el tracto intestinal causa


bacteriemia con mayor frecuencia?

Seleccione una:

a. Campylobacter jejuni.
b. Shigella flexneri

c. Vibrio cholerae.

d. Salmonella

Retroalimentación

La respuesta correcta es: Salmonella

Pregunta 30

Correcta

Puntúa 1,00 sobre 1,00

Marcar pregunta
Enunciado de la pregunta

Paciente que acude a Urgencias con fiebre intermitente, escalofríos, cefalea,


debilidad y abundante sudoración nocturna de 3 días de evolución. Como
antecedente epidemiológico destaca un viaje a Cayambe hace 3 semanas donde
consumieron leche cruda y queso sin pasteurizar. El modo más rápido de
diagnosticar la infección sospechada es:

Seleccione una:

a. Prueba del Rosa de Bengala y toma de hemocultivo.

b. Test para detección de anticuerpos heterófilos (Paul-Bunnell).

c. Tinciones de micobacterias en esputo u orina.

d. Gram directo del líquido cefalorraquídeo.

Retroalimentación

La respuesta correcta es: Prueba del Rosa de Bengala y toma de hemocultivo.

Pregunta 31

Incorrecta

Puntúa 0,00 sobre 1,00

Marcar pregunta
Enunciado de la pregunta
Qué patología de las siguientes es una condición define Sida:

Seleccione una:

a. herpes tipo 1: úlceras crónicas > 1 mes

b. displasia cervical

c. candidiasis oral

d. leucoplasia vellosa

Retroalimentación

La respuesta correcta es: herpes tipo 1: úlceras crónicas > 1 mes

Pregunta 32

Correcta

Puntúa 1,00 sobre 1,00

Marcar pregunta
Enunciado de la pregunta

Ante los hallazgos que cabe esperar del análisis del líquido cefalorraquídeo en un
cuadro de meningitis bacteriana, NO se encuentra:

Seleccione una:

a. Cultivo positivo en el 40% de los casos

b. Abundantes hematíes.

c. Glucosa < 40 mg/dl.

d. Proteínas > 45 mg/dl.

Retroalimentación

La respuesta correcta es: Abundantes hematíes.

Pregunta 33

Correcta

Puntúa 1,00 sobre 1,00

Marcar pregunta
Enunciado de la pregunta

En la clasificación de FOD cuál considera usted como urgente y debe ser tratada de
inmediato:

Seleccione una:

a. FOD Clásico

b. FOD asociado a HIV

c. FOD neutropénico

d. FOD Nosocomial

Retroalimentación

La respuesta correcta es: FOD neutropénico

Pregunta 34

Correcta

Puntúa 1,00 sobre 1,00

Marcar pregunta

Enunciado de la pregunta

Tras la infección, ¿qué plasmodios parásitos humanos persisten como hipnozoitos


en el hígado y pueden causar recidivas?

Seleccione una:

a. P. falciparum.

b. P. cynmology y P. knowlesi.

c. P. malariae.

d. P. vivax y P. ovale.

Retroalimentación

La respuesta correcta es: P. vivax y P. ovale.

Pregunta 35

Correcta
Puntúa 1,00 sobre 1,00

Marcar pregunta

Enunciado de la pregunta

La BCG es una vacuna atenuada derivada del:

Seleccione una:

a. M. tuberculosis

b. M. avium - intracellulare

c. M. leprae

d. M. bovis

Retroalimentación

La respuesta correcta es: M. bovis


ISAMAR CASTRO

Se describen síntomas y signos más frecuentes en la Enfermedad por Coronavirus


COVID19 entre éstos cuál es el que se presenta con más frecuencia :

a) Tos seca (50,7%)

b) Fiebre (87,9%)

c) Astenia (99,1%)

d) Expectoración (33,4%)

ALEJANDRA CORVALAN
¿Cuál es el receptor al que se une el virus para ingresar a las células pulmonares?

-
a. AT1
b. ECA
c. AT2
d. Bradicinina

NADIA FALCÓN
Cuál es la respuesta a la que puede conducir la hiperactivación de macrofagos y
granulocitos que puede conllevar a la muerte del paciente:
A. Síndrome de liberación de inmunoglobulinas
B. Síndrome de liberación de linfocitos T
C. Síndrome de liberación de citoquinas
D. Síndrome de liberación de factores de crecimiento.

LUIS GRANDA
Seleccione la respuesta correcta.
Dentro de las especies del género Plasmodium que pueden causar infecciones palúdicas en
seres humanos se incluyen:

a) Falciparum, Vivax, Ovale,Malariae, Knowlesi.


b) Falciparum, Vivax, Marburg, Ovale
c) DENV-1, DENV-2, DENV-3, DENV-4
d) Cruzi, Gambiense, Rhodesiense
ANTONIO GUZMAN

La malaria es una de las enfermedades endémicas en nuestro país. ¿Cuál de estos


enunciados es falsa?:
a) La gravedad de la infección se relaciona con el grado de parasitemia
b) La infección por Plasmodium falciparum es la presentación más grave.
c) La malaria cerebral es una de las manifestaciones típicas por Plasmodium vivax
d) El dato analítico más frecuente en la malaria es una anemia normocítica

SEBASTIAN LARA

Señale la respuesta correcta en relación a la Malaria:

a) Plasmodium ovale es el causante de la mayoría de las muertes por malaria


b) El examen de gota gruesa o de extendido de sangre periférica no sirven para el
diagnóstico.
c) El Anopheles infectado, al picar, inoculan los hipnozoitos, forma infectante del
parásito
d) Es una enfermedad causada por protozoarios del género Plasmodium

GLORIA LOYA
Cual es una característica principal para que el dengue pueda diseminarse?:

A. El vector es muy cercano a los asentamientos humanos.


B. Debido a la gran capacidad de diseminación del vector Anopheles.
C. Una característica precoz es la trombocitosis y neutrofilia.
D. Es sumamente frecuente la progresión de daño encefálico.

ALEXIS MENA

Uno de los criterios de DENGUE SEVERO es:


a) Dolor abdominal
b) Tos
c) Fiebre
d) Exantema

EVELIN ORTEGA
¿En cuánto tiempo se puede producir la muerte del paciente por dengue hemorrágico?
a) 7- 8 horas
b) 12 -24 horas
c) 10 - 12 horas
d) 24 - 48 horas
SONIA PANTOJA

¿A qué familia y género pertenece el virus del sarampión?


a) familia Flaviviridae - género Flavivirus
b) familia Herpesviridae - género Varicellovirus
c) familia Paramyxoviridae - género Morbillivirus
d) familia Togaviridae - género Rubivirus

RAUL PEÑAFIEL
¿Cuál de los siguientes signos y síntomas son considerados el patognomónico del
sarampión?
a)Fiebre entre 39 y 40 °C
b) Conjuntivitis
c) lesiones de Köplik
d) Coriza y tos

GEOVANNY RAYO
El segundo microorganismo más frecuente causante de neumonía en la comunidad en
población adulta.
a) haemophilus influenzae
b) estafilococo weeks
c) estreptococo pneumoniae
d) mycobacterium tuberculosis

Respuesta: A

Respecto al esquema permanente de vacunación de sarampión y rubéola.


a) la primera dosis debe darse al primer mes de nacimiento.
b) la primera dosis debe darse entre los 6-9 meses de edad.
c) la primera dosis debe darse entre los 12 -23 meses de edad.
d) la primera dosis debe darse después de los 5 años de edad.

Respuesta: C

KIMBERLY ROMAN

1.- Paciente de 25 años, presenta fiebre, tos, malestar general durante una semana, acude
el dia de hoy a consulta ,se le realizaron examen de esputo en el cual se detectó ?: según la
literatura cuál es el germen más común que causa neumonía:
1. Pneumocystis jiroveci
2. virus de la gripe
3. streptococcus pneumoniae
4. Pseudomona aeruginosa
JONATHAN SALAZAR

Un paciente de 30 años adicto a drogas por vía parenteral, se presenta con un cuadro de 3
días de evolución, fiebre, dolor torácico y tos con expectoración verdosa con hilos de
sangre; en la radiografía de tórax presenta varios infiltrados pulmonares con cavitación
central en alguno de ellos. El diagnóstico más probable sería:

1. Neumonía neumocócica.
2. Tuberculosis pulmonar.
3. Neumonía por anaerobios.
4. Neumonía hematógena.
5. Neumonía por H. Influenzae.

PAMELA TABANGO

Dentro de los factores de riesgo de neumonía extrahospitalaria (CAP) escoja la opción


correcta:
a. Alcoholismo, asma, inmunodepresión, hospitalización y tener ≥70 años.
b. Niños <5 años, asma, inmunosuprimidos
c. Tabaquismo, alcoholismo y asma
d. Edad >60 años, asma, uso de corticoides

Harrison edicion 19 , capitulo 153, pag 805

JESSICA TARIS

¿En la infección tuberculosa cuál es el factor principal para adquirir dicha infección? Indique
la respuesta correcta:

a. Factor endógeno

b. Factor exógeno

c. Factor social

d. Factor ultravioletas
CAMILA URBANO

¿Cuál es el porcentaje de pacientes en los cuales se les reactiva la Tuberculosis dentro de

-
los 2 años posteriores, teniendo la enfermedad activa?
A. 50-80%
B. 2-10%
C. 30-50%
D. 99%

MARITZA VELARDE

Es correcto en cuanto a la patogenia y transmisión de Tuberculosis. Excepto:

A. M. tuberculosis se transmite mediante partículas en suspensión en el aire, o núcleos de


gotitas, que pueden generarse cuando las personas tuberculosas estornudan, tosen o
hablan
B. La infección, que es generalmente asintomática, ocurre cuando una persona
susceptible inhala núcleos de gotitas que contienen M. tuberculosis y los
microorganismos llegan a los alvéolos pulmonares.
C. Una vez en el pulmón, los bacilos son fagocitados por los linfocitos T
D. Puede presentarse forma extrapulmonares como ganglionar, pleural, abdominal, renal,
pericárdica, entre otras

MISHELL ZURITA

Señale cuál de estos enunciados respecto a la tuberculosis es incorrecto:

A. La transmisión ocurre por la diseminación aérea de núcleos de gotitas producidas


por los paciente con tuberculosis.
B. El Mycobacterium tuberculosis es el agente causal que mide 0.5-3 um y en su pared
tiene lipoarabinomanano que interviene en la patogenia.

-
C. La tuberculosis la enfermedad primaria es también conocida como tuberculosis de
reinfección.
D. Para el diagnóstico se realizan pruebas como baciloscopia,broncoscopia, radiografía
de tórax, prueba de tuberculina, cultivo, PCR.
2
ARIEL GUSTAVO TORO AGUILAR

Área personal  Mis cursos  Quito  CIENCIAS DE LA SALUD  MEDICINA - PRESENCIAL  ABR 2020 - AGO 2020
 INVESTIGACIÓN I - Prl: MD NVD Pen: 961  13 de junio: Examen Primer Parcial Investigación I  Examen Primer Parcial
Investigación I

Comenzado el sábado, 13 de junio de 2020, 10:10


Estado Finalizado
Finalizado en sábado, 13 de junio de 2020, 10:59
Tiempo empleado 49 minutos 46 segundos
Calificación 3,04 de 4,00 (76%)

Pregunta 1 Correcta Puntúa 0,16 sobre 0,16

Cuál de los siguientes es un signo característico de las vías respiratorias de las personas que sufren
asma:

Seleccione una:

o a. In ltración basó la

o b. Transformación de los basó los en macrófagos

o c. Adhesión de los glóbulos rojos al endotelio

@ d. In ltración eosinofílica

La respuesta correcta es: In ltración eosinofílica

/
Pregunta 2 Correcta Puntúa 0,16 sobre 0,16

¿Cuál es la clasi cación anatómica de las bronquiectasias?

Seleccione una:

o a. Lobares, arteriales, globulares

@ b. Cilíndricas, varicosas, saculares

o c. Vasculares, semilunares saculares

o d. Cubicas, cilíndricas, alargadas

La respuesta correcta es: Cilíndricas, varicosas, saculares

Pregunta 3 Correcta Puntúa 0,16 sobre 0,16

¿Cuál es la siopatología de la hemoptisis?

Seleccione una:

@ a. Hipervascularización de la circulación brónquica, hipertensión pulmonar y neovascularización

o b. Hipervascularización de la circulación pulmonar, ebre y regeneración alveolar

o c. Hipervascularización de la circulación pulmonar, hipertensión pulmonar y remodelación.

o d. Hipervascularización de la circulación brónquica, hipertensión pulmonar y disminución de


coagulabilidad

La respuesta correcta es: Hipervascularización de la circulación brónquica, hipertensión pulmonar y


neovascularización

/
Pregunta 4 Correcta Puntúa 0,16 sobre 0,16

De los siguientes factores, cuál es el que debe estar alterado para que un paciente infectado de
tuberculosis se convierta en persona enferma:

Seleccione una:

@ a. Huésped: Inmunidad celular de ciente, en especial CD4

o b. Huésped: que genéticamente esté predispuesto a infectarse

o c. Medio ambiente: Que haya elevada prevalencia de tuberculosis

o d. Medio Ambiente: Que viva en un lugar frío

La respuesta correcta es: Huésped: Inmunidad celular de ciente, en especial CD4

Pregunta 5 Correcta Puntúa 0,16 sobre 0,16

Dentro de las manifestaciones tardías de la sí lis tenemos trastornos cardiovasculares de cual podemos
desprender el siguiente:

Seleccione una:

o a. Trastornos del endotelio

o b. Insu ciencia cardiaca congestiva.

o c. Flebitis irritativa.

@ d. Aneurismas

La respuesta correcta es: Aneurismas

/
Pregunta 6 Correcta Puntúa 0,16 sobre 0,16

¿De qué depende la inmunidad contra Leptospira?

Seleccione una:

o a. Producción de anticuerpos contra los distintos serotipos.

o b. Producción de anticuerpos contra LPS inespecí cos.

o c. Producción de anticuerpos contra proteínas inespecí cas de un serotipo.

@ d. Producción de anticuerpos circulantes contra LPS especí cos de un serotipo.

La respuesta correcta es: Producción de anticuerpos circulantes contra LPS especí cos de un serotipo.

Pregunta 7 Correcta Puntúa 0,16 sobre 0,16

El hallazgo más frecuente en la radiografía de tórax en un paciente con asma es:

Seleccione una:

o a. Engrosamiento de paredes bronquiales.

@ b. Radiografía de tórax normal.

o c. Condensaciones alveolares bilaterales y difusas.

o d. Hiperinsu ación pulmonar.

La respuesta correcta es: Radiografía de tórax normal.

/
Pregunta 8 Correcta Puntúa 0,16 sobre 0,16

El mecanismo por el cual la infección por cólera produce una diarrea tan intensa es por:

Seleccione una:

o a. Toxina Shiga-like

o b. Vibrios entero hemorrágicos.

o c. Vibrios enteroagregativos.

@ d. Toxina enterogénica

La respuesta correcta es: Toxina enterogénica

Pregunta 9 Correcta Puntúa 0,16 sobre 0,16

El siguiente enunciado: Un estado parecido al sueño profundo en el que el paciente permanece con los
ojos cerrados y el paciente no puede ser despertado, se re ere a:

Seleccione una:

@ a. Estado de coma

o b. Estado vegetativo

o c. Somnolencia

o d. Estupor

La respuesta correcta es: Estado de coma

/
Pregunta 10 Correcta Puntúa 0,16 sobre 0,16

El tratamiento de las Bronquiectasias se basa en 3 pilares, excepto:

Seleccione una:

@ a. Revertir el remodelamiento bronquia

o b. Mejorar la eliminación de las secreciones, que se consigue con una adecuada hidratación, con
sioterapia respiratoria y drenaje postural mantenidos.

o c. Controlar las infecciones con el uso de antibióticos en las agudizaciones durante 10-15 días.

o d. Eliminar la obstrucción bronquial.

La respuesta correcta es: Revertir el remodelamiento bronquia

Pregunta 11 Incorrecta Puntúa 0,00 sobre 0,16

En cuanto a la pro laxis antitetánica en el tratamiento sistemático de las heridas, en una herida
pequeña limpia sin ningún antecedente de vacunación antitetánica Usted recomendaría:

Seleccione una:

@ a. Vacunación antitetánica de inmediato. X


o b. Es indicativo de inmunoglobulina.

o c. Solo limpiar la herida y dar indicaciones de cuidado al paciente, ofrecer la anti toxina.

o d. Dar antibiótico de manera pro láctica.

La respuesta correcta es: Solo limpiar la herida y dar indicaciones de cuidado al paciente, ofrecer la anti
toxina.

/
Pregunta 12 Correcta Puntúa 0,16 sobre 0,16

En cuanto al diagnóstico funcional del asma:

Seleccione una:

o a. Si la relación VEF1/CVF es menor a 0.7 (patrón restrictivo) y post broncodilatador obtenemos un


aumento igual o mayor al 12% en el VEF1, nos orienta hacia diagnóstico de asma

o b. Si la relación VEF1/CVF es mayor 0.7 se considera patrón obstructivo, diagnosticamos asma

@ c. Si la relación VEF1/CVF es menor a 0.7 (patrón obstructivo) y post broncodilatador obtenemos un


aumento igual o mayor al 12% en el VEF1, nos orienta hacia diagnóstico de asma

o d. Valores espirométricos no tienen importancia en el diagnóstico del asma.

La respuesta correcta es: Si la relación VEF1/CVF es menor a 0.7 (patrón obstructivo) y post broncodilatador
obtenemos un aumento igual o mayor al 12% en el VEF1, nos orienta hacia diagnóstico de asma

Pregunta 13 Correcta Puntúa 0,16 sobre 0,16

En el manejo de un paciente con delirium agitado (o hiperactivo), de las siguientes opciones, señale la
que NO incluiría en su prescripción médica:

Seleccione una:

o a. Evitar uso de restricciones físicas (amarras) para controlar el movimiento del paciente

o b. Antipsicóticos atípicos como la quetiapina en vía oral

@ c. Benzodiacepina como el clonazepam en vía oral

o d. Manejo no farmacológico como: colocar un reloj visible para el paciente

La respuesta correcta es: Benzodiacepina como el clonazepam en vía oral

/
Pregunta 14 Correcta Puntúa 0,16 sobre 0,16

En la clasi cación de OMS de adultos con VIH – Sida en el estadio C3 tenemos a:

Seleccione una:

o a. Mayor de 400 células CD4+ con síntomas no de nidores de Sida.

@ b. Menor de 200 células CD4+ con síntomas de nidores de Sida.

o c. Menor de 300 células CD4+ con síntomas de nidores de Sida.

o d. Mayor de 500 células CD4+ Asintomático.

La respuesta correcta es: Menor de 200 células CD4+ con síntomas de nidores de Sida.

Pregunta 15 Correcta Puntúa 0,16 sobre 0,16

En pacientes con catatonía, usted sospecharía:

Seleccione una:

o a. Lesión de la región orbitaria frontal

o b. Secuela del coma

o c. Herniación central

@ d. Trastorno psiquiátrico

La respuesta correcta es: Trastorno psiquiátrico

/
Pregunta 16 Incorrecta Puntúa 0,00 sobre 0,16

En relación a los siguientes enunciados acerca del ictus isquémico o enfermedad cerebrovascular
isquémica, señale el verdadero:

Seleccione una:

o a. El uso de estatinas reduce el riesgo de ictus isquémico inclusive con niveles normales de LDL o
niveles bajos de HDL

o b. Cualquier fármaco antiplaquetario (aspirina, clopidogrel, ticlopidina) son e caces y aprobados para
el uso en fase aguda de un ictus isquémico

c. Un paciente con brilación auricular valvular (ej enfermedad valvular reumática) requiere uso de
anticoagulantes como prevención primaria o secundaria X
o d. El tratamiento de HTA como prevención primaria con cifras de TA sistólica &lt;120 mm Hg reduce
en 43% la presencia de ictus y ataques del corazón

La respuesta correcta es: Cualquier fármaco antiplaquetario (aspirina, clopidogrel, ticlopidina) son e caces y
aprobados para el uso en fase aguda de un ictus isquémico

Pregunta 17 Incorrecta Puntúa 0,00 sobre 0,16

En un paciente de 68 años de edad, con APP: HTA, que ingresa con un cuadro de crisis focales motoras
sin pérdida del estado de conciencia y limitadas al miembro superior derecho, que se han repetido 1
crisis cada 3 días desde hace 2 semanas, además se acompaña de cefalea desde hace 3 meses con
características de empeoramiento progresivo, en los estudios de imagen se encontró una lesión
expansiva frontal izquierda; en relación al tratamiento farmacológico de elección, señale el que
escogería:

Seleccione una:

(!) a. Acido valproico X


o b. Lamotrigina

o c. Fenobarbital

o d. Clonazepam

La respuesta correcta es: Lamotrigina

/
Pregunta 18 Correcta Puntúa 0,16 sobre 0,16

La causa principal de un fracaso en el tratamiento de la tuberculosis pulmonar es

Seleccione una:

o a. Resistencia secundaria a las drogas

o b. Resistencia primaria a las drogas

@ c. Abandono del tratamiento por el paciente

o d. Toxicidad hepática

La respuesta correcta es: Abandono del tratamiento por el paciente

Pregunta 19 Incorrecta Puntúa 0,00 sobre 0,16

La ictericia en un paciente con Malaria es frecuente a partir:

Seleccione una:

o a. La ictericia no aparece en la malaria.

@ b. Siempre existirá ebre e ictericia debido a la destrucción de eritrocitos. X


o c. La Ictericia en Malaria es frecuente a partir de los 15 años.

o d. Ocasionalmente en niños debido a su alta susceptibilidad

La respuesta correcta es: La Ictericia en Malaria es frecuente a partir de los 15 años.

/
Pregunta 20 Correcta Puntúa 0,16 sobre 0,16

Paciente de 16 años de género masculino, sin APP, empieza con crisis convulsivas tónico clónicas
generalizadas hace 1 año, 1 crisis cada 2 meses, su examen neurológico es normal al igual que la
Resonancia Magnética cerebral, el electroencefalograma demostró actividad epileptiforme, cuál de las
siguientes etiologías consideraría en el diagnóstico

Seleccione una:

o a. Epilepsia por enfermedad degenerativa

@ b. Epilepsia por desorden genético

o c. Epilepsia secundaria a disturbios hidroelectrolíticos

o d. Epilepsia secundaria a autoanticuerpos

La respuesta correcta es: Epilepsia por desorden genético

Pregunta 21 Correcta Puntúa 0,16 sobre 0,16

Paciente de 22 años, género femenino, tiene desde hace 1 año dolor de cabeza de forma episódica (2
episodios al mes), hemicránea derecha o izquierda, pulsátil, intensidad moderada, se acompaña de
náusea y fotofobia, el dolor es precedido por escotomas centellantes que duran 90 minutos
aproximadamente, en los últimos 3 meses el dolor se ha tornado más frecuente (3 episodios semanales)
y al toser exacerba en intensidad a dolor severo, de los siguientes enunciados cuál NO considera
signo/síntoma de alarma:

Seleccione una:

@ a. Localización del dolor

o b. Valsalva positivo

o c. Aura de duración larga

o d. Empeoramiento del dolor (más frecuente)

La respuesta correcta es: Localización del dolor

/
Pregunta 22 Incorrecta Puntúa 0,00 sobre 0,16

Paciente de 38 años de género femenino, con APP diagnóstico de fenómeno de Raynaud en estudio de
un posible Lupus eritematoso sistémico, también diagnóstico de migraña con aura desde la juventud,
acude por un ataque agudo de cefalea, usted qué fármaco NO recomendaría:

Seleccione una:

o a. AINES

o b. Aspirina + metoclopramida

@ c. Clorpromazina X
o d. Sumatriptán

La respuesta correcta es: Sumatriptán

Pregunta 23 Correcta Puntúa 0,16 sobre 0,16

Paciente masculino de 66 años de edad, con antecedentes de deterioro cognitivo leve, HTA, ingresa para
una cirugía electiva de resección prostática por una hipertro a prostática benigna, en su postoperatorio
se encuentra con sondaje (cateterización), vesical permanente, dolor pélvico, y un cuadro de delirium (o
sd confusional agudo), se reinició el enalapril en el postoperatorio, de las siguientes opciones señale
cuál NO es un factor de riesgo para el aparecimiento de delirium

Seleccione una:

o a. Dolor en el postoperatorio

o b. Antecedentes de deterioro cognitivo leve

@ c. Administración de enalapril

o d. Sondaje vesical

La respuesta correcta es: Administración de enalapril

/
Pregunta 24 Incorrecta Puntúa 0,00 sobre 0,16

Tras la infección de un niño con el virus de la In uenza con la siguientes características: ebre intensa de
inicio súbito, disnea y cianosis a la Rx de tórax se aprecia patrón asociado con in ltrados intersticiales
difusos e hipoxia intensa, usted sospecharía de:

Seleccione una:

@ a. Neumonía viral secundaria. X


o b. Neumonía viral primaría.

o c. Neumonía bacteriana primaria.

o d. Neumonía bacteriana secundaria.

La respuesta correcta es: Neumonía viral primaría.

Pregunta 25 Correcta Puntúa 0,16 sobre 0,16

Una de las características principales por las cuales el dengue puede diseminarse es:

Seleccione una:

o a. Es sumamente frecuente la progresión de daño encefálico.

o b. Una característica precoz es la trombocitosis y neutro lia.

o c. Debido a la gran capacidad de diseminación del vector Anopheles.

@ d. El vector es muy cercano a los asentamientos humanos.

La respuesta correcta es: El vector es muy cercano a los asentamientos humanos.

◄ Avisos Ir a... V Link sesión zoom examen primer parcial ►

/
~UTE
l. U I L.I FACULTAD
CIENCIAS DE LA SALUD
~ EUGENIO ESPEJO

NICOLE CAROLINA CORTEZ TORRES

Área personal  Mis cursos  Quito  CIENCIAS DE LA SALUD  MEDICINA - PRESENCIAL  ABR 2020 - AGO 2020
 INVESTIGACION I-RESPI-TEORIA - Prl: MD NVA Pen: 961  6 de julio - 12 de julio  EVALUACION 2

Comenzado el viernes, 10 de julio de 2020, 16:40


Estado Finalizado
Finalizado en viernes, 10 de julio de 2020, 16:46
Tiempo empleado 6 minutos 5 segundos
Puntos 3,00/6,00
Cali cación 5,00 de 10,00 (50%)

Pregunta 1 Incorrecta Puntúa 0,00 sobre 1,00

La oligohemia distal a un vaso ocluido, se denomina:

Seleccione una:

o a. Horner

o b. Westermark

c. Hamman Rich X
o d. Damaseau

Respuesta incorrecta.

La respuesta correcta es: Westermark

/
Pregunta 2 Incorrecta Puntúa 0,00 sobre 1,00

Qué técnica o espécimen ofrece el rendimiento más alto para el diagnóstico de la tuberculosis pleural?

Seleccione una:

o a. Estudio directo de biopsia con histología de pleura

o b. Adenosin deaminasa en lìquido pleural

o c. BAAR del lìquido pleural

@ d. Zhiel Nielsen y cultivo en Lowenstein del esputo X

Respuesta incorrecta.

La respuesta correcta es: Estudio directo de biopsia con histología de pleura

Pregunta 3 Correcta Puntúa 1,00 sobre 1,00

Paciente femenino de 71 años con fractura de tobillo, presenta dolor en punta de costado con una
intensidad de 7/10, que se agrava al movimiento y la inspiración, se acompaña de disnea de aparición
súbita, tos y hemoptisis, FC: 114, FR: 26, T: 37.8 ºC. APP: Adenocarcinoma pulmonar con metástasis óseas en
columna lumbar y metástasis cerebral cosiderado etapa 4. Al valorarlo por escala de Weels usted considera
que:

Seleccione una:

@ a. El paciente se encuentra en un riesgo elevado de presentar tromboembolia pulmonar

o b. El paciente se encuentra en un riesgo moderado de presentar tromboembolia pulmonar

o c. El paciente se encuentra en un riesgo bajo de presentar tromboembolia pulmonar y no debe


considerarse pruebas de imagen

o d. El paciente se encuentra en un riesgo bajo de presentar tromboembolia pulmonar por lo que se debe
considerar otros diagnósticos diferenciales.

Respuesta correcta

La respuesta correcta es: El paciente se encuentra en un riesgo elevado de presentar tromboembolia pulmonar

/
Pregunta 4 Correcta Puntúa 1,00 sobre 1,00

La acumulación de líquido en el espacio pleural, según Sahn se debe a:

1. La acumulación de líquido en el espacio pleural, según Sahn se debe a:

Seleccione una:

o a. Disminución de la presión negativa del espacio pleural

o b. Aumento presión oncótica

o c. Disminución presión hidrostática

@ d. Aumento de permeabilidad capilar

Respuesta correcta

La respuesta correcta es: Aumento de permeabilidad capilar

Pregunta 5 Correcta Puntúa 1,00 sobre 1,00

Cuál de los siguientes signos radiográ cos no corresponde a un hallazgo de neumotórax?

Seleccione una:

o a. Aumento de la radiolucidez en el área de acumulación del aire.

o b. Desplazamiento de las estructuras del mediastino hacia el lado contrario del afectado.

@ c. Joroba de Hampton.

o d. Colapso pulmonar parcial o completo del pulmón del lado afectado.

Respuesta correcta

La respuesta correcta es: Joroba de Hampton.

/
Pregunta 6 Incorrecta Puntúa 0,00 sobre 1,00

Cuál es la mejor opción al identi car un paciente sintomático respiratorio.

Seleccione una:

o a. Realizo un PPD

o b. Iniciar tratamiento contra Tuberculosis

@ c. Pedir una Rx de tórax AP y lateral X


o d. 2 muestras de BAAR en esputo

Respuesta incorrecta.

La respuesta correcta es: 2 muestras de BAAR en esputo

◄ ENFERMEDAD PULMONAR INTERSTICIAL Ir a... V

/
Pregunta 1 Incorrecta Puntúa 0,00 sobre 1,00

En relación a la encefalitis, señale el enunciado FALSO:

Seleccione una:

a. En caso de comprobación de una encefalitis viral por arbovirus debe iniciarse inmediatamente tratamiento
con Aciclovir intravenoso por 14 a 21 días

b. El proceso infeccioso y respuesta inflamatoria se produce en el parénquima cerebral, en algunos casos puede
haber meningitis asociada (meningoencefalitis)

c. En adultos inmunocompetentes brotes episódicos de infección son causados por herpesvirus (HSV,
VZV, EBV), pero puede haber epidemias causadas por arbovirus

d. La clínica puede expresarse como alucinaciones, agitación, cambios de personalidad, trastornos conductuales y
a veces un estado psicótico franco

Respuesta incorrecta.

La respuesta correcta es: En caso de comprobación de una encefalitis viral por arbovirus debe iniciarse inmediatamente
tratamiento con Aciclovir intravenoso por 14 a 21 días

/
Pregunta 2 Incorrecta Puntúa 0,00 sobre 1,00

Señale el enunciado FALSO:

Seleccione una:

a. La presión intracraneal elevada siempre se produce a nivel de todos los compartimentos cerebrales en un mismo
momento

b. El incremento de la presión intracraneal produce una alteración global del flujo sanguíneo cerebral

c. Situaciones como hidrocefalia pueden producir incremento de la presión intracraneal al desplazar los otros
compartimentos

d. El desarrollo de papiledema se produce en las primeras 12 a 24 horas de un trauma craneal o de una


hemorragia cerebral

X
Respuesta incorrecta.

La respuesta correcta es: La presión intracraneal elevada siempre se produce a nivel de todos los compartimentos cerebrales
en un mismo momento

Pregunta 3 Incorrecta Puntúa 0,00 sobre 1,00

En relación a los siguientes enunciados, señale el falso:

Seleccione una:

a. Las crisis convulsivas tónico-clónicas generalizadas son el tipo más frecuente de crisis como consecuencia
de trastornos metabólicos

b. La fase tónica en una crisis T-C se acompaña de grito “ictal”, cianosis, mordedura lingual y aumento del tono
simpático

c. La epilepsia traduce la presencia de una o más crisis no provocadas producido por un proceso crónico
subyacente

d. Las crisis de ausencia típicas constituyen un grupo de epilepsia de origen genético que inicia en la niñez o
adolescencia

Respuesta incorrecta. X
La respuesta correcta es: La epilepsia traduce la presencia de una o más crisis no provocadas producido por un proceso crónico
subyacente

/
Pregunta 4 Correcta Puntúa 1,00 sobre 1,00

En relación a las infecciones del SNC, señale el enunciado falso:

Seleccione una:

a. Una de las complicaciones graves en Meningitis bacteriana aguda puede ser la hipertensión intracraneal la
misma que produce disminución del estado de conciencia

b. La otitis, mastoiditis y sinusitis son entidades predisponentes para sufrir meningitis por
estreptococo, estafilococo o anaerobios gram negativos

c. La frecuencia de meningitis por haemofilus influenza tipo b en niños ha disminuido gracias a la vacuna contra
dicho gérmen

d. Una de las vías de ingreso al SNC por parte de S pneumoniae y N meningitidis es por contigüidad desde el
epitelio nasofaríngeo

Respuesta correcta

La respuesta correcta es: Una de las vías de ingreso al SNC por parte de S pneumoniae y N meningitidis es por contigüidad desde el
epitelio nasofaríngeo

Pregunta 5 Incorrecta Puntúa 0,00 sobre 1,00

En relación a crisis, de los siguientes enunciados señale el FALSO:

Seleccione una:
a. Dentro de la exploración de crisis se debe investigar causas metabólicas con estudios séricos (Mg,
Na, Ca), infecciosas con LCR, etc

1 , b. Las crisis son la consecuencia de un desequilibrio entre la excitación y la inhibición dentro del SNC

c. Para el tratamiento de Epilepsia con crisis T.C generalizadas como fármaco de primera línea consta la fenitoína y
el fenobarbital

d. Como causa de crisis o epilepsia en el período neonatal o primera infancia consta anomalías congénitas
del SNC, etc

Respuesta incorrecta.

La respuesta correcta es: Para el tratamiento de Epilepsia con crisis T.C generalizadas como fármaco de primera línea consta la
fenitoína y el fenobarbital

/
Pregunta 6 Correcta Puntúa 1,00 sobre 1,00

En relación a la cisticercosis señale el enunciado FALSO:

Seleccione una:

a. La infección por cisticerco se deriva de la teniasis por Tenia saginata, cuyo hospedario es el ser humano

b. El tratamiento se lo debe hacer con un fármaco cisticida como el praziquantel o el albendazol

1 /
c. La infección puede afectar a cualquier órgano de la economía, a nivel cerebral puede invadir el parénquima,
sistema ventricular o subaracnoideo, etc

d. Las infecciones de cisticercosis pueden ser asintomáticas, pero también presentarse con crisis convulsivas o
no convulsivas, cefalea, hidrocefalia, etc

Respuesta correcta

La respuesta correcta es: La infección por cisticerco se deriva de la teniasis por Tenia saginata, cuyo hospedario es el ser
humano

Pregunta 7 Correcta Puntúa 1,00 sobre 1,00

En relación a la neurocisticercosis, señale el enunciado falso:

Seleccione una:

a. Un individuo que viajó a una zona endémica tendría criterio epidemiológico y sería un potencial foco de
infección

b. La determinación de antígenos o anticuerpos por ELISA se los puede realizar, pero no constan en los criterios
1
\
/ diagnósticos

c. Para el diagnóstico se mencionan criterios epidemiológicos: entre los que consta: residir en un área endémica de
cisticercosis

d. Uno de los criterios absolutos es: identificación neuroradiológica de lesiones quísticas que contengan un
escólex en su interior

Respuesta correcta

La respuesta correcta es: La determinación de antígenos o anticuerpos por ELISA se los puede realizar, pero no constan en los
criterios diagnósticos

/
Pregunta 8 Correcta Puntúa 1,00 sobre 1,00

En relación a las encefalopatías, señale el enunciado falso:

Seleccione una:

a. La causa pulmonar expresada como insuficiencia pulmonar o respiratoria se basa en hipoxemia e hipercapnia

b. El aporte diagnóstico del electroencefalograma se basa en el hallazgo de una lentitud difusa o simétrica que
traduce una disfunción cerebral difusa

c. Dentro de la etiología o causa hepática se menciona al shunt porto-sistémico, así como a la cirrosis

d. Dentro de las causas hidroelectrolíticas se mencionan las hiperpotasemias y las hipermagnesemias

Respuesta correcta

La respuesta correcta es: Dentro de las causas hidroelectrolíticas se mencionan las hiperpotasemias y las hipermagnesemias

Pregunta 9 Incorrecta Puntúa 0,00 sobre 1,00

En relación a cefalea, de los siguientes enunciados señale el FALSO:

Seleccione una:

a. El dolor de una cefalea tipo tensión comúnmente requerirá el uso de analgésicos como paracetamol,
ácido acetilsalicílico o AINES

b. El consumo frecuente de analgésicos especialmente los que contienen barbitúricos pueden inducir cefalea crónica
por sobre-uso de analgésicos

c. El tratamiento de un ataque agudo de cefalea tipo cluster se basa en inhalación de O2 al 100% 10-
12 l/minuto por 15 a 20 minutos
1 ,
d. La cefalea tipo tensión puede clasificarse en aguda y crónica, aguda si el dolor ocurre de 1-7 días/mes,
crónica si ocurre <15 días/mes X

Respuesta incorrecta.

La respuesta correcta es: La cefalea tipo tensión puede clasificarse en aguda y crónica, aguda si el dolor ocurre de 1-7
días/mes, crónica si ocurre <15 días/mes

/
Pregunta 10 Incorrecta Puntúa 0,00 sobre 1,00

En relación a la migraña, señale el enunciado falso:

Seleccione una:

a. Dentro de los criterios diagnósticos consta: cefalea de intensidad moderada a grave, dolor holocraneo,
no pulsátil, acompañado de aura luego del dolor

b. Dentro del manejo de migraña aguda se puede usar AINES como el ibuprofeno o agonistas 5HT1 como la
ergotamina o los triptanes

c. En la fisiopatología se ven implicados el péptido del gen relacionado con la calcitonina así como el
neurotransmisor 5-hidroxitriptamina, entre otros

d. Dentro de causas genéticas de migraña consta la migraña hemipléjica familiar con alteración del gen CACNA1A,
entre otros
En la fisiopatología se ven implicados el péptido del gen relacionado con la calcitonina así como el
neurotransmisor 5-hidroxitriptamina, entreotros X

Respuesta incorrecta.

La respuesta correcta es: Dentro de los criterios diagnósticos consta: cefalea de intensidad moderada a grave, dolor holocraneo, no
pulsátil, acompañado de aura luego del dolor

◄ Prueba pre-requisitos 1 18 Nov Ir a... V

Clase 4 24 Nov ►

/
Área personal J Mis cursos J Quito J CIENCIAS DE LA SALUD L MEDICINA - PRESENCIAL L OCT 2020 - FEB 2021
□ INVESTIGACION I-NEURO-TEORIA - Prl: MD NVA Pen: 961 Per:OCT 2020 - FEB 2021 Car:MEDICINA - PRESENCIAL [ 23 de
noviembre - 29 de noviembre □ Examen 2 28 Nov

Comenzado el sábado, 28 de noviembre de 2020, 16:00


Estado Finalizado
Finalizado en sábado, 28 de noviembre de 2020, 16:20
Tiempo empleado 20 minutos
Calificación 9,00 de 10,00 (90%)

Pregunta 1 Correcta Puntúa 1,00 sobre 1,00

Paciente que sufrió un cuadro de gastroenteritis aguda y 10 días después, desarrolla debilidad de las 4 extremidades que
evolucionó en el lapso de 4 días respiratoria, en el examen físico se encontró
cuadriparesia fláccida arrefléctica con respuesta plantar flexora, sin trastorno sensitivo, cuál sería el diagnóstico cor recto y la
conducta indicada:

Seleccione una:

a. Mielitis transversa, indicar corticoterapia

b. Mielitis transversa, indicar inmunoglobulina IV

c. Sd Guillain Barré, indicar corticoterapia

d. Sd Guillain Barré, indicar plasmaféresis

Respuesta correcta

La respuesta correcta es: Sd Guillain Barré, indicar plasmaféresis

/
Pregunta 2 Incorrecta Puntúa 0,00 sobre 1,00

En relación a la hemorragia cerebral, señale el enunciado falso:

Seleccione una:

a. Puede ocurrir en el espacio epidural o subdural en forma de hematomas traumáticos, o en parénquima,


espacio subaracnoideo o intraventricular por enfermedades médicas

b. La hemorragia hipertensiva produce ruptura de arterias perforantes por lo que es habitual encontrar
sangrado en: lóbulo frontal y temporal a nivel de la corteza cerebral

c. En pacientes ancianos puede producirse hemorragia parenquimatosa por Angiopatía Amiloide que es producida
por depósito de amiloide y degeneración arteriolar

d. En las horas que siguen a la hemorragia parenquimatosa existe riesgo de expansión del volumen del sangrado la
misma que se relaciona con cifras elevadas de HTA

X
Respuesta incorrecta.

La respuesta correcta es: La hemorragia hipertensiva produce ruptura de arterias perforantes por lo que es habitual encontrar
sangrado en: lóbulo frontal y temporal a nivel de la corteza cerebral

Pregunta 3 Correcta Puntúa 1,00 sobre 1,00

Una lesión completa del cordón medular, produce cuadriplejía alta si la lesión se localiza en:

Seleccione una:
a. C5-C8

b. Bajo T1

c. C1-C4

d. Sobre T12

Respuesta correcta

La respuesta correcta es: C1-C4

/
Pregunta 4 Correcta Puntúa 1,00 sobre 1,00

De los siguientes enunciados señale el verdadero:

Seleccione una:

a. La fiebre y la hiperglicemia no tienen injerencia en la isquemia pues no alteran el flujo sanguíneo cerebral

b. Los signos y síntomas neurológicos que se resuelven dentro de las primeras 48 h se conocen como isquemia
transitoria
r

c. El infarto cerebral ocurre por 2 vías: vía apoptóica y ruta necrótica, la primera ocurre ante la falta de sustratos
energéticos

d. El tejido que rodea al centro del infarto sufre isquemia pero su disfunción es reversible (penumbra isquémica)

! l)

Respuesta correcta r
La respuesta correcta es: El tejido que rodea al centro del infarto sufre isquemia pero su disfunción es reversible (penumbra
isquémica)

Pregunta 5 Correcta Puntúa 1,00 sobre 1,00

En relación al vértigo, señale el enunciado verdadero:

Seleccione una:

a. Un vértigo agudo prolongado puede ser debido a vértigo postural paroxístico

b. Una causa de vértigo episódico es la migraña vestibular

c. El nistagmo se inhibe al fijar la mirada si la causa es de origen central

d. Uno de los fármacos utilizados es el atenolol


r
Respuesta correcta

La respuesta correcta es: Una causa de vértigo episódico es la migraña vestibular

/
Pregunta 6 Correcta Puntúa 1,00 sobre 1,00

Entre los mecanismos de trauma craneal se mencionan, EXCEPTO:

Seleccione una:

a. Quemadura
,,..
b. Penetración de cuerpo extraño

c. Aceleración-desaceleración

d. Explosión o aplastamiento

Respuesta correcta

La respuesta correcta es: Quemadura

Pregunta 7 Correcta Puntúa 1,00 sobre 1,00

En un paciente con diagnóstico de Trauma cráneo encefálico, de acuerdo a su nivel o categoría de trauma y la acción
a realizar, cuál sería el enunciado verdadero:

Seleccione una:

a. En cualquier caso, requerirá TAC cerebral contrastada

b. Trauma leve – realizar TAC Cerebral simple

c. Trauma severo – TAC cerebral simple

d. Trauma moderado – alta con signos de alarma


• 1,
,,..
;

Respuesta correcta

La respuesta correcta es: Trauma severo – TAC cerebral simple

/
Pregunta 8 Correcta Puntúa 1,00 sobre 1,00

En relación a la esclerosis múltiple, señale el enunciado FALSO:

Seleccione una:

a. La edad de inicio es típicamente de entre 20 a 40 años

b. En la fisiopatología interviene el sistema inmune humoral

c. La lesión axonal es el dato patológico patognomónico de la enfermedad


~J:
d. Es más frecuente en mujeres que en hombres
,.
Respuesta correcta

La respuesta correcta es: La lesión axonal es el dato patológico patognomónico de la enfermedad

Pregunta 9 Correcta Puntúa 1,00 sobre 1,00

En relación al hematoma epidural, señale el enunciado FALSO:

Seleccione una:

a. Resulta de injuria de la arteria meníngea media

; b. Su imagen tomográfica es en forma de lente bi-convexa

; c. Se localiza entre la tabla interna del hueso y la piamadre

d. Evoluciona rápidamente
IJ)
,.
._

Respuesta correcta

La respuesta correcta es: Se localiza entre la tabla interna del hueso y la piamadre

/
Pregunta 10 Correcta Puntúa 1,00 sobre 1,00

En la exploración de un paciente con vértigo, señale cuál método o examen NO utilizaría:

Seleccione una:

a. Test calórico

b. Electronistagmografía

c. Punción lumbar

d. Videonistagmografía

Respuesta correcta

La respuesta correcta es: Punción lumbar

◄ Tarea Análisis de caso clínico 25 Nov Ir a... V

Caso clínico 2 ►

/
2
AYLEN MARILYN VERGARA TITO

Área personal  Mis cursos  Quito  CIENCIAS DE LA SALUD  MEDICINA - PRESENCIAL  ABR 2020 - AGO 2020
 INVESTIGACIÓN I - Prl: MD NVD Pen: 961  Sábado 4 de julio 8:00 am. Investigación I: Examen nal 1  Examen nal 1:
Investigación I

Comenzado el sábado, 4 de julio de 2020, 08:10


Estado Finalizado
Finalizado en sábado, 4 de julio de 2020, 09:27
Tiempo empleado 1 hora 16 minutos
Cali cación 4,13 de 5,33 (78%)

Pregunta 1 Correcta Puntúa 0,13 sobre 0,13

Ante un paciente con infarto cerebral en fase aguda, seleccione la respuesta correcta en cuanto al
tratamiento.

Seleccione una:

o a. Debe reducirse la presión arterial siempre.

o b. Se debe valorar tratamiento trombolítico intravenoso si llega antes de 6 horas del inicio de los
síntomas.

c. Se debe utilizar manitol cuando el edema cerebral produce disminución de la conciencia o se observa
desplazamiento de la línea media en la TC.

o d. Se debe colocar solución de Dextrosa 5% para hidratación.

La respuesta correcta es: Se debe utilizar manitol cuando el edema cerebral produce disminución de la conciencia
o se observa desplazamiento de la línea media en la TC.

/
Pregunta 2 Correcta Puntúa 0,13 sobre 0,13

¿A qué género pertenece el virus del dengue?

Seleccione una:

@ a. Flaviviridae

o b. Hantavirus

o c. Coronavirus

o d. Poxvirus

La respuesta correcta es: Flaviviridae

Pregunta 3 Incorrecta Puntúa 0,00 sobre 0,13

¿Cómo se denomina el vector transmisor de ebre amarilla?

Seleccione una:

@ a. Anopheles X
o b. Aedes aegypti

o c. Culecoides

o d. Lutzomya

La respuesta correcta es: Aedes aegypti

/
Pregunta 4 Correcta Puntúa 0,13 sobre 0,13

¿Cuál de las siguientes alternativas es una de las medidas para evitar la diseminación del Dengue entre la
población?

Seleccione una:

o a. Introducción experimental de mosquitos transgénicos

o b. Administración de la vacuna a poblaciones susceptibles.

o c. Pro laxis con Cloroquina – Primaquina

@ d. Control vectorial

La respuesta correcta es: Control vectorial

Pregunta 5 Incorrecta Puntúa 0,00 sobre 0,13

¿Cuál de las siguientes es una causa de hemoptisis?

Seleccione una:

o a. Vàrices esofágicas

o b. Estenosis Mitral

@ c. Edema agudo de Pulmòn X


o d. Fibrosis Pulmonar

La respuesta correcta es: Estenosis Mitral

/
Pregunta 6 Correcta Puntúa 0,13 sobre 0,13

Cuál de los siguientes enunciados corresponde a la de nición de neumonía nosocomial tardía

Seleccione una:

o a. Aparece después de 21 días del ingreso

o b. Aparece después de 10 días del ingreso

@ c. Aparece después de 7 días del ingreso

o d. Aparece después de 14 días del ingreso

La respuesta correcta es: Aparece después de 7 días del ingreso

Pregunta 7 Correcta Puntúa 0,13 sobre 0,13

¿Cuál es la acción de los nitratos en el síndrome coronario agudo?

Seleccione una:

@ a. vasodilatador coronario

o b. antiagregante

o c. revascularizador

o d. sedante

La respuesta correcta es: vasodilatador coronario

/
Pregunta 8 Correcta Puntúa 0,13 sobre 0,13

¿Cuál es la característica de la cefalea en racimos?

Seleccione una:

o a. Síntomas bilaterales

o b. Dolor leve

@ c. Síntomas ipsilaterales

o d. Dolor en la región occipital del cráneo

La respuesta correcta es: Síntomas ipsilaterales

Pregunta 9 Correcta Puntúa 0,13 sobre 0,13

¿Cuál es la diferencia entre una crisis focal y una crisis generalizada?

Seleccione una:

a. Una crisis focal se limita a un solo hemisferio mientras que una crisis generalizada se distribuye en
ambos hemisferios.

o b. No existe diferencia

o c. Una crisis focal se origina en el interior conectándose con las redes de ambos hemisferios mientras
que una crisis generalizada se limita a un solo hemisferio.

o d. Una crisis focal se origina en un lóbulo del cerebro mientras que una generalizada se origina en las
redes límites de todo un hemisferio.

La respuesta correcta es: Una crisis focal se limita a un solo hemisferio mientras que una crisis generalizada se
distribuye en ambos hemisferios.

/
Pregunta 10 Correcta Puntúa 0,13 sobre 0,13

¿Cuál es la función de la aspirina en el síndrome coronario agudo?

Seleccione una:

o a. Trombolisis

o b. Anticoagulantes

@ c. Evitar el incremento de la agregación plaquetaria en la placa ateromatosa accidentada

o d. Analgésico antin amatorio

La respuesta correcta es: Evitar el incremento de la agregación plaquetaria en la placa ateromatosa accidentada

Pregunta 11 Correcta Puntúa 0,13 sobre 0,13

¿Cuáles son signos de insu ciencia cardíaca derecha?

Seleccione una:

o a. Dolor abdominal, astenia

o b. Ortópnea, dolor precordial

@ c. Edema de miembros inferiores, hepatomegalia

o d. Disnea, estertores crepitantes

La respuesta correcta es: Edema de miembros inferiores, hepatomegalia

/
Pregunta 12 Correcta Puntúa 0,13 sobre 0,13

¿Cuántos días abarca el período de incubación de la leptospirosis?

Seleccione una:

o a. De 10 a 20 días (aproximadamente 2 semanas)

o b. De 1 a 60 días (aproximadamente 3 semanas)

@ c. De 1 a 30 días (aproximadamente 2 semanas)

o d. De 1 a 15 días (aproximadamente 1 semana)

La respuesta correcta es: De 1 a 30 días (aproximadamente 2 semanas)

Pregunta 13 Correcta Puntúa 0,13 sobre 0,13

De las encefalitis virales seleccione la que se bene cia del tratamiento con Aciclovir:

Seleccione una:

o a. Encefalitis equina

o b. Citomegalovirus

@ c. Herpes virus

o d. Enterovirus

La respuesta correcta es: Herpes virus

/
Pregunta 14 Correcta Puntúa 0,13 sobre 0,13

¿De qué depende el mayor riesgo de ENFERMARSE de Tuberculosis?

Seleccione una:

o a. Prevalencia de Tuberculosis

o b. Estilo de vida

o c. Genètica

@ d. Inmunidad celular, CD4

La respuesta correcta es: Inmunidad celular, CD4

Pregunta 15 Incorrecta Puntúa 0,00 sobre 0,13

El derrame pleural puede ser causado por diferentes causas. ¿En cuál de ellos usted sugiere realizar una
pleurodesis?

Seleccione una:

o a. Por Insu ciencia Cardíaca

o b. Por Tuberculosis

@ c. Por Neumonía X
o d. Por Cáncer

La respuesta correcta es: Por Cáncer

/
Pregunta 16 Correcta Puntúa 0,13 sobre 0,13

El dolor tipo pleurítico se caracteriza por ser:

Seleccione una:

o a. Opresivo intenso que disminuye con la inspiración

o b. Punzante intenso que disminuye con la inspiración

@ c. Punzante intenso que aumenta con la inspiración

o d. Opresivo intenso que aumenta con la inspiraciòn

La respuesta correcta es: Punzante intenso que aumenta con la inspiración

Pregunta 17 Correcta Puntúa 0,13 sobre 0,13

El uso de betabloqueantes debe ser analizado en el paciente con insu ciencia cardiaca ya que este grupo
farmacológico posee ciertas características importantes. Señale lo correcto respecto a estos productos:

Seleccione una:

o a. Puede utilizarse con seguridad en pacientes con asma

o b. Pueden utilizarse con seguridad en combinación con verapamilo

o c. Pueden utilizarse con seguridad en pacientes con hipotensión sintomático

@ d. Pueden utilizarse con seguridad en pacientes con insu ciencia cardiaca sistólica y angina de esfuerzo

La respuesta correcta es: Pueden utilizarse con seguridad en pacientes con insu ciencia cardiaca sistólica y
angina de esfuerzo

/
Pregunta 18 Correcta Puntúa 0,13 sobre 0,13

¿En cuál sistema arterial se producen la mayoría de las hemoptisis y por qué? Señale lo CORRECTO

Seleccione una:

o a. Las arterias bronquiales forman parte de la circulación sistémica, tienen menor presión y mucho
mayor ujo, de ellas depende la irrigación de los bronquios y la pleura visceral.

b. Las arterias bronquiales forman parte de la circulación sistémica, tienen mayor presión y mucho
menor ujo, de ellas depende la irrigación de los bronquios y la pleura visceral.

o c. Las arterias pulmonares son un sistema de alta presión por el que circula todo el gasto cardíaco

o d. Las arterias pulmonares son un sistema de baja presión por el que circula todo el gasto cardiaco y son
las responsables del intercambio gaseoso

La respuesta correcta es: Las arterias bronquiales forman parte de la circulación sistémica, tienen mayor presión
y mucho menor ujo, de ellas depende la irrigación de los bronquios y la pleura visceral.

Pregunta 19 Correcta Puntúa 0,13 sobre 0,13

¿En qué caso complementamos el electrocardiograma con derivaciones derechas?

Seleccione una:

o a. Sospecha de edema agudo de pulmón

@ b. Sospecha de infarto de ventrículo derecho

o c. Taquicardia ventricular monomorfa

o d. Ritmo de galope

La respuesta correcta es: Sospecha de infarto de ventrículo derecho

/
Pregunta 20 Correcta Puntúa 0,13 sobre 0,13

En una paciente de 30 años es llevada inconsciente a la emergencia de un hospital. Uno de los signos
clínicos que excluye el diagnóstico de muerte cerebral es:

Seleccione una:

o a. ausencia de re ejo nauseoso y tusígeno

@ b. postura de decerebración bilateral

o c. Glasgow 3

o d. pupilas midriáticas arreactivas

La respuesta correcta es: postura de decerebración bilateral

Pregunta 21 Correcta Puntúa 0,13 sobre 0,13

Forma de transmisión del virus de la hepatitis A

Seleccione una:

@ a. vía fecal- oral

o b. sexual

o c. vía respiratoria

o d. Por animales

La respuesta correcta es: vía fecal- oral

/
Pregunta 22 Correcta Puntúa 0,13 sobre 0,13

La causa más frecuente de hemoptisis leve-moderada es:

Seleccione una:

@ a. Bronquiectasias.

o b. Infarto pulmonar.

o c. Carcinoma broncogénico.

o d. Diátesis hemorrágica.

La respuesta correcta es: Bronquiectasias.

Pregunta 23 Correcta Puntúa 0,13 sobre 0,13

La enfermedad pulmonar obstructiva crónica no tratada puede conducir a la siguiente complicación


cardíaca:

Seleccione una:

o a. Síndrome coronario agudo

o b. Bradicardia

o c. Pericarditis

@ d. Insu ciencia cardíaca derecha

La respuesta correcta es: Insu ciencia cardíaca derecha

/
Pregunta 24 Correcta Puntúa 0,13 sobre 0,13

La insu ciencia cardíaca izquierda puede provocar edema agudo de pulmón por aumento de la:

Seleccione una:

o a. Presión sistólica del ventrículo derecho

o b. Presión venosa central

@ c. Presión capilar pulmonar

o d. Presión arterial

La respuesta correcta es: Presión capilar pulmonar

Pregunta 25 Correcta Puntúa 0,13 sobre 0,13

Los promastigotes son la forma de leishmania que se inyectan por medio de la probóscide del ebótomo de
sexo femenino en la piel del hospedador, ¿qué células son las encargadas de fagocitar a estas formas
parasitarias?

Seleccione una:

@ a. Neutró los

o b. Macrófagos

o c. Eosinó los

o d. Monocitos

La respuesta correcta es: Neutró los

/
Pregunta 26 Correcta Puntúa 0,13 sobre 0,13

María, 65 años, acude a la consulta re riendo cansancio a los moderados esfuerzos, re ere haber tenido
ebre reumática, al examen físico presenta PA 130/80, FC 88lpm, soplo sistólico en quinto espacio
intercostal línea media clavicular izquierda III/VI, con irradiación a la axila. ¿Cuál es el diagnóstico más
probable en esta paciente?

Seleccione una:

o a. Insu ciencia aortica

o b. Estenosis mitral

@ c. Insu ciencia mitral

o d. Estenosis aortica

La respuesta correcta es: Insu ciencia mitral

Pregunta 27 Correcta Puntúa 0,13 sobre 0,13

Paciente acude con trastornos osteomusculares crónicos, junto con dé cit visual inespecí co, pérdida de
peso y ebre de larga evaluación, el paciente proviene de una zona meramente ganadera, Usted sospecha
de brucelosis cuya base de tratamiento es:

Seleccione una:

o a. Penicilina y Metronidazol

@ b. Dicloxacilina y Estreptomicina

o c. Gentamicina y Dapsona

o d. Isoniacida y Estreptomicina

La respuesta correcta es: Dicloxacilina y Estreptomicina

/
Pregunta 28 Correcta Puntúa 0,13 sobre 0,13

Paciente de 25 años que re ere viaje reciente a Filipinas presenta: ebre, dolor abdominal, diarrea,
badicardia relativa. ¿Cuál es el diagnóstico más probable?

Seleccione una:

@ a. Fiebre tifoidea

o b. Paludismo

o c. Gastroenteritis complicada

o d. Micosos sistémica

La respuesta correcta es: Fiebre tifoidea

Pregunta 29 Sin contestar Puntúa como 0,13

Paciente masculino de 55 años de edad, sin antecedentes patológicos familiares de relevancia. Acude por
presentar desde hace 6 días dolor intenso y alodinia a nivel de región lumbar derecha. Hoy en la mañana
nota erupciones muy pruriginosas en la piel. Antecedentes patológicos personales: varicela en su niñez. Al
examen físico presenta vesículas agrupadas sobre una placa eritematosa elevada que siguen un mismo
dermatoma. También se observa un desprendimiento de costras dejando zonas expuestas de piel. ¿Cuál
sería el diagnóstico probable y su terapéutica?

Seleccione una:

o a. Herpes Zoster cuyo tratamiento ideal es Aciclovir, valaciclovir o famciclovir formando parte estos
fármacos de la familia de los inhibidores de la transcriptasa inversa.

o b. Herpes Zoster y su tratamiento más adecuado es Aciclovir 800 mg cada 4 horas durante 7 días.

o c. Infección por Herpes Zoster el mismo que necesitará tratamiento inmediato con Fluconazol 250 mg VO
cada día por 5 días.

o d. Dermatitis de contacto y deberá averiguarse sobre factores de predisposición alérgica en el paciente.

La respuesta correcta es: Herpes Zoster y su tratamiento más adecuado es Aciclovir 800 mg cada 4 horas durante
7 días.

/
Pregunta 30 Incorrecta Puntúa 0,00 sobre 0,13

Qué antihipertensivo se debe usar con cautela en un paciente con arteriopatia coronaria grave:

Seleccione una:

o a. Hidralazina

@ b. Espironolactona X
o c. Atenolol

o d. Clonidina

La respuesta correcta es: Hidralazina

Pregunta 31 Correcta Puntúa 0,13 sobre 0,13

Qué endocrinopatía es causa secundaria de hipertensión arterial:

Seleccione una:

o a. Hipogonadismo

@ b. Síndrome de Cushing

o c. Hipocalcemia

o d. Hiperprolactinemia

La respuesta correcta es: Síndrome de Cushing

/
Pregunta 32 Correcta Puntúa 0,13 sobre 0,13

¿Qué es una convulsión?

Seleccione una:

o a. Trastorno en el que una persona tiene convulsiones o crisis recurrentes debido a proceso crónico
subyacente

o b. Episodio paroxístico producido por descargas anormales disminuidas o actividad neuronal no


sincrónica en el cerebro

o c. Ninguna de las anteriores

@ d. Episodio paroxístico producido por descargas anormales excesivas o actividad neuronal sincrónica en
el cerebro

La respuesta correcta es: Episodio paroxístico producido por descargas anormales excesivas o actividad neuronal
sincrónica en el cerebro

Pregunta 33 Incorrecta Puntúa 0,00 sobre 0,13

¿Qué parámetros se consideran para tener migraña crónica?

Seleccione una:

o a. Episodios de migraña todos los días o casi a diario

@ b. Episodios de migraña cada semana X


o c. Episodios de migraña con alteraciones visuales

o d. Episodios de migraña por más de tres años

La respuesta correcta es: Episodios de migraña todos los días o casi a diario

/
Pregunta 34 Incorrecta Puntúa 0,00 sobre 0,13

Recibe el reporte de una muestra de líquido pleural con los siguientes resultados: exudado amarillento,
recuento del 90% de linfocitos. Pertenece a un paciente que presenta ebre y baja de peso, ¿en cuál de los
siguientes diagnósticos usted pensaría?

Seleccione una:

o a. Empiema

@ b. Neoplasia X
o c. Enfermedad del colágeno

o d. Tuberculosis

La respuesta correcta es: Tuberculosis

Pregunta 35 Incorrecta Puntúa 0,00 sobre 0,13

Señale las características del síndrome atípico de la Neumonía Adquirida en la Comunidad.

Seleccione una:

o a. Clínica aguda.

@ b. Se caracteriza por ebre elevada, escalofríos, tos productiva y dolor pleurítico X


o c. Disociación clínico radiológica

o d. Más evidente en personas ancianas

La respuesta correcta es: Disociación clínico radiológica

/
Pregunta 36 Correcta Puntúa 0,13 sobre 0,13

Su paciente proviene de la amazonia del Ecuador y presenta aproximadamente 15 días con ebre alta con
escalofríos, pero sin pérdida de peso considerable, solo con ligera palidez, al examen físico se parecía muy
levemente una hepato espleno megalia, y Biometría sin cambios aparente, sería prudente realizar lo
siguiente a su criterio:

Seleccione una:

@ a. Gota gruesa y extendido seriado

o b. Ecografía y determinar si existe ascitis y derrame pleural (Extravasación)

o c. Hemocultivo y esperar resultados

o d. Biopsia de Hígado y descartar procesos de ebre de larga evolución (FOD clásico)

La respuesta correcta es: Gota gruesa y extendido seriado

Pregunta 37 Correcta Puntúa 0,13 sobre 0,13

Una mujer de 40 años presenta debilidad progresiva y dolor en las piernas después de una semana de
haber presentado un episodio gripal. El examen físico demuestra tensión arterial 80/40 mm Hg, pulso 150
por minuto, temperatura bucal 37 grados C. Fuerza normal en la cara. Paresia 2/5 en la extremidad inferior
derecha y 3/5 en las extremidad inferior izquierda. Arre exia global con respuestas plantares en exión.
Uno de los signos o síntomas que podría hacer dudar en el diagnóstico de un síndrome de Guillain-Barré es:

Seleccione una:

o a. ausencia de paresia facial

@ b. debilidad asimétrica en las extremidades inferiores

o c. dolor en las piernas

o d. disfunción autonómica

La respuesta correcta es: debilidad asimétrica en las extremidades inferiores

/
Pregunta 38 Correcta Puntúa 0,13 sobre 0,13

Uno de los criterios para considerar la probabilidad de muerte cerebral y proceder a realizar el test de
apnea es:

Seleccione una:

o a. postura de descerebración luego de una reanimación cardiopulmonar de 20 minutos

o b. pupilas mióticas de 2 mm débilmente reactivas a la luz

o c. temperatura corporal de 36 grados centígrados

@ d. TC cerebral con una lesión estructural cerebral compatible con muerte cerebral

La respuesta correcta es: TC cerebral con una lesión estructural cerebral compatible con muerte cerebral

Pregunta 39 Correcta Puntúa 0,13 sobre 0,13

Un paciente de 35 años, consulta por cuadro de 4 semanas de evolución, de tos, con expectoración
mucopurulenta, con estrías de sangre en algunas ocasiones, asociado a compromiso del estado general,
baja de peso, ebre intermitente y sudoración nocturna. Ha tomado amoxicilina en varias oportunidades,
sin respuesta. Al examen físico destacan crépitos escasos, mayores en el ápice derecho. El diagnóstico más
probable es:

Seleccione una:

o a. Bronquiectasias

@ b. Tuberculosis

o c. Cáncer pulmonar

o d. Absceso pulmonar

La respuesta correcta es: Tuberculosis

/
Pregunta 40 Incorrecta Puntúa 0,00 sobre 0,13

Un paciente de 50 años, es trasladado inconsciente a la emergencia del hospital. Se procede a intubarlo


para protección de la vía aérea. Al estímulo doloroso no abre por los ojos, presenta exión de la extremidad
superior derecha con extensión plantar del pie derecho. La puntuación en la escala de Glasgow sería:

Seleccione una:

o a. 6

o b. 7

@ c. 4 X
o d. 5

La respuesta correcta es: 5

◄ Link sesión zoom examen primer parcial Ir a... V

Link sesión zoom examen nal 1: Investigación I ►

/
~UTE
l. U I L.I FACULTAD
CIENCIAS DE LA SALUD
~ EUGENIO ESPEJO

FRANCISCO JAVIER IZURIETA GAONA

Área personal  Mis cursos  Quito  CIENCIAS DE LA SALUD  MEDICINA - PRESENCIAL  OCT 2020 - FEB 2021
 INVESTIGACION I-RESPI-TEORIA - Prl: MD NVD Pen: 961 Per:OCT 2020 - FEB 2021 Car:MEDICINA - PRESENCIAL  2 de
noviembre - 8 de noviembre  EVALUACION PATOLOGIA RESPIRATORIA

Comenzado el domingo, 8 de noviembre de 2020, 11:00


Estado Finalizado
Finalizado en domingo, 8 de noviembre de 2020, 11:28
Tiempo empleado 28 minutos 36 segundos
Puntos 19,00/19,00
Cali cación 10,00 de 10,00 (100%)

Pregunta 1 Correcta Puntúa 1,00 sobre 1,00

Es la mejor prueba complementaria para el diagnóstico de asma:

Seleccione una:

Seleccione una:

o a. Determinación de inmunoglobulinas

b. Espirometría

o c. Lavado bronquial

o d. Broncoscopia

Respuesta correcta

La respuesta correcta es: Espirometría

/
Pregunta 2 Correcta Puntúa 1,00 sobre 1,00

Las tres alteraciones funcionales básicas del asma son:

Seleccione una:

Seleccione una:

o a. Hipertensión arterial, hiperreactividad bronquial y obstrucción nasal

o b. Obstrucción al ujo de aire, obstrucción al ujo venoso e irreversibilidad de la vía aérea

o c. Tos, obstrucción ujo aéreo y somnolencia

@ d. Obstrucción al ujo de aire, reversibilidad de la vía aérea e hiperreactividad bronquial

Respuesta correcta

La respuesta correcta es: Obstrucción al ujo de aire, reversibilidad de la vía aérea e hiperreactividad bronquial

Pregunta 3 Correcta Puntúa 1,00 sobre 1,00

En la exploración funcional de un paciente con enfermedad pulmonar obstructiva crónica, son esperables
todos los hallazgos MENOS uno:

Seleccione una:

Seleccione una:

@ a. Volúmenes pulmonares disminuidos

o b. FEV1 menor del 80%.

o c. Prueba broncodilatadora negativa

o d. Cociente FEV1/FVC inferior al 0.7

Respuesta correcta

La respuesta correcta es: Volúmenes pulmonares disminuidos

/
Pregunta 4 Correcta Puntúa 1,00 sobre 1,00

Cuáles son los 3 síntomas o signos más frecuentes de la EPOC

Seleccione una:

Seleccione una:

o a. Tos crónica, disnea crónica y ebre

@ b. Tos crónica, disnea crónica y expectoración mucosa

o c. Tos crónica, disnea crónica y utilización de músculos accesorios

o d. Tos crónica, disnea crónica y sibilancias

Respuesta correcta

La respuesta correcta es: Tos crónica, disnea crónica y expectoración mucosa

/
Pregunta 5 Correcta Puntúa 1,00 sobre 1,00

Masculino de 65 años de edad se presenta con una historia de disnea progresiva y tos no productiva de 3
días de evolución. Había sido hospitalizado previamente hace 2 años por insu ciencia cardiaca congestiva.
Sus signos vitales: TA 90/55 mmHg, frecuencia cardiaca de 110 latidos por minuto, temperatura de 37.6C, y
saturación de oxígeno al aire ambiente es de 86%. A la auscultación de tórax, se detectan estertores y
roncus bilaterales. La radiografía de tórax revela in ltrados bilaterales y un aumento discreto del tamaño de
la silueta cardiaca. ¿Cuál es el diagnóstico clínico más probable?

Seleccione una:

Seleccione una:

o a. Angina estable

o b. Disección de Aorta

o c. Tromboembolia pulmonar

@ d. Edema agudo de pulmón

Respuesta correcta

La respuesta correcta es: Edema agudo de pulmón

/
Pregunta 6 Correcta Puntúa 1,00 sobre 1,00

Con respecto a la EPOC, NO es cierto:

Seleccione una:

Seleccione una:

o a. Espirométricamente, se detecta obstrucción por un cociente FEV1/FVC inferior a 0,70

o b. El hábito tabáquico es el factor más importante para desarrollar EPOC

o c. El factor genético para desarrollar EPOC mejor documentado es el dé cit de alfa1- antitripsina

@ d. Contrariamente al asma, no hay componente in amatorio

Respuesta correcta

La respuesta correcta es: Contrariamente al asma, no hay componente in amatorio

Pregunta 7 Correcta Puntúa 1,00 sobre 1,00

Cuál de las siguientes anomalías electrocardiográ cas suele asociarse a tromboembolia pulmonar?

Seleccione una:

Seleccione una:

o a. Inversión de ondas T en la derivación de ondas laterales I, AVL y V5- V6

o b. Ondas Q en las derivaciones anteriores de V1- V4

@ c. S1Q3T3, ondas T negativas en las derivaciones anteriores de V1 a V4

o d. Bloqueo A-V de primer grado

Respuesta correcta

La respuesta correcta es: S1Q3T3, ondas T negativas en las derivaciones anteriores de V1 a V4

/
Pregunta 8 Correcta Puntúa 1,00 sobre 1,00

Actualmente se utilizan 3 clases de broncodilatadores en pacientes con asma, señale lo correcto:

Seleccione una:

Seleccione una:

@ a. Agonistas β2, anticolinérgicos, teo lina

o b. Agonistas β2, corticoesteroides inhalados, corticoesteroides sistémicos

o c. Antileucotrienos, teo lina, agonistas β2

o d. Anticolinérgicos, antileucotrienos, agonistas β2

Respuesta correcta

La respuesta correcta es: Agonistas β2, anticolinérgicos, teo lina

Pregunta 9 Correcta Puntúa 1,00 sobre 1,00

La decisión del ingreso hospitalario de un/a paciente con NAC depende de la capacidad del médico para
predecir la probabilidad de muerte. Según el índice CURB65, ¿cuándo se recomienda la hospitalización?

Seleccione una:

Seleccione una:

o a. Cuando la puntuación sea de 1 puntos.

o b. Cuando la puntuación sea de 1,5 puntos.

o c. Cuando la puntuación sea de 0 puntos.

@ d. Cuando la puntuación sea > de 2 puntos.

Respuesta correcta

La respuesta correcta es: Cuando la puntuación sea > de 2 puntos.

/
Pregunta 10 Correcta Puntúa 1,00 sobre 1,00

Criterios para derrame pleural exudativo, excepto:

Seleccione una:

Seleccione una:

o a. LDH del líquido pleural >66% del límite superior normal para el suero

@ b. LDH del líquido pleural/LDH sérica ˂0.4

o c. Proteínas de líquido pleural/proteínas séricas >0.5

o d. LDH del líquido pleural/LDH sérica >0.6

Respuesta correcta

La respuesta correcta es: LDH del líquido pleural/LDH sérica ˂0.4

/
Pregunta 11 Correcta Puntúa 1,00 sobre 1,00

Señale la respuesta incorrecta con relación a la tuberculosis:

Seleccione una:

Seleccione una:

o a. La enfermedad tuberculosa es aquella situación en la que el individuo presenta datos clínicos.

@ b. La primoinfección tuberculosa es aquella situación en la que el individuo entra en contacto con el


germen y desarrolla por primera vez la enfermedad.

o c. La tuberculosis posprimaria es aquella situación en la que el individuo presenta una reactivación de la


enfermedad tuberculosa.

o d. La infección tuberculosa es aquella situación en la que el individuo entra en contacto con el germen

Respuesta correcta

La respuesta correcta es: La primoinfección tuberculosa es aquella situación en la que el individuo entra en
contacto con el germen y desarrolla por primera vez la enfermedad.

Pregunta 12 Correcta Puntúa 1,00 sobre 1,00

Cuál es el cuadro clínico característico de los pacientes con Bronquitis crónica?

Seleccione una:

Seleccione una:

o a. llamados tosedores rosados, hemoptisis sin tos productiva

o b. llamados tosedores rosados, con espiración prolongada y aumento de peso

o c. llamados sopladores azules, aumento de peso y coloración normal de la piel

@ d. llamados sopladores azules, cianóticos y caquécticos

Respuesta correcta

La respuesta correcta es: llamados sopladores azules, cianóticos y caquécticos

/
Pregunta 13 Correcta Puntúa 1,00 sobre 1,00

Cuáles son los principales factores de riesgo para el desarrollo de NAC?

Seleccione una:

Seleccione una:

o a. Alcoholismo, parejas sexuales múltiples, uso de anticonceptivos orales, antecedentes de cirugía


cardiaca o pulmonar.

b. Extremos de edad (ancianos y niños), tabaquismo, alcoholismo, diabetes, asma, insu ciencia cardíaca,
EPOC, cáncer, inmunodeprimidos.

o c. Múltiples pareja sexuales, EPOC, tabaquismo, Inmunodeprimidos, adultos mayores sanos, y personas
adultas jóvenes.

o d. Enfermedad de Alzheimer, brosis quística, pacientes entre 30 y 50 años de edad, uso de


anticonceptivos orales, antecedentes de cirugía cardiaca o pulmonar ..

Respuesta correcta

La respuesta correcta es: Extremos de edad (ancianos y niños), tabaquismo, alcoholismo, diabetes, asma,
insu ciencia cardíaca, EPOC, cáncer, inmunodeprimidos.

/
Pregunta 14 Correcta Puntúa 1,00 sobre 1,00

Hombre de 68 años con antecedente de neoplasia de páncreas en curso de quimioterapia. Consulta en


Emergencias por dolor y edema de todo el miembro inferior desde ingle. ¿Qué prueba diagnóstica es más
coste-efectiva para con rmar la sospecha diagnóstica?

Seleccione una:

Seleccione una:

@ a. Ecografía doppler venosa

o b. Tomografía Axial Computarizada

o c. Resonancia Magnética

o d. Dímero D

Respuesta correcta

La respuesta correcta es: Ecografía doppler venosa

Pregunta 15 Correcta Puntúa 1,00 sobre 1,00

Señale el germen que no produce la tuberculosis pulmonar:

Seleccione una:

Seleccione una:

o a. Mycobacterium tuberculae

o b. Mycobacterium africanum.

o c. Mycobacterium bovis.

@ d. Mycobacterium leprae.

Respuesta correcta

La respuesta correcta es: Mycobacterium leprae.

/
Pregunta 16 Correcta Puntúa 1,00 sobre 1,00

Mujer de 54 años es encontrada desorientada por unos transeúntes. Al llegar el equipo de emergencias la
encontraron con una saturación de oxígeno del 80% respirando aire ambiente y al examen físico pupilas
puntiformes. Se la traslada a la emergencia del hospital, donde la gasometría arterial basal muestra: pH
7,25, PaC02 60 mmHg, Pa02 56 mmHg, bicarbonato de 26 mEq/l y exceso de bases de -1. En sangre el sodio
es 136 mEq/1 y el cloruro 100 mEq/1. Desde el punto de vista gasométrico la paciente tiene: Seleccione una:

Seleccione una:

o a. La gasometría solo puede ser de sangre venosa

o b. Acidosis metabòlica.

@ c. Acidosis respiratoria pura

o d. Insu ciencia respiratoria parcial

Respuesta correcta

La respuesta correcta es: Acidosis respiratoria pura

Pregunta 17 Correcta Puntúa 1,00 sobre 1,00

En la gasometría realizada en un paciente con Insu ciencia Respiratoria Aguda esperaríamos encontrar:

Seleccione una:

Seleccione una:

o a. PaO2 menor de 60 mmHg-PaCO2 menor de 45 mmHg

@ b. PaO2 menor de 60 mmHg-PaCO2 mayor de 45 mmHg

o c. PaO2 mayor de 60 mmHg-PaCO2 menor de 45 mmHg

o d. PaO2 mayor de 60 mmHg-PaCO2 mayor de 45 mmHg

Respuesta correcta

La respuesta correcta es: PaO2 menor de 60 mmHg-PaCO2 mayor de 45 mmHg

/
Pregunta 18 Correcta Puntúa 1,00 sobre 1,00

Cuál de las siguientes premisas es FALSA respecto del diagnóstico de Tromboembolia Pulmar:

Seleccione una:

Seleccione una:

o a. La disnea y la taquipnea son las manifestaciones clínicas más frecuentes

o b. Los escores de Ginebra y de Wells permite estrati car la probabilidad diagnóstica de un


Troboembolismo de Pulmón.

c. El ecodoppler venoso es el estudio de mayor sensibilidad y especi cidad para el diagnóstico de


Tromboembolismo de Pulmón.

o d. Ante una baja probabilidad clínica de Tromboembolismo de Pulmón un dímero-D de alta sensibilidad
negativo permite excluirlo.

Respuesta correcta

La respuesta correcta es: El ecodoppler venoso es el estudio de mayor sensibilidad y especi cidad para el
diagnóstico de Tromboembolismo de Pulmón.

Pregunta 19 Correcta Puntúa 1,00 sobre 1,00

Son factores de riesgo para neumonía nosocomial

Seleccione una:

Seleccione una:

o a. Diabetes Mellitus

o b. Edad y Etnia

@ c. Aspiración de contenido gástrico, Reintubación

o d. Extremos de la vida

Respuesta correcta

La respuesta correcta es: Aspiración de contenido gástrico, Reintubación

/
◄ INVESTIGACION PATOLOGIA RESPIRATORIA Ir a... V

PATOLOGIA RESPIRATORIA ►

/
~UTE
l. U I L.I FACULTAD
CIENCIAS DE LA SALUD
~ EUGENIO ESPEJO

FRANCISCO JAVIER IZURIETA GAONA

Área personal  Mis cursos  Quito  CIENCIAS DE LA SALUD  MEDICINA - PRESENCIAL  OCT 2020 - FEB 2021
 INVESTIGACION I-NEURO-TEORIA - Prl: MD NVD Pen: 961 Per:OCT 2020 - FEB 2021 Car:MEDICINA - PRESENCIAL  23 de
noviembre - 29 de noviembre  INVESTIGACION

Comenzado el jueves, 26 de noviembre de 2020, 14:00


Estado Finalizado
Finalizado en jueves, 26 de noviembre de 2020, 14:25
Tiempo empleado 24 minutos 53 segundos
Puntos 4,00/16,00
Cali cación 1,00 de 4,00 (25%)

Pregunta 1 Incorrecta Puntúa 0,00 sobre 1,00

Una mujer de 50 años presenta una debilidad progresiva en las piernas 7 días después de un síndrome
respiratorio. Niega dolor en las extremidades. El examen físico demuestra una presión arterial de 70/40
mmHg, pulso 130 por minuto, temperatura bucal 36.8 grados C. Paresia 3/5 en extremidades inferiores.
Ausencia de nivel algésico sensitivo en el tronco. Arre exia aquílea y rotuliana, con hiporre exia bicipital y
tricipital. Re ejos cutáneo plantares exores.

Uno de los síntomas adicionales que apoya el diagnóstico de Guillain Barré es:

Seleccione una:

o A. Hipotensión arterial y taquicardia

B. Debilidad progresiva en las extremidades inferiores X


o C. Arre exia rotuliana y aquílea

o D. Ausencia de dolor

La respuesta correcta es: Hipotensión arterial y taquicardia

/
Pregunta 2 Correcta Puntúa 1,00 sobre 1,00

Una mujer de 35 años sufre una caída accidental desde un autobús en movimiento, lo que ocasiona un
trauma craneal con pérdida de la conciencia. La valoración inicial objetiva una puntuación en la escala de
Glasgow de 9 (O2V3M4). La TC cerebral muestra una colección extra-axial de 15 cc en la región temporal
izquierda, con desvio de la línea media hacia la derecha de 7 cm. Indicaría:

Seleccione una:

o A. Inducir coma barbitúrico

o B. Hiperventilación manteniendo una pCO2 de 40 mmHg

@ C. Valoración por neurocirugía

o D. Manitol y diuréticos osmóticos

La respuesta correcta es: Valoración por neurocirugía

Pregunta 3 Incorrecta Puntúa 0,00 sobre 1,00

Una característica importante que permitiría establecer el diagnóstico de crisis parciales complejas en lugar
de ausencias es:

Seleccione una:

o A. Presencia de automatismos oromandibulares

o B. Pueden ser provocadas por hiperventilación

o C. Electroencefalograma con actividad punta-onda a 3 Hz generalizada

@ D. Afectación de la conciencia X

La respuesta correcta es: Presencia de automatismos oromandibulares

/
Pregunta 4 Incorrecta Puntúa 0,00 sobre 1,00

La medida terapéutica inmediata que indicaría en un paciente de 70 años con hipertensión arterial y
brilación auricular crónica que presenta un infarto cerebral de 2.5 horas de evolución con una puntuación
de 3 en la escala del NIHSS sería:

Seleccione una:

o A. Adecuado manejo de la presión arterial y glicemia

@ B. Trombolisis intravenosa con activador del plasminógeno tisular tPA 0.9 mg/Kg de peso X
o C. Antiagregación plaquetaria dual con aspirina y clopidogrel

o D. Antiarritmicos

La respuesta correcta es: Adecuado manejo de la presión arterial y glicemia

Pregunta 5 Incorrecta Puntúa 0,00 sobre 1,00

Un hombre de 65 años de edad, diabético, es trasladado inconsciente a la emergencia del hospital. Al


examen físico presenta una herida de 3 cm en el cuero cabelludo. Glasgow 7. Hiperventilación. Anisocoria
pupilar (derecha 5 mm no reactiva, izquierda 3 mm reactiva). Mirada desconjugada. Postura extensora
bilateral anormal. Fondo de ojo normal.

Los signos clínicos sugieren herniación cerebral a nivel de:

Seleccione una:

o A. Diencéfalo-mesencéfalo

@ B. Protuberancia-bulbo raquídeo X
o C. Prosencéfalo-diencéfalo

o D. Mesencéfalo-protuberancia

La respuesta correcta es: Mesencéfalo-protuberancia

/
Pregunta 6 Incorrecta Puntúa 0,00 sobre 1,00

Una mujer de 55 años acude por presentar desde hace 3 meses 1 episodio semanal de dolor lancinante en
la mejilla derecha de 10 minutos de duración desencadenados por tensión emocional. Se encuentra en
tratamiento farmacológico por cuadro depresivo desde hace 1 año. La valoración neurológica objetiva
hipoalgesia en la mejilla derecha. La consideración diagnóstica sería:

Seleccione una:

o A. Dolor psicógeno

o B. Neuralgia del trigémino sintomática

@ C. Dolor facial atípico X


o D. Neuralgia del trigémino clásica

La respuesta correcta es: Neuralgia del trigémino clásica

Pregunta 7 Incorrecta Puntúa 0,00 sobre 1,00

Un hallazgo sugestivo de infarto cerebral en la circulación vertebro-basilar en presencia de hemiparesia


proporcionada de instauración súbita es:

Seleccione una:

o A. Hemianopsia homónima

o B. Parálisis facial de tipo periférico contralateral a la hemiparesia

@ C. Parálisis del motor ocular externo ipsilateral al dé cit motor X


o D. Afasia motora

La respuesta correcta es: Parálisis facial de tipo periférico contralateral a la hemiparesia

/
Pregunta 8 Correcta Puntúa 1,00 sobre 1,00

Un criterio diagnóstico necesario para el diagnóstico de neurocisticercosis parenquimatosa de nitiva es:

Seleccione una:

o A. Múltiples vesículas parequimatosas sin escolex

@ B. Múltiples vesículas parenquimatosas sin escolex con test inmunológico positivo en LCR o sangre

o C. Calci caciones parenquimatosas múltiples en una persona que resida en área endémica

o D. Una vesícula parenquimatosa sin escolex con test inmunológico positivo en LCR o sangre

La respuesta correcta es: Múltiples vesículas parenquimatosas sin escolex con test inmunológico positivo en LCR
o sangre

Pregunta 9 Correcta Puntúa 1,00 sobre 1,00

Un hombre de 40 años acude por presentar una cefalea supraorbitaria izquierda, intensa, de 45 minutos de
duración acompañada de nausea, ptosis y miosis ipsilaterales. Re ere haber presentado al menos 8
episodios de iguales características en los últimos 8 meses, alguno de ellos desencadenado por estrés
laboral. La consideración diagnóstica sería:

Seleccione una:

o A. Cefalea secundaria aneurisma cerebral

o B. Migraña con aura

@ C. Cefalea en racimos

o D. Migraña con aura del tronco encefálico

La respuesta correcta es: Cefalea en racimos

/
Pregunta 10 Incorrecta Puntúa 0,00 sobre 1,00

Una paciente de 30 años es llevada inconsciente a la emergencia de un hospital. Uno de los signos clínicos
que excluye el diagnóstico de muerte cerebral es:

Seleccione una:

o A. Puntuación en la escala de Glasgow de 3

o B. Postura de descerebración bilateral

o C. Ausencia de los re ejos nauseoso y tusígeno

@ D. Pupilas midriáticas y arreactivas X

La respuesta correcta es: Postura de descerebración bilateral

Pregunta 11 Incorrecta Puntúa 0,00 sobre 1,00

Una mujer de 40 años presenta cefalea intensa y alteración de la visión en el ojo derecho. Está preocupada
porque su madre falleció de una hemorragia cerebral. Tiene como antecedentes lupus e hipertensión
arterial mal controlada. Fuma 20 cigarrillos diarios los últimos 5 años. El examen neurológico es normal
excepto que en su ojo derecho presenta ptosis y pupila dilatada. La TC cerebral es normal.

Recomendaría:

Seleccione una:

o A. Punción lumbar

o B. Angiografía cerebral

o C. TC cerebral contrastada

@ D. Valoración oftalmológica X

La respuesta correcta es: Punción lumbar

/
Pregunta 12 Incorrecta Puntúa 0,00 sobre 1,00

El estado epiléptico se de ne como:

Seleccione una:

o A. Paroxismos de contracciones musculares involuntarias de 4 minutos de duración

o B. Crisis epilépticas focales de al menos 5 minutos de duración

@ C. Dos o más crisis epilépticas con recuperación breve de la conciencia entre ellas X
o D. Crisis epilépticas recurrentes no provocadas por insulto neurológico o sistémico

La respuesta correcta es: Crisis epilépticas focales de al menos 5 minutos de duración

Pregunta 13 Correcta Puntúa 1,00 sobre 1,00

Jerarquice en orden de mayor a menor importancia los criterios clínicos para considerar una evento
cerebrovascular agudo como hemorrágico más que como isquémico.

Seleccione una:

A. Alteración del estado de conciencia, antecedente de hipertensión arterial, cefalea holocraneal,


hemiparesia

o B. Antecedente de hipertensión arterial, edad mayor de 70 años, cefalea occipital, afasia

o C. Cefalea occipital con nausea y vómito, antecedente de hipertensión arterial, alteración del estado de
conciencia, diplopia

o D. Edad mayor de 70 años, antecedente de hipertensión arterial, alteración del estado de conciencia,
ataxia cerebelosa

La respuesta correcta es: Alteración del estado de conciencia, antecedente de hipertensión arterial, cefalea
holocraneal, hemiparesia

/
Pregunta 14 Incorrecta Puntúa 0,00 sobre 1,00

Uno de los criterios para diferenciar la cefalea tensional crónica de la migraña crónica es:

Seleccione una:

@ A. Cefalea durante 15 o más días al mes durante más de 3 meses X


o B. Cefalea que empeora con la actividad física habitual

o C. Cefalea bilateral de intensidad leve a moderada

o D. Cefalea que mejora con un triptano

La respuesta correcta es: Cefalea bilateral de intensidad leve a moderada

Pregunta 15 Incorrecta Puntúa 0,00 sobre 1,00

El hematoma subdural por traumatismo cráneo-encefálico:

Seleccione una:

o A. Es la lesión focal postraumática más frecuente

o B. Se asocia frecuentemente a fractura del hueso temporal

o C. Siempre requiere tratamiento quirúrgico

@ D. Es debido a ruptura de la arteria meníngea media X

La respuesta correcta es: Es la lesión focal postraumática más frecuente

/
Pregunta 16 Incorrecta Puntúa 0,00 sobre 1,00

El signo clínico más importante para considerar el coma estructural más que metabólico es:

Seleccione una:

@ A. Respiración de Cheyne-Stokes X
o B. Anisocoria

o C. Crisis epilépticas

o D. Postura extensora anormal

La respuesta correcta es: Anisocoria

◄ Traumatismo cráneo-encefálico Ir a... V

/
CUTE
\.. U I L.I
FACULTAD
CIENCIAS DE LA SALUD
~ EUGENIO ESPEJO

Jonathan Esteban Miranda

Área personal  Mis cursos  Quito  CIENCIAS DE LA SALUD  MEDICINA - PRESENCIAL  OCT 2020 - FEB
2021  INVESTIGACION I-INFECTO-TEORIA - Prl: MD NVD Pen: 961 Per:OCT 2020 - FEB 2021 Car:MEDICINA -
PRESENCIAL  9 de noviembre - 15 de noviembre  Evaluación 9D

Comenzado el martes, 10 de noviembre de 2020, 09:05


Estado Finalizado
Finalizado en martes, 10 de noviembre de 2020, 09:50
Tiempo empleado 45 minutos 36 segundos
Puntos 32,00/35,00
Cali cación 3,66 de 4,00 (91%)

Pregunta 1 Correcta Puntúa 1,00 sobre 1,00

¿Cuál de los siguientes microorganismos que infectan el tracto intestinal causa bacteriemia
con mayor frecuencia?

Seleccione una:

a. Salmonella

o b. Campylobacter jejuni.

o c. Shigella exneri

o d. Vibrio cholerae.

La respuesta correcta es: Salmonella

/
Pregunta 2 Correcta Puntúa 1,00 sobre 1,00

En caso que se enfrente a una situación de tétanos una medida de soporte importante para
el control de espasmos es:

Seleccione una:

@ a. Diazepam

o b. Antitoxina

o c. Gabapentina

o d. Metronidazol

La respuesta correcta es: Diazepam

Pregunta 3 Correcta Puntúa 1,00 sobre 1,00

Su paciente presenta las siguientes características ebre alta prolongada dolor abdominal
diarrea y estreñimiento ocasionales esplenomegalia y roséola usted sospecha de:

Seleccione una:

@ a. Salmonelosis

o b. Shigella

o c. Entamoeba histolytica

o d. Giardiasis

La respuesta correcta es: Salmonelosis

/
Pregunta 4 Correcta Puntúa 1,00 sobre 1,00

Paciente que acude a Urgencias con ebre intermitente, escalofríos, cefalea, debilidad y
abundante sudoración nocturna de 3 días de evolución. Como antecedente epidemiológico
destaca un viaje a Cayambe hace 3 semanas donde consumieron leche cruda y queso sin
pasteurizar. El modo más rápido de diagnosticar la infección sospechada es:

Seleccione una:

o a. Gram directo del líquido cefalorraquídeo.

o b. Test para detección de anticuerpos heteró los (Paul-Bunnell).

o c. Tinciones de micobacterias en esputo u orina.

@ d. Prueba del Rosa de Bengala y toma de hemocultivo.

La respuesta correcta es: Prueba del Rosa de Bengala y toma de hemocultivo.

Pregunta 5 Correcta Puntúa 1,00 sobre 1,00

En la clasi cación de FOD cuál considera usted como urgente y debe ser tratada de
inmediato:

Seleccione una:

@ a. FOD neutropénico

o b. FOD Clásico

o c. FOD Nosocomial

o d. FOD asociado a HIV

La respuesta correcta es: FOD neutropénico

/
Pregunta 6 Correcta Puntúa 1,00 sobre 1,00

El interferón es un medicamento útil para las siguientes infecciones excepto:

Seleccione una:

@ a. VIH

o b. HPV

o c. Hepatitis B

o d. Sarcoma de Kaposi

La respuesta correcta es: VIH

Pregunta 7 Correcta Puntúa 1,00 sobre 1,00

Para el tratamiento de Leishmania cutánea de primera línea se utiliza:

Seleccione una:

o a. Anfotericina B

o b. Dapsona

o c. Fluconazol

@ d. Antimonio pentavalente

La respuesta correcta es: Antimonio pentavalente

/
Pregunta 8 Correcta Puntúa 1,00 sobre 1,00

En relación al tratamiento de la sí lis, ¿cuál de las siguientes es la respuesta correcta?

Seleccione una:

@ a. La penicilina es el tratamiento de elección en cualquier estadio.

o b. El tratamiento de elección son las tetraciclinas.

o c. No hay ningún tratamiento efectivo.

o d. La penicilina es el tratamiento de elección sólo en la neurosí lis.

La respuesta correcta es: La penicilina es el tratamiento de elección en cualquier estadio.

Pregunta 9 Correcta Puntúa 1,00 sobre 1,00

¿Qué es el dengue?

Seleccione una:

o a. Una enfermedad vírica que ocasiona un eritema que evoluciona a mácula y pápula
afectando fundamentalmente a la población infantil.

@ b. Una enfermedad vírica que puede producir una ebre hemorrágica.

o c. Una zoonosis que afecta al hombre ocasionalmente.

o d. Una enfermedad causada por un poxvirus.

La respuesta correcta es: Una enfermedad vírica que puede producir una ebre hemorrágica.

/
Pregunta 10 Correcta Puntúa 1,00 sobre 1,00

En el análisis de líquido cefalorraquídeo usted encuentra linfocitos aumentados, glucosa


disminuida, usted podría pensar en que la etiología es:

Seleccione una:

o a. parasitaria

o b. bacteriana

@ c. Micobacterias

o d. viral

La respuesta correcta es: Micobacterias

Pregunta 11 Correcta Puntúa 1,00 sobre 1,00

La BCG es una vacuna atenuada derivada del:

Seleccione una:

o a. M. avium - intracellulare

o b. M. tuberculosis

@ c. M. bovis

o d. M. leprae

La respuesta correcta es: M. bovis

/
Pregunta 12 Incorrecta Puntúa 0,00 sobre 1,00

Con respecto a la infección por gripe indiqué la premisa verdadera:

Seleccione una:

o a. casi siempre se asocia con febrícula, o sin ebre sino con tos seca

o b. puede causar una neumonía viral primaria

@ c. la Clínica generalmente se asocia a trastornos respiratorios altos X


o d. El virus solamente tiene mutaciones puntuales

La respuesta correcta es: puede causar una neumonía viral primaria

Pregunta 13 Correcta Puntúa 1,00 sobre 1,00

En la malaria, la forma de plasmodio transmitida del mosquito al hombre es el:

Seleccione una:

o a. Merozoito.

@ b. Esporozoito.

o c. Hipnozoito.

o d. Gametocito.

La respuesta correcta es: Esporozoito.

/
Pregunta 14 Correcta Puntúa 1,00 sobre 1,00

Mujer de 47 años de edad, nacida y residente en Orellana A mediados del año 2017 comienza
con palpitaciones, disnea y edemas progresivos en extremidades inferiores. Es diagnosticada
de insu ciencia cardíaca secundaria a miocardiopatía por Chagas. ¿Cuál es el agente
etiológico de esta enfermedad endémica?

Seleccione una:

@ a. Trypanosoma cruzi.

o b. Giardia lamblia.

o c. Leishmania donovani.

o d. Trypanosoma brucei.

La respuesta correcta es: Trypanosoma cruzi.

Pregunta 15 Incorrecta Puntúa 0,00 sobre 1,00

Artritis reactiva es una complicación de:

Seleccione una:

o a. Giardiasis

@ b. Salmonelosis X
o c. Shigella

o d. Entamoeba histolytica

La respuesta correcta es: Shigella

/
Pregunta 16 Correcta Puntúa 1,00 sobre 1,00

Si durante el examen físico en el área genital encuentra una úlcera lisa no purulenta bien
delimitado única super cial con linfadenopatía bilateral usted piensa en:

Seleccione una:

o a. Leishmania

o b. herpes tipo 2

o c. chancroide

@ d. sí lis

La respuesta correcta es: sí lis

Pregunta 17 Correcta Puntúa 1,00 sobre 1,00

A qué grupo poblacional no restringiría la vacuna de la gripe:

Seleccione una:

o a. Personas con antecedente de Guillain Barré

@ b. Personas alérgicas a las proteínas del huevo

o c. Niños con antecedentes de síndrome de Reye

o d. Personas con trastornos crónicos metabólicos

La respuesta correcta es: Personas alérgicas a las proteínas del huevo

/
Pregunta 18 Correcta Puntúa 1,00 sobre 1,00

Paciente diabética de 65 años de edad que consulta por un cuadro de dolor en la cara, ebre
y aparición de una lesión in amatoria bien delimitada que diagnosticamos de erisipela. ¿Cuál
es la etiología del proceso?

Seleccione una:

o a. Staphylococcus epidermidis.

o b. Staphylococcus aureus.

o c. Streptococcus agalactiae.

@ d. Streptococcus pyogenes.

La respuesta correcta es: Streptococcus pyogenes.

Pregunta 19 Correcta Puntúa 1,00 sobre 1,00

Cuál es el grupo sanguíneo y con mayor susceptibilidad al contagio de cólera:

Seleccione una:

o a. Grupo A

@ b. Grupo O

o c. Ninguno

o d. Grupo AB

La respuesta correcta es: Grupo O

/
Pregunta 20 Correcta Puntúa 1,00 sobre 1,00

Un varón de 19 años consulta por un cuadro de febrícula, mialgias y odinofagia, acompañado


de máculas cutáneas generalizadas no pruriginosas durante los últimos 7 días. En el
interrogatorio re ere, como único suceso médico, haber presentado una úlcera anal que le
causaba dolor a la defecación en el mes anterior que desapareció progresivamente sin
ninguna intervención. ¿Cuál es el germen causante de este cuadro clínico?

Seleccione una:

o a. Gonococo.

o b. Virus de herpes simple tipo 2.

o c. Chlamydia trachomatis.

@ d. Treponema pallidum.

La respuesta correcta es: Treponema pallidum.

Pregunta 21 Incorrecta Puntúa 0,00 sobre 1,00

Qué patología de las siguientes es una condición de ne Sida:

Seleccione una:

o a. leucoplasia vellosa

@ b. candidiasis oral X
o c. displasia cervical

o d. herpes tipo 1: úlceras crónicas > 1 mes

La respuesta correcta es: herpes tipo 1: úlceras crónicas > 1 mes

/
Pregunta 22 Correcta Puntúa 1,00 sobre 1,00

La tuberculosis osteoarticular ataca principalmente a columna a lo que se denomina:

Seleccione una:

o a. Trastorno de Jacob

o b. Osteo tosis

@ c. Mal de Pott

o d. Enfermedad de Wegener

La respuesta correcta es: Mal de Pott

Pregunta 23 Correcta Puntúa 1,00 sobre 1,00

La infección de herpes en recién nacidos es principalmente dado por:

Seleccione una:

o a. VHS 1

o b. VHH 8

o c. HVZ

@ d. VHS 2

La respuesta correcta es: VHS 2

/
Pregunta 24 Correcta Puntúa 1,00 sobre 1,00

Erisipela una infección de tejido blando es debido principalmente a:

Seleccione una:

o a. S. aureus

@ b. S. pyogenes

o c. Enterobacterias

o d. Pseudomona aeruginosa

La respuesta correcta es: S. pyogenes

Pregunta 25 Correcta Puntúa 1,00 sobre 1,00

Que caracteriza a una primoinfección por herpes tipo 1:

Seleccione una:

o a. distribución en dermatoma en especial tórax

o b. pápulas y pústulas intersticio entre piel y mucosa

o c. trastornos de córnea y conjuntiva

@ d. faringitis o gingivoestomatitis

La respuesta correcta es: faringitis o gingivoestomatitis

/
Pregunta 26 Correcta Puntúa 1,00 sobre 1,00

Joven de 23 años, que trabaja como DJ en un bar nocturno y tiene una historia de
promiscuidad bisexual e ingesta de drogas sintéticas, consulta por ebre, malestar general y
dolor en glande. La exploración física muestra múltiples tatuajes, piercings y la presencia de
tres lesiones vesiculosas en glande y adenopatías inguinales bilaterales. ¿Cuál es el
diagnóstico más probable?

Seleccione una:

@ a. Herpes genital.

o b. Infección por citomegalovirus.

o c. Primoinfección VIH.

o d. Condilomas acuminados.

La respuesta correcta es: Herpes genital.

Pregunta 27 Correcta Puntúa 1,00 sobre 1,00

Cuando la infección de la Tripanosoma cruzi ingresa por piel puede haber una reacción
in amatoria en el sitio de la infección que dura hasta ocho semanas denominado:

Seleccione una:

@ a. Chagoma

o b. Reacción de Fernández.

o c. Complejo oftalmo ganglionar del Chagas.

o d. Signo de Romaña - Mazza.

La respuesta correcta es: Chagoma

/
Pregunta 28 Correcta Puntúa 1,00 sobre 1,00

Cuál consideraría como primera opción para el diagnóstico de ebre tifoidea ( ebre
enterica):

Seleccione una:

o a. rosa de bengala

o b. biopsia intestinal

o c. Reacción de Widal

@ d. hemocultivo

La respuesta correcta es: hemocultivo

Pregunta 29 Correcta Puntúa 1,00 sobre 1,00

Tras la infección, ¿qué plasmodios parásitos humanos persisten como hipnozoitos en el


hígado y pueden causar recidivas?

Seleccione una:

o a. P. malariae.

o b. P. cynmology y P. knowlesi.

o c. P. falciparum.

@ d. P. vivax y P. ovale.

La respuesta correcta es: P. vivax y P. ovale.

/
Pregunta 30 Correcta Puntúa 1,00 sobre 1,00

¿Cuál de los siguientes componentes de la estructura de los virus gripales es el principal


responsable de su infecciosidad?

Seleccione una:

@ a. Hemaglutinina.

o b. ARN polimerasa.

o c. Envoltura lipídica.

o d. Neuraminidasa.

La respuesta correcta es: Hemaglutinina.

Pregunta 31 Correcta Puntúa 1,00 sobre 1,00

Lesión de Ghon es:

Seleccione una:

@ a. pequeño nódulo calci cado

o b. Ganglio de ápice de pulmón

o c. Diseminación miliar de tuberculosis.

o d. linfadenopatia de cuello

La respuesta correcta es: pequeño nódulo calci cado

/
Pregunta 32 Correcta Puntúa 1,00 sobre 1,00

Con respecto al tétanos, ¿cuál de las siguientes frases es la correcta?

Seleccione una:

o a. En el diagnóstico del tétanos, es fundamental haber efectuado cultivos previos al inicio


del tratamiento.

o b. La mortalidad del tétanos es todavía en la actualidad, superior al 50%.

@ c. El tétanos está caracterizado por una rigidez generalizada junto a crisis de espasmos
musculares.

o d. El tétanos aparece tras un periodo mínimo de incubación de 14 días.

La respuesta correcta es: El tétanos está caracterizado por una rigidez generalizada junto a crisis
de espasmos musculares.

Pregunta 33 Correcta Puntúa 1,00 sobre 1,00

Ante un niño de 7 meses con ebre e irritabilidad, fontanela abombada y un estudio de


líquido cefalorraquídeo con 110 células/ mm 3(75% linfocitos), proteínas 120 mg/dl y glucosa
28 mg/dl (glucemia sérica 89 mg/ dl), ¿cuál es la sospecha diagnóstica más razonable?

Seleccione una:

o a. Meningitis vírica.

o b. Síndrome mononucleósico.

@ c. Meningitis tuberculosa.

o d. Meningitis bacteriana.

La respuesta correcta es: Meningitis tuberculosa.

/
Pregunta 34 Correcta Puntúa 1,00 sobre 1,00

Un trabajador en el oleoducto en el Coca de 36 años de edad, sufre desde hace dos meses un
cuadro abdominal intermitente de náuseas, heces pastosas, atulencia, meteorismo y que le
han llevado a perder tres kg de peso. ¿Cuál de los siguientes microorganismos sería con
mayor probabilidad el responsable del cuadro?

Seleccione una:

o a. Vibrio cholerae.

@ b. Giardia lamblia

o c. Entamoeba coli.

o d. Trichomonas hominis.

La respuesta correcta es: Giardia lamblia

Pregunta 35 Correcta Puntúa 1,00 sobre 1,00

Ante los hallazgos que cabe esperar del análisis del líquido cefalorraquídeo en un cuadro de
meningitis bacteriana, NO se encuentra:

Seleccione una:

o a. Glucosa < 40 mg/dl.

o b. Proteínas > 45 mg/dl.

o c. Cultivo positivo en el 40% de los casos

@ d. Abundantes hematíes.

La respuesta correcta es: Abundantes hematíes.

/
Ir a... V

◄ Evaluación 9D

/
2
AYLEN MARILYN VERGARA TITO

Área personal  Mis cursos  Quito  CIENCIAS DE LA SALUD  MEDICINA - PRESENCIAL  ABR 2020 - AGO 2020
 INVESTIGACIÓN I - Prl: MD NVD Pen: 961  13 de junio: Examen Primer Parcial Investigación I  Examen Primer Parcial
Investigación I

Comenzado el sábado, 13 de junio de 2020, 10:10


Estado Finalizado
Finalizado en sábado, 13 de junio de 2020, 10:55
Tiempo empleado 44 minutos 48 segundos
Cali cación 2,88 de 4,00 (72%)

Pregunta 1 Correcta Puntúa 0,16 sobre 0,16

Cuál de los siguientes es un signo característico de las vías respiratorias de las personas que sufren
asma:

Seleccione una:

o a. Adhesión de los glóbulos rojos al endotelio

o b. In ltración basó la

o c. Transformación de los basó los en macrófagos

d. In ltración eosinofílica

La respuesta correcta es: In ltración eosinofílica

/
Pregunta 2 Correcta Puntúa 0,16 sobre 0,16

¿Cuál es la clasi cación anatómica de las bronquiectasias?

Seleccione una:

o a. Cubicas, cilíndricas, alargadas

o b. Lobares, arteriales, globulares

o c. Vasculares, semilunares saculares

d. Cilíndricas, varicosas, saculares

La respuesta correcta es: Cilíndricas, varicosas, saculares

Pregunta 3 Correcta Puntúa 0,16 sobre 0,16

¿Cuál es la siopatología de la hemoptisis?

Seleccione una:

a. Hipervascularización de la circulación brónquica, hipertensión pulmonar y neovascularización

o b. Hipervascularización de la circulación pulmonar, ebre y regeneración alveolar

o c. Hipervascularización de la circulación pulmonar, hipertensión pulmonar y remodelación.

o d. Hipervascularización de la circulación brónquica, hipertensión pulmonar y disminución de


coagulabilidad

La respuesta correcta es: Hipervascularización de la circulación brónquica, hipertensión pulmonar y


neovascularización

/
Pregunta 4 Correcta Puntúa 0,16 sobre 0,16

De los siguientes factores, cuál es el que debe estar alterado para que un paciente infectado de
tuberculosis se convierta en persona enferma:

Seleccione una:

o a. Medio Ambiente: Que viva en un lugar frío

o b. Huésped: que genéticamente esté predispuesto a infectarse

o c. Medio ambiente: Que haya elevada prevalencia de tuberculosis

d. Huésped: Inmunidad celular de ciente, en especial CD4

La respuesta correcta es: Huésped: Inmunidad celular de ciente, en especial CD4

Pregunta 5 Correcta Puntúa 0,16 sobre 0,16

Dentro de las manifestaciones tardías de la sí lis tenemos trastornos cardiovasculares de cual podemos
desprender el siguiente:

Seleccione una:

a. Aneurismas

o b. Insu ciencia cardiaca congestiva.

o c. Flebitis irritativa.

o d. Trastornos del endotelio

La respuesta correcta es: Aneurismas

/
Pregunta 6 Correcta Puntúa 0,16 sobre 0,16

¿De qué depende la inmunidad contra Leptospira?

Seleccione una:

o a. Producción de anticuerpos contra LPS inespecí cos.

o b. Producción de anticuerpos contra proteínas inespecí cas de un serotipo.

o c. Producción de anticuerpos contra los distintos serotipos.

@ d. Producción de anticuerpos circulantes contra LPS especí cos de un serotipo.

La respuesta correcta es: Producción de anticuerpos circulantes contra LPS especí cos de un serotipo.

Pregunta 7 Incorrecta Puntúa 0,00 sobre 0,16

El hallazgo más frecuente en la radiografía de tórax en un paciente con asma es:

Seleccione una:

o a. Engrosamiento de paredes bronquiales.

o b. Condensaciones alveolares bilaterales y difusas.

o c. Radiografía de tórax normal.

@ d. Hiperinsu ación pulmonar. X

La respuesta correcta es: Radiografía de tórax normal.

/
Pregunta 8 Correcta Puntúa 0,16 sobre 0,16

El mecanismo por el cual la infección por cólera produce una diarrea tan intensa es por:

Seleccione una:

o a. Vibrios entero hemorrágicos.

o b. Toxina Shiga-like

o c. Vibrios enteroagregativos.

d. Toxina enterogénica

La respuesta correcta es: Toxina enterogénica

Pregunta 9 Correcta Puntúa 0,16 sobre 0,16

El siguiente enunciado: Un estado parecido al sueño profundo en el que el paciente permanece con los
ojos cerrados y el paciente no puede ser despertado, se re ere a:

Seleccione una:

o a. Estado vegetativo

b. Estado de coma

o c. Somnolencia

o d. Estupor

La respuesta correcta es: Estado de coma

/
Pregunta 10 Correcta Puntúa 0,16 sobre 0,16

El tratamiento de las Bronquiectasias se basa en 3 pilares, excepto:

Seleccione una:

o a. Eliminar la obstrucción bronquial.

o b. Controlar las infecciones con el uso de antibióticos en las agudizaciones durante 10-15 días.

@ c. Revertir el remodelamiento bronquia

o d. Mejorar la eliminación de las secreciones, que se consigue con una adecuada hidratación, con
sioterapia respiratoria y drenaje postural mantenidos.

La respuesta correcta es: Revertir el remodelamiento bronquia

Pregunta 11 Correcta Puntúa 0,16 sobre 0,16

En cuanto a la pro laxis antitetánica en el tratamiento sistemático de las heridas, en una herida
pequeña limpia sin ningún antecedente de vacunación antitetánica Usted recomendaría:

Seleccione una:

a. Solo limpiar la herida y dar indicaciones de cuidado al paciente, ofrecer la anti toxina.

o b. Dar antibiótico de manera pro láctica.

o c. Vacunación antitetánica de inmediato.

o d. Es indicativo de inmunoglobulina.

La respuesta correcta es: Solo limpiar la herida y dar indicaciones de cuidado al paciente, ofrecer la anti
toxina.

/
Pregunta 12 Correcta Puntúa 0,16 sobre 0,16

En cuanto al diagnóstico funcional del asma:

Seleccione una:

a. Si la relación VEF1/CVF es menor a 0.7 (patrón obstructivo) y post broncodilatador obtenemos un


aumento igual o mayor al 12% en el VEF1, nos orienta hacia diagnóstico de asma

o b. Valores espirométricos no tienen importancia en el diagnóstico del asma.

o c. Si la relación VEF1/CVF es menor a 0.7 (patrón restrictivo) y post broncodilatador obtenemos un


aumento igual o mayor al 12% en el VEF1, nos orienta hacia diagnóstico de asma

o d. Si la relación VEF1/CVF es mayor 0.7 se considera patrón obstructivo, diagnosticamos asma

La respuesta correcta es: Si la relación VEF1/CVF es menor a 0.7 (patrón obstructivo) y post broncodilatador
obtenemos un aumento igual o mayor al 12% en el VEF1, nos orienta hacia diagnóstico de asma

Pregunta 13 Incorrecta Puntúa 0,00 sobre 0,16

En el manejo de un paciente con delirium agitado (o hiperactivo), de las siguientes opciones, señale la
que NO incluiría en su prescripción médica:

Seleccione una:

o a. Manejo no farmacológico como: colocar un reloj visible para el paciente

o b. Evitar uso de restricciones físicas (amarras) para controlar el movimiento del paciente

c. Antipsicóticos atípicos como la quetiapina en vía oral X


o d. Benzodiacepina como el clonazepam en vía oral

La respuesta correcta es: Benzodiacepina como el clonazepam en vía oral

/
Pregunta 14 Correcta Puntúa 0,16 sobre 0,16

En la clasi cación de OMS de adultos con VIH – Sida en el estadio C3 tenemos a:

Seleccione una:

o a. Menor de 300 células CD4+ con síntomas de nidores de Sida.

o b. Mayor de 500 células CD4+ Asintomático.

c. Menor de 200 células CD4+ con síntomas de nidores de Sida.

o d. Mayor de 400 células CD4+ con síntomas no de nidores de Sida.

La respuesta correcta es: Menor de 200 células CD4+ con síntomas de nidores de Sida.

Pregunta 15 Correcta Puntúa 0,16 sobre 0,16

En pacientes con catatonía, usted sospecharía:

Seleccione una:

a. Trastorno psiquiátrico

o b. Lesión de la región orbitaria frontal

o c. Secuela del coma

o d. Herniación central

La respuesta correcta es: Trastorno psiquiátrico

/
Pregunta 16 Incorrecta Puntúa 0,00 sobre 0,16

En relación a los siguientes enunciados acerca del ictus isquémico o enfermedad cerebrovascular
isquémica, señale el verdadero:

Seleccione una:

a. El tratamiento de HTA como prevención primaria con cifras de TA sistólica &lt;120 mm Hg reduce
en 43% la presencia de ictus y ataques del corazón X
o b. Cualquier fármaco antiplaquetario (aspirina, clopidogrel, ticlopidina) son e caces y aprobados para
el uso en fase aguda de un ictus isquémico

o c. Un paciente con brilación auricular valvular (ej enfermedad valvular reumática) requiere uso de
anticoagulantes como prevención primaria o secundaria

o d. El uso de estatinas reduce el riesgo de ictus isquémico inclusive con niveles normales de LDL o
niveles bajos de HDL

La respuesta correcta es: Cualquier fármaco antiplaquetario (aspirina, clopidogrel, ticlopidina) son e caces y
aprobados para el uso en fase aguda de un ictus isquémico

Pregunta 17 Correcta Puntúa 0,16 sobre 0,16

En un paciente de 68 años de edad, con APP: HTA, que ingresa con un cuadro de crisis focales motoras
sin pérdida del estado de conciencia y limitadas al miembro superior derecho, que se han repetido 1
crisis cada 3 días desde hace 2 semanas, además se acompaña de cefalea desde hace 3 meses con
características de empeoramiento progresivo, en los estudios de imagen se encontró una lesión
expansiva frontal izquierda; en relación al tratamiento farmacológico de elección, señale el que
escogería:

Seleccione una:

o a. Fenobarbital

o b. Clonazepam

® c. Lamotrigina

o d. Acido valproico

La respuesta correcta es: Lamotrigina

/
Pregunta 18 Correcta Puntúa 0,16 sobre 0,16

La causa principal de un fracaso en el tratamiento de la tuberculosis pulmonar es

Seleccione una:

o a. Toxicidad hepática

o b. Resistencia secundaria a las drogas

o c. Resistencia primaria a las drogas

d. Abandono del tratamiento por el paciente

La respuesta correcta es: Abandono del tratamiento por el paciente

Pregunta 19 Incorrecta Puntúa 0,00 sobre 0,16

La ictericia en un paciente con Malaria es frecuente a partir:

Seleccione una:

o a. La ictericia no aparece en la malaria.

o b. La Ictericia en Malaria es frecuente a partir de los 15 años.

@ c. Siempre existirá ebre e ictericia debido a la destrucción de eritrocitos. X


o d. Ocasionalmente en niños debido a su alta susceptibilidad

La respuesta correcta es: La Ictericia en Malaria es frecuente a partir de los 15 años.

/
Pregunta 20 Correcta Puntúa 0,16 sobre 0,16

Paciente de 16 años de género masculino, sin APP, empieza con crisis convulsivas tónico clónicas
generalizadas hace 1 año, 1 crisis cada 2 meses, su examen neurológico es normal al igual que la
Resonancia Magnética cerebral, el electroencefalograma demostró actividad epileptiforme, cuál de las
siguientes etiologías consideraría en el diagnóstico

Seleccione una:

@ a. Epilepsia por desorden genético

o b. Epilepsia por enfermedad degenerativa

o c. Epilepsia secundaria a autoanticuerpos

o d. Epilepsia secundaria a disturbios hidroelectrolíticos

La respuesta correcta es: Epilepsia por desorden genético

Pregunta 21 Incorrecta Puntúa 0,00 sobre 0,16

Paciente de 22 años, género femenino, tiene desde hace 1 año dolor de cabeza de forma episódica (2
episodios al mes), hemicránea derecha o izquierda, pulsátil, intensidad moderada, se acompaña de
náusea y fotofobia, el dolor es precedido por escotomas centellantes que duran 90 minutos
aproximadamente, en los últimos 3 meses el dolor se ha tornado más frecuente (3 episodios semanales)
y al toser exacerba en intensidad a dolor severo, de los siguientes enunciados cuál NO considera
signo/síntoma de alarma:

Seleccione una:

o a. Localización del dolor

o b. Empeoramiento del dolor (más frecuente)

o c. Valsalva positivo

d. Aura de duración larga X

La respuesta correcta es: Localización del dolor

/
Pregunta 22 Incorrecta Puntúa 0,00 sobre 0,16

Paciente de 38 años de género femenino, con APP diagnóstico de fenómeno de Raynaud en estudio de
un posible Lupus eritematoso sistémico, también diagnóstico de migraña con aura desde la juventud,
acude por un ataque agudo de cefalea, usted qué fármaco NO recomendaría:

Seleccione una:

@ a. Clorpromazina X
o b. AINES

o c. Aspirina + metoclopramida

o d. Sumatriptán

La respuesta correcta es: Sumatriptán

Pregunta 23 Correcta Puntúa 0,16 sobre 0,16

Paciente masculino de 66 años de edad, con antecedentes de deterioro cognitivo leve, HTA, ingresa para
una cirugía electiva de resección prostática por una hipertro a prostática benigna, en su postoperatorio
se encuentra con sondaje (cateterización), vesical permanente, dolor pélvico, y un cuadro de delirium (o
sd confusional agudo), se reinició el enalapril en el postoperatorio, de las siguientes opciones señale
cuál NO es un factor de riesgo para el aparecimiento de delirium

Seleccione una:

o a. Dolor en el postoperatorio

b. Administración de enalapril

o c. Antecedentes de deterioro cognitivo leve

o d. Sondaje vesical

La respuesta correcta es: Administración de enalapril

/
Pregunta 24 Incorrecta Puntúa 0,00 sobre 0,16

Tras la infección de un niño con el virus de la In uenza con la siguientes características: ebre intensa de
inicio súbito, disnea y cianosis a la Rx de tórax se aprecia patrón asociado con in ltrados intersticiales
difusos e hipoxia intensa, usted sospecharía de:

Seleccione una:

@ a. Neumonía bacteriana secundaria. X


o b. Neumonía viral primaría.

o c. Neumonía viral secundaria.

o d. Neumonía bacteriana primaria.

La respuesta correcta es: Neumonía viral primaría.

Pregunta 25 Correcta Puntúa 0,16 sobre 0,16

Una de las características principales por las cuales el dengue puede diseminarse es:

Seleccione una:

o a. Una característica precoz es la trombocitosis y neutro lia.

o b. Debido a la gran capacidad de diseminación del vector Anopheles.

@ c. El vector es muy cercano a los asentamientos humanos.

o d. Es sumamente frecuente la progresión de daño encefálico.

La respuesta correcta es: El vector es muy cercano a los asentamientos humanos.

◄ Avisos Ir a... Link sesión zoom examen primer parcial ►

/
2
AYLEN MARILYN VERGARA TITO

Área personal  Mis cursos  Quito  CIENCIAS DE LA SALUD  MEDICINA - PRESENCIAL  ABR 2020 - AGO 2020
 INVESTIGACIÓN I - Prl: MD NVD Pen: 961  18 de julio: 12:00 H: Examen Segundo parcial Investigación I  Examen Segundo
Parcial Investigación I

Comenzado el sábado, 18 de julio de 2020, 12:10


Estado Finalizado
Finalizado en sábado, 18 de julio de 2020, 12:52
Tiempo empleado 42 minutos
Cali cación 3,68 de 4,00 (92%)

Pregunta 1 Correcta Puntúa 0,16 sobre 0,16

Causa de riesgo de Trombosis adquirida tanto venosa como arterial, la cual amerita tratamiento
anticoagulante?

Seleccione una:

o a. Inmovilización

o b. Cirugía mayor

@ c. Síndrome Antifosfolipídico

o d. Embarazo

La respuesta correcta es: Síndrome Antifosfolipídico


/
Pregunta 2 Correcta Puntúa 0,16 sobre 0,16

¿Cuál es la reacción transfusional más frecuente?

Seleccione una:

@ a. Reacción febril no hemolítica

o b. Reacción alérgica

o c. Reacción Hemolítica

o d. Reacción Ana láctica

La respuesta correcta es: Reacción febril no hemolítica

Pregunta 3 Correcta Puntúa 0,16 sobre 0,16

Cuál glomerulopatía no produce daño vascular:

Seleccione una:

o a. Crioglobulinemia

o b. Nefritis lúpica

o c. Granulomatosis de Wegener

@ d. Nefropatía por IgA

La respuesta correcta es: Nefropatía por IgA


/
Pregunta 4 Correcta Puntúa 0,16 sobre 0,16

Cuál no es una glomerulopatía de vasos pequeños ANCA:

Seleccione una:

o a. Síndrome de Churg-Strauss

@ b. Púrpura de Henoch-Schönlein

o c. Granulomatosis de Wegener

o d. Poliangitis microscópica

La respuesta correcta es: Púrpura de Henoch-Schönlein

Pregunta 5 Correcta Puntúa 0,16 sobre 0,16

Cuánto es la dosis de dabigatrán con CCr >30 mL/min:

Seleccione una:

o a. 150 mg cada 24 horas

@ b. 150 mg cada 12 horas

o c. 150 mg cada 6 horas

o d. 150 mg cada 8 horas

La respuesta correcta es: 150 mg cada 12 horas


/
Pregunta 6 Correcta Puntúa 0,16 sobre 0,16

Cuántos Joules se usa en la brilación auricular con hipotensión profunda, edema pulmonar o angina:

Seleccione una:

o a. 150 Joules

o b. 100 Joules

o c. 250 Joules

@ d. 200 Joules

La respuesta correcta es: 200 Joules

Pregunta 7 Incorrecta Puntúa 0,00 sobre 0,16

El riesgo de contaminación Bacteriana debido a una transfusión ha aumentado conforme disminuye el


riesgo de infecciones virales, muchas bacterias no proliferan en frío. Cuál de estos derivados representa el
mayor riesgo de contaminación?

Seleccione una:

o a. Concentrado de plaquetas

o b. Críoprecipitado

o c. Plasma

@ d. Concentrado de glóbulos rojos X

La respuesta correcta es: Concentrado de plaquetas


/
Pregunta 8 Correcta Puntúa 0,16 sobre 0,16

La clase IV de la nefritis lúpica se llama:

Seleccione una:

@ a. Nefritis difusa

o b. Nefritis esclerótica

o c. Nefritis focal

o d. Proliferación mesangial

La respuesta correcta es: Nefritis difusa

Pregunta 9 Correcta Puntúa 0,16 sobre 0,16

La clase VI de la nefritis lúpica se llama:

Seleccione una:

o a. Nefritis difusa

o b. Proliferación mesangial

@ c. Nefritis esclerótica

o d. Nefritis focal

La respuesta correcta es: Nefritis esclerótica


/
Pregunta 10 Correcta Puntúa 0,16 sobre 0,16

La fragmentación de Eritrocitos (Esquitocitos) imagen en sangre periférica que observamos en pacientes


con:

Seleccione una:

@ a. Válvulas Cardíacas Mecánicas

o b. Anemia por trastorno de maduración (A. megaloblástica)

o c. Daño medular. Hipoplasia

o d. Dé cit de hierro

La respuesta correcta es: Válvulas Cardíacas Mecánicas

Pregunta 11 Correcta Puntúa 0,16 sobre 0,16

La hepatitis B que nefropatía produce:

Seleccione una:

o a. Nefritis lúpica

o b. Nefropatía por IgA

o c. Granulomatosis de Wegener

@ d. Glomerulonefritis membranoproliferativa

La respuesta correcta es: Glomerulonefritis membranoproliferativa


/
Pregunta 12 Correcta Puntúa 0,16 sobre 0,16

La hipertensión esencial corresponde a

Seleccione una:

o a. Mas frecuente en pacientes jóvenes

@ b. Se relaciona con alteración en la excreción de sodio

o c. La minoriade paciente hipertensos

o d. Se relaciona con alteración en la excreción de calcio

La respuesta correcta es: Se relaciona con alteración en la excreción de sodio

Pregunta 13 Incorrecta Puntúa 0,00 sobre 0,16

La obstrucción ureteral bilateral aguda se caracteriza por:

Seleccione una:

o a. Disminución de la liberación de óxido nítrico

o b. Disminución de la producción de angiotensina

@ c. Disminución del ujo sanguíneo medular X


o d. Disminución de prostaglandinas vasodilatadoras

La respuesta correcta es: Disminución de la liberación de óxido nítrico


/
Pregunta 14 Correcta Puntúa 0,16 sobre 0,16

La obstrucción ureteral bilateral crónica se caracteriza por:

Seleccione una:

o a. Diuresis posterior a la obstrucción

@ b. Hipertensión

o c. Dolor renal

o d. Anuria

La respuesta correcta es: Hipertensión

Pregunta 15 Correcta Puntúa 0,16 sobre 0,16

La proteinuria sostenida es aquella que se expulsan más de:

Seleccione una:

o a. 7 a 8 g/24 h

o b. 5 a 6 g/24 h

o c. 3 a 4 g/24 h

@ d. 1 a 2 g/24 h

La respuesta correcta es: 1 a 2 g/24 h


/
Pregunta 16 Correcta Puntúa 0,16 sobre 0,16

Los Eritrocitos jóvenes que con la coloración de azul de metileno observamos restos de ARN y su aumento
indica buena respuesta de la médula ante una anemia. Cuál célula nos referimos?

Seleccione una:

o a. Células en Diana

o b. Esquitocitos

o c. Cuerpos de Howel Jolly

@ d. Reticulocitos

La respuesta correcta es: Reticulocitos

Pregunta 17 Correcta Puntúa 0,16 sobre 0,16

Paciente con reacción transfusional Ana láctica a repetición. Debe ser estudiado para descartar:

Seleccione una:

@ a. Dé cit de Ig A

o b. Dé cit de Ig M

o c. Dé cit de Ig E

o d. Dé cit de Ig G

La respuesta correcta es: Dé cit de Ig A


/
Pregunta 18 Correcta Puntúa 0,16 sobre 0,16

Paciente hipertenso con uso de 3 antihipertensivos uno de los cuales es un diurético que persiste con
presiones arteriales elevadas. Usted pensaría en

Seleccione una:

@ a. Hipertension resistente

o b. Hipertension de bata blanca

o c. Hipertension enmascarada

o d. Hipertension refractaria

La respuesta correcta es: Hipertension resistente

Pregunta 19 Correcta Puntúa 0,16 sobre 0,16

Paciente que en monitoreo ambulatorio de presión arterial presenta cifras tensionales en parámetros
adecuados, pero en el consultorio presenta cifras tensionales elevadas. Usted pensaría en

Seleccione una:

@ a. Hipertension de bata blanca

o b. d)Hipertension refractaria

o c. c)Hipertension resistente

o d. Hipertension enmascarada

La respuesta correcta es: Hipertension de bata blanca


/
Pregunta 20 Correcta Puntúa 0,16 sobre 0,16

Paciente que presenta cifras tensionales en el consultorio dentro de parámetros normales, pero evidencia
de retinopatía hipertensiva. Usted pensaría en

Seleccione una:

o a. Hipertension resistente

o b. Hipertension refractaria

o c. Hipertension de bata blanca

@ d. Hipertension enmascarada

La respuesta correcta es: Hipertension enmascarada

Pregunta 21 Correcta Puntúa 0,16 sobre 0,16

Qué antidepresivo causa disfunción del nódulo sinoauricular:

Seleccione una:

@ a. Amitriptilina

o b. Sertralina

o c. Fluoxetina

o d. Midazolam

La respuesta correcta es: Amitriptilina


/
Pregunta 22 Correcta Puntúa 0,16 sobre 0,16

Qué endocrinopatía no causa prolongación de QT y de taquicardia ventricular polimorfa:

Seleccione una:

o a. Hipotiroidismo

@ b. Hipogonadismo

o c. Hiperladosteronismo

o d. Hiperparatiroidismo

La respuesta correcta es: Hipogonadismo

Pregunta 23 Correcta Puntúa 0,16 sobre 0,16

Qué familia de antibiótico no causa prolongación de QT y de taquicardia ventricular polimorfa:

Seleccione una:

@ a. Cefalosporinas

o b. Macrólidos

o c. Quinolonas

o d. Antagonistas del folato

La respuesta correcta es: Cefalosporinas


/
Pregunta 24 Correcta Puntúa 0,16 sobre 0,16

Qué vitamina causa insu ciencia cardiaca con alto gasto:

Seleccione una:

@ a. B

o b. D

o c. E

o d. A

La respuesta correcta es: B

Pregunta 25 Correcta Puntúa 0,16 sobre 0,16

Una anemia con un Índice de Producción de Reticulocítos ( IPR ) mayor de 2 % es una anemia
hiperproliferativa . Cuál sería la causa de anemia?

Seleccione una:

o a. Hipoplasia medular

o b. Disfunción Renal

o c. Dé cit de hierro

@ d. Hemorragia y Hemólisis

La respuesta correcta es: Hemorragia y Hemólisis

◄ Link sesión zoom examen nal 1: Investigación I Ir a... V

Link sesión zoom examen segundo parcial ►


/
CARDIO
Pregunta 1
Correcta
Puntúa 1,00 sobre 1,00

Marcar pregunta

Enunciado de la pregunta

LA HIPERTROFIA CONCENTRICA SE ASOCIA A AUMENTO DE POSCARGA


Seleccione una:

r. Verdadero
r Falso

Retroalimentación
La respuesta correcta es 'Verdadero'

Pregunta 2
Correcta
Puntúa 1,00 sobre 1,00

Marcar pregunta

Enunciado de la pregunta

Dentro de los signos y sintomas de insuficiencia cardiaca derecha son

Seleccione una:
r a. a ) disnea paroxistica nocturna
r b. Ninguna
r c. b) edema agudo de pulmon

r. d. Hepatomegalia

Retroalimentación

Respuesta correcta
La respuesta correcta es: Hepatomegalia

Pregunta 3
Incorrecta
Puntúa 0,00 sobre 1,00
Marcar pregunta

Enunciado de la pregunta

Una oclusion total de coronarias por placas ateromatomatosas se asocia


Seleccione una:
r a. b) Angina vasoespastica

b. c)Angina inestable
r c. d) Inf arto agudo de miocardio
r d. a )Angina estable

Retroalimentación
Respuesta incorrecta.
La respuesta correcta es: d) Inf arto agudo de miocardio

Pregunta 4
Incorrecta
Puntúa 0,00 sobre 1,00

Marcar pregunta
Enunciado de la pregunta

El ANP y Pro Bnp se asocia a disminucion de natriuresis


Seleccione una:

Verdadero
r Falso

Retroalimentación
La respuesta correcta es 'Falso'

Pregunta 5
Correcta
Puntúa 1,00 sobre 1,00

Marcar pregunta

Enunciado de la pregunta
Este paciente se realiza un electrocardiograma en el que se evidencia R en AVL MAS
S en V3 de 30 mv
Usted considera que es

Seleccione una:
r a. )Hipertrof ia ventricular derecha
r b. )Bloqueo de rama izquierda

c. )Hipertrof ia ventricular izquierda


r d. )Bloqueo de rama derecha

Retroalimentación

Respuesta correcta
La respuesta correcta es: )Hipertrof ia ventricular izquierda

Pregunta 6
Correcta
Puntúa 1,00 sobre 1,00

Marcar pregunta

Enunciado de la pregunta

Usted evalua un paciente de 75 anos que acude a Emergencias por dolor precordial
tipo opresivo
de 30 minutos de evolucion ,al examen fisico evidencia un soplo entre R1 y R2 que
aumenta con maniobra de
Valsalva y se ubica de predominio en segundo espacio intercostal derecho y se irradia
al cuello.
Su diagnostico
del soplo es

Seleccione una:
r a. Insuf iciencia Aortica

r. b. Estenosis aortica
r c. Insuf iciencia mitral
r d. Estenosis Mitral

Retroalimentación
Respuesta correcta
La respuesta correcta es: Estenosis aortica

Pregunta 7
Correcta
Puntúa 1,00 sobre 1,00

Marcar pregunta
Enunciado de la pregunta

Dentro de las causas de hipertension secundaria estan excepto


Seleccione una:
r a. Sindrome de apnea obstructiva del sueno
r b. a) Hiperparatiroidismo

c. insulinoma
r d. b)Feocromocitoma

Retroalimentación

Respuesta correcta
La respuesta correcta es: insulinoma

Pregunta 8
Incorrecta
Puntúa 0,00 sobre 1,00

Marcar pregunta

Enunciado de la pregunta

Este paciente se realiza Ecocardiograma en donde se evidencia un area valvular de


0,8 cm2,razon por la que presenta criterios para cirugia valvular
Seleccione una:
r Verdadero

r. Falso

Retroalimentación
La respuesta correcta es 'Verdadero'

Pregunta 9
Correcta
Puntúa 1,00 sobre 1,00
Marcar pregunta

Enunciado de la pregunta

Usted valora a un paciente de 25 anos con hipofosfatemia,hipercalcemia mas


hipertension
Usted pensaria
Seleccione una:
r a. Hiperaldosteronismo

b. Hiperparatiroidismo
r c. Coartacion de la aorta
r d. f eocromocitoma

Retroalimentación
Respuesta correcta
La respuesta correcta es: Hiperparatiroidismo

Pregunta 10
Incorrecta
Puntúa 0,00 sobre 1,00

Marcar pregunta

Enunciado de la pregunta

La disnea es uno de los criterios mayores de insuficiencia cardiaca


Seleccione una:
r. Verdadero
r Falso

Retroalimentación

La respuesta correcta es 'Falso'

Pregunta 11
Correcta
Puntúa 1,00 sobre 1,00

Marcar pregunta
Enunciado de la pregunta

Usted valora a un paciente de 15 anos con cefalea mas hipertension,diferencia de


presiones entre brazos mayor a 10 mm Hg
y pulsos asimetricos. Usted pensaria en

a)

Seleccione una:
r a. b)Hiperparatiroidismo
(.'
b. Coartacion de la aorta
r c. Feocromocitoma
r d. Hiperaldosteronismo

Retroalimentación

Respuesta correcta
La respuesta correcta es: Coartacion de la aorta

Pregunta 12
Correcta
Puntúa 1,00 sobre 1,00

Marcar pregunta

Enunciado de la pregunta

Usted valora un paciente de 25 anos con diaforesis,palpitaciones ,hipertension


paroxistica,cefalea
Usted pensaria en

Seleccione una:
r a. Hiperaldosteronismo
r b. Hiperparatiroidismo

c. Feocromocitoma
r d. Coartacion de la aorta

Retroalimentación

Respuesta correcta
La respuesta correcta es: Feocromocitoma

Pregunta 13
Correcta
Puntúa 1,00 sobre 1,00
Marcar pregunta

Enunciado de la pregunta

Usted valora a un paciente de 25 anos con alcalemia metabolica,hipocalemia mas


hipertension
Usted pensaria en

Seleccione una:
r a. Coartacion de aorta
r b. Feocromocitoma
r c. Hiperparatiroidismo

d. Hiperaldosteronismo

Retroalimentación
Respuesta correcta
La respuesta correcta es: Hiperaldosteronismo

Pregunta 14
Incorrecta
Puntúa 0,00 sobre 1,00

Marcar pregunta

Enunciado de la pregunta
La aldosterona provoca
Seleccione una:
r a. )Perdida de sodio por la orina
r b. hipercalemia
('
c. f ibrosis miocardica

d. Ninguna de las anteriores

Retroalimentación
Respuesta incorrecta.
La respuesta correcta es: f ibrosis miocardica
HEMATO

Pregunta 1
Correcta
Puntúa 1,00 sobre 1,00

Marcar pregunta

Enunciado de la pregunta

El Indice de producción de los reticulocitos (IPR) ayuda a clasificar las anemias en


arregerativas y regenerativas, si tenemos un IPR >2. ¿Cuál de estas anemias
corresponde a ser regenerativas?
Seleccione una:

a. Hemorragia aguda
r b. Anemia por déf icit de hierro
r c. Anemia de proceso crónico
r d. Hipoplasia medular

Retroalimentación
Respuesta correcta
La respuesta correcta es: Hemorragia aguda

Pregunta 2
Correcta
Puntúa 1,00 sobre 1,00

Marcar pregunta

Enunciado de la pregunta

¿Cuáles son los criterios de hipoplasia medular severa?


Seleccione una:
r a. La f iebre y anemia
r b. Petequias, las mucositis
r c. Epistaxis, sangrado por mucosas

d. Reticulocitos <de 1 corregido, neutrof ilos < de 500 cif ras absolutas

Retroalimentación
Respuesta correcta
La respuesta correcta es: Reticulocitos <de 1 corregido, neutrof ilos < de 500 cif ras absolutas
Pregunta 3
Correcta
Puntúa 1,00 sobre 1,00

Marcar pregunta

Enunciado de la pregunta

¿Cuál es la prueba que hace diagnóstico de anemia hemolítica autoinmune?


Seleccione una:
r a. Electrof orésis de hemoglobina
r b. Anticuerpos antinucleares

r. c. La prueba de coombs directa


r d. El f rótis perif érico

Retroalimentación
Respuesta correcta
La respuesta correcta es: La prueba de coombs directa

Pregunta 4
Correcta
Puntúa 1,00 sobre 1,00

Marcar pregunta

Enunciado de la pregunta

Las anemias hemolíticas congénitas son intracorpusculares porque tienen un defecto


en los hematíes, ¿Dónde ocurre la hemólisis de los hematies?
Seleccione una:
r a. En los macróf agos de higado.
r b. En la microcirculación

r. c. En el bazo, (SRE) macróf ago, son Extravascular


r d. En los vasos sanguineos Intravascular

Retroalimentación
Respuesta correcta
La respuesta correcta es: En el bazo, (SRE) macróf ago, son Extravascular

Pregunta 5
Correcta
Puntúa 1,00 sobre 1,00
Marcar pregunta

Enunciado de la pregunta

Las anemias microangiopáticas,, son anemias agudas graves como la CID, SUH.
intravasculares, que datos morfológicos en el frotis la identifican?
Seleccione una:
r a. Cuerpos de Heinz

f.'
b. Hematíes f ragmentados (esquistocitos)
r c. Polisementación de neutrof ilos
r d. Células en diana

Retroalimentación
Respuesta correcta
La respuesta correcta es: Hematíes f ragmentados (esquistocitos)

Pregunta 6
Correcta
Puntúa 1,00 sobre 1,00

Marcar pregunta

Enunciado de la pregunta

El plasma es la parte líquida de la sangre separada de la parte célular, señale su


indicación en estas patologías?
Seleccione una:
r a. Hemof ilia A

f.'
b. Hemof ilia B
r c. El niño con PTI con petequias
r d. Para ayudar a cicatrizar heridas

Retroalimentación
Respuesta correcta
La respuesta correcta es: Hemof ilia B

Pregunta 7
Correcta
Puntúa 1,00 sobre 1,00
Marcar pregunta

Enunciado de la pregunta

¿Cuál es el grupo sanguíneo Donante Universal,pero el no puede recibir de ningún


otro grupo?
Seleccione una:
r a. Grupo AB -
r b. Grupo O +

c. Grupo O -
r d. Grupo AB+

Retroalimentación
Respuesta correcta
La respuesta correcta es: Grupo O -

Pregunta 8
Correcta
Puntúa 1,00 sobre 1,00

Marcar pregunta

Enunciado de la pregunta

La anemia por déficit de hierro es la mas frecuente que existe en todas las edades,
¿Cuál es la causa y cuanto tarda en producirse?
Seleccione una:

a. Tarda un año y su causa es porque solo se absorbe el 10% de hierro en la dieta


('
b. Tarda 4 meses y se absorbe el 80%
r c. Tiene buena absorcíón
r d. Tarda 4 años y tiene depósitos

Retroalimentación
Respuesta correcta
La respuesta correcta es: Tarda un año y su causa es porque solo se absorbe el 10% de hierro
en la dieta

Pregunta 9
Correcta
Puntúa 1,00 sobre 1,00
Marcar pregunta

Enunciado de la pregunta

Cuando cumplimos un concentrado globular para tratar una anemia,


aportando oxigeno. ¿Cuánto aumenta la hemoglobina por 1 unidad de concentrado
transfundido?
Seleccione una:

a. Aumenta 1.5 g/dl y 4% del hematocríto


r b. Aumenta un 8%
r c. Aumenta un 3%
r d. Aumenta un 5%

Retroalimentación
Respuesta correcta
La respuesta correcta es: Aumenta 1.5 g/dl y 4% del hematocríto

Pregunta 10
Correcta
Puntúa 1,00 sobre 1,00

Marcar pregunta

Enunciado de la pregunta

Una pareja del grupo O. ¿Qué posibilidad tiene de tener un hijo con estos grupo?
Seleccione una:
r a. 25% de posibilidad de tener un hijo grupo A
r b. 25% de probabilidad de tener un grupo B
r c. Otro 25% del grupo AB

d. La posibilidad 100% un hijo grupo O

Retroalimentación
Respuesta correcta
La respuesta correcta es: La posibilidad 100% un hijo grupo O

NEFROLOGIA

Pregunta 1
Finalizado
Puntúa 1,00 sobre 1,00

Marcar pregunta

Enunciado de la pregunta

Cuál no es un biomarcador?

Seleccione una:

A. Adiponectina
r B. KIM 1
r C. IL 18
r D. NGAL

Retroalimentación
La respuesta correcta es: Adiponectina

Pregunta 2
Finalizado
Puntúa 0,00 sobre 1,00

Marcar pregunta

Enunciado de la pregunta

Según Harrison, cuántas semanas se requieren para catalogar cistitis


recurrente?

Seleccione una:
r A. 2 semanas

B. 8 semanas
r C. 6 semanas
r D. 4 semanas

Retroalimentación
La respuesta correcta es: 2 semanas

Pregunta 3
Finalizado
Puntúa 0,00 sobre 1,00
Marcar pregunta

Enunciado de la pregunta

Según Harrison, cuál no es un factor anatómico que retrasa el vaciamiento


vesical para ITU en postmenopáusicas?

Seleccione una:

A. Orina residual
r B. Estenosis ureteral
r C. Cistoceles
r D. Incontinencia urinaria

Retroalimentación

Las respuestas correctas son: Incontinencia urinaria, Estenosis ureteral

Pregunta 4
Finalizado
Puntúa 0,00 sobre 1,00

Marcar pregunta

Enunciado de la pregunta

Según Harrison, cuál NO es un factor independiente de riesgo para cistitis no


complicada en mujeres?
Seleccione una:
r A. Uso reciente de T de cobre

B. Coitos f recuentes
r C. Antecedentes de UTI
r D. Uso reciente de un diaf ragma con espermicida.

Retroalimentación

La respuesta correcta es: Uso reciente de T de cobre

Pregunta 5
Finalizado
Puntúa 1,00 sobre 1,00
Marcar pregunta

Enunciado de la pregunta

Un RIFLE 2 se caracteriza por?

Seleccione una:
r A. Creatinina al cuádruple
r B. Creatinina al triple
r. C. Creatinina al doble
r D. Creatinina al quíntuple

Retroalimentación

La respuesta correcta es: Creatinina al doble

Pregunta 6
Finalizado
Puntúa 1,00 sobre 1,00

Marcar pregunta

Enunciado de la pregunta

Cuál no se considera causa para azotemia prerrenal?

Seleccione una:
r. A. Hiperplasia prostática
r B. Diarrea aguda
r C. Vómito a repetición
r D. Hemorragia digestiva alta

Retroalimentación

La respuesta correcta es: Hiperplasia prostática

Pregunta 7
Finalizado
Puntúa 1,00 sobre 1,00
Marcar pregunta

Enunciado de la pregunta

Según Harrison, los factores que predisponen a las mujeres a padecer cistitis,
aumentan el peligro de?
Seleccione una:
r A. Uretritis
r B. Absceso perirenal
r C. Vaginitis

D. Pielonef ritis

Retroalimentación

La respuesta correcta es: Pielonef ritis

Pregunta 8
Finalizado
Puntúa 0,00 sobre 1,00

Marcar pregunta

Enunciado de la pregunta

Cuál no se considera un tratamiento para hiperpotasemia?

Seleccione una:
r A. Furosemida
r B. Bicarbonato de calcio
r C. Gluconato de calcio

D. Solución polarizante

Retroalimentación
La respuesta correcta es: Bicarbonato de calcio

Pregunta 9
Finalizado
Puntúa 0,00 sobre 1,00

Marcar pregunta
Enunciado de la pregunta

Si el cálculo se ubica en la porción inferior del uréter, el dolor puede irradiarse


en mujeres hacia?

Seleccione una:
.:)
A.
Útero
J
B.
Pubis
J
C.
Labio mayor ipsilateral
J
D. Miometrio

Retroalimentación
La respuesta correcta es:
Labio mayor ipsilateral

Pregunta 10
Finalizado
Puntúa 0,00 sobre 1,00

Marcar pregunta

Enunciado de la pregunta

El Síndrome de Alport que nefropatía produce?

Seleccione una:
.:)
A. Glomerulonef ritis membranoprolif erativa
J
B. Granulomatosis de Wegener
J
C. Nef ropatía por IgA
.J
D. Glomeruloesclerosis segmentaria f ocal

Retroalimentación
La respuesta correcta es: Glomeruloesclerosis segmentaria f ocal

Pregunta 11
Finalizado
Puntúa 0,00 sobre 1,00

Marcar pregunta
Enunciado de la pregunta

Qué es el MDRD?

Seleccione una:
r. A. Dieta baja en la enf ermedad renal
r B. Dieta modif icada en la enf ermedad renal
r C. Dieta única en la enf ermedad renal
r D. Dieta básica en la enf ermedad renal

Retroalimentación

La respuesta correcta es: Dieta modif icada en la enf ermedad renal

Pregunta 12
Finalizado
Puntúa 0,00 sobre 1,00

Marcar pregunta
Enunciado de la pregunta

La obstrucción ureteral bilateral crónica se caracteriza por?

Seleccione una:
r A. Hipertensión
r B. Dolor renal
r C. Anuria
r. D. Diuresis posterior a la obstrucción

Retroalimentación
La respuesta correcta es: Hipertensión

Pregunta 13
Finalizado
Puntúa 1,00 sobre 1,00

Marcar pregunta
Enunciado de la pregunta

Qué valor define oliguria?


Seleccione una:
r A. 0 a 50 ml

B. 100 a 400 ml
r C. 1000 a 2000 ml
r D. 500 a 1000 ml

Retroalimentación
La respuesta correcta es: 100 a 400 ml

Pregunta 14
Sin contestar
Puntúa como 1,00

Marcar pregunta

Enunciado de la pregunta

Un AKIN 3 se caracteriza por?

Seleccione una:
r A. Descenso del FG superior a 15 %
r B. Descenso del FG superior a 25 %
r C. Descenso del FG superior a 75%
r D. Descenso del FG superior a 50 %

Retroalimentación
La respuesta correcta es: Descenso del FG superior a 75%

Pregunta 15
Finalizado
Puntúa 0,00 sobre 1,00

Marcar pregunta

Enunciado de la pregunta

Según Harrison, cuál es un microorganismo virulento para ITU por propagación


hematógena:
postmenopáusicas?

Seleccione una:
r A. Acinetobacter Baumani
.:)
B. Proteus mirabilis
J
C. Estreptococo aureus
J
D. Estaf ilococo aureus

Retroalimentación
La respuesta correcta es: Estaf ilococo aureus

RESPIRATORIO

Pregunta 1
Correcta
Puntúa 1,00 sobre 1,00

Marcar pregunta
Enunciado de la pregunta

En la gasometría realizada en un paciente con Insuficiencia Respiratoria Aguda


esperaríamos encontrar:

Seleccione una:
.:)
a. PaO2 menor de 60 mmHg-PaCO2 mayor de 45 mmHg
J
b. PaO2 menor de 60 mmHg-PaCO2 menor de 45 mmHg
J
c. PaO2 mayor de 60 mmHg-PaCO2 menor de 45 mmHg
J
d. PaO2 mayor de 60 mmHg-PaCO2 mayor de 45 mmHg

Retroalimentación

Respuesta correcta
La respuesta correcta es: PaO2 menor de 60 mmHg-PaCO2 mayor de 45 mmHg

Pregunta 2
Correcta
Puntúa 1,00 sobre 1,00

Marcar pregunta

Enunciado de la pregunta

Señale el germen que no produce la tuberculosis pulmonar:


Seleccione una:
r a. Mycobacterium bovis.
r b. Mycobacterium af ricanum.
r c. Mycobacterium tuberculae.

f.'
d. Mycobacterium leprae.

Retroalimentación

Respuesta correcta
La respuesta correcta es: Mycobacterium leprae.

Pregunta 3
Correcta
Puntúa 1,00 sobre 1,00

Marcar pregunta

Enunciado de la pregunta

Masculino de 65 años de edad se presenta con una historia de disnea progresiva y


tos no productiva de 3 días de evolución. Había sido hospitalizado previamente hace
2 años por insuficiencia cardiaca congestiva. Sus signos vitales: TA 90/55 mmHg,
frecuencia cardiaca de 110 latidos por minuto, temperatura de 37.6C, y saturación de
oxígeno al aire ambiente es de 86%. A la auscultación de tórax, se detectan estertores
y roncus bilaterales. La radiografía de tórax revela infiltrados bilaterales y un aumento
discreto del tamaño de la silueta cardiaca. ¿Cuál es el diagnóstico clínico más
probable?

Seleccione una:
r a. Disección de Aorta
r b. Angina estable
('
c. Tromboembolia pulmonar

f.'
d. Edema agudo de pulmón

Retroalimentación

Respuesta correcta
La respuesta correcta es: Edema agudo de pulmón

Pregunta 4
Correcta
Puntúa 1,00 sobre 1,00
Marcar pregunta

Enunciado de la pregunta

La pérdida de gran volumen de sangre, conocida como hemoptisis masiva se define


de manera variable como:
Seleccione una:
r a. Pérdida que rebasa los 600 a 900 ml en 24 h
r b. Pérdida que rebasa los 100 a 300 ml en 24 h

c. Pérdida que rebasa los 400 a 600 ml en 24 h


r d. Pérdida que rebasa los 200 a 600 ml en 24 h

Retroalimentación
Respuesta correcta
La respuesta correcta es: Pérdida que rebasa los 400 a 600 ml en 24 h

Pregunta 5
Correcta
Puntúa 1,00 sobre 1,00

Marcar pregunta

Enunciado de la pregunta

Hombre de 68 años con antecedente de neoplasia de páncreas en curso de


quimioterapia. Consulta en Emergencias por dolor y edema de todo el miembro
inferior desde ingle. ¿Qué prueba diagnóstica es más coste-efectiva para
confirmar la sospecha diagnóstica?

Seleccione una:
r a. Resonancia Magnética
r b. Tomograf ía Axial Computarizada
r c. Dímero D

d. Ecograf ía doppler venosa

Retroalimentación
Respuesta correcta
La respuesta correcta es: Ecograf ía doppler venosa

Pregunta 6
Correcta
Puntúa 1,00 sobre 1,00

Marcar pregunta

Enunciado de la pregunta

Las tres alteraciones funcionales básicas del asma son:


Seleccione una:
r a. Obstrucción al f lujo de aire, obstrucción al f lujo venoso e irreversibilidad de la vía aérea

b. Obstrucción al f lujo de aire, reversibilidad de la vía aérea e hiperreactividad bronquial


r c. Hipertensión arterial, hiperreactividad bronquial y obstrucción nasal
r d. Tos, obstrucción f lujo aéreo y somnolencia

Retroalimentación
Respuesta correcta
La respuesta correcta es: Obstrucción al f lujo de aire, reversibilidad de la vía aérea e
hiperreactividad bronquial

Pregunta 7
Correcta
Puntúa 1,00 sobre 1,00

Marcar pregunta

Enunciado de la pregunta

Es la mejor prueba complementaria para el diagnóstico de asma:

Seleccione una:
r a. Broncoscopia
r b. Lavado bronquial
r c. Determinación de inmunoglobulinas

d. Espirometría

Retroalimentación

Respuesta correcta
La respuesta correcta es: Espirometría

Pregunta 8
Correcta
Puntúa 1,00 sobre 1,00
Marcar pregunta

Enunciado de la pregunta

Señale la respuesta incorrecta con relación a la tuberculosis:


Seleccione una:
r a. La enf ermedad tuberculosa es aquella situación en la que el individuo presenta datos
clínicos.
r b. La inf ección tuberculosa es aquella situación en la que el individuo entra en contacto con el
germen

c. La primoinf ección tuberculosa es aquella situación en la que el individuo entra en contacto

con el germen y desarrolla por primera vez la enf ermedad .


r d. La tuberculosis posprimaria es aquella situación en la que el individuo presenta una
reactivación de la enf ermedad tuberculosa.

Retroalimentación

Respuesta correcta
La respuesta correcta es: La primoinf ección tuberculosa es aquella situación en la que el
individuo entra en contacto con el germen y desarrolla por primera vez la enf ermedad .

Pregunta 9
Correcta
Puntúa 1,00 sobre 1,00

Marcar pregunta

Enunciado de la pregunta

Cuál de las siguientes premisas es FALSA respecto del diagnóstico de


Tromboembolismo de Pulmón:

Seleccione una:
r a. Los escores de Ginebra y de Wells permite estratif icar la probabilidad diagnóstica de un
Troboembolismo de Pulmón.
r b. Ante una baja probabilidad clínica de Tromboembolismo de Pulmón un dímero -D de alta
sensibilidad negativo permite excluirlo.

c. El ecodoppler venoso es el estudio de mayor sensibilidad y especif icidad para el diagnóstico

de Tromboembolismo de Pulmón.
r d. La disnea y la taquipnea son las manif estaciones clínicas más f recuentes

Retroalimentación
Respuesta correcta
La respuesta correcta es: El ecodoppler venoso es el estudio de mayor sensibilidad y
especif icidad para el diagnóstico de Tromboembolismo de Pulmón.

Pregunta 10
Correcta
Puntúa 1,00 sobre 1,00

Marcar pregunta

Enunciado de la pregunta

Son factores de riesgo para neumonía nosocomial


Seleccione una:

a.

Aspiración de contenido gástrico, Reintubación


r b.
Diabetes Mellitus
r c. Edad y Etnia
r d.
Extremos de la vida

Retroalimentación
Respuesta correcta
La respuesta correcta es:
Aspiración de contenido gástrico, Reintubación

Pregunta 11
Correcta
Puntúa 1,00 sobre 1,00

Marcar pregunta

Enunciado de la pregunta

La decisión del ingreso hospitalario de un/a paciente con NAC depende de la


capacidad del médico para predecir la probabilidad de muerte. Según el índice
CURB65, ¿cuándo se recomienda la hospitalización?

Seleccione una:
r a. Cuando la puntuación sea de 0 puntos.

b. Cuando la puntuación sea > de 2 puntos.


r c. Cuando la puntuación sea de 1 puntos.
r d. Cuando la puntuación sea de 1,5 puntos.

Retroalimentación
Respuesta correcta
La respuesta correcta es: Cuando la puntuación sea > de 2 puntos.

Pregunta 12
Correcta
Puntúa 1,00 sobre 1,00

Marcar pregunta
Enunciado de la pregunta

Cuál es el cuadro clínico característico de los pacientes con Bronquitis crónica?


Seleccione una:
r a. llamados sopladores azules, aumento de peso y coloración normal de la piel
r b. llamados tosedores rosados, con espiración prolongada y aumento de peso

c. llamados sopladores azules, cianóticos y caquécticos


r d. llamados tosedores rosados, hemoptisis sin tos productiva

Retroalimentación

Respuesta correcta
La respuesta correcta es: llamados sopladores azules, cianóticos y caquécticos

Pregunta 13
Correcta
Puntúa 1,00 sobre 1,00

Marcar pregunta

Enunciado de la pregunta

Cuáles son los principales factores de riesgo para el desarrollo de NAC?.

Seleccione una:
r a. Alcoholismo, parejas sexuales múltiples, uso de anticonceptivos orales, antecedentes de
cirugía cardiaca o pulmonar
r b. Enf ermedad de Alzheimer, f ibrosis quística, pacientes entre 30 y 50 años de edad, uso de
anticonceptivos orales, antecedentes de cirugía cardiaca o pulmonar .
r c. Múltiples pareja sexuales, EPOC, tabaquismo, Inmunodeprimidos, adultos mayores sanos, y
personas adultas jóvenes
r. d. Extremos de edad (ancianos y niños), tabaquismo, alcoholismo, diabetes, asma,

insuf iciencia cardíaca, EPOC, cáncer, inmunodeprimidos.

Retroalimentación
Respuesta correcta
La respuesta correcta es: Extremos de edad (ancianos y niños), tabaquismo, alcoholismo,
diabetes, asma, insuf iciencia cardíaca, EPOC, cáncer, inmunodeprimidos.

Pregunta 14
Correcta
Puntúa 1,00 sobre 1,00

Marcar pregunta
Enunciado de la pregunta

Mujer de 54 años es encontrada desorientada por unos transeúntes. Al llegar el equipo


de emergencias la encontraron con una saturación de oxígeno del 80% respirando aire
ambiente y al examen físico pupilas puntiformes. Se la traslada a la emergencia del
hospital, donde la gasometría arterial basal muestra: pH 7,25, PaC02 60 mmHg, Pa02
56 mmHg, bicarbonato de 26 mEq/l y exceso de bases de -1. En sangre el sodio es
136 mEq/1 y el cloruro 100 mEq/1. Desde el punto de vista gasométrico la
paciente tiene:

Seleccione una:

r. a. Acidosis respiratoria pura


r b. Insuf iciencia respiratoria parcial
r c. Acidosis metabòlica.
r d. La gasometría solo puede ser de sangre venosa

Retroalimentación
Respuesta correcta
La respuesta correcta es: Acidosis respiratoria pura

Pregunta 15
Correcta
Puntúa 1,00 sobre 1,00
Marcar pregunta

Enunciado de la pregunta

Criterios para derrame pleural exudativo, excepto:

Seleccione una:
r a. Proteínas de líquido pleural/proteínas séricas >0.5
r b. LDH del líquido pleural >66% del límite superior normal para el suero

c. LDH del líquido pleural/LDH sérica ˂0.4


r d. LDH del líquido pleural/LDH sérica >0.6

Retroalimentación
Respuesta correcta
La respuesta correcta es: LDH del líquido pleural/LDH sérica ˂0.4

Pregunta 16
Correcta
Puntúa 1,00 sobre 1,00

Marcar pregunta

Enunciado de la pregunta

Cuáles son los 3 síntomas o signos más frecuentes de la EPOC

Seleccione una:
r a. Tos crónica, disnea crónica y sibilancias
r b. Tos crónica, disnea crónica y utilización de músculos accesorios

c. Tos crónica, disnea crónica y expectoración mucosa


r d. Tos crónica, disnea crónica y f iebre

Retroalimentación
Respuesta correcta
La respuesta correcta es: Tos crónica, disnea crónica y expectoración mucosa

Pregunta 17
Correcta
Puntúa 1,00 sobre 1,00
Marcar pregunta

Enunciado de la pregunta

Con respecto a la EPOC, no es cierto:


Seleccione una:
r a. Espirométricamente, se detecta obstrucción por un cociente FEV1/FVC inf erior a 0,70

b. Contrariamente al asma, no hay componente inf lamatorio


r c. El f actor genético para desarrollar EPOC mejor documentado es el déf icit de alf a1-
antitripsina
r d. El hábito tabáquico es el f actor más importante para desarrollar EPOC

Retroalimentación

Respuesta correcta
La respuesta correcta es: Contrariamente al asma, no hay componente inf lamatorio

Pregunta 18
Correcta
Puntúa 1,00 sobre 1,00

Marcar pregunta

Enunciado de la pregunta

En la exploración funcional de un paciente con enfermedad pulmonar obstructiva


crónica, son esperables todos los hallazgos MENOS uno:

Seleccione una:
r a. Cociente FEV1/FVC inf erior al 0.7
r b. FEV1 menor del 80%.
r c. Prueba broncodilatadora negativa

d. Volúmenes pulmonares disminuidos

Retroalimentación

Respuesta correcta
La respuesta correcta es: Volúmenes pulmonares disminuidos

Pregunta 19
Correcta
Puntúa 1,00 sobre 1,00

Marcar pregunta

Enunciado de la pregunta

Actualmente se utilizan 3 clases de broncodilatadores en pacientes con asma, señale


lo correcto:

Seleccione una:
r a. Agonistas β2, corticoesteroides inhalados, corticoesteroides sistémicos

b. Agonistas β2, anticolinérgicos, teof ilina


r c. Anticolinérgicos, antileucotrienos, agonistas β2
r d. Antileucotrienos, teof ilina, agonistas β2

Retroalimentación

Respuesta correcta
La respuesta correcta es: Agonistas β2, anticolinérgicos, teof ilina

Pregunta 20
Correcta
Puntúa 1,00 sobre 1,00

Marcar pregunta

Enunciado de la pregunta

Cuál de las siguientes anomalías electrocardiográficas suele asociarse a


tromboembolia pulmonar?

Seleccione una:
r a. Inversión de ondas T en la derivación de ondas laterales I, AVL y V5- V6
r b. Ondas Q en las derivaciones anteriores de V1- V4
r c. Bloqueo A-V de primer grado

d. S1Q3T3, ondas T negativas en las derivaciones anteriores de V1 a V4

Retroalimentación

Respuesta correcta
La respuesta correcta es: S1Q3T3, ondas T negativas en las derivaciones anteriores de V1 a
V4
NEURO

1. La presencia de episodios de 2 minutos de disuasión caracterizaos por ¿?olfatoria, automatismois


oromandibulares y afectación de la conciencia sugieren

Ausencia típicas

2. En el trabajo dx de un pct de 60 A q presenta crisis parciales con gneralizacion secundaria además


de un adecuada HCL es necesario

EEG en vigilia, TC cerebral simple y contrastada

3. Una característica importante q permitiría diferencia las ausencias de las crisis parciales complejas
es

Afectación de la conciencia

4. Un hombre de 60 A diestro con antecedenetes de HTA y alcoholismo es encontrado por su hija


inconciente y con hemiparesia derecha a las 7 am. En la madrugada refirió una cefale pulsatil
holocraneana. Cual es el orden de un evento hemorrágico

Alteración del estado de conciencia / HTA / cefalea holocraneana / edad / hemiparesia derecha

5. La medida terapéutica en un pct hipertenso y con fibrilación auricular q presenta un infarto


cerebral de 3h de evolución con puntiacion de 14 en escala de NIHSS es

Trombosis intravenosa con activador del plasminogeno tisular 0.9mg-kg

6. En orden de mayor a menor importancia, los FR para enferemdad cerebro vascular se consideran

HTA, fibrilación auricular, edad, hipercolesterolemia

7. Un hallazgo clínico sugestivo de infarto cerebral en la circulación posterior o vertebro basiar en


presencia de una hemiparesia proporcionada de instauración aguda es

Paralisis del motor ocular externo ipsilateral al déficit motor

8. Pct de 50 A es trasladada inconsciente a la emergencia. Se intuba para protección de via aérea. Al


estimulo doloroso no abre los ojos, presenta flexión de la extremidad superior derecha con flexión
plantar del pie derecho. Babinski bilateral

9. Signo clínico importante q oriente a considerar un coma de origen metabolico mas que estructural
en un pct en coma es

Pupilas de 3mm con reflejo fotomotor presente

10. El contenido de la conciencia se encuentra conservado en

Sx de encerramiento

11. En el manejo mergente de un pct con alteración del estado de conciencia con pupiklas normales,
sin focalidad neurologica se debe considerar

Realizar glicemia capilar y administrar tiamina


12. Uno de los criterios para considerar la probabilidad de muerte cerebral y proceder a realizar el
teste de apnea es

Neuroimagen con una lesión estructural cerebral compatible con muerte cerebral

13. El examen confirmatorio mas adecuado de dx de muerte cerebral es

Electroencefalograma

14. Hombre 60 A presenta debilidad progresía y adormecimiento en las piernas después de 7 días de
haber presentado diarrea . niega dolor en las extremidades inferiores, el examen físico denuesta:
PA 80/50 140x T37 pares craneales normales, paresia 3/5 en las 4 extremidades, ausencia de nivel
sensitivo. Arreflexia global con respuestas plantares en flexión. Uno de los síntomas adicionales q
podría soportar el Dx de Sx de guillan barre es

Hipotensión arterial y taquicardia

15. El hematoma subdural por traumatismo cráneo encefálico

Es debido a ruptura de venas comunicantes entre el cortex y la duramadre

16. De los criterios diagnosticos de la sociedad internacional de la cefalea, para el Dx de migraña con
aura considera el mas importante

La duración de los síntomas del aura

17. Pct 21 A con historia de migraña desde la niñez acude con cefalea de presentaion diariam de un
mes de duración. Tiene cefalea holocraneana pulstil acompañada de nausea vomito fotofobia,
toma medicación para la migraña de ceta libre en la farmacio….la TAC cerebral simple es normal

Tiene migraña sin aura

18. Uno de los criterios diagnósticos clínico de la neuralgia del trigémino clásica es

Examen neurológico normal

19. A las cuantas horas se apreciaría en una tomografía un infarto cerebral

24h

20. Que forma tienen las lesiones observadas en TAC de un infarto establecido

Triangular

21. Se define como distonia a

Contracciones musculares sostenidas o repetidas con perfil involuntario q acompaña de movimientos


de retorcimiento y psoturas anormales

22. Con relación al Sx parkinsoniano, seleccione la resp correcta

La diminución del braceo es un signo de hipocinesia

23. Cual es el trastorno al qu hace referencia la siguiente definición:…estereoespecificada,


involuntaria aparentemente repetitiva rápida y breve q ….musculos individuales

Tics

24. El estado profundo similar al sueño del cual es imposible despertar a la persona es
Coma

25. En el estado en el que un pct ha despertado del coma y tiene aspecto despierto pero no responde,
se conoce como

Vegetativo

26. El Sx de hipomovilidad y mutismo q aparece como parte de psicosis intensa se llama

Catatonia

27. Cual de las siguientes estructuras encefálicas cuando presentan daño provocan coma?

El sistema de activación reticular

28. el consumo de oxigeno en el tej cerebral es

<115 mmol/l

29. La respiración de kusmaul representa la existencia de

Acidosis metabolica

30. Cual es la incidencia de epilepsia a nivel mundial

0.3-0.8%

31. El fenómeno de crisis parciales q empieza en partes distales de los miembros y q avanza en forma
gradual hasta casi la totalidad del miembro se conoce como

Marcha jacksoniana

32. Las crisis q se caracterizan por breves y repentinos lapsos de perdida de consciencia sin perdida de
postura se conoce como

Ausencias típicas

33. Las crisis convulsivas tónico clínicas de inicio generalizado son el tipo de crisis principal en
aproximación q porcentaje de personas sufren epilepsia

10%

34. Los traumatismos craneales penetrantes graves cuanto riesgo de desarrollar epilepsia tienen

45%

35. Las crisis convulsivas más frecuentes q aparecen al final de la lactancia y comienzo …

Crisis febriles

36. Cual es el riesgo de muerte de un pct con epilepsia

2-3 veces mas q población general

37. Cual es el mejor estudio para observar lesiones traumáticas

TAC

38. Q porcentaje de pcts con apoplejía isquémica quedan sin definir su causa

30%
39. Cual es la causa mas frecuente de embolia pulmonar

Fibrilación auricular no reumática

40. Cual es el riesgo de apoplejía después de una TIA en los primeros 3 meses

10-15%

41. Cual es la ubicación mas frecuente de la hemorragia hipertnsiva

Ganglios basales

42. Las malformaciones arteriovenosas son mas frecuentes en

Varones

43. La nagiopatia amilode es una enfermedad q aparece en

Adultos mayores

44. Cual es el estudio de imagen principal para infarto cerebral

RMS en perfusión difusión

45. La imagen típicamente asociada a la hemorragia intracerebral es

Lesión hiperdensa redondeada u oval dentro del parénquima cerebral

46. Sobre las enfermedades cerebrovasculares, seleccione la resp correcta

El ictus es sinónimo de ECV sin síntomas clínicos

47. Cual es la nomenclatura etiopatogenica de un infarto cerebral menos a 15mm q afecta territorio
vascular de vasos perforantes

Infarto cerebral lacunar

48. Con relación tto en fase aguda del ictus, seleccione lo incorrecto

Deben utilizarse la dextrosa al 5% u otras soluciones hipotónicas

49. Para un pct de 55ª con infarto cerebral y ateromatosis carotidea del 75% de la luz del vaso. Q
opción seria útil para evitar el tto de recurrencias

Antiplaquetarios mas estatinas

50. seleccione el factor de riesgo de mayor peso para el ictus

HTA

51. Seleccione la respuesta correcta. Las cefaleas trigeminales autonómicas se caracterizan por

Unilateral, con manifestaciones autonómicas hiperfocales

52. Ante un pct con cefalea y signos de alarma seleccione la respuesta correcta

Se requieren exámenes complementarios de forma urgente para el dx

53. Identifique lo q no es una manifestación clínica del Dx de hipertensión intracraneal

Rigidez nucal

54. Sobre el tto profiláctico de la migraña, seleccione la resp correcta


Deben identificarse factores desencadenantes de las crisis y controlarlos

55. Sobre el tto de la cefalea tipo tensión, seleccione la opción adecuada

Para la forma crónica puede indicarse amitriptilina

56. Con relación a los efectos adversos de los medicamentos q se utilizan en la profilaxis de la
migraña, seleccione lo correcto

El propanolol esta contraindicado en pcts con asma y EPOC

57. Sobre el tto de las cefaleas trigeminales autonómicas marque lo correcto

Los corticoides son de elección para el tto a mediano plazo de la cefalea en racimos

58. Las guias terapéuticas plantean el uso de AINES como primera línea para el manejo de la migraña.
De acuerdo al mecanismo de acción de este grupo farmacológico. Como se logra controlar este
cuadro?

Provoca q los eicosanoides sensibilicen las terminaciones nociceptoras

59. El manejo farmacológico de la cefalea plantea el uso de los AINES de inicio, en caso de
refractariedad del dolor se plantean otros grupos farmacológicos como

Agonistas serotoninergicos

60. Sobre las crisis epilépticas focales selecciones la resp adecauda

La crisis deja vu es de tipo pasiquica y se origina en el lóbulo tmeporal

61. Con relación al Dx diferencialy etiológico de la epilepsia, marque lo correcto

Los trastornos del mocimiento hipercineticos pueden confundir con crisis epilepticas motoras

62. En cuanto al tto de las crisis epilepticasm elija estas opciones

La lamotrigina debe titularse lentamente para evitar rash, q constituye sus complicación ams
frecuente

63. La epilepsia en la mujer tiene connotaciones especiales, seleccione la resp correcta

El efecto de las convulsiones es mas dañina para el feto q los FAEs

64. Que porcentaje de ECV isquémico hace hemiparesia

85%

65. La interrupción cmpleta de la circulación cerebral cauas la muerte del tejido encefálico

4-10 min

66. Respecto a las conductas de prevención en la diseminación de meningitis bacteriana usted


recomendaría

Uso de dexametasona previo a la administración del esquema antibiótico

67. Con relación a nomenclatura y clasificación de las crisis epilépticas seleccione lo correcto

Para el Dx de la epiplepsia se requiere al menos de una crisis epiléptica

68. Sobre las crisis epilépticas focales selecciones lo adecuado


Las crisis con postura asimétrica son características del lóbulo Fontal

69. En cuanto al tto de las crisis epilépticas elija la correcta

El acido valproico es útil para las ausencia y mioclonias

70. La epilepsia en la mujer es especial, elija lo correcto

El embrazo debe ser planificado, con un año de control de crisis, uso de Fas menos tetatogenicos y con
administración previoa de acido fólico

71. Los derivados ergoticos son una herramienta terapéutica en el manejpo sintomático de la crisis
migrañosa su acción se base en

Agonismo general a los alfa adrenorecptores

72. Con relación a la anatomia y sisiologia de las meninges y al LCR seleccione lo correto

El LCR se produce en los plexos corides de los ventrículos cebrales

73. Sobre el Sx de irritación meníngea selección lo adecuado

El Dx etiológico se basa sobre todo en el estudio del LCR

74. Elija la resp correcta sobre las características del LCR en meningitis micotica

Células en decenas o cientos a predominio de linfocitos proteínas muy elevadas, glucosa baja

75. Con relación a las complicaciones de las meningoencefalitis selección lo correcto

Las vasculitis pueden producir isquemia cerebral en el curso de una meningoencefalitis

76. Sobre el tto de la meningoencefalitis en el adulto seleccione la resp adecuada

El tto con Aciclovir es de utilidad en la meningoencefalitis herpética

77. La puncion lumbar tiene contraindicaciones. Deleccione la correcta

Trastorno de la coagulación

78. Si un pct con una hemiplejia aguda con duración mayor a 24h con signo de babinski postivo y no
se observa lesión en la TAC de cráneo significa q

Es un infarto cerebral en etapas iniciales

79. A un pct con infarto cerbeal le realizan EKG, EEG y ecodoppler de carótidas. Tiene hipertrofia
ventricular izq y placa de ateroma en inicio de carótida interna con estenosis del 70%. Dx?

Infarto cerebral aterotrombotico

80. Con relación al tto en fase aguda del ictus, selccione la resp correcta

Debe colocarse la cabecera del pct a 30°, hacer cambios de posición frecuentes e iniciatr fisioterapia
temprana para evitar complicaciones

81. Ante un pct con infarto cerebral tipo cardioembolico elija el tto de elección para prevención de las
recurrencias

Anticoagulantes orales
82. Q núcleo de la protuberancia esta relacionado con el control descendente nonciceptivo hacia el
trigémino

Locus cerúleo

83. Q neuropeptido se encuentra relacionado con la fisiopatología de la migraña

Péptido relacionado con el gen de la calcitonina

84. Pct de 30 A con dolor retroorbitario de gran intensidad quien refiere q las crisis de dolor se
producen casi todos los días mas o menos a la misma hora. Usted q cefalea pensaría

Cefalea en racimos

85. Q tipo de cefalea se caracteriza por presentar episodios de cefalea frecuentes, unilaterales,
intenso, de corta duración, epifora, congestión nasal y son retro orbitarios

Hemicránea paroxística

86. En SUNCT para el Dx cuantos ataques se requiere y en q tiempo

20 ataques de 5-240 seg

87. Cual es el tipo de cefalea q depende de la posición del pct en donde esta comienza cuando la
persona se sienta o se pone de pie erecta o cuando adopta el decúbito

Cefalea por disminución del volumen de LCR

88. Q tipo de cefalea se caracteriza por los trastornos en q el dolor y sus características se presentan
sin q exista una causa exógena

Cefalea Primaria

89. Con q nombre se le conoce a la inervación de grandes vasos intracraneales y la duramadre por
arte del trigémino

Sistema trigeminovascular

90. La cefalea en la migraña debe ser

Unilateral

……………………

 En relación a la ECV isquémica señale el enunciado flaso


La fiebre en la fase aguda de la enfermedad se debe evitar y de existir debe ser tratada con
antipiréticos o medios físicos

 En relación a la ECV isquémica señale lo flaso


Las causas cardioembolicas pueden producir hemorragia petequiales la cual es frecuente en la
parte distal de la carótida interna

 Delos siguientes enunciados señale cual es el falso


La oclusión total del tronco de la cerebral medio ocasiona hemiplejia, hemianestesia, hemianopsia
homónima contralaterales a la lesión

 En relación a las hemorragias cerebrales, señale enunciado falso


Luego del sangrado puede haber expansión del volumen de sangre

 En relación a la hemorragia cerebral señale el enunciado falso


Las hemorragias causadas por anticoagulacione xcesiva afecta al sistema ventriculatr y progresa
en 72h

 En relación a las convulsiones, señale lo falso


Las crisis focales o parciales se originan en las redes neuronales limitadas a un hemisferio cerebral
producidas usualmente por anormalidades estructurales cerebrales

 En relación a los siguientes enunciados, señale lo falso


El termino epileptogenesis corresponde a un estado de hiperexitabilidad producidad de forma
aguda por exesiva inhibición cortical

 En relación a la epilepsia señale lo falso


El fármaco de primera elección para epilpsia generalizada es la fenitoina

 En relación a los siguiente enunciados de ceflea señale el falso


La cefalea secundaria es aquella en la q el dolor y sis características constituyen el cuadro
intrínseco

 Dentro de los síntomas de cefalea q sugieren un trastorno grave o banderas rojas de dolor
constante, señale lo falso
Vomito q antecede a la cefalea

 En relación a migraña señale lo falso


Dentro de los criterios diagnósticos se señala dolor q dura 4-72h

 En relación a las polineuropatias señale lo falso


El Sx de guillan barre requiere un manejo rápido a través de dosis altar de corticoides parenterales

 En relación al Sx de Guillasn barre señale lo falso


En el estudio de LCR q apoya al Dx debe buscarse una hiperproteinorraquia con pleiocitosis

 En relación a las infecciones del SNC señale lo falso


La infección del espacio subaracnoideo se conoce como epiema

 en relación al tto de infecciones del SNC señale lo falso


la meningitis bacteriana aguda se debe a usar cefalosporinas de 3ra-4ta + Vancomicina

………………………………………

1. En la etiología neurológica a qué grupo poblacional afecta en mayor porcentaje la migraña? (c-
pag2586)

-a. Hombres
b. Mujeres
c. Ancianos
d. Niños
2. Las regiones del tronco encefálico que participan en la modulación descendente del dolor del
trigémino son: (cap447-pag2588)

a. Mesencefalo
1b. Núcleo del locus cerúleo en la protuberancia
c. Corteza
d. Hipotálamo
3. Cuál es el criterio diagnóstico para la SUNCT? (Cap447-pag2596)

a. Al menos 20 ataques con duración de 5 a 240 segundos


b. Al menos 10 ataques en una hora
c. Al menos 5 ataques en un día
d. Al menos 24 ataques en las últimas 24h
4. El dolor es profundo por lo general retroorbitario, a menudo de gran intensidad no fluctuante,
explosivo y se caracteriza por tener periodicidad (cap447-pag2596)

a. Cefalea tensional
b. Hemicranea paroxistica
c. Hemicranea continúa
d. Cefalea en racimo
Cuál de las siguientes Causas no son frecuentes de cefalea primaria. Cap 447

a) tensional
b) Migrañosa
c) Lesion craneoencefálica
d) Idiopática

6.La inervación de los gruesos vasos intracraneales y la duramadre por el nervio trigémino se conoce
como: Cap 447

a) sistema trigeminovascular.
b) Sistema microvascular
c) Sistema intersticial
d) Sistema conector

¿Cual es la causa mas frecuente de epilepsia en lactantes ( mayores a un mes y menores de 12


años)?(pagina 2546)
a.-tumores cerebrales
b.-apoplejia
c.-crisis febriles
d.-autoanticuerpos

¿cual es el método diagnostico mas efectivo para encontrar un foco epileptogeno ¿


a.- Electroencefalograma (pagina 2550)
b.- tomografía
c.-resonancia magnética
d.- angiografía

91. en una meningitis vírica cual es el valor de las proteínas


50-100

92. Cual es el germen mas frecuente en meningitis comunitaria


Strepto pneumoniae

93. Cual es la prueba de lab mas importasnte para el Dx de meningitis

Estudio de LCR

94. Al solicitar examen de LCR la principal característica q hace pensar en meningitis bacteriana es

Glucosa < 40mg /100 ml

95. Status epiléptico se considera cuando la crisis conulsiva dura mas de

5 min

96. El grito ictal es un signo característico de q tipo de crisis convulsivas

Crisis tonico clónicas generalizadas

97. Pct llega a emergencia de 45 A sin APP en crisis convulsiva según efiere familiar a presentado 3
crisis convulsivas dese hace 20 min, no ha existido recuperación de la conciencia desde la primera
crisis y ha criterio familiar la última ha durado como 5 minutos, Al examen físico se encuentra
inconsciente, cianótico con midriasis bilateral. Dx

Lesión ocupativa cebral que necesita TAC

98. La terapia trombolitica con rTPA tienen actual evidencia en q ventana

0-3h

99. Ingresa a la emergencia señor 35 A sin AAP con clínica de focalidad neurológica caracterizada por
hemiparsia Facio braquio crural izquierda además acompaña de cefalea intensa holocraneal el
cuadro fue súbito desde hace horas y los síntomas se han incrementado

Evento cereborvascular hemorrágico que necesita TAC

100. Las neuronas cuanto tiempo soportan sin oxigeno

2 minutos
~UTE
l. U I L.I FACULTAD
CIENCIAS DE LA SALUD
~ EUGENIO ESPEJO

FERNANDA MARISOL SANDOVAL FARINANGO

Área personal  Mis cursos  Quito  CIENCIAS DE LA SALUD  MEDICINA - PRESENCIAL  ABR 2020 - AGO 2020
 INVESTIGACION I-NEURO-TEORIA - Prl: MD NVA Pen: 961  General  INVESTIGACIÓN I

Comenzado el viernes, 26 de junio de 2020, 14:15


Estado Finalizado
Finalizado en viernes, 26 de junio de 2020, 14:40
Tiempo empleado 24 minutos 52 segundos
Cali cación 1,50 de 4,00 (38%)

Pregunta 1 Incorrecta Puntúa 0,00 sobre 0,25

En la hemorragia subaracnoidea por ruptura de un aneurisma cerebral:

Seleccione una:

a. Alteración del estado de conciencia es inusual al inicio del evento X


o b. Una punción lumbar está indicada cuando la neuroimagen es normal

o c. El vasoespasmo es una complicación frecuente dentro de los 3 primeros días del evento

o d. La ruptura de aneurismas de la circulación posterior es la causa más frecuente

La respuesta correcta es: Una punción lumbar está indicada cuando la neuroimagen es normal

/
Pregunta 2 Incorrecta Puntúa 0,00 sobre 0,25

La localización cortical es una característica de la hemorragia secundaria a:

Seleccione una:

o a. Fármacos simpaticomiméticos

o b. Activador del plasminógeno tisular

o c. Cocaina

@ d. Hipertensión arterial X

La respuesta correcta es: Activador del plasminógeno tisular

Pregunta 3 Incorrecta Puntúa 0,00 sobre 0,25

En el trabajo diagnóstico de un hombre de 55 años que presenta crisis parciales motoras con generalización
secundaria, además de una adecuada historia clínica es necesario:

Seleccione una:

@ a. Electroencefalograma con privación de sueño. Opcional tomografía cerebral contrastada X


o b. Resonancia magnética cerebral simple, contrastada y electroencefalograma con privación sueño

o c. Resonancia magnética cerebral simple, contrastada y electroencefalograma en vigilia

o d. Estudio del líquido cefalorraquídeo. Opcional electroencefalograma con privación de sueño

La respuesta correcta es: Resonancia magnética cerebral simple, contrastada y electroencefalograma con
privación sueño

/
Pregunta 4 Correcta Puntúa 0,25 sobre 0,25

Una característica en común de las ausencias y de las crisis parciales complejas es:

Seleccione una:

o a. Automatismos oromandibulares

o b. Electroencefalograma con punta onda a 3 Hz por segundo

@ c. Afectación de la conciencia

o d. Duración del evento

La respuesta correcta es: Afectación de la conciencia

Pregunta 5 Correcta Puntúa 0,25 sobre 0,25

De los criterios diagnósticos de la Sociedad Internacional de la Cefalea para el diagnóstico de migraña sin
aura, considera el más importante

Seleccione una:

o a. Que la cefalea debe ser siempre pulsátil y unilateral

o b. Dolor de intensidad moderada o severa

@ c. El número de episodios de la cefalea

o d. Los síntomas acompañantes como nausea y/o vómito o fotofobia y fonofobia

La respuesta correcta es: El número de episodios de la cefalea

/
Pregunta 6 Correcta Puntúa 0,25 sobre 0,25

Un signo clínico sugestivo de coma metabólico es:

Seleccione una:

o a. Patrón respiratorio normal

o b. Mirada desconjugada

o c. Postura de decorticación bilateral

@ d. Pupilas isocóricas y reactivas

La respuesta correcta es: Pupilas isocóricas y reactivas

Pregunta 7 Correcta Puntúa 0,25 sobre 0,25

El dato clínico más importante para considerar un evento cerebrovascular como hemorrágico más que
isquémico es:

Seleccione una:

o a. Hemiparesia de instauración súbita

@ b. Alteración temprana del estado de conciencia

o c. Edad mayor de 70 años

o d. Antecedente personal de hipertensión arterial

La respuesta correcta es: Alteración temprana del estado de conciencia

/
Pregunta 8 Correcta Puntúa 0,25 sobre 0,25

Una mujer de 40 años es trasladada inconsciente a la emergencia del hospital. En el examen clínico no abre
los ojos a estímulos dolorosos; emite palabras incompresibles y realiza una extensión y pronación de la
extremidad superior izquierda, con exión plantar izquierda. La puntuación en la escala de Glasgow sería:

Seleccione una:

o a. 5

o b. 4

o c. 7

@ d. 6

La respuesta correcta es: 6

Pregunta 9 Incorrecta Puntúa 0,00 sobre 0,25

En orden de mayor a menor importancia los factores de riesgo para enfermedad cerebrovascular
trombótica se consideran:

Seleccione una:

@ a. Hipertensión arterial, diabetes, obesidad, hipercolesterolemia X


o b. Diabetes, hipercolesterolemia, soplo carotídeo, poliglobulia

o c. Hipertensión arterial, edad, brilación auricular, sedentarismo

o d. Edad, prótesis valvular mecánica cardíaca, diabetes, tabaquismo

La respuesta correcta es: Edad, prótesis valvular mecánica cardíaca, diabetes, tabaquismo

/
Pregunta 10 Incorrecta Puntúa 0,00 sobre 0,25

Un hombre de 79 años, hipertenso y con brilación auricular crónica en tratamiento con betabloqueantes y
warfarina 2.5 mg diarios, presenta un infarto cerebral en el territorio carotídeo de 3.5 horas de evolución.
La valoración inicial objetiva: presión arterial 180/100 mmHg, NIHSS 18. Glucosa 300 mg/dL, plaquetas
150000, INR normal. TC cerebral sin evidencia de hemorragia cerebral.

El médico neurólogo de emergencia no considera oportuno la administración de activador del


plasminógeno tisular (rTPA) debido a:

Seleccione una:

o a. 3.5 horas de evolución del evento

o b. Presión arterial 188/100 mmHg

o c. Tratamiento con warfarina 2.5 mg diarios con INR normal

@ d. NIHSS 18 X

La respuesta correcta es: Tratamiento con warfarina 2.5 mg diarios con INR normal

Pregunta 11 Incorrecta Puntúa 0,00 sobre 0,25

Un criterio para el diagnóstico de neurocisticercosis parenquimatosa de nitiva es:

Seleccione una:

o a. Test inmunológico positivo en sangre o líquido cefalorraquídeo

o b. Quistes parenquimatosos en diferentes estadios de evolución

@ c. Una o más calci caciones en el parenquima cerebral X


o d. Quiste coloidal con test inmunológico positivo en liquido cefalorraquídeo

La respuesta correcta es: Quistes parenquimatosos en diferentes estadios de evolución

/
Pregunta 12 Incorrecta Puntúa 0,00 sobre 0,25

Una mujer de 28 años con historia de migraña desde los 10 años acude con cefalea holocraneal, pulsátil,
acompañada de nausea, vómito y fotofobia, de presentación diaria los últimos 15 días. Ha tomado
medicación para la migraña de venta libre, consiguiendo un alivio parcial. Al momento la cefalea es de 3/10,
se incrementa con la actividad física. El examen neurológico no muestra focalidad, no hay papiledema en el
examen de fondo de ojo. La TC cerebral simple es normal. Consideraría como posibilidad diagnóstica:

Seleccione una:

o a. Cefalea tensional episódica

o b. Migraña sin aura

@ c. Migraña crónica X
o d. Cefalea por abuso de medicación

La respuesta correcta es: Migraña sin aura

Pregunta 13 Correcta Puntúa 0,25 sobre 0,25

Un hombre de 66 años en tratamiento con anticoagulantes sufre un accidente de tránsito violento, lo que le
ocasiona una pérdida de la conciencia. La valoración neurológica inicial objetiva una herida en el cuero
cabelludo en la región fronto-temporal derecha, Glasgow 5 (O1V1M3) con postura de decorticación en las
extremidades izquierdas, anisocoria pupilar (derecha 5 mm no reactiva, izquierda 3 mm reactiva.

La primera hipótesis diagnóstica a considerar es:

Seleccione una:

o a. Laceración

@ b. Hematoma epidural

o c. Hematoma subdural

o d. Contusión

La respuesta correcta es: Hematoma epidural

/
Pregunta 14 Sin contestar Puntúa como 0,25

Una mujer de 58 años presenta una parálisis facial bilateral con tetraparesia arre éctica sin trastorno
sensitivo y di cultad para respirar. 7 días antes presentó varias deposiciones diarréicas. El coprocultivo fue
positivo para campylobacter jejuni. El líquido cefalorraquídeo con 10 células monocleadas por mm3. Las
velocidades de conducción motora muy alteradas en las 4 extremidades.

Recomendaría para su tratamiento:

Seleccione una:

o a. Esteroides 1 mg Kg diario por 30 días

o b. Inmunoglobulina 0.4 g/Kg por 5 días y esteroides 1 mg/Kg

o c. Plasmaféresis 5 sesiones

o d. Plasmaferesis 5 sesiones e inmunoglobulina 0.4 g/kg diario por 5 días

La respuesta correcta es: Plasmaféresis 5 sesiones

Pregunta 15 Incorrecta Puntúa 0,00 sobre 0,25

Con un test de apnea con hipotensión arterial o arritmia cardíaca y pCO2 mayor de 60 mmHg, una vez
desconectado del respirador se debe considerar:

Seleccione una:

o a. Solicitar un Doppler transcraneal

o b. Diagnóstico clínico de muerte cerebral

o c. Repetir el procedimiento después de 10 minutos

@ d. Solicitar una angiografía cerebral X

La respuesta correcta es: Diagnóstico clínico de muerte cerebral

/
Pregunta 16 Incorrecta Puntúa 0,00 sobre 0,25

El tratamiento pro láctico de elección para la neuralgia del trigémino clásica es:

Seleccione una:

o a. Opioides

o b. Agonistas de los receptores 5HT/1B-1D

@ c. Anti-in amatorios no esteroidales X


o d. Bloqueantes de los canales de sodio

La respuesta correcta es: Bloqueantes de los canales de sodio

◄ CEFALEA Ir a... V EPILEPSIA ►

/
¿Patrón electrocardiográfico?

Respuesta: S1 Q3T3 onda Q Negativo

Factores de riesgo de desarrollo de neumonía:

RP: Niño o ancianos (edad), comorbilidades (DM, HTA), Asinamiento, inmunosupresión,


institucionalización, demencia, exceso de peso.

Alteraciones básicas del asma:

RP: Obstrucción al flujo de la vía aérea, reversibilidad de la vía área, hiperactividad bronquial.

EPOC no lo que no es cierto:

RP: No hay componente inflamatorio contrario al asma

Tratamientos broncodilatadores del asma:

RP: Beta agonista, leucotrienos (teofilina), anticolinérgico.

Mujer 54 años encontrada desorientada, saturación 80% al ambiente, pupilas puntiformes, pH: 7,25,
pCO2: 60, pO2: 56 Na 136 Cl 100, Según la gasometría:

RP: (ojo leer) Acidosis respiratoria pura.


TB Cual no es la correcta:

RP: la primoinfección es cuando el paciente entra en contacto y desarrolla la infección.

TEP cual es LA INCORRECTA:

RP: Ecodopler es el mas sensibles: no es muy sensible (es la angiografía pulmonar eso la correcta
angio TAC)

Gasometría lo verdadero de

RP: Patrón invertido PCO2 aumentado (+ 45) pO2 disminuida (-60).

Masculino de 65 años disnea progresiva, con tos no productiva de 3 días de evolución. Signos vitales:
Pa 90/55 p 110, t 37,6 saturación 86%. Diagnostico probable. Estertores, aumento de la silueta
cardiaca, infiltrados.

RP: Edema de pulmón cardiogénico.

TEP 68años edad, neoplasia de páncreas más quimioterapia viene a consulta por dolor y edema de
miembros inferiores.

Rp: eco Doppler venoso. (trombosis venosa profunda).

Hemoptisis masiva:

Rp: de 400 a 600 ml

3 signos del EPOC:


Rp: Tos crónica expectoración productiva mucosa, disnea crónica.

Exploración funciona con EPOC son esperables todos los hallazgos menos a uno

Pp: Disminución de volúmenes pulmonares.

De lo siguiente cual es incorrecta: Germen que no produce tuberculosis pulmonar

RP: Mc Leprae.

Definición de ingreso NAC, Hospitalización

RP: más de 2 hospitalización.

Cual cuadro clínico del paciente con bronquitis crónica

RP: Soplador azules, caquéctico, cianótico (flaco azulado).

Mejor prueba de Asma

Espirometría.

Factores de riesgo neumonía nosocomial:

Rp: Re intubación, Aspiración de contenido gástrico.

Criterios de derrame pleural exudativo excepto:

Rp: LDH menor a 0,4.


Área personal 7 Mis cursos 7 Quito 7 CIENCIAS DE LA SALUD í MEDICINA - PRESENCIAL í OCT 2020 - FEB 2021
□ INVESTIGACION I-NEURO-TEORIA - Prl: MD NVA Pen: 961 Per:OCT 2020 - FEB 2021 Car:MEDICINA - PRESENCIAL [ 16 de
noviembre - 22 de noviembre LJ Prueba pre-requisitos 1 18 Nov

Comenzado el miércoles, 18 de noviembre de 2020, 20:10


Estado Finalizado
Finalizado en miércoles, 18 de noviembre de 2020, 20:15
Tiempo empleado 4 minutos 43 segundos
Puntos 3/4
Calificación 8 de 10 (75%)

Pregunta 1 Correcta Puntúa 1 sobre 1

En relación al cuerpo calloso y las crisis epilépticas, señale el enunciado VERDADERO:

Seleccione una:

a. Induce la propagación de un flujo excitatorio a áreas corticales más distales

b. Induce la iniciación de un flujo excitatorio a áreas corticales más distales

c. Induce la iniciación de un flujo excitatorio en áreas corticales contiguas

d. Induce la propagación de un flujo excitatorio en áreas corticales contiguas

Respuesta correcta

La respuesta correcta es: Induce la propagación de un flujo excitatorio a áreas corticales más distales

/
Pregunta 2 Incorrecta Puntúa 0 sobre 1

De las sustancias que intervienen en dolor, especialmente dolor a nivel craneal como en Migraña, señale que sustancia NO
interviene:

Seleccione una:

a. Serotonina

b. Acetilcolina

c. Péptido del gen relacionado con la calcitonina


X
d. Dopamina

Respuesta incorrecta.

La respuesta correcta es: Acetilcolina

Pregunta 3 Correcta Puntúa 1 sobre 1

En relación a las estructuras y compartimentos cerebrales, señale el enunciado FALSO:

Seleccione una:

a. El falx separa los 2 hemisferios cerebrales

b. Existe la región supratentorial y la infratentorial

c. El agujero occipital separa la región supra de la infratentorial

d. El tentorio se localiza por encima del cerebelo

Respuesta correcta

La respuesta correcta es: El agujero occipital separa la región supra de la infratentorial

/
Pregunta 4 Correcta Puntúa 1 sobre 1

Dentro de los mecanismos de producción de crisis, señale el enunciado FALSO:

Seleccione una:

a. Generación de potenciales de acción repetitivos

b. Despolarización de la membrana neuronal

c. Bloqueo del influjo de Calcio y K intracelulares

d. Apertura de los canales de sodio voltaje dependientes

Respuesta correcta

La respuesta correcta es: Bloqueo del influjo de Calcio y K intracelulares

◄ Foro 1 Ir a... V Examen 1 21 Nov ►

/
Área personal J Mis cursos J Quito J CIENCIAS DE LA SALUD [ MEDICINA - PRESENCIAL [ OCT 2020 - FEB 2021
LJ INVESTIGACION I-NEURO-TEORIA - Prl: MD NVA Pen: 961 Per:OCT 2020 - FEB 2021 Car:MEDICINA - PRESENCIAL L 23 de

noviembre - 29 de noviembre LJ Prueba pre-requisitos 2 24 Nov

Comenzado el martes, 24 de noviembre de 2020, 20:10


Estado Finalizado
Finalizado en martes, 24 de noviembre de 2020, 20:25
Tiempo empleado 15 minutos 17 segundos
Puntos 5,00/5,00
Calificación 10,00 de 10,00 (100%)

Pregunta 1 Correcta Puntúa 1,00 sobre 1,00

En relación a la irrigación del área sensitiva primaria o postrolándica, señale el verdadero:

Seleccione una:

a. Arteria lenticuloestriada

b. Arteria cerebral posterior

c. Arteria paramediana

d. Arteria cerebral media

Respuesta correcta

La respuesta correcta es: Arteria cerebral media

/
Pregunta 2 Correcta Puntúa 1,00 sobre 1,00

La estructura a través de la cual se comunica el sistema ventricular con el espacio subaracnoideo se conoce como:

Seleccione una:

a. Agujero de Magendie

b. Agujero de Paccionni

1 /
c. Agujero de Silvio

d. Agujero de Monro

Respuesta correcta

La respuesta correcta es: Agujero de Magendie

Pregunta 3 Correcta Puntúa 1,00 sobre 1,00

Las ramas de la arteria carótida interna son, señale el EXCEPTO:

Seleccione una:

a. Arteria cerebral media

b. Arteria cerebral posterior

c. Arteria comunicante posterior

d. Arteria cerebral anterior

Respuesta
1 \ correcta

La respuesta correcta es: Arteria cerebral posterior

/
Pregunta 4 Correcta Puntúa 1,00 sobre 1,00

En relación a la barrera hemato-encefálica (BHE), señale el enunciado FALSO:

Seleccione una:

a. Separa la sangre del líquido intracelular encefálico

'
b. Contiene moléculas transportadoras específicas por ej para hormonas

c. En ciertas zonas como el área postrema no hay BHE

d. Es muy permeable al agua, es liposoluble

Respuesta correcta

La respuesta correcta es: Separa la sangre del líquido intracelular encefálico

Pregunta 5 Correcta Puntúa 1,00 sobre 1,00

En relación a los territorios de irrigación a nivel del cerebro, señale el enunciado FALSO:

Seleccione una:

a. El área motora primaria y suplementaria recibe irrigación de la arteria cerebral media

b. La corteza visual en el surco calcarino recibe irrigación de la arteria coroidea posterior

c. La capsula interna recibe irrigación de la arteria cerebral media-ramas penetrantes

d. El uncus del hipocampo recibe irrigación de la arteria coroidea anterior


'
Respuesta correcta

La respuesta correcta es: La corteza visual en el surco calcarino recibe irrigación de la arteria coroidea posterior

◄ Foro 2
<

Ir a...

Tarea Análisis de caso clínico 25 Nov ►

/
1
Charlie Rivadeneira Mieles

Área personal  Mis cursos  Quito  CIENCIAS DE LA SALUD  MEDICINA - PRESENCIAL  ABR 2020 - AGO 2020
 INVESTIGACIÓN I - Prl: MD NVD Pen: 961  22 de agosto Dr Carlos Romero  Examen nal 2

Comenzado el sábado, 22 de agosto de 2020, 09:15


Estado Finalizado
Finalizado en sábado, 22 de agosto de 2020, 10:24
Tiempo empleado 1 hora 9 minutos
Cali cación 2,94 de 4,67 (63%)

Pregunta 1 Incorrecta Puntúa 0,00 sobre 0,13

¿Cuál de las siguientes a rmaciones es causa potencial de IRA de origen isquémico?

Seleccione una:

@ a. Mayor vasodilatación glomerular en respuesta al óxido nítrico X


o b. Menor vasoconstricción glomerular en respuesta a la concentración elevada de endotelina

o c. Mayor adherencia leucocítica en el glomérulo

o d. Apoptosis y necrosis del epitelio ureteral

La respuesta correcta es: Mayor adherencia leucocítica en el glomérulo

/
Pregunta 2 Correcta Puntúa 0,13 sobre 0,13

Cuál de los siguientes agentes es el causante de erisipela y celulitis:

Seleccione una:

o a. Cándida

(j) b. Estreptococo del grupo A

o c. Esta lococo Aureus

o d. Estreptococo del grupo B

La respuesta correcta es: Estreptococo del grupo A

Pregunta 3 Incorrecta Puntúa 0,00 sobre 0,13

¿Cuál de los siguientes casos con IRA son exhibirán hidronefrosis bilateral en ecografía renal?

Seleccione una:

o a. Varón de 19 años de edad con púrpura fulminante por septicemia gonocócica

(j) b. Mujer de 85 años de edad que vive en un asilo de ancianos y padece pielonefritis y septicemia X
o c. Mujer de 37 años de edad sometida a quimioterapia y radioterapia por cáncer cervicouterino
avanzado

o d. Varón de 48 años de edad con insu ciencia renal crónica por hipertensión que recibió medio de
contraste yodado para una angiografía abdominal

La respuesta correcta es: Mujer de 37 años de edad sometida a quimioterapia y radioterapia por cáncer
cervicouterino avanzado

/
Pregunta 4 Incorrecta Puntúa 0,00 sobre 0,13

¿Cuál de los siguientes es un biomarcador de función glomerular pronostica el comienzo de IRA después
de una crisis de isquemia o hipotensión?

Seleccione una:

o a. Interleucina 18

o b. Nitrógeno ureico sanguíneo

o c. Molécula 1 de lesión renal (KIM-1)

(j) d. Lipocalina relacionada con gelatinasa neutrofílica (NGAL) X

La respuesta correcta es: Nitrógeno ureico sanguíneo

Pregunta 5 Correcta Puntúa 0,13 sobre 0,13

Cuál es la meta de HbA1c en un paciente de 60 años sin complicaciones y comorbilidades graves

Seleccione una:

o a. HbA1c > 7%

o b. HbA1c 7.5-8%

o c. HbA1c 7-7.5%

(j) d. HbA1c < 7%.

La respuesta correcta es: HbA1c < 7%.

/
Pregunta 6 Correcta Puntúa 0,13 sobre 0,13

¿Cuál fármaco podría descompensar a un paciente con diabetes?

Seleccione una:

@ a. Prednisona

o b. Atorvastatina

o c. Insulina

o d. Gem brozilo

La respuesta correcta es: Prednisona

Pregunta 7 Correcta Puntúa 0,13 sobre 0,13

Cuál NO es un factor de riesgo de insu ciencia renal aguda posoperatoria:

Seleccione una:

@ a. Género femenino

o b. Hipotensión transoperatoria

o c. Diabetes mellitus

o d. Cirugía cardíaca con derivación cardiopulmonar

La respuesta correcta es: Género femenino

/
Pregunta 8 Correcta Puntúa 0,13 sobre 0,13

Dentro del diagnóstico etiológico en el hipotiroidismo primario las hormonas tiroideas se van a
encontrar:

Seleccione una:

o a. FT4 y TSH normales

o b. Únicamente FT4 baja

@ c. FT4 baja y TSH alta.

o d. Únicamente TSH alta

La respuesta correcta es: FT4 baja y TSH alta.

Pregunta 9 Correcta Puntúa 0,13 sobre 0,13

El grupo sanguíneo O + es el grupo sanguíneo más frecuente. ¿Cuál es el menos frecuente?

Seleccione una:

o a. Grupo B+

o b. Grupo B+

o c. Grupo AB+

@ d. Grupo AB-

La respuesta correcta es: Grupo AB-

/
Pregunta 10 Incorrecta Puntúa 0,00 sobre 0,13

El mejor tratamiento ara el estómago en sandia (hipertensión Portal) es:

Seleccione una:

@ a. Antrectomía X
o b. Fármacos que promuevan la motilidad.

o c. Gastrectomía total.

o d. Agentes para reducir el acido

La respuesta correcta es: Agentes para reducir el acido

Pregunta 11 Correcta Puntúa 0,13 sobre 0,13

El tratamiento estándar de un tumor estromal gastrointestinal del estómago es:

Seleccione una:

o a. Gastrectomía total.

o b. Gastrectomía subtotal.

o c. Ablación endoscópica.

@ d. Resección en cuña con márgenes limpios.

La respuesta correcta es: Resección en cuña con márgenes limpios.

/
Pregunta 12 Incorrecta Puntúa 0,00 sobre 0,13

El tratamiento para el síndrome de vaciamiento rápido temprano grave después de gastrectomía es:

Seleccione una:

@ a. Conversión quirúrgica a drenaje tipo Roux en Y. X


o b. Tratamiento expectante.

o c. Octreotido

o d. Glucosa oral para los síntomas

La respuesta correcta es: Octreotido

Pregunta 13 Correcta Puntúa 0,13 sobre 0,13

En cuanto al tratamiento de la dermatitis atópica todo es correcto Excepto

Seleccione una:

o a. Algunos medicamentos liberadores de histamina (ácido acetil salicílico, codeína) o vasodilatadores


pueden agravar el prurito

b. Es muy importante aclarar que la dermatitis atópica es un tipo de alergia y que es fundamental
realizar pruebas alérgicas

o c. Antes de iniciar el tratamiento es esencial explicar con detalle a la familia del niño qué son la atopia
y la dermatitis atópica, además de su pronóstico.

o d. Algunos alimentos ricos en histamina, en cantidades considerables y en niños muy sensibles,


pueden agravar el prurito.

La respuesta correcta es: Es muy importante aclarar que la dermatitis atópica es un tipo de alergia y que es
fundamental realizar pruebas alérgicas

/
Pregunta 14 Correcta Puntúa 0,13 sobre 0,13

Entre las siguientes a rmaciones elija la correcta para el fenómeno isomór co de Köebner:

Seleccione una:

o a. Presión tangencial sobre la piel que provoca un despegamiento cutáneo.

o b. Disrupción dermoepidérmica traumática que produce una pústula en el lugar agredido, reproduce
lesiones propias de la dermatosis.

o c. Aparición de un habón tras el rascado de una lesión, debido a la degranulación mastocitaria.

@ d. Es la aparición de lesiones propias de una dermatosis en zonas de presión o traumatizadas. Típico


de enfermedades eritematodescamativas

La respuesta correcta es: Es la aparición de lesiones propias de una dermatosis en zonas de presión o
traumatizadas. Típico de enfermedades eritematodescamativas

Pregunta 15 Correcta Puntúa 0,13 sobre 0,13

En un alto porcentaje 80% los anticoagulantes son utilizados en el área cardiovascular. Los problemas
coronarios que obedecen a trombos arteriales. ¿Qué medicamento resulta bene cioso para su
tratamiento?

Seleccione una:

o a. Anticoagulantes

@ b. Antiagregantes

o c. Diuréticos

o d. Digitálico

La respuesta correcta es: Antiagregantes

/
Pregunta 16 Incorrecta Puntúa 0,00 sobre 0,13

Es característico la rigidez matutina en las articulaciones de pacientes con enfermedades inmunes


sistémica crónica. ¿Cuál patología lo presenta?

Seleccione una:

o a. Síndrome antifosfolipídico SAFL

o b. Fiebre Reumática

o c. Lupus eritematoso sistémico

(j) d. Artritis reumatoide X

La respuesta correcta es: Lupus eritematoso sistémico

Pregunta 17 Correcta Puntúa 0,13 sobre 0,13

Escoja la opción correcta

Seleccione una:

o a. La diabetes insípida es producida por aumento de la hormona vasopresina

o b. La acromegalia es producida por tumores hipo siarios o por tumores primarios no ectópicos
localizados en el pulmón y el páncreas.

o c. Las causas básicas a largo plazo de la menopausia son demencia, osteoartritis, trastornos del
equilibrio, aumento de la masa ósea.

(j) d. Entre las causas de diabetes insípida de origen central son el trauma cráneo encefálico, procesos
isquémicos cerebrales, procesos infecciosos que afectan a la hiposisis e igualmente procesos
in ltrativos como sarcoidosis e histiocitosis

La respuesta correcta es: Entre las causas de diabetes insípida de origen central son el trauma cráneo
encefálico, procesos isquémicos cerebrales, procesos infecciosos que afectan a la hiposisis e igualmente
procesos in ltrativos como sarcoidosis e histiocitosis

/
Pregunta 18 Correcta Puntúa 0,13 sobre 0,13

Escoja la opción correcta

Seleccione una:

a. los factores de riesgo de cáncer de la glándula tiroides son:exposición a radiación ionizante


,carcinoma familiar de tiroides,síndromes genéticos como poliposis familiar,síndrome de Werner

o b. el hiperparatiroidismo como síntomas produce litiasis renal,puede provocar hiporexia, nausea y


vomitos ,nunca produce ulcera gástrica y peor aún pancreatitis .

o c. la diabetes mellitus se diagnostica por un valor de hemoglobina glicosilada mayor a 6 , glicemia


menor a 199 mg en la curva d e tolerancia a la glicemia.

o d. para el tratamiento del hiperparatiroidismo usamos cinacalcet a dosis de 100 mg vo cada 12 horas
y bifosfonatos puede ser usada pero no siempre se necesita esta .

La respuesta correcta es: los factores de riesgo de cáncer de la glándula tiroides son:exposición a radiación
ionizante ,carcinoma familiar de tiroides,síndromes genéticos como poliposis familiar,síndrome de Werner

Pregunta 19 Correcta Puntúa 0,13 sobre 0,13

Escoja la respuesta correcta

Seleccione una:

a. la hipocalcemia crónica puede desarrollar cataratas subcapsulares .uñas quebradizas pelo áspero.

o b. la hipocalcemia en el electrocardiograma puede dar cambios como alargamiento del PR.QRS


levemente acortado,prolongación del QT

o c. otros trastornos de la hipocalcemia en el electrocardiograma son encontrar ondas T invertidas y


acortamiento del QT.

o d. la hipocalcemia aguda produce espasmos en los miembros, hay espasmos


carpopedal,laringoespasmo pero nunca hay tetania

La respuesta correcta es: la hipocalcemia crónica puede desarrollar cataratas subcapsulares .uñas
quebradizas pelo áspero.

/
Pregunta 20 Incorrecta Puntúa 0,00 sobre 0,13

La dermatitis atópica es:

Seleccione una:

o a. A y C son correctas

o b. Su prevalencia es mayor en mujeres

@ c. Una condición hereditaria que predispone a padecer una serie de enfermedades X


o d. En su siopatología presenta alteraciones de la inmunidad celular y de la reactividad vascular.

La respuesta correcta es: A y C son correctas

Pregunta 21 Correcta Puntúa 0,13 sobre 0,13

La hemoglobina glicosilada es un tipo de hemoglobina que tiene la importancia de mantener un buen


control del paciente con diabetes. ¿A qué valores se debe mantener para un buen control?

Seleccione una:

o a. A1c 9%

o b. A1c > de 7%

o c. A1c 8%

@ d. A1c < de 7%

La respuesta correcta es: A1c < de 7%

/
Pregunta 22 Incorrecta Puntúa 0,00 sobre 0,13

La prueba diagnóstica más exacta para establecer el síndrome de Zollinger-Ellison es:

Seleccione una:

@ a. Prueba de estimulación con secretina X


o b. Gastrina sérica en ayuno

o c. Endoscopia

o d. Tomografía por computadora

La respuesta correcta es: Gastrina sérica en ayuno

Pregunta 23 Correcta Puntúa 0,13 sobre 0,13

Lo más frecuente es que la lesión de Dieulafoy derive en:

Seleccione una:

o a. Progresión a cáncer gástrico.

@ b. Hemorragia gastrointestinal superior

o c. Gastroparesia

o d. Obstrucción de la salida gástrica.

La respuesta correcta es: Hemorragia gastrointestinal superior

/
Pregunta 24 Correcta Puntúa 0,13 sobre 0,13

Mecanismo de acción de la metformina:

Seleccione una:

o a. Prolonga la vida del GLP-1

(j) b. Disminuye la producción hepática de glucosa

o c. Apertura de los canales de K en la célula B

o d. Inhibe la dipeptidil peptidasa IV

La respuesta correcta es: Disminuye la producción hepática de glucosa

Pregunta 25 Incorrecta Puntúa 0,00 sobre 0,13

Paciente adolescente con obesidad y glicemia al azar de 300mg/dl. ¿Cuál es su orientación diagnóstica?

Seleccione una:

o a. DM LADA

@ b. DM MODY X
o c. DM-2

o d. DM- 1

La respuesta correcta es: DM-2

/
Pregunta 26 Correcta Puntúa 0,13 sobre 0,13

Paciente de 10 años normopeso, clínica cardinal de diabetes y glicemia al azar de 300mg/dl. ¿Cuál es su
orientación diagnóstica?

Seleccione una:

o a. DM-2

o b. DM MODY

o c. DM LADA

(j) d. DM- 1

La respuesta correcta es: DM- 1

Pregunta 27 Incorrecta Puntúa 0,00 sobre 0,13

Paciente de 24 años quien presentó clínica caracterizada por irregularidades menstruales, aumento de
peso, así como acné en cara, espalda y aparición de estrías en piel.Peso: 70 Kg, Talla: 1,62 m, IMC: 26,7
Kg/m2, TA: 130/85 mmHg, Pulso: 70 ppm. Distribución centrípeta de la grasa corporal, cara de luna llena
y acumulo de grasa en el dorso del cuello. Piel: acné a nivel de cara y espalda, vello no en cara, estrías
violáceas en abdomen y brazos, no se observó iperpigmentación ni signos de fragilidad capilar. Glándula
tiroides no palpable. La auscultación cardiopulmonar fue normal; a nivel de abdomen, aumento del
panículo adiposo, sin visceromegalia. Extremidades con masa muscular conservada y debilidad
proximal. Escoja la opción correcta

Seleccione una:

o a. clínicamente impresiona un síndrome de Cushing y la causa más probable es un tumor hipo sario
no secretor de ACTH.

o b. ninguna de las opciones son correctas

(j) c. clínicamente impresiona como un síndrome de Cushing y para su con rmación solicitaríamos
cortisol plasmático el mismo que debería estar bajo, y solicitar RMN de silla turca porque la causa mas
probable es un tumor hipo sario. X
o d. clínicamente impresiona como una enfermedad de Cushing y podría estar relacionada con la
administración exógena de glucocorticoides .

La respuesta correcta es: clínicamente impresiona como una enfermedad de Cushing y podría estar
relacionada con la administración exógena de glucocorticoides .

/
Pregunta 28 Correcta Puntúa 0,13 sobre 0,13

Para transfundir concentrado globular a un niño con anemia que pesa 20 kg. ¿Cuánto le tocaría a la
dosis que le corresponde?

Seleccione una:

o a. Concentrado globular 400 cc

o b. Concentrado globular 300 cc (1 unidad)

@ c. Concentrado globular 200 cc

o d. Concentrado globular 500 cc

La respuesta correcta es: Concentrado globular 200 cc

Pregunta 29 Correcta Puntúa 0,13 sobre 0,13

Señale la respuesta incorrecta en cuanto a la clínica de la dermatitis atópica

Seleccione una:

o a. Las manifestaciones clínicas típicas de la dermatitis atópica se dividen en tres etapas, que suelen
denominarse del lactante, infantil y del adulto.

@ b. No se asocia a otras enfermedades cutáneas y extracutáneas

o c. Puede presentar manifestaciones atípicas

o d. La xerosis es una manifestación constante dentro del cuadro

La respuesta correcta es: No se asocia a otras enfermedades cutáneas y extracutáneas

/
Pregunta 30 Correcta Puntúa 0,13 sobre 0,13

Una de las siguientes a rmaciones es verdadera, respecto a la infección por c. di cile.

Seleccione una:

o a. Se debe esperar el cultivo mas antibiograma antes de iniciar tratamiento.

o b. Aproximadamente un 70% sufren una recaída al nalizar el tratamiento.

@ c. Ante un caso de colitis fulminante, si pasados 3 días de tratamiento antibiótico no hay respuesta,
está indicado el tratamiento quirúrgico.

o d. La vía de contagio es oral-oral.

La respuesta correcta es: Ante un caso de colitis fulminante, si pasados 3 días de tratamiento antibiótico no
hay respuesta, está indicado el tratamiento quirúrgico.

Pregunta 31 Incorrecta Puntúa 0,00 sobre 0,13

Una mujer con 65 años de edad con ulcera duodenal conocida se trata con dieta y un bloqueador H2.
Ingresa al hospital con hemorragia gastrointestinal superior de con compromiso hemodinámico. Junto a
iniciar la reposición sanguínea, el próximo paso en el tratamiento sería

Seleccione una:

o a. Endoscopia y coagulación del vaso sangrante

o b. Iniciar omeprazol

@ c. Iniciar bismuto, tetraciclina y metronidazol X


o d. Piloroduodenotomía y cierre del vaso sangrante con sutura.

La respuesta correcta es: Iniciar omeprazol

/
Pregunta 32 Correcta Puntúa 0,13 sobre 0,13

Una mujer de 25 años re ere una historia de 12 meses de cólicos abdominales recurrentes en hemi-
abdomen inferior acompañado de distensión abdominal y diarreas. No presenta ebre, pérdida de peso
ni anorexia ni deposición con sangre. Este paciente cumple con los criterios de Roma IV para Sd de colon
irritable con predominio de diarrea, no presenta signos de alarma. En que otra patología usted puede
pensar?.

Seleccione una:

o a. Hipertiroidismo.

o b. Helicobacter pylori.

@ c. Enfermedad celiaca.

o d. Enfermedad in amatoria intestinal.

La respuesta correcta es: Enfermedad celiaca.

Pregunta 33 Incorrecta Puntúa 0,00 sobre 0,13

Un hombre de 62 años con cirrosis secundaria a hemocromatosis con várices grado III y con signos rojos
en la endoscopía. El es un diabético insulino-dependiente que ha presentado algunos episodios de
hipoglicemia. Su escala de Child es de 5. La terapia de primera línea para pro laxis primaria de
hemorragia varicial en este paciente puede ser?.

Seleccione una:

o a. Mononitrato de isossorbide.

o b. Ligadura varicial endoscópica.

o c. Shunt transyugular intrahepático portosistémico ( TIPS).

@ d. Betabloqueadores no selectivos. X

La respuesta correcta es: Ligadura varicial endoscópica.

/
Pregunta 34 Incorrecta Puntúa 0,00 sobre 0,13

Un varón de 50 años con alcoholismo y cirrosis con Child A es visto en emergencia por sangrado
gastrointestinal agudo. Hace 1 año tuvo sangrado por várices esofágicas y fue sometido a ligadura con
bandas en forma emergente. El ha venido utilizando betabloqueadores y lo ha hecho en forma
obediente. El paciente ahora se presenta con sangrado gastrointestinal recurrente y actual. Cuál de los
siguientes es el mejor tratamiento para este paciente antes de la endoscopía?.

Seleccione una:

o a. Octreotide y cefriaxone intravenosos.

@ b. Colocar TIPS emergentes. X


o c. Trasfundir hasta conseguir una hemoglobina alrededor de 10 g/dl.

o d. Octreotide y nor oxacina oral.

La respuesta correcta es: Octreotide y cefriaxone intravenosos.

Pregunta 35 Correcta Puntúa 0,13 sobre 0,13

Varón de 45 años con azotemia prerrenal secundaria a deshidratación, con una creatinina de 2.1
mg/100 ml. Se queja de lumbalgia leve y el médico le receta naproxeno en forma intermitente. ¿Cuál es
el mecanismo por medio del cual este fármaco podría deteriorar aún más su función renal?

Seleccione una:

o a. Efectos adversos en el túbulo proximal

@ b. Vasoconstricción arteriolar aferente

o c. Vasoconstricción arteriolar eferente

o d. Vasodilatación arteriolar aferente

La respuesta correcta es: Vasoconstricción arteriolar aferente

◄ Link de sesión examen nal 2 Ir a... V

/
Pregunta 1 Correcta Puntúa1,00sobre1,00 f" M.irc.ir pregunt.i

SEÑALE LO CORRECTO DE LA DERMATITIS ATOPICA

seleccione uno:

a. NO ESTÁ INFLUIDA POR FACTORES AMBIENTALES

b. SE ASOCIA CON ANTECEDENTES FAMILIARES DE DERMATITIS ATÓPICA, RINITIS ALÉRGICA Y ASMA .

c. SIEMPRE HAY ANTECEDENTES PERSONALES DE INFECCIONES GASTROINTESTINALES


d. ES MÁS COMÚN EN MUJERES

Respuesta correcta

Ll respuesta correcta es: SE ASOCIA CON ANTECEDENTES FAMILIARES DE DERMATIT1S ATÓPICA, RINITIS ALÉRGICA Y ASMA

Pregunta 2 Incorrecta Puntua 0,00 sobre 1,00 f" Ma rcar pregunta

MODELO DE 27 AÑOS DE EDAD QUE PRESENTA PLACAS ERITEMATO ESCAMOSAS EN ZONA DEL ARETE Y POR DEBAJO DEL
OMBLIGO, ESTA ÚLTIMA EN FORMA CIRCULAR DE MONEDA. ¿CUÁL ES SU DIAGNÓSTICO?

Seleccione una:

a. DERMATITIS ALERGICA DE CONTACTO

b. DERMATITIS O ECZEMA NUMULAR X

c. TIÑA CORPORIS
d . PSORIASlS

Respuesta incorrecta.

Ll respuesta correcta es: DERMATms AlERGICA DE CONTACTO

Pregunta 3 Correcta Punrúa 1,00sobre 1,00 f" Marcar pregunta

LAS LESIONES DE LA DERMAT1T1S ATÓPICA EN EL LACTANTE. SEÑALE LO CORRECTO

seleccione una:

a. SON MÁS HABITUALES ES EN LA CARA. NO RESPETAN LAS ZONAS ALREDEDOR DE LOS OJOS, LA NARIZ Y LA BOCA.

b. LA DERMATITIS ATÓPICA NO SE PRESENTA EN LACTANTES

c. SON MÁS HABITUALES ES EN EL CUERPO, DE PREDOMINIO EN ZONAS EXTENSORAS COMO CODOS Y RODILLAS
d . SON MÁS HABITUALES ES EN LA CARA. RESPETAN LAS ZONAS ALREDEDOR DE LOS OJOS, LA NARIZ Y LA BOCA

Respuesto correcta

Ll respuesta correcto es: SON MÁS HABITUALES ES EN tA CARA. RESPETAN LAS ZONAS ALREDEDOR DE LOS OJOS, tA NARIZ Y LA BOCA

Preg unta 4 Correcta Puntúa 1,00sobre 1,00 f" Marcar pregu n ta

MODELO DE 27 AÑOS DE EDAD QUE PRESENTA PLACAS ERITEMATO ESCAMOSAS EN ZONA DEL ARETE Y POR DEBAJO DEL
OMBLIGO, ESTA ÚLTIMA EN FORMA CIRCULAR DE MON: DA. EL ALERGÉNO MÁS PROBABLE ES:

Seleccione una:

a. NIQUELADOS Y CROMADOS

b. LATEX

c. FRAGANCIAS
d. FORMALDEHIDO

Respuesta correcta

La respuesta correcta es: NIQUELADOS y CROMAOOS

Pregu nta 5 Incorrecta Puntua 0,00 sobre 1,00 f" M.irur pregunta

Señale la respuesta incorrecta

seleccione una:

a. El cuadro eritrodérmico se presenta con un enrojecimiento generalizado de la piel.

b. La eritrodermia se acompaña de prurito. exudación y descamación

c. La Eritrodermia es la forma más grave de la dermatitis atópica X


d. En un cuadro eritrodérmico es raro encont rar manifestaciones generales, como fiebre. escalofríos, trastornos digestivos y
adenopatías

Respuesta incorrecta.

Ll respuesta correcta es: En un cuodro eritrodérmieo es raro encontrar marnfestaciones generales, como fiebre, escaloh-íos, trastornos
digestivos y adenopatías
Pregunta 6 lnco,ncta P,mtúa 0,00 sobre 1,00 f' Mucu p regunt.a

SEÑALE LO FALSO DE LAS ENFERMEDADES BACTERIANAS DE LA PIEL

Seleccione una:

a. EN EL IMPÉTIGO HAY RIESGO DE DESARROLLAR PIELONEFRITIS POS ESTREPTOCÓCICA

b. EL ECTIMA SE CARACTERIZA POR ULCERA PROFUNDA DOLOROSA X

c. EN EL IMPÉTIGO SE ACOMPAÑA SIEMPRE DE ADENOPATiAS REGIONALES


d. LA UNFANGITIS SE CARACTERIZA POR TRAYECTOS LINEALES ERITEMATOSOS

Respuesta incorrecta.

La respuesta correcta es: EN El IMPÉTIGO SE ACOMPAÑA SIEMPRE DE ADENOPATÍAS REGIONALES

Pregunta 7 Correcta PuntÚ!I 1,00sobre 1,00 f' M.arc.ar pregunt.a

¿EN QUE EDAD PREDOMINA EL IMPÉTIGO?

Seleccione una:

a . LACTANCIA

b. INFANCIA

c. MADUREZ
d .JUVENTUD

Respuesta correcta

La respuesta correcta es: INFANCIA.

Pregunta 8 Correcta PuntW 1,00sobre 1,00 f' M.arc.ar pregunt.a

SEÑALE LO CORRECTO DEL AC\IE

Seleccione una:

EL ACNE SE CONTAGIA POR MANIPULACIÓN DE LAS LESIONES

b. EL TRATAMIENTO INCLLYE QUERATOLincos YANTIBIÓT1COS TÓPICOS

EL ACNE SE CARACTERIZA POR UNA PIEL SECA POR DISMINUCIÓN DEL SEBO

d. LOS CORTICOIDES VlA ORAL SE USAN EN TODOS LOS CASOS

Respuesta correcta

la respuesta correcta es: EL TRA~AMIENTO INCLUYE QUERATOlÍTICOS y ANTIBIÓTICOS TÓPICOS

Pregunta 9 Correcta PuntW 1,00sobre1,00 f' M.arc.ar pregunt.a

SEÑALE LO VERDADERO SOBRE LA DERMATITIS DE CONTACTO JRRITATIVA

Seleccione una:

a . DENTRO DEL TRATAMIENTO NO SE DEBERÍA EVlTAR LA REEXPOSICIÓN

b . NO ES LA FORMA MÁS COMÚN DE ENFERMEDAD CUTÁNEA OCUPACIONAL

c. LOS PIES SON A MENUDO LOS MÁS AFECTADOS


d. SON IRRITANTES:JABONES, DETERGENTES, ÁCIDOS, PLANTAS

Respuesta correcta

la respuesta correcta es: SON IRRITANTES: JABONES. DETERGENTES, ÁCIDOS. PlANTAS


Pregunta 10 lnconewi Puntúa 0,00 sobre 1,00 'f" M.ircilr preguntil

Señale lo falso de la CELULITIS

Seleccione una:

a. Requiere en algunos casos de antibioticot erapia par enter al IV

b. La b acteria causal m ás frecuente es el estafilococo aureus X

c. La lesión elemental es el nódulo

d. Se acompaña de fiebre

Respuesta incorrecta.

La respuesta correcta es: La lesión elemental es el nódulo

....... ,.00-1.00 f' llarar pnpnta

SEÑALE LO FALSO SOBRE LA DERMATITIS DE CONTACTO ALERGICA

Seleccione una:

a. REQUIERE DE UNA SENSIBILIZACIÓN O EXPOSICIÓN PREVIA

b. SE PRESENTA CON SÍNTOMAS COMO SENSACIÓN QUEMAZÓN O PRURITO

C. SIEMPRE HACE LESIONES A DISTANCIA EN OTRAS ÁREAS DEL SITIO DE CONTACTO ./

d . ES POCO COMÚN EN NIÑOS PEQUEÑOS Y EN MAYORES DE 70 AÑOS DE EDAD

S E~P~E LE LES O ES~ O STANC A EN OTAAS A EAS DEL $ TO DE CONTACTO

Pregunta 2 Correcta Puntua 1,00 sobre 1,00


f Marcar pregunta

SEÑALE LO FALSO DE DERMATITIS ATOPICA

Seleccion e una:

a. ES COMUN LA COLONIZACION POR S AURE S

b ~E Pl,E""JE ENCONTRAR NCRH.'EI\TO DE .A 1 ·E E ER

.A EOS NOF LA ESTA PRES E" TE E~ AL

FE O E O E

Pn&unt-3 Corrttl4 Pumutt 1,00 !.obre 1.00


f Marcar pregunta

La Dermat ,tts atóplca del lactante se caracte, 1,a por ,xcEPTO·


Select1one una:

a Algunos n iños desarrollan lesiones de eccet"l,1St'burre,co que de torma g1adual va adqumendo el aspecto de a dt
o e ccema atop1co.

b La lesiones ~uelen ser papula, o placa

aoc 1z 1 nmshltual e
'T€
a oc;a ,zaclón m.is hab,tual e s e n el 11 0,1<0

Pntgunta4 Correcta Puntúa 1,00 sob.-e 1,00 'P' Marcar pregunta

S ENALE LO VERDADERO SOBRE LA DERMATITIS DE CONTACTO IRRITATIYA

Se ecc1one una:

,3 <;Qi', IRR TANTE:S JABO ES DETERGENTES AC.IDOS PLANTAS

b N A FORMA MAS COMUN DE ENFER'IIIEDAD C.l TANEA C'C.UPACIONAl

A ME IDO LC' MA'- A E~ TAD

A PC"

Pregunta S C0r1ecta PYf'tua 1.00 so~" 1.00 l Marcar prqunta

H PARASllO CUYAS ESIONES SE OCALIZAN PREHRENTEMENTE EN LOS ESPACIOS 'NTE


SUB V PERI UNGUEAL DORSO DE PIE V TOBILLO ES·

AA

Pregunta 6
Corr~ Putitt:.O, 1,00 sobr@1,CO
f Marcu pregunta

Sei\ale lo verdadero de la ESCABIOSIS

Respuesta: D
Pregunta7 Correcta PuntU.!I 1,00sobr~ 1.00 't,. Marcar preaunt.a

CUANDO SE CONSIDERA QUE lA URTICARIA ES CRÓNICA

~lecc1one una·

d MAS DE 6 l1 8 EMANAS

t, DURA MAS DE ' SEMANA

D YA, DE NANO

ANA
Pngunt118 CorrK:ta Puntua 1,00 sobre 1.00
, . Marcar pr99unta

LAS LESIONES DE LA DERMATITIS ATÓP,CA EN EL LAtTANTf SENAlt LO CORRECTO

Seleccione una:

a SON MAS HABITUALES ES EN LA CARA, NO RfSPE íA!', LA l NA' N RWEOOR DE LOS OJC S. LA '<AR Z ,A BOCA
b. LA DERMAT •15 ATÓP 'A NO ~E PRE<E'ITA EN LALTAN E

<DN MAS rABIT Al.ES E EN CUERP D <1ED<l ,<¡ ro·~ >:T N.. !A ce~··
dOMÁHA!:T EEENLA,,1E-r•¡ ·owollDEL

Pnaunta9 Correcta Puntual 00 sob1e 1,00


f Marcar presunta

CUAL [5 El PROC[SQ MEDIANTE EL CUAL LAS ClLULAS IN


AT A PASAN AL SITIO DE LA N~.AMAC ÓN

Pngunta10 Correct.! Pu,itua 1.00 sobre- 1.00


f M.arur pregunta

Señale lo falso de la DERMATITIS ATOPICA

Seleccione una:

a E OIAGNÓSTICO SE BASA EN LAS MANl~ESTAC OMS <LIN CA.

b SE C.ARAC"'ER ZA POR RESEQ EDAD CL TAN-A V P RIT

A t-;[.• LABA RE:AC /TANEA

AA •• • J IAX_ C LAE
Pregunta 1 Con'"ecta P.mtlla 1,00 sobre 1,00 f' M~ruir pregun1 a

SELECCIONE LA OPCIÓN QUE MEJOR DEFINA EL SIGUIENTE CUADRO CLINICO: PACIENTE DES AÑOS DE EDAD, PRESENTA EN EL
TERCIO DISTAL DE LAS PLANTAS Y LA CARA PLANTAR DE LOS DEDOS ERITEMA. DE ASPECTO BRILLANTE Y APERGAMINADO, NO
EXUDATIVO, CON ALGUNAS FISURAS.

Seleccione una:
a. PSORIASIS
b. TIÑAPEDIS

c. NINGUNA DE LAS ANTERIORES

d. DERMATITIS PLANTARJWENIL ,/

Respuesta correcta

U respues~ correcta es: DERMATITIS PLANTAR JUVENIL

Pregunta 2 Incorrecta PumúaO,OOsobre 1,00 f Marca r pregunta

EN UNA URTICARIA CRONlCA LA RONCHA O HABON DESAPARECE A LOS:

Seleccione una:
a.AL M ES

b. EN OCASIONES A LOS SEIS MESES X


c. MINUTOS U HORAS

d. A LOS 4 DIAS

Respuesta incorrecta.

La respuesta correcta es: MINUTOS U HORAS

D (!3!), M Recibi O Fema 0 Rev,s ~ Vol46 0 099-1 1 A.ct1.J.11 A.ctua ID (:6¡ V ~ PRl >< .<J Oiagl' ,..; Ectim,c (:

~ ➔ C O i lms~fcsee.ute.edu.ec/mod/quiZ/rev1ew.php?attempt=23124&cm td=34573

: ~ A.phc..5Ciones G ¡Vay.11! Google Chro... N Netflix O Facebook a YouTube [il MyEnglishl.ab 0 Horarios Ciencias CI.. C MEGA 0 M Dosifica

La respuesta correcta es: MINUTOS U HORAS

Pregunta 3 Incorrecta Punnia 0,00 sobre 1,00 f Marcar pregunta

SEÑALE LO FALSO SOBRE LA DERMATITIS DE CONTACTO

Seleccione una:

a. LA DERMATITIS !RRITATIVA OCURRE DESPUÉS DE 1A EXPOSICIÓN A UNA SUSTANCIA CÁUSTICA

b. LA ATOPIA ES UN FACTOR PREDlSPONENTE PARA SUFRIR DERMATITIS DE CONTACTO

c. EL SULFATO DE NIQUEL NO ES UN ALÉRGENO COMÚN PARA 1A DERMATITIS DE CONTACTO ALÉRGICA

d. EXISTEN DOS TIPOS PRINCIPALES DE DERMATITIS DE CONTACTO IRRITATIVA Y ALÉRGICA X

Respuesta incorrecta

La respuesta correcta es: El SULFATO DE NIQUEL NO ES UN ALÉRGENO COMÚN PARA LA DERMATITIS DE CONTACTO ALÉRGICA

Pregunta 4 Incorrecta Puntúa 0,00 sobre 1,00 'f Marcar pregunta

SEÑALE LO FALSO SOBRE LA DERMATITIS DE CONTACTO ALERGICA

Seleccione una;

a. REQUIERE DE UNA SENSIBILIZACIÓN O EXPOSICIÓN PREVIA

b. ES POCO COMÚN EN NIÑOS PEQUEÑOS Y EN MAYORES DE 70 AÑOS DE EDAD X


c. SIEMPRE HACE LESIONES A DISTANCIA EN OTRAS ÁREAS DEL smo DE CONTACTO

d. SE PRESENTA CON SÍNTOMAS COMO SENSACIÓN QUEMAZÓN O PRURITO

Respuest a incorrecta.

La respuesta correcta es: SIEMPRE HACE LESIONES A DISTANCIA EN OTRAS ÁREAS DEL smo DE CONTACTO

Pregunta 5 Correcta Puntúa 1,00 sobre 1,00 f Marcar pregu nta

MODELO DE 27 AÑOS DE EDAD QUE PR ESENTA PLACAS ERITEMATO ESCAMOSAS EN ZONA DEL ARETE Y POR DEBAJO DEL
OMBLIGO, ESTA ÚLTIMA EN FORMA CIRCULAR DE MONEDA ELALERGÉNO MÁS PROBABLE ES:
Pregunta 5 Corre= Punt\l<l 1,00 sobre 1,00 f' Mar car pregunta

MODELO DE 27 AÑOS DE EDAD QUE PRESENTA PLACAS ERITEMATO ESCAMOSAS EN ZONA DEL ARETE Y POR D EBAJO DEL
OMBLIGO, ESTA ÚLTIMA EN FORMA CIRCULAR DE MONEDA. EL ALERGÉNO MÁS PROBABLE ES:

Seleccione una:

a . NIQUELADOS Y CROMADOS

b. LATEX

c. FRAGANCIAS
d. FORMALDEHIDO

Respuest a correcta

La respuesta correcta es: NIQUELADOS Y CROMADOS

Pregunta 6 Correcta Puntúa 1,00sobre1,00 f' Marcar pregunta

Pregunta 6 Correcta Puntú.!1 1,00sobre 1,00 f' Marcar preg unta

EL PARÁSITO CUYAS LESIONES SE LOCALIZAN PRHERENTEM ENTE EN LOS ESPACIOS INTERD IGITALES DE LOS PIES, REGION ES
SUB Y PERI UNGUEAL DORSO DE PIE Y TOBILLO ES:

Seleccione una:

a. GARRAPATA

b . OEMODEX FOUCULORUM

c. lXODIDAE
d. TUNGA PENETRANS

Resp uesta correct a

La respuesta correcta es: TUNGA PENETRA.NS

Pregunta 7 Corre= Puntúa 1,00 sobre 1,00 f' Marcar pregunta

CUÁLES SON LAS PRIMERAS CÉLULAS EN MIGRAR DURANTE EL PROCESO DE LA INFLAMACIÓN

Seleccione una:

a . LINFOCITOS Y MONOCITOS

b. PMN Y MACRÓFAGOS

c. ERITROCITOS Y PLAQUETAS
d. EOSINOFILOS Y BASÓFtLOS.

Respuesta correcta

La respuesta correcta es: PM N Y MACRÓFAGOS

Pregunta 8 Corre= Puntúa 1.00 sobre 1,00 f' Marcar pregunta

LA FASCITIS NECROTIZANTE, SE CARACTERIZA POR:

Seleccione una:

a . AFECTAR A LA EPIDERM IS

b. INFECCIÓN AGUDA Y RÁPIDAMENTE PROGRESIVA DEL TEJIDO CELULAR SUBCUT ÁNEO

c. LA LOCALIZACIÓN MÁS FRECUENTE ES El TÓRAX


d. NO POSEE UNA ELEVADA MORBIMORTAU DAD

Respuesta correcta

La respuesta correcta es: INFECCIÓN AGUDA Y RÁPlDAMENTE PROGRESIVA DEL TEJIDO CELU LAR SUBCUTÁNEO
Pregunta 9 Correrui Pumú.!i 1.(Xlsobre 1,00 f J.o1arcar pregunta

N IÑO DE 11 AÑOS Q UE CO NSULTA POR UNA LESIÓ N ER ITEMATO EXUDATIVA CUBIERTA DE COSTRA MELISÉRICA PO ST ERIOR A
SÍNDROME GRIPAL CUAL ES El DIAGNÓSTICO MÁS PROBABLE JE ESTA LESIÓN

Seleccione una·

a. IMPETIGO CONTAGIOSO

b. ERISIPELA

c. ECT1MA
d. DERMATITIS ATOPICA

Respuesta correcta

La respuesta correcta es: IMPETl GO CONTAGIOSO

Pregunta 10 COt"rKta Pu ntúa 1,00 sobr" 1,00 f Marcar pregunta

Señal e l o verdad ero de l HERPES ZOST ER

Seleccione una:

a. una característica es el contagio intrafamitiar

b. Es causado por et virus del papiloma

c. La lesión elemental son vesículas umbilicadas agrupadas


d. Nunca se acompa ña de dolor o neur algia

Respuesta correcta

La respuesta correcta es: La lesión elemental son vesículas umbil icacas agrupadas
PRIMER PARCIAL 9A

Pregunta 1
Enunciado de la pregunta
Carlos paciente de 50 años, fumador, con obesidad, que presenta desde hace 2
semanas polidipsia polifagia, poliurea, pérdida de peso, que examen como parte de los
criterios de la ADA nos daría el diagnóstico de diabetes
Seleccione una:
a. Glucosa en ayunas mayor de 100 mg/dl
b. Glucosa mayor de 200 tomada al azar.
c. Glucosa postprandial mayor de 150 mg/dl
d. Glucosa mayor a 126 mg/dl durante prueba oral de tolerancia a la glucosa
Retroalimentación
La respuesta correcta es: Glucosa mayor de 200 tomada al azar.

Pregunta 2
Enunciado de la pregunta
Una de las características principales por las cuales el dengue puede diseminarse es:
Seleccione una:
a. Es sumamente frecuente la progresión de daño encefálico.
b. Una característica precoz es la trombocitosis y neutrofilia.
c. Debido a la gran capacidad de diseminación del vector Anopheles.
d. El vector es muy cercano a los asentamientos humanos.
Retroalimentación
La respuesta correcta es: El vector es muy cercano a los asentamientos humanos.

Pregunta 3
Enunciado de la pregunta
Paciente masculino de 66 años de edad, con antecedentes de deterioro cognitivo leve,
HTA, ingresa para una cirugía electiva de resección prostática por una hipertrofia
prostática benigna, en su postoperatorio se encuentra con sondaje (cateterización),
vesical permanente, dolor pélvico, y un cuadro de delirium (o sd confusional agudo), se
reinició el enalapril en el postoperatorio, de las siguientes opciones señale cuál NO es
un factor de riesgo para el aparecimiento de delirium
Seleccione una:
a. Dolor en el postoperatorio
b. Sondaje vesical
c. Administración de enalapril
d. Antecedentes de deterioro cognitivo leve
Retroalimentación
La respuesta correcta es: Administración de enalapril

Pregunta 4
Enunciado de la pregunta
Paciente de 38 años de género femenino, con APP diagnóstico de fenómeno de
Raynaud en estudio de un posible Lupus eritematoso sistémico, también diagnóstico
de migraña con aura desde la juventud, acude por un ataque agudo de cefalea, usted
qué fármaco NO recomendaría:
Seleccione una:
a. Clorpromazina
b. AINES
c. Sumatriptán
d. Aspirina + metoclopramida
Retroalimentación
La respuesta correcta es: Sumatriptán

Pregunta 5
Enunciado de la pregunta
Hombre de 54 años que acude a chequeo general. Se detecta un índice de masa
corporal de 32,8 kg/m2 y glucemia en ayunas 138 mg/l. Un mes después, glucemia
123 mg/dl. ¿Qué recomendación terapéutica efectuaría en primer lugar?
Seleccione una:
a. Prescribir una sulfonilurea.
b. Administrar metformina.
c. Cambios conductuales, dieta y ejercicio físico.
d. Insulina antes de cada comida.
Retroalimentación
La respuesta correcta es: Cambios conductuales, dieta y ejercicio físico.

Pregunta 6
Enunciado de la pregunta
Paciente de 16 años de género masculino, sin APP, empieza con crisis convulsivas
tónico clónicas generalizadas hace 1 año, 1 crisis cada 2 meses, su examen
neurológico es normal al igual que la Resonancia Magnética cerebral, el
electroencefalograma demostró actividad epileptiforme, cuál de las siguientes
etiologías consideraría en el diagnóstico
Seleccione una:
a. Epilepsia por enfermedad degenerativa
b. Epilepsia por desorden genético
c. Epilepsia secundaria a disturbios hidroelectrolíticos
d. Epilepsia secundaria a autoanticuerpos
Retroalimentación
La respuesta correcta es: Epilepsia por desorden genético

Pregunta 7
Enunciado de la pregunta
Mujer de 68 años de edad en estado de coma. Al examen: PA: 90/50 mmHg; Urea: 56
mg%, Creatinina: 2,5 mg%; Glucosa: 1150 mg%; Na: 130 mEq/L; Bicarbonato: 23
mEq/L; pH: 7,32. Volumen urinario de 24 horas: 450 mL. El diagnóstico MÁS probable
es:
Seleccione una:
a. Coma por hiponatremia.
b. Coma por cetoacidosis diabética.
c. Coma urémico.
d. Coma hiperosmolar no cetosico.
Retroalimentación
La respuesta correcta es: Coma hiperosmolar no cetosico.

Pregunta 8
Enunciado de la pregunta
¿De qué depende la inmunidad contra Leptospira?
Seleccione una:
a. Producción de anticuerpos contra LPS inespecíficos.
b. Producción de anticuerpos circulantes contra LPS específicos de un serotipo.
c. Producción de anticuerpos contra proteínas inespecíficas de un serotipo.
d. Producción de anticuerpos contra los distintos serotipos.
Retroalimentación
La respuesta correcta es: Producción de anticuerpos circulantes contra LPS
específicos de un serotipo.
Pregunta 9
Enunciado de la pregunta
El mecanismo por el cual la infección por cólera produce una diarrea tan intensa es
por:
Seleccione una:
a. Vibrios enteroagregativos.
b. Toxina enterogénica
c. Toxina Shiga-like
d. Vibrios entero hemorrágicos.
Retroalimentación
La respuesta correcta es: Toxina enterogénica

Pregunta 10
Enunciado de la pregunta
Un paciente con una glucemia de 300mg/dl puede referir los siguientes signos y
síntomas: señale lo correcto:
Seleccione una:
a. Cataratas, cetosis, uremia
b. Polidipsia, astenia, calambres musculares
c. Amputación de miembro inferior, HTA, IVU
d. Rinorrea, glucosuria, Ceguera
Retroalimentación
La respuesta correcta es: Polidipsia, astenia, calambres musculares

Pregunta 11
Enunciado de la pregunta
En un paciente con diabetes mellitus tipo 2, índice de masa corporal mayor de 30 cuál
sería el antidiabético oral más indicado:
Seleccione una:
a. Metformina
b. Tiazolidinedionas
c. Ninguna de las anteriores
d. Glibenclamida
Retroalimentación
La respuesta correcta es: Metformina

Pregunta 12
Enunciado de la pregunta
La ictericia en un paciente con Malaria es frecuente a partir:
Seleccione una:
a. La ictericia no aparece en la malaria.
b. Ocasionalmente en niños debido a su alta susceptibilidad
c. Siempre existirá fiebre e ictericia debido a la destrucción de eritrocitos.
d. La Ictericia en Malaria es frecuente a partir de los 15 años.
Retroalimentación
La respuesta correcta es: La Ictericia en Malaria es frecuente a partir de los 15 años.

Pregunta 13
Enunciado de la pregunta
En pacientes con catatonía, usted sospecharía:
Seleccione una:
a. Lesión de la región orbitaria frontal
b. Secuela del coma
c. Herniación central
d. Trastorno psiquiátrico
Retroalimentación
La respuesta correcta es: Trastorno psiquiátrico

Pregunta 14
Enunciado de la pregunta
Tras la infección de un niño con el virus de la Influenza con la siguientes
características: fiebre intensa de inicio súbito, disnea y cianosis a la Rx de tórax se
aprecia patrón asociado con infiltrados intersticiales difusos e hipoxia intensa, usted
sospecharía de:
Seleccione una:
a. Neumonía bacteriana secundaria.
b. Neumonía viral primaría.
c. Neumonía bacteriana primaria.
d. Neumonía viral secundaria.
Retroalimentación
La respuesta correcta es: Neumonía viral primaría.

Pregunta 15
Enunciado de la pregunta
En cuanto a la profilaxis antitetánica en el tratamiento sistemático de las heridas, en
una herida pequeña limpia sin ningún antecedente de vacunación antitetánica Usted
recomendaría:
Seleccione una:
a. Es indicativo de inmunoglobulina.
b. Solo limpiar la herida y dar indicaciones de cuidado al paciente, ofrecer la anti
toxina.
c. Dar antibiótico de manera profiláctica.
d. Vacunación antitetánica de inmediato.
Retroalimentación
La respuesta correcta es: Solo limpiar la herida y dar indicaciones de cuidado al
paciente, ofrecer la anti toxina.

Pregunta 16
Enunciado de la pregunta
Carlos paciente de 18 años sin antecedentes de importancia acude a la sala de
Emergencia con aliento de manzana, somnoliento, se le realiza una glicemia y se
encuentra glucosa de 300 mg / dl, se le realiza una gasometría y se encuentra un PH
de 7 . Bicarbonato de 9, presencia de cetonas en orina ++, cuál sería su apreciación
diagnostica
Seleccione una:
a. Estado hiperosmolar con debut diabético
b. Hiperglucemia sostenida con debut diabético
c. Cetoacidosis diabética con debut diabético
d. Descompensación simple
Retroalimentación
La respuesta correcta es: Cetoacidosis diabética con debut diabético

Pregunta 17
Enunciado de la pregunta
Masculino de 52 años consulta por poliuria, polidipsia intensa y pérdida involuntaria de
10 kg de peso; es diagnosticado de diabetes mellitus por una glucemia plasmática de
322 mg/dL y una HbA1c de 9,8%. Se le recomienda dieta, ejercicio físico, e iniciar
tratamiento con metformina 850 mg cada 12 horas y glimepirida 6 mg/dia. En las
semanas siguientes los controles glucémicos se van reduciendo progresivamente. A
los 4 meses la glucemia es de 94 mg/dL y la HbA1c de 5,9%. El paciente se queja de
episodios frecuentes de pérdida del equilibrio “mareo”, dolor epigástrico, visión
borrosa, sudoración y temblor, que mejoran con la ingesta de alimentos y que ocurren
sobre todo al final de la mañana y al final de la tarde. ¿Qué modificación propondría en
su tratamiento?
Seleccione una:
a. Suspender la metformina.
b. Sustituir la metformina por un inhibidor de la DPP4
c. Suspender la sulfonilurea
d. Revisar la distribución de hidratos de carbono de su dieta.
Retroalimentación
La respuesta correcta es: Suspender la sulfonilurea

Pregunta 18
Enunciado de la pregunta
Un hombre de 62 años con una diabetes mellitus tipo 2, de 10 años de evolución
realiza tratamiento con metformina y sitagliptina. Hace ejercicio físico escaso y realiza
una dieta adecuada. En los últimos 6 meses ha perdido peso y tiene más astenia. Sus
controles glucémicos se han deteriorado pasando de glucemias basales de 110-140
mg/dl a glucemias de 170-200 mg/dl, así como su hemoglobina glicosilada que ha
pasado de 7,1 a 8,5%. La medida terapéutica más adecuada a realizar es, señale la
alternativa correcta:
Seleccione una:
a. Sustituir metformina por glimepirida
b. Asociar al tratamiento ascarbosa.
c. Sustituir sitagliptina por pioglitazona
d. Asociar al tratamiento una dosis de insulina basal.
Retroalimentación
La respuesta correcta es: Asociar al tratamiento una dosis de insulina basal.

Pregunta 19
Enunciado de la pregunta
El siguiente enunciado: Un estado parecido al sueño profundo en el que el paciente
permanece con los ojos cerrados y el paciente no puede ser despertado, se refiere a:
Seleccione una:
a. Estado de coma
b. Estupor
c. Somnolencia
d. Estado vegetativo
Retroalimentación
La respuesta correcta es: Estado de coma

Pregunta 20
Enunciado de la pregunta
Dentro de las manifestaciones tardías de la sífilis tenemos trastornos cardiovasculares
de cual podemos desprender el siguiente:
Seleccione una:
a. Insuficiencia cardiaca congestiva.
b. Aneurismas
c. Trastornos del endotelio
d. Flebitis irritativa.
Retroalimentación
La respuesta correcta es: Aneurismas

Pregunta 21
Enunciado de la pregunta
Niña de 11 años es hospitalizada por presentar nerviosismo y palpitaciones de 24
horas de evolución. El apetito ha sido bueno pero registra pérdida de peso. Al examen
físico: FC 100lpm, temperatura 37.5°C, transpiración excesiva y ROT rápidos. Los ojos
presentaban una apariencia vidriosa. El diagnóstico MÁS probable es:
Seleccione una:
a. Disautonomía familiar.
b. Tirotoxicosis juvenil.
c. Diabetes mellitus tipo 1.
d. Psicosis juvenil.
Retroalimentación
La respuesta correcta es: Diabetes mellitus tipo 1.

Pregunta 22
Enunciado de la pregunta
En el manejo de un paciente con delirium agitado (o hiperactivo), de las siguientes
opciones, señale la que NO incluiría en su prescripción médica:
Seleccione una:
a. Manejo no farmacológico como: colocar un reloj visible para el paciente
b. Evitar uso de restricciones físicas (amarras) para controlar el movimiento del
paciente
c. Antipsicóticos atípicos como la quetiapina en vía oral
d. Benzodiacepina como el clonazepam en vía oral
Retroalimentación
La respuesta correcta es: Benzodiacepina como el clonazepam en vía oral

Pregunta 23
Enunciado de la pregunta
En un paciente de 68 años de edad, con APP: HTA, que ingresa con un cuadro de
crisis focales motoras sin pérdida del estado de conciencia y limitadas al miembro
superior derecho, que se han repetido 1 crisis cada 3 días desde hace 2 semanas,
además se acompaña de cefalea desde hace 3 meses con características de
empeoramiento progresivo, en los estudios de imagen se encontró una lesión
expansiva frontal izquierda; en relación al tratamiento farmacológico de elección,
señale el que escogería:
Seleccione una:
a. Clonazepam
b. Lamotrigina
c. Acido valproico
d. Fenobarbital
Retroalimentación
La respuesta correcta es: Lamotrigina

Pregunta 24
Enunciado de la pregunta
Paciente de 22 años, género femenino, tiene desde hace 1 año dolor de cabeza de
forma episódica (2 episodios al mes), hemicránea derecha o izquierda, pulsátil,
intensidad moderada, se acompaña de náusea y fotofobia, el dolor es precedido por
escotomas centellantes que duran 90 minutos aproximadamente, en los últimos 3
meses el dolor se ha tornado más frecuente (3 episodios semanales) y al toser
exacerba en intensidad a dolor severo, de los siguientes enunciados cuál NO
considera signo/síntoma de alarma:
Seleccione una:
a. Aura de duración larga
b. Localización del dolor
c. Empeoramiento del dolor (más frecuente)
d. Valsalva positivo
Retroalimentación
La respuesta correcta es: Localización del dolor

Pregunta 25
Enunciado de la pregunta
En la clasificación de OMS de adultos con VIH – Sida en el estadio C3 tenemos a:
Seleccione una:
a. Mayor de 500 células CD4+ Asintomático.
b. Menor de 300 células CD4+ con síntomas definidores de Sida.
c. Menor de 200 células CD4+ con síntomas definidores de Sida.
d. Mayor de 400 células CD4+ con síntomas no definidores de Sida.
Retroalimentación
La respuesta correcta es: Menor de 200 células CD4+ con síntomas definidores de
Sida.

PRIMER PARCIAL 9B

Pregunta 1 Correcta Puntúa 1,00 sobre 1,00


Cuánto es la dosis sublingual de nitroglicerina en angina de pecho:
Seleccione una:
a. 1.3 a 1.6 mg
b. 0.8 a 1.0 mg
c. 3 a 6 mg
d. 0.3 a 0.6 mg
La respuesta correcta es: 0.3 a 0.6 mg

Pregunta 2 Incorrecta Puntúa 0,00 sobre 1,00


Mujer de 68 años de edad en estado de coma. Al examen: PA: 90/50 mmHg; Urea: 56
mg%, Creatinina: 2,5 mg%; Glucosa: 1150 mg%; Na: 130 mEq/L; Bicarbonato: 23
mEq/L; pH: 7,32. Volumen urinario de 24 horas: 450 mL. El diagnóstico MÁS probable
es:
Seleccione una:
a. Coma por cetoacidosis diabética.
b. Coma por hiponatremia.
c. Coma hiperosmolar no cetosico.
d. Coma urémico.
La respuesta correcta es: Coma hiperosmolar no cetosico.

Pregunta 3 Correcta Puntúa 1,00 sobre 1,00


Cuánto es la dosis máxima de dinitrato de isosorbide de liberación prolongada en
angina de pecho:
Seleccione una:

-
a. 40 mg
b. 300 mg
c. 120 mg
d. 160 mg
La respuesta correcta es: 300 mg

Pregunta 4 Correcta Puntúa 1,00 sobre 1,00


Un paciente con una glucemia de 300mg/dl puede referir los siguientes signos y
síntomas: señale lo
correcto: Seleccione una:
a. Rinorrea, glucosuria, Ceguera
b. Cataratas, cetosis, uremia
c. Amputación de miembro inferior, HTA, IVU
d. Polidipsia, astenia, calambres musculares
La respuesta correcta es: Polidipsia, astenia, calambres musculares

Pregunta 5 Correcta Puntúa 1,00 sobre 1,00


El mecanismo por el cual la infección por cólera produce una diarrea tan intensa es
por:
Seleccione una:
a. Toxina Shiga-like
b. Toxina enterogénica
c. Vibrios enteroagregativos.
d. Vibrios entero hemorrágicos.
La respuesta correcta es: Toxina enterogénica

Pregunta 6 Incorrecta Puntúa 0,00 sobre 1,00


Niña de 11 años es hospitalizada por presentar nerviosismo y palpitaciones de 24
horas de evolución. El apetito ha sido bueno pero registra pérdida de peso. Al examen
físico: FC 100lpm, temperatura 37.5°C, transpiración excesiva y ROT rápidos. Los ojos
presentaban una apariencia vidriosa. El diagnóstico MÁS probable es:
Seleccione una:
a. Diabetes mellitus tipo 1.
b. Disautonomía familiar.
c. Psicosis juvenil.
d. Tirotoxicosis juvenil.
La respuesta correcta es: Diabetes mellitus tipo 1.

Pregunta 7 Correcta Puntúa 1,00 sobre 1,00


Carlos paciente de 18 años sin antecedentes de importancia acude a la sala de
Emergencia con aliento de manzana, somnoliento, se le realiza una glicemia y se
encuentra glucosa de 300 mg / dl, se le realiza una gasometría y se encuentra un PH
de 7 . Bicarbonato de 9, presencia de cetonas en orina ++, cuál sería su apreciación
diagnostica
Seleccione una:
a. Hiperglucemia sostenida con debut diabético
b. Descompensación simple
c. Estado hiperosmolar con debut diabético
d. Cetoacidosis diabética con debut diabético
La respuesta correcta es: Cetoacidosis diabética con debut diabético

Pregunta 8 Correcta Puntúa 1,00 sobre 1,00


¿De qué depende la inmunidad contra Leptospira?
Seleccione una:
a. Producción de anticuerpos circulantes contra LPS específicos de un serotipo.
b. Producción de anticuerpos contra proteínas inespecíficas de un serotipo.
c. Producción de anticuerpos contra LPS inespecíficos.
d. Producción de anticuerpos contra los distintos serotipos.
La respuesta correcta es: Producción de anticuerpos circulantes contra LPS
específicos de un serotipo.

Pregunta 9 Correcta Puntúa 1,00 sobre 1,00


Una de las características principales por las cuales el dengue puede diseminarse es:
Seleccione una:
a. El vector es muy cercano a los asentamientos humanos.
b. Una característica precoz es la trombocitosis y neutrofilia.
c. Es sumamente frecuente la progresión de daño encefálico.
d. Debido a la gran capacidad de diseminación del vector Anopheles.
La respuesta correcta es: El vector es muy cercano a los asentamientos humanos.

Pregunta 10 Correcta Puntúa 1,00 sobre 1,00


Un hombre de 62 años con una diabetes mellitus tipo 2, de 10 años de evolución
realiza tratamiento con metformina y sitagliptina. Hace ejercicio físico escaso y realiza
una dieta adecuada. En los últimos 6 meses ha perdido peso y tiene más astenia. Sus
controles glucémicos se han deteriorado pasando de glucemias basales de 110-140
mg/dl a glucemias de 170-200 mg/dl, así como su hemoglobina glicosilada que ha
pasado de 7,1 a 8,5%. La medida terapéutica más adecuada a realizar es, señale la
alternativa correcta:
Seleccione una:
a. Sustituir metformina por glimepirida
b. Sustituir sitagliptina por pioglitazona
c. Asociar al tratamiento ascarbosa.
d. Asociar al tratamiento una dosis de insulina basal.
La respuesta correcta es: Asociar al tratamiento una dosis de insulina basal.

Pregunta 11 Correcta Puntúa 1,00 sobre 1,00


Dentro de las manifestaciones tardías de la sí􀃕lis tenemos trastornos cardiovasculares
de cual podemos desprender el siguiente:
Seleccione una:
a. Aneurismas
b. Trastornos del endotelio
c. Insuficiencia cardiaca congestiva.
d. Flebitis irritativa.
La respuesta correcta es: Aneurismas

Pregunta 12 Correcta Puntúa 1,00 sobre 1,00


Cuál síndrome genético es común en la insuficiencia aórtica:
Seleccione una:
a. Síndrome de Marfán
b. Anomalía de Ebstein
c. Síndrome de Eisenmenger
d. Síndrome de Rastelli
La respuesta correcta es: Síndrome de Marfán

Pregunta 13 Correcta Puntúa 1,00 sobre 1,00


Qué infección causa bloqueo auriculoventricular:
Seleccione una:
a. Varicela

-
b. Tripanosomiasis
c. Sífilis
d. Sarampión
La respuesta correcta es: Sífilis

Pregunta 14 Correcta Puntúa 1,00 sobre 1,00


La ictericia en un paciente con Malaria es frecuente a partir:
Seleccione una:
a. Ocasionalmente en niños debido a su alta susceptibilidad
b. La ictericia no aparece en la malaria.
c. La Ictericia en Malaria es frecuente a partir de los 15 años.
d. Siempre existirá fiebre e ictericia debido a la destrucción de eritrocitos.
La respuesta correcta es: La Ictericia en Malaria es frecuente a partir de los 15 años.

Pregunta 15 Correcta Puntúa 1,00 sobre 1,00


Masculino de 52 años consulta por poliuria, polidipsia intensa y pérdida involuntaria de
10 kg de peso; es diagnosticado de diabetes mellitus por una glucemia plasmática de
322 mg/dL y una HbA1c de 9,8%. Se le recomienda dieta, ejercicio físico, e iniciar
tratamiento con metformina 850 mg cada 12 horas y glimepirida 6 mg/dia. En las
semanas siguientes los controles glucémicos se van reduciendo progresivamente. A
los 4 meses la glucemia es de 94 mg/dL y la HbA1c de 5,9%. El paciente se queja de
episodios frecuentes de pérdida del equilibrio “mareo”, dolor epigástrico, visión
borrosa, sudoración y temblor, que mejoran con la ingesta de alimentos y que ocurren
sobre todo al final de la mañana y al final de la tarde. ¿Qué modificación propondría en
su tratamiento?
Seleccione una:
a. Sustituir la metformina por un inhibidor de la DPP4
b. Suspender la metformina.
c. Revisar la distribución de hidratos de carbono de su dieta.
d. Suspender la sulfonilurea
La respuesta correcta es: Suspender la sulfonilurea

Pregunta 16 Correcta Puntúa 1,00 sobre 1,00


Cuánto es la frecuencia de administración de propanolol en cardiopatía isquémica:
Seleccione una:
a. Cada 8 horas
b. Cada 6 horas
c. Cada 24 horas
d. Cada 12 horas
La respuesta correcta es: Cada 12 horas

Pregunta 17 Correcta Puntúa 1,00 sobre 1,00


Hombre de 54 años que acude a chequeo general. Se detecta un índice de masa
corporal de 32,8 kg/m2 y glucemia en ayunas 138 mg/l. Un mes después, glucemia
123 mg/dl. ¿Qué recomendación terapéutica efectuaría en primer lugar?
Seleccione una:
a. Administrar metformina.
b. Prescribir una sulfonilurea.
c. Cambios conductuales, dieta y ejercicio físico.
d. Insulina antes de cada comida.
La respuesta correcta es: Cambios conductuales, dieta y ejercicio físico.

Pregunta 18 Correcta Puntúa 1,00 sobre 1,00


Tras la infección de un niño con el virus de la In􀃕uenza con la siguientes
características: 􀃕ebre intensa de inicio súbito, disnea y cianosis a la Rx de tórax se
aprecia patrón asociado con in􀃕ltrados intersticiales difusos e hipoxia intensa, usted
sospecharía de:
Seleccione una:
a. Neumonía bacteriana secundaria.
b. Neumonía viral secundaria.
c. Neumonía viral primaría.
d. Neumonía bacteriana primaria.
La respuesta correcta es: Neumonía viral primaría.
Pregunta 19 Correcta Puntúa 1,00 sobre 1,00
Cuál es una causa de taquicardia sinusal fisiológica:
Seleccione una:
a. Hipotiroidismo
b. Hipertensión pulmonar
c. Sedenterismo
d. Feocromocitoma
La respuesta correcta es: Feocromocitoma

Pregunta 20 Correcta Puntúa 1,00 sobre 1,00


En cuanto a la profilaxis antitetánica en el tratamiento sistemático de las heridas, en
una herida pequeña limpia sin ningún antecedente de vacunación antitetánica Usted
recomendaría:
Seleccione una:
a. Vacunación antitetánica de inmediato.
b. Dar antibiótico de manera pro􀃕láctica.
c. Solo limpiar la herida y dar indicaciones de cuidado al paciente, ofrecer la anti
toxina.
d. Es indicativo de inmunoglobulina.
La respuesta correcta es: Solo limpiar la herida y dar indicaciones de cuidado al
paciente, ofrecer la anti toxina.

Pregunta 21 Correcta Puntúa 1,00 sobre 1,00


Cuánto es la frecuencia de administración de atenolol en cardiopatía isquémica:
Seleccione una:
a. Cada 12 horas
b. Cada 6 horas
c. Cada 8 horas
d. Cada 24 horas
La respuesta correcta es: Cada 24 horas

Pregunta 22 Correcta Puntúa 1,00 sobre 1,00


En un paciente con diabetes mellitus tipo 2, índice de masa corporal mayor de 30 cuál
sería el antidiabético oral más indicado:
Seleccione una:
a. Metformina
b. Tiazolidinedionas
c. Glibenclamida
d. Ninguna de las anteriores
La respuesta correcta es: Metformina

Pregunta 23 Correcta Puntúa 1,00 sobre 1,00


Un paciente hipertenso con descompensación aguda “típica” de insu􀃕ciencia cardiaca
sin sobrecarga de volumen, que fármaco usaría:
Seleccione una:
a. Clortalidona
b. Dobutamina
c. Furosemida
d. Nitroprusiato
La respuesta correcta es: Nitroprusiato

Pregunta 24 Correcta Puntúa 1,00 sobre 1,00


Qué enfermedad reumatológica causa insu􀃕ciencia aórtica:
Seleccione una:
a. Espondilitis anquilosante
b. Vasculitis leucocitoclástica
c. Lupus eritematoso sistémico
d. Artritis reumatoidea
La respuesta correcta es: Espondilitis anquilosante

Pregunta 25 Incorrecta Puntúa 0,00 sobre 1,00


Paciente diabético que acude a Emergencia obnubilado con los siguientes resultados
de laboratorio: glucemia 340 mg/dL, pH 7.20 mg/dL, Na+ 136rnEq/L, K+ 4,5 mEq/L, y
Bicarbonato estándar 11,2mMol/L, con cetonuria. En relación con el tratamiento y la
evolución ¿cuál de los siguientes enunciados NO es correcto?:
Seleccione una:
a. Proporcionaremos aporte de suero glucosado dextrosa a 5% cuando la glucemia
sea igual o inferior a 250 mg/dL.
b. La infusión endovenosa con insulina debe iniciarse dos horas después de la
fluidoterapia de resucitación con solución salina.
c. Puede observarse respiración de Kussmaul.
d. Son frecuentes el dolor abdominal, náuseas y vómitos; también puede alterarse el
estado de conciencia.
La respuesta correcta es: La infusión endovenosa con insulina debe iniciarse dos
horas después de la fluidoterapia de resucitación con solución salina.

PRIMER PARCIAL 9C

Pregunta 1
Correcta
Puntúa 1,00 sobre 1,00
Marcar pregunta
Enunciado de la pregunta
Cuánto es la dosis intravenosa de sostén para digoxina en arritmias:
Seleccione una:
a. 0.145-0.250 mg/día
b. 0.175-0.250 mg/día
c. 0.125-0.250 mg/día
d. 0.200-0.250 mg/día
Retroalimentación
La respuesta correcta es: 0.125-0.250 mg/día

Pregunta 2
Correcta
Puntúa 1,00 sobre 1,00
Marcar pregunta
Enunciado de la pregunta
¿De qué depende la inmunidad contra Leptospira?
Seleccione una:
a. Producción de anticuerpos circulantes contra LPS específicos de un serotipo.
b. Producción de anticuerpos contra los distintos serotipos.
c. Producción de anticuerpos contra LPS inespecíficos.
d. Producción de anticuerpos contra proteínas inespecíficas de un serotipo.
Retroalimentación
La respuesta correcta es: Producción de anticuerpos circulantes contra LPS
específicos de un serotipo.

Pregunta 3
Correcta
Puntúa 1,00 sobre 1,00
Marcar pregunta
Enunciado de la pregunta
Paciente de sexo masculino, de 48 años de edad que presenta una tumefacción
eritematosa de bordes mal definidos, muy dolorosa a la palpación en tórax posterior de
consistencia semidura, con un pequeño orificio central a través del cual drena material
purulento de mal olor de 8 días de evolución. ¿Cuál de las siguientes alternativas

-
corresponde al diagnóstico más probable?
Seleccione una:
a. Forúnculo
b. Celulitis
c. Foliculitis profunda
d. Erisipela
Retroalimentación
La respuesta correcta es: Forúnculo

Pregunta 4
Correcta
Puntúa 1,00 sobre 1,00
Marcar pregunta
Enunciado de la pregunta
Cuánto es la dosis máxima de dinitrato de isosorbide de liberación prolongada en
angina de pecho:
Seleccione una:
a. 160 mg
b. 120 mg

-
c. 40 mg
d. 300 mg
Retroalimentación
La respuesta correcta es: 300 mg

Pregunta 5
Correcta
Puntúa 1,00 sobre 1,00
Marcar pregunta
Enunciado de la pregunta
Dentro de las manifestaciones tardías de la sífilis tenemos trastornos cardiovasculares
de cual podemos desprender el siguiente:
Seleccione una:
a. Aneurismas
b. Flebitis irritativa.
c. Trastornos del endotelio
d. Insuficiencia cardiaca congestiva.
Retroalimentación
La respuesta correcta es: Aneurismas

Pregunta 6
Correcta
Puntúa 1,00 sobre 1,00
Marcar pregunta
Enunciado de la pregunta
Cuánto es la frecuencia de administración de propanolol en cardiopatía isquémica:
Seleccione una:
a. Cada 8 horas
b. Cada 6 horas
c. Cada 24 horas
d. Cada 12 horas
Retroalimentación
La respuesta correcta es: Cada 12 horas

Pregunta 7
Correcta
Puntúa 1,00 sobre 1,00
Marcar pregunta
Enunciado de la pregunta
En caso que se enfrente a una situación de tétanos una medida de soporte importante
para el control de espasmos es:
Seleccione una:
a. Gabapentina
b. Metronidazol
c. Antitoxina
d. Diazepam
Retroalimentación
La respuesta correcta es: Diazepam

Pregunta 8
Correcta
Puntúa 1,00 sobre 1,00
Marcar pregunta
Enunciado de la pregunta
En la clasificación de OMS de adultos con VIH – Sida en el estadio C3 tenemos a:
Seleccione una:
a. Mayor de 400 células CD4+ con síntomas no definidores de Sida.
b. Menor de 200 células CD4+ con síntomas definidores de Sida.
c. Menor de 300 células CD4+ con síntomas definidores de Sida.
d. Mayor de 500 células CD4+ Asintomático.
Retroalimentación
La respuesta correcta es: Menor de 200 células CD4+ con síntomas definidores de
Sida.

Pregunta 9
Incorrecta
Puntúa 0,00 sobre 1,00
Marcar pregunta
Enunciado de la pregunta
La ictericia en un paciente con Malaria es frecuente a partir:
Seleccione una:
a. La ictericia no aparece en la malaria.
b. Siempre existirá fiebre e ictericia debido a la destrucción de eritrocitos.
c. Ocasionalmente en niños debido a su alta susceptibilidad
d. La Ictericia en Malaria es frecuente a partir de los 15 años.
Retroalimentación
La respuesta correcta es: La Ictericia en Malaria es frecuente a partir de los 15 años.

Pregunta 10
Correcta
Puntúa 1,00 sobre 1,00
Marcar pregunta
Enunciado de la pregunta
Un paciente hipertenso con descompensación aguda “típica” de insuficiencia cardiaca
sin sobrecarga de volumen, que fármaco usaría:
Seleccione una:
a. Dobutamina
b. Furosemida
c. Nitroprusiato
d. Clortalidona
Retroalimentación
La respuesta correcta es: Nitroprusiato

Pregunta 11
Correcta
Puntúa 1,00 sobre 1,00
Marcar pregunta
Enunciado de la pregunta
Cuál síndrome genético es común en la insuficiencia aórtica:
Seleccione una:
a. Anomalía de Ebstein
b. Síndrome de Eisenmenger
c. Síndrome de Marfán
d. Síndrome de Rastelli
Retroalimentación
La respuesta correcta es: Síndrome de Marfán

Pregunta 12
Correcta
Puntúa 1,00 sobre 1,00
Marcar pregunta
Enunciado de la pregunta
Paciente de sexo masculino, de 48 años de edad que presenta una tumefacción
eritematosa de bordes mal definidos, muy dolorosa a la palpación en tórax posterior de
consistencia semidura, con un pequeño orificio central a través del cual drena material
purulento de mal olor de 8 días de evolución. ¿Cuál de las siguientes alternativas
indica los antibióticos de elección en este caso?
Seleccione una:
a. Amoxicilina + ácido clavulanico
b. Dicloxacilina y linezolid
c. Ceftriaxona y gentamicina
d. Dicloxacilina y trimetroprim sulfametoxazol
Retroalimentación
La respuesta correcta es: Dicloxacilina y trimetroprim sulfametoxazol

Pregunta 13
Correcta
Puntúa 1,00 sobre 1,00
Marcar pregunta
Enunciado de la pregunta
Qué infección causa bloqueo auriculoventricular:
Seleccione una:

-
a. Varicela
b. Tripanosomiasis
c. Sífilis
d. Sarampión
Retroalimentación
La respuesta correcta es: Sífilis

Pregunta 14
Correcta
Puntúa 1,00 sobre 1,00
Marcar pregunta
Enunciado de la pregunta
Cuál consideraría como primera opción para el diagnóstico de fiebre tifoidea ( fiebre
enterica):
Seleccione una:
a. biopsia intestinal
b. Reacción de Widal
c. rosa de bengala
d. hemocultivo
Retroalimentación
La respuesta correcta es: hemocultivo

Pregunta 15
Correcta
Puntúa 1,00 sobre 1,00
Marcar pregunta
Enunciado de la pregunta
Cuál es una causa de taquicardia sinusal fisiológica:
Seleccione una:
a. Feocromocitoma
b. Hipotiroidismo
c. Hipertensión pulmonar
d. Sedenterismo
Retroalimentación
La respuesta correcta es: Feocromocitoma

Pregunta 16
Correcta
Puntúa 1,00 sobre 1,00
Marcar pregunta
Enunciado de la pregunta
Una de las características principales por las cuales el dengue puede diseminarse es:
Seleccione una:
a. Debido a la gran capacidad de diseminación del vector Anopheles.
b. Es sumamente frecuente la progresión de daño encefálico.
c. El vector es muy cercano a los asentamientos humanos.
d. Una característica precoz es la trombocitosis y neutrofilia.
Retroalimentación
La respuesta correcta es: El vector es muy cercano a los asentamientos humanos.

Pregunta 17
Correcta
Puntúa 1,00 sobre 1,00
Marcar pregunta
Enunciado de la pregunta
¿Cuál de las siguientes alternativas es el tratamiento sistémico de elección en
impétigo contagioso?
Seleccione una:
a. Eritromicina
b. Azitromicina
c. Ampicilina
d. Dicloxacilina
Retroalimentación
La respuesta correcta es: Dicloxacilina

Pregunta 18
Correcta
Puntúa 1,00 sobre 1,00
Marcar pregunta
Enunciado de la pregunta
En cuanto a la profilaxis antitetánica en el tratamiento sistemático de las heridas, en
una herida pequeña limpia sin ningún antecedente de vacunación antitetánica Usted
recomendaría:
Seleccione una:
a. Es indicativo de inmunoglobulina.
b. Dar antibiótico de manera profiláctica.
c. Solo limpiar la herida y dar indicaciones de cuidado al paciente, ofrecer la anti
toxina.
d. Vacunación antitetánica de inmediato.
Retroalimentación
La respuesta correcta es: Solo limpiar la herida y dar indicaciones de cuidado al
paciente, ofrecer la anti toxina.

Pregunta 19
Correcta
Puntúa 1,00 sobre 1,00
Marcar pregunta
Enunciado de la pregunta
Paciente de sexo femenino de 30 años de edad que presenta desde la infancia placas
eritematodescamativas de bordes bien definidos en codos, y rodilla con descamación
intensa y prurito moderado, además presenta eritema y descamación profusa en cuero
cabelludo. ¿Cuál de las siguientes alternativas corresponde al diagnóstico clínico más
probable?

-
Seleccione una:
a. Psoriasis + dermatitis seborreica de cuero cabelludo
b. Psoriasis
c. Dermatitis atópica + dermatitis seborreica de cuero cabelludo
d. Dermatitis seborreica + tiña corporis
Retroalimentación
La respuesta correcta es: Psoriasis

Pregunta 20
Correcta
Puntúa 1,00 sobre 1,00
Marcar pregunta
Enunciado de la pregunta
Cuánto es la frecuencia de administración de atenolol en cardiopatía isquémica:
Seleccione una:
a. Cada 8 horas
b. Cada 6 horas
c. Cada 24 horas
d. Cada 12 horas
Retroalimentación
La respuesta correcta es: Cada 24 horas

Pregunta 21
Correcta
Puntúa 1,00 sobre 1,00
Marcar pregunta
Enunciado de la pregunta
Qué enfermedad reumatológica causa insuficiencia aórtica:
Seleccione una:
a. Espondilitis anquilosante
b. Artritis reumatoidea
c. Lupus eritematoso sistémico
d. Vasculitis leucocitoclástica
Retroalimentación
La respuesta correcta es: Espondilitis anquilosante

Pregunta 22
Correcta
Puntúa 1,00 sobre 1,00
Marcar pregunta
Enunciado de la pregunta
Paciente de sexo masculino, de 48 años de edad que presenta una tumefacción
eritematosa de bordes mal definidos, muy dolorosa a la palpación en tórax posterior de
consistencia semidura, con un pequeño orificio central a través del cual drena material
purulento de mal olor de 8 días de evolución. ¿Cuál de las siguientes alternativas
indica las bacterias que con más frecuencia ocasionan este cuadro?
Seleccione una:
a. Estafilococo aureus, bacterias anaerobias, gramnegativos
b. Estafilococo aureus, clostridium, pseudomona
c. Estreptococo beta hemolítico del grupo a, pseudomona, h. Influenza
d. Estreptococo beta hemolítico del grupo a, neumococo, eschericia coli
Retroalimentación
La respuesta correcta es: Estafilococo aureus, bacterias anaerobias, gramnegativos

Pregunta 23
Correcta
Puntúa 1,00 sobre 1,00
Marcar pregunta
Enunciado de la pregunta
Tras la infección de un niño con el virus de la Influenza con la siguientes
características: fiebre intensa de inicio súbito, disnea y cianosis a la Rx de tórax se
aprecia patrón asociado con infiltrados intersticiales difusos e hipoxia intensa, usted
sospecharía de:
Seleccione una:
a. Neumonía bacteriana secundaria.
b. Neumonía viral secundaria.
c. Neumonía bacteriana primaria.
d. Neumonía viral primaría.
Retroalimentación
La respuesta correcta es: Neumonía viral primaría.

Pregunta 24
Correcta
Puntúa 1,00 sobre 1,00
Marcar pregunta
Enunciado de la pregunta
El mecanismo por el cual la infección por cólera produce una diarrea tan intensa es
por:
Seleccione una:
a. Vibrios entero hemorrágicos.
b. Toxina Shiga-like
c. Vibrios enteroagregativos.
d. Toxina enterogénica
Retroalimentación
La respuesta correcta es: Toxina enterogénica

Pregunta 25
Correcta
Puntúa 1,00 sobre 1,00
Marcar pregunta
Enunciado de la pregunta
Cuánto es la dosis sublingual de nitroglicerina en angina de pecho:
Seleccione una:
a. 3 a 6 mg
b. 0.3 a 0.6 mg
c. 0.8 a 1.0 mg
d. 1.3 a 1.6 mg
Retroalimentación
La respuesta correcta es: 0.3 a 0.6 mg

PRIMER PARCIAL 9D

Pregunta 1 Correcta Puntúa 0,16 sobre 0,16


Cuál de los siguientes es un signo característico de las vías respiratorias de las
personas que sufren
asma:
Seleccione una:
a. Adhesión de los glóbulos rojos al endotelio
b. Infiltración basófila
c. Transformación de los basófilos en macrófagos
d. Infiltración eosinofílica
La respuesta correcta es: Infiltración eosinofílica

Pregunta 2 Correcta Puntúa 0,16 sobre 0,16


¿Cuál es la clasificación anatómica de las bronquiectasias?
Seleccione una:
a. Cubicas, cilíndricas, alargadas
b. Lobares, arteriales, globulares
c. Vasculares, semilunares saculares
d. Cilíndricas, varicosas, saculares
La respuesta correcta es: Cilíndricas, varicosas, saculares

Pregunta 3 Correcta Puntúa 0,16 sobre 0,16


¿Cuál es la fisiopatología de la hemoptisis?
Seleccione una:
a. Hipervascularización de la circulación brónquica, hipertensión pulmonar y
neovascularización
b. Hipervascularización de la circulación pulmonar, _ebre y regeneración alveolar
c. Hipervascularización de la circulación pulmonar, hipertensión pulmonar y
remodelación.
d. Hipervascularización de la circulación brónquica, hipertensión pulmonar y
disminución de coagulabilidad
La respuesta correcta es: Hipervascularización de la circulación brónquica,
hipertensión pulmonar y neovascularización
Pregunta 4 Correcta Puntúa 0,16 sobre 0,16
De los siguientes factores, cuál es el que debe estar alterado para que un paciente
infectado de tuberculosis se convierta en persona enferma:
Seleccione una:
a. Medio Ambiente: Que viva en un lugar frío
b. Huésped: que genéticamente esté predispuesto a infectarse
c. Medio ambiente: Que haya elevada prevalencia de tuberculosis
d. Huésped: Inmunidad celular deficiente, en especial CD4
La respuesta correcta es: Huésped: Inmunidad celular deficiente, en especial CD4

Pregunta 5 Correcta Puntúa 0,16 sobre 0,16


Dentro de las manifestaciones tardías de la sí_lis tenemos trastornos cardiovasculares
de cual podemos
desprender el siguiente:
Seleccione una:
a. Aneurismas
b. Insuficiencia cardiaca congestiva.
c. Flebitis irritativa.
d. Trastornos del endotelio
La respuesta correcta es: Aneurismas

Pregunta 6 Correcta Puntúa 0,16 sobre 0,16


¿De qué depende la inmunidad contra Leptospira?
Seleccione una:
a. Producción de anticuerpos contra LPS inespecíficos.
b. Producción de anticuerpos contra proteínas inespecíficas de un serotipo.
c. Producción de anticuerpos contra los distintos serotipos.
d. Producción de anticuerpos circulantes contra LPS específicos de un serotipo.
La respuesta correcta es: Producción de anticuerpos circulantes contra LPS
específicos de un serotipo.

Pregunta 7 Incorrecta Puntúa 0,00 sobre 0,16


El hallazgo más frecuente en la radiografía de tórax en un paciente con asma es:
Seleccione una:
a. Engrosamiento de paredes bronquiales.
b. Condensaciones alveolares bilaterales y difusas.
c. Radiografía de tórax normal.
d. Hiperinsuflación pulmonar.
La respuesta correcta es: Radiografía de tórax normal.

Pregunta 8 Correcta Puntúa 0,16 sobre 0,16


El mecanismo por el cual la infección por cólera produce una diarrea tan intensa es
por:
Seleccione una:
a. Vibrios entero hemorrágicos.
b. Toxina Shiga-like
c. Vibrios enteroagregativos.
d. Toxina enterogénica
La respuesta correcta es: Toxina enterogénica
Pregunta 9 Correcta Puntúa 0,16 sobre 0,16
El siguiente enunciado: Un estado parecido al sueño profundo en el que el paciente
permanece con los
ojos cerrados y el paciente no puede ser despertado, se re_ere a:
Seleccione una:
a. Estado vegetativo
b. Estado de coma
c. Somnolencia
d. Estupor
La respuesta correcta es: Estado de coma

Pregunta 10 Correcta Puntúa 0,16 sobre 0,16


El tratamiento de las Bronquiectasias se basa en 3 pilares, excepto:
Seleccione una:
a. Eliminar la obstrucción bronquial.
b. Controlar las infecciones con el uso de antibióticos en las agudizaciones durante 10-
15 días.
c. Revertir el remodelamiento bronquia
d. Mejorar la eliminación de las secreciones, que se consigue con una adecuada
hidratación, con fisioterapia respiratoria y drenaje postural mantenidos.
La respuesta correcta es: Revertir el remodelamiento bronquia

Pregunta 11 Correcta Puntúa 0,16 sobre 0,16


En cuanto a la profilaxis antitetánica en el tratamiento sistemático de las heridas, en
una herida pequeña limpia sin ningún antecedente de vacunación antitetánica Usted
recomendaría:
Seleccione una:
a. Solo limpiar la herida y dar indicaciones de cuidado al paciente, ofrecer la anti
toxina.
b. Dar antibiótico de manera profiláctica.
c. Vacunación antitetánica de inmediato.
d. Es indicativo de inmunoglobulina.
La respuesta correcta es: Solo limpiar la herida y dar indicaciones de cuidado al
paciente, ofrecer la antitoxina.

Pregunta 12 Correcta Puntúa 0,16 sobre 0,16


En cuanto al diagnóstico funcional del asma:
Seleccione una:
a. Si la relación VEF1/CVF es menor a 0.7 (patrón obstructivo) y post broncodilatador
obtenemos un aumento igual o mayor al 12% en el VEF1, nos orienta hacia
diagnóstico de asma
b. Valores espirométricos no tienen importancia en el diagnóstico del asma.
c. Si la relación VEF1/CVF es menor a 0.7 (patrón restrictivo) y post broncodilatador
obtenemos un aumento igual o mayor al 12% en el VEF1, nos orienta hacia
diagnóstico de asma
d. Si la relación VEF1/CVF es mayor 0.7 se considera patrón obstructivo,
diagnosticamos asma
La respuesta correcta es: Si la relación VEF1/CVF es menor a 0.7 (patrón obstructivo)
y post broncodilatador obtenemos un aumento igual o mayor al 12% en el VEF1, nos
orienta hacia diagnóstico de asma

Pregunta 13 Incorrecta Puntúa 0,00 sobre 0,16


En el manejo de un paciente con delirium agitado (o hiperactivo), de las siguientes
opciones, señale la que NO incluiría en su prescripción médica:
Seleccione una:
a. Manejo no farmacológico como: colocar un reloj visible para el paciente
b. Evitar uso de restricciones físicas (amarras) para controlar el movimiento del
paciente
c. Antipsicóticos atípicos como la quetiapina en vía oral
d. Benzodiacepina como el clonazepam en vía oral
La respuesta correcta es: Benzodiacepina como el clonazepam en vía oral

Pregunta 14 Correcta Puntúa 0,16 sobre 0,16


En la clasificación de OMS de adultos con VIH – Sida en el estadio C3 tenemos a:
Seleccione una:
a. Menor de 300 células CD4+ con síntomas definidores de Sida.
b. Mayor de 500 células CD4+ Asintomático.
c. Menor de 200 células CD4+ con síntomas definidores de Sida.
d. Mayor de 400 células CD4+ con síntomas no definidores de Sida.
La respuesta correcta es: Menor de 200 células CD4+ con síntomas definidores de
Sida.

Pregunta 15 Correcta Puntúa 0,16 sobre 0,16


En pacientes con catatonía, usted sospecharía:
Seleccione una:
a. Trastorno psiquiátrico
b. Lesión de la región orbitaria frontal
c. Secuela del coma
d. Herniación central
La respuesta correcta es: Trastorno psiquiátrico

Pregunta 16 Incorrecta Puntúa 0,00 sobre 0,16


En relación a los siguientes enunciados acerca del ictus isquémico o enfermedad
cerebrovascular isquémica, señale el verdadero:
Seleccione una:
a. El tratamiento de HTA como prevención primaria con cifras de TA sistólica &lt;120
mm Hg reduce en 43% la presencia de ictus y ataques del corazón
b. Cualquier fármaco antiplaquetario (aspirina, clopidogrel, ticlopidina) son eficaces y
aprobados para el uso en fase aguda de un ictus isquémico
c. Un paciente con fibrilación auricular valvular (ej enfermedad valvular reumática)
requiere uso de anticoagulantes como prevención primaria o secundaria
d. El uso de estatinas reduce el riesgo de ictus isquémico inclusive con niveles
normales de LDL o niveles bajos de HDL
La respuesta correcta es: Cualquier fármaco antiplaquetario (aspirina, clopidogrel,
ticlopidina) son eficaces y aprobados para el uso en fase aguda de un ictus isquémico

Pregunta 17 Correcta Puntúa 0,16 sobre 0,16


En un paciente de 68 años de edad, con APP: HTA, que ingresa con un cuadro de
crisis focales motoras sin pérdida del estado de conciencia y limitadas al miembro
superior derecho, que se han repetido 1 crisis cada 3 días desde hace 2 semanas,
además se acompaña de cefalea desde hace 3 meses con características de
empeoramiento progresivo, en los estudios de imagen se encontró una lesión
expansiva frontal izquierda; en relación al tratamiento farmacológico de elección,
señale el que
escogería:
Seleccione una:
a. Fenobarbital
b. Clonazepam
c. Lamotrigina
d. Acido valproico
La respuesta correcta es: Lamotrigina

Pregunta 18 Correcta Puntúa 0,16 sobre 0,16


La causa principal de un fracaso en el tratamiento de la tuberculosis pulmonar es
Seleccione una:
a. Toxicidad hepática
b. Resistencia secundaria a las drogas
c. Resistencia primaria a las drogas
d. Abandono del tratamiento por el paciente
La respuesta correcta es: Abandono del tratamiento por el paciente

Pregunta 19 Incorrecta Puntúa 0,00 sobre 0,16


La ictericia en un paciente con Malaria es frecuente a partir:
Seleccione una:
a. La ictericia no aparece en la malaria.
b. La Ictericia en Malaria es frecuente a partir de los 15 años.
c. Siempre existirá fiebre e ictericia debido a la destrucción de eritrocitos.
d. Ocasionalmente en niños debido a su alta susceptibilidad
La respuesta correcta es: La Ictericia en Malaria es frecuente a partir de los 15 años.

Pregunta 20 Correcta Puntúa 0,16 sobre 0,16


Paciente de 16 años de género masculino, sin APP, empieza con crisis convulsivas
tónico clónicas generalizadas hace 1 año, 1 crisis cada 2 meses, su examen
neurológico es normal al igual que la Resonancia Magnética cerebral, el
electroencefalograma demostró actividad epileptiforme, cuál de las
siguientes etiologías consideraría en el diagnóstico
Seleccione una:
a. Epilepsia por desorden genético
b. Epilepsia por enfermedad degenerativa
c. Epilepsia secundaria a autoanticuerpos
d. Epilepsia secundaria a disturbios hidroelectrolíticos
La respuesta correcta es: Epilepsia por desorden genético
Pregunta 21 Incorrecta Puntúa 0,00 sobre 0,16
Paciente de 22 años, género femenino, tiene desde hace 1 año dolor de cabeza de
forma episódica (2 episodios al mes), hemicránea derecha o izquierda, pulsátil,
intensidad moderada, se acompaña de náusea y fotofobia, el dolor es precedido por
escotomas centellantes que duran 90 minutos aproximadamente, en los últimos 3
meses el dolor se ha tornado más frecuente (3 episodios semanales) y al toser
exacerba en intensidad a dolor severo, de los siguientes enunciados cuál NO
considera signo/síntoma de alarma:
Seleccione una:
a. Localización del dolor
b. Empeoramiento del dolor (más frecuente)
c. Valsalva positivo
d. Aura de duración larga
La respuesta correcta es: Localización del dolor

Pregunta 22 Incorrecta Puntúa 0,00 sobre 0,16


Paciente de 38 años de género femenino, con APP diagnóstico de fenómeno de
Raynaud en estudio de un posible Lupus eritematoso sistémico, también diagnóstico
de migraña con aura desde la juventud, acude por un ataque agudo de cefalea, usted
qué fármaco NO recomendaría:
Seleccione una:
a. Clorpromazina
b. AINES
c. Aspirina + metoclopramida
d. Sumatriptán
La respuesta correcta es: Sumatriptán

Pregunta 23 Correcta Puntúa 0,16 sobre 0,16


Paciente masculino de 66 años de edad, con antecedentes de deterioro cognitivo leve,
HTA, ingresa para una cirugía electiva de resección prostática por una hipertro_a
prostática benigna, en su postoperatorio se encuentra con sondaje (cateterización),
vesical permanente, dolor pélvico, y un cuadro de delirium (o sd confusional agudo), se
reinició el enalapril en el postoperatorio, de las siguientes opciones señale cuál NO es
un factor de riesgo para el aparecimiento de delirium
Seleccione una:
a. Dolor en el postoperatorio
b. Administración de enalapril
c. Antecedentes de deterioro cognitivo leve
d. Sondaje vesical
La respuesta correcta es: Administración de enalapril

Pregunta 24 Incorrecta Puntúa 0,00 sobre 0,16


Tras la infección de un niño con el virus de la In􀃕uenza con la siguientes
características: fiebre intensa de inicio súbito, disnea y cianosis a la Rx de tórax se
aprecia patrón asociado con infiltrados intersticiales difusos e hipoxia intensa, usted
sospecharía de:
Seleccione una:
a. Neumonía bacteriana secundaria.
b. Neumonía viral primaría.
c. Neumonía viral secundaria.
d. Neumonía bacteriana primaria.
La respuesta correcta es: Neumonía viral primaría.

Pregunta 25 Correcta Puntúa 0,16 sobre 0,16


Una de las características principales por las cuales el dengue puede diseminarse es:
Seleccione una:
a. Una característica precoz es la trombocitosis y neutrofilia.
b. Debido a la gran capacidad de diseminación del vector Anopheles.
c. El vector es muy cercano a los asentamientos humanos.
d. Es sumamente frecuente la progresión de daño encefálico.
La respuesta correcta es: El vector es muy cercano a los asentamientos humanos.
CUESTIONARIO

INFECTOLOGIA

-
1. Cuál de los siguientes parásitos puede infectar personas y causas cisticercosis?

Tenia solium

2. ¿Cuál de los siguientes fármacos es el indicado para el tratamiento de Infección por


Áscaris Lumbricoides?

Mebendazol 100mg vía oral 2 veces al día por 3 días

3. Cuál es el tratamiento y la dosis para plasmodium falciparum?

Artemeter + Lumefantrina 20mg/120mg y Primaquina 7.5mg dosis única

4. La salmonela es una bacteria:

bacilo anaerobio gram (-)

5. Los fumadores tienen 4 veces más posibiliades para adquirir una neumonía?

'Verdadero'

6. La transmisión del dengue está dada por:

A través de mosquitos Aedes aegypti. Pica en las primeras horas de la mañana y en las últimas
horas de la tarde

7. ¿Qué es el Síndrome de inmunodeficiencia humana?

Etapa más avanzada de la infección por VIH, en la que se ha llegado al agotamiento de las células
CD4+, con un recuento <200 células/mL

8. El virus del Covid- 19 es un virus RNA y su fisiopatología depende de la respuesta


inmunológica de cada individuo?

'Verdadero'

9. ¿Cuál es el vector transmisor del paludismo?

Anopheles

NEUROLOGIA

1. En relación al cuerpo calloso y las crisis epilépticas, señale el enunciado FALSO:

r. Induce la propagación de un flujo excitatorio a áreas corticales más distales

2. De las sustancias que intervienen en dolor, especialmente dolor a nivel craneal como en
Migraña, señale que sustancia NO interviene:

d. Acetilcolina

3. Dentro de los mecanismos de producción de crisis, señale el enunciado FALSO:


Bloqueo del influjo de Calcio y K intracelulares

4. En relación al sistema reticular activante, señale el enunciado FALSO:

Se localiza a nivel diencefálico y supratentorial

5. Señale el enunciado FALSO:

La presión intracraneal elevada siempre se produce a nivel de todos los compartimentos


cerebrales en un mismo momento

6. En relación a cefalea, de los siguientes enunciados señale el FALSO:

La cefalea tipo tensión puede clasificarse en aguda y crónica, aguda si el dolor ocurre de 1-7
días/mes, crónica si ocurre <15 días/mes

7. En relación a la migraña, señale el enunciado falso:

Dentro de los criterios diagnósticos consta: cefalea de intensidad moderada a grave, dolor
holocraneo, no pulsátil, acompañado de aura luego del dolor

8. En relación a los siguientes enunciados, señale el FALSO:

La epilepsia traduce la presencia de una o más crisis no provocadas producido por un proceso
crónico subyacente

9. En relación a la cisticercosis señale el enunciado FALSO:

-
La infección por cisticerco se deriva de la teniasis por Tenia saginata, cuyo hospedario es el ser
humano

10. En relación a la encefalitis, señale el enunciado FALSO:

En caso de comprobación de una encefalitis viral por arbovirus debe iniciarse inmediatamente
tratamiento con Aciclovir intravenoso por 14 a 21 días

11. En relación a las encefalopatías, señale el enunciado FALSO:

Dentro de las causas hidroelectrolíticas se mencionan las hiperpotasemias y las hipermagnesemias

12. En relación a crisis, de los siguientes enunciados señale el FALSO:

Para el tratamiento de Epilepsia con crisis T.C generalizadas como fármaco de primera línea
consta la fenitoína y el fenobarbital

13. En relación a las infecciones del SNC, señale el enunciado FALSO:

Una de las vías de ingreso al SNC por parte de S pneumoniae y N meningitidis es por contigüidad
desde el epitelio nasofaríngeo

14. En relación a la neurocisticercosis, señale el enunciado FALSO:

La determinación de antígenos o anticuerpos por ELISA se los puede realizar, pero no constan en
los criterios diagnósticos

15. De los siguientes enunciados, señale el FALSO:


-
El tramo final de arterias y venas cerebrales penetra al parénquima cerebral, arrastrando a la
duramadre

16. En relación al líquido cefalorraquídeo (LCR), señale el enunciado FALSO:

La hipotensión de LCR produce edema de papila

17. Las ramas de la arteria carótida interna son, señale el EXCEPTO:

Arteria cerebral posterior

18. En relación a los territorios de irrigación a nivel del cerebro, señale el enunciado FALSO:

La corteza visual en el surco calcarino recibe irrigación de la arteria coroidea posterior

19. Paciente que sufrió un cuadro de gastroenteritis aguda y 10 días después, desarrolla
debilidad de las 4 extremidades que evolucionó en el lapso de 4 días respiratoria, en el
examen físico se encontró cuadriparesia fláccida arrefléctica con respuesta plantar flexora,
sin trastorno sensitivo, cuál sería el diagnóstico correcto y la conducta indicada:

Síndrome de Guillain Barré, indicar plasmaféresis

20. En un paciente con diagnóstico de Trauma cráneo encefálico, de acuerdo a su nivel o


categoría de trauma y la acción a realizar, cuál sería el enunciado verdadero:

Trauma severo – TAC cerebral simple

21. En relación a la esclerosis múltiple, señale el enunciado FALSO:

La lesión axonal es el dato patológico patognomónico de la enfermedad

22. Una lesión completa del cordón medular, produce cuadriplejía alta si la lesión se localiza

-
en:

C1-C4

23. En relación al vértigo, señale el enunciado verdadero:

Una causa de vértigo episódico es la migraña vestibular

24. En relación a la hemorragia cerebral, señale el enunciado verdadero:

La hemorragia por coagulopatía puede ocurrir en cualquier localización cerebral

25. En relación a la Hemorragia subaracnoidea, señale el enunciado FALSO:

La etiología más frecuente es la traumática, seguida de la farmacológica (uso de anticoagulación)

26. Paciente masculino de 50 años de edad, fue diagnosticado hace 1 semana de Neuralgia
del Trigémino izquierdo, de los siguientes enunciados cuál es el verdadero:

Esperaría que tenga gatillantes de dolor como cepillarse los dientes

27. En relación al hematoma epidural, señale el enunciado FALSO:

Se localiza entre la tabla interna del hueso y la piamadre

28. En relación a la esclerosis múltiple señale el enunciado FALSO:

En el curso de la enfermedad el más frecuente es la primaria progresiva


29. De los criterios diagnósticos de la sociedad internacional de la cefalea, para el
diagnóstico de migraña sin aura, considera el más importante:

a. Que la cefalea sea de gran intensidad e incapacitante


b. Que la cefalea debe ser siempre unilateral y pulsátil
c. Los episodios deben durar entre 4-72 horas
d. Que se acompañe de vómito y fotofobia
30. Uno de los criterios diagnósticos importantes para la neuralgia del trigémino es:
a. Afecta siempre la división oftálmica del trigémino
b. Ataques paroxísticos recurrentes de hasta 5 minutos de duración
c. Puede encontrarse hipoalgesia en rama del trigémino afectada
d. Sin mejor explicación para otro diagnostico
31. Las crisis parciales motoras se caracterizan por:
a. Afectación de la conciencia
b. Actitud tónica postural de una de las extremidades superiores
c. Pueden ir precedidas por crisis uncinadas
d. Duración menor de 60 segundos
32. Un fármaco de elección para tratar una crisis tónico-clónica generalizada es:
a. Carbamazepina
b. Lamotrigina
c. Valproato de sodio
d. Clonazepan
33. La medida terapéutica inmediata en un paciente de 70 años, hipertenso y con
fibrilación auricular crónica, que presenta un infarto cerebral de 2.5 horas de
evolución, con puntuación de 2 en la escala de NIHSS es:
a. Adecuado manejo de la presión arterial y de la glucosa en sangre
b. Antiarrítmicos
c. Antiagregación plaquetaria dual con aspirina y clopidrogrel
d. Trombolisis intravenosa con activador de plasminógeno tisular 1.1 mg/kg peso
34. Un hallazgo clínico sugestivo de infarto cerebral en la circulación anterior o carotidea
en presencia de una hemiparesia desproporcionada de instauración aguda es:
a. Alteración de estado de conciencia
b. Hemianopsia homónima
c. Parálisis facial periférica contralateral del déficit motor
d. Parálisis del motor ocular común ipsilateral al déficit motor
35. Un signo clínico importante que oriente a considerar un coma como de origen
estructural más metabólico en un paciente en coma es:
a. Respiración de Cheyne Stokes

-
b. Postura de descerebración
c. Glasgow menos a 8
d. Anisocoria
36. Uno de los criterios para considerar la probabilidad de muerte cerebral y proceder a
realizar el test de apnea es:
a. Pupilas midriáticas de 8 mm muy débilmente reactivas a la luz
b. Temperatura corporal de 30 grados centígrados
c. Postura de descerebración luego de una reanimación cardiopulmonar de 10 minutos
d. TC cerebral con una lesión estructural cerebral compatible con muerte cerebral
37. El hematoma subdural por traumatismo cráneo-encefálico:
a. Es debido a ruptura de venas comunicantes entre el cortex y la duramadre
b. Es la lesión focal post-traumática menos frecuente
c. Puede estar asociado a fractura del hueso temporal
d. Requiere tratamiento con corticoides más que quirúrgico
38. Un criterio clínico necesario para establecer el diagnostico de Guillan-Barré es:
a. Paresia facial bilateral
b. Disociación albúmino-citológica en el LCR
c. Incontinencia urinaria como síntoma inicial
d. Arreflexia en los miembros débiles
39. El tratamiento de elección en la forma racemosa y extraparenquimatosa de la

-
neurocisticercosis es:
a. Dexamentasona
b. Quirúrgico
c. Albendazol
d. Praziquantel
40. Una característica de la hemorragia intracerebral primaria por hipertensión arterial es:
a. Van precedidas de crisis epilépticas focales con generalización secundaria
b. Sea asocian de manera frecuente con hemorragia subaracnoidea
c. Localización preferente subcortical
d. Son más frecuentes en pacientes con deterioro cognitivo

41. De los criterios diagnósticos de la sociedad internacional de la cefalea, para el


diagnóstico de migraña sin aura, considera el más importante:

a. Que la cefalea sea de gran intensidad e incapacitante


b. Que la cefalea debe ser siempre unilateral y pulsátil
c. Los episodios deben durar entre 4-72 horas
d. Que se acompañe de vómito y fotofobia
42. Uno de los criterios diagnósticos importantes para la neuralgia del trigémino es:
a. Afecta siempre la división oftálmica del trigémino
b. Ataques paroxísticos recurrentes de hasta 5 minutos de duración
c. Puede encontrarse hipoalgesia en rama del trigémino afectada
d. Sin mejor explicación para otro diagnostico
43. Las crisis parciales motoras se caracterizan por:
a. Afectación de la conciencia
b. Actitud tónica postural de una de las extremidades superiores
c. Pueden ir precedidas por crisis uncinadas
d. Duración menor de 60 segundos
44. Un fármaco de elección para tratar una crisis tónico-clónica generalizada es:
a. Carbamazepina
b. Lamotrigina
c. Valproato de sodio
d. Clonazepan
45. La medida terapéutica inmediata en un paciente de 70 años, hipertenso y con
fibrilación auricular crónica, que presenta un infarto cerebral de 2.5 horas de
evolución, con puntuación de 2 en la escala de NIHSS es:
a. Adecuado manejo de la presión arterial y de la glucosa en sangre
b. Antiarrítmicos
c. Antiagregación plaquetaria dual con aspirina y clopidrogrel
d. Trombolisis intravenosa con activador de plasminógeno tisular 1.1 mg/kg peso
46. Un hallazgo clínico sugestivo de infarto cerebral en la circulación anterior o carotidea
en presencia de una hemiparesia desproporcionada de instauración aguda es:
a. Alteración de estado de conciencia
b. Hemianopsia homónima
c. Parálisis facial periférica contralateral del déficit motor
d. Parálisis del motor ocular común ipsilateral al déficit motor
47. Un signo clínico importante que oriente a considerar un coma como de origen
estructural más metabólico en un paciente en coma es:
a. Respiración de Cheyne Stokes

-
b. Postura de descerebración
c. Glasgow menos a 8
d. Anisocoria
48. Uno de los criterios para considerar la probabilidad de muerte cerebral y proceder a
realizar el test de apnea es:
a. Pupilas midriáticas de 8 mm muy débilmente reactivas a la luz
b. Temperatura corporal de 30 grados centígrados
c. Postura de descerebración luego de una reanimación cardiopulmonar de 10 minutos
d. TC cerebral con una lesión estructural cerebral compatible con muerte cerebral
49. El hematoma subdural por traumatismo cráneo-encefálico:
a. Es debido a ruptura de venas comunicantes entre el cortex y la duramadre
b. Es la lesión focal post-traumática menos frecuente
c. Puede estar asociado a fractura del hueso temporal
d. Requiere tratamiento con corticoides más que quirúrgico
50. Un criterio clínico necesario para establecer el diagnostico de Guillan-Barré es:
a. Paresia facial bilateral
b. Disociación albúmino-citológica en el LCR
c. Incontinencia urinaria como síntoma inicial
d. Arreflexia en los miembros débiles
51. El tratamiento de elección en la forma racemosa y extraparenquimatosa de la

-
neurocisticercosis es:
a. Dexamentasona
b. Quirúrgico
c. Albendazol
d. Praziquantel
52. Una característica de la hemorragia intracerebral primaria por hipertensión arterial es:
a. Van precedidas de crisis epilépticas focales con generalización secundaria
b. Sea asocian de manera frecuente con hemorragia subaracnoidea
c. Localización preferente subcortical
d. Son más frecuentes en pacientes con deterioro cognitivo

53. De los criterios diagnósticos de la sociedad internacional de la cefalea para el diagnóstico


de migraña considera el más importante

a. El número de episodios de cefalea


b. Que la cefalea debe ser unilateral y pulsátil
c. Antecedentes familiares de migraña
d. Que se acompañe de vómito y fotofobia

54. Indicaría la realización de tomografía cerebral simple en un paciente con cefalea, en la


siguiente situación:

a. Cambio del patrón habitual de la cefalea en un paciente con diagnóstico de migraña


b. Primer episodio de tipo pulsátil, con nausea, vómito y fotofobia que dure 4 horas
c. Cefalea a un trauma posterior a un trauma craneal leve, con examen neurológico normal
d. Cefalea acompañada de fiebre y dolor a la presión de los senos paranasales
RESPIRATORIO

-
1.
a)
b)
c)
d)
Señale la respuesta incorrecta respecto a las características del estatus asmático:
Es agudo
Estado inflamatorio crónico
Presencia de sibilancias
Broncoespasmo

2. El asma es una enfermedad respiratoria muy común, cuál es la edad donde se presenta
con más frecuencia?
a) Recién nacido
b) Mujer postmenopaúsica
c) Adulto mayor
d) Niños de 3 años

3. Cual es la causa más común de hemoptisis?


a) Bronquitis aguda
b) Tuberculosis
c) Bronquitis crónica
d) Estatus asmático

4. Cual es el hallazgo más importante a la auscultación pulmonar en caso de insuficiencia


respiratoria
a) Sibilancias
b) Roncus
c) Disminución del murmullo vesicular
d) Crepitantes

5. Cuál es el germen más común a sospechar en una Neumonía extra hospitalaria?


a) Haemophilus influenzae
b) Klebsiella Penumoniae
c) Streptococcus penumoniae
d) S aureus

6. Dentro de los microorganimos que producen Neumonia asociada al uso de ventilador…


Pesudomona Auriginosa
7. Para el tratamiento farmacológico. Cual es el orden adecuado para la antibiótico terapia?
- Amoxicilina + AC --- macrolidos --- cefalosporinas ----- quinolonas ---- carbapenemicos ---
otros (vancomicina, amikacina)
8. Cuando surge dolor pleurítico y disnea en que debemos pensar

Neumotórax espontaneo y embolia pulmonar


9. El patrón clásico de tuberculosis en una RX de torax

Enfermedad del lóbulo superior con infiltrados y cavidades

10. En la tuberculosis extrapulmonar, los sitios con mayor frecuencia son:

Ganglios linfáticos

11. En un paciente con alcoholismo y neumonía extrah. Cual de los gérmenes sospecharía?

Klebsiella Pneumoniae

12. Tratamiento empírico de la neumonía extrah, la doxiciclina esta indicada:

Pacientes ambulatorios que no han recibido Tx en los últimos 3 meses

13. En la enfermedad pulmonar obstructiva crónica:

Se incluye a la bronquitis crónica, enfisema, y enfermedad de las vías respiratorias finas.

14. Respecto al tabaquismo

El elemento más relevante que predice el FEV1 es el número de cajetillas de cigarrillos al año.

15. En la etapa GOLD1, el paciente con EPOC presenta

FEV1 > 80% del valor previsible

16. Uso de corticoides en EPOC descompensado y hospitalizado, ha demostrado

Aumentan la posibilidad de exarcebaciones

17. En la insuficiencia respiratoria tipo 1

Existe insuficiencia hipoxica aguda

18. Paciente con hipoperfusión y choque

Insuficiencia respiratoria tipo IV

ASMA:

1. Cual es un factor endógeno que desencadene los procesos del asma:

a) Aire frío

-
b)
c)
d)
Alérgenos
Atopia
Tabaquismo pasivo
2. En un paciente que acude con una crisis asmática, ¿cuál de los siguientes hallazgos es el que
indica peor evolución?

a) Silencio auscultatorio
b) Taquipnea.
c) Espiración prolongada.
d) Presencia de sibilancias a la auscultación.

3. ¿Cuál es el porcentaje en cuanto a la remodelación del tejido bronquial presentada en la fase


crónica del asma?

a) 5%
b) 10%

-c)
d)
25%
15%

4.- En cuanto al diagnóstico funcional del asma:

a) Si la relación VEF1/CVF es mayor 0.7 se considera patrón obstructivo, diagnosticamos


asma
b) Si la relación VEF1/CVF es menor a 0.7 (patrón restrictivo) y post broncodilatador
obtenemos un aumento igual o mayor al 12% en el VEF1, nos orienta hacia diagnóstico de
asma
c) Si la relación VEF1/CVF es menor a 0.7 (patrón obstructivo) y post broncodilatador
obtenemos un aumento igual o mayor al 12% en el VEF1, nos orienta hacia diagnóstico de
asma
d) Valores espirométricos no tienen importancia en el diagnóstico del asma.
5.- Paciente que presenta dos veces por semana síntomas de asma durante el día, no presenta
síntomas nocturnos, utiliza 4 veces al mes tratamiento de rescate y tiene una función pulmonar
normal, y en el último año no ha presentado exacerbaciones; usted está frente a un paciente:

a) No controlado

-
b) Parcialmente Controlado
c) Controlado

HEMOPTISIS:

1. La hemoptisis es la expulsión por la boca de sangre procedente del aparato respiratorio a nivel
subglótico y dentro de la fisiopatología los factores que influyen son:

a) Hipervascularización de la circulación bronquial, hipotensión del sistema funcional pulmonar,


neovascularización.
b) Hipovascularización de la circulación bronquial, hipertensión del sistema funcional pulmonar,
neovascularización.
c) Hipervascularización de la circulación bronquial, hipertensiondel sistema funcional pulmonar,
neovascularización.
d) Hipovascularización de la circulación bronquial, hipotensión del sistema funcional pulmonar,
neovascularización.
2. Según el SEPAR, la clasificación de hemoptisis es?

a. Expectoración hemoptóica, hemoptisis franca, hemoptisis amenazante.


b. Esputo hemoptóico, hemoptisis mayor, hemoptisis amenazante.
c. Esputo hemoptóico, hemoptisis franca, hemoptisis mayor.
d. Hemoptisis aguda, hemoptisis crónica
3. Una causa extrapulmonar de hemoptisis puede ser:

a) Asma Leve
b) Ca broncogénico
c) Cuerpo extraño
d) TEP

4. En la hemoptisis amenazante está indicado que:


a) Iniciar un estudio diagnóstico, manejo ambulatorio
b) Reposo absoluto con ingreso hospitalario
c) Administración de antitusígenos, control intrahospitalario
d) El paciente debe ingresar a UCI con entubación orotraqueal

5. El tratamiento quirúrgico de la hemoptisis está asociado a:

a) Éxito al 100% por esto se recomienda como primera línea si el paciente lo decide

-
b) Alta tasa de morbimortalidad por lo que es un método de última elección y en casos
específicos
c) Baja tasa de morbimortalidad por lo que es recomendada una vez estabilizado el paciente
d) Poca eficacia y no es recomendada en casos de malformaciones arteriovenosas

BRONQUIECTASIAS:

1. Las Bronquiectasias destruye el componente elástico y muscular de la pared bronquial y su


patogenia corresponde a:

a) Alteración de la motilidad ciliar y aclaramiento, contacto entre bacterias y epitelio, respuesta


inflamatoria sistémica, liberación de proteasa, radicales y daño de epitelio
b) Alteración de la motilidad ciliar y aclaramiento, contacto entre bacterias y epitelio, respuesta
inflamatoria local, liberación de proteasa, radicales y daño de epitelio
c) Alteración de la motilidad ciliar y aclaramiento, contacto entre bacterias y epitelio, respuesta
inflamatoria sistémica y daño de epitelio
d) Alteración de la motilidad ciliar y aclaramiento, contacto entre bacterias y epitelio, respuesta
inflamatoria sistémica.

2. La manifestación clínica más importante en bronquiectasias es:

a) Tos crónica con expectoración matutina tipo mucopurulenta


b) hemoptisis
c) disnea
d) fiebre
3. Respecto a las bronquiectasias cilíndricas, señale lo correcto:
a) 80% estas bronquiectasias están en los lóbulos superiores en la zona apical y posterior
b) 80% estas bronquiectasias están en los lóbulos medios en la zona medial y lateral
c) 80% estas bronquiectasias están en los lóbulos inferiores en la zona dorso- basal y latero-
1111
basal
a) 80% estas bronquiectasias están lóbulos inferiores en la zona basal media
4. Las bronquiectasias se considera una enfermedad Pulmonar Obstructiva Crónica. Cual es un
signo temprano de limitación al flujo aéreo:

a) Una relación FEV1/FVC < 70% con una FEV1 > 80% del teórico.
b) Una relación FEV1/FVC > 70% con una FEV1 > 80% del teórico.
c) Una relación FEV1/FVC < 70% con una FEV1 < 80% del teórico.
d) Una relación FEV1/FVC > 70% con una FEV1 < 80% del teórico.
5. El tratamiento de las bronquiectasias se basa en 3 pilares, escoja la respuesta incorrecta:
a) Eliminar la obstrucción bronquial.
b) Mejorar la eliminación de las secreciones, que se consigue con una adecuada hidratación,
con fisioterapia respiratoria y drenaje postural mantenidos.
c) Controlar las infecciones con el uso de antibióticos en las agudizaciones durante 10-15 días.
d) Revertir el remodelamiento bronquial

DERRAME PLEURAL:

1. A que valor de la cuenta de leucocitos corresponde un empiema pleural?

a) Más de 10 000 leucocitos/mm3


b) Más de 100 000 leucocitos/mm3
c) Más de 6000 leucocitos/mm3
d) Más de 7000 leucocitos/mm3
2. Cuál de las siguientes alternativas es una característica del exudado pleural :

a) La relación entre la cantidad de proteínas del líquido pleural y la del suero es mayor a 0.5
b) La relación entre la concentración LDH en liquido pleural y la del suero es superior a 0.5
c) La concentración de LDH en liquido pleural es superior a 105 UI/L
d) Nada de lo anterior

3. Un hombre acude a su consulta por disnea. En la exploración fisica del tórax se aprecia una
disminución de la movilidad del hemitórax derecho, con disminución de las vibraciones vocales en
ese mismo lado y desviación de la tráquea hacia el lado izquierdo. Además, a la percusión se aprecia
matidez en todo el hemitórax derecho y ausencia de murmullo vesicular en ese mismo hemitórax.
Usted pensaría en:

a) Neumotórax derecho.
b) Atelectasia obstructiva derecha.
c) Condensación pulmonar derecha.
d) Derrame pleural derecho.
4. Paciente de 50 años que presenta un derrame pleural con las siguientes características: aspecto
pajizo, pH 7.3, cociente de proteínas pleura/suero 0.8, cociente de LDH pleura/suero 0.9, Gram y
Ziehl negativos, lípidos totales, colesterol y triglicéridos normales, células mesoteliales <5%, intensa
linfocitosis sin atipias, ADA 64 U/l. ¿Qué diagnóstico le sugiere?

a) Empiema pleural.
b) Derrame pleural por insuficiencia cardiaca (trasudado).
c) Derrame pleural tuberculoso.
d) Derrame secundario a infarto pulmonar.

5. El neumotórax a tensión debe tratarse como una urgencia médica, cuál de las siguientes acciones
es la conducta terapéutica más adecuada:

a) Iniciar ventilación con mascarilla 5L al 100%


b) Administrar adrenalina 1m IV e instalar un tubo de toracotomía
c) Realizar un Rx AP y lateral de toráx para determinar la causa
d) Punción pleural en el segundo espacio intercostal línea medio clavicular

TUBERCULOSIS:

1. La cadena epidemiológica de Transmisión de Tuberculosis consta de:

a) Paciente sintomático respiratorio, mecanismo de transmisión, contacto expuesto

-
b) Agente causal, reservorio (fuente de infección), mecanismo de trasmisión, huésped
susceptible
c) Agente causal, contacto expuesto, reservorio (fuente de infección), paciente sintomático
respiratorio
d) Huésped, agente causal, mecanismo de transmisión, paciente sintomático respiratorio

2. Dentro del mecanismo de transmisión de la tuberculosis, es importante conocer la facilidad con la


que los bacilos se propagan por vía aérea. Indique la cantidad de bacilos liberados mientras una
persona habla, tose y estornuda:

a) Hablando 0-100, tosiendo 100-200, estornudando 200 – 1000 de bacilos liberados


b) Hablando 0 – 100, tosiendo 500, estornudando 5000 o más de bacilos liberados
c) Hablando 0 – 200, tosiendo 0 – 1000, estornudando 2000 – 10000 de bacilos liberados
d) Hablando 0 – 200, tosiendo 0 – 3500, estornudando 4500 – 1millón de bacilos liberados
3. De los siguiente factores, cuál es el que debe estar alterado para que un paciente infectado de
tuberculosis se convierta en persona enferma:

a) Medio ambiente: Que haya elevada prevalencia de tuberculosis


b) Huésped: que genéticamente esté predispuesto a infectarse
c) Medio Ambiente: Que viva en un lugar frío
d) Huésped: Inmunidad celular deficiente, en especial CD4
4. Paciente de 46 años llega a la emergencia refiriendo fiebre, tos con esputo por más de 15 días. A

-
este tipo de paciente lo considera un sintomático respiratorio. Señale verdadero o falso.

a) Verdadero
b) Falso

5. ¿Cuál de los siguientes fármacos antituberculosos puede colorear la orina?


a) Estreptomicina
b) Moxifloxacina
c) Pirazinamida
d) Rifampicina

TROMBOEMBOLIA PULMONAR:

1. El tromboembolismo pulmonar consiste en el enclavamiento en las arterias pulmonares de un


trombo desprendido desde parte del territorio venoso, fisiopatológicamente cuales son los factores
que influyen en su desarrollo?
a) Lesion endotelial, hipercoagulabilidad, estasis venosa.
b) Lesion endotelial, hipercoagulabilidad, estasis arterial.
c) Disminucion de aclaramiento linfático
d) Aumento de producción de líquido pleural
e) Todas son correctas.

2. Es falso sobre la tromboembolia pulmonar:

a) Dependiendo del área de circulación pulmonar bloqueada, aumento de Resistencias


Vasculares
b) Si el bloqueo > 50 %, Hipertensión Pulmonar y Cor Pulmonale Agudo o Colapso
Hemodinámico
c) El diagnóstico de TEPA es confuso porque la presentación clínica puede ser atípica o estar
enmascarada por otra enfermedad coexistente.
d) La muerte súbita es la manifestación clínica inicial en aproximadamente el 15% de los
pacientes con embolismo pulmonar.

3. ¿Qué evalúan los criterios de Wells para tromboembolia pulmonar?

a) La probabilidad clínica de tener tromboembolia pulmonar


b) Diagnóstico definitivo de tromboembolia pulmonar
c) Pronostico de supervivencia para pacientes con tromboembolia pulmonar
d) Criterios radiológicos para diagnosticar tromboembolia pulmonar

4. En el tratamiento de mantenimiento con warfarina en la TEP el seguimiento se hace con?

-
a. Midiendo el INR, si se encuentra en un rango de 2 a 3 significa que está bien
anticoagulado
b. Midiendo el INR, si se encuentra valor menor a 2 está bien anticoagulado
c. Midiendo el INR, si sencuentra valor mayor a 3 está bien anticoagulado
d. Midiendo el INR, si está en un rango de 2 a 3 significa que demos aumentar la dosis de
anticoagulantes
5. En TEP después de un periodo de dosis de heparina de 7-10 días, ¿Cuánto tiempo como mínimo
han de mantenerse los anticoagulantes orales?:

a.
b.
c.
d.
-
4 semanas
3 meses
1 año
Toda la vida

FIBROSIS PULMONAR:

1. Dentro de las fibrosis pulmonares con causa aparente, en el sexo femenino ¿cual es más probable
que sea su etiología?:
a) LES (Lupus Eritematoso Sistémico)
b) Consumo de amiodarona
c) Infeccioso
d) Genético

2. La fibrosis pulmonar puede no tener causa conocida, en cuyo caso se denomina fibrosis pulmonar
idiopática, o puede ser causada por un gran número de variables, en cuyo caso suele remitir al
eliminar el factor desencadenante. Algunos de los factores que pueden asociarse al desarrollo de
fibrosis pulmonar son:
a) Exposición a material particulado de asbesto, polvo de sílice, alveolitis alérgica extrínseca.
b) Medicamentos como la bleomicina, amiodarona, metotrexato, infliximab y etanercept.
c) Enfermedades autoinmunitarias como el lupus, la artritis reumatoide, la sarcoidosis y la
esclerodermia.
d) Todas son correctas.

3. Según los criterios de oxigenoterapia crónica domiciliaria ¿A partir de qué valor de PaO2 en reposo
y respirando aire a nivel del mar es indicación de oxigenoterapia crónica domiciliaria?:

a) Menor o igual a 80 mmHg.


b) Menor o igual a 65 mmHg.
c) Menor o igual a 60 mmHg.
d) Menor o igual a 55 mmHg
4. En la actualidad el manejo de la fibrosis pulmonar está dirigido a evitar los procesos fibrótico e
inflamatorio, típicos de la enfermedad, seleccione el medicamento que posee dichas características:

a) Rituximab
b) Nitrofurantoína

-
c)
d)
Ibuprofeno
Pirfenidona

5. ¿De cuántos años es la supervivencia media de pacientes con fibrosis pulmonar?

-a)
b)
c)
d)
5 años
3 años
1 años
7 años
CARDIOCIRCULATORIO

1. Cual síndrome genético es común en la insuficiencia aórtica?

Síndrome de Marfán

2. Cuanto es la dosis intravenosa de sostén para digoxina en arritmias?

0,125 – 0,250 mg/día

3. Cuanto es la dosis máxima de dinitrato de isosorbide de liberación prolongada en angina

-
de pecho?

300 mg

-
4. Cual es la dosis sublingual de nitroglicerina en angina de pecho?

0,3 – 0,6 mg

-
5. Cuanto es la frecuencias de atenolol en cardiopatía isquémica?

Cada 24 horas

-
6. Cuanto es la frecuencia de propanolol en cardiopatía isquémica?

Cada 12 horas

7. Qué enfermedad reumatológica causa insuficiencia aórtica?

Espondilitis anquilosante

8. Qué infección causa bloqueo auriculoventricular?

1111
Sífilis

9. Un paciente hipertenso con descompensación aguda típica de insuficiencia cardiaca sin

-
sobrecarga de volumen, que fármaco usaría

Nitroprusiato

10. EL INFARTO AGUDO DE MIOCARDIO SIN ELEVACION DEL SEGMENTO ST ES

-
TRIBUTARIO DE FIBRINOLISIS

'Falso'

11. PACIENTE DE 65 ANOS CON APP DE DIABETES MELLITUS MELLITUS TIPO


2,HIPERTENSION ARTERIAL QUE ACUDE POR DOLOR PRECORDIAL,OPRESIVO DE
3 HORAS DE EVOLUCION NAUSEA,VOMITO CON TA 80/40 MAS ALTERACION DEL
ESTADO DE CONCIENCIA. CAMBIOS ELECTROCARDIOGRAFICOS CON
SUPRADESNIVEL DEL ST SIGNIFICATIVO DE V1 A V4

-
ESTE PACIENTE SE CLASIFICARIA COMO

KILLIP 4

12. EL PACIENTE DESCRITO ANTERIORMENTE REQUIERE PROCEDIMINETO DE


ANGIOPLASTIA EN
-
NINGUNA

-
13. LA ELEVACION DEL PRO BNP CONTITUYE UN FACTOR DE MAL PRONOSTICO

'Verdadero'

14. SE TRATA DE UN PACIENTE DE 25 ANOS CON HIPERTENSION DE RECIENTE


DIAGNOSTICO,REFIERE DEBILIDAD GENERALIZADA Y EN EXAMENES DE
LABORATORIO SE EVIDENCIA HIPOCALEMIA,ALCALEMIA METABOLICA. USTED
SOSPECHARIA DE

HIPERALDOSTERONISMO

15. DENTRO DE LAS CAUSAS DE HIPERTENSION SECUNDARIA SE ENCUENTRA LA

-
INSUFICIENCIA SUPRARENAL

'Falso'

-
16. EL USO DE DIGOXINA ESTA INDICADO CON UNA FEVI ENTRE 55 A 60 POR CIENTO

'Falso'

17. LA CREACION DE UNA FISTULA ARTERIOVENOSA PUEDE DESARROLLAR UNA

-
INSUFICIENCIA CARDIACA

ALTO GASTO

18. DENTRO DE LOS EFECTOS ADVERSOS DE LOS DIURETICOS TIAZIDICOS ESTAN


EXCEPTO

HIPERCALCIURIA

19. LA ELEVACION DE CREATININA MAS DEL 30 POR CIENTO CON EL USO DE IECAS O

-
ARA 2 ES UNA INDICACION DE MANTENER EL TRATAMINETO

'Falso'

20. DENTRO DE LAS CAUSAS DE HIPERTENSION SECUNDARIA ESTAN EXCEPTO

HIPOPARATIROIDISMO

21. EN PACIENTES CON ESTENOSIS DE VALVULA MITRAL MAS CRECIMINETO DE


AURICULA MAS FIBRILACION AURICULAR MAS FEVI DE 30 POR CIENTO EL

-
FARMACO DE ELECCION PARA LA FIBRILACION AURICULAR SERIA AMIODARONA

'Falso'

22. ¿Cuál de las siguientes es una causa de hemoptisis?

Seleccione una:

a. Vàrices esofágicas

b. Fibrosis Pulmonar

c. Estenosis Mitral

d. Edema agudo de Pulmòn


23. ¿Cuál es la acción de los nitratos en el síndrome coronario agudo?

Seleccione una:

a. antiagregante

b. vasodilatador coronario

c. revascularizador

d. sedante

24. ¿Cuál es la función de la aspirina en el síndrome coronario agudo?

Seleccione una:

a. Anticoagulantes

b. Evitar el incremento de la agregación plaquetaria en la placa ateromatosa accidentada

c. Analgésico antin􀃖amatorio

d. Trombolisis

25. ¿Cuáles son signos de insuficiencia cardíaca derecha?

Seleccione una:

a. Ortópnea, dolor precordial

b. Edema de miembros inferiores, hepatomegalia

c. Dolor abdominal, astenia

d. Disnea, estertores crepitantes

26. El uso de betabloqueantes debe ser analizado en el paciente con insuficiencia cardiaca ya
que este grupo farmacológico posee ciertas características importantes. Señale lo correcto
respecto a estos productos:

Seleccione una:

a. Puede utilizarse con seguridad en pacientes con asma

-
b. Pueden utilizarse con seguridad en pacientes con insuficiencia cardiaca sistólica y angina de

esfuerzo

c. Pueden utilizarse con seguridad en pacientes con hipotensión sintomático

d. Pueden utilizarse con seguridad en combinación con verapamilo

27. ¿En qué caso complementamos el electrocardiograma con derivaciones derechas?

Seleccione una:
a. Sospecha de infarto de ventrículo derecho

b. Taquicardia ventricular monomorfa

c. Ritmo de galope

d. Sospecha de edema agudo de pulmón

28. María, 65 años, acude a la consulta re_riendo cansancio a los moderados esfuerzos, re_ere
haber tenido fiebre reumática, al examen físico presenta PA 130/80, FC 88lpm, soplo sistólico en
quinto espacio intercostal línea media clavicular izquierda III/VI, con irradiación a la axila. ¿Cuál es
el diagnóstico más probable en esta paciente?

Seleccione una:

a. Estenosis mitral

b. Insu_ciencia aortica

c. Insu_ciencia mitral

d. Estenosis aortica

29. Qué antihipertensivo se debe usar con cautela en un paciente con arteriopatia coronaria grave:

-
Seleccione una:

a. Hidralazina

b. Clonidina

c. Atenolol

d. Espironolactona

30. Qué endocrinopatía es causa secundaria de hipertensión arterial:

Seleccione una:

a. Hipogonadismo

b. Hiperprolactinemia

c. Síndrome de Cushing

d. Hipocalcemia

31. Causa de taquicardia sinusal fisiológica

Feocromocitoma

32. síndrome genético común en la insuficiencia aortica

Síndrome de marfan
NEFROLOGÍA

1. La furosemida que categoria de riesgo en gestantes tiene?

Seleccione una:

a. A

1
b. C

c. D

d. B

2. Como se encuentra la creatinina sérica en RIFLE 2

Seleccione una:

a. Triple

b. Cuádruple

-
c. Quíntuple

d. Doble

3. Si la evidencia cientíca aprobaría el uso de tiazidas en gestantes, ¿En cuánto tiempo


aproximadamente se observaría la efacacia antihipertensiva del uso de las tiazidas?

Seleccione una:

-
a. 12 semanas

b. 14 a 30 días

c. 2 meses

d. 1 semana

4- Cuánto de sodio en una hipernatremia se requiere para producirse hemorragias cerebrales?

Seleccione una:

a. Mas de 145 mEq/L

b. Mas de 125 mEq/L

c. Mas de 135 mEq/L

d. Mas de 160 mEq/L

5. El síndrome de guillain barre desencadena?

Seleccione una:

a. Acidosis metabólica
b. Alcalosis metabólica

c. Alcalosis respiratoria

d. Acidosis respiratoria

6. El valor de oliguria de_ne?

Seleccione una:

a. 100 a 400 ml

b. 1000 a 1500 ml

c. 0 a 50 ml

d. 500 a 1000 ml

7. La fiebre por meningitis que trastorno ácido básico produce?

Seleccione una:

a. Alcalosis metabólica

b. Alcalosis respiratoria

c. Acidosis metabólica

d. Acidosis respiratoria

8. Cuántos mEq de Na contiene 500 ml de Cloruro de sodio al 0.09%?

Seleccione una:

-
a. 130 mEq

b. 154 mEq

c. 87 mEq

d. 250 mEq

9.Cuál es el peligro del uso de espironolactona en gestantes?

Seleccione una:

a. Aumento del flujo placentario

b. Retención de sodio

c. Estimulación de la aldosterona

d. Efecto antiandrogénico en fetos masculinos


10.La diarrea produce?

Seleccione una:

a. Acidosis metabólica

b. Alcalosis respiratoria

c. Alcalosis metabólica

d. Acidosis respiratoria

11.Cuál NO es un sistema amortiguador del equilibrio ácido base?

Seleccione una:

a. Bicarbonato

b. Fosfato

c. Hemoglobina

d. Proteinas

12. En que valores se encuentra una hiponatremia leve?

Seleccione una:

a. 115 a 125 mEq/L

b. 105 a 115 mEq/L

c. 95 a 105 mEq/L

d. 125 a 135 mEq/L

13. Cuál NO es una posibilidad en el periodo de resolución de una azotemia aguda?

Seleccione una:

a. Muerte por coma urémico

b. Evolución a una IRC.

c. Reagudización de lesión renal permanente

d. Reversibilidad total

14. Cuál es el tratamiento de la hipermagnesemia grave?

Seleccione una:

a. Bicarbonato de sodio al 10%

b. Cloruro de calcio al 10%


c. Gluconato de calcio al 10%

d. Carbonato de calcio al 10%

15. Si existen una intoxicacion crónica por Sal Andrews que trastorno ácido básico se produce?

Seleccione una:

a. Alcalosis metabólica

b. Acidosis metabólica

c. Alcalosis respiratoria

d. Acidosis respiratoria

16. no se considera causa para azotemia prerrenal

Hiperplasia prostática

17. cual no se considera un tratamiento para hiperpotasemia

Bicarbonato de calcio

19. El síndrome de Alport que nefropatía prodce

Glomerulonefritis segmentaria focal

20. Que valor define anuria?

0 a 100 ml

21. Los favtores que predisponen a las mujeres a padecer cistitis, aumentan el peligro de

Pielonefritis

22. Cuál glomerulopatía no produce daño vascular: Seleccione una:

a. Crioglobulinemia

b. Nefropatía por IgA

c. Nefritis lúpica

d. Granulomatosis de Wegener

23. Cuál no es una glomerulopatía de vasos pequeños ANCA: Seleccione una:

a. Púrpura de Henoch-Schönlein

b. Síndrome de Churg-Strauss

c. Poliangitis microscópica
d. Granulomatosis de Wegener

24. La clase IV de la nefritis lúpica se llama: Seleccione una:

a. Nefritis difusa

b. Proliferación mesangial

c. Nefritis esclerótica

d. Nefritis focal

25. La hepatitis B que nefropatía produce: Seleccione una:

a. Nefropatía por IgA

b. Glomerulonefritis membranoproliferativa

c. Nefritis lúpica

d. Granulomatosis de Wegener

26. La obstrucción ureteral bilateral aguda se caracteriza por: Seleccione una:

a. Disminución del flujo sanguíneo medular

b. Disminución de la liberación de óxido nítrico

c. Disminución de la producción de angiotensina

d. Disminución de prostaglandinas vasodilatadoras

-
27. La obstrucción ureteral bilateral crónica se caracteriza por: Seleccione una:

a. Hipertensión

b. Anuria

c. Diuresis posterior a la obstrucción

d. Dolor renal

28. La proteinuria sostenida es aquella que se expulsan más de: Seleccione una:

a. 3 a 4 g/24 h

b. 7 a 8 g/24 h

-
c. 5 a 6 g/24 h

d. 1 a 2 g/24 h
29. En que valores se encuentra una hiponatremia leve? Seleccione una:

a. 115 a 125 mEq/l

b. 125 a 135 mEq/l

c. 105 a 115 mEq/l

d. 95 a 105 mEq/l

30. Cistitis recurrente: 2 semanas.


31. Cálculo en uréter: labio mayor ipsilateral.
32. No es biomarcador: adiponectina.
33. Microorganismo virulento para ITU por propagación hematógena postmenopáusica:
estafolococo aureus.
34. AKIN 3: descenso de FG superior a 75%
35. MDRD: dieta modificada en la enfermedad renal.
36. No es un factor independiente de riesgo para cistitis no complicada en mujeres: uso
reciente de T de cobre.
37. No es un factor anatómico que retrasa el vaciamiento vesical para ITU en
postmenopausicas: incontinencia urinaria, estenosis ureteral.

-
HEMATO

Ó GUÍA DEL DONANTE


. DE - ...

GRUPO PUEDE DONAR A PUEDE RECIBIR DE


A+ A+,AB+ A A--, Di-, D-
O+ A+, O+, B+, AB+ O+, 0-
B
AB+
A-
0-
8-
B ,AB
AB
__
A+,A-,AB+,AB-
TODOS
8+, B-,AB+,AB-
B ,S-,0+,0-

____..._ A- Q..__
-- TODOS
_.._

0-
B-,0-
AB- AB AB-
, A- O- 8- AB-
I I t
,,
CCf:I ON ll>E LOS
DOS
~E.;;RIVADOS PLASMÁncos
--■o

, a;m .
Receptor ).\ ciecu21d21 Opdón 1 Opción 2

ION DE
~ tlVADOS
CONCENTRADO GLOBULAR
Grupo
Receptor Adecuada Opción 1 Opción 2

o o
A A o -
e B o -
AB AB AoB* o
1. Un paciente grupo A (+) sufrió quemaduras en un 30 % de su cuerpo amerita una
transfusión de Plasma.

¿Cuál de estos derivados plasmático puede recibir?

Seleccione una:

a. Plasma grupo (O+)

b. Plasma grupo B

c. Plasma grupo (O-)


d. Plasma grupo AB

2. Un paciente con un 30% de quemaduras en su cuerpo, presenta edema generalizado tipo


anasarca, se le indica cumplir Albumina. ¿Porqué?

Seleccione una:

a. Hace que las plaquetas actúen más activas

b. Estimula la célula endotelial en sus funciones.

c. Ayuda al proceso infeccioso de las lesiones

d. Para reponer la disminución de las proteínas y presión coloidosmótica

3. El plasma es la parte líquida de la sangre separada de la parte celular, señale su indicación


en estas patologías

Seleccione una:

a. Hemofilia A

b. Para ayudar a cicatrizar heridas

-
c. El niño con PTI con petequias

d. Hemofilia B

4. ¿Cuál es el grupo sanguíneo Donante Universal, pero no puede recibir de ningún otro
grupo?

Seleccione una:

a. Grupo AB+

-
b. Grupo AB -

c. Grupo O -

d. Grupo O +

5. Cuando cumplimos un concentrado globular para tratar una anemia, ¿Cuánto aumenta la
hemoglobina por 1 unidad de concentrado transfundido?

Seleccione una:

a. Aumenta 1.5 g/dl y 8% del hematocríto

b. Aumenta 2.5 g/dl y 4% del hematocríto

c. Aumenta 3 g/dl y 12% del hematocríto

d. Aumenta 1.5 g/dl y 4% del hematocrito


6. Una pareja del grupo O. ¿Qué posibilidad tiene de tener un hijo con este grupo?

Seleccione una:

a. 25% de probabilidad de tener un grupo B

b. La posibilidad 100% un hijo grupo O

c. 25% de posibilidad de tener un hijo grupo A

d. Otro 25% del grupo AB

7. ¿Cuál de estos fármacos tiene acción potenciadora de los anticoagulantes?

Seleccione una:

a. Barbitúrico

b. Rifampicina

c. Vitamina K

d. Ácido acetíl salicílico

8. Después de una transfusión, puede ocurrir una reacción adversa, y estas pueden ser
inmediatas o tardías. ¿Cuál de estas es una reacción transfusional tardía?

Seleccione una:

a. Reacción alérgica

b. Fiebre y escalofrío

c. Enfermedad Injerto contra huesped

d. Reacción hemolítica aguda

9. ¿Cuál es la causa más frecuente de Trombo_lia congénita?

Seleccione una:

a. Déficit de proteína S

b. Déficit de antitrombina III

c. Déficit de proteína C

d. Resistencia a la Proteína C

10. Un paciente con una patología de sistema nervioso central. ¿Cuál de estos medicamentos

está contraindicado?
Seleccione una:

a. Heparina de bajo peso molecular

-
b. Esteroides

c. Fibrinolítico

d. Antihipertensivo

Respuesta correcta

11. Un joven de 14 años con signos de varicela. ¿Cuál de estos medicamentos está
contraindicado su uso?

Seleccione una:

a. Ácido acetíl salicílico

b. Paracetamol

c. Ibuprofeno

d. Aciclovir

12. La resistencia a la Proteina C, es la más frecuente de los cuadros trombóticos congénitos


¿Cuál es la

causa fisiopatológica?

Seleccione una:

a. Por la mutación del factor V Leidern

b. Inhibe a la antitrombina III

c. Inhibe la _brinolisis

d. Resistencia a la proteína S

13. Los trombos arteriales son trombos blancos se producen por lesión endotelial. ¿Por qué en
esta trombosis está indicado los antiagregantes?

Seleccione una:

a. Ayuda a la acción de la _brinolisis

b. Porque estos trombos están compuestos principalmente por plaquetas

c. Porque ellos rompen la _brina

d. Ayuda a la acción de los anticoagulantes

Respuesta correcta

La respuesta correcta es: Porque estos trombos están compuestos principalmente por plaquetas
14. ¿Responda cuál anticoagulante está contraindicado en el embarazo?

Seleccione una:

a. Las nuevas heparinas

b. Heparina no fraccionada (HNF)

c. La warfarina sódica

d. Heparina de bajo peso molecular (HBPM)

15. El índice de producción de reticulocitos (IPR), clasi_ca las anemias en arregenerativas y


regenerativas. Un IPR >2, ¿Cuál de estas anemias se presenta?

Seleccione una:

a. Anemia por dé_cit de hierro

b. En Hipoplasia medular

c. Anemia de los procesos Crónico

d. En una hemorrágia aguda

Respuesta correcta

La respuesta correcta es: En una hemorrágia aguda

16. ¿Cuáles son los criterios de hipoplasia medular severa?

Seleccione una:

a. Petequias, las mucositis

b. Reticulocitos <de 1 corregido, neutro_los < de 500 cifras absolutas

c. La _ebre y anemia

d. Epistaxis, sangrado por mucosas

17. Las anemias microangiopáticas,, son anemias agudas graves como la CID, SUH.
intravasculares, que datos morfológicos en el frotis la identi_can?

Seleccione una:

a. Hematíes fragmentados (esquistocitos)

b. Cuerpos de Heinz

c. Células en diana

d. Polisegmentación de neutro_los

Respuesta correcta

La respuesta correcta es: Hematíes fragmentados (esquistocitos)


18. Unas reacciones alérgicas tipo urticaria o ana􀃖lácticas que reincide. ¿Qué estudio
debemos pedir a ese paciente?

Seleccione una:

a. Fosfatasas alcalina

b. Deshidrogenasa Láctica (DHL)

c. Proteína C in􀃖amatoria

d. Anticuerpo Ig A por de􀃖ciencia

Respuesta correcta

La respuesta correcta es: Anticuerpo Ig A por de􀃖ciencia

19. La hipersensibilidad tipo III o enfermedad del suero, es una reacción que se da por una
transfusión o por antibióticos, vacunas, como se diferencia clínicamente de la Enfermedad
injerto contra huésped (transplante), porque ambas son parecidas.

Seleccione una:

a. Por la 􀃖ebre y la diarrea

b. Por la hemólisis que se presenta

c. Erupción cutánea, urticaria

d. Por la presencia de adenomegalia, glomerulonefrítis, vasculitis

Pregunta 14 Incorrecta Puntúa 0,00 sobre 1,00

¿Cuándo se habla de reacción transfusional aguda?

Seleccione una:

a. A los 7 días después.

b. Aquella que ocurre durante la transfusión hasta 24 horas

c. Después de las 24 horas

d. La que ocurre durante la transfusión hasta 24 horas

20. ¿Cuál es la reacción más frecuente que se presenta en una transfusión sanguínea?

Seleccione una:

a. La hemolítica crónica

b. La alérgica, urticaria

c. La reacción febril no hemolítica


d. La hemolítica aguda

21. La de_ciencia de ácido fólico, además de producir anemia. ¿Qué complicación produce en
el recién nacido?

Seleccione una:

a. Luxación congénita de cadera

b. Defectos en el tubo neural

c. Onfalitis

d. Ictericia

Respuesta correcta

22. Cual de estas patologías con sangrado purpurico se identifica en su frotis da sangre
periférica por la presencia de plaquetas gigantes?

Enfermedad de Bernard Soulier

23. El IPR es un estudio que mide la capacidad de respuesta de la medula. Si tenemos un IPR
menor de 2. Que anemia es?

Hipopalsia medular

24. La heparina no fraccionada su uso puede causar algunas complicaciones por su acción
anticoagulante, sobre cual factor actua principalmente?

Factor ii prototrombina

25. Los productos de degradación de fibrina podemos medirlos a través de Dimero D , su


contaje elevado mayor de 1000 (5000) indicaria

Tromboembolismo pulmonar

26. Si se necesita un aumento de plaquetas mas efectivo en una trombocitopenia severa


inmune. Que utilizo de primera línea?

Inmunoglobulina

27. Causa de riesgo de Trombosis adquirida tanto venosa como arterial, la cual amerita
tratamientoanticoagulante? Seleccione una:

a. Embarazo

b. Inmovilización

c. Síndrome Antifosfolipídico

d. Cirugía mayor

28. ¿Cuál es la reacción transfusional más frecuente? Seleccione una:

a. Reacción febril no hemolítica


b. Reacción alérgica

c. Reacción Hemolítica

d. Reacción Anafiláctica

29. El riesgo de contaminación Bacteriana debido a una transfusión ha aumentado conforme


disminuye el riesgo de infecciones virales, muchas bacterias no proliferan en frío. Cuál de estos
derivados representa el mayor riesgo de contaminación. Seleccione una:

a. Concentrado de glóbulos rojos

b. Críoprecipitado

c. Concentrado de plaquetas

d. Plasma

30. La fragmentación de Eritrocitos (Esquitocitos) imagen en sangre periférica que observamos en


pacientes con: Seleccione una:

a. Anemia por trastorno de maduración (A. megaloblástica)

b. Válvulas Cardíacas Mecánicas

c. Daño medular. Hipoplasia

d. Déficit de Hierro

31. Los Eritrocitos jóvenes que con la coloración de azul de metileno observamos restos de ARN y
su aumento indica buena respuesta de la médula ante una anemia. Cuál célula nos referimos?

-
Seleccione una:

a. Reticulocitos

b. Células en Diana

c. Esquitocitos

d. Cuerpos de Howel Jolly

-
32. Paciente con reacción transfusional Anafiláctica a repetición. Debe ser estudiado para
descartar: Déficit de IgA

33. Una anemia con un Índice de Producción de Reticulocítos ( IPR ) mayor de 2 % es una anemia
hiperproliferativa . Cuál sería la causa de anemia? Seleccione una:

a. Disfunción Renal

b. Hemorragia y Hemólisis

c. Hipoplasia medular

d. Déficit de hierro

34. El concentrado plaquetario constituye el derivado con mayor riesgo de


contaminación. ¿Cuál es la causa? Seleccione una:

a. Se mantiene a 6 grados en nevera

b. Se encuentra a 35 grados de temperatura.


c. Debido a mantener a -20 grados en congelación

d. Por estar a 26 grados de temperatura en el ambiente

35. Este paciente con quemaduras le van a cumplir posteriormente plasma, el paciente
es grupo O (-) Bancod e sangre envía un plasma. ¿Cuál de estos podemos
cumplir? Seleccione una:

a. Plasma O +

b. Plasma A +

c. Plasma B +

d. Plasma A B –

36. La anemia hemolítica microangiopática es un cuadro clínico, que se presenta en


patologías como la coagulación intravascular diseminada, la presencia de
esquitocitos en el frotis periférico es una imagen característica. ¿Qué representa?
Seleccione una:

a. Hematíes fragmentados

b. Reticulocitos

c. Fibrina fragmentada

d. Buena respuesta de la médula

37. Un paciente con quemaduras de un 30% de su cuerpo, presenta además de sus


lesiones un edema importante. ¿Cuál de estos derivados pedirías para ayudar a
resolver el edema? Seleccione una:

a. Concentrado plaquetario

-
b. Concentrado globular

c. Albumina

d. Inmunoglobulina

DIGESTIVO Y ENDOCRINO

1. Un paciente con una glucemia de 300 mg/dl puede referir los siguientes signos y sintomas.

- Señale lo correcto: Polidipsia, astenia, calambres musculares.


2. Carlos de 18 años sin antecedentes de importancia acude a la sala de emergencia con
aliento a manzana, somnoliento, glucosa de 300 mg/dl, se le realiza gasometría y se
encuentra un Ph de 7. Bicarbonato, cetona en orina ++ ¿Cuál es su apreciación
diagnóstica?: Cetoacidosis diabética con debut diabético.
3. Carlos de 50 años, fumador, con obesidad, que presenta hace 2 semanas polidipsia,
polifagia, poliuria, pérdida de peso. ¿Qué examen como parte de los criterios de la ADA
nos daría el diagnostico de diabetes?
Glucosa mayor de 200 tomada al azar

Glucosa mayor a 126 mg/dl durante prueba oral de tolerancia a la glucosa.

4. En un paciente con diabetes mellitus tipo 2, IMC mayor de 30 ¿Cuál sería el antidiabético
oral más indicado? Metformina
5. Niña 11 años es hospitalizada por presentar nerviosismo y palpitaciones de 24 horas. El
apetito ha sido bueno…… al examen físico FC 100lpm, T 37,5 transpiración excesiva, ROT
rápidos. Los ojos presentaban una apariencia vidriosa ¿Cuál es el diagnóstico MÁS
probable? Diabetes mellitus tipo 1
6. Mujer de 68 años de edad en estado de coma. Al examen PA 90/50 mmHg, Urea 56 mg,
Creatinina 2,5 mg, glucosa , Na 130 mEq, Bicarbonato 23 mEq, Ph 7,32. Volumen urinario
de 24 horas: 450 ml. El diagnostico MAS probable es: Coma por cetoacidosis diabética.
7. Hombre de 64 años que acude a chequeo general por fatigabilidad fácil de unos meses de
evolución y pérdida de apetito, aunque sin pérdida de peso. Nos llama la atención, al
abservarla, facies de anima, edema palpebral y una llamativa falta de cola de la ceja. Con
el interrogatorio y la exploración clínica, se nos ocurre una hipótesis diagnostica ¿Cuál de
las pruebas analíticas crees que es mejor? Hemograma, hierro, ferritina.
8. Hombre de 54 años que acude a chequeo general. Se detecta IMC de 32, 8 kg/m2 y
glucemia en ayunas, después de 123 mg/dl ¿Qué recomendación terapéutica efectuaría en
primer lugar? Cambios conductuales, dieta, y ejercicio físico.

-
9. ¿Cuáles no son causas de hipotiroidismo congénito? Déficit de ingesta de yodo
10. ¿Cuál es la relación entre el yodo y la tiroides? El yodo es utilizado por la tiroides para
fabricar sus hormonas
11. Muchacha de 26 años hospitalizada por ictericia de instauración reciente, asociada a dolor
en hipocondrio derecho. Se detecta hepatomegalia sensible sin esplenomegalia. Hay

-
telangectasias faciales. Reconoce antecedentes de promiscuidad sexual, consumo de
drogas intravenosas. AST 135, BT 16 mg/dl, ALT 110, GCT 680, FA 280, Trgliceridos 600
colesterol 280. Eco: patrón ecogenico del hígado. DX probable: Hepatitis vírica.
12. Cual es TX de elección de la colitis pseudomembranosa que cursa con ileo paralítico?
Metronidazol por vía intravenosa
13. Cual hallazgo histológico es necesario para establecer el diagnóstico de cirrosis hepática.
Presencia de fibrosis.
14. ¿Cual de las siguientes alternativas es el carcinoma que prevalece en el tercio superior del
esófago? Carcinoma escamoso.
15. Mujer de 25 años refiere una historia de 12 meses de cólicos abdominales recurrentes en
hemiabdomen inferior acompañado de distensión abdominal y diarreas. No presenta fiebre,
pérdida de peso ni deposición con sangre. Esta paciente cumple con los criterios ROMA IV
para Sd de colon irritable con predominio de diarrea, no presenta signos de alarma ¿En
qué patología usted puede pensar? Enfermedad celiaca
16. El esofago de Barret se carateriza por: presencia de metaplasia intestinal
17. Respecto a la cirrosis hepática, escoja la respuesta correcta: las complicaciones son
consecuencia de la hipertensión portal y la insuficiencia hepática.

-
18. Hombre 69 años con hepatitis vírica HVC y cirrosis sin antecedentes previos de
hemorragia variceal, acude por el hallazgo endoscópico de várices esofágicas grado III

- ¿Cuál es el agente farmacológico para la prevención del sangrado? Carvedilol

-
19. Ejecutivo de 55 años, con dolor epigástrico intenso, en la endoscopia gastrointestinal se
encuentra una ulcera grande en el bulbo duodenal. Se indican una dieta e inhibidor de
bomba de protones, pero los síntomas persisten. Sugerencia apropiada: tratamiento de
erradicación del Helicobacter Pylori
20. Con respecto a la disfagia motora, señale lo correcto. La causa más común es la estenosis
pilórica.
--
21. La ubicación de la ulcera que presenta mayor sangrado es. Ulcera gástrica de curvatura
menor.
22. Que endocrinopatía no causa prolongación del QT y de taquicadia ventricular polimorfa:

-
Hipogonadismo
23. Paciente de 50 años, con obesidad. DM II de 10 años de evolución, función renal
preservada e infarto agudo de miocardio. Mejor combinación farmacológica: Metformina/
Inhibidores SGLT-2
24. Evalue la siguiente función tiroidea. TSH 8. T4L 1,15 ¿Cuál de las siguientes alternativas
corresponde? Hipotiroidismo primario subclínico
25. Evalue …..: TSH 15, T4L 0,70 ¿Cuál corresponde? Hipotiroidismo tiroideo clínico

-- - -
26. En un paciente de 80 años con cardiopatía isquémica y función tiroidea: TSH 12, T4L 0,85.
Dosis de levotiroxina: 50 mcg
27. Cual es la causa mas frecuente de Hiperth en una mujer de 25 años? Enfermedad de

-
Graves
28. Causa mas frecuente de hipertiroidismo en una mujer de 70 años? Bocio tóxico.

-
29. Cuál es la primera causa endocrina de retraso de crecimiento: hipotiroidismo.
30. En un paciente de 25 años de edad con DM I y clínica de I Suprarrenal ¿Cuál es el test
diagnóstico? Cortisol basal.
31. Paciente a quien se le administra grandes cantidades de cortisol por un periodo prolongado

- ¿Qué acción se verá incrementada? Concentración de glucosa plasmática


32. Cuales son los agentes antirresortivos del hueso en osteoporosis? Estrógenos,
bifosfonatos, vitamina D, y calcitriol.
33. Paciente acude con trastornos osteomusculares crónicos, junto con déficit visual
inespecífico, pérdida de peso, y fiebre de larga evolución. El paciente proviene de una
zona meramente ganadera. Usted sospecharía de brucelosis cuya base de tratamiento es:
dicloxacilina y estreptomicina.
34. Masculino de 52 años consulta por poliuria, polidipsia intensa y pérdida involuntaria de 10
kg de peso; es diagnosticado de diabetes mellitus por una glucemia plasmática de 322
mg/dL y una HbA1c de 9,8%. Se le recomienda dieta, ejercicio físico, e iniciar tratamiento
con metformina 850 mg cada 12 horas y glimepirida 6 mg/dia. En las semanas siguientes
los controles glucémicos se van reduciendo progresivamente. A los 4 meses la glucemia es
de 94 mg/dL y la HbA1c de 5,9%. El paciente se queja de episodios frecuentes de pérdida
del equilibrio “mareo”, dolor epigástrico, visión borrosa, sudoración y temblor, que mejoran
con la ingesta de alimentos y que ocurren sobre todo al... Suspender la sulfonilurea

DERMATOLOGÍA

1. Cuál de las siguientes alternativas es el tratamiento sistémico de elección en impétigo


contagioso? Dicloxacilina
2. Paciente masculino 48 años, presenta tumefacción eritematosa de bordes mal definidos,
muy dolorosa a la palpación, en tórax posterior de consistencia semidura y con un pequeño
orificio central a través del cual drena material purulento de mal olor de 8 días de
evolución. ¿Cuál de las siguientes alternativas indica las bacterias que con más frecuencia
ocasionan este cuadro? Estafilococo aureus, bacterias anaerobias, gramnegativos.
3. Masculino 48 años…. ¿Cuál de las siguientes alternativas indica los antibióticos de
elección en este caso? Dicloxacilina y Trimetropin sulfametoxazol.
4. Femenina de 30 años que presenta desde la infancia placas eritematosas descamativas de
bordes bien definidos en codos y rodillas, con descamación intensa y prurito moderado,
además presenta eritema y descamación profusa en cuero cabelludo ¿Cuál de las
siguientes alternativas corresponde al diagnóstico clínico más probable? Psoriasis
5. Femenina de 30 años que presenta… ¿Cuál de las siguientes alternativas corresponde al
tratamiento tópico? Corticoesteroides tópicos asociados a calcipotriol
6. El dermatofito más común en la tiña pedia interdigital es, señale la alternativa correcta.
Tricophyten Mentagrophytes.
7. En la urticaria crónica autoinmune un medicamento de gran utilidad con pocos efectos
secundarios es: Seleccione una: Ciclosporina
8. En la urticaria crónica resistente a antihistaminicos anti h1 se puede utilizar. Seleccione
una: Ciproheptadina.

-
9. Para el manejo de infecciones dermatológicas la Penicilina G es uno de los betalactámicos
más eficiente por su gran actividad sobre Pseudomonas, responda verdadero o falso.
FALSO
Cuál de los siguientes es un signo característico de las vías respiratorias de las personas que
sufren asma:
Seleccione una:

a. Infiltración eosinofílica
b. Transformación de los basófilos en macrófagos
c. Adhesión de los glóbulos rojos al endotelio
d. Infiltración basófila
Retroalimentación
La respuesta correcta es: Infiltración eosinofílica

Pregunta 2
Correcta
Puntúa 0,16 sobre 0,16

Marcar pregunta
Enunciado de la pregunta

¿Cuál es la clasificación anatómica de las bronquiectasias?


Seleccione una:
a. Cubicas, cilíndricas, alargadas
b. Lobares, arteriales, globulares
c. Vasculares, semilunares saculares

d. Cilíndricas, varicosas, saculares


Retroalimentación
La respuesta correcta es: Cilíndricas, varicosas, saculares

Pregunta 3
Correcta
Puntúa 0,16 sobre 0,16

Marcar pregunta
Enunciado de la pregunta

¿Cuál es la fisiopatología de la hemoptisis?


Seleccione una:
a. Hipervascularización de la circulación brónquica, hipertensión pulmonar y disminución de
coagulabilidad
b. Hipervascularización de la circulación brónquica, hipertensión pulmonar y neovascularización
c. Hipervascularización de la circulación pulmonar, hipertensión pulmonar y remodelación.
d. Hipervascularización de la circulación pulmonar, fiebre y regeneración alveolar
Retroalimentación
La respuesta correcta es: Hipervascularización de la circulación brónquica, hipertensión pulmonar y
neovascularización

Pregunta 4
Correcta
Puntúa 0,16 sobre 0,16

Marcar pregunta
Enunciado de la pregunta

Cuál es una causa de taquicardia sinusal fisiológica:


Seleccione una:

a. Feocromocitoma
b. Sedenterismo
c. Hipotiroidismo
d. Hipertensión pulmonar
Retroalimentación
La respuesta correcta es: Feocromocitoma

Pregunta 5
Correcta
Puntúa 0,16 sobre 0,16

Marcar pregunta
Enunciado de la pregunta

Cuál síndrome genético es común en la insuficiencia aórtica:


Seleccione una:
a. Síndrome de Rastelli
b. Síndrome de Eisenmenger

c. Síndrome de Marfán
d. Anomalía de Ebstein
Retroalimentación
La respuesta correcta es: Síndrome de Marfán

Pregunta 6
Correcta
Puntúa 0,16 sobre 0,16

Marcar pregunta
Enunciado de la pregunta

Cuánto es la dosis intravenosa de sostén para digoxina en arritmias:


Seleccione una:
a. 0.200-0.250 mg/día

b. 0.125-0.250 mg/día
c. 0.145-0.250 mg/día
d. 0.175-0.250 mg/día
Retroalimentación
La respuesta correcta es: 0.125-0.250 mg/día

Pregunta 7
Correcta
Puntúa 0,16 sobre 0,16

Marcar pregunta
Enunciado de la pregunta

Cuánto es la dosis sublingual de nitroglicerina en angina de pecho:


Seleccione una:
a. 1.3 a 1.6 mg

b. 0.3 a 0.6 mg
c. 3 a 6 mg
d. 0.8 a 1.0 mg
Retroalimentación
La respuesta correcta es: 0.3 a 0.6 mg

Pregunta 8
Incorrecta
Puntúa 0,00 sobre 0,16
Marcar pregunta
Enunciado de la pregunta

Cuánto es la frecuencia de administración de atenolol en cardiopatía isquémica:


Seleccione una:
a. Cada 8 horas

b. Cada 12 horas
c. Cada 24 horas
d. Cada 6 horas
Retroalimentación
La respuesta correcta es: Cada 24 horas

Pregunta 9
Correcta
Puntúa 0,16 sobre 0,16

Marcar pregunta
Enunciado de la pregunta

De los siguientes factores, cuál es el que debe estar alterado para que un paciente infectado de
tuberculosis se convierta en persona enferma:
Seleccione una:

a. Huésped: Inmunidad celular deficiente, en especial CD4


b. Medio Ambiente: Que viva en un lugar frío
c. Medio ambiente: Que haya elevada prevalencia de tuberculosis
d. Huésped: que genéticamente esté predispuesto a infectarse
Retroalimentación
La respuesta correcta es: Huésped: Inmunidad celular deficiente, en especial CD4

Pregunta 10
Correcta
Puntúa 0,16 sobre 0,16

Marcar pregunta
Enunciado de la pregunta

El hallazgo más frecuente en la radiografía de tórax en un paciente con asma es:


Seleccione una:

a. Radiografía de tórax normal.


b. Condensaciones alveolares bilaterales y difusas.
c. Engrosamiento de paredes bronquiales.
d. Hiperinsuflación pulmonar.
Retroalimentación
La respuesta correcta es: Radiografía de tórax normal.

Pregunta 11
Correcta
Puntúa 0,16 sobre 0,16

Marcar pregunta
Enunciado de la pregunta

El tratamiento de las Bronquiectasias se basa en 3 pilares, excepto:


Seleccione una:
a. Eliminar la obstrucción bronquial.

b. Revertir el remodelamiento bronquia


c. Mejorar la eliminación de las secreciones, que se consigue con una adecuada hidratación, con
fisioterapia respiratoria y drenaje postural mantenidos.
d. Controlar las infecciones con el uso de antibióticos en las agudizaciones durante 10-15 días.
Retroalimentación
La respuesta correcta es: Revertir el remodelamiento bronquia

Pregunta 12
Correcta
Puntúa 0,16 sobre 0,16

Marcar pregunta
Enunciado de la pregunta

El valor de oliguria define?


Seleccione una:
a. 0 a 50 ml
b. 500 a 1000 ml

c. 100 a 400 ml
d. 1000 a 1500 ml
Retroalimentación
La respuesta correcta es: 100 a 400 ml

Pregunta 13
Incorrecta
Puntúa 0,00 sobre 0,16

Marcar pregunta
Enunciado de la pregunta

El virus de hepatitis B, puede ser causa de algunas enfermedades como agente biológico. ¿Cuál
de estas enfermedades?
Seleccione una:
a. Anemia por déficit de hierro
b. Hipoplasia medular
c. Anemia megaloblástica

d. Anemia de proceso crónico


Retroalimentación
La respuesta correcta es: Hipoplasia medular

Pregunta 14
Correcta
Puntúa 0,16 sobre 0,16

Marcar pregunta
Enunciado de la pregunta

En cuanto al diagnóstico funcional del asma:


Seleccione una:
a. Si la relación VEF1/CVF es menor a 0.7 (patrón obstructivo) y post broncodilatador obtenemos un

aumento igual o mayor al 12% en el VEF1, nos orienta hacia diagnóstico de asma
b. Si la relación VEF1/CVF es menor a 0.7 (patrón restrictivo) y post broncodilatador obtenemos un
aumento igual o mayor al 12% en el VEF1, nos orienta hacia diagnóstico de asma
c. Si la relación VEF1/CVF es mayor 0.7 se considera patrón obstructivo, diagnosticamos asma
d. Valores espirométricos no tienen importancia en el diagnóstico del asma.
Retroalimentación
La respuesta correcta es: Si la relación VEF1/CVF es menor a 0.7 (patrón obstructivo) y post
broncodilatador obtenemos un aumento igual o mayor al 12% en el VEF1, nos orienta hacia diagnóstico
de asma

Pregunta 15
Incorrecta
Puntúa 0,00 sobre 0,16

Marcar pregunta
Enunciado de la pregunta

Existen fármacos que aumentan o disminuyen la acción de los anticoagulantes ¿Cuál de estos
fármacos aumenta su acción?
Seleccione una:
a. Aspirina
b. Rifampicina

c. Vitamina K
d. Anticonceptivo
Retroalimentación
La respuesta correcta es: Aspirina

Pregunta 16
Correcta
Puntúa 0,16 sobre 0,16

Marcar pregunta
Enunciado de la pregunta

La causa principal de un fracaso en el tratamiento de la tuberculosis pulmonar es


Seleccione una:

a. Abandono del tratamiento por el paciente


b. Resistencia secundaria a las drogas
c. Toxicidad hepática
d. Resistencia primaria a las drogas
Retroalimentación
La respuesta correcta es: Abandono del tratamiento por el paciente
Pregunta 17
Correcta
Puntúa 0,16 sobre 0,16

Marcar pregunta
Enunciado de la pregunta

La fiebre por meningitis que trastorno ácido básico produce?


Seleccione una:
a. Acidosis respiratoria
b. Acidosis metabólica
c. Alcalosis metabólica

d. Alcalosis respiratoria
Retroalimentación
La respuesta correcta es: Alcalosis respiratoria

Pregunta 18
Incorrecta
Puntúa 0,00 sobre 0,16

Marcar pregunta
Enunciado de la pregunta

La triada de Virchow está definida por lesión endotelial, hipercoagulabilidad y la inamovilidad.


¿Cuál de estas es un riesgo de hipercoagulabilidad?
Seleccione una:
a. Apoplejía lesión de columna
b. Edad avanzada

c. Cirugía
d. Estrógenos
Retroalimentación
La respuesta correcta es: Estrógenos

Pregunta 19
Correcta
Puntúa 0,16 sobre 0,16
Marcar pregunta
Enunciado de la pregunta

La trombosis venosa tiene sus características por estar constituidos por mucha fibrina y
hematíes (trombo rojo) ¿Cuál de estos factores es de riesgo para una trombosis venosa?
Seleccione una:
a. Dislipidemia
b. Diabetes

c. Estasis, inamovilidad
d. Hipertensión
Retroalimentación
La respuesta correcta es: Estasis, inamovilidad

Pregunta 20
Correcta
Puntúa 0,16 sobre 0,16

Marcar pregunta
Enunciado de la pregunta

Qué infección causa bloqueo auriculoventricular:


Seleccione una:
a. Sarampión
b. Tripanosomiasis
c. Varicela

d. Sífilis
Retroalimentación
La respuesta correcta es: Sífilis

Pregunta 21
Correcta
Puntúa 0,16 sobre 0,16

Marcar pregunta
Enunciado de la pregunta

Según Harrison, cuál es un microorganismo virulento para ITU por propagación hematógena:
Seleccione una:

a. Estafilococo aureus
b. Acinetobacter Baumani
c. Estreptococo aureus
d. Proteus mirabilis
Retroalimentación
La respuesta correcta es: Estafilococo aureus

Pregunta 22
Correcta
Puntúa 0,16 sobre 0,16

Marcar pregunta
Enunciado de la pregunta

Según Harrison, cuántas semanas se requieren para catalogar cistitis recurrente?


Seleccione una:
a. 8 semanas
b. 6 semanas
c. 4 semanas

d. 2 semanas
Retroalimentación
La respuesta correcta es: 2 semanas

Pregunta 23
Correcta
Puntúa 0,16 sobre 0,16

Marcar pregunta
Enunciado de la pregunta

Si existen una intoxicacion crónica por Sal Andrews que trastorno ácido básico se produce?
Seleccione una:
a. Acidosis metabólica
b. Alcalosis respiratoria
c. Alcalosis metabólica
d. Acidosis respiratoria
Retroalimentación
La respuesta correcta es: Alcalosis metabólica

Pregunta 24
Incorrecta
Puntúa 0,00 sobre 0,16

Marcar pregunta
Enunciado de la pregunta

Una reacción transfusional aguda o inmediata, se define como la que ocurre durante la
transfusión o en las 24 horas. ¿Cuál de estas opciones es una reacción aguda?
Seleccione una:

a. Enfermedad del Suero


b. Purpura post transfusional
c. Reacción Alérgica Urticaria
d. Transmisión de un HIV
Retroalimentación
La respuesta correcta es: Reacción Alérgica Urticaria

Pregunta 25
Correcta
Puntúa 0,16 sobre 0,16

Marcar pregunta
Enunciado de la pregunta

Un paciente hipertenso con descompensación aguda “típica” de insuficiencia cardiaca sin


sobrecarga de volumen, que fármaco usaría:
Seleccione una:
a. Clortalidona

b. Nitroprusiato
c. Dobutamina
d. Furosemida
Retroalimentación
La respuesta correcta es: Nitroprusiato
Pregunta 1
Correcta
Puntúa 0,16 sobre 0,16

Marcar pregunta
Enunciado de la pregunta

Con respecto a la infección por Virus del Papiloma Humano los tipos (oncogénicos) e
alto riesgo tenemos a:
Seleccione una:

a. 13, 15 y 17.

b. 16, 19, 31 y 22

c. 13, 12,23 y 21

d. 16, 18, 31, 33 y 45.


Retroalimentación
La respuesta correcta es: 16, 18, 31, 33 y 45.

Pregunta 2
Correcta
Puntúa 0,16 sobre 0,16

Marcar pregunta
Enunciado de la pregunta

Con respecto a la vacunación de la fiebre amarilla este tiene una inmunidad con un
plazo de:
Seleccione una:

a. Plazo no mayor a 3 años.

b. Únicamente 12 meses.

c. De por vida del huésped susceptible.

d. Un plazo de 10 días a 10 años.


Retroalimentación
La respuesta correcta es: Un plazo de 10 días a 10 años.

Pregunta 3
Correcta
Puntúa 0,16 sobre 0,16
Marcar pregunta
Enunciado de la pregunta

¿Cuál de las siguientes alternativas indica el carcinoma que prevalece en el tercio


superior del esófago?
Seleccione una:

a. Carcinoma in situ

b. Carcinoma escamoso

c. Adenocarcinoma

d. Carcinoma atípico
Retroalimentación
La respuesta correcta es: Carcinoma escamoso

Pregunta 4
Correcta
Puntúa 0,16 sobre 0,16

Marcar pregunta
Enunciado de la pregunta

¿Cuál de los siguientes hallazgos histológicos es necesario para establecer el


diagnóstico de cirrosis hepática?
Seleccione una:

a. Degeneración baloniforme de los hepatocitos.

b. Destrucción de los conductos biliares.

c. Cuerpos de Mallory.

d. Presencia de fibrosis.
Retroalimentación
La respuesta correcta es: Presencia de fibrosis.

Pregunta 5
Correcta
Puntúa 0,16 sobre 0,16
Marcar pregunta
Enunciado de la pregunta

Dentro de la fisiopatogenia de la infección por VIH tenemos a los coreceptores


principales en la infección estos son:
Seleccione una:

a. CCR5 y CXCR4

b. CCR5 y gp 120

c. Ácido siálico e integrasa.

d. Ácido hialurónico y CD59


Retroalimentación
La respuesta correcta es: CCR5 y CXCR4

Pregunta 6
Correcta
Puntúa 0,16 sobre 0,16

Marcar pregunta
Enunciado de la pregunta

Dentro de las complicaciones extra pulmonares en influenza se han descrito:


Seleccione una:

a. Síndrome de Reye

b. Trombocitopenia y trastornos de la coagulación.

c. Síndrome urémico.

d. Coagulación intravascular diseminada.


Retroalimentación
La respuesta correcta es: Síndrome de Reye

Pregunta 7
Correcta
Puntúa 0,16 sobre 0,16

Marcar pregunta
Enunciado de la pregunta

Dentro de las manifestaciones tardías de la sífilis tenemos trastornos cardiovasculares


de cual podemos desprender el siguiente:
Seleccione una:

a. Insuficiencia cardiaca congestiva.

b. Trastornos del endotelio

c. Flebitis irritativa.

d. Aneurismas
Retroalimentación
La respuesta correcta es: Aneurismas

Pregunta 8
Correcta
Puntúa 0,16 sobre 0,16

Marcar pregunta
Enunciado de la pregunta

El dermatofito más común en la tiña pedis interdigital es, señale la alternativa correcta:
Seleccione una:

a. Tricophyton rubrum

b. Epidermophyton

c. Tricophyton mentagrophytes.

d. Microsporun canis
Retroalimentación
La respuesta correcta es: Tricophyton mentagrophytes.

Pregunta 9
Correcta
Puntúa 0,16 sobre 0,16

Marcar pregunta
Enunciado de la pregunta

El esófago de Barret se caracteriza por, señale la alternativa correcta:


Seleccione una:
a. Se lo considera carcinoma in situ de esófago distal
b. Metaplasia intestinal

c. Se asocia con el anillo de Schatzki.

d. Se asocia con divertículo de Zenker


Retroalimentación
La respuesta correcta es: Metaplasia intestinal

Pregunta 10
Correcta
Puntúa 0,16 sobre 0,16

Marcar pregunta
Enunciado de la pregunta

El tratamiento de elección en colitis pseudomembranosa que cursa con íleo paralítico


es, señale la alternativa correcta:
Seleccione una:

a. Metronidazol por vía intravenosa.

b. Vancomicina por vía intravenosa.

c. Vancomicina por vía oral.

d. Metronidazol por vía oral.


Retroalimentación
La respuesta correcta es: Metronidazol por vía intravenosa.

Pregunta 11
Correcta
Puntúa 0,16 sobre 0,16

Marcar pregunta
Enunciado de la pregunta

En la clasificación de OMS de adultos con VIH – Sida en el estadio C3 tenemos a:


Seleccione una:

a. Mayor de 500 células CD4+ Asintomático.

b. Menor de 200 células CD4+ con síntomas definidores de Sida.

c. Menor de 300 células CD4+ con síntomas definidores de Sida.

d. Mayor de 400 células CD4+ con síntomas no definidores de Sida.


Retroalimentación
La respuesta correcta es: Menor de 200 células CD4+ con síntomas definidores de Sida.

Pregunta 12
Incorrecta
Puntúa 0,00 sobre 0,16

Marcar pregunta
Enunciado de la pregunta

En la urticaria crónica autoinmune un medicamento de gran utilidad con pocos efectos


secundarios es:
Seleccione una:

a. Loratadina

b. Cimetidina

c. Levocetirizina

d. Ciclosporina
Retroalimentación
La respuesta correcta es: Ciclosporina

Pregunta 13
Incorrecta
Puntúa 0,00 sobre 0,16

Marcar pregunta
Enunciado de la pregunta

En la urticaria crónica resistente a antihistaminicos anti h1 se puede utilizar


Seleccione una:

a. Infliximab

b. Rituximab

c. Ciproheptadina

d. Ciclosporina
Retroalimentación
La respuesta correcta es: Ciproheptadina

Pregunta 14
Correcta
Puntúa 0,16 sobre 0,16
Marcar pregunta
Enunciado de la pregunta

En una infección pro Cándida en mucosa oral, indique la premisa correcta:


Seleccione una:

a. Invasión profunda con daño extenso de tejido.

b. El tratamiento de elección es la caspofungina vía parenteral.

c. Constituye un hongo hifa, filamentoso de colonias secas.

d. Es importante saber el estado inmunitario del paciente para establecer el posible

diagnóstico.
Retroalimentación
La respuesta correcta es: Es importante saber el estado inmunitario del paciente para
establecer el posible diagnóstico.

Pregunta 15
Correcta
Puntúa 0,16 sobre 0,16

Marcar pregunta
Enunciado de la pregunta

La primoinfeccion de VHS 1 se basa principalmente es cuales aspectos clínicos:


Seleccione una:

a. Herpes labial exfoliativo.

b. Irritabilidad y llanto fácil del lactante.

c. Panadizo herpético.

d. Gingivoestomatitis y faringitis.
Retroalimentación
La respuesta correcta es: Gingivoestomatitis y faringitis.

Pregunta 16
Correcta
Puntúa 0,16 sobre 0,16
Marcar pregunta
Enunciado de la pregunta

Muchacha de 26 años hospitalizada por ictericia de instauración reciente, asociada a


dolor en hipocondrio derecho. Se detecta hepatomegalia sensible sin esplenomegalia.
Hay telangiectasias faciales. Reconoce antecedentes de promiscuidad sexual, pero no
consumo de droga intravenosa. AST 315 ;BT 16 mg/dl; alt 110 ; GGT 680; FA 280 ;
triglicéridos 600, colesterol 280. Ecografía con patrón ecogénico del hígado. Cuál de
las siguientes alternativas indica el diagnóstico más probable:
Seleccione una:

a. Hepatopatía alcoholica

b. Hepatitis Autoinmune

c. Hepatitis tóxica

d. Hepatitis vírica
Retroalimentación
La respuesta correcta es: Hepatitis vírica

Pregunta 17
Correcta
Puntúa 0,16 sobre 0,16

Marcar pregunta
Enunciado de la pregunta

Paciente de sexo femenino de 30 años de edad que presenta desde la infancia placas
eritematodescamativas de bordes bien definidos en codos, y rodilla con descamación
intensa y prurito moderado, además presenta eritema y descamación profusa en cuero
cabelludo. ¿Cuál de las siguientes alternativas corresponde al tratamiento tópico?
Seleccione una:

a. Tacrolimus + tazaroteno

b. Emolientes y corticoides topicos

c. Inhibidores de la calcineurina asociados a calcipotriol

d. Corticoesteroides tópicos asociados a calcipotriol


Retroalimentación
La respuesta correcta es: Corticoesteroides tópicos asociados a calcipotriol

Pregunta 18
Correcta
Puntúa 0,16 sobre 0,16

Marcar pregunta
Enunciado de la pregunta

Para el manejo de infecciones dermatológicas la Penicilina G es uno de los


betalactámicos más eficientes por su gran actividad sobre Pseudomonas, responda
verdadero o falso
Seleccione una:

a. Falso

b. Verdadero
Retroalimentación
La respuesta correcta es: Falso

Pregunta 19
Correcta
Puntúa 0,16 sobre 0,16

Marcar pregunta
Enunciado de la pregunta

Respecto a la cirrosis hepática, escoja la respuesta correcta.


Seleccione una:

a. La cirrosis es la etapa final de cualquier enfermedad aguda del hígado.

b. Es un proceso reversible.

c. Es un proceso localizado caracterizado por fibrosis y la conversión de la arquitectura normal


en nódulos de estructura alterada.

d. Las complicaciones son consecuencia de la hipertensión portal y la insuficiencia

hepática.
Retroalimentación
La respuesta correcta es: Las complicaciones son consecuencia de la hipertensión portal y la
insuficiencia hepática.

Pregunta 20
Correcta
Puntúa 0,16 sobre 0,16
Marcar pregunta
Enunciado de la pregunta

Sobra la Enfermedad Inflamatoria Intestinal, escoja la respuesta INCORRECTA:


Seleccione una:

a. El factor de necrosis tumoral y el interferón alfa contribuyen directamente al daño epitelial del
intestino.

b. Los niveles tisulares de IL 1 están elevados en la Enfermedad de Crohn y los niveles séricos

de IL 2 están igualmente elevados en la Colitis ulcerosa.

c. En la EII se encuentra aumentada la producción de las citokinas proinflamatorias, IL 1, IL 6,


IL 8, y además del FNT (factor de necrosis tumoral) alfa.

d. La mayor fuente de producción de estas interleukinas proinflamatorias son los macrófagos


activados de la lámina propia que estimulan las células inmunes.
Retroalimentación
La respuesta correcta es: Los niveles tisulares de IL 1 están elevados en la Enfermedad de
Crohn y los niveles séricos de IL 2 están igualmente elevados en la Colitis ulcerosa.

Pregunta 21
Correcta
Puntúa 0,16 sobre 0,16

Marcar pregunta
Enunciado de la pregunta

Tras la infección de un niño con el virus de la Influenza con la siguientes


características: fiebre intensa de inicio súbito, disnea y cianosis a la Rx de tórax se
aprecia patrón asociado con infiltrados intersticiales difusos e hipoxia intensa, usted
sospecharía de:
Seleccione una:

a. Neumonía viral secundaria.

b. Neumonía bacteriana primaria.

c. Neumonía viral primaría.

d. Neumonía bacteriana secundaria.


Retroalimentación
La respuesta correcta es: Neumonía viral primaría.

Pregunta 22
Correcta
Puntúa 0,16 sobre 0,16

Marcar pregunta
Enunciado de la pregunta

Una de las características principales por las cuales el dengue puede diseminarse es:
Seleccione una:

a. Debido a la gran capacidad de diseminación del vector Anopheles.

b. Es sumamente frecuente la progresión de daño encefálico.

c. El vector es muy cercano a los asentamientos humanos.

d. Una característica precoz es la trombocitosis y neutrofilia.


Retroalimentación
La respuesta correcta es: El vector es muy cercano a los asentamientos humanos.

Pregunta 23
Correcta
Puntúa 0,16 sobre 0,16

Marcar pregunta
Enunciado de la pregunta

Una mujer de 25 años refiere una historia de 12 meses de cólicos abdominales


recurrentes en hemi-abdomen inferior acompañado de distensión abdominal y
diarreas. No presenta fiebre, pérdida de peso ni anorexia ni deposición con sangre.
Este paciente cumple con los criterios de Roma IV para Sd de colon irritable con
predominio de diarrea, no presenta signos de alarma. ¿En qué otra patología Usted
puede pensar?
Seleccione una:

a. Sobrecrecimiento bacteriano de intestino delgado.

b. Helicobacter pylori.

c. Enfermedad celiaca.

d. Enfermedad inflamatoria intestinal.


Retroalimentación
La respuesta correcta es: Enfermedad celiaca.

Pregunta 24
Correcta
Puntúa 0,16 sobre 0,16
Marcar pregunta
Enunciado de la pregunta

Un ejecutivo de 55 años de edad se atiende por dolor epigástrico intenso; en la


endoscopia gastroduodenal se encuentra una ulcera grande en el bulbo duodenal.
Se indican una dieta e inhibidor de bomba de protones, pero los síntomas persisten.
En este momento lo más apropiado seria sugerir, señale la opción correcta:
Seleccione una:

a. Tratamiento de erradicación de helicobacter pylori.

b. Vagotomía altamente selectiva.

c. Vagotomía troncal y antrectomia

d. Vagotomía troncal y piloroplastia.


Retroalimentación
La respuesta correcta es: Tratamiento de erradicación de helicobacter pylori.

Pregunta 25
Correcta
Puntúa 0,16 sobre 0,16

Marcar pregunta
Enunciado de la pregunta

Un hombre de 69 años con hepatitis crónica por virus C y cirrosis sin antecedentes
previos de hemorragia varicial, acude por el hallazgo endoscópico de várices
esofágicas grado III. ¿Cuál es el agente farmacológico de preferencia para la
prevención de sangrado varicial?
Seleccione una:
a. Atenolol.

b. Pantoprazol.

c. Carvedilol.

d. 5 mononitrato de isosorbide.
Retroalimentación
La respuesta correcta es: Carvedilol.
Pregunta 1
Correcta
Puntúa 0,16 sobre 0,16

Marcar pregunta
Enunciado de la pregunta

Con respecto a DM2 señale lo correcto:


Seleccione una:

a. Se asocia con una fuerte predisposición genética o antecedentes familiares en familiares de

primer grado.

b. Existe una destrucción de las células beta del páncreas con un déficit absoluto de insulina

c. Un de los criterios para su diagnóstico es PTOG a las dos horas con un valor de 140-199
mg/dl

d. Representa 2– 4% de todas las diabetes


Retroalimentación
La respuesta correcta es: Se asocia con una fuerte predisposición genética o antecedentes
familiares en familiares de primer grado.

Pregunta 2
Correcta
Puntúa 0,16 sobre 0,16

Marcar pregunta
Enunciado de la pregunta

¿Cuál de las siguientes es una causa secundaria de Diabetes?


Seleccione una:
a. Hipoparatiroidismo, hipertiroidismo, pancreatitis

b. Acromegalia, embarazo, enfermedad de Addison

c. Síndrome de ovario poliquístico, Síndrome de Cushing, acromegalia.

d. Hipotiroidismo, embarazo, enfermedad de Addison


Retroalimentación
La respuesta correcta es: Síndrome de ovario poliquístico, Síndrome de Cushing, acromegalia.

Pregunta 3
Correcta
Puntúa 0,16 sobre 0,16
Marcar pregunta
Enunciado de la pregunta

¿Cuál de las siguientes es una complicación aguda?


Seleccione una:

a. Cetoacidosis.

b. Nefropatía (grado I)

c. Cardiopatía isquémica

d. Retinopatía proliferativa
Retroalimentación
La respuesta correcta es: Cetoacidosis.

Pregunta 4
Correcta
Puntúa 0,16 sobre 0,16

Marcar pregunta
Enunciado de la pregunta

¿Cuál de los siguientes corresponde a un criterio de diagnóstico en DM2?


Seleccione una:

a. Glucosa al azar (2 tomas ≥ 200mg/dl).

b. Glucosas en ayunas 100-125mg/dl

c. Glucosa al azar ≥ 126 mg/dl junto con las tres P (poliuria, polidipsia, polifagia)

d. HbAc1 de 5,7 – 6,4%


Retroalimentación
La respuesta correcta es: Glucosa al azar (2 tomas ≥ 200mg/dl).

Pregunta 5
Correcta
Puntúa 0,16 sobre 0,16

Marcar pregunta
Enunciado de la pregunta
De los siguientes enunciados señale el verdadero:
Seleccione una:

a. El falx divide al cerebro en hemisferio derecho e izquierdo

b. La herniación se refiere al desplazamiento de un compartimento de menor a mayor presión

c. El tentorio divide al cerebral en región infratentorial de región medular

d. El desplazamiento del mesencéfalo con compresión del pedúnculo origina midriasis


ipsilateral
Retroalimentación
La respuesta correcta es: El falx divide al cerebro en hemisferio derecho e izquierdo

Pregunta 6
Correcta
Puntúa 0,16 sobre 0,16

Marcar pregunta
Enunciado de la pregunta

El dato clínico más importante para considerar un evento cerebrovascular como


hemorrágico más que como isquémico es:
Seleccione una:

a. antecedente de hipertensión arterial

b. glicemia en urgencias mayor de 300 mg/dL

c. hemiparesia de instauración súbita

d. afectación temprana del estado de conciencia


Retroalimentación
La respuesta correcta es: afectación temprana del estado de conciencia

Pregunta 7
Incorrecta
Puntúa 0,00 sobre 0,16

Marcar pregunta
Enunciado de la pregunta

En relación a los factores de riesgo en infarto cerebral, señale el enunciado verdadero:


Seleccione una:

a. El tratamiento de prevención secundaria evita la posibilidad de recurrencia de un infarto


b. La presencia de fibrilación auricular valvular requiere anticoagulación total crónicamente

c. Factores no modificables como el sedentarismo no requiere manejo específico

d. Los niveles de control de lípidos son: HDL >130 mg/dl, colesterol total >250 mg/dl
Retroalimentación
La respuesta correcta es: El tratamiento de prevención secundaria evita la posibilidad de
recurrencia de un infarto

Pregunta 8
Correcta
Puntúa 0,16 sobre 0,16

Marcar pregunta
Enunciado de la pregunta

En relación al tratamiento de Encefalitis viral, señale el enunciado verdadero:


Seleccione una:

a. El tratamiento antiviral debe ser iniciado cuando se tenga el resultado del cultivo

b. Aciclovir debe administrarse en vía IV a dosis de 30 mg/kg/día

c. Se debe añadir al antiviral tratamiento a largo plazo con prednisona oral

d. El aciclovir en vía oral tiene igual efectividad que el IV


Retroalimentación
La respuesta correcta es: Aciclovir debe administrarse en vía IV a dosis de 30 mg/kg/día

Pregunta 9
Correcta
Puntúa 0,16 sobre 0,16

Marcar pregunta
Enunciado de la pregunta

En relación al tratamiento de meningitis bacteriana aguda, señale el enunciado


verdadero:
Seleccione una:

a. En Haemophilus influenzae se prescribiría ceftriaxona

b. En estafilococo meticilino resistente se prescribiría penicilina G

c. Si hay resistencia a la penicilina se prescribiría Amikacina

d. En Neisseria Meningitidis se prescribiría metronidazol


Retroalimentación
La respuesta correcta es: En Haemophilus influenzae se prescribiría ceftriaxona

Pregunta 10
Correcta
Puntúa 0,16 sobre 0,16

Marcar pregunta
Enunciado de la pregunta

Entre los factores de riesgo de enfermedad de Graves tenemos escoja la opción


correcta
Seleccione una:

a. no se relaciona con la herencia familiar , el tabaquismo es un factor de riesgo intermedio , se


ve en gemelos idénticos

b. historia familiar de enfermedad de Hashimoto, presencia de otras enfermedades

autoinmunes , más común en mujeres que en hombres , tabaquismo.

c. se ve en misma proporción como factor de riesgo en mujeres y en hombres, el tabaquismo y


se asocia con enfermedades autoinmunes

d. presencia de otras enfermedades autoinmunes , tabaquismo más frecuente en hombres que


el mujeres , antecedente de enfermedad de Graves .
Retroalimentación
La respuesta correcta es: historia familiar de enfermedad de Hashimoto, presencia de otras
enfermedades autoinmunes , más común en mujeres que en hombres , tabaquismo.

Pregunta 11
Correcta
Puntúa 0,16 sobre 0,16

Marcar pregunta
Enunciado de la pregunta

En un paciente con síntomas de diplopía, con antecedentes de una infección


gastrointestinal 10 días atrás y que al examen se lo encuentra con oftalmoparesia y
arreflexia, al estudiarlo se encuentra disociación albúmino-citológica, cuál sería su
proceder:
Seleccione una:

a. Se debe solicitar determinación de anticuerpos contra los lípidos del axón neuronal

b. Le prescribiría corticoides en vía IV durante 5 días

c. Si lleva 3 días de evolución tendría que realizarse una electromiografía


d. Le diagnosticaría Sd de Miller-Fisher como variante del Sd de Guillain-Barré
Retroalimentación
La respuesta correcta es: Le diagnosticaría Sd de Miller-Fisher como variante del Sd de
Guillain-Barré

Pregunta 12
Correcta
Puntúa 0,16 sobre 0,16

Marcar pregunta
Enunciado de la pregunta

En un paciente de 77 años con antecedentes de fumador, estenosis carotídea,


diabetes mellitus y dislipidemia, presentó un episodio de pérdida del estado de
conciencia al parecer con crisis convulsivas, sin testigos directos, se sospecha en
crisis convulsivas, con qué entidad debería hacer diagnóstico diferencial:
Seleccione una:
a. Migraña basilar

b. Hipotensión ortostática

c. Crisis psicogénica

d. Ataques de pánico
Retroalimentación
La respuesta correcta es: Hipotensión ortostática

Pregunta 13
Incorrecta
Puntúa 0,00 sobre 0,16

Marcar pregunta
Enunciado de la pregunta

En un paciente que acude a urgencias por presentar un cuadro de cefalea de inicio


súbito con deterioro del estado de conciencia y una crisis tónico clónica generalizada,
qué estudio requeriría con mayor urgencia:
Seleccione una:

a. Biometría hemática

b. Electrolitos

c. Electroencefalograma
d. TAC cerebral simple
Retroalimentación
La respuesta correcta es: TAC cerebral simple

Pregunta 14
Correcta
Puntúa 0,16 sobre 0,16

Marcar pregunta
Enunciado de la pregunta

Escoja la respuesta CORRECTA sobre la tiroiditis crónica o de Hashimoto


Seleccione una:

a. Produce una glándula blanda, de gran crecimiento, dolorosa y con consistencia lobulada

b. Es una enfermedad autoinmune donde actúan los linfocitos T CD4 que libran IL-2 y FNT y

los linfocitos T citotóxicos que produce apoptosis de tirocitos

c. Es más frecuente en hombres (1:10) vs mujeres (1:20).

d. No tiene factores desencadenantes como estrés, déficit de selenio, infección por Yersinia
Enterocolítica o hepatitis C
Retroalimentación
La respuesta correcta es: Es una enfermedad autoinmune donde actúan los linfocitos T CD4
que libran IL-2 y FNT y los linfocitos T citotóxicos que produce apoptosis de tirocitos

Pregunta 15
Correcta
Puntúa 0,16 sobre 0,16

Marcar pregunta
Enunciado de la pregunta

Hablando de la fisiopatología de la oftalmopatía de Graves .Escoja la opción correcta


Seleccione una:

a. la disminución de las células T juegan un papel importante en la generación de la


oftalmopatía .

b. las células T dismunuidas , los fibroblastos producen ácido hialuronico y glucosamino


glucanos y esto aumenta el tejido retro ocular .

c. el volumen de los músculos retro oculares aumentan debido a la disminución de los


fibroblastos .
d. la presencia aumentada d e células T juegan un papel importantes en la oftalmopatía , y los

fibroblastos retro oculares secretan glucosaminoglucanos


Retroalimentación
La respuesta correcta es: la presencia aumentada d e células T juegan un papel importantes en
la oftalmopatía , y los fibroblastos retro oculares secretan glucosaminoglucanos

Pregunta 16
Correcta
Puntúa 0,16 sobre 0,16

Marcar pregunta
Enunciado de la pregunta

Hay que hacer una historia clínica minuciosa en el hipotiroidismo, tomando especial
importancia a ciertos fármacos que producen disfunción de hormona tiroidea. ¿Cuál de
los siguientes NO está asociado con hipotiroidismo?
Seleccione una:

a. Yodo

b. Cefalosporinas.

c. Amiodarona

d. Interferon alfa
Retroalimentación
La respuesta correcta es: Cefalosporinas.

Pregunta 17
Incorrecta
Puntúa 0,00 sobre 0,16

Marcar pregunta
Enunciado de la pregunta

La hemorragia cerebral primaria o por hipertensión arterial se localiza en:


Seleccione una:

a. Bulbo raquídeo

b. Región lobar parietal

c. Núcleo putaminal

d. Ventrículos
Retroalimentación
La respuesta correcta es: Núcleo putaminal

Pregunta 18
Correcta
Puntúa 0,16 sobre 0,16

Marcar pregunta
Enunciado de la pregunta

Paciente de 4 0 años de sexo femenino que se ha pedido exámenes por control


médico anual y en el Examen Físico se encontró BOCIO regresa con resultados: Eco
tiroideo: reporta imagen nodular solida de 1 cm hipo eco génica con vascularidad
central y periférica con margen irregular . TSH 4 . 2 u/ IU/ ml T 4 1.6 ng / dl. Escoja la
opción correcta.
Seleccione una:

a. el reporte ecográfico no hace sospechar de malignidad se debe hacer seguimiento


únicamente en 6 meses

b. le derivo al endocrinólogo y al cirujano de cuello para que le le opren inmediatamente , y no


necesita de otros exámenes .

c. es una paciente con sospecha de malignidad mas de un 70 % se debe solicitar punción con

aguja fina y se solicita marcadores tumorales.

d. es una paciente con poca sospecha de malignidad como de 30 % y se debe pedir un nuevo
control en 3 meses
Retroalimentación
La respuesta correcta es: es una paciente con sospecha de malignidad mas de un 70 % se
debe solicitar punción con aguja fina y se solicita marcadores tumorales.

Pregunta 19
Correcta
Puntúa 0,16 sobre 0,16

Marcar pregunta
Enunciado de la pregunta

Paciente de 45 años acude a consulta por presentar 6 meses de evolución de caída de


cabello y no refiere otros síntomas, Al examen físico se encuentra escaso cabello y
hay piel seca lo demás de examen físico esta normal , se solicita exámenes los
mismos que reportan : TSH5.17 u/IU/ml , T3 3 ,3 n mol/ L- T 4 1.0 ng / dl. Escoja la
opción correcta.
Seleccione una:
a. el paciente tiene diagnóstico de hipotiroidismo subclínico

b. el paciente tiene diagnóstico de eutiroidismo

c. el paciente tiene diagnóstico de hipertiroidismo

d. paciente tiene diagnóstico de hipotiroidismo central


Retroalimentación
La respuesta correcta es: el paciente tiene diagnóstico de hipotiroidismo subclínico

Pregunta 20
Correcta
Puntúa 0,16 sobre 0,16

Marcar pregunta
Enunciado de la pregunta

Paciente de 65 años con antecedente de cáncer de estómago, tiene una dieta


fraccionada con 5 comidas ha bajado de peso mucho como 10 kilogramos y refiere
dolor de epigastrio en forma continua ,además refiere que se ha sentido con polidipsia
y poliuria por lo que se solicita exámenes los mismos que reportan: Glicemia 103 mg /
dl Hemoglobina Glicosilada 6.4 mg y curva de tolerancia a la glicemia reporta a la 99
mg , 30 minutos, 108 mg, 1 horas 199 mg , 2 horas 155 mg. Escoja la opción correcta
Seleccione una:

a. la paciente tiene valores normales de laboratorio y no necesita recomendaciones

b. la paciente tiene diagnóstico de prediabetes tiene que remitirse a nutrición y debe regresar a

un control en 3 meses con glicemia y nueva hemoglobina glicosilada.

c. tiene cáncer gástrico y tiene diabetes y debe recibir ya tratamiento con metformina de 1000
mg cada 12 horas

d. la paciente tiene gastrectomía por cáncer gástrico y al momento cursa además con diabetes
y se le recomienda valoración nutricional , ejercicio y control en 1 mes con nueva glicemia
Retroalimentación
La respuesta correcta es: la paciente tiene diagnóstico de prediabetes tiene que remitirse a
nutrición y debe regresar a un control en 3 meses con glicemia y nueva hemoglobina
glicosilada.

Pregunta 21
Incorrecta
Puntúa 0,00 sobre 0,16

Marcar pregunta
Enunciado de la pregunta
Señale la respuesta CORRECTA sobre las complicaciones del hipotiroidismo
Seleccione una:

a. En el hipotiroidismo severo se puede generar hipernatremia dilucional.

b. En el hipotiroidismo leve puede haber apnea del sueño y síndrome del túnel carpiano

c. Puede producir hipertensión, cardiomegalia y disfunción diastólica

d. Algunas personas desarrollan incremento de memoria, estado de ánimo eufórica


Retroalimentación
La respuesta correcta es: Puede producir hipertensión, cardiomegalia y disfunción diastólica

Pregunta 22
Incorrecta
Puntúa 0,00 sobre 0,16

Marcar pregunta
Enunciado de la pregunta

Si un paciente es detectado con HbA1c: > 9%, la acción correcta para su tratamiento
sería:
Seleccione una:

a. Se debe realizar a triple terapia de ADO

b. Administrar metformina junto con un ADO

c. Iniciar inmediatamente con insulina.

d. La primera línea indicada es la metformina en busca de HbAc1 de < 7%


Retroalimentación
La respuesta correcta es: Iniciar inmediatamente con insulina.

Pregunta 23
Incorrecta
Puntúa 0,00 sobre 0,16

Marcar pregunta
Enunciado de la pregunta

Un hallazgo clínico sugestivo de infarto cerebral en la circulación carotídea es:


Seleccione una:

a. parálisis facial ipsilateral a la lesión con hemiplejia contralateral

b. alteración del estado de conciencia


c. afasia de Broca sin hemiparesia

d. parálisis oculomotora con tetraplejia


Retroalimentación
La respuesta correcta es: afasia de Broca sin hemiparesia

Pregunta 24
Incorrecta
Puntúa 0,00 sobre 0,16

Marcar pregunta
Enunciado de la pregunta

Un paciente joven es encontrado en coma, con pupilas isocóricas de 2 mm reactivas al


reflejo fotomotor, reflejo oculocefálico normal, al dolor la respuesta era de flexión
parcial en 4 extremidades, con estos datos qué estudio hubiera solicitado con mayor
urgencia:
Seleccione una:

a. Electrocardiograma

b. Tiempos de coagulación

c. Pruebas toxicológicas

d. Electroencefalograma
Retroalimentación
La respuesta correcta es: Pruebas toxicológicas

Pregunta 25
Incorrecta
Puntúa 0,00 sobre 0,16

Marcar pregunta
Enunciado de la pregunta

Un paciente que ingresa con un cuadro de amaurosis fugax del ojo derecho que
revirtió en 10 minutos, usted sospecharía en:
Seleccione una:

a. Neuritis óptica secundario a vasculitis

b. Infarto occipital agudo secundario a aterosclerosis vertebral

c. Compresión del quiasma óptico por macroadenoma hipofisario

d. Accidente isquémico transitorio secundario a aterosclerosis carotídea


Retroalimentación
La respuesta correcta es: Accidente isquémico transitorio secundario a aterosclerosis carotídea
Con respecto a DM2 señale lo correcto:
Seleccione una:

a. Se asocia con una fuerte predisposición genética o antecedentes familiares en familiares de

primer grado.

b. Existe una destrucción de las células beta del páncreas con un déficit absoluto de insulina

c. Representa 2– 4% de todas las diabetes

d. Un de los criterios para su diagnóstico es PTOG a las dos horas con un valor de 140-199
mg/dl
Retroalimentación
La respuesta correcta es: Se asocia con una fuerte predisposición genética o antecedentes
familiares en familiares de primer grado.

Pregunta 2
Correcta
Puntúa 0,16 sobre 0,16

Desmarcar
Enunciado de la pregunta

¿Cuál de las siguientes es una causa secundaria de Diabetes?


Seleccione una:

a. Acromegalia, embarazo, enfermedad de Addison

b. Hipotiroidismo, embarazo, enfermedad de Addison

c. Hipoparatiroidismo, hipertiroidismo, pancreatitis

d. Síndrome de ovario poliquístico, Síndrome de Cushing, acromegalia.


Retroalimentación
La respuesta correcta es: Síndrome de ovario poliquístico, Síndrome de Cushing, acromegalia.

Pregunta 3
Correcta
Puntúa 0,16 sobre 0,16

Desmarcar
Enunciado de la pregunta

¿Cuál de las siguientes es una complicación aguda?


Seleccione una:
a. Nefropatía (grado I)

b. Cardiopatía isquémica

c. Cetoacidosis.

d. Retinopatía proliferativa
Retroalimentación
La respuesta correcta es: Cetoacidosis.

Pregunta 4
Correcta
Puntúa 0,16 sobre 0,16

Desmarcar
Enunciado de la pregunta

¿Cuál de los siguientes corresponde a un criterio de diagnóstico en DM2?


Seleccione una:

a. Glucosa al azar (2 tomas ≥ 200mg/dl).

b. Glucosa al azar ≥ 126 mg/dl junto con las tres P (poliuria, polidipsia, polifagia)

c. HbAc1 de 5,7 – 6,4%

d. Glucosas en ayunas 100-125mg/dl


Retroalimentación
La respuesta correcta es: Glucosa al azar (2 tomas ≥ 200mg/dl).

Pregunta 5
Correcta
Puntúa 0,16 sobre 0,16

Desmarcar
Enunciado de la pregunta

Cuál de los siguientes no es un factor de riesgo para el desarrollo de cáncer gástrico


Seleccione una:

a. Infección por h pylori.

b. Poliposis gástrica

c. Hipotiroidismo
d. Familiar con antecedente de cáncer digestivo
Retroalimentación
La respuesta correcta es: Hipotiroidismo

Pregunta 6
Correcta
Puntúa 0,16 sobre 0,16

Desmarcar
Enunciado de la pregunta

El tratamiento de elección para un hombre de 40 años de edad con diagnóstico de


linfoma MALT después de endoscopia y biopsia es:
Seleccione una:

a. Radioterapia

b. Tratamiento de erradicación de Helicobacter Pylori.

c. Gastrectomía subtotal

d. Quimioterapia
Retroalimentación
La respuesta correcta es: Tratamiento de erradicación de Helicobacter Pylori.

Pregunta 7
Incorrecta
Puntúa 0,00 sobre 0,16

Desmarcar
Enunciado de la pregunta

El tratamiento que se utiliza para reducir la IGE en la urticaria crónica es:


Seleccione una:

a. Rituximab

b. Ciproheptadina

c. Ciclosporina

d. Omalizumab
Retroalimentación
La respuesta correcta es: Omalizumab

Pregunta 8
Correcta
Puntúa 0,16 sobre 0,16
Desmarcar
Enunciado de la pregunta

En que situaciones es mandatorio dar tratamiento para erradicación de helicobacter


pylori
Seleccione una:

a. En síndrome de malabsorción

b. Anemia ferropenica

c. En cirugía intestinal

d. Cuando hay episodios de constipación


Retroalimentación
La respuesta correcta es: Anemia ferropenica

Pregunta 9
Correcta
Puntúa 0,16 sobre 0,16

Desmarcar
Enunciado de la pregunta

Entre los factores de riesgo de enfermedad de Graves tenemos escoja la opción


correcta
Seleccione una:

a. presencia de otras enfermedades autoinmunes , tabaquismo más frecuente en hombres que


el mujeres , antecedente de enfermedad de Graves .

b. historia familiar de enfermedad de Hashimoto, presencia de otras enfermedades

autoinmunes , más común en mujeres que en hombres , tabaquismo.

c. no se relaciona con la herencia familiar , el tabaquismo es un factor de riesgo intermedio , se


ve en gemelos idénticos

d. se ve en misma proporción como factor de riesgo en mujeres y en hombres, el tabaquismo y


se asocia con enfermedades autoinmunes
Retroalimentación
La respuesta correcta es: historia familiar de enfermedad de Hashimoto, presencia de otras
enfermedades autoinmunes , más común en mujeres que en hombres , tabaquismo.

Pregunta 10
Correcta
Puntúa 0,16 sobre 0,16
Desmarcar
Enunciado de la pregunta

En un paciente con disfagia esofágica que refiere además regurgitación y pirosis


frecuente de predominio nocturno que diagnostico consideraría
Seleccione una:

a. Diverticulo de zenker

b. Enfermedad por reflujo gastroesofagico

c. Disfagia orofaringea

d. Esofagitis infecciosa
Retroalimentación
La respuesta correcta es: Enfermedad por reflujo gastroesofagico

Pregunta 11
Correcta
Puntúa 0,16 sobre 0,16

Desmarcar
Enunciado de la pregunta

Erisipela una infección de tejido blando es debido principalmente a:


Seleccione una:

a. S. pyogenes

b. Enterobacterias

c. S. aureus

d. Pseudomona aeruginosa
Retroalimentación
La respuesta correcta es: S. pyogenes

Pregunta 12
Correcta
Puntúa 0,16 sobre 0,16

Desmarcar
Enunciado de la pregunta

Hablando de la fisiopatología de la oftalmopatía de Graves .Escoja la opción correcta


Seleccione una:

a. el volumen de los músculos retro oculares aumentan debido a la disminución de los


fibroblastos .

b. la disminución de las células T juegan un papel importante en la generación de la


oftalmopatía .

c. las células T dismunuidas , los fibroblastos producen ácido hialuronico y glucosamino


glucanos y esto aumenta el tejido retro ocular .

d. la presencia aumentada d e células T juegan un papel importantes en la oftalmopatía , y los

fibroblastos retro oculares secretan glucosaminoglucanos


Retroalimentación
La respuesta correcta es: la presencia aumentada d e células T juegan un papel importantes en
la oftalmopatía , y los fibroblastos retro oculares secretan glucosaminoglucanos

Pregunta 13
Correcta
Puntúa 0,16 sobre 0,16

Desmarcar
Enunciado de la pregunta

La causa mas frecuente de gastritis es


Seleccione una:

a. Aines

b. Helicobacter pylori

c. Alcohol

d. Radiacion
Retroalimentación
La respuesta correcta es: Helicobacter pylori

Pregunta 14
Correcta
Puntúa 0,16 sobre 0,16

Desmarcar
Enunciado de la pregunta
La fase inicial del melanoma maligno es:
Seleccione una:

a. Fase de crecimiento vertical

b. Fase de crecimiento radial

c. Fase de crecimiento acral

d. Fase de crecimiento lentiginoso


Retroalimentación
La respuesta correcta es: Fase de crecimiento radial

Pregunta 15
Correcta
Puntúa 0,16 sobre 0,16

Desmarcar
Enunciado de la pregunta

La infección por estafilococo aureus a nivel epidérmico ocasiona ampollas por


Seleccione una:

a. Lesión de los desmosomas basales

b. Lesión de los desmosomas granulosos

c. Lesión del estrato espinoso

d. Lesión del estrato basal


Retroalimentación
La respuesta correcta es: Lesión de los desmosomas granulosos

Pregunta 16
Correcta
Puntúa 0,16 sobre 0,16

Desmarcar
Enunciado de la pregunta

La presencia de superficie perlada, telangiectasias arboriformes y nódulos con


pigmento son característicos de
Seleccione una:

a. Carcinoma escamocelular

b. Melanoma maligno
c. Carcinoma basocelular

d. Tumor de celulas de merkel


Retroalimentación
La respuesta correcta es: Carcinoma basocelular

Pregunta 17
Correcta
Puntúa 0,16 sobre 0,16

Desmarcar
Enunciado de la pregunta

Paciente de 4 0 años de sexo femenino que se ha pedido exámenes por control


médico anual y en el Examen Físico se encontró BOCIO regresa con resultados: Eco
tiroideo: reporta imagen nodular solida de 1 cm hipo eco génica con vascularidad
central y periférica con margen irregular . TSH 4 . 2 u/ IU/ ml T 4 1.6 ng / dl. Escoja la
opción correcta.
Seleccione una:

a. es una paciente con sospecha de malignidad mas de un 70 % se debe solicitar punción con

aguja fina y se solicita marcadores tumorales.

b. es una paciente con poca sospecha de malignidad como de 30 % y se debe pedir un nuevo
control en 3 meses

c. el reporte ecográfico no hace sospechar de malignidad se debe hacer seguimiento


únicamente en 6 meses

d. le derivo al endocrinólogo y al cirujano de cuello para que le le opren inmediatamente , y no


necesita de otros exámenes .
Retroalimentación
La respuesta correcta es: es una paciente con sospecha de malignidad mas de un 70 % se
debe solicitar punción con aguja fina y se solicita marcadores tumorales.

Pregunta 18
Correcta
Puntúa 0,16 sobre 0,16

Desmarcar
Enunciado de la pregunta

Paciente de 45 años acude a consulta por presentar 6 meses de evolución de caída de


cabello y no refiere otros síntomas, Al examen físico se encuentra escaso cabello y
hay piel seca lo demás de examen físico esta normal , se solicita exámenes los
mismos que reportan : TSH5.17 u/IU/ml , T3 3 ,3 n mol/ L- T 4 1.0 ng / dl. Escoja la
opción correcta.
Seleccione una:

a. paciente tiene diagnóstico de hipotiroidismo central

b. el paciente tiene diagnóstico de hipotiroidismo subclínico

c. el paciente tiene diagnóstico de eutiroidismo

d. el paciente tiene diagnóstico de hipertiroidismo


Retroalimentación
La respuesta correcta es: el paciente tiene diagnóstico de hipotiroidismo subclínico

Pregunta 19
Correcta
Puntúa 0,16 sobre 0,16

Desmarcar
Enunciado de la pregunta

Paciente de 65 años con antecedente de cáncer de estómago, tiene una dieta


fraccionada con 5 comidas ha bajado de peso mucho como 10 kilogramos y refiere
dolor de epigastrio en forma continua ,además refiere que se ha sentido con polidipsia
y poliuria por lo que se solicita exámenes los mismos que reportan: Glicemia 103 mg /
dl Hemoglobina Glicosilada 6.4 mg y curva de tolerancia a la glicemia reporta a la 99
mg , 30 minutos, 108 mg, 1 horas 199 mg , 2 horas 155 mg. Escoja la opción correcta
Seleccione una:

a. tiene cáncer gástrico y tiene diabetes y debe recibir ya tratamiento con metformina de 1000
mg cada 12 horas

b. la paciente tiene gastrectomía por cáncer gástrico y al momento cursa además con diabetes
y se le recomienda valoración nutricional , ejercicio y control en 1 mes con nueva glicemia

c. la paciente tiene diagnóstico de prediabetes tiene que remitirse a nutrición y debe regresar a

un control en 3 meses con glicemia y nueva hemoglobina glicosilada.

d. la paciente tiene valores normales de laboratorio y no necesita recomendaciones


Retroalimentación
La respuesta correcta es: la paciente tiene diagnóstico de prediabetes tiene que remitirse a
nutrición y debe regresar a un control en 3 meses con glicemia y nueva hemoglobina
glicosilada.

Pregunta 20
Correcta
Puntúa 0,16 sobre 0,16
Desmarcar
Enunciado de la pregunta

Que de lo siguiente usted consideraría como tratamiento inicial ante un diagnóstico de


disfagia orofaringea
Seleccione una:

a. La disfagia orofaringea es indicacion absoluta de gastrostomia endoscopica percutanea

b. Colocacion de sonda de alimentacion duodenal ya que es necesario prevenir la


deshidratacion

c. Debido a la facil descompensacion de los pacientes durante los procedimientos


endoscopicos se prefiere una gastrostomia quirurgica

d. Corregir la enfermedad de base, terapia deglutoria, espesantes alimenticios


Retroalimentación
La respuesta correcta es: Corregir la enfermedad de base, terapia deglutoria, espesantes
alimenticios

Pregunta 21
Incorrecta
Puntúa 0,00 sobre 0,16

Desmarcar
Enunciado de la pregunta

Que examen considera el más adecuado para evaluar disfagia orofaringea en un


paciente con antecedente de enfermedad de parkinson
Seleccione una:

a. Endoscopia digestiva alta

b. Videodeglucion

c. Manometria esofagica de alta resolución

d. Rx detorax
Retroalimentación
La respuesta correcta es: Videodeglucion

Pregunta 22
Incorrecta
Puntúa 0,00 sobre 0,16
Desmarcar
Enunciado de la pregunta

Qué parámetros se usan para valorar la severidad en la evolución de pancreatitis


aguda.
Seleccione una:

a. Amilasa

b. Pcr

c. Bisap

d. Apache 2
Retroalimentación
La respuesta correcta es: Amilasa

Pregunta 23
Correcta
Puntúa 0,16 sobre 0,16

Desmarcar
Enunciado de la pregunta

Señale en vesícula biliar el tipo de cálculo más frecuente


Seleccione una:

a. colesterol

b. pigmento

c. ácido úrico

d. calcico
Retroalimentación
La respuesta correcta es: colesterol

Pregunta 24
Correcta
Puntúa 0,16 sobre 0,16

Desmarcar
Enunciado de la pregunta

Señale lo falso sobre el helicobacter pylori


Seleccione una:

a. La erradicación de helicobacter pylori mejora los síntomas dispépticos

b. Es un bacilo gram positivo

c. Es un bacilo gram negativo

d. El tratamiento de erradicación evita la progresión a lesiones premalignas


Retroalimentación
La respuesta correcta es: Es un bacilo gram positivo

Pregunta 25
Correcta
Puntúa 0,16 sobre 0,16

Desmarcar
Enunciado de la pregunta

Si un paciente es detectado con HbA1c: > 9%, la acción correcta para su tratamiento
sería:
Seleccione una:

a. La primera línea indicada es la metformina en busca de HbAc1 de <7%

b. Administrar metformina junto con un ADO

c. Se debe realizar a triple terapia de ADO

d. Iniciar inmediatamente con insulina.

También podría gustarte